David Madore's WebLog: Mathematics

Vous êtes sur le blog de David Madore, qui, comme le reste de ce site web, parle de tout et de n'importe quoi (surtout de n'importe quoi, en fait), des maths à la moto et ma vie quotidienne, en passant par les langues, la politique, la philo de comptoir, la géographie, et beaucoup de râleries sur le fait que les ordinateurs ne marchent pas, ainsi que d'occasionnels rappels du fait que je préfère les garçons, et des petites fictions volontairement fragmentaires que je publie sous le nom collectif de fragments littéraires gratuits. • Ce blog eut été bilingue à ses débuts (certaines entrées étaient en anglais, d'autres en français, et quelques unes traduites dans les deux langues) ; il est maintenant presque exclusivement en français, mais je ne m'interdis pas d'écrire en anglais à l'occasion. • Pour naviguer, sachez que les entrées sont listées par ordre chronologique inverse (i.e., celle écrite en dernier est en haut). Certaines de mes entrées sont rangées dans une ou plusieurs « catégories » (indiqués à la fin de l'entrée elle-même), mais ce système de rangement n'est pas très cohérent. Cette page-ci rassemble les entrées de la catégorie Mathématiques : il y a une liste de toutes les catégories à la fin de cette page, et un index de toutes les entrées. Le permalien de chaque entrée est dans la date, et il est aussi rappelé avant et après le texte de l'entrée elle-même.

You are on David Madore's blog which, like the rest of this web site, is about everything and anything (mostly anything, really), from math to motorcycling and my daily life, but also languages, politics, amateur(ish) philosophy, geography, lots of ranting about the fact that computers don't work, occasional reminders of the fact that I prefer men, and some voluntarily fragmentary fictions that I publish under the collective name of gratuitous literary fragments. • This blog used to be bilingual at its beginning (some entries were in English, others in French, and a few translated in both languages); it is now almost exclusively in French, but I'm not ruling out writing English blog entries in the future. • To navigate, note that the entries are listed in reverse chronological order (i.e., the latest written is on top). Some entries are classified into one or more “categories” (indicated at the end of the entry itself), but this organization isn't very coherent. This page lists entries in category Mathematics: there is a list of all categories at the end of this page, and an index of all entries. The permalink of each entry is in its date, and it is also reproduced before and after the text of the entry itself.

[Index of all entries / Index de toutes les entréesLatest entries / Dernières entréesXML (RSS 1.0) • Recent comments / Commentaires récents]

Entries with category Mathematics / Entrées de la catégorie Mathématiques:

↓Entry #2784 [older| permalink|newer] / ↓Entrée #2784 [précédente| permalien|suivante] ↓

(dimanche)

La réalisabilité propositionnelle, et ce qu'elle nous apprend sur les algorithmes

Je veux ici de nouveau parler d'informatique théorique, dans son intersection avec les maths (logique, calculabilité, typage), et plus précisément d'un sujet appelé la réalisabilité propositionnelle. Je reconnais qu'il a un petit côté « opération de pub » de ma part : il s'agit d'un sujet qu'on peut qualifier d'obscur, voire ésotérique (largement confiné à une poignée de publications soviétiques des années 1960–1990, dont certaines n'ont même pas été traduites du russe en quoi que ce soit d'autre[#], ce qui le dessert cruellement), et pourtant il me semble que d'une part il a des choses à nous apprendre sur l'informatique, sur ce que peut ou ne peut pas faire un algorithme, et sur le rapport entre ça et le typage, et d'autre part il n'est techniquement pas très compliqué à exposer (j'en ai dit un mot — bien moins que ce qui va suivre, certes — dans mon cours à Télécom qui s'est récemment fini, et je ne crois pas avoir largué tout le monde). Donc je trouve qu'il mérite plus d'attention.

[#] Bon, le fait que ce soit en russe ne devrait pas être un obstacle sérieux, surtout maintenant qu'on a des OCR et des traducteurs automatiques, il n'y a vraiment plus de raison. Le problème, c'est que souvent, et en plus d'être en russe, l'article a le défaut d'être super mal écrit : extrêmement concis et pas toujours super rigoureux comme les mathématiciens soviétiques avaient tendance à l'être, références vagues, et fautes de frappe à foison. (Voyez par exemple cette question que j'ai posée sur MathOverflow en essayant de comprendre l'argument de Ânkov sur la non-réalisabilité de l'axiome de Scott que j'expose plus bas ici de façon, j'espère, nettement plus clair que le texte original : mon russe est enfoui au fond des oubliettes, mais ce n'est vraiment ce qui était le plus problématique en l'occurrence par rapport aux fautes de frappe et au style de rédaction vraiment minimaliste.)

Je ne sais pas si l'entrée de blog qui va suivre peut aider à convaincre qui que ce soit de s'y réintéresser, mais en tout cas, si on lit ce qui suit on pourra se dire qu'on sait une proportion significative de tout le savoir de l'Humanité sur la réalisabilité propositionnelle (i.e., pas grand-chose !), et il n'y a pas beaucoup de sujets techniques pour lesquels on puisse en dire autant.

Ajout () : Pour les gens qui veulent savoir de quoi ce billet cause mais qui n'ont pas la patience de lire ce pavé (ni même l'introduction), j'ai écrit un résumé en seulement 13 tweets, ici sur Twitter et ici sur BlueSky [on peut lire le fil BlueSky sans avoir de compte dessus].

Table des matières

Résumé et publicité préalable

Avant de rentrer dans le cœur du sujet, j'essaie d'expliquer informellement[#2] (et de manière — j'espère — assez peu technique) de quoi il s'agit, et pourquoi je pense que c'est assez intéressant pour prendre le temps de déchiffrer des articles à moitié oubliés dans des journaux soviétiques et de pondre un pavé de 1729 pages dans ce blog pour réexpliquer le tout aux enfants. (Si on veut passer directement à la définition, c'est possible, mais je pense que c'est intéressant de donner d'abord une idée grossière de ce dont je vais parler, au risque de prendre un peu un ton introduction de manuscrit de thèse. Je remercie d'ailleurs mes peluches pour leur soutien et leur patience tout au long de l'écriture de cette entrée.)

[#2] Autrement dit, comme le corps de cette entrée est très long, je commence par une introduction très longue pour essayer de motiver cette longueur. Je n'ai fait celle-ci plus longue que parce que je n'ai pas eu le loisir de la faire plus courte. (Blaise Pascal)

Introduction : de quoi s'agit-il ? Cadre et motivations

La calculabilité s'intéresse à décrire de façon générale ce que peut ou ne peut pas faire un algorithme informatique. La réalisabilité propositionnelle s'intéresse (du moins selon le point de vue que j'adopte ici) spécifiquement à certaines sortes de manipulations que je pourrais être qualifier de manipulations de données génériques (ou abusivement, fonctions polymorphes). On va décrire ces manipulations génériques par des formules logiques, du style ABBA (logiquement, celle-ci se lit : si A et B alors B et A). La formule représente une sorte de « contrat » qui stipule la manipulation censément effectuée : par exemple, la formule ABBA représente le contrat je prends une donnée de type A et une donnée de type B et je renvoie une donnée de type B et une donnée de type A (et on peut faire ça algorithmiquement, de façon à peu près évidente, en échangeant les deux données qu'on a reçues). Lorsqu'il y a un algorithme qui remplit le contrat, on dira qu'il réalise la formule, ou que celle-ci est réalisable.

Je donnerai une définition précise plus bas de comment lire la formule comme un contrat et ce que ça signifie de la réaliser, mais je peux tout de suite donner un aperçu informel : on s'intéresse à des formules écrites avec les connecteurs binaires ‘∧’, ‘∨’ et ‘⇒’, ainsi que les constantes ‘⊤’ et ‘⊥’ (et ‘¬’ qui est juste une abréviation de X⇒⊥). Les variables (A,B dans l'exemple que je viens de donner) représentent des types de données quelconques et non spécifiés, et les connecteurs logiques représentent des façons de mettre ces données ensemble : grosso modo,

  • AB (le « et » logique) représente la donnée d'une donnée de type A et d'une de type B,
  • AB (le « ou » logique) représente la donnée d'une donnée de type A ou d'une de type B (mais avec l'information de laquelle on a),
  • AB (l'implication) représente une fonction (au sens informatique) qui prend une donnée de type A et en renvoie une de type B,
  • ⊤ (le « vrai ») représente une donnée triviale, et ⊥ (le « faux ») représente une donnée impossible (inexistante ou inobtenable),
  • ¬A (la négation) représente une promesse qu'il n'y a pas de A.

Ainsi, par exemple, pour faire une manipulation représentée par la formule (« contrat ») ABBA, on prend une donnée de type A et une donnée de type B et on les renvoie dans l'ordre contraire, c'est-à-dire la donnée de type B et la donnée de type A, et on pourra dire que la formule ABBA est réalisable (et que ce programme la réalise, i.e., il « remplit le contrat »).

J'ai utilisé le terme type ci-dessus, mais il n'est peut-être pas vraiment approprié, parce que justement, la réalisabilité ce n'est pas pareil que le typage, même si ça y ressemble beaucoup (et c'est la différence qui est très intéressante). Je m'explique un peu plus.

Il y a un formalisme appelé correspondance de Curry-Howard (j'en ai parlé dans un billet précédent, dont il est peut être intéressant de lire au moins l'introduction même si ce n'est pas strictement nécessaire pour la suite) qui transforme une démonstration d'une formule dans un certain système logique (le calcul propositionnel intuitionniste) en un programme qui fait la manipulation décrite par cette formule. Et le fait que ce programme fasse bien cette manipulation est attesté par ce qu'on appelle un système de typage, en l'occurrence le λ-calcul simplement typé enrichi de types produits et sommes, 1 et 0 mais peu importe : si on se limite aux programmes conformes à ce système de typage, alors la correspondance est exacte — je veux dire, bijective : on peut vraiment identifier les preuves de tel système logique avec les programmes conformes à tel système de typage. D'ailleurs, la correspondance n'est pas bien compliquée et fait vraiment correspondre de façon assez simple les différentes règles de la logique avec les différentes constructions admises par le système de typage.

Le système de typage (disons ce fameux λ-calcul simplement typé enrichi de types produits et sommes, 1 et 0), il contraint les programmes a priori : il les contraint dans toutes les étapes de leur construction, il exige que toutes les opérations qu'ils font soient conformes au typage. Et au bout du compte, les formules obtenues décrivant les manipulations qu'on peut faire, ce sont justement leurs types (à la notation près), et ce sont exactement les formules démontrables dans le système logique (c'est ça que dit la correspondance de Curry-Howard).

Mais ce n'est pas de ça que je veux parler. Ça c'est ce dont j'ai déjà parlé (dans la première partie du billet sur Curry-Howard).

La réalisabilité[#3], elle ne contraint pas les programmes a priori comme le fait le typage. Elle correspond à la philosophie suivante : le programme a le droit de faire ce qu'il veut comme manips (il faut juste que ce soit un algorithme au sens usuel de la calculabilité, celle de Church-Turing, l'idéalisation standard de ce que c'est qu'un ordinateur) et réaliser une formule ça signifie qu'il remplit un certain contrat. Peut-être qu'entre temps, au sein de leurs manips, les programmes violent toutes les règles du typage, mais au final, on demande juste qu'ils remplissent le contrat représenté par la formule : c'est le résultat qui compte, et seul le résultat qui compte. (Peut-être aussi qu'il est impossible de prouver que le programme remplit bien son contrat, mais ce qui importe c'est que mathématiquement il le fait : là aussi, c'est juste le résultat qui compte.)

[#3] Enfin, la réalisabilité propositionnelle que j'évoque ici, et qu'on peut qualifier de réalisabilité propositionnelle non typée pour insister, parce qu'il y a 1001 variantes de la notion (ce qui a causé un certain dialogue de sourds entre Andrej Bauer et moi dans cette question MathOverflow — sa réponse est très intéressante mais elle ne répond pas vraiment à la question que je voulais poser, quoiqu'elle réponde peut-être à la question que j'ai posée).

Par exemple, ABBA c'est le contrat tu vas recevoir deux données, la première de type A et la seconde de type B (et, précision importante, tu n'auras ni accès ni information sur ce que sont A et B), tu dois terminer en temps fini et renvoyer une donnée de type B et une donnée de type A, et effectivement on peut se convaincre que la seule façon de faire ça c'est d'échanger l'ordre des coordonnées.

Quand la formule est démontrable (en calcul propositionnel intuitionniste) ou, ce qui revient au même par Curry-Howard, quand on peut faire un programme typé correspondant à la formule, alors certainement ce programme remplit le contrat représenté par la formule, donc elle est réalisable.

Mais la grande surprise (en tout cas pour moi quand j'ai appris ça, mais aussi, de ce que je comprends, pour Kleene qui contrairement à moi n'était pas un idiot), c'est que la réciproque n'est pas vraie : il y a des contrats qu'on peut remplir — des formules qu'on peut réaliser — sans que la formule soit prouvable, ou, ce qui revient au même, sans qu'on puisse le faire dans le système de typage naturel dans cette situation (le λ-calcul simplement typé enrichi gnagnagna). Un des buts de ce billet est de donner des exemples de telles formules (jetez un coup d'œil plus bas si vous voulez voir à quoi ça ressemble).

Ces formules réalisables mais non démontrables illustrent donc, si on veut, des choses que peut faire un algorithme, remplissant un contrat de manipulation qui ressemble à du typage, mais qui ne sont pas faisables dans le cadre du typage — en tout cas du typage le plus naturel dans cette situation — ou, si on veut, ces choses sont faisables, mais la vérification que le contrat est bien rempli est plus compliquée qu'une vérification locale telle que fournie par le typage.

Bien sûr, il n'y avait aucun doute que les algorithmes généraux sont plus généraux que les algorithmes typés : notamment, les systèmes de typage que j'évoque dans cette introduction ne permettent pas de faire de boucle infinie, donc il est évident qu'ils limitent structuralement les algorithmes. Mais ce qui est surprenant dans l'histoire, c'est que l'algorithme va quand même remplir un contrat qui se lit exactement comme un type, i.e., ils font des manipulations de données génériques potentiellement utiles, pas quelque chose comme une boucle infinie ni même une fonction sur un type spécifique comme les entiers.

Je précise pour dissiper un possible malentendu que ce que je dis là n'est pas destiné à être une attaque contre le typage ou son utilité, juste un signe que les choses sont plus délicates que ce qu'on pourrait imaginer naïvement : certes Curry-Howard nous dit que les formules logiques qui correspondent à des programmes typés sont exactement les formules démontrables, mais il y a des contrats qu'on peut remplir de façon plus subtile que le typage.

Et du coup, il me semble que c'est un problème très naturel pour l'informatique et la calculabilité que d'étudier cette réalisabilité propositionnelle, parce que ça a directement trait à ce qu'un algorithme peut ou ne peut pas faire. (Notons que c'est là l'approche que je donne à la question dans ce billet, tout le monde ne sera pas forcément d'accord que c'est la bonne motivation.)

Organisation de ce billet

Dans la suite, je commence par définir de façon précise ce que c'est que la réalisabilité propositionnelle, et je donne des exemples très simples d'une formule réalisable (AB ⇒ BA) et d'une formule qui ne l'est pas (A ∨ ¬A) pour montrer comment on manie la notion. Ensuite je dois faire un certain nombre de remarques générales (plusieurs desquelles sont en petits caractères pour montrer qu'on peut les ignorer et passer à la suite).

Puis je me tourne vers des exemples de formules réalisables mais non démontrables, parce que c'est quand même ça la grosse surprise, que de telles formules existent : pour chacune, j'ai essayé de donner non seulement un algorithme qui la réalise (et comment il fonctionne) mais aussi des explications informelles à son sujet. En revanche, je ne détaille pas l'argument expliquant que la formule n'est pas démontrable (en calcul propositionnel intuitionniste) : il n'y a que la première pour laquelle j'ai fait cet effort, et encore succinctement et en petits caractères, parce que ce n'est pas vraiment mon sujet.

Je commence par la formule qui me semble la plus simple à comprendre intuitivement, celle de Ceitin, puis j'en passe en revue un certain nombre d'autres. On peut traiter ça comme autant d'énigmes : une fois qu'on a compris le principe, c'est un petit jeu de regarder chaque formule comme un contrat à remplir et de se demander comment je vais faire une telle chose avec un algorithme ? (en tout cas, c'est comme ça que j'ai approché le problème : je n'ai pas lu les preuves de réalisabilité qu'on trouve dans la littérature, c'était bien plus instructif de les retrouver moi-même).

Ensuite, je passe à des exemples de formules non réalisables mais intéressantes et notamment « presque » réalisables, à travers deux exemples importants : la formule de Kreisel-Putnam et celle de Scott. Donc cette fois il faut à la fois montrer qu'elles ne sont pas réalisables et que, pourtant, on ne passe pas loin de l'être (et j'explique en quoi elles sont « presque » réalisables, même si je n'ai pas réussi à avoir le recul pour comprendre si on peut définir cette notion proprement).

Enfin je finis par des généralités et spéculations peut-être vaseuses et certainement mal écrites, qui font un peu le pendant de l'introduction ci-dessus.

Pourquoi j'estime que c'est intéressant et important

Bien sûr, comme je l'explique ci-dessus, on peut traiter chacune des formules que je vais lister ci-dessous (aussi bien celles qui sont réalisables que celles qui ne le sont pas) comme un exercice de calculabilité : voici une formule représentant un certain contrat, trouver une façon de le remplir [= réaliser la formule] ou montrer que ce n'est pas possible ; ou même comme une sorte d'énigme (à la manière dont je l'ai fait ici et ). Les outils de base utilisés par ces algorithmes sont toujours un peu les mêmes, et ce sont les techniques fondamentales de la calculabilité : lancer deux tâches en parallèle (sachant qu'au moins une des deux terminera), exécuter un programme pour un certain nombre d'étapes pour voir s'il termine, parcourir tous les entiers naturels en sachant qu'on finira par en trouver un qui remplit une certaine condition — ce genre de choses qui sont les bases de la calculabilité de Church-Turing. Donc déjà c'est au moins intéressant parce que c'est instructif à ce niveau-là.

Mais je pense que ça va plus loin que ça. Comme je le dis plus haut, la réalisabilité propositionnelle est une façon d'aborder le problème, qui me semble central en informatique, de ce qu'un algorithme peut ou ne peut pas faire, et en l'occurrence il est important de comprendre ce que le typage laisse ou ne laisse pas passer, et comme le typage est un outil lui aussi central, comprendre ce qu'il ne laisse pas passer est justement une question cruciale, et même si on peut trouver que la réalisabilité propositionnelle est un prisme un peu étroit pour étudier cette question (j'en conviens), je pense qu'elle a des choses à dire, et je n'ai pas l'impression qu'on les comprenne bien dans l'état actuel de l'art (d'où le fait que les exemples de formules réalisables soient un peu… hétéroclites), mais ça vaut certainement la peine de méditer un peu sur chacune de ces formules certes disparates et de ce qu'elle signifie et ce qu'elle représente. En tout cas, ce n'est certainement pas un hasard si la réalisabilité de ces différentes formules fait appel à ce que je viens d'appeler les techniques fondamentales de la calculabilité.

En outre, ça nous apprend aussi — il me semble — des choses sur la richesse de la logique intuitionniste, même au niveau simplement propositionnel (en nous forçant à nous demander ce que disent logiquement certaines de ces formules qu'on n'aurait sans doute pas pensé à considérer sinon). Ma motivation à moi était justement de me familiariser mieux à la fois avec la calculabilité en général et avec les techniques à l'interface entre logique, calculabilité et typage : en réexpliquant chacune de ces preuves, je m'assure que je les ai bien comprises. (Et il y a plein de choses que j'ai été obligé de revoir en écrivant ce billet.)

↑Entry #2784 [older| permalink|newer] / ↑Entrée #2784 [précédente| permalien|suivante] ↑

↓Entry #2782 [older| permalink|newer] / ↓Entrée #2782 [précédente| permalien|suivante] ↓

(vendredi)

Mélanges probabilistes et superpositions quantiques

Précisions d'emblée que le but de ce billet, qui évoque le rapport entre (pour prendre un exemple célèbre) un chat vivant avec probabilité ½ et mort avec probabilité ½ (« mélange probabiliste ») et un chat dans un état quantique qui combine vivant et mort (« superposition quantique ») n'est pas vraiment de faire de la vulgarisation, encore moins de la physique. Je m'en sers, comme je fais parfois, surtout pour gribouiller rapidement ce que j'ai (moi matheux) réussi à comprendre de textes que je trouve souvent obscurs, et pour noter des questions que je (me) pose si je veux y réfléchir plus attentivement Un Jour™, mais ça ne signifie pas que d'autres trouveront mes explications plus claires que ce qu'on peut trouver ailleurs. Il s'agit essentiellement de choses très classiques, mais que je trouve généralement très mal expliquées (notamment par le fait qu'on prend rarement le soin d'essayer de décrire le parallèle entre mélanges probabilistes et superpositions quantiques séparément avant de dire comment ces deux choses se combinent), quoique certaines des questions que je soulève au passage n'ont pas l'air d'être beaucoup discutées, et c'est dommage.

Quoi qu'il en soit, ce qui suit s'adresse à des lecteurs qui savent au moins un peu d'algèbre linéaire (en gros, pour qui les mots espace de Hilbert ont un sens — je ne parlerai que de la dimension finie donc on peut préférer espace hermitien), et sont globalement familiers avec le fait qu'une matrice hermitienne est diagonalisable. Et encore une fois, mon point de vue va être celui d'un matheux, pas d'un physicien (témoin le fait que je vais à peine évoquer de lois de la physique) : la question est celle de la représentation mathématique d'états d'un système physique. (Et j'en profite pour pointer du doigts certains faits géométriques sur cette représentation.) Mais au passage, ça soulève des questions, qui me semblent intéressantes, sur la philosophie de la physique (notamment que signifient les probabilités, et dans quelle mesure elles font partie de la réalité du monde).

Je dois aussi préciser que j'ai changé plein de fois d'avis sur ce que je voulais raconter ici, que j'ai fait mon plan a posteriori et que j'ai réécrit plein de fois des passages sans vérifier la cohérence avec ce qui était déjà ailleurs, ce qui explique sans doute des virages un peu bizarres, des redites ou incohérences de propos et des digressions inutiles (comme d'habitude, j'essaie de rédiger de manière à ce qu'on puisse les sauter, mais je ne sais pas dans quelle mesure j'y arrive). Par ailleurs, comme ça m'arrive souvent, j'ai écrit ce texte jusqu'au point où j'en ai eu marre de l'écrire, ce qui explique qu'après être parti dans toutes les directions il s'arrête un peu brutalement et sans vraie conclusion — mais je pense que mes lecteurs (enfin, ceux qui sont assez patients pour lire mes billets jusqu'au bout) ont l'habitude de ça.

Plan

Mélanges probabilistes et superpositions quantiques séparément

Comme promis, je commence comme un matheux. Supposons que A soit un ensemble, que je vais prendre fini pour simplifier et que j'imagine comme les états basiques que peut prendre un système physique.

Mon but est dans un premier temps de définir deux types de constructions[#] qu'on peut faire sur cet ensemble A, que je vais ensuite comparer et contraster, et que je vais appeler mélanges probabilistes et superpositions quantiques ; puis, dans la suite, je discuterai comment on peut les combiner.

[#] Techniquement, j'imagine qu'on doit pouvoir faire de chacune de ces constructions une monade, mais je ne veux pas tomber dans ce trou de lapin-là.

Mélanges probabilistes

Si je ne sais pas exactement dans quel état se trouve mon système, je peux représenter mon ignorance sous la forme d'une distribution de probabilités sur A : concrètement, ça va prendre la forme d'une fonction de A vers les réels positifs (donnant la probabilité de chaque état) dont la somme totale est 1. De façon équivalente, si je note [a], lorsque aA la fonction qui vaut 1 en a et 0 ailleurs (i.e., la distribution de probabilités concentrée en a), une distribution de probabilités p quelconque sur A s'écrit comme une combinaison convexe des [a], c'est-à-dire une combinaison linéaire à coefficients positifs de somme 1 (les coefficients étant justement la probabilité p(a) de chaque a, i.e., on a p = ∑aA p(a) · [a], avec, je répète, p(a)≥0 pour chaque a, et ∑aA p(a) = 1). Je parlerai aussi de mélange probabiliste des éléments de a pour une telle combinaison convexe.

À titre d'exemple, si A = {vivant, mort} désigne les deux états possibles de vie d'un chat, le mélange probabiliste ½([vivant] + [mort]) désigne un chat qui a 50% de chances d'etre vivant et 50% de chances d'être mort.

Géométriquement, il faut penser à cet ensemble de distributions de probabilités / combinaisons convexes / mélanges probabilistes (selon le terme qu'on préfère) comme un simplexe dont les sommets sont les éléments de A (i.e., lorsque A a 2 éléments, c'est un segment les reliant, quand il en a 3 c'est un triangle ayant ces sommets, quand il en a 4 c'est un tétraèdre, etc.) ; et les coordonnées barycentriques dans le simplexe sont les valeurs p(a) de la distribution de probabilités p considérée.

Je n'ai rien dit d'intelligent, ou même d'intéressant dans tout ça : c'est complètement standard, c'est juste différents points de vue un tout petit peu différents sur la même chose.

Superpositions quantiques

Maintenant, quand on fait de la mécanique quantique, il y a autre chose qui intervient, et qu'il faut bien distinguer de ce qui précède : ce sont les superpositions quantiques. Cette fois, je vais considérer un espace vectoriel complexe[#2], et même hilbertien (= hermitien), dont une base orthonormée est formée de vecteurs notés |a⟩ où aA, et je m'intéresse aux vecteurs de norme 1 dans cet espace (éventuellement : modulo la phase, c'est-à-dire modulo multiplication par les complexes de module 1, ce qui en fait un « espace projectif » complexe, cf. ici). Autrement dit, les éléments de l'espace sont les ∑aA u(a) · |a⟩ avec u(a) des nombres complexes et ∑aA |u(a)|² = 1 ; le produit scalaire hermitien de deux tels éléments |u⟩ := ∑aA u(a) · |a⟩ et |v⟩ := ∑aA v(a) · |a⟩ est donné par ⟨u|v⟩ := ∑aA u(a)* · v(a) où z* désigne ici le conjugué d'un nombre complexe (normalement noté avec une barre au-dessus, mais c'est pénible à faire en HTML) (j'ai pris ici la convention des physiciens selon laquelle le produit scalaire hermitien est antilinéaire dans sa première variable et linéaire dans la seconde). La notation suggère de définir ⟨a| comme la forme linéaire valant 1 en |a⟩ et 0 sur tous les autres |b⟩, si bien que le produit scalaire par ∑aA u(a) · |a⟩ à gauche s'écrit comme la forme linéaire ∑aA u(a)* · ⟨a|. Bon, là je n'ai rien dit d'intelligent.

[#2] Le fait qu'on ait apparemment nécessairement affaire à des coefficients complexes, quel que soit le système physique décrit, me laisse un peu perplexe, et apparemment je ne suis pas le seul.

Pour reprendre l'exemple précédent, si A = {vivant, mort} désigne les deux états possibles de vie d'un chat, alors (|vivant⟩ + |mort⟩)/√2 désigne un chat dans un état quantique qui superpose ces deux états. Mais on notera que (|vivant⟩ − |mort⟩)/√2 est aussi un tel état, qui semble très analogue, mais qui est orthogonal au précédent comme on le voit en calculant le produit hermitien (et on peut légitimement se demander ce que tout ça veut dire). Et de même, (|vivant⟩ + i·|mort⟩)/√2 et (|vivant⟩ − i·|mort⟩)/√2 devraient avoir un sens et être orthogonaux l'un à l'autre (quoique pas aux précédents). Ceci étant dit, autant c'est rigolo de donner mes exemples avec des chats vivants ou morts ou en superposition quantique entre les deux, ce n'est peut-être pas un très bon exemple[#3][#3b], en fait, justement à cause de la difficulté de donner un sens à ces états que je viens d'écrire, donc dans la suite je vais passer à un exemple plus abstrait du genre A = {0,1}, c'est-à-dire les états basiques de ce qu'on appelle un qubit : vous pouvez imaginer ‘0’ et ‘1’ comme signifiant qu'un chat est vivant et mort si vous voulez, mais si vous voulez des exemples physiquement plus plausibles, l'article Wikipédia que je viens de lier a divers exemples, et je vais juste dire un mot de deux d'entre eux dans les paragraphes suivants.

[#3] Je ne sais plus qui me faisait le reproche je ne sais où d'utiliser le chat de Schrödinger comme exemple de superposition quantique, alors que c'est justement un exemple censé illustrer le doute qu'on peut avoir sur l'existence ou le sens de superpositions quantiques sur des objets macroscopiques. Si on croit la mécanique quantique jusqu'au bout, et notamment si on croit sa linéarité exacte, alors oui, on peut faire des superpositions quantiques macroscopiques, et même c'est ce qui arrive à l'Univers tout entier dès qu'on fait une « mesure », et il y a toutes sortes de tentatives d'explications, ou de bouts d'explications (décohérence, interprétation « multi-mondes » d'Everett-DeWitt) sur pourquoi on ces superpositions ne se manifestent pas de façon visible dans notre expérience quotidienne. Mais mon but ici n'est pas vraiment de parler de ces choses-là (même si je ne peux pas faire l'économie d'au moins une mention au passage — dont acte ; cf. aussi la note #8 plus bas).

[#3b] Ajout () : Bien sûr, le problème avec le chat, ce n'est pas juste qu'il est macroscropique, c'est qu'il a bien plus d'états que {vivant, mort} : il y a peut-être quelque chose comme 101027 états qualifiables de vivant et de mort. En quoi ceci est vraiment pertinent pour toute la discussion n'est pas clair pour moi, ni si on choisit de les regrouper en deux paquets (i.e., de fabriquer deux sous-espaces de grande dimension) ni si on décide d'en choisir un très particulier dans chaque paquet (mais je note quand même que, par un phénomène de concentration de la mesure, si on choisit un état vivant au hasard et un état mort au hasard, ils seront essentiellement orthogonaux — donc au moins ça justifie de travailler avec comme des états basiques).

À titre d'exemple de qubit, il y a la polarisation d'un photon : si |↺⟩ représente un photon polarisé circulairement d'hélicité droite[#4] et |↻⟩ un photon polarisé circulairement d'hélicité gauche, alors (|↺⟩ + |↻⟩)/√2 et (|↺⟩ − |↻⟩)/√2 peuvent représenter des photons respectivement polarisés horizontalement et verticalement[#5][#6], tandis que (|↺⟩ + i·|↻⟩)/√2 et (|↺⟩ − i·|↻⟩)/√2 peuvent en représenter de polarisations diagonales.

[#4] Comme bien expliqué sur Wikipédia, il y a deux conventions opposées sur ce qu'une polarisation circulaire horaire ou anti-horaire signifie, selon qu'on prend le point de vue de la source qui voit l'onde partir ou de la cible qui voit l'onde arriver. Par contre, l'hélicité, il me semble que ça devrait être inambigu : on met le pouce (droit ou gauche, selon qu'on parle d'hélicité droite ou gauche) dans le sens de propagation de l'onde et en courbant les autres doigts ils indiquent dans quel sens l'onde tourne autour de son sens de propagation. Donc pour moi, hélicité droite = sens anti-horaire (= trigonométrique) vu par la cible = sens horaire (= rétrograde) vu par la source, tandis que hélicité gauche = sens horaire (= rétrograde) vu par la cible = sens anti-horaire (= trigonométrique) vu par la source. (Et les petits dessins ‘↺’ et ‘↻’ que j'utilise évoquent ce que voit la cible.) Mais apparemment, toujours si j'en crois Wikipédia, des gens ont aussi réussi à mélanger les conventions gauche/droite, et là je ne comprends pas comment ils ont pu faire un truc pareil. Enfin bon, tout ça n'a aucune importance pour ce que je veux raconter ici.

[#5] Là aussi, on trouve des conventions contradictoires, bien sûr, mais la convention moderne semble être de dire qu'une onde se propageant horizontalement a une polarisation horizontale par référence à la direction du champ électrique oscillant : le champ magnétique, lui, oscille dans une direction perpendiculaire au champ électrique et au vecteur de propagation de l'onde, donc verticalement pour une onde de polarisation horizontale.

[#6] Mathématiquement, imaginez que |↺⟩ est la fonction exp(2iπν·t) où ν est la fréquence du photon et t est le temps retardé par la distance depuis la source, et le composantes réelle et complexe sont, disons, les composantes horizontale et verticale du champ électrique ou quelque chose comme ça, tandis que |↻⟩ est exp(−2iπν·t) ; alors (|↺⟩ + |↻⟩)/√2 et (|↺⟩ − |↻⟩)/√2 décrivent les fonctions cos(2πν·t) et i·sin(2πν·t) respectivement.

La raison pour laquelle je précise l'exemple du paragraphe précédent est pour souligner que ces états en superposition quantique sont parfaitement valables (je vais dire ci-dessous que, contrairement à la situation probabiliste, il n'y a rien d'objectif qui distingue les états basiques que j'ai choisis des autres états fabriqués par combinaisons linéaires de ceux-ci : le fait qu'un état soit « superposé » n'a pas de sens en soi), et ça se voit bien sur cet exemple-là : les polarisations circulaires n'ont rien de plus naturel que les polarisations horizontales/verticales ou diagonales. Par ailleurs, on se dit que ce sont des choses qui ont un vrai sens physique, pas des expressions de notre ignorance.

J'ai évoqué les photons ci-dessus pas juste pour le plaisir d'utiliser les caractères ‘↺’ et ‘↻’, mais aussi parce que je pense que c'est raisonnablement simple à comprendre — modulo les prises de tête sur les conventions contradictoires quant au sens de la polarisation — mais on peut aussi dire un mot du qubit décrivant le spin de l'électron au repos. Là les deux états basiques pourraient être |↑⟩ et |↓⟩ représentant un électron avec un spin dirigé vers le haut ou vers le bas respectivement : alors (|↑⟩ + |↓⟩)/√2 et (|↑⟩ − |↓⟩)/√2 peuvent représenter un électron avec un spin dirigé vers la droite et la gauche respectivement, tandis que (|↑⟩ + i·|↓⟩)/√2 et (|↑⟩ − i·|↓⟩)/√2 peuvent en représenter un avec un spin dirigé vers l'avant et l'arrière respectivement[#7].

[#7] Le lecteur astucieux me demandera mais ça dépend très hautement du fait que l'espace est de dimension 3, ça : que se passe-t-il en d'autres dimensions ? — et, en effet, c'est une particularité de la dimension 3 que l'état de spin d'une particule de spin ½ soit représenté par un qubit. En général, en dimension d le spin d'une telle particule devrait avoir 2d/2⌋ états basiques (c'est la dimension de la représentation spinorielle du groupe Spind ; je ne sais d'ailleurs pas comment Wikipédia réussit à cacher cette information aussi efficacement dans la page que je viens de lier), c'est-à-dire l'équivalent de ⌊d/2⌋ qubits. Donc, oui, c'est particulier à la dimension 3 qu'on puisse décrire ça aussi simplement que vers le haut et vers le bas. Pour la polarisation du photon, il me semble que c'est d−1 états basiques (donc, en grande dimension, il y a beaucoup moins d'information dans le spin d'un photon que d'un électron, vous interprétez ça comme vous voulez).

Ressemblances et différences entre les deux

Il y a des ressemblances entre mélange probabiliste et superposition quantique, et j'ai fait exprès de choisir une description analogue avec des combinaisons linéaires pour faire ressortir ces ressemblances (et je regrette que toute description de la mécanique quantique ne commence pas par une telle discussion). Il y a aussi des différences cruciales, à la fois physiques et mathématiques.

↑Entry #2782 [older| permalink|newer] / ↑Entrée #2782 [précédente| permalien|suivante] ↑

↓Entry #2780 [older| permalink|newer] / ↓Entrée #2780 [précédente| permalien|suivante] ↓

(jeudi)

Sur la notion d'« étendue » en optique et en géométrie

Je voudrais essayer de parler ici d'un concept de géométrie (euclidienne), notamment important pour son application en optique, que je trouve à la fois joli pourtant trop peu connu[#] : celui d'« étendue ». En gros, l'étendue est une façon de mesurer la taille d'un ensemble de droites (disons, dans l'espace) de la même manière que le volume est une façon de mesurer la taille d'un ensemble de points. Mais pour commencer, et pour expliquer une façon dont cette notion apparaît, je veux parler des unités et grandeurs lumineuses que sont le lumen, le lux, la candela et la candela par mètre carré, et le lien entre ces unités, et ce qu'elles nous disent. Parce que même le grand public commence à avoir entendu parler des lumens (c'est écrit sur toutes les boîtes d'ampoules, de nos jours), et peut-être de lux (pour le niveau d'éclairement d'une pièce), mais ne sait pas forcément la différence entre les deux (divulgâchis : un lux, c'est un lumen par mètre carré).

[#] À titre d'exemple, je n'en vois pas de trace dans les programmes de classes préparatoires scientifiques françaises, ce que je trouve un peu surprenant. Et ce n'est pas une évolution récente parce que je ne crois pas qu'on m'en ait parlé quand j'étais moi-même en prépa.

La difficulté de l'exercice, c'est que comme je veux parler de divers concepts (l'étendue en géométrie euclidienne, l'étendue en optique géométrique, les unités de mesure photométriques, et quelques conséquences de tout ça) et ce à différents niveaux de vulgarisation, je mais certainement faire de la bouillie. En plus de ça, je suis infoutu de faire le moindre dessin, ce qui n'aide pas pour expliquer un concept éminemment géométrique.

Mais voilà, essayons quand même (bear with me), parce cette histoire de conservation de l'étendue est quelque chose qui me semble important pour comprendre les bases de l'optique, et c'est aussi un concept mathématique pas compliqué mais néanmoins digne d'être noté. Et j'en ai marre de toujours oublier ces choses, alors ce billet de blog est surtout un aide-mémoire pour moi-même.

*

☞ La « luminosité » décroît-elle avec la distance ? (Oui et non.)

Pour commencer, une observation toute simple qui me fascine depuis que je suis petit, qui n'est pas difficile à comprendre, et qui renferme l'essence de cette notion de conservation de l'étendue.

Considérez le Soleil, vu depuis la Terre, en plein jour (et par beau temps). Ne le regardez surtout pas directement, c'est très dangereux pour vos yeux (Newton a failli se rendre aveugle comme ça). Oui mais pourquoi est-ce si dangereux, au juste ? Parce que le Soleil est très brillant, bien sûr. Certes, mais il est aussi très loin : est-ce que la luminosité ne s'atténue pas avec la distance ? Après tout, l'étoile Sirius (enfin, Sirius A), qui est 23 fois plus lumineux que le Soleil mais 540 000 fois plus loin de nous, ne nous aveugle pas quand nous la regardons directement. Alors, oui et non : ça dépend surtout de ce qu'on appelle luminosité.

D'abord, il y a une absorption de la lumière par l'atmosphère terrestre, qui atténue surtout le bleu et l'ultra-violet. C'est important pour rendre le Soleil moins dangereux pour nous, mais ce n'est pas de ça que je veux parler, donc faisons comme si elle n'existait pas. Ensuite, il y a effectivement une décroissance de la luminosité avec la distance (quadratique : si on est 2 fois plus loin d'une étoile, elle apparaît 4 fois moins lumineuse), mais cette décroissance correspond précisément à une décroissance de la taille apparente (ou « angle solide ») de l'étoile : si on va 2 fois plus loin du Soleil, il apparaîtra 2 fois plus petit dans chaque direction, donc 4 fois plus petit en surface apparente (« angle solide »), et sa luminosité sera 4 fois plus faible juste pour cette raison, c'est-à-dire que la luminosité par unité d'angle solide (ou luminance) sera la même.

Quand l'objet qu'on regarde est suffisamment petit pour qu'il n'en reste qu'un seul point (on dit qu'il n'est pas « résolu optiquement »), on ne voit plus l'angle solide, et tout ce qui compte est la luminosité totale qu'on en reçoit[#2] (dont je vais dire qu'elle s'appelle correctement éclairement et se mesure en lux) ; mais si l'objet est de taille visible, ce qui est le cas du Soleil, alors ce qui compte est cette luminosité par angle solide (appelée luminance, et mesurée en lux par stéradian, ou ce qui revient au même, en candelas par mètre carré), et elle ne change pas avec la distance.

[#2] Je ne sais pas si je l'ai déjà raconté ici, mais depuis que je suis petit, quand je suis passager dans une voiture la nuit, j'aime parfois jouer à retirer mes lunettes. Comme je suis très très myope, chaque feu de véhicule devient alors une grosse tache (en gros l'image de mon cristallin qui, vu que c'est la nuit, est bien ouvert) : c'est très joli et assez poétique, ça fait une sorte de ballet hypnotisant de taches de couleurs rouges, orange et blanches. Ce qui est perturbant, c'est que la taille de ces taches ne change pas quand le véhicule s'éloigne, puisqu'elle est avant tout due à l'imperfection optique de mon œil : l'éloignement se manifeste non pas par une diminution de la taille mais par une diminution de la luminosité de la tache. C'est ce que j'appelle une situation non résolue optiquement. C'est pareil pour n'importe qui quand on regarde une étoile, mais dans mon cas les taches sont vraiment très grosses (d'ailleurs, du coup, sans mes lunettes, je ne peux absolument pas voir la moindre étoile).

En bref, si vous étiez deux fois plus loin du Soleil, ce serait tout aussi dangereux pour vos yeux de le regarder directement et fixement : la surface de la rétine qui se ferait endommager serait plus petite, mais la surface qui serait endommagée le serait tout autant. (Bon, il y a plein de petits caractères à ajouter là : par exemple pour rappeler que je parle de regarder fixement pendant une certaine durée, parce que les yeux bougent toujours un peu tout le temps, ce qui aide justement à nous protéger contre le soleil.)

Inversement, en principe, même si vous étiez à cent mètres de la surface du Soleil, cette surface vous apparaîtrait aussi brillante (absorption atmosphérique mise à part) que ce que vous en voyez depuis la Terre — c'est juste que ça occuperait la moitié de votre champ visuel au lieu d'être un tout petit truc (de 68 microstéradians) dans le ciel. (Bon, bien sûr, à 100m du Soleil vous auriez plein de problèmes, et je déconseille très fortement d'y aller.)

Et ça, quand j'étais petit, ça me fascinait profondément, de me dire que nous voyons le Soleil vraiment lumineux comme il est, juste plus petit.

*

☞ Flux lumineux, éclairement, intensité lumineuse, et luminance

Bon, mais dans tout ça j'ai utilisé le terme de luminosité de façon très vague. Essayons d'être plus clair, en distinguant quatre notions, et en expliquant comment elles se relient et quelles sont les unités qui les mesurent (si vous trouvez ça trop verbeux, j'ai mis un résumé à la fin) :

  • La première, c'est la notion de flux lumineux, qui est mesuré en lumens (symbole : lm) dans le Système International. C'est analogue à la puissance lumineuse, c'est-à-dire la quantité d'énergie par unité de temps : donc c'est analogue à des watts (symbole : W).

    Je dis analogue, parce qu'un lumen n'est pas un watt : le lumen reflète le fait que notre œil est plus ou moins sensible à différentes parties du spectre électromagnétique, donc qu'on ne voit pas 1W dans le vert autant que 1W dans le rouge ou le bleu, et si c'est 1W dans l'ultraviolet ou l'infrarouge on ne le voit pas du tout : donc le lumen tient compte de cette différence de sensibilité en pondérant la puissance des différentes longueurs d'onde en fonction de la sensibilité de l'œil humain (mais à cette pondération près, c'est comme des watts).

↑Entry #2780 [older| permalink|newer] / ↑Entrée #2780 [précédente| permalien|suivante] ↑

↓Entry #2778 [older| permalink|newer] / ↓Entrée #2778 [précédente| permalien|suivante] ↓

(dimanche)

Réflexions à 2 femtozorkmids sur l'informatique en tant que science

Les quelques derniers billets de ce blog qui avaient un contenu scientifique (notamment celui-ci et celui-ci) tournaient autour de l'informatique théorique. (Et je compte en écrire encore plusieurs.) Je ne sais pas si le lecteur non spécialiste aura vu la différence avec des maths. Je ne sais pas si je vois une différence avec des maths. Après tout, j'ai rangé ces billets dans la catégorique « maths » (pas que le mécanisme de catégories sur ce blog serve à quoi que ce soit, et c'est peut-être surtout que j'avais la flemme de créer une nouvelle catégorie, mais quand même, c'est une info) ; alors que par le passé, j'ai écrit des entrées sur la physique théorique (comme celle-ci ou celle-là) et je les ai rangées dans une catégorie différente. Peut-être même qu'avec assez de mauvaise foi on pourrait prétendre que la correspondance de Curry-Howard affirme qu'il n'y a pas de différence entre maths et info (mais soyons clairs, ça demande une bonne dose de mauvaise foi, justement).

Ce n'est pas la première fois que je médite sur cette question à laquelle je n'ai pas vraiment de réponse : qu'est-ce que c'est que l'informatique ? — et secondairement, quelle position a-t-elle par rapport aux maths ? et cette position est-elle analogue à celle de la physique ?

(Je précise que ce billet, qui est une méditation décousue et sans véritable point d'arrivée, se veut non-technique, i.e., lisible par tout le monde, mais que fatalement je vais être amené à faire des remarques techniques çà et là : si on ne sait pas de quoi je parle, on peut juste ignorer ces passages, ils ne sont là que pour appuyer ou illustrer mes propos, ils n'en sont pas le cœur. Si vous n'avez vraiment pas le temps de lire, regardez au moins le dernier paragraphe pour un résumé de ce que j'essaie de dire.)

☞ Science informatique et génie informatique

Mettons-nous d'accord sur le fait que je parle de l'informatique en tant que science : computer science en anglais, par opposition à computer engineering, ce dernier étant le génie informatique (lui-même au sens d'ingénierie, pas au sens du fait d'être génial…), comme on peut parler de la physique comme science par opposition au génie mécanique / électrique / électronique / etc. Mais la frontière entre la science et le génie n'est pas super claire non plus, et peut-être que ça fait aussi partie de la question : en première approximation, une science est une discipline qui cherche à élargir nos connaissances tandis que le génie cherche à appliquer des connaissances pour résoudre des problèmes, mais évidemment ce n'est pas toujours net, et il y a des sciences appliquées (que je range quand même parmi les sciences) qui montrent que la limite est assez floue.

Néanmoins, j'aime bien souligner que ce n'est pas parce qu'une limite est floue qu'elle n'existe pas : entre l'étude des modèles du λ-calcul et l'optimisation d'un jeu vidéo il y a bien une différence d'approche, ce qui se mesure ne serait-ce qu'empiriquement au modèle économique qui paye les gens qui font l'une et l'autre activité, et il me semble que cette différence est assez analogue à celle entre, pour ce qui est de la physique, l'étude de la théorie des solutions exactes de l'équation d'Einstein et l'optimisation d'un moteur de moto. (Et je devance l'accusation qu'il y aurait là un jugement de valeur de ma part : d'une part je mets ma vie entre les mains des gens de chez Yamaha qui conçoivent des moteurs de moto, d'autre part j'avoue platement que programmer un jeu vidéo ou concevoir un moteur de moto est quelque chose qui me semble complètement hors de mes capacités à la différence de l'étude des solutions exactes des équations d'Einstein ou des modèles du λ-calcul, que j'ai au moins pratiquées de loin.)

Alors maintenant, si la physique est l'étude du monde matériel, ou quelque chose qui y ressemble, l'informatique, c'est quoi ?

☞ L'informatique n'est pas la science des ordinateurs

La réponse la plus évidente serait que c'est la science des ordinateurs. À ça il faut répondre par une citation souvent attribuée à Dijkstra, mais comme toutes les meilleures citations sont apocryphes, elle n'est probablement pas de lui :

Computer science is no more about computers than astronomy is about telescopes.

Les ordinateurs ne sont que le moyen d'étude de l'informatique, en même temps que son application : ils ne sont pas son objet.

Le grand public a peut-être dans l'idée qu'un chercheur en informatique cherche à rendre les ordinateurs plus rapides. (Bon, à vrai dire, le grand public a peut-être dans l'idée qu'un chercheur en informatique est un pro en Excel ou sait réparer une imprimante qui dysfonctionne, et ce sont des idées à ranger avec celle du mathématicien comme quelqu'un qui passe ses journées à faire des calculs immensément compliqués : juste naïves, mais révélatrices d'un triste manque d'information sur le fonctionnement de la science.) Ce n'est pas complètement faux, bien sûr, il y a des informaticiens qui cherchent effectivement à rendre les ordinateurs plus rapides. D'autres qui cherchent à les rendre plus sûrs. Mais ce sont un peu les arbres qui cachent la forêt.

Bon, mais alors c'est quoi, l'objet de l'informatique, si ce n'est pas les ordinateurs ?

À vrai dire je ne sais pas bien quoi répondre. Ma première approximation de la réponse (peut-être désespérément vague) serait : l'informatique est la science de l'information, des données, de la communication, de l'automatisation et du calcul.

☞ La physique et l'informatique vis-à-vis des maths

L'ennui, si on dit ça, c'est que ça ressemble beaucoup à des maths. Bon, à ce stade, il faut peut-être aussi que je tente de définir les maths, et je suis tout autant embarrassé : c'est essentiellement aussi naïf de s'imaginer que les maths sont la science des nombres que que l'informatique est celle des ordinateurs, mais là aussi, on est en droit de se demander de quoi elles sont l'étude, alors. Je dirais bien l'étude des structures abstraites, mais on m'objectera que ça ressemble trop à la philosophie, ça. En tout cas, l'information, les données, la communication, l'automatisation et le calcul, ça ressemble beaucoup à des notions abstraites que les maths pourraient étudier. Et de fait, il y a une intersection énorme entre les maths et l'informatique théorique ; mais intersection ne signifie pas inclusion c'est un peu le cœur de mon propos.

Peut-être parce qu'elle est née plus récemment que la physique, l'informatique semble avoir une crise identitaire vis-à-vis des mathématiques un peu plus compliquée que celle de la physique. Crise identitaire qui peut signifier qu'elle tarde à prendre son indépendance, ou au contraire prendre la forme d'une volonté excessive de « tuer le père ».

Je pense qu'informatique et physique occupent une place relativement analogue (voire, symétrique) par rapport aux mathématiques. Les deux sont à la fois une inspiration pour les mathématiques et un domaine d'application de celles-ci, et, dans leur partie la plus théorique, elles intersectent largement les maths. (Elles ont aussi, dans leur incarnation moderne, émergé des mathématiques : Turing était mathématicien au moins autant qu'informaticien, comme Newton était mathématicien au moins autant que physicien.) Les maths qui intéressent l'informatique (par exemple la logique et l'algèbre discrète) et les maths qui intéressent la physique (par exemple la géométrie différentielle et la théorie des groupes continus) sont généralement distinctes, mais il y a de grands domaines qui intéressent les deux (comme l'analyse de Fourier ou les probabilités).

☞ Une digression sur mon papa physicien

Je devrais peut-être faire une digression à ce sujet pour parler de mon papa, qui était physicien théoricien, et faisait de la physique qui pourrait franchement être considérée comme des maths (genre, ça), mais qui avait une relation difficile avec les maths et les mathématiciens (il aimait bien citer Feynman qui comparait la rigueur mathématique à la rigor mortis ; il faudrait que je vérifie si elle n'est pas apocryphe aussi celle-là, mais en tout cas ça montre que si son sujet d'étude était très mathématique, son approche n'était pas vraiment celle d'un mathématicien).

Mon papa était en outre persuadé que la physique était la source d'inspiration des maths, et que toutes les idées mathématiques naissaient soient de la recherche d'applications à la physique soit de généralisations assez évidentes de ça. J'aurais dû lui rétorquer plein de choses quand il me disait ça (par exemple, parmi les choses qu'il aurait dû bien savoir en tant que relativiste, que la géométrie riemannienne était là bien avant qu'Einstein n'en ait besoin pour décrire un espace-temps courbe), mais je pense qu'il reflétait là surtout son propre biais d'intérêt dans les domaines qu'il connaissait des mathématiques (à chaque fois que je lui ai parlé de corps de caractéristique p>0, par exemple, il haussait les yeux au ciel comme si c'était vraiment une lubie saugrenue d'étudier des choses aussi déconnectées du monde réel dans lesquelles en ajoutant p fois le nombre 1 on retombe sur 0). C'est intéressant, donc, parce que mon père ne semblait pas du tout avoir considéré cette possibilité que l'informatique soit, de façon analogue à la physique, elle aussi une source féconde d'inspiration en même temps que de débouchés pour les maths. Mais c'est aussi intéressant parce que sa relation compliquée avec les maths, qui dans son cas était plutôt idiosyncratique, est quelque chose que je crois retrouver de façon collective dans des tentatives de trouver un démarquage entre les maths et l'info pour des disciplines qui sont à cheval entre les deux.

☞ L'enseignement de l'informatique comme science

Il me faut sans doute d'abord dire un mot de l'enseignement. Ne sachant pas comment ça se passe ailleurs, je vais évoquer le cas de la France.

Malgré N timides tentatives par le passé, n'ayant jamais vraiment abouti, ce n'est que tout récemment (~2018, peut-être ?) que l'enseignement secondaire français s'est doté, sous le nom de numérique et sciences informatiques (NSI), d'un véritable enseignement (optionnel…) de l'informatique. (Cela me semble impossiblement tardif, mais je crois qu'il vaut mieux que je ne regarde pas à quel moment historique on a commencé à enseigner la physique dans les lycées français, parce que j'ai peur de faire une attaque.) N'ayant pas de contact direct avec le monde de l'enseignement secondaire, je ne peux juger que par les programmes, qui sont un peu bizarres (p.ex., l'enseignement du SQL me semble vraiment bizarrement précis à côté de généralités très vagues sur d'autres plans) mais néanmoins intéressants par certains côtés. Donc je ne vais pas en dire plus.

☞ La génération Z et l'informatique

Même si ce n'est que tangentiel à mon propos, comme c'est quelque chose qui revient régulièrement si on évoque la nécessité de l'enseignement de l'informatique au lycée, il faut que je dénonce un lieu commun, qui dit quelque chose comme ceci :

❝Les jeunes, de nos jours, ils n'ont pas besoin qu'on leur enseigne l'informatique, parce qu'ils ont grandi avec les ordinateurs. D'ailleurs, ils en savent certainement plus que leurs profs de lycée.❞

Il y a tellement de choses fausses là-dedans que je ne sais pas bien par où commencer. Par le fait que l'informatique n'est pas la science des ordinateurs ? Je l'ai déjà signalé. Mais je pourrais aussi remarquer le ridicule de l'idée analogue que parce qu'on a tous grandi avec les voitures thermiques on saurait expliquer comment fonctionne un moteur à explosion[#].

[#] Je serais d'ailleurs sincèrement curieux de savoir quelle proportion de la population prise au hasard dans la rue serait capable d'expliquer approximativement le principe d'un moteur à explosion (disons par exemple, de nommer les quatre phases du cycle d'un moteur à quatre temps). Mais j'ai déjà souligné l'incongruité du fait que ce genre de connaissances ne soient pas considérées comme faisant partie de la culture générale.

Un autre problème avec ce cliché est que, en fait, la génération qui a grandi avec les ordinateurs, c'est plutôt la mienne et la suivante ; la génération de ceux nés au XXIe siècle, ils ont grandi avec des smartphones (et autres interfaces graphiques complètes), et c'est bien différent. Or les interfaces graphiques comme celles des smartphones, comme leur but était de mettre l'outil entre les mains de tout le monde en évitant que Monsieur Toutlemonde ait à connaître quoi que ce soit du fonctionnement interne de la machine, elles ont tout fait pour cacher (dans des skeuomorphes plus ou moins heureux) la moindre chose qui ressemble à de la programmation. En même temps que le monde informatique derrière est devenu de plus en plus compliqué par l'empilement de couches séparant l'utilisateur du matériel (et ceci vaut aussi pour les vrais PC, pas seulement pour les smartphones), l'accès de l'utilisateur à ces couches et devenu de plus en plus compliqué.

Donc je ne pense pas du tout qu'un jeune né en 2000–2010 soit automatiquement doué pour la programmation, encore moins pour comprendre la notion abstraite de machine de Turing, pas plus que quelqu'un qui a grandi avec les voitures modernes — par opposition à la Ford modèle T — est automatiquement doué pour la mécanique auto, encore moins pour expliquer la thermodynamique du cycle d'Otto.

(Loin de moi l'idée de faire mon vieux con en mode c'était mieux âââvant, hein ! Je n'ai rien contre les interfaces graphiques pas plus que je n'en ai contre les voitures modernes. Si on n'est pas obligé de taper mkdir pour créer un répertoire, c'est très bien. Le problème c'est plutôt quand on ne peut même pas taper mkdir pour créer un répertoire. Je ne suis pas non plus en train de reprocher à la génération Z de ne rien savoir faire en-dehors de se servir de leur smartphone — autre cliché à la con que je déteste : je dis juste qu'il ne faut pas automatiquement supposer que grandir avec un smartphone rend ipso facto compétent pour se servir d'un ordinateur, encore moins pour comprendre le fonctionnement d'un ordinateur ou la théorie derrière.)

☞ L'informatique en prépa et à l'agreg

S'agissant de la formation des futurs ingénieurs (i.e., post-baccalauréat), les classes préparatoires scientifiques françaises ont tout récemment (à la rentrée 2022) vu la création d'une filière MPI (pour maths, physique, informatique), qui ouvre un véritablement enseignement de l'informatique en tant que science en classes prépa, à côté des filières MP (maths-physique) et PC (physique-chimie). Un de mes collègues a d'ailleurs participé à l'écriture des programmes de cette filière (du coup, la partie sur les automates finis est fortement inspirée d'un cours que je donnais sur ce sujet à Télécom[#2] et dans lequel ce collègue intervenait). Je suis certainement favorable à cette évolution, et au fait que l'informatique soit traitée comme une vraie manière autonome et pas une sorte d'excroissance des maths ni une distraction proposée en marge des cours et visant uniquement les applications dans d'autres domaines. Pour autant, je pense qu'il faut aussi se méfier de la fausse impression que cela pourrait causer dans l'esprit des préparationnaires qui risquent de croire faussement que parce que c'est de l'informatique ce n'est pas des maths (cf. ce que je dis plus bas sur la grande intersection entre ces domaines) : ils risquent du coup de se construire une image mentale trop exiguë des mathématiques (limitée en gros à l'algèbre linéaire et à l'analyse classique, ce qu'on leur enseigner en cours de maths), alors qu'en fait de grands pans de ce programme informatique auraient très bien pu être enseignés dans un cours de maths.

[#2] Cours dont voici les notes et ici une vidéo des séances de 2020–2021. À ce sujet, en avril 2021 on m'a demandé d'assurer un cours sur le sujet destiné aux profs de prépa (et qui va donc un peu au-delà du programme). Mes slides sont ici, et les vidéos de l'exposé lui-même sont ici pour les 2h du matin et ici pour les 2h de l'après-midi.

Cette création d'une filière MPI en prépa scientifique fait suite de peu à la création d'une agrégation d'informatique (premier concours à l'été 2022). Auparavant, il y avait eu une option informatique à l'agreg de maths (option proposée à l'écrit entre ~1990 et ~1997 je crois, puis à l'oral entre ~2006 et 2022 ; entre les deux il n'y avait rien). Il y a certainement lieu de s'interroger sur la question de savoir s'il était idéal d'avoir une agreg complètement différente plutôt qu'une option, ou une solution intermédiaire (une question analogue se pose certainement aussi sur le rapport entre physique et chimie, ou entre biologie et géologie), ou d'ailleurs s'il était évident que la création d'une agreg d'informatique dût entraîner la suppression de l'option informatique de l'agreg de maths. (Bizarrement, en prépa, il continue à exister une option info en filière MP, qui est distincte de la filière MPI : c'est confusant, pas forcément super cohérent, et un peu compliqué à gérer en aval dans l'enseignement, mais je suis tenté de croire que cette multiplicité des choix a des vertus.)

Sans vouloir suggérer que c'est un mauvais choix scientifique ou pédagogique (au contraire), je pense que ces décisions relèvent largement d'une décision d'affirmer l'indépendance de l'informatique comme discipline scientifique à part entière, et de ne pas laisser l'idée que c'est une sorte d'appendice des mathématiques.

☞ Le problème des limites entre sciences

Au-delà de l'enseignement, parlons un peu de la science pour la science.

Les frontières entre disciplines sont, évidemment, un peu arbitraires. D'abord, je crois fermement à l'unité fondamentale de tous les domaines du savoir humain, c'est-à-dire au moins que je suis sûr qu'on peut passer de proche en proche de la théorie des cordes à la philologie grecque, ou de la biochimie à la musicologie, sans jamais rencontrer de barrière nette. Si cela peut paraître farfelue, je rappelle au passage qu'un concept très important de l'informatique théorique (à l'intersection avec les maths), la notion de grammaire hors contexte a été inventé par un linguiste[#3] (d'ailleurs peut-être plus connu pour son activisme politique), Noam Chomsky.

[#3] Je sais que l'importance de cette notion en linguistique (ou de façon générale toute la linguistique chomskienne, et la notion de grammaire universelle) est assez controversée. Je m'abstiens explicitement de me prononcer sur ces questions sur lesquelles je suis incompétent. Mais quoi qu'on en pense dans le cadre de la linguistique, en informatique théorique c'est une notion fondamentale.

Mais je ne veux pas tomber dans le sophisme des sorites que je ne cesse de dénoncer : ce n'est pas parce que la frontière est floue et/ou largement arbitraire qu'elle n'existe pas, ou qu'on n'a pas le droit de discuter le meilleur endroit où la faire passer.

Les limites administratives entre sciences causent toutes sortes de maux de tête, par exemple pour les candidats à un poste académique dont la spécialité est proche d'une de ces frontières arbitraires : a priori on pourrait imaginer que cela tourne à leur avantage (par exemple, quelqu'un qui fait quelque chose à cheval entre les maths et l'info pourrait candidater aussi bien en maths qu'en info), mais dans la réalité c'est presque toujours un handicap (il y aura toujours quelqu'un pour vous reprocher de faire des choses trop informatiques quand vous candidatez en maths et de faire des choses trop mathématiques quand vous candidatez en info, avec le prétexte que vous auriez dû candidater dans l'autre domaine ; je pense que ceci vaut à chaque fois qu'il y a une interface de ce genre : sur le papier tout le monde dit du bien de la transdisciplinarité, mais dans la réalité ça a tendance à nuire aux candidats de ne pas rentrer dans les bonnes petites cases). Mais ce n'est pas tellement de ça que je veux parler.

Donc, l'informatique fait-elle partie des mathématiques ? Une partie de l'informatique fait-elle partie des mathématiques ? Où est le meilleur endroit pour tracer la limite ?

↑Entry #2778 [older| permalink|newer] / ↑Entrée #2778 [précédente| permalien|suivante] ↑

↓Entry #2774 [older| permalink|newer] / ↓Entrée #2774 [précédente| permalien|suivante] ↓

(dimanche)

Une énigme avec un dragon et de la calculabilité

Allez, en cadeau de Noël, un deuxième billet à deux jours d'intervalle, ce qui est devenu très rare sur ce blog : je vous propose une devinette dont je posterai la solution ultérieurement. Je la trouve très jolie et surprenante et je pense qu'elle aurait beaucoup plu à Raymond Smullyan.

Méta : Elle m'est venue en réfléchissant à des questions de maths (autour de la réalisabilité propositionnelle). La formulation est de moi, mais c'est en fait une réécriture d'un théorème (de Plisko, je donnerai la référence précise en même temps que la solution) dont j'ai lu l'énoncé et que j'ai reformulé comme une énigme pour y réfléchir parce que je ne voulais pas lire la preuve mais la retrouver moi-même. J'ai eu énormément de mal à la résoudre, donc je pense qu'elle est vraiment dure (d'un autre côté, je n'étais pas sûr d'avoir trouver la bonne formulation, donc je réfléchissais à la fois à la question et à la réponse), mais quand je l'ai présentée sur Twitter, au moins une personne a trouvé la solution, donc elle n'est certainement pas insoluble. Même si elle a un rapport distant avec mon très long billet tout récent sur Curry-Howard (et je compte écrire une autre entrée qui fait référence aux deux, et qui explique l'intérêt mathématique de cette énigme), il n'est pas nécessaire, ni même spécialement utile, d'avoir lu ce billet (ni aucun autre) pour réfléchir à cette énigme. En revanche, il est certainement pertinent d'avoir des notions de calculabilité pour espérer y arriver (même si je vais présenter l'énoncé pour qu'il soit compréhensible sans ça). J'éditerai ce billet d'ici quelques jours pour présenter la réponse ainsi que quelques commentaires : pour l'instant je me contente de l'énoncé de l'énigme, suivi de quelques commentaires.

L'énigme

Vous êtes un aventurier dans un donjon. Devant vous se trouvent trois portes, identiques à part leur étiquette : A, B et C. Derrière une de ces portes se trouve un dragon, qui vous dévorera si vous ouvrez la porte, mais vous ne savez pas laquelle. Votre but est d'ouvrir une des portes « sûres », c'est-à-dire sans dragon (et qui conduisent à la sortie et, soyons généreux, à un trésor pour vous récompenser d'avoir résolu l'énigme). Peu importe quelle porte sûre vous ouvrez, la seule chose qui importe est de ne pas ouvrir la porte au dragon.

Pour vous aider à trouver une porte sûre, il y a un indice apposé sur chaque porte. Mais comme ceci est un donjon très moderne, pas ces trucs poussiéreux des magiciens d'il y a quelques siècles, chaque indice prend la forme d'un programme informatique. Ça tombe bien, vous avez un ordinateur capable de faire tourner ces programmes (ou pour en examiner le contenu si vous le voulez).

L'idée générale est que le programme apposé sur chaque porte devrait, quand vous l'exécutez, vous indiquer une des deux autres portes qui soit sûre. Mais ce n'est pas si simple ! Car le programme demande quelque chose en entrée, et ne fonctionnera correctement que si on lui a fourni le bon quelque chose. Et ce quelque chose est aussi un programme (qui, lui, ne prend rien en entrée).

Les règles précises sont les suivantes :

  • Le programme apposé sur la porte au dragon, quoi qu'on lui passe en entrée, va terminer quoi qu'il arrive et produire comme résultat l'étiquette d'une des deux autres portes. (Comme il n'y a qu'un dragon, les deux autres portes sont sûres, donc ce sera de toute façon une porte sûre. Son choix peut dépendre de ce que vous avez donné en entrée, mais il doit produire un résultat dans tous les cas, même si le programme qu'on lui a fourni en entrée n'a pas de sens ou ne termine pas ou quoi que ce soit du genre.)

  • Le programme apposé sur une porte « sûre » va terminer et produire comme résultat l'étiquette de l'autre porte sûre (il n'y en a qu'une autre), mais à condition qu'on lui ait fourni comme entrée un autre programme qui lui-même (exécuté sans entrée) produit comme résultat l'étiquette de l'autre porte sûre. Si cette condition n'est pas remplie, le comportement du programme n'est pas précisé (il pourrait ne pas terminer, produire une mauvaise porte, ou afficher 42 ou n'importe quoi d'autre ; mais il ne va pas vous tuer, quand même : c'est un programme, pas un dragon, et votre ordinateur peut le faire tourner sans risque).

Je redis ça pour être bien sûr d'avoir été clair. Appelons X la porte au dragon, et Y,Z les deux autres portes. Appelons pX,pY,pZ les programmes apposés aux portes X,Y,Z respectivement, et q un programme qu'on décide de passer en entrée à un de pX,pY,pZ. Les garanties sont : d'abord, concernant la porte au dragon, pX(q) termine quoi que soit q et produit comme résultat soit Y soit Z ; ensuite, concernant les deux autres portes : si q termine et produit Z comme résultat alors pY(q) termine et produit lui aussi Z comme résultat, et de même, si q termine et produit Y comme résultat alors pZ(q) termine et produit Y comme résultat.

Vous avez bien sûr le droit de lancer les programmes plusieurs fois (séquentiellement ou en parallèle) avec des entrées différentes. Vous avez le droit de créer vos propres programmes (par exemple écrire un programme qui produit toujours la sortie B, et fournir ça au programme de la porte C est légitime ; ou même faire un programme qui exécute le programme de la porte C sur le programme qui produit toujours B en sortie, et fournir ça au programme de la porte A : tout ça est légitime). Vous avez aussi tout le temps nécessaire, mais il faut quand même arriver à un résultat de façon certaine au bout d'un temps fini (donc on a le droit d'attendre longtemps la sortie d'un programme, mais s'il ne termine jamais c'est un échec).

En fait, votre réponse sera elle-même algorithmique. C'est-à-dire que techniquement, ce que je demande c'est d'écrire un programme qui prend en entrée les programmes pX,pY,pZ apposés aux trois portes et qui, quelle que soit la porte au dragon et quels que soient pX,pY,pZ vérifiant les contraintes, termine et produit en sortie l'étiquette d'une porte qui n'a pas de dragon.

Voilà, c'est un peu long, mais j'espère que c'est parfaitement clair. S'il y a quelque chose qui ne l'est pas, dites-le moi en commentaire et j'essaierai de préciser les règles.

Résumé : Il y a une seule porte avec un dragon et on ne sait pas laquelle, les deux autres sont sûres et le but est de trouver une porte sûre. Chaque porte a un programme apposé comme indice. Ce programme termine et produit comme résultat l'étiquette d'une des deux autres portes qui soit sûre ; mais, s'agissant du programme apposé sur une porte sûre, ceci n'est garanti que sous condition qu'on lui passe comme entrée un programme (sans entrée) qui lui-même termine et produise comme résultat l'étiquette de l'autre autre porte qui soit sûre.

Comment vous tirez-vous (de façon certaine) de cette fâcheuse situation ?

Pourquoi elle semble insoluble

J'ai fini l'énoncé de l'énigme et on peut s'arrêter de lire là si on veut y réfléchir ; mais je vais quand même faire un commentaire qui, sans vraiment constituer un indice, attire l'attention sur ce qui est difficile dans cette énigme.

↑Entry #2774 [older| permalink|newer] / ↑Entrée #2774 [précédente| permalien|suivante] ↑

↓Entry #2773 [older| permalink|newer] / ↓Entrée #2773 [précédente| permalien|suivante] ↓

(vendredi)

Une introduction à la correspondance de Curry-Howard

Méta : Le présent billet, quoique indépendant, est une sorte de suite de celui-ci où j'exposais quelques points d'informatique théorique que j'ai appris, ou mieux/enfin compris, en enseignant un cours que je crée cette année à Télécom. En fait, j'étais parti pour écrire quelque chose sur un sujet plus spéculatif, mais je me suis dit que pour commencer il fallait que j'explique proprement la correspondance de Curry-Howard, et comme c'est justement une des choses que j'enseigne et que je trouve généralement très mal expliqué, autant en faire aussi une entrée de blog ; puis comme j'étais trop débordé par l'écriture des transparents du cours lui-même, j'ai laissé ça de côté. J'ai de nouveau un petit peu de temps avec les vacances, donc je publie et complète séparément ce bout sur Curry-Howard comme un texte autonome, dans l'espoir de faire ensuite un autre billet sur les questions qui, au-delà de mon enseignement du moment, m'avaient initialement motivé à en parler. Mais comme je tombe dans un trou de lapin dès que je me mets à écrire sur un sujet, j'ai quand même élargi le sujet à des questions connexes.

Table des matières

Introduction

[Sean Connery dans le rôle de Guillaume de Baskerville dans le film “Le Nom de la Rose”] Preuves mathématiques
(contemplatives ?)

[Sean Connery dans le rôle de Zed dans le film “Zardoz”] Programmes informatiques
(dynamiques ?)

Bref, mon but est d'exposer (avec des prérequis aussi limités que possible) la correspondance de Curry-Howard entre preuves et programmes.

De quoi s'agit-il ? On résume parfois la correspondance de Curry-Howard en disant que, comme un bon acteur capable de prendre des rôles très différents, les preuves mathématiques et les programmes informatiques sont, en fait, fondamentalement la même chose (et d'ailleurs je suis très content de l'illustration ci-contre).

À vrai dire, ce raccourci, qui a le mérite d'être mémorable, est un peu… raccourci. Ce ne sont pas n'importe quelles preuves et surtout, ce ne sont pas n'importe quels programmes. Côté preuves, il s'agit a priori de preuves en logique intuitionniste (c'est-à-dire sans le tiers exclu ; je vais définir les choses précisément ci-dessous), et même si on peut étendre Curry-Howard à la logique classique grâce à la fonction call/cc, la correspondance obtenue n'est peut-être pas aussi satisfaisante. Côté programmes, il s'agit de programmes écrits dans des langages fortement typés dans lesquels la terminaison de tout programme est garanti : donc ce ne sont pas du tout les langages de programmation habituels, lesquels permettent des boucles infinies et/ou des appels récursifs illimités[#] ; il n'y a rien non plus qui corresponde à des effets de bord (modification de valeurs, entrées-sorties, rien de tel). Parmi les limites de Curry-Howard, il y a aussi des différences subtiles dans la signification de la quantification existentielle (‘∃’) en logique par rapport aux types sommes (‘Σ’) qui vont faire l'objet de mon « addendum 1 » plus bas.

Toujours est-il que, malgré ces limitations, Curry-Howard établit une correspondance entre, côté logique propositions et preuves de ces propositions, et, côté informatique, types dans un langage informatique fortement typé et termes (i.e., programmes) de ces types. À un certain niveau c'est même une trivialité (observer qu'on exactement les mêmes règles de formation des deux côtés : c'est juste deux façons de voir le même acteur, cet acteur étant une sorte de λ-calcul qui peut servir soit à désigner des preuves soit à coder des programmes) ; mais quand même, cette correspondance permet d'interpréter une preuve comme quelque chose qui va s'exécuter, prendre des données en entrée et renvoyer des résultats à la sortie, et c'est assez frappant, et c'est l'essence de ce qui permet d'extraire d'une preuve constructive un programme réalisant le calcul qu'elle prétend construire (j'agite un peu les mains ici). À titre d'exemple, le programme qui correspond à la preuve « évidente » de AB ⇒ BA (si A et B sont vrais, alors B et A sont vrais) est le programme qui prend en entrée un couple et échange les deux coordonnées de ce couple (lesquelles peuvent être de types arbitraires).

[#] Comme je l'ai déjà signalé ici, autoriser ces appels récursifs revient très précisément, au niveau preuves, à autoriser les raisonnements du style suivant : Je veux démontrer que les poules ont des dents. Je tiens l'affirmation suivante : Si j'ai raison, alors les poules ont des dents. Clairement, si j'ai raison (c'est-à-dire, si cette affirmation est vraie), alors les poules ont des dents. Mais c'était justement mon affirmation : donc j'ai raison. Donc, par ce qui vient d'être démontré, les poules ont des dents. Il est peut-être intéressant de s'intéresser à des logiques paraconsistantes dans lesquelles une telle démonstration serait permise (et du coup, tout est démontrable, de même que dans un langage de programmation usuel on peut fabriquer un programme de n'importe quel type — même le type vide, c'est juste que le programme ne terminera pas), mais ce n'est pas ce qu'on appelle usuellement une « démonstration » mathématique.

De façon un petit peu plus détaillée, la correspondance de Curry-Howard met en regard (je vais essayer d'expliquer ces différents points dans la suite, donc ceci est un divulgâchis qui a pour but d'aider à se retrouver dans la masse de mes explications si on décide de la lire en diagonale) :

  • propositions avec types ;
  • preuves avec termes (= programmes) ;
  • implication logique (PQ) avec types fonctions (στ, qui correspond aux fonctions prenant une valeur de type σ et renvoyant une valeur de type τ) ;
  • modus ponens (application d'une implication : si PQ et P alors Q) avec application d'une fonction (une fonction στ s'applique à un type σ pour donner un type τ) ;
  • ouverture d'une hypothèse (pour démontrer une implication) avec λ-abstraction (= définition d'une fonction à partir d'un argument) ;
  • hypothèses actuellement ouvertes avec variables libres (en contexte) dans le programme ;
  • hypothèses déchargées avec variables liées dans le programme ;
  • conjonction (et logique : PQ) avec types produits (σ×τ, qui correspond aux données d'un couple d'une valeur de type σ et d'une valeur de type τ) ;
  • disjonction (ou logique : PQ) avec types produits (σ+τ, qui correspond aux données soit d'une valeur de type σ soit d'une valeur de type τ, avec un sélecteur qui indique dans quel cas on se trouve) ;
  • le vrai (noté ‘⊤’, affirmation tautologiquement vraie) avec le type unité (ayant une seule valeur, triviale) ;
  • le faux (noté ‘⊥’, affirmation tautologiquement fausse) avec le type vide (n'ayant aucune valeur) ;
  • quantification universelle ∀(v:U).P(v) (qui est une sorte de « conjonction en famille » ⋀v:U P(v) sur tous les v de type U) avec types produits en famille ∏v:U σ(v) (type des fonctions prenant un v de type U et renvoyant une valeur de type σ(v)) ;
  • quantification existentielle ∃(v:U).P(v) (qui est une sorte de « disjonction en famille » ⋁v:U P(v) sur les v de type U) avec types sommes en famille ∑v:U σ(v) (type des données d'un v₀ de type U et d'une valeur de type σ(v₀)).

La correspondance est surtout satisfaisante dans le sens des preuves vers les programmes, i.e., donnée une démonstration, arriver à construire un programme. C'est cette direction que je vais essayer d'exposer ci-après, en laissant globalement de côté la question de si on obtient tous les programmes de cette façon (ce qui demanderait d'expliquer précisément ce que sont « tous les programmes », et j'ai peur qu'on ne puisse avoir qu'une réponse un peu triviale en prenant la définition de programme qui fait rend Curry-Howard bijectif, c'est-à-dire de prendre pile-poil ceux qu'on peut fabriquer de la sorte : c'est en quelque sorte tout l'objet du typage que de limiter les programmes écrivibles à ceux que la correspondance de Curry-Howard atteint).

Bref, mon but dans la suite est d'exposer ça plus précisément, d'abord dans le cas purement propositionnel, c'est-à-dire sans quantificateurs (c'est nettement plus simple et si ça peut sembler ennuyeux je pense qu'il a déjà énormément à nous apprendre), et ensuite dire des choses plus vagues sur les quantificateurs (les quantificateurs posent toutes sortes de difficultés pénibles sur ce qu'on a le droit de quantifier exactement), en regardant d'un peu plus près la logique du premier ordre (qui est la forme de quantification la plus simple) ; je finis par parler de deux points indépendants (et que je ne comprends que de façon très imparfaite, donc je serai encore plus vague et à la limite de l'agitage de mains) : la question du sens du ‘∃’ d'une part, et l'imprédicativité de l'autre.

L'exposition que je vais faire ici suit pour le début la même idée que mon cours, pour laquelle on peut se référer à ce jeu de transparents pour la partie propositionnelle, mais je vais plutôt partir du côté preuves et ne pas entrer dans une description précise de ce qu'est le λ-calcul simplement typé. En revanche, la fin de ce billet va nettement au-delà de ce que je compte exposer dans mon cours (la partie de mon cours sur les quantificateurs est encore à l'état d'ébauche au moment où j'écris ce paragraphe, mais de toute façon je ne vais pas dire grand-chose).

Puisque ce sera le côté « preuves », je commence par des explications sur la logique intuitionniste (propositionnelle pour commencer) et sur ses règles de démonstration : on peut aussi se référer à ce billet passé pour le contexte général (y compris historique) sur les maths constructives, et cet autre billet (assez mal écrit) pour une description un peu différente des règles de la logique (en style « calcul des séquents »).

↑Entry #2773 [older| permalink|newer] / ↑Entrée #2773 [précédente| permalien|suivante] ↑

↓Entry #2771 [older| permalink|newer] / ↓Entrée #2771 [précédente| permalien|suivante] ↓

(vendredi)

Où j'apprends aussi l'informatique en l'enseignant

Ce que j'aime surtout dans le fait d'enseigner, c'est qu'on apprend souvent soi-même plein de choses sur le sujet qu'on enseigne, surtout la première fois, et surtout si on crée soi-même le programme du cours : on croyait bien savoir les choses de loin, mais être obligé de les regarder de près pour les enseigner oblige souvent à se rendre compte qu'il y avait une subtilité là où on ne le savait pas (ou qu'on avait oubliée) ; ça peut éventuellement conduire à une panique zut, l'approche que je croyais avoir tracée pour ce cours ne marche pas ! comment est-ce que je vais m'en sortir ?, mais souvent on en ressort bien plus compétent qu'avant : même si la subtilité qu'on a découverte, finalement, n'est pas enseignée, l'enseignant sera quand même meilleur (ceci étant, les personnes à qui on enseigne la première fois risquent de moins goûter l'expérience, et il faut parfois quelques années pour que la découverte de surprises se tarisse et que l'enseignant soit rôdé).

Pour prendre un exemple passé précis, en enseignant mon cours de théories des jeux (notez le pluriel à théories !) à Télécom — dont les notes sont ici si ça intéresse quelqu'un — je me suis rendu compte d'une subtilité importante dont je n'avais pas du tout pris conscience avant : quand on parle de stratégie dans un jeu (à information parfaite), on peut soit s'intéresser aux stratégies qui, pour prescrire un coup à jouer, dépendent seulement de la configuration actuelle du jeu (appelons-les stratégies positionnelles), soit de celles, plus générales, qui dépendent de tout l'historique de la confrontation jusqu'à ce point (appelons-les stratégies historiques) ; or, même si la règle du jeu ne dépend que de la configuration actuelle et pas de l'historique qui a mené à cette configuration, ce n'est pas du tout évident a priori qu'une stratégie gagnante dans le sens plus général (historique) soit forcément traduisible en une stratégie gagnante dans le sens plus restreint (positionnel) ; et j'avais un peu paniqué, parce que j'avais construit mon cours autour de la démonstration de l'existence d'une stratégie gagnante dans un sens via la théorie des jeux de Gale-Stewart (qui sont intrinsèquement « historiques »), alors que j'en avais besoin ensuite au sens plus fort (« positionnel ») pour la théorie de Sprague-Grundy. J'ai réussi à m'en sortir (j'ai écrit des notes expliquant correctement le lien, voir la section 3.5 du PDF lié ci-dessus), en cours je demande aux élèves d'admettre ce point en les renvoyant au poly s'ils ne veulent pas l'admettre, donc finalement je n'enseigne pas vraiment cette subtilité, mais j'ai été content d'en prendre conscience, et je suis étonné qu'il soit si peu évoqué, ou si mal expliqué, dans la littérature scientifique sur le sujet.

Même en donnant un cours d'analyse (dont je n'étais, cette fois, pas le responsable ni le créateur, mais simple intervenant) j'ai appris un certain nombre de choses sur le sujet, voyez par exemple ce billet passé pour quelque chose que j'ai appris au détour d'un exercice. Et en donnant un cours d'initiation à la géométrie algébrique, je me rends surtout compte… que c'est très difficile d'enseigner la géométrie algébrique et que je ne sais même pas produire une définition d'un morphisme entre variétés quasiprojectives (l'objet fondamental du domaine, quoi) qui ne soit pas abominable dans sa complication.

Bref. Cette année, comme je l'ai raconté ici et , j'inaugure un cours intitulé Logique et Fondements de l'Informatique où je dois parler de calculabilité, λ-calcul, logique, typage, isomorphisme de Curry-Howard, ce genre de choses (un programme à vrai dire assez ambitieux, mais on m'a donné comme consigne de profiter de la création de la filière MPI en prépa pour viser quelque chose d'assez sérieux). Des choses, à cheval entre mathématiques et informatique théorique, qui me plaisent beaucoup (sur la calculabilité voyez par exemple ce long billet) et sur lesquelles je pensais ne pas avoir grand-chose à apprendre… Famous last words!

(La suite de ce billet est essentiellement un brain dump de quelques unes des choses que j'ai apprises, réapprises, ou mieux comprises en préparant ce cours jusqu'à présent. Noter que ça ne veut pas dire que je vais les enseigner ! Ou du moins pas forcément sous cette forme. Comme les petits bouts de ce billet, de longueur très inégales, sont assez indépendants, je les ai séparés par des fleurons : si on n'aime pas ce que je raconte à un endroit donné, on peut sauter jusqu'au fleuron suivant.)

Il y a des choses que j'ai apprises en marge de la préparation de ce cours, sans que ça ait vraiment d'impact dessus. Par exemple, comment on fabrique des modèles du lambda-calcul non typé (pour ceux qui veulent en savoir plus là-dessus, j'ai bien aimé l'article From computation to foundations via functions and application: The λ-calculus and its webbed models de Chantal Berline, et notamment la partie sur les K-modèles, qui m'ont semblé les plus parlants de tout cette histoire : de façon très sommaire, on représente un terme par l'ensemble de tous les types qu'on peut lui attribuer dans un certain système de typage dont les types ne sont pas disjoints ; en plus ça semble avoir un rapport avec la réalisabilité, ce qui n'est pas déplaisant) : ce n'est pas spécialement une difficulté que j'ai rencontrée, juste quelque chose que j'ai appris au passage.

Il y a aussi des choses que je pensais savoir sans vraiment y avoir suffisamment réfléchi. Notamment, comment fonctionne au juste le combinateur de point fixe Y de Curry. J'avais déjà appris il y a longtemps que dans un langage fonctionnel non typé on peut faire des appels récursifs sans faire d'appels récursifs, par exemple (en Scheme) :

(define proto-fibonacci
  (lambda (self)
    (lambda (n) 
      (if (<= n 1) n
	  (+ ((self self) (- n 1)) ((self self) (- n 2)))))))
(define fibonacci (proto-fibonacci proto-fibonacci))

— code la fonction définie récursivement[#] par F(n)=n si n≤1 et F(n) = F(n−1) + F(n−2) sinon, sans jamais que la fonction fasse appel à elle-même dans sa définition (c'est, en fait, l'astuce de Quine — qui n'est pas due à Quine, c'est Hofstadter qui lui a donné ce nom-là, mais à Cantor, Gödel, Turing et Kleene — et sur laquelle j'avais écrit une page alors que mes élèves actuels n'étaient même pas nés). L'astuce (de Quine qui n'est pas de Quine), donc, c'est qu'on passe la fonction proto-fibonacci en argument à la fonction proto-fibonacci, et quand elle a besoin de faire appel à elle-même, elle applique son argument (self) en prenant bien soin de lui passer une copie de lui-même, d'où le self self dans ce code.

[#] Oui, je sais que la suite de Fibonacci est un très mauvais exemple de récursion parce qu'en vrai il ne faut pas la coder de façon récursive, c'est épouvantable pour la complexité ; mais c'est un exemple facile à lire, donc je le reprends avec cet avertissement qu'il ne faut pas faire comme ça pour autre chose que pour illustrer les appels récursifs.

Vous noterez bien, donc, qu'il n'y a pas d'appels récursifs dans ce code. La même astuce de Quine permet de faire un programme qui s'écrit lui-même même si le langage ne permet pas de faire référence à lui-même (voyez ma vieille page liée ci-dessus pour tous les détails), à Gödel de fabriquer un énoncé qui dit je suis indémontrable, etc. Ici ça sert à ce qu'une fonction s'appelle elle-même même si le langage ne permettait pas les appels récursifs comme construction spéciale (par exemple en λ-calcul, il n'y a pas de construction récursive).

Ici on est dans un langage fonctionnel donc on peut juste appeler une fonction passé en argument, mais dans un langage non fonctionnel capable d'écrire un interpréteur de lui-même, on pourrait quand même simuler les appels récursifs en lançant l'interpréteur (sur une représentation du code de la fonction !) à chaque fois qu'il est écrit self self dans le code ci-dessus, ce qui est la façon la plus mind-blowing de faire de la récursion, et je ne m'étais pas rendu compte de ça avant de commencer à préparer ce cours.

↑Entry #2771 [older| permalink|newer] / ↑Entrée #2771 [précédente| permalien|suivante] ↑

↓Entry #2769 [older| permalink|newer] / ↓Entrée #2769 [précédente| permalien|suivante] ↓

(dimanche)

Étude critique de vulgarisation mathématique : une petite vidéo d'Arte

La chaîne de télé Arte produit une série de petites vidéos de vulgarisation scientifique (à destination du grand public) sur les mathématiques appelée Voyages au pays des maths (on les trouve ici sur le site web d'Arte ou ici sur YouTube) : je n'en ai regardé qu'une partie (via leur site Web : je ne suis pas tombé dessus à l'antenne, mais c'est juste parce que je n'allume jamais la télé), mais celles que j'ai vues me semblent globalement correctes : il y a parfois des affirmations douteuses ou qui peuvent induire des idées fausses mais je n'ai rien entendu dans celles que j'ai regardées qui me fasse bondir au plafond ; la présentation est plutôt pas mal au sens où j'ai l'impression que tout le monde peut accrocher au moins un peu, et comme chacune dure 10 minutes, même si on n'aime pas, on n'a pas le temps de s'endormir et je pense que ça peut convaincre des gens d'essayer au moins de s'intéresser un minimum au genre de choses sur lesquelles les maths se penchent. (Je peux éventuellement reprocher au choix des sujets, même s'il est agréablement éclectique, de mélanger des choses qui sont des problèmes profonds et difficiles avec des petites curiosités qui ne font pas l'objet de recherches ; ce n'est pas grave en soi, mais il faudrait peut-être mieux expliquer au public ce qui tombe dans chaque catégorie.)

Comme je l'avais raconté dans cette entrée passée de ce blog, je suis moi-même intéressé par la vulgarisation mathématique, pas pour le contenu de ce que ça raconte, mais pour apprendre à améliorer ma propre présentation des choses, qu'il s'agisse de vulgarisation, d'enseignement (ou même d'exposition à des pairs), et à tous les niveaux (du grand public aux chercheurs).

Or il se trouve justement que dans cette série Voyages au pays des maths est paru un épisode intitulé L'Entscheidungsproblem ou la fin des mathématiques ? (visible ici sur le site web d'Arte ou ici sur YouTube) qui porte sur le même sujet — la calculabilité — sur lequel j'ai récemment publié les transparents d'un cours que j'inaugure cette année à Télécom Paris (et aussi un billet qui se veut grand public sur un thème apparenté).

Du coup je suis curieux de savoir comment ce genre de vulgarisation est jugé par le grand public : j'apprécierais si des personnes qui lisent mon blog, surtout celles qui ne sont pas mathématiciennes, idéalement même pas scientifiques, pouvaient regarder cette vidéo de 10 minutes et me dire ce qu'elles en pensent : ce qu'elles en retiennent, si elles ont l'impression de comprendre les enjeux évoqués, quels passages sont clairs, lesquels ne le sont pas, ce genre de choses, et si elles sont d'accord avec mes critiques qui vont suivre.

(Si possible, merci de regarder la vidéo avant de lire la suite pour ne pas se laisser influencer par mes propres remarques qui vont suivre ; et aussi tout bêtement parce qu'elles sont sans doute difficiles à lire si on n'a pas vu la vidéo avant.)

↑Entry #2769 [older| permalink|newer] / ↑Entrée #2769 [précédente| permalien|suivante] ↑

↓Entry #2767 [older| permalink|newer] / ↓Entrée #2767 [précédente| permalien|suivante] ↓

(mercredi)

Transparents de cours de calculabilité

Comme je le disais il y a un mois, je me suis engagé à organiser à Télécom Paris(PlusÀParis) un cours de Logique et Fondements de l'Informatique où je dois parler de calculabilité, logique et typage. Je suis évidemment complètement à la bourre sur la préparation de ce cours, qui commence , mais bon, j'ai au moins provisoirement fini d'écrire des transparents pour la partie « calculabilité » qui devrait occuper, je pense, bien 6h de cours (sur 22h au total pour le cours magistral ; il y a des TD/TP à côté). Autant que je rende ça public dès maintenant, des fois que des gens voudraient m'aider à corriger les fautes. Donc :

Ces transparents sont en ligne ici

(Je compte concaténer ceux de la suite du cours quand il seront écrits. Je ne sais pas encore comment je veux diviser ça donc ce n'est pas évident de choisir des adresses intelligemment.)

J'ai à peine commencé à les relire, donc c'est certainement bourré de typos et de fautes plus ou moins graves. J'espère quand même qu'il n'y a pas d'erreur tellement grave qu'elle m'obligerait à tout restructurer.

Si vous faites des commentaires, pensez à me donner l'identifiant Git (en bas du premier transparent) auxquels ils se rapportent. (Au moment où j'écris ce billet, c'est 31080ea Wed Nov 1 11:06:40 2023 +0100 ; l'arbre Git avec le source est ici.)

Il faut préciser que ça s'adresse à des élèves ayant déjà fait de l'informatique en prépa (filière MPI ou filière MP option info, plus quelques uns venus de licences d'info ; les programmes des classes prépa sont ici), donc d'une part ce n'est pas comme s'ils découvraient tout, d'autre part on peut les espérer motivés par le sujet.

Dans cette partie calculabilité, je présente les fonctions primitives récursives et surtout générales récursives, les machines de Turing et le λ-calcul non typé, l'esquisse de l'équivalence entre les trois présentations de la calculabilité (fonctions générales récursives, machines de Turing et λ-calcul non typé), ainsi que divers résultats classiques fondamentaux : théorème s-m-n, théorème de récursion de Kleene, existence d'une machine universelle, résultats élémentaires sur les ensembles décidables (= calculables) et semi-décidables (= calculablement énumérables), et bien sûr l'indécidabilité du problème de l'arrêt (et aussi l'incalculabilité de la fonction « castor affairé »). Dans la suite du cours, il est prévu de parler de λ-calcul simplement typé en lien avec le calcul propositionnel intuitionniste, puis de diverses extensions (logique classique, logique du premier ordre, et évoquer divers bouts du cube de Barendregt). Le fil conducteur du cours est censé être quelque chose comme ceci : L'indécidabilité du problème de l'arrêt signifie que tout langage informatique qui garantit la terminaison des programmes est nécessairement limité ; des systèmes de typage de plus en plus puissants cherchent à rendre cette limitation aussi faible que possible.

J'ai renoncé à parler, même allusivement, de machines avec oracle ou de degrés de Turing ; mais les gens qui veulent en savoir plus sur ce sujet peuvent se référer à ce billet interminable pour lequel les notes ci-dessus suffisent largement en matière de prérequis.

Ajout () : Suivant ce qu'on m'a fait remarquer en commentaire, j'ai ajouté (Git 1cdc719 Thu Nov 2 17:08:48 2023 +0100) des choses sur le théorème de Rice et les réductions (many-to-one et de Turing). Il est cependant vraisemblable que j'en saute au moins une partie.

Le fait de me replonger dans le λ-calcul non typé, et de vouloir en savoir plus que le minimum que j'enseigne, m'a obligé à réapprendre plein de choses à son sujet[#], que j'avais complètement oublié ou jamais sues, et redécouvrir toutes les petites crottes de ragondin qui polluent un sujet qui a superficiellement l'air simple et élégant (comme : la différence entre β-réduction et βη-réduction, la différence entre termes normalisables et fortement normalisables, la différence entre stratégie de réduction extérieure gauche et intérieure gauche, la différence entre forme normale, forme normale de tête et forme normale de tête faible, etc.) ; le livre de Barendregt (The Lambda Calculus: Its Syntax and Semantics) est assez abominable en matière de dissection de crottes de ragondin, et celui de Krivine (Lambda-calcul : types et modèles — disponible en ligne en traduction anglaise) ne l'est pas moins. Un des problèmes est sans doute qu'on n'a pas vraiment idée de ce que sont les termes du λ-calcul non typé (prima facie, ce sont des fonctions qui prennent en entrée une autre fonction de même sorte et renvoient une autre fonction de même sorte : ce n'est pas du tout clair qu'on puisse fabriquer un objet qui soit aussi l'objet des morphismes de lui-même dans lui-même !) : divers gens (en commençant par Dana Scott à la fin des années 1960) ont réussi à en donner des modèles, ce qui éclaircit un peu la sémantique, mais là aussi on se perd entre les différentes manières de fabriquer des modèles du λ-calcul et les zillions de relations d'équivalence entre types que fournissent ces façons de fabriquer des modèles. (J'ai commencé à lire plein de choses sur le sujet, et surtout à me noyer dans les notations pourries. J'espère que l'article From computation to foundations via functions and application: The λ-calculus and its webbed models de Chantal Berline m'aidera à y voir plus clair.)

Je suis assez étonné, en revanche, de ne pas trouver d'implémentation (libre, flexible et largement disponible) du λ-calcul non typé, qui permettrait de tester un peu les choses (transformer les notations, réécrire les termes à la main ou de façon automatisée, comparer les stratégies de réduction, etc.). Est-ce que j'en ai raté une évidente ?

[#] La première fois que j'ai appris des choses sur le λ-calcul, ça devait être vers 1990 quand on m'a offert le livre de vulgarisation scientifique The Emperor's New Mind de Roger Penrose (j'en ai parlé dans une section d'une entrée récente), qui décrit un peu le λ-calcul et les entiers de Church, et ça m'a complètement fasciné que des règles typographiques aussi simples et élégantes (← mais bon, en fait, une bonne quantité de poussière avait été glissée sous le tapis) puissent donner quelque chose d'aussi puisant.

Ajout () : Par pure coïncidence, la chaîne de télé Arte vient de produire, dans le cadre de sa série Voyages au pays des maths, un mini-documentaire de vulgarisation (10 minutes) intitulé L'Entscheidungsproblem ou la fin des mathématiques ? (ici sur YouTube, ici sur le site web d'Arte) et qui porte justement sur le sujet dont je parle ici. Je ne suis pas d'accord avec tous les choix de présentation, mais ça donne au moins une idée de ce dont il est question (et notamment, tenter de vulgariser l'équivalence entre fonctions générales récursives, machines de Turing et λ-calcul était un défi pas du tout évident, et je trouve qu'il s'en sort pas mal). Surajout : voir ce nouveau billet où je décortique un peu plus cette vidéo.

↑Entry #2767 [older| permalink|newer] / ↑Entrée #2767 [précédente| permalien|suivante] ↑

↓Entry #2764 [older| permalink|newer] / ↓Entrée #2764 [précédente| permalien|suivante] ↓

(mercredi)

Oracles en calculabilité : degrés de Turing et diverses généralisations

Avant-propos et motivation

Avant-propos : J'ai publié il y a quelque temps un billet de vulgarisation sur la notion d'« oracle » en informatique théorique. Ce billet-là (qui se veut grand public) était initialement destiné à être l'introduction à celui-ci (plus technique — mais, j'espère, pas incompréhensible pour autant), mais j'ai décidé de les publier séparément parce qu'ils sont, en fait, à peu près indépendants : on peut donc commencer par lire celui-là si on veut une sorte d'explication introductive et de motivation du sujet, mais ce n'est pas nécessaire non plus.

Ce billet-ci, plus technique mais j'espère pas incompréhensible pour autant (voir le paragraphe suivant pour les prérequis), vise à présenter la notion classique de degré de Turing, et ensuite diverses généralisations de celles-ci dans la recherche contemporaine en logique / calculabilité. Mais je trouve aussi intéressant de vulgariser la notion (classique) de non-déterminisme et celle (pas du tout classique, mais que je trouve très intéressante) de « co-non-déterminisme » (le terme est de moi).

Pour ce qui est des prérequis, ce billet-ci s'adresse à des lecteurs qui savent déjà les bases de la calculabilité : c'est-à-dire en gros, ce qu'est une machine de Turing ou un algorithme (c'est-à-dire au moins approximativement : je n'ai pas l'intention de rentrer dans quelques détails que ce soit sur les état et les bandes), et une fonction calculable (c'est-à-dire, calculable au sens de Church-Turing), ce genre de choses. Je vais essayer de faire en sorte de ne supposer connu (outre des maths générales, du genre ce que c'est qu'une fonction, un ensemble, une partie, une bijection, un ordre, une relation d'équivalence…) que le contenu du chapitre 5 (Introduction à la calculabilité) des notes de mon cours Théorie des langages à Télécom, auquel je peux donc renvoyer pour ces notions de base en calculabilité (en attendant un nouveau cours sur le sujet). • Ajout () : la première partie des transparents de mon nouveau cours est disponible ici.

En tout cas, je ne suppose pas connu la notion de degré de Turing puisque mon but est justement de l'expliquer et de voir comment on peut aller plus loin. (Néanmoins, soyons honnête, les lecteurs déjà un minimum familiers du concept trouveront sans doute mon billet plus facile à suivre que si on le découvre pour la première fois ici.)

Mon but ici est d'abord de définir la notion (tout à fait standard) de réduction de Turing et de degré de Turing, puis de présenter des extensions de ces notions qui me semblent à la fois très importantes et profondément naturelles. J'ai appris l'existence de ces notions en lisant deux articles d'un certain Takayuki Kihara, Lawvere-Tierney topologies for computability theorists et Rethinking the notion of oracle (et dans une moindre mesure Degrees of incomputability, realizability and constructive reverse mathematics, mais je n'ai pas fini celui-là) : je cherche donc à la fois à faire de la pub pour ces articles et pour les notions qu'ils contiennent (parce qu'elles ont vraiment changé la manière dont je pense à la calculabilité), à montrer que ces notions ne sont pas terriblement techniques, et aussi simplement à assurer ma propre compréhension de ces articles en en réexposant certains bouts à ma façon. En outre, j'espère avoir apporté quelques éléments d'intuition utiles derrière certaines des définitions ou des concepts que j'expose. Je trouve particulièrement intéressante la notion de « co-non-déterminisme » (le passage du niveau T2 au niveau T3), donc mon but est notamment de faire de la pub pour ce concept (qui pourrait sans doute s'avérer fécond en-dehors de la calculabilité).

(Notons que j'attribue ces notions à Kihara, qui les a au moins synthétisées, et c'est par lui que je les ai apprises ; mais je ne prétends pas qu'il a tout inventé — il a plutôt réussi à relier, reformuler et réexposer de façon extrêmement convaincante des notions dont certaines figuraient déjà ailleurs : je renvoie aux références de ses papiers pour les citations antérieures, mais je peux par exemple mentionner Basic Subtoposes of the Effective Topos de Lee & van Oosten, ou Instance reducibility and Weihrauch degrees de Bauer, qui sont dignes d'intérêt si on apprécie le sujet.)

Bref, je veux commencer par expliquer ce qu'est un degré de Turing ordinaire (celui d'une fonction — totale, simplement valuée — ℕ→ℕ), puis donner trois extensions successives de cette notion (je vais parler de degrés T1, T2 et T3 faute de meilleure terminologie — Kihara n'en introduit pas vraiment). en expliquant ce qu'elles changent, et si possible pourquoi elles sont naturelles et intéressantes, et ce qu'on peut en dire : d'abord (T1) aux fonctions partielles, puis (T2) aux fonctions multivaluées (ou non-déterministes), et enfin (T3) aux « fonctions avec conseil » (une sorte de « co-non-déterminisme »). Enfin, je veux essayer d'expliquer pourquoi on a fait la « bonne » généralisation, et pour ça, je donne, en guise de dessert, une construction tout à fait différente des degrés (T1, T2 et surtout) T3 qu'on aura définis par des opérateurs effectifs locaux (et évoquer brièvement le lien avec le topos effectif, sur lequel j'ai récemment écrit un billet, mais je ne suppose pas qu'on ici l'a lu).

Conseil de lecture (en guise de leitfaden) : Je sais que j'ai tendance à entrer parfois dans de grandes digressions pas forcément tellement utiles. J'ai essayé de les marquer comme telles (par des petits caractères, ou en disant dès le début d'une section qu'on peut sauter celle-ci) ; je n'ai sans doute pas toujours marqué tout ce qui pouvait être sauté, mais les dépendances entre sections ne sont pas énormes. Même si on n'a pas lu tout ce qui précède, je pense que ça vaut la peine de goûter le dessert (surtout si on n'est pas convaincu par l'intérêt des définitions qui ont précédé, car, après tout, il s'agit plus ou moins de les justifier). Et surtout, je pense que ça vaut la peine de jeter un coup d'œil à la définition des degrés T3 pour la définition du « co-non-déterminisme », ou simplement parce que le jeu à trois joueurs entre Arthur, Nimué et Merlin est vraiment rigolo.

Table des matières

T0 : Degrés de Turing ordinaires (fonctions ℕ→ℕ totales simplement valuées)

Réduction de Turing ordinaire et degrés de Turing

La notion la plus standard est la réduction de Turing ordinaire (j'ajoute ordinaire parce que toutes les notions qui viennent peuvent légitiment être aussi qualifiées de réduction de Turing), et je vais commencer par la définir précisément et dire quelques choses basiques à son sujet. Elle concerne les fonctions ℕ→ℕ (c'est-à-dire, totales, prenant une seule valeur pour chaque entier naturel), ou éventuellement ℕ→{0,1} mais ça ne changera rien à l'histoire.

Une fonction f:ℕ→ℕ est dite réductible au sens de Turing (=Turing-réductible) à une fonction g:ℕ→ℕ, ou bien calculable avec g pour oracle, et on note (disons) f ≼T g, lorsqu'il existe une machine de Turing (i.e., un algorithme) qui calcule f en ayant accès à un oracle calculant g, c'est-à-dire un gadget magique capable de fournir à l'algorithme la valeur de g en tout point souhaité.

Plus précisément, f ≼T g signifie qu'il existe une machine de Turing qui, quand on lui donne un entier n en entrée, termine toujours en temps fini et calcule f(n) comme sortie (peu importent les manières dont n et f(n) sont codés sur la bande de la machine de Turing tant que c'est raisonnable), sachant que la machine dispose de l'accès à un oracle qui peut calculer g(m) pour n'importe quel m donné. (Voici un exemple de protocole d'interrogation de l'oracle : la machine écrit m sur un ruban dédié, entre dans un état spécial interrogation de l'oracle, l'oracle remplace m par g(m) et place la machine dans l'état réponse de l'oracle ; mais les détails sont peu importants tant qu'on parle de calculabilité.) La machine peut interroger l'oracle autant de fois qu'elle le souhaite (y compris pas du tout, mais ça ne changerait d'ailleurs rien si on imposait d'appeler l'oracle puisqu'on peut ignorer sa réponse), et elle l'interroge sur les valeurs qu'elle veut et fait ce qu'elle veut des réponses : la réponse de l'oracle pour une valeur m est toujours exactement g(m), et c'est tout. (Le temps d'interrogation de l'oracle est une étape de calcul, mais ceci est peu important puisqu'on parle de calculabilité et pas de complexité.)

En particulier, toutes les fonctions calculables (c'est-à-dire : sans oracle) sont réductibles à n'importe quelle fonction, tout simplement en utilisant un programme qui ne fait aucun appel à l'oracle (en particulier, elles sont réductibles les unes aux autres, ou par exemple à la fonction constante égale à zéro).

J'insiste bien (parce que c'est ce qui va changer dans la suite) sur le fait que cette réduction de Turing « ordinaire » concerne des fonctions totales. Notamment, l'algorithme réputé calculer f en ayant g pour oracle est censé terminer pour toute valeur n qu'on lui fournit en entrée (du moins tant que n est une représentation légitime d'un entier naturel, et la valeur calculée doit aussi en être une ; mais là non plus, ces subtilités ne changent essentiellement rien). Je réitère aussi le fait que la consultation de l'oracle n'est pas limitée (ou payante) : l'algorithme a le droit de s'en servir aussi souvent qu'il le veut.

La réduction de Turing (ordinaire, mais ça restera vrai pour les généralisations que je vais introduire après) ≼T forme ce qu'on appelle un préordre (relation réflexive et transitive) sur l'ensemble des fonctions ℕ→ℕ, c'est-à-dire que :

  • d'une part, f ≼T f : toute fonction f est Turing-réductible à elle-même (c'est évident) ; et
  • d'autre part, f ≼T g et g ≼T h impliquent f ≼T h : si f est Turing-réductible à g et que g est Turing-réductible à h, alors f est Turing-réductible à h (c'est assez facile à voir : si je sais calculer f en utilisant g et que je sais calculer g en utilisant h, alors pour calculer f en utilisant h je vais utiliser l'algorithme qui me permet f au moyen de g et, à chaque fois que cet algorithme fait appel à l'oracle, j'utilise le programme qui me permet g au moyen de h à la place, en utilisant l'oracle donnant h comme boîte noire).

Ce qui manque à une relation de « préordre » pour être un ordre, c'est l'antisymétrie, et c'est justement ce qui justifie la définition suivante.

On dit que f et g sont Turing-équivalentes ou de même degré de Turing quand chacune est réductible à l'autre (i.e., on peut calculer f en ayant un oracle qui donne g, et réciproquement) : c'est-à-dire qu'on note (disons) f ≡T g lorsque f ≼T g et g ≼T f. Ceci est une relation d'équivalence (c'est la notion de relation d'équivalence associée à un préordre) : les classes d'équivalence sont appelées les degrés de Turing. Autrement dit, le degré de Turing [f]T de f est l'ensemble {g:ℕ→ℕ : g ≡T f} de toutes les fonctions ℕ→ℕ qui sont Turing-équivalentes à f, et notamment, elles sont toutes Turing-réductibles les unes aux autres. C'est-à-dire que deux fonctions sont Turing-équivalentes exactement quand elles ont le même degré de Turing.

Intuitivement, un degré de Turing est la mesure de la puissance d'un oracle. On peut aussi considérer que c'est une forme d'« impossibilité à calculer » la fonction ou bien de « quantité d'information » (totale) contenue dans la fonction f (mais ce n'est pas exactement la même façon de la mesurer que, disons, la complexité de Kolmogorov, pour ceux qui savent ce que c'est que ça : il y a des rapports, bien sûr, mais disons très sommairement que le degré de Turing est une mesure plus grossière que la complexité de Kolmogorov ; comme ce n'est pas mon propos ici, je ne vais pas en dire plus).

Le « préordre » de la réduction de Turing devient un vrai ordre (partiel) sur les degrés de Turing : on dit qu'un degré de Turing f est inférieur ou égal à un degré de Turing g, et on note fg, lorsque une (ou, ce qui revient au même, toute) fonction du degré f est Turing-réductible à une (ou, ce qui revient au même, toute) fonction du degré g. (Autrement dit, [f]T ≤ [g]T signifie par définition exactement la même chose que f ≼T g.)

Parmi les degrés de Turing, il en est un plus petit, qu'on note 0, c'est celui des fonctions calculables (c'est notamment celui de la fonction constante égale à 0, c'est-à-dire un oracle qui ne répond rien d'intéressant, et on peut voir ça comme une raison de cette notation ; si on préfère, c'est la puissance des oracles qui n'ont aucun intérêt comme oracles parce qu'on peut les calculer directement sans oracle).

↑Entry #2764 [older| permalink|newer] / ↑Entrée #2764 [précédente| permalien|suivante] ↑

↓Entry #2761 [older| permalink|newer] / ↓Entrée #2761 [précédente| permalien|suivante] ↓

(vendredi)

Vulgarisation sur la notion d'oracle en informatique théorique

Méta : Ce billet est un rejeton d'un autre que j'ai commencé à rédiger sur un sujet technique (à savoir : les oracles en calculabilité au sens de Kihara) : en écrivant ce dernier je me suis dit que j'allais commencer par une introduction générale essentiellement grand public à la notion d'oracle (qu'est-ce que c'est que ce truc et pourquoi on les étudie ?). Mais finalement, comme cette introduction générale devenait un peu longue, et surtout comme elle ne vise en fait pas du tout le même public (ce qui suit ne se veut pas technique, le billet dont il est détaché présuppose des connaissances en calculabilité), et aussi parce qu'il y a toujours un risque que je ne finisse pas ce que je commence, je choisis de la publier séparément, plutôt que risquer de rebuter les lecteurs de l'un ou de l'autre. Du coup, je développe aussi un peu plus largement que ce qui était initialement prévu, au risque de rendre ce billet un peu bancal parce que j'ai repris plusieurs fois certains passages en changeant mon approche (notamment à force d'insérer des explications plus détaillées de choses que j'avais évoquées ailleurs, j'ai créé un certain nombre de redites : je pense qu'elles ne sont pas graves et j'ai la flemme de les traquer).

Quand (si !) je publierai l'autre billet, j'ajouterai un lien ici et réciproquement, pour qu'on puisse les lire successivement comme c'était initialement prévu. • Ajout : c'est ici.

À cause du fait que j'essaie de viser un public très large, je ne vais pas donner de définitions mathématiques précises (si j'utilise un terme technique comme exponentiel, on peut se contenter d'une idée très approximative de ce que c'est) ; mais du coup, il faut que prévienne que je vais dire des choses qui seront parfois techniquement incorrectes (même quand j'oublie de prendre les précautions oratoires du type très grossièrement, approximativement parlant, etc.). J'espère ne rien avoir dit de violemment faux, quand même, mais tout est à prendre avec les pincettes de rigueur : il s'agit juste de donner une toute petite idée du type de questions qu'on peut se poser dans quelques parties de l'informatique théorique et d'un outil intéressant (la notion d'oracle) pour y répondre et poser d'autres questions, pas de rentrer dans le moindre détail technique (pour ça, il y aura le deuxième volet, indépendant de celui-ci).

Pour les lecteurs qui ont la patience d'aller jusqu'au bout, il y a une petite énigme de logique(?) à la fin ; en fait, elle ne nécessite pas d'avoir lu ce qui précède, donc on peut la lire directement. (Bon, je ne sais pas si elle est intéressante, et je ne sais pas non plus à quel point elle est difficile. Je ne suis même pas complètement convaincu qu'elle ait un rapport avec le sujet, même si le contexte où je l'ai lue a clairement un rapport.)

Le concept d'oracle dont il est question ici est vient de l'informatique théorique (le terme oracle apparaît dans la thèse d'Alan Turing en 1938). Je voudrais essayer d'expliquer de façon très informelle, très vague aussi malheureusement (mais, j'espère, du coup, assez grand public) ce dont il est question.

Une partie de l'informatique théorique (l'algorithmique) s'intéresse, de façon positive, à trouver des moyens de résoudre des problèmes (mathématiquement bien définis), c'est-à-dire à concevoir des algorithmes qui les résolvent : un algorithme est un programme, un plan d'opération susceptible d'être mené dans un ordinateur — mais aussi en principe par un humain particulièrement patient — qui effectue une tâche calculatoire donnée. Par exemple, la manière dont on apprend à faire des additions et des multiplications à l'école primaire sont des algorithmes, même si ce ne sont pas forcément les plus intéressants, ils ont le mérite de nous rappeler qu'un algorithme ne tourne pas forcément sur un ordinateur, on peut aussi l'exécuter à la main (ce sera juste quelques milliards de fois plus lent…) ; d'ailleurs, le mot algorithme fait référence au mathématicien persan (écrivant en langue arabe) Muḥammad ibn Mūsá al-H̱wārizmī parce qu'il a décrit toutes sortes de méthodes systématiques pour résoudre des problèmes mathématiques, qu'on peut donc légitimement qualifier d'algorithmes, et c'était plus de 1000 ans avant l'invention des ordinateurs.

(Petite digression sur le terme algorithme : il semble qu'un mélange entre l'incompréhension des journalistes face à tout ce qui est technique et l'approximation des termes utilisés dans le contexte de l'intelligence artificielle ait fait dévier le sens du mot, dans l'image qu'en a le grand public, vers une sorte de synonyme de système opaque auquel on ne comprend rien. Certainement beaucoup de programmes utilisés dans l'ingénierie informatique moderne (notamment tout ce qui relève de l'IA) comportent énormément d'heuristiques et de méthodes mal comprises, parfois même mal comprises de leurs programmeurs ou concepteurs, et dont la réponse est parfois fausse ou mal spécifiée. Mais le sens d'algorithme en algorithmique désigne au contraire un plan bien défini et bien compris qui arrive à un résultat bien spécifié.)

Mais d'autres parties de l'informatique théorique s'intéressent non pas à concevoir des algorithmes efficaces pour résoudre tel ou tel problème, mais à se pencher sur la notion même d'algorithme et les limites théoriques du concept : que peut-on faire avec un ordinateur ?

Spécifiquement, on s'intéresse alors plutôt à des résultats négatifs : plutôt que résoudre tel ou tel problème c'est-à-dire concevoir un algorithme qui le résout, on s'attache à montrer que tel ou tel problème est difficile (long et coûteux) voire impossible à résoudre par un algorithme, ou au moins, à étudier cette difficulté, à la classifier et à la jauger. Je reste délibérément vague sur ce qu'un problème peut recouvrir ici et ce que difficile veut dire, mais je peux délimiter ici trois grands domaines où ce que je veux dire s'applique, et j'en profite pour parler un peu plus longuement de chacun des trois, en gros du plus théorique vers le plus appliqué (il n'est pas nécessaire de lire tout ce que j'écris sur chacun ci-dessous pour passer à la suite, mais je cherche à donner un minimum de contexte) :

  • La théorie de la calculabilité est la branche de l'informatique théorique (possiblement la plus proche des mathématiques pures, et notamment de la logique) qui étudie ce qu'on peut faire algorithmiquement avec des ressources illimitées, c'est-à-dire, en disposant d'un temps illimité et d'une mémoire illimitée. (Le mot illimité signifie ici fini mais arbitrairement grand, sans limite a priori : c'est-à-dire que les algorithmes en question doivent s'arrêter un jour en disant j'ai terminé, mais ils ont le droit de prendre autant de temps qu'ils veulent, tant que ce temps est fini ; idem pour la mémoire : ils ont le droit de stocker toutes les données qu'ils veulent tant que cet espace utilisé reste fini à tout moment. C'est donc la forme la plus abstraite et théorique de la question que peut-on faire avec un ordinateur ? qui est étudiée ici, et elle déborde sur des questions du type comment peut-on imaginer des types de machines fondamentalement et théoriquement plus puissantes qu'un ordinateur même si elles ne sont pas réalisables en pratique ? — mentionnons par exemple que même un ordinateur quantique, qui si on réussissait à en construire serait pour certains types de problèmes incroyablement plus efficace qu'un ordinateur tel que nous disposons actuellement, ne représente pas un saut qualitatif tel qu'il peut intéresser la calculabilité : pour la calculabilité, un ordinateur quantique, un ordinateur classique, un système d'engrenages mécaniques ou encore un humain extrêmement patient et systématique muni d'un stylo et d'un papier et exécutant mécaniquement des opérations prédéfinies valent exactement la même chose.)

    La calculabilité est un peu la mère de l'informatique, parce que c'est au travers de la recherche d'une formalisation de ce qu'est un algorithme, et ce qu'un algorithme peut faire, qu'Alonzo Church et son étudiant Alan Turing sont arrivés au concept de calculabilité ; la définition précise d'algorithme à laquelle Turing est arrivé, la machine de Turing, est considéré comme une préfiguration théorique de ce qu'est un ordinateur.

    Un des thèmes majeurs de la calculabilité, donc, c'est de montrer que, pour des raisons théoriques, certains types de problèmes (mathématiquement bien posés) ne sont pas résolubles algorithmiquement : même avec un temps illimité(-mais-fini) à votre disposition, et autant de mémoire, vous ne pourrez pas écrire un algorithme qui répond à coup sûr à certaines questions qui admettent pourtant une réponse bien définie.

    L'exemple archétypal à ce sujet est le problème de l'arrêt : très grossièrement, ce problème demande justement, donné un algorithme, i.e., une suite d'instructions à exécuter (qui peut, bien sûr, contenir des boucles, du type répéter les instructions suivantes tant qu'une certaine condition n'est pas vérifiée), et des valeurs à fournir en entrée à cet algorithme, si l'algorithme en question finit (i.e., si le calcul aboutit à un résultat). Le théorème majeur de Turing, qui est à la base de la théorie de la calculabilité (et donc, dans un certain sens, de l'informatique ; mais à l'origine il s'y intéressait pour des raisons liées à la logique mathématique et spécifiquement au théorème de Gödel), c'est qu'aucun algorithme ne peut résoudre le problème de l'arrêt : autrement dit, aucun algorithme ne peut dire à coup sûr si un autre algorithme s'arrête au bout d'un temps fini. (La seule façon de savoir est de l'exécuter, ce qu'on peut faire, mais quand on l'exécute, tant qu'il n'est pas fini, on n'est jamais sûr si le programme finira par terminer plus tard.) La raison de cette impossibilité est d'ailleurs étonnamment bête une fois qu'on a fait le travail de formalisation pour rendre rigoureuse la théorie : en gros, si un algorithme existait qui puisse dire à coup sûr si un algorithme donné s'arrête, on pourrait faire un algorithme qui l'interroge et fait le contraire de ce qu'il a prédit, ce qui le met en défaut. (C'est ce qu'on appelle un argument diagonal de Cantor.)

    La calculabilité ne se contente pas, en fait, de définir des problèmes résolubles ou non résolubles algorithmiquement, et de le montrer : il y a des problèmes plus ou moins impossibles à résoudre, et l'étude des degrés d'impossibilité (par exemple les degrés de Turing dont je dirai un mot plus bas, et que je définirai précisément dans le volet technique de ce billet) est un sujet de recherche qui court depuis Turing, mais l'outil de base pour ne serait-ce que formuler ce genre de questions est la notion d'oracle que je vais chercher à introduire ici.

  • La théorie de la complexité [algorithmique], cherche à être plus fine que la calculabilité : alors que la calculabilité donne aux algorithmes l'accès à des ressources (temps, mémoire) illimitées-mais-finies, la complexité s'intéresse à des problèmes résolubles par un algorithme, mais cherche à mesurer combien de temps ou de mémoire (ou parfois d'autres ressources) un algorithme devra utiliser, en fonction de la taille du problème qu'on leur pose, lorsque cette taille devient très grande. (Par exemple, combien d'étapes faut-il pour multiplier deux nombres de 100 chiffres ? de 1000 chiffres ? de 10 000 chiffres ? Avec l'algorithme qu'on apprend à l'école primaire, la réponse est en gros 10 000, 1 000 000 et 100 000 000 respectivement, parce qu'il faut multiplier chaque chiffre du multiplicande par chaque chiffre du multiplicateur avant d'ajouter tout ça : en complexité on appelle ça un algorithme quadratique. En fait, on peut faire beaucoup plus efficacement, et d'ailleurs au moins un algorithme de multiplication plus efficace sur les grands nombres que l'algorithme appris à l'école primaire peut vraiment servir à la main — enfin, pourrait servir à la main si on était dans un monde où nous n'avions pas un ordinateur en permanence avec nous.)

    La complexité est donc une discipline un peu plus applicable au monde réel que la calculabilité : alors que la calculabilité va nous donner des réponses du type vous ne pouvez pas résoudre ce problème avec un ordinateur, quel que soit le temps que vous soyez prêt à attendre ou oui vous pouvez, mais je n'ai aucune idée du temps nécessaire parce que ce n'est pas mon sujet, la complexité va chercher à voir plus précisément dans la deuxième catégorie entre des problèmes théoriquement-résolubles-mais-absolument-pas-en-pratique (un peu comme si vous cherchiez à multiplier à la main deux nombres de 1 000 000 000 000 chiffres… oui, en principe c'est possible) et des problèmes un peu plus abordables en pratique. (La complexité va aussi faire la différence entre différents types d'ordinateurs qui, pour la calculabilité sont équivalents, par exemple un ordinateur quantique permet plus de choses du point de vue de la complexité parce qu'il peut en quelque sorte mener plein de calculs en parallèle ; alors qu'en calculabilité il ne change rien du tout par rapport à un ordinateur classique.)

    Néanmoins, la complexité reste une discipline assez théorique parce qu'elle se penche sur le temps (ou la mémoire, ou une autre ressource) utilisés asymptotiquement : ici, asymptotiquement veut dire quand la taille de l'entrée de notre problème (i.e. l'instance précise) devient extrêmement grande (tend vers l'infini). C'est-à-dire que le but de la complexité n'est pas de savoir si vous allez prendre exactement tel ou tel temps pour multiplier deux grands entiers (disons) mais comment ce temps grandit quand les entiers deviennent très grands (est-ce que doubler la taille des nombres double le temps qu'il faut pour faire la multiplication ? plus ? moins ?). Tout simplement parce que c'est un peu plus abordable comme type de question (et que c'est déjà utile dans la pratique).

    La complexité définit toutes sortes de catégories de problèmes en fonction de la difficulté à les résoudre, mesurée sous la forme des ressources qu'on accepte d'allouer à un algorithme qui les résout. (Par exemple, de façon approximative, un problème est dit EXPTIME ou EXPSPACE s'il existe un algorithme qui le résout et dont le temps d'exécution ou respectivement la mémoire utilisée croît au plus exponentiellement dans la taille de la donnée ; on peut faire énormément de choses avec un temps ou une mémoire exponentielle, donc énormément de problèmes sont dans ces classes, qui ne sont pas d'un grand intérêt pratique, mais c'est quand même plus restrictif que les problèmes tout simplement calculables en ressource illimitées que j'ai évoqués plus haut.)

    Deux classes particulièrement importantes en calculabilité sont les classes P et NP. La classe P est très grossièrement celle des problèmes faciles (un peu plus précisément, ce sont ceux qui sont résolubles par un algorithme qui utilise un temps au plus polynomial dans la taille de l'entrée : linéaire, quadratique, cubique, quelque chose comme ça, mais exponentiel n'est pas permis) ; la classe NP est plus subtile : très grossièrement, ce sont les problèmes faciles à vérifier (mais pas forcément faciles à résoudre ; c'est-à-dire que si vous avez la réponse, la vérification qu'elle est correcte se fait essentiellement selon la classe P, mais par contre, si vous ne connaissez pas la réponse, il est possible qu'elle soit très difficile à trouver ; j'insiste sur le fait que mes explications sont très grossières et que je passe sur plein de subtilités théoriques).

    Un exemple de problème NP très simple à décrire est le suivant : je vous donne un tas de nombres (entiers positifs, disons) et un nombre-cible, et votre but est d'exprimer le nombre-cible comme somme de certains des nombres donnés. (C'est une variante très simple des « chiffres et les lettres » où on n'a le droit qu'à l'addition ! Ou bien imaginez que ce sont des pièces de monnaie dans un système monétaire bizarre, et vous voulez réussir à payer exactement un certain montant cible en ayant dans votre poche des pièces de certains montants donnés. Par exemple, si je vous donne les nombres 1, 3, 8, 17, 32, 51, 82, 127, 216, 329, 611, 956 et 1849, et que je vous demande de faire la somme 2146 avec, ce n'est pas évident d'y arriver sauf à tester énormément de combinaisons ; par contre, vérifier que 1 + 8 + 32 + 82 + 127 + 329 + 611 + 956 = 2146 est un calcul facile.)

    Parmi les classes que j'ai pas-vraiment-définies-mais-un-peu-évoquées, on a P ⊆ NP ⊆ EXPTIME, le symbole ‘⊆’ signifiant ici est inclus dans, c'est-à-dire que tout problème P (facile à résoudre) est en particulier NP (facile à vérifier), et que tout problème NP est lui-même, en particulier, EXPTIME (résoluble en temps exponentiel). (Ce dernier est à son tour inclus dans EXPSPACE mais peu importe.) On sait par ailleurs montrer qu'il existe des problèmes qui sont dans EXPTIME mais qui ne sont pas dans P (et on sait en décrire explicitement), autrement dit, des problèmes résolubles en temps exponentiels mais pas faciles pour autant (pas résolubles en temps polynomial) : ce n'est pas très surprenant, parce qu'une exponentielle grandit vraiment très vite, donc il n'est pas surprenant que certains problèmes puissent se résoudre en un temps exponentiel mais pas en un temps beaucoup plus limité, mais encore fallait-il le prouver (ce n'est pas très difficile, mais ce n'est pas complètement évident non plus). Bref, si voit P ⊆ NP ⊆ EXPTIME comme trois boîtes imbriquées, la plus à gauche est effectivement strictement plus petite que la plus à droite. Mais la question se pose de savoir comment est celle du milieu. En fait, on ne sait pas la situer ni par rapport à celle de gauche ni par rapport à celle de droite.

    Spécifiquement, une question centrale de la théorie de la complexité est de savoir si P=NP (en gros, est-ce que tout problème facile à vérifier est, en fait, facile à résoudre ?). On pense très fortement que la réponse est non (i.e., qu'il existe des problèmes qui sont dans NP et qui ne sont pas dans P ; on en a même plein de candidats, d'ailleurs celui que j'ai donné plus haut en est un), mais on ne sait pas le prouver (l'enjeu, ici, est de prouver rigoureusement qu'un problème est difficile au sens où il ne peut pas exister d'algorithme qui le résout facilement). Cette question PNP est même mise à prix à 1 000 000 $ (et attire régulièrement des « solutions » incorrectes de toutes parts).

    Pour en savoir plus sur la situation en complexité, cet article récent de vulgarisation (costaud !) dans le magazine Quanta, quoique long, n'est pas mauvais pour donner un aperçu, et expliquer un peu mieux que ce que je l'ai fait ce que c'est que cette histoire de PNP, ce qu'on sait dire à son sujet et quelle est la difficulté. Ce n'est pas vraiment mon propos ici d'en dire plus ici.

  • La cryptographie (enfin, peut-être que je devrais plutôt dire cryptologie ici) est l'étude scientifique de la sécurité de l'information : il s'agit de développer des techniques de chiffrement (comment transformer un message en un chiffré qui ne puisse être déchiffré qu'en ayant accès à une clé) ou ayant trait à d'autres questions de sécurité de l'information (signature électronique, authenticité, non-répudiation, partage de secrets, ce genre de choses). Mais plus précisément, ici, au sein de la cryptographie, j'ai à l'esprit la cryptographie à clés publiques (voir ce billet récent pour une explication de ce que ça signifie ; ce n'est pas très important ici).

    Une différence importante entre la calculabilité/complexité et la cryptographie est qu'en cryptographie il y a un adversaire (par exemple, s'agissant d'un chiffrement, l'adversaire est un attaquant hypothétique qui essaie d'obtenir des informations sur le message sans avoir la clé de déchiffrement, ou peut-être sur la clé en ayant connaissance du message et du chiffré, ou des variantes autour de ces questions). En calculabilité ou en complexité on s'intéresse à la difficulté de problèmes parce que ça mettra une borne sur ce que nous pourrons faire. En cryptographie, on s'intéresse à la difficulté plutôt parce que ça peut représenter un travail pour l'attaquant (donc la difficulté est souhaitable !). Spécifiquement, on voudrait concevoir des méthodes de chiffrement qui soient faciles à utiliser (c'est-à-dire, peu coûteuses algorithmiquement) quand on connaît la clé, mais extrêmement difficiles à casser, c'est-à-dire pour l'attaquant qui ne connaît pas la clé, et si possible, on voudrait prouver cette difficulté, ou au moins avoir des raisons un peu plus tangibles que je ne sais pas faire, donc c'est sans doute compliqué.

Bref, dans ces trois domaines (et il y en a sans doute d'autres dont j'ignore tout ou qui ne me viennent pas à l'esprit), pour des raisons un peu différentes et avec des notions de problème et de difficulté différentes, on peut chercher à montrer qu'un certain problème est difficile (voire, s'agissant de la calculabilité, impossible, mais je vais ranger ça sous l'étiquette vague difficile), ou étudier sa difficulté.

Bon, mais comment montre-t-on qu'un problème est difficile ?

↑Entry #2761 [older| permalink|newer] / ↑Entrée #2761 [précédente| permalien|suivante] ↑

↓Entry #2752 [older| permalink|newer] / ↓Entrée #2752 [précédente| permalien|suivante] ↓

(mardi)

Sur le topos effectif

Avant-propos

Le but ce de billet interminable est de donner (sans trop de prérequis, cf. plus bas) quelques explications, la définition, et quelques propriétés autour du topos effectif de Hyland. C'est un sujet sur lequel je m'étais promis de parler sur ce blog il y a un moment déjà. Je n'ai cependant pas vraiment suivi mon plan initial, qui était plutôt de faire d'abord une série de billets (en principe indépendant mais qu'il aurait été conseillé de lire dans l'ordre) pour expliquer au préalable l'idée générale du concept de topos et leur logique interne, puis les topos de faisceaux sur un espace topologique, avant d'en venir au topos effectif (que j'ai tendance à imaginer comme plus difficile à comprendre, mais peut-être que je me trompe, en fait). Seulement je me suis retrouvé (pour lire des choses sur un sujet connexe[#]) à ré-apprendre la définition du topos effectif après l'avoir oubliée pour la 42e fois environ, et j'ai pensé que la meilleure façon de la retenir et de m'assurer que j'en avais compris les bases, serait de me forcer à l'expliquer ici, autant que possible de mémoire, et, de fait, ça m'a permis de me rendre compte de plein de subtilités qui m'avaient d'abord échappé : voici pour la genèse de cette entrée, qui s'est évidemment avérée beaucoup plus longue qu'initialement prévue, et que sans doute personne ne lira mais ce n'est pas grave parce qu'elle m'aura servi à moi.

[#] En fait, pour expliquer d'où je viens pour les gens qui connaissent déjà le sujet, ces jours-ci j'essaie épisodiquement de comprendre trois articles que je considère à la fois très intéressants et très importants (pas tellement pour leurs résultats que pour le point de vue, surtout s'agissant des deux derniers), et dont je reparlerai sans doute une autre fois, d'un certain Takayuki Kihara : ① Degrees of incomputability, realizability and constructive reverse mathematics, ② Lawvere-Tierney topologies for computability theorists et ③ Rethinking the notion of oracle (ils peuvent se lire indépendamment les uns des autres et aussi indépendamment de la notion de topos effectif, mais toutes ces choses s'éclairent nettement les unes les autres). Notamment, les deux derniers articles suggèrent que la « bonne » notion d'oracle en calculabilité est celle de topologie de Lawvere-Tierney sur le topos de Kleene-Vesley, et donc j'essaie de me faire une intuition du pourquoi et du comment.

Cette genèse peut aussi expliquer le style inhabituellement brouillon, vu que j'avais commencé par me dire que j'allais juste donner la définition et rien d'autre, et au fur et à mesure que je l'écrivais je m'apercevais soit que je devais d'abord expliquer ceci ou cela, soit qu'il fallait bien que je dise un mot sur telle chose que j'avais mal comprise, ou simplement parce que je me suis dit que je ne pouvais pas m'affranchir d'un bout d'explication intuitive pour adoucir une présentation trop formelle. Bref, c'est un peu le bordel, surtout que j'ai plusieurs fois changé l'ordre dans lequel je disais certaines choses, et je n'exclus pas d'avoir commis des cercles vicieux de références, mais j'espère que ce qui suit a quand même un intérêt, surtout que ce n'est pas évident de trouver des textes où les choses sont bien expliquées. (Notamment si on ne veut pas savoir ce qu'est un tripos — et personnellement je préfère ne pas avoir à savoir ce qu'est un tripos, donc je ne parlerai pas du tout de tripos dans la suite.)

En plus de ça, je me suis très peu relu, et maintenant que ce billet est fini je n'ai plus vraiment le courage de le relire systématiquement. (Cet avertissement est valable pour toutes les entrées de ce blog, mais celle-ci est particulièrement propice à engendrer des fautes de frappe idiotes donc il y en a certainement à foison.)

Bref, je ne sais pas à quel point ce qui suit est compréhensible, mais j'ai fait un certain effort pour limiter les prérequis : en principe, pour l'essentiel de ce billet je ne suppose du lecteur qu'une familiarité avec la théorie élémentaire des ensembles et les rudiments de la calculabilité ; il n'est pas nécessaire, notamment, de savoir ce qu'est un topos (et d'ailleurs, je ne l'expliquerai pas, je me contente de définir le topos effectif), ni même une catégorie (mais ça doit quand même aider). Je suppose qu'on sait ce que c'est qu'une formule logique (connecteurs et quantificateurs, ce genre de choses), mais guère plus : il n'est pas vraiment nécessaire de savoir ce qu'est la logique intuitionniste, mais c'est utile, surtout à partir de la partie qui parle de réalisabilité, d'en avoir une certaine idée, et la lecture de ce billet ou surtout celui-là peuvent remplir ce prérequis faible. Je ne suppose pas non plus que le lecteur a lu mon précédent billet sur la réalisabilité, mais ça peut aider à motiver les définitions (le topos effectif est une généralisation de la réalisabilité de Kleene). Vers la fin du billet, je suppose la familiarité avec quelques concepts plus sophistiqués (coupures de Dedekind, ordinaux, des choses de ce genre), mais comme il s'agit de petits bouts assez indépendants les uns des autres, on doit pouvoir sauter ce qu'on ne comprend pas.

En tout cas, j'espère au moins avoir réussi à écrire quelque chose de plus clair que l'article Wikipédia ou celui du nLab.

Si on veut en savoir plus que ce qui est expliqué ici (ou corriger les bêtises que j'aurai certainement écrites), le mieux est sans doute de se tourner vers le livre de van Oosten, Realizability: An Introduction to its Categorical Side (2008), qui contient essentiellement tout ce que je raconte ici, mais il vaut mieux sauter directement au chapitre 3 si on ne veut pas entendre parler de tripos. On peut aussi regarder l'article de Bernardet & Lengrand, A simple presentation of the effective topos (ici sur l'arXiv), même si je ne suis pas persuadé que leur présentation soit vraiment plus simple que la version usuelle, elle a le mérite d'être assez compacte et contenue. L'article original de Hyland s'appelle The Effective Topos, et il est paru p. 165–216 dans les actes The L.E.J. Brouwer Centenary Symposium édité par Troelstra & van Dalen (1982) : en voici une version retypographiée et le scan de la version d'origine (disponible sur Sci Hub si vous n'y avez pas accès par ce lien).

Ajout () : on me signale en commentaire ce texte d'Ingo Blechschmidt (un chapitre du livre Exploring mathematical objects from custom-tailored mathematical universes édité par Oliveri, Ternullo et Boscolo, qui est plutôt de portée philosophique) ; cela semble en effet une bonne introduction : il développe de façon pédagogique (et en commençant de façon très informelle) le point de vue les topos comme des mondes mathématiques alternatifs ; néanmoins, il ne donne pas une vraie définition du topos effectif (juste comme une complétion de la catégorie des assemblées). Tant qu'à faire, je peux aussi signaler les notes d'Andrej Bauer sur la réalisabilité, qui sont à l'état d'ébauche au moment où j'écris : ce qui est là est très bien expliqué, mais c'est encore incomplet, et notamment, il n'y a pas de définition du topos effectif.

Ajout () : ce billet ultérieur sur diverses généralisations des degrés de Turing (et sur ce qui est, en fait, les topologies de Lawvere-Tierney sur le topos effectif), quoique indépendant de celui-ci, a néanmoins un rapport assez étroit.

Ajout () : ce billet ultérieur sur la réalisabilité propositionnelle (qu'on peut lire avant, ou après, ou indépendamment) a aussi un rapport étroit avec celui-ci, puisque la réalisabilité propositionnelle est la logique propositionnelle interne du topos effectif.

Table des matières

Motivation

☞ Un monde de la calculabilité

Bref, le but est de donner ci-dessous la définition du topos effectif, c'est-à-dire des « objets » et des « morphismes » du topos effectif, et ensuite de la « réalisabilité » des formules logiques dont les variables sont « typées » par les objets du topos effectif. (Tous les mots entre guillemets doivent être expliqués plus bas.) Pour essayer de donner quand même un avant-goût de quoi il est question avant de passer aux définitions proprement dites, un objet du topos effectif va être une structure qui ressemble à un ensemble, et un morphisme va ressembler à une application, mais l'idée est de construire une sorte de monde mathématique alternatif (sujet aux lois de la logique intuitionniste, cf. ici et surtout ), ça c'est en gros le sens du mot topos, mais ce topos précis ayant la propriété remarquable que toute fonction des entiers vers eux-mêmes est calculable (au sens de Church-Turing), d'où le terme de effectif, et accessoirement toute fonction réelle est continue. (Ces affirmations étant certainement réfutables en logique classique, il est nécessaire de passer à une logique plus faible comme la logique intuitionniste pour espérer les rendre possibles.)

☞ Vérité par témoignages

Toujours pour donner une idée très vague et en agitant les mains de ce dont il va s'agir, le cœur de l'idée du topos effectif est d'utiliser en quelque sorte, les parties de ℕ (i.e., les ensembles d'entiers naturels) comme des sortes de valeurs de vérité (avec les opérations ⊓,⊔,⇛ que je vais définir ci-dessous pour tenir lieu de la conjonction, de la disjonction, et de l'implication logique). De façon un tout petit peu moins vague, pour différentes sortes d'affirmations logiques, le topos effectif va introduire une partie de ℕ (plus bas, je noterai ⟦φ⟧ pour la « valeur de vérité » associée à une formule logique φ), qu'on appellera l'ensemble de ses « réalisateurs » : le terme classique est de dire que n réalise φ pour dire que n appartient à la partie en question (n ∈ ⟦φ), mais je préfère, intuitivement, parler de témoignages de la véracité de φ.

Même si j'en suis juste à une explication très vague et informelle, profitons-en pour dissiper un malentendu possible dans l'idée qu'on peu se faire : la taille de la partie n'importe pas, il suffit de disposer d'un seul « témoignage » pour conclure que φ est valable dans le topos effectif ; ce n'est pas parce que la partie est plus grande qu'on conclut que l'affirmation est plus vraie, la seule chose qui importe est d'arriver à trouver un élément dedans : si ∅ représente le « faux », le « vrai » peut se représenter aussi bien par ℕ ou {0} ou {42}. Du coup, on peut se demander pourquoi il n'y a pas juste deux valeurs de vérité, le vide qui représente le faux, et n'importe quel ensemble habité[#2] qui représente le vrai, et de fait c'est bien le cas quand il n'y a aucun paramètre libre, mais dès que la partie dépend de paramètres, il va y avoir des parties habitées où il sera plus ou moins facile de trouver un élément. Dit comme ça, c'est désespérément vague, mais j'espère que ça aidera à comprendre un peu mieux les définitions précises qui vont suivre.

[#2] Le mot habité (pour un ensemble) signifie simplement non-vide. Je vais dire plus bas pourquoi je l'utilise.

☞ Ensembles avec égalité-existence

Toujours de façon très vague, un objet du topos effectif sera la donnée (X,E) d'un ensemble X (au sens usuel) muni d'une « fonction d'égalité-existence », qui à un couple (x,y) d'éléments de X va associer une partie de ℕ qui indique la « valeur de vérité » du fait que x et y existent et sont égaux (on pourrait certainement séparer les deux rôles en une fonction d'existence et une fonction d'égalité, mais ce serait techniquement moins commode), cette fonction étant symétrique et transitive en un sens qu'on va expliquer plus bas (mais pas réflexive, parce que la réflexivité va servir à mesurer l'existence). Et un morphisme entre objets du topos effectifs est défini, en gros, par une fonction qui indique, de même, la « valeur de vérité » du fait que x est envoyé sur y par le morphisme. Maintenant il s'agit de rendre précises ces idées que je viens de dire de façon très vague.

↑Entry #2752 [older| permalink|newer] / ↑Entrée #2752 [précédente| permalien|suivante] ↑

↓Entry #2721 [older| permalink|newer] / ↓Entrée #2721 [précédente| permalien|suivante] ↓

(vendredi)

(Nouvelle tentative d')introduction aux mathématiques constructives : histoire, motivations et principes

Méta : J'ai déjà fait plusieurs tentatives pour expliquer sur ce blog ce que sont les mathématiques constructives et comment elles fonctionnent (notamment ici — où j'ai tenté d'expliquer les règles de la logique intuitionniste mais en même temps je me suis embourbé dans des explications sur ce que je devrais ou voudrais écrire —, et ici — où j'ai publié l'introduction / motivation d'une entrée que j'avais commencé à écrire et qui, à cause de ça, s'est complètement embourbée). Je considère ces tentatives comme des échecs. Une raison de cet échec est que je n'ai pas correctement expliqué, pour commencer, de quoi il s'agit et pourquoi on s'y intéresse. Plus tard, j'ai participé à un podcast avec mes collègues Sylvie Benzoni-Gavage et David Monniaux sur le thème mathématiques honnêtes (l'expression vient d'une citation de Poincaré, qui a ensuite donné lieu à un échange sur Twitter), où il a été question au passage d'essayer d'expliquer ce que sont les maths constructives ; mais là aussi, j'ai peur de m'être très mal débrouillé quand j'ai évoqué le sujet. Je voudrais donc faire une nouvelle tentative, en reprenant à zéro. Comme cette tentative-ci est de nouveau en train de s'embourber (ça fait maintenant quelque chose comme six mois que j'ai commencé à l'écrire), je me force à en publier le début comme une entrée autonome, où je parle un petit peu de l'histoire et des motivations, puis je commence à développer quelques principes, quitte à ce que la fin soit un peu abrupte.

Au moins le début de cette entrée (où je parle plus d'histoire des mathématiques que de mathématiques) devrait être très largement compréhensible, quitte à sauter quelques passages un peu plus techniques.

De façon extrêmement schématique (et juste pour lancer le sujet : ceci ne se prétend pas être une explication), les mathématiques constructives sont des mathématiques faites dans une logique particulière appelée logique intuitionniste. (Les termes constructif et intuitionniste sont un peu — mais pas complètement — interchangeables.) Cette logique intuitionniste diffère de la logique usuelle dans laquelle on fait des mathématiques (logique classique) en ce qu'elle abandonne une règle de raisonnement, à savoir la loi du tiers exclu, laquelle affirme — schématiquement — que ❝si quelque chose n'est pas faux alors ce quelque chose est vrai❞. (Ou, ce qui revient au même, la logique intuitionniste abandonne le principe du raisonnement par l'absurde où, pour montrer que P est vrai, on suppose « par l'absurde » que P est faux, on aboutit à une contradiction, et on en conclut que P devait être vrai.) La logique intuitionniste est donc plus faible que la logique classique : du coup, prouver quelque chose en logique intuitionniste est plus difficile ou, si on veut, il y a moins de théorèmes (un théorème en logique intuitionniste est encore un théorème en logique classique, mais un théorème en logique classique n'est pas forcément un théorème en logique intuitionniste) ; donc obtenir un résultat « constructivement » est plus fort que l'obtenir « classiquement », et l'étude des maths constructives consiste en bonne partie à se demander quels résultats classiques sont encore valables constructivement, ou, à défaut, comment on peut les démontrer autrement, ou sinon, les reformuler, pour obtenir quelque chose de constructif.

☞ Oui mais pourquoi donc faire ça ? Pourquoi affaiblir la logique ? Pourquoi précisément comme ça ? Pourquoi remettre en cause la loi du tiers exclu ? Quel est l'intérêt de la démarche ? Quelles sont les règles du jeu ? Et pourquoi ces mots constructif et intuitionniste ? C'est ce que je veux essayer d'expliquer ici.

Plan :

Un (tout petit) peu d'histoire du constructivisme en mathématiques

Commençons par essayer d'expliquer comment cette notion est apparue. Qu'on me permette de faire de l'histoire des maths très schématique et simplifiée, juste pour situer un peu les choses et sans prétendre décrire complètement des positions philosophiques forcément assez complexes :

La controverse Hilbert-Brouwer

L'histoire commence au début du XXe siècle à un moment où les fondements des mathématiques commencent à se mettre en place : le monde mathématique a vu se mettre en place des approches rendant l'Analyse plus rigoureuse (Cauchy, Weierstraß, Dedekind…), l'axiomatisation de l'Arithmétique (Peano), la naissance de la théorie des ensembles (Cantor) et de l'idée que celle-ci peut servir à soutenir l'ensemble des mathématiques (Frege). Deux courants apparentés émergent en philosophie des mathématiques, le logicisme et le formalisme (voir ici pour une explication de la différence — qui ne m'intéresse pas tellement ici), qui proposent de ramener, autant que possible, la pratique mathématique à l'application de règles de déduction logique à partir d'un jeu d'axiomes (voire de pure logique dans le cas du logicisme).

Chef de file du courant formaliste, David Hilbert propose, en 1904, un programme visant à fonder les mathématiques sur une base axiomatique : au moins pour une branche donnée des mathématiques, on devrait (selon le programme de Hilbert), trouver des axiomes, formaliser ces axiomes (c'est-à-dire leur donner une forme extrêmement précise ramenant, en principe, la démonstration, à un simple jeu de manipulation de symboles), et idéalement, prouver mathématiquement que les axiomes en question permettent de démontrer ou réfuter tout énoncé syntaxiquement licite, et qu'ils ne comportent pas de contradiction. (La dernière partie de ce programme sera sérieusement mise à mal à cause des limitations posées par le théorème d'incomplétude de Gödel — voir notamment ici —, mais ce n'est pas ce qui me préoccupe ici. On dit parfois que Gödel a porté le coup de grâce au programme de Hilbert, mais il me semble, au contraire que, une fois acceptées ces limitations, le programme de Hilbert a été un grand succès et qu'il est largement admis que les mathématiques ont besoin d'axiomes et de règles de déductions claires même si, dans la pratique, les démonstrations se font généralement en langage informel.)

Hilbert accueille aussi avec enthousiasme la théorie des ensembles, qu'il qualifie de paradis créé par Cantor, parce qu'il permet de rendre précises les constructions admises en la matière, et il accepte, au passage, ses infinis de différentes tailles. (Qui font maintenant partie des mathématiques « standard », un nouveau signe de succès du programme de Hilbert.) Il s'oppose en cela au courant finitiste, dans lequel s'inscrit notamment Kronecker (selon lequel les entiers naturels ont été créés par le bon Dieu, tout le reste est l'œuvre de l'homme) et dans une moindre mesure Poincaré, qui rejettent ou regardent au moins avec soupçon les constructions infinies.

Les règles de logique admises par le programme formaliste, les règles de la logique « classique », permettent souvent de montrer qu'un certain objet mathématique existe sans pour autant exhiber cet objet. Ces raisonnements prennent typiquement une forme du style : je veux montrer qu'il existe un <machin> ; supposons au contraire que <machin> n'existe pas : dans ce cas <…diverses conséquences sont tirées…>, ce qui est une contradiction : ce n'est donc pas possible, et ceci prouve que <machin> existe. À aucun moment le <machin> n'est construit : il est simplement montré qu'il ne peut pas ne pas exister : classiquement, cela revient exactement au même qu'exister, mais cela ne permet pas d'expliciter <machin> ; on peut donc dire que la preuve n'est pas constructive.

Deux exemples significatifs de telles preuves non constructives (ou considérées à l'époque comme non constructives, parce qu'en fait, tout dépend de la manière précise dont on les formalise et/ou démontre) sont donnés par deux théorèmes mathématiques très importants et dus, justement, aux deux protagonistes de notre histoire. Il s'agit d'une part du théorème de la base de Hilbert (1888), avec comme conséquence le fait que les anneaux d'invariants polynomiaux (peu importe de quoi il s'agit) sont finiment engendrés, sans que la démonstration (au moins dans sa forme initiale) exhibe explicitement un système générateur ni ne permette de le calculer, ce qui aurait fait dire à Paul Gordan, le grand spécialiste des invariants, ce n'est pas des mathématiques, c'est de la théologie (en fait, cette phrase, comme toutes les meilleures citations, est probablement apocryphe). Et d'autre part, le théorème du point fixe de Brouwer (c. 1910), dû au topologiste Luitzen Egbertus Jan Brouwer, lequel théorème affirme que toute fonction continue d'une boule dans elle-même a un point fixe, sans que la démonstration (au moins dans sa forme initiale) exhibe un tel point fixe ni ne permette de le calculer.

Ces preuves non-constructives heurtent la conception philosophique de Brouwer, selon lequel prouver l'existence d'un objet ne doit pouvoir se faire qu'en construisant l'objet en question. Il est également en désaccord, plus généralement, avec l'idée formaliste de ramener les mathématiques — au moins en principe — à une application mécanique de règles logiques à partir d'axiomes : pour Brouwer, la créativité de la démarche du mathématicien ne peut pas se ramener à une application formelle de règles. Par ailleurs, Brouwer se rapproche de l'école finitiste par son scepticisme au sujet des constructions infinies arbitraires autorisées par la théorie des ensembles de Cantor (même si on ne peut pas vraiment dire que Brouwer soit un finitiste). Enfin, son intuition du continu, c'est-à-dire de la droite réelle, ne s'accorde pas vraiment avec la formalisation des nombres réels par Dedekind, mais je dois dire que je ne prétends pas vraiment comprendre ce que Brouwer pensait exactement des nombres réels (par opposition aux réinterprétations ultérieures de l'intuitionnisme). Bref, pour ces différentes raisons, Brouwer s'oppose à la philosophie formaliste défendue par Hilbert (ainsi qu'à sa cousine, le logicisme) et, une fois qu'il a obtenu un poste permanent en 1912, il développe ses propres idées auxquelles il donne le nom d'intuitionnisme.

↑Entry #2721 [older| permalink|newer] / ↑Entrée #2721 [précédente| permalien|suivante] ↑

↓Entry #2698 [older| permalink|newer] / ↓Entrée #2698 [précédente| permalien|suivante] ↓

(mercredi)

Sur une entrée en cours d'écriture

Métamotivations : [bon, sérieusement, je ne sais pas ce que sont des métamotivations, mais je trouve ce mot trop rigolo pour ne pas l'écrire]

J'ai commencé il y a quelques semaines à écrire une entrée de ce blog (je vais dire dans un instant à quel sujet, mais pour l'instant ce n'est pas important), et il est arrivé ce qui m'arrive trop souvent : je pars plein d'enthousiasme en me disant que je vais réussir à condenser à sa substantifique moëlle un sujet fort copieux, en même temps que le vulgariser, je me mets à taper, taper, taper, la condensation espérée n'a pas vraiment lieu, au contraire, je me rends compte qu'il faut que je parle de ceci, puis de cela, le texte que j'écris devient de plus en plus indigeste, et, pire, alors que je partais tout content de me dire que l'effort d'exposition me permettra d'y voir plus clair sur le sujet dont je parle, plus je l'écris plus je commence à en avoir marre de l'écrire, donc plus je traîne à m'y mettre, et en plus quand je m'y mets je mets tellement de temps à relire ce que j'ai déjà écrit que les progrès initialement rapides deviennent de plus en plus lent, et cette lente agonie ne prend fin que quand je décide de bouger le billet en cours d'écriture dans un fichier d'entrées inachevées qui prend de plus en plus la forme d'un cimetière, parce que sinon il bloque l'écriture de toute nouvelle entrée.

(Oui, mes phrases sont trop longues. Je sais.)

Que faire quand ce genre de choses se produit ? (C'est fréquent, et j'ai déjà dû en parler plein de fois.) Je n'ai toujours pas trouvé de réponse satisfaisante. Je peux jeter l'éponge et publier l'entrée dans son état inachevé, mais c'est un peu renoncer à toute possibilité de l'améliorer ultérieurement, ce qui me déplaît (il m'arrive bien sûr de modifier des entrées déjà publiées, mais c'est pour ajouter des petites précisions mineures : personne ne va tout relire pour ça). Je peux publier les parties qui sont à peu près présentables (ces entrées fort longues se découpant en parties et sous-parties, je peux publier, disons, le premier chapitre), mais je n'aime pas beaucoup non plus parce que je risque de vouloir quand même y faire des additions ultérieures, et aussi parce que ça rompt ma promesse implicite d'essayer autant que raisonnablement possible d'écrire des entrées indépendantes sur ce blog. (Promesse qui me semble nécessaire si je veux qu'on puisse lire l'entrée #2698 de ce blog sans avoir lu les 2697 qui ont précédé, ce que sans doute personne n'a fait, je ne suis même pas sûr de l'avoir fait moi-même.) Bon, je suis de mauvaise foi, je peux bien dire je suppose qu'on a lu <telle entrée passée>, d'ailleurs il a dû m'arriver quelques fois de le faire, mais si la partie 2 met des années à arriver, personne ne la lira parce que tout le monde aura oublié la partie 1 et que personne n'aura envie de la relire. Même vis-à-vis de moi-même, quand je publie quelque chose sur ce blog, qui est un peu l'espace de swap de mon cerveau, c'est pour me décharger mentalement en me disant que je peux l'oublier puisque je pourrai toujours me relire plus tard (et du coup je ne le fais généralement pas, justement), et ça ne marche plus vraiment si je continue à vouloir donner une suite.

Mais faute d'avoir décidé, je me dis que je peux au moins publier en avance (quitte à la recopier quand même plus tard, ce n'est pas long ce ne sera pas grave) la partie motivations de l'entrée en question. Ne serait-ce que parce que les motivations, c'est un peu comme les remerciements de thèse, c'est la seule chose que doit lire la majorité des gens ; et ici, ils ont, je crois, un intérêt autonome parce que c'est beaucoup moins technique que ce qui vient (enfin, est censé venir) après ; et je ne crois pas avoir envie de les changer plus tard, et elles ont toute leur place ici puisque, finalement, je suis en train de parler de ma motivation.

(En plus, là, j'ai été aspiré par d'autres préoccupations qui m'ont bouffé plein de temps, donc l'écriture de ce billet-ci a elle-même traîné en longueur !)

Voici donc les motivations de ce billet partiellement écrit :

Motivations et introduction générales

Il y a un certain temps, j'avais publié un billet fort mal écrit sur la logique intuitionniste et les mathématiques constructives, pour essayer d'expliquer de quoi il s'agit. J'aimerais réessayer d'en parler, mais en prenant une approche différente : dans le billet précédent (que je ne vais pas supposer que le lecteur a lu, même si ça peut aider de l'avoir au moins parcouru), j'avais mis l'accent sur les règles de la logique intuitionniste (la syntaxe, si on veut), maintenant je voudrais mettre l'accent sur les « mondes » (j'utilise ce terme de façon délibérément vague et informelle, je vais revenir là-dessus) dans lesquels la logique intuitionniste s'applique — la sémantique si on veut. Je pense en effet qu'on comprend beaucoup mieux l'intérêt de cette logique (au-delà de se dire tiens, je vais m'interdire d'appliquer le tiers exclu et voyons ce qui se passe) si on commence par avoir à l'esprit quelques situations où elle s'applique.

Comme on n'a pas besoin de savoir d'avance ce qu'est la logique intuitionniste pour comprendre ces « mondes », cela peut aider, justement, à l'approcher : de l'« intérieur » ils sont régis par la logique intuitionniste, mais de l'« extérieur » ce sont des objets mathématiques classiques (d'ailleurs manipulés régulièrement par des mathématiciens qui n'ont aucune appétence particulière pour la logique, comme les faisceaux en topologie ou géométrie algébrique). Ils forment donc un pont par lequel un mathématicien classique peut comprendre ou visualiser les objets intuitionnistes.

Mais par ailleurs, ces « mondes » sont intéressants, au-delà de l'aspect logique, pour la source de contre-exemples qu'ils fournissent : un monde dans lequel toutes les fonctions ℝ→ℝ sont continues, par exemple, ou dans lequel toutes les fonctions ℕ→ℕ sont calculables, ou dans lequel le théorème des valeurs intermédiaires ne peut pas être affirmé, ou dans lequel on ne peut pas affirmer qu'une suite croissante bornée de réels converge, cela mérite qu'on s'y intéresse même si on n'a aucun intérêt particulier pour la logique : je pense, et c'est en fait ma principale motivation pour m'y intéresser, que cela permet une meilleure compréhension des objets basiques que sont les entiers naturels, les réels, les fonctions entre eux, de comment ils sont construits et de ce qui permet de démontrer ceci ou cela. (Et en comparaison au degré de technicité nécessaire pour construire des mondes dans lesquels l'axiome du choix ne vaut pas, les constructions ici sont raisonnablement peu élaborées bien que plus dépaysantes, ce qui augmente leur intérêt pédagogique.)

J'utilise ci-dessus le mot monde, délibérément vague : disons un mot à ce sujet. En logique classique, le terme correct pour désigner un monde dans lequel vaut une théorie est celui de modèle (pour montrer qu'une théorie T n'implique pas un énoncé φ, on va construire un modèle de T ne vérifiant pas φ). Il y a plusieurs notions de modèle permettant de donner une sémantique à la logique intuitionniste : modèles de Kripke, par exemple, modèles à valeurs dans une algèbre de Heyting, ou encore topoï. Les « mondes » dont je veux parler sont, en l'occurrence, des topoï, mais je ne compte pas expliquer ce qu'est un topos en général, uniquement en donner des exemples. Pourquoi ? Pas que ce soit immensément compliqué si on connaît un peu de théorie des catégories (encore qu'il y a la subtilité qu'il y a deux sens, apparentés mais distincts, du mot topos : topos de Grothendieck [dans ce cas le pluriel est un topos, des topos] ou topoï élémentaires [dans ce cas le pluriel est un topos, des topoï], ces derniers étant plus généraux que ces premiers), mais je pense que la notion de topos ne peut être correctement comprise qu'en ayant au préalable un stock d'exemples, et il s'agit donc de commencer par là. D'autre part, je ne prétends pas énoncer de résultat de complétude. Disons un mot à ce sujet.

[Le paragraphe suivant est une digression par rapport à mon propos général. On peut le sauter sans perdre le fil.]

Quand on a une théorie logique (ce mot étant pris ici, informellement, dans un sens excessivement vague et général), lui définir une sémantique, c'est définir un tas de mondes possibles (même remarque) qui valident certains énoncés (i.e., certaines choses sont vraies dans tel ou tel de ces mondes), et la moindre des choses est que la sémantique donne raison à la théorie, c'est la partie qu'on appelle la soundness en anglais, et je ne sais pas quel mot utiliser en français (cohérence ?) : dire que la sémantique Muf est sound (sensée ?) pour la théorie Truc, c'est dire que tout ce que la théorie Truc permet de démontrer sera vérifié dans chacun des « mondes » Muf. En général, ce n'est pas difficile, c'est même souvent une évidence tellement évidente qu'on ne se fatigue pas à le dire. (Mais remarquez que les « mondes » dont je veux parler, s'ils sont sound pour la logique intuitionniste, ne le sont pas pour la logique classique, et c'est bien pour ça qu'il y aura des théorèmes des mathématiques classiques qui ne seront pas valables dans certains d'entre eux, du genre le théorème des valeurs intermédiaires.) Mais l'autre chose qu'on veut souvent (mais pas forcément) avec une sémantique, c'est qu'elle soit complète, et là ça veut dire la réciproque : dire que la sémantique Muf est complète pour la théorie Truc, c'est dire que tout ce qui est vérifié dans chacun des « mondes » Muf pourra être démontré dans la théorie Truc. En logique classique du premier ordre avec la sémantique fournie par la notion habituelle de modèle, si vous savez ce que c'est, on a effectivement complétude, et le résultat en question s'appelle le théorème de complétude de Gödel (qui affirme que, dans le contexte du calcul des prédicats du premier ordre en logique classique, si φ est vrai dans tout modèle d'une théorie T alors φ est démontrable à partir de T). La question de la mesure dans laquelle les topoï définissent une sémantique complète par rapport à la logique intuitionniste dépend crucialement de ce qu'on appelle exactement la logique intuitionniste (le problème n'étant pas les règles de déduction mais ce sur quoi on s'autorise à quantifier), et essayer d'expliquer ça, sur quoi je n'ai d'ailleurs pas les idées aussi claires que je voudrais, m'entraînerait à parler de logique d'ordre supérieur, de théorie des types et de choses dont je ne veux pas parler. Donc je vais faire complètement l'impasse sur tout ce qui concerne la complétude, et du coup il n'est pas vraiment pertinent que j'explique ce qu'est un topos, ce qui m'importe ce sont les exemples que je veux exposer et le fait qu'ils soient sound.

[Fin de la digression.]

Ce qui est intéressant à comprendre, aussi, c'est le rapport entre le point de vue « interne » et le point de vue « externe » sur les « mondes » en question : on peut soit se plonger dans un de ces « mondes », le regarder avec les lunettes « internes », auquel cas on a affaire à des ensembles dans un monde de maths intuitionnistes, ou au contraire le regarder « de l'extérieur », auquel cas on a affaire à des objets un peu plus compliqués (faisceaux, par exemple, en tout cas des objets d'une catégorie et c'est cette dernière qu'on appelle topos) mais dans un monde plus familier puisque la logique classique s'applique. Les deux points de vue se traduisent l'un l'autre (du moins, tout énoncé « interne » peut se réécrire de façon « externe », la réciproque n'est pas forcément vraie), ils s'éclairent et se complètent. Ce que je voudrais faire c'est expliquer, sur quelques exemples, comment marche cet aller-retour.

↑Entry #2698 [older| permalink|newer] / ↑Entrée #2698 [précédente| permalien|suivante] ↑

↓Entry #2687 [older| permalink|newer] / ↓Entrée #2687 [précédente| permalien|suivante] ↓

(vendredi)

Comment manœuvrer une voiture dans un tunnel : un peu de géométrie

Mon poussinet s'est acheté une nouvelle voiture. Ou plus exactement, a remplacé un joujou, rouge de chez Honda, thermique et bruyant, par un autre joujou, blanc de chez Tesla, électrique et beaucoup moins bruyant et beaucoup plus Internet of Shit ; mais ce n'est pas mon propos ici d'en parler : le point de départ de ce que je veux dire ici, c'est que le nouveau joujou est assez long et large, et que la descente du parking de notre immeuble est compliquée.

Nous avons trois véhicules et trois places de parking (une achetée avec l'appartement et deux louées à des voisins). Ma moto, bizarrement, ne pose aucun problème pour monter et descendre au parking. Pour la Tuture préférée (qui fait censément 1.74m de large, et 4.15m de long dont 2.51m entre les essieux), le poussinet s'en sort bien (le plus souvent sans marche arrière), mais moi je n'ose pas trop essayer, et c'est d'ailleurs mon principal obstacle à circuler en voiture. Et pour le nouveau joujou (qui fait censément 1.85m de large, et 4.69m de long dont 2.83m entre les essieux), c'est beaucoup plus compliqué : à ce stade, à chaque fois que nous voulons entrer ou sortir du parking, je descends de la voiture, je passe devant, et j'indique au poussinet par des signes de main la distance à gauche et à droite (certes, le joujou a plein de capteurs, mais ils sont essentiellement inutiles dans ces circonstances, parce qu'ils passent juste leur temps à faire bip, et ne détectent pas forcément la « bonne » distance) : il arrive à sortir en une seule fois, mais pour rentrer dans le parking il faut trois ou quatre marches arrière (et la question de pourquoi ce n'est pas symétrique est une de celles que je veux discuter ici).

Pourtant, la descente du parking n'est pas si étroite : elle fait 2.67m de large au point le plus resserré que j'aie mesuré ; cette largeur est un peu stupidement grignotée par le fait qu'il y a des garde-corps (je ne sais pas comment les appeler : une sorte de marche ou de micro trottoirs de part et d'autre de la descente, au niveau du sol, large d'environ 15cm et haute d'environ autant), qui réduit l'espace disponible au niveau des roues à 2.38m au point le plus étroit. Mais bien sûr, la descente est courbe : entre la porte côté rue et la base de la rampe, il y a un changement de cap de 90° (on entre perpendiculairement à la rue Simonet, et on finit parallèlement à elle).

Géométriquement, si j'en crois les plans d'architecte de l'immeuble que j'ai récupérés je ne sais plus comment, la forme est très simple (cf. figure ci-contre) : prenez un quart de cercle de rayon 6.50m et un autre de rayon 5.50m dont les centres sont décalés de 1.75m selon chacun des axes du quart de cercle (donc à distance 1.75m×√2 ≈ 2.47m l'un de l'autre), le centre du petit cercle étant évidemment plus éloigné des arcs eux-mêmes que celui du grand cercle, et ensuite prolongez tangentiellement chacune des deux extrémités de chacun des deux arcs. (En notation SVG et en exprimant les distances en mètres : M 0.00 0.00 L 9.37 0.00 A 6.50 6.50 0 0 1 15.87 6.50 L 15.87 18.38 pour le bord extérieur et M 0.00 2.75 L 7.62 2.75 A 5.50 5.50 0 0 1 13.12 8.25 L 13.12 18.38 pour le bord intérieur.) Ceci fait donc une rampe dont la largeur est de (6.50m−5.50m)+1.75m = 2.75m dans ses parties rectilignes, et de (6.50m−5.50m)+1.75m×√2 ≈ 3.47m dans sa partie courbe, à quoi il faut retirer environ 15cm de garde-corps de part et d'autre comme je l'ai expliqué ci-dessus. J'ai fait apparaître sur la figure des rectangles à l'échelle du joujou du poussinet, mais il n'est pas évident de le placer comme je viens de le montrer (cf. plus loin). L'épaisseur des traits sur la figure est de 30cm (le milieu du trait est au niveau du mur lui-même, le bord intérieur correspond à peu près au bord du garde-corps).

Bon, en plus, la rampe est bien sûr en pente (de 14.38% selon le plan d'architecte, l'hélice étant orientée à main gauche : sur le schéma ci-contre, la sortie côté rue est en bas de la figure, la base de la rampe côté parking est à gauche), mais je ne pense pas que ça joue énormément sur le problème géométrique que je vais évoquer. Par ailleurs, la construction de l'immeuble n'a pas respecté précisément les plans d'architecte et les arcs de cercle ont été approchés par des polygones, donc il y a trois ou quatre points anguleux sur le mur extérieur : je ne sais pas bien si ça joue dans l'explication de l'asymétrie ressentie entre montée et descente, je vais y revenir ; la largeur de la rampe, comme je l'ai dit plus haut, n'est, d'après mes mesures, pas tout à fait égale aux 2.75m contractuels de mur à mur, je l'ai déjà noté.

Mes lecteurs savent que j'aime faire des typologies, alors allons-y. Je peux distinguer trois niveaux au problème d'entrer ou sortir la voiture :

  1. La question purement géométrique (entrer ou sortir la voiture, en supposant une connaissance parfaite de ses dimensions, sa position, la forme de la rampe, etc., donc toutes les distances impliquées), que je vais elle-même ci-dessous subdiviser en trois.
  2. La complication supplémentaire que, assis à la place du conducteur, on voit mal ce qu'on fait, on évalue mal les distances, malgré les rétroviseurs et les diverses caméras de la Tesla, en tout cas, plus mal que quelqu'un qui se tient à distance et qui regarde la voiture de l'extérieur.
  3. La complication supplémentaire qu'une voiture n'avance pas exactement comme on veut (je parle du point de vue de la traction : je mets la question du rayon de braquage sur le chapeau géométrique) : ceci ne concerne pas la Tesla, dont le moteur électrique permet d'avancer aussi lentement qu'on veut, presque millimètre par millimètre, aussi bien en montée qu'en descente, mais le problème se pose avec une voiture thermique si on ne veut pas vitrifier l'embrayage en patinant trop longtemps.

Je veux surtout parler ici du (A), même si (B) et (C) sont aussi problématiques en pratique. Maintenant, même si je le simplifie à outrance en traitant la voiture comme un simple rectangle et en ramenant tout le problème dans le plan, ce que je vais faire, le (A) se subdivise lui-même en trois niveaux de difficulté :

↑Entry #2687 [older| permalink|newer] / ↑Entrée #2687 [précédente| permalien|suivante] ↑

↓Entry #2686 [older| permalink|newer] / ↓Entrée #2686 [précédente| permalien|suivante] ↓

(mercredi)

Trucs et astuces pour tirer au hasard diverses choses

Je rassemble dans cette entrée quelques faits algorithmiques et informatiques qui sont généralement « bien connus » (et franchement assez basiques) mais souvent utiles, et qu'il est possiblement difficile de trouver rassemblés en un seul endroit. Le problème général est de tirer algorithmiquement des variables aléatoires selon différentes distributions, typiquement à partir d'un générateur aléatoire qui produit soit des bits aléatoires (indépendants et non biaisés) soit des variables aléatoires réelles (indépendantes) uniformément réparties sur [0;1]. Je parle d'algorithmique, mais ce n'est pas uniquement sur un ordinateur : ça peut être utile même dans la vie réelle, par exemple si on a une pièce avec laquelle on peut tirer à pile ou face et qu'on veut s'en servir pour jouer à un jeu qui réclame des dés à 6 faces, ou si on a des dés à 6 faces et qu'on veut jouer à un jeu d'aventure qui réclame des dés à 20 faces.

Comment tirer des nombres aléatoires en conditions adversariales ? Je commence par ce problème-ci qui n'a pas de rapport direct avec la suite, mais que je trouve quand même opportun de regrouper avec : Alice et Bob veulent jouer à pile ou face, ou plus généralement tirer un dé à n faces, mais ils n'ont pas de pièce ou de dé en lequel ils fassent tous les deux confiance. Par exemple, Alice a sa pièce fétiche que Bob soupçonne d'être truquée et symétriquement (ou peut-être même que chacun est persuadé de pouvoir tirer des nombres aléatoires dans sa tête mais ne fait évidemment pas confiance à l'autre). La solution est la suivante : chacun fait un tirage avec son propre moyen de son côté, sans connaître le résultat de l'autre, et on combine ensuite les résultats selon n'importe quelle opération (choisie à l'avance !) qui donne tous les n résultats possibles pour chaque valeur fixée d'une quelconque des entrées (un carré latin, par exemple une loi de groupe) ; par exemple, s'il s'agit de tirer à pile ou face, on peut décider (à l'avance !) que le résultat sera pile (0) si les deux pièces ont donné le même résultat et face (1) si elles ont donné un résultat différent ; s'il s'agit de dés à n faces donnant un résultat entre 0 et n−1, on fait la somme modulo n (c'est-à-dire qu'on fait la somme et qu'on soustrait n si elle vaut au moins n, pour se ramener à un résultat entre 0 et n−1). Bien sûr, il faut un protocole pratique pour faire en sorte que chacun fasse son tirage sans connaître le résultat de l'autre (sinon, s'il a moyen de tricher, il pourra adapter le résultat en conséquence) : physiquement, chacun peut faire son tirage en secret et écrire le résultat secrètement sur un papier placé dans une enveloppe scellée, qu'on ouvrira une fois les deux tirages effectués (en fait, il n'y a que le premier tirage qui a besoin d'être fait de la sorte) ; cryptographiquement, on procède à une mise en gage (typiquement au moyen d'une fonction de hachage, mais je ne veux pas entrer dans ces questions-là). On peut bien sûr généraliser à plus que deux joueurs (en faisant la somme modulo n de nombres tirés par chacun des participants). Le protocole garantit que le résultat sera un tirage uniforme honnête si l'un au moins des participants désire qu'il le soit (et a les moyens de réaliser un tirage honnête) : bien sûr, si aucun des participants ne le souhaite, c'est leur problème, donc on s'en fout. Même si on abandonne toute prétention à ce que les participants tirent leur valeur aléatoirement et qu'on s'imagine qu'ils la choisissent, tant qu'il s'agit d'un jeu à somme nulle, la stratégie optimale est bien de tirer au hasard (et encore une fois, s'ils veulent coopérer pour un autre résultat, tant qu'il n'y a pas d'autre partie impliquée, c'est leur problème, de même s'ils s'imaginent pouvoir faire mieux que le hasard en utilisant, par exemple, une prédiction psychologique).

Je ne sais plus où j'avais lu que ce protocole a été découvert (il l'a certainement été de nombreuses fois !) à la renaissance. Dans mon souvenir, le découvreur proposait même que, pour une question de la plus haute importance, on demande au pape de faire un des tirages en plus de tous les autres participants. Je ne sais d'ailleurs pas si ce protocole a un nom standard.

Comment tirer une variable de Bernoulli de paramètre p à partir de bits aléatoires ? Autrement dit, ici, on a fixé p, et on veut faire un tirage aléatoire qui renvoie oui avec probabilité p et non avec probabilité 1−p, et pour ça, on dispose simplement d'une pièce qui renvoie des bits aléatoires en tirant à pile (0) ou face (1), et on souhaite effectuer les tirages de façon économique. Par exemple, combien de tirages de pièce faut-il, en moyenne, pour générer un événement de probabilité 1/3 ? Il s'avère, en fait, que quel que soit p on peut s'en tirer avec deux (2) tirages en moyenne (je veux dire en espérance). Pour cela, on peut procéder ainsi : on effectue des tirages répétés et on interprète les bits aléatoires ainsi produits comme l'écriture binaire d'un nombre réel x uniformément réparti entre 0 et 1 : on compare x à 1−p en binaire, c'est-à-dire qu'on s'arrête dès qu'on dispose d'assez de bits pour pouvoir décider si x < 1−p ou x > 1−p (on peut considérer le cas x = 1−p comme s'il était impossible vu qu'il est de probabilité 0), en notant qu'on va avoir x < 1−p lorsque le k-ième bit tiré est 0 et que le k-ième bit de l'écriture binaire de p vaut 0, et x > 1−p lorsque le k-ième bit tiré est 1 et que le k-ième bit de p vaut 1 ; et si x < 1−p on renvoie non, sinon oui. Concrètement, donc, faire des tirages aléatoires jusqu'à ce que le k-ième bit tiré soit égal au k-ième bit de l'écriture de p, et alors s'arrêter et renvoyer ce bit-là. Il est clair que cet algorithme fonctionne, mais pour qu'il soit encore plus évident qu'il conduit à faire deux tirages en moyenne, on peut le reformuler de la façon encore plus élégante suivante (il suffit d'échanger les résultats 0 et 1 pour x, qui sont complètement symétriques, lorsque le bit correspondant de p vaut 0) : tirer des bits aléatoires jusqu'à tomber sur 1, et lorsque c'est le cas, s'arrêter et renvoyer le k-ième bit de p (où k est le nombre de bits aléatoires qui ont été tirés). Je trouve ça incroyablement élégant et astucieux (même si c'est très facile), et je ne sais pas d'où sort ce truc. (Cela revient encore à tirer une variable aléatoire k distribuée selon une loi géométrique d'espérance 1, comme je l'explique plus bas, c'est-à-dire valant k avec probabilité (½)k+1, et renvoyer le (k+1)-ième bit bk+1 de p, ce qui, quand on écrit p = ∑k=0+∞ bk+1·(½)k+1, est finalement assez évident.)

Comment tirer un entier aléatoire entre 0 et n−1 à partir de bits aléatoires ? (Ma première réaction en entendant ce problème a été de dire : considérer x uniforme dont l'écriture binaire est donnée par la suite des bits tirés, générer suffisamment de bits pour calculer la valeur de ⌊n·x⌋, où ⌊—⌋ désigne la partie entière, et renvoyer celle-ci. Ceci fonctionne, mais ce n'est pas le plus efficace. Un autre algorithme avec rejet consiste à générer r := ⌈log(n)/log(2)⌉ bits, qui, lus en binaire, donnent un entier aléatoire c entre 0 et 2r−1, renvoyer c s'il est <n, et sinon tout recommencer. Mais ce n'est pas très efficace non plus, quoique dans des cas un peu différents.) Je décris ce problème plus en détails dans ce fil Twitter, mais donnons juste l'algorithme : on utilise deux variables internes à l'algorithme, notées v et c, qu'on initialise par v←1 et c←0 (il s'agit d'un réservoir d'entropie, et la garantie est que c est aléatoire uniformément réparti entre 0 et v−1) ; puis on effectue une boucle : à chaque étape, on génère un bit aléatoire b (valant 0 ou 1 avec probabilité ½ pour chacun, et indépendant de tous les autres, donc) et on remplace v ← 2v et c ← 2c+b ; puis on compare v avec n et c avec n : si v<n (ce qui implique forcément c<n) on continue simplement la boucle (il n'y a pas assez d'entropie) ; si vn et c<n, on termine l'algorithme en renvoyant la valeur c ; enfin, si cn, on effectue v ← vn et c ← cn et on continue la boucle. Le calcul du nombre moyen de tirages effectués est fastidieux (voir cette référence citée dans le fil Twitter référencée ci-dessus), mais c'est optimal.

L'algorithme que je viens de décrire s'adapte assez bien pour tirer un entier uniforme entre 0 et n−1 à partir d'une source de entiers uniformes entre 0 et m−1 (le cas que je viens de décrire est le cas m=2), autrement dit : comment fabriquer un dé à n faces à partir d'un dé à m faces ? Je n'ai pas vraiment envie de réfléchir à si c'est optimal (mise à jour : on l'a fait pour moi), mais c'est en tout cas assez élégant : on utilise deux variables internes à l'algorithme, notées v et c, qu'on initialise par v←1 et c←0 ; puis on effectue une boucle : à chaque étape, on génère un tirage aléatoire b entre 0 et m−1 à partir de la source dont on dispose et on remplace v ← m·v et c ← m·c+b ; puis on effectue la division euclidienne de v et de c par n : si les deux quotients calculés sont différents (⌊c/n⌋ < ⌊v/n⌋), on termine l'algorithme en renvoyant le reste c%n := cn·⌊c/n⌋ de la division de c par n, tandis que si les deux quotients sont égaux, on remplace chacun par son reste, c'est-à-dire v ← v%n et c ← c%n et on continue la boucle.

Introduisons maintenant aussi des tirages continus.

↑Entry #2686 [older| permalink|newer] / ↑Entrée #2686 [précédente| permalien|suivante] ↑

↓Entry #2685 [older| permalink|newer] / ↓Entrée #2685 [précédente| permalien|suivante] ↓

(vendredi)

Configurations de points et droites : un petit projet mathématico-physico-artistique

Je parle souvent de maths un peu compliquées sur ce blog, alors pour changer (et pour me changer les idées) je vais parler de choses un peu plus simples : de géométrie plane, et plus précisément de points et de droites. Je voudrais évoquer un petit projet que j'ai — qui, comme beaucoup de projets que j'ai, risque de n'aboutir nulle part — et qui ferait intervenir les maths (pour le sujet de fond), la mécanique classique (pour l'animation), l'informatique (pour le calcul) et un côté artistique (parce que le but serait surtout de faire quelque chose de joli à regarder). Enfin, ça c'était l'idée initiale, sauf que, comme d'habitude quand j'écris une entrée dans ce blog (mais bon, c'est un peu l'idée, aussi), je suis tombé dans un terrier de lapin, je me suis perdu dans les méandres de ce que je dois raconter, et au final ça ne ressemble à rien.

La configuration (9₃)① (ou configuration de Pappus) La configuration (9₃)② La configuration (9₃)③

J'ai expliqué par le passé (et souvent fait référence depuis au fait) que j'étais fasciné par la symétrie et par les objets mathématiques très symétriques. En même temps, il ne faut pas oublier que je suis géomètre, et au sein de la géométrie, j'aime beaucoup ce qui en est la forme la plus épurée, la géométrie projective (plane, disons) où il n'est question que de points et de droites. (Je faisais d'ailleurs souvent remarquer à mes élèves agrégatifs quand ils faisaient des développements sur les constructions à la règle et au compas qu'il pouvait être bienvenu de consacrer une certaine attention aux constructions à la règle seule, qui sont les constructions « pures » de la géométrie projective, où on ne peut que relier deux points à la règle et intersecter deux droites.)

À la croisée de ces deux intérêts, il y a la notion de configuration de points et de droites (dans le plan) : une configuration est simplement un ensemble fini de points et un ensemble fini de droites[#]. Les figures ci-contre à droite sont des exemples de configurations de neuf points et neuf droites telles que par chaque point de la configuration passent exactement trois droites et chaque droite passe par exactement trois points. Bien sûr, on peut voir sur cette figure d'autres points, à l'intersection de deux droites de la configuration, mais ceux qui sont des points de la configuration sont uniquement ceux que j'ai marqués en rouge, pas n'importe quel point que vous pouvez voir comme intersection de deux droites (et symétriquement, on peut considérer d'autres droites en reliant deux des points, mais ceux qui sont des droites de la configuration sont celles qui ont été tracées, pas n'importe quelle droite que vous pouvez faire apparaître en reliant deux points).

[#] Pour éviter de considérer des objets sans intérêt, on demandera que chaque point de la configuration soit situé sur au moins une des droites de la configuration (sinon c'est un point isolé qui ne sert à rien), voire deux (sinon c'est un point isolé sur sa droite), voire trois (sinon c'est juste le marqueur d'une intersection), et symétriquement, que chaque droite passe par au moins un des points, voire deux, voire trois. De toute façon, comme je le dis plus bas, on demande en général que la configuration soit régulière, c'est-à-dire que par chaque point passe le même nombre q de droites et que chaque droite passe par le même nombre k de points.

Je ne veux pas parler longuement des configurations de points et de droites, parce que ce n'est pas tellement mon sujet, mais disons-en quand même quelques mots. (Enfin, quelques mots qui, comme d'habitude, se sont multipliés en quelques pages.) Ceux qui veulent juste savoir ce qu'est mon projet peuvent sauter directement plus bas.

Généralement on s'intéresse aux configurations possédant un certain degré de régularité, au moins numérique, c'est-à-dire que par chaque point passe le même nombre de droites et chaque droite passe par le même nombre de points (voire que ces deux nombres sont égaux, ce qui est le cas sur mes exemples), voire un certain degré de symétrie. Spécifiquement, on dit qu'une configuration est de type (pq,nk), où p,q,n,k sont quatre entiers ≥2 (ou en fait plutôt ≥3), lorsqu'elle comporte p points et n droites, que par chaque point passent q droites et que chaque droite passe par k points (ces informations sont donc redondantes et on a pq = nk, ce qui se voit en comptant le nombre total d'incidences d'un point et d'une droite) ; la plupart des textes sur les configurations de points et droites utilisent le mot configuration pour désigner spécifiquement les configurations régulières, c'est-à-dire celles qui sont de type (pq,nk) pour certains paramètres p,q,n,k≥3. Lorsque de plus p=n (ou ce qui revient au même, q=k), on dit simplement qu'on a affaire à une configuration de type (nk), c'est-à-dire n points, n droites, chaque droite passant par k points et par chaque point passant k droites : mes figures à droite sont donc des configurations de type (9₃).

Pour être plus précis, je dois distinguer la notion de configuration abstraite et de réalisation géométrique de la configuration : deux configurations géométriques ont la même configuration abstraite lorsqu'on peut étiqueter (i.e., donner des noms, ce que je n'ai volontairement pas fait sur les figures ci-contre) aux points et aux droites des deux configurations de manière à ce qu'elles se correspondent avec les mêmes incidences, c'est-à-dire que si la droite nommée passe par le point nommé P sur une figure, ça doit aussi être le cas sur l'autre. (Cela pourrait être le cas parce qu'on a déplacé juste un petit peu les points et les droites d'une des figures pour former l'autre, mais ce n'est pas forcément le cas qu'on puisse passer continûment de l'une à l'autre.) Une configuration abstraite est donc la manière de demander quelles droites doivent passer par quels points (par exemple, un triangle abstrait consisterait à dire trois points A,B,C et trois droites a,b,c de manière que a passe par B et C, que b passe par C et A et que c passe par A et B ; et une réalisation géométrique de cette configuration abstraite est simplement un triangle).

Bref, une configuration abstraite est simplement la donnée de deux ensembles finis d'objets, arbitrairement appelés points et droites, et d'une relation d'incidence entre points et droites (on peut dire qu'une droite [abstraite] passe par un point [abstrait] lorsque cette relation est satisfaite) ; si on veut, c'est un graphe bipartite ; et on demandera en outre qu'il existe au plus une droite incidente avec deux points distincts donnés et au plus un point incident avec deux droites distinctes données (ceci correspond au fait que, dans le plan, deux points distincts définissent une droite et que deux droites distinctes se coupent en au plus un point). Une réalisation géométrique d'une configuration abstraite est une façon de trouver des points distincts et des droites distincts dans le plan (encore qu'il faut préciser quel plan : plan euclidien, ou ce qui revient au même, affine réel, plan projectif réel, ou des plans affines ou projectifs différents), en correspondance avec la configuration abstraite à réaliser, de façon qu'une droite passe par un point exactement quand l'incidence a lieu dans la configuration abstraite.

Il y a donc plusieurs questions qui se posent naturellement : quelles sont les configurations abstraites possibles ? (peut-on, par exemple, les dénombrer ? les classifier ? a priori non, cela reviendrait en gros à classifier les graphes bipartites, ce qui n'a guère de sens, il y a juste trop de possibilités, mais on peut s'intéresser à celles qui vérifient certaines contraintes, par exemple ont beaucoup de symétries ; ou on peut simplement en chercher qui sont particulièrement remarquables et intéressantes, et je vais donner quelques exemples ci-dessous) ; parmi elles, quelles sont celles qui sont réalisables ? (peut-on tester ce fait efficacement sur tel ou tel corps ? je dois mentionner que cela revient en fait à tester si un système d'équations polynomiales tout à fait général a des solutions, ce qui est décidable mais très coûteux sur les complexes, décidable mais extraordinairement coûteux sur les réels, et possiblement indécidable sur les rationnels) ; puis on peut encore se poser des questions sur les réalisations d'une configuration donnée, par exemple peut-on passer continûment de l'une à l'autre ? Malheureusement, je doute qu'on puisse dire quoi que ce soit de vraiment intelligent sur aucune de ces questions à ce niveau de généralité (il faut se contenter de résultats du type : pour tout n≥9, il existe au moins une configuration géométrique de type (n₃) dans le plan euclidien).

Quelle est cette configuration ?

Il n'est pas toujours évident, visuellement, de reconnaître quand une configuration abstraite est la même qu'une autre. Par exemple, les trois configurations (9₃) ci-dessus à droite sont distinctes non seulement géométriquement (c'est évident) mais même abstraitement ; et celle qui est ci-contre à gauche, est une réalisation géométrique (différente) d'une des trois configurations abstraites en question, et ce n'est pas forcément immédiat de reconnaître laquelle ! Le lecteur saura-t-il reconnaître laquelle, et saura-t-il montrer que les trois de départ sont bien distinctes ? Pour ça, on peut suggérer l'indication consistant à relier (d'une couleur différente, disons) les paires de points qui ne sont pas situées sur une même droite de la configuration, et regarder le graphe ainsi formé (par exemple, y a-t-il des triplets de points dont aucune paire n'est située sur une droite de la figure ? combien de tels « anti-triangles » y a-t-il ?).

Plus difficile, on peut chercher à montrer qu'il n'y a que trois configurations (9₃) abstraites possibles, et que je les ai donc toutes les trois réalisées géométriquement. (La plus en haut, (9₃)①, s'appelle la configuration de Pappus, parce qu'elle est celle qui intervient dans l'énoncé du théorème de Pappus.) Il y a une unique configuration (8₃) abstraite possible, la configuration de Möbius-Kantor, mais elle n'est pas réalisable géométriquement dans le plan réel même si elle l'est sur les complexes (on peut par exemple l'obtenir en retirant un point et les droites qui vont avec à une autre, de type (9₄,12₃) celle-là, la configuration de Hesse, elle aussi non réalisable sur les réels mais réalisable sur les complexes, qui est celle des neuf points d'inflection d'une courbe cubique lisse). Il y a aussi une unique configuration (7₃) abstraite possible, la configuration de Fano, mais celle-ci n'est réalisable que sur un corps de caractéristique 2.

↑Entry #2685 [older| permalink|newer] / ↑Entrée #2685 [précédente| permalien|suivante] ↑

↓Entry #2683 [older| permalink|newer] / ↓Entrée #2683 [précédente| permalien|suivante] ↓

(samedi)

Sur le modèle SIR avec susceptibilité hétérogène

Je continue dans ce billet de blog une série sur l'épidémiologie mathématique que j'avais commencée avec cette entrée sur le modèle SIR classique, celle-ci sur une variante de SIR où le rétablissement se fait en temps constant, accessoirement celle-ci sur la différence entre seuil d'immunité collective et taux d'attaque, et plus indirectement celle-ci sur des modèles d'hétérogénéité basés sur les graphes aléatoires ; je ne présuppose pas la lecture des billets en question, même si celle du premier a un intérêt, mais je vais en résumer rapidement le contenu.

Mon but aujourd'hui est d'expliquer un peu en détails, mathématiquement, comment on peut modifier le modèle SIR classique (dont le vais rappeler les grandes lignes dans un instant), lequel décrit l'évolution d'une épidémie dans laquelle tout le monde est également susceptible à l'infection, pour le cas d'une susceptibilité hétérogène, c'est-à-dire que certains individus sont plus ou moins susceptibles d'être infectés (= ont plus ou moins de chances d'être infectés dans des circonstances identiques), et on va voir que ces hétérogénéités de susceptibilité ont un impact important. (Je ne me prononce pas sur la cause de ces différences de susceptibilité : elles pourraient être dues à des différences biologiques — certaines personnes s'infectent plus facilement que d'autres — ou sociales — certaines personnes sont plus fréquemment exposées à des conditions infectieuses. Néanmoins, comme le modèle que je vais développer ici suppose que la variation de susceptibilité n'est pas corrélée à une variation d'infectiosité, c'est-à-dire que les personnes plus susceptibles ne sont pas spécialement plus infectieuses — si c'était le cas l'effet que je décris ici serait encore plus accentué — il vaut peut-être mieux imaginer le cas d'une origine biologique, parce qu'une hétérogénéité sociale a plus de chances d'être symétrique.)

Ce qui est assez surprenant, c'est que cette idée, qui peut paraître compliquée à traiter, complique en fait extrêmement peu le modèle SIR, et qu'on peut trouver des réponses exactes à essentiellement les mêmes questions que pour SIR classique (du genre quel est le nombre maximal d'infectés ?) dans ce cadre plus complexe, donnée la distribution (initiale) de susceptibilité dans la population. En général les réponses feront intervenir la transformée de Laplace de la distribution de susceptibilité (je vais expliquer ce que c'est plus bas), mais dans un cas particulier assez naturel (celui d'une distribution Γ, par exemple la distribution exponentielle), on peut tout traiter complètement.

[Un résumé de ce post de blog est contenu dans ce fil Twitter (17 tweets ; ici sur ThreadReaderApp), pour ceux qui préfèrent ce format ou qui veulent surtout les points importants (noter que tweet 11/17 il y a une typo, il faut lire φ′(0)=−1 et pas φ′(0)=1). ※ Une version anglaise (un petit peu plus longue) est contenu dans ce fil Twitter (25 tweets ; ici sur ThreadReaderApp).]

Ajout () : Comme je commence à avoir écrit pas mal de choses au sujet d'épidémiologie sur Twitter, voici un méta-fil (ici sur ThreadReaderApp) rassemblant les plus importants fils que j'ai pondus.

Je commence par rappeler les grandes lignes du modèle SIR classique.

Le modèle SIR classique, donc, étudie l'évolution d'une épidémie dans une population en distinguant trois classes d'individus : les Susceptibles, les Infectieux (qui dans ce modèle sont les mêmes que les infectés) et les Rétablis (qui sont immuns — ou, en fait, morts). Parmi les nombreuses hypothèses simplificatrices faites par ce modèle, il y a les suivantes (j'en oublie certainement) : l'immunité acquise par l'infection est parfaite et permanente, les individus sont infectieux dès qu'ils sont infectés, et ils vont donc soit rester dans l'état S, soit passer succesivement par les étapes S,I,R ; la population est homogène, c'est-à-dire que tous les individus sont également susceptibles et également infectieux une fois infectés, ils ont les mêmes probabilités de se faire infecter, la taille de la population est constante, et elle est assez grande pour être traitée de façon continue déterministe, et les contacts obéissent à une hypothèse de mélange parfait (au sens où tous les contacts sont également plausibles) ; le comportement de la population est constant dans le temps et notamment indépendant de l'évolution de l'épidémie ; les contaminations et le rétablissement obéissent à une cinétique du premier ordre I+S → I+I et I → R respectivement, avec des constantes β (d'infectiosité) et γ (de rétablissement) respectivement, c'est-à-dire le nombre de nouveau infectés par unité de temps est simplement proportionnel au produit du nombre d'infectieux par le nombre de susceptibles, et que le nombre de nouvellement rétablis est simplement proportionnel au nombre d'infectieux.

Bref, si on note s,i,r (quantités réelles entre 0 et 1, fonctions du temps) les proportions de la population formées d'individus susceptibles, infectieux et rétablis respectivement, alors les nouvelles infections par unités de temps se représentent par le terme β·i·s, et les rétablissements par γ·i, du coup le modèle SIR est décrit par le système d'équations différentielles ordinaires (autonomes) du premier ordre suivant :

  • ds/dt = −β·i·s
  • di/dt = β·i·sγ·i
  • dr/dt = γ·i
  • (s+i+r=1)

où on impose en outre généralement les conditions initiales telles que s(−∞)=1, i(−∞)=0 et r(−∞)=0 (je parle bien sûr des limites en −∞), avec i croissant exponentiellement pour t assez proche de −∞ (cf. ci-dessous). La constante β d'infectiosité représente le nombre moyen de personnes qu'une personne infectieuse donnée infecte par unité de temps dans une population entièrement susceptible, tandis que la constante γ de rétablissement représente la proportion moyenne d'infectés qui se rétablissent par unité de temps (donc l'inverse du temps moyen de rétablissement, le temps de rétablissement suivant en fait une loi exponentielle). Notons que β peut aussi, symétriquement, se comprendre comme une constante de susceptibilité, c'est-à-dire comme le nombre moyen de personnes par lesquelles une personne susceptible donnée sera infectée par unité de temps dans une population entièrement infectieuse : c'est la raison pour laquelle je parlerai tantôt de β comme représentant une infectiosité et tantôt une susceptibilité (et comme ici on veut modéliser des variations de susceptibilité, c'est plutôt le deuxième qui va être mis en lumière).

Rappelons quelques uns des points saillants de ce modèle concernant le début, le pic et la fin de l'épidémie, résumé que je recopie de ce billet (plus exactement, comme je viens de le dire, on s'intéresse aux solutions pour lesquelles s→1 quand t→−∞) ; on notera κ := β/γ le nombre de reproduction, que je suppose >1 :

  • tant que s reste très proche de 1 (si on veut, t→−∞), les proportions i et r croissent comme des exponentielles de pente logarithmique βγ = β·((κ−1)/κ), avec un rapport 1/(κ−1) entre les deux, autrement dit comme i = c·exp((βγt) = c·exp(β·((κ−1)/κt) et r = c·(γ/(βγ))·exp((βγt) = c·(1/(κ−1))·exp(β·((κ−1)/κt) (ergotage : dans l'entrée sur le sujet, j'avais mis un −1 aux exponentielles pour r, parce que je voulais partir de r=0, mais je me rends compte maintenant qu'il est plus logique de partir d'une solution où i/r tend vers une constante en −∞, cette constante étant κ−1) ;
  • au moment du pic épidémique (maximum de la proportion i d'infectieux), on a s = 1/κ et i = (κ−log(κ)−1)/κ et r = log(κ)/κ ; notamment, le moment où l'épidémie commence à régresser correspond à i+r = 1 − 1/κ (seuil d'immunité collective) ;
  • quand t→+∞, la proportion i tend vers 0 (bien sûr) et s tend vers Γ := −W(−κ·exp(−κ))/κ (en notant W la fonction de Lambert) l'unique solution strictement comprise entre 0 et 1 de l'équation Γ = exp(−κ·(1−Γ)) (qui vaut 1 − 2·(κ−1) + O((κ−1)²) pour κ proche de 1, et exp(−κ) + O(κ·exp(−2κ)) pour κ grand), tandis qu'évidemment r, lui, tend vers 1−Γ (taux d'attaque final).

J'ai parlé dans cette entrée de la différence entre seuil d'immunité collective et taux d'attaque final (qui sont les deux quantités essentielles que le modèle calcule) dans le modèle SIR.

*

↑Entry #2683 [older| permalink|newer] / ↑Entrée #2683 [précédente| permalien|suivante] ↑

↓Entry #2675 [older| permalink|newer] / ↓Entrée #2675 [précédente| permalien|suivante] ↓

(mercredi)

La réalisabilité de Kleene (comme prélude au topos effectif)

Interrompons un petit peu la succession de rants au sujet du covid pour parler un peu de maths. Je déterre pour le terminer un vieux texte que j'avais commencé il y a environ deux ans et que j'avais abandonné, mais auquel je repense parce que j'ai reréfléchi à des questions adjacentes. Le but est ici de définir un concept à cheval entre la logique et la calculabilité qui s'appelle la réalisabilité de Kleene. Plus tard (un jour, si j'en trouve le temps et la patience) j'aimerais parler du topos effectif, et comme le topos effectif contient (généralise, donne un cadre catégorique à) la réalisabilité de Kleene, il sera utile que j'aie au moins écrit à ce sujet avant, même si en principe on n'a pas besoin de passer par cette étape intermédiaire. [Ajout : le billet sur le topos effectif est ici.]

Tout ceci a un rapport avec la logique intuitionniste : j'ai déjà écrit un post de blog à ce sujet, qu'il peut être utile d'avoir lu, mais il faut admettre qu'il est extrêmement brouillon et mal structuré [ajout : celui-ci, écrit ultérieurement, est sans doute bien plus clair]. En tout état de cause, il n'est pas nécessaire de savoir ce que c'est que la logique intuitionniste pour comprendre la définition de la réalisabilité de Kleene. (Le principal prérequis à ce qui va suivre est de savoir les bases de la calculabilité : ce qu'est une machine de Turing — ou toute autre représentation des fonctions partielles calculables —, et savoir qu'on peut les encoder par des entiers naturels ; donc comprendre une expression comme le résultat de l'exécution de la machine de Turing codée par l'entier e sur l'entier n en entrée. c'est-à-dire l'image de n par la e-ième fonction calculable partielle, qui sera noté en ci-dessous.) Néanmoins, comme la réalisabilité de Kleene est compatible avec les règles de la logique intuitionniste (elle réalise tous les théorèmes de l'arithmétique de Heyting), et est inspirée par (et rend rigoureux) les principes (informels) de Brouwer-Heyting-Kolmogorov, c'est bien d'en avoir au moins une idée ; cette connexion avec la logique intuitionniste sera encore plus forte dans le topos effectif : on peut dire sommairement qu'un topos est une sorte de « monde mathématique alternatif » régis par les lois de la logique intuitionniste (et le topos effectif est un tel monde où la calculabilité joue un rôle central et notamment toutes les fonctions ℕ→ℕ sont calculables).

Mais le but de cette entrée-ci n'est pas de décrire le topos effectif (pas qu'il soit très long à définir, mais ça devient long si on veut en dire le minimum pour que ce soit intéressant). C'est de présenter une notion plus ancienne, la réalisabilité de Kleene, qui sert de fondement ou de prolégomène à la construction du topos effectif. Je dois avouer que le sens profond de cette notion m'échappe encore, et j'ai vaguement l'impression que personne ne la comprend aussi profondément qu'il voudrait ; je ne sais même pas bien ce que Kleene cherchait à faire en introduisant cette notion. Mais superficiellement, l'idée est qu'on définit une notion que je vais noter n ⊪ φ, lire n réalise φ, entre un entier naturel n et une formule logique φ de l'arithmétique du premier ordre (je vais rappeler ce que c'est plus bas), et dont le sens intuitif est que n apporte une sorte de témoignage(?) algorithmique de la véracité de φ (néanmoins, on va voir que φ n'est pas forcément vraie dans le monde classique, et il va s'agir d'expliquer le rapport entre ces notions).

Le rapport avec le topos effectif sera notamment que les énoncés arithmétique du premier ordre qui sont réalisables (au sens où il existe un n qui les réalise) seront exactement ceux qui seront vrais dans le topos effectif. En particulier, la réalisabilité permet de faire un pont vers la logique intuitionniste, même en partant de la logique classique : on n'a pas besoin de savoir ce qu'est la logique intuitionniste ni comment elle fonctionne pour définir la relation n ⊪ φ, mais une fois qu'on l'a définie, on constate qu'elle fonctionne de façon fondamentalement intuitionniste en φ (et notamment, réaliser la double négation ¬¬φ n'est pas du tout pareil que réaliser φ). Par ailleurs, la relation n ⊪ φ est elle-même une formule logique (avec une variable n de plus que la formule de départ), on peut se demander sous quelle condition elle-même est réalisable, ou démontrable (ou démontrablement réalisable), etc., et les réponses sont assez satisfaisantes.

Ajout : ce billet ultérieur, sur la réalisabilité propositionnelle (qu'on peut lire avant, ou après, ou indépendamment) a aussi un rapport étroit avec celui-ci, puisque la réalisabilité propositionnelle est la logique propositionnelle interne du topos effectif de même que la réalisabilité de Kleene dont il est question ici est l'arithmétique du premier ordre dans le topos effectif.

Avant de donner la définition, je dois rappeler des choses sur la logique, qu'on peut sans doute se contenter de lire en diagonale parce que ce qui suit est vraiment standard et peu surprenant.

D'abord, qu'est-ce que c'est qu'une formule de l'arithmétique du premier ordre ? Il s'agit d'une formule fabriquée à partir des connecteurs logiques et de quantificateurs qui ne peuvent porter que sur les entiers naturels. En voici une définition plus précise :

Pour commencer, un terme (de l'arithmétique du premier ordre) est une expression formée à partir d'un stock illimité de variables (que j'appellerai généralement k, , m, n, p, q, etc.) et des constantes représentant les entiers naturels (0, 1, 2, etc.) au moyen des opérations de somme (+), produit (×, souvent noté par simple concaténation) et, pour me simplifier la vie, d'élévation à la puissance (↑, souvent noté en plaçant le deuxième argument en exposant du premier) ; une formule atomique (de l'arithmétique du premier ordre) est celle exprimant l'égalité entre deux termes, par exemple m=2n ou pk+qk=nk sont des formules atomiques ; mettons qu'on accepte aussi comme atomiques les formules d'inégalité, comme ij, cela me simplifiera aussi la vie ; les variables libres d'une formule atomique sont toutes les variables qui y apparaissent (par exemple, dans m=2n, il y a deux variables libres, m et n). Une formule (de l'arithmétique du premier ordre) est définie inductivement de la façon suivante : (A) toute formule atomique est une formule, ainsi que les formules ⊤ (tautologiquement vraie, qu'on peut considérer comme synonyme de 0=0 si on préfère) et ⊥ (tautologiquement fausse, qu'on peut considérer comme synonyme de 0=1 si on préfère), (B) si φ et ψ sont deux formules, alors φψ (conjonction logique), φψ (disjonction logique) et φψ (implication logique) en sont, et leurs variables libres sont celles de φ et de ψ, ainsi que ¬φ (négation de φ, qu'on considérera comme une abréviation pour φ⇒⊥) qui a les mêmes variables libres que φ, et (C) si φ est une formule alors ∀n.φ et ∃n.φ sont des formules ayant les variables libres de φ sauf n (notons qu'elle avait parfaitement le droit de ne pas figurer dans φ, par exemple ∃n.(0=0) est une formule légitime — et d'ailleurs vraie). Il faudrait ajouter des parenthèses dans ce que je viens d'écrire pour éviter les ambiguïtés d'écriture, mais je vais supposer que mon lecteur saura le faire sans plus d'explication. Par ailleurs, il faudrait définir la substitution d'un terme pour une variable libre : si φ(n) désigne une formule ayant possiblement la variable libre n, et si t est un terme, alors φ(t) désigne la substitution de t pour la variable n (là où elle est libre, donc pas à l'intérieur d'éventuels quantificateurs ∀n ni ∃n).

Si t est un terme ne faisant pas intervenir la variable n, on utilise les notations ∀nt.φ et ∃nt.φ comme abréviations (sucre syntaxique) de ∀n.(nt ⇒ φ) et ∃n.(nt ∧ φ). Une formule dont tous les quantificateurs sont de cette forme est dite (arithmétique) à quantificateurs bornés ou (arithmétique) Δ₀. L'intérêt des formules à quantificateurs bornés est que leur véracité peut se tester de façon « finitaire » (si on veut, on a un algorithme qui, donnée une formule à quantificateur bornés, termine de façon certaine en temps fini en renvoyant vrai ou faux selon que la formule est vraie ou fausse).

Une formule n'ayant aucune variable libre est appelée un énoncé.

Ensuite, j'ai besoin de deux notions de codage classiques. Premièrement, on peut coder (=représenter) les couples d'entiers naturels par des entiers naturels : pour ça, je choisis une bijection ℕ²→ℕ calculable (et même primitive récursive, disons (p,q)↦2p(2q+1)−1 pour fixer les idées) dont je note ⟨p,q⟩ l'image du couple (p,q). Deuxièmement, j'ai aussi besoin de coder (=représenter) les machines de Turing (ou toute autre façon de représenter les fonctions calculables partielles ℕ⇢ℕ) par des entiers naturels. Il y a plusieurs notations vaguement standard pour désigner l'exécution de la e-ième machine de Turing (i.e., celle codée par l'entier naturel e) sur l'entier n, c'est-à-dire l'image de n par la e-ième fonction calculable partielle : on note ça parfois Φe(n) (voire φe(n), mais j'ai choisi la lettre ‘φ’ pour désigner des formules logiques donc je ne peux pas), parfois {e}(n), ce qui est une notation franchement pourrie ; une notation plus rare, mais moins problématique, et c'est celle que je vais utiliser, est en (cf. ci-dessous pour l'écriture formelle de cette expression dans l'arithmétique du premier ordre). J'écrirai en↓ pour signifier que en est défini, y compris dans des expressions comme en↓=v (de nouveau, cf. ci-dessous).

Bref, ⟨p,q⟩ désignera le couple (p,q) codé sous forme d'un entier naturel, et en désignera le résultat de l'exécution du programme codé par l'entier naturel e sur l'entier naturel n passé en entrée (et peut donc ne pas être définie si le programme en question n'est pas correct, ou si son exécution ne termine pas, ou si le résultat n'est pas un entier naturel correct).

L'opération en ne fait pas partie du langage de l'arithmétique, mais on peut l'y définir : c'est-à-dire que l'affirmation l'exécution du programme [codé par] e sur l'entrée n termine et renvoie la valeur v (qu'on peut abréger en↓=v) peut s'écrire comme une formule de l'arithmétique du premier ordre. Pour être même un chouïa plus précis, il existe une formule T(e,n,x), le prédicat T de Kleene, qu'on peut écrire explicitement mais je ne le ferai pas, et qui est même à quantificateurs bornés (Δ₀ : cf. ci-dessus), dont le sens intuitif est l'exécution du programme e sur l'entrée n termine avec pour trace d'exécution x (la trace d'exécution étant le détail de tous les calculs qu'a fait, par exemple, la machine de Turing désignée par e) ; et une fonction U(x) dont on peut supposer — et je le ferai — que c'est simplement la projection x=⟨t,v⟩ ↦ v sur la seconde coordonnée, qui transforme une trace d'exécution x en le résultat v renvoyé par le calcul. Ainsi, en↓ (l'exécution du programme e sur l'entrée n termine) s'écrit/signifie ∃x.T(e,n,x) ; et en↓=v (l'exécution du programme e sur l'entrée n termine et renvoie v) s'écrit/signifie ∃t.(T(e,n,⟨t,v⟩)).

★ Voici maintenant la définition de la réalisabilité de Kleene, par induction sur la complexité de la formule réalisée :

  • lorsque φ est une formule atomique (y compris ⊤ ou ⊥), n ⊪ φ signifie simplement φ (i.e., que φ est vraie) : autrement dit, n'importe quel entier naturel réalise une formule atomique vraie, et aucun entier naturel ne réalise une formule atomique fausse ;
  • on a n ⊪ (φψ) lorsque n=⟨p,q⟩ où p ⊪ φ et q ⊪ ψ : autrement dit, les entiers naturels qui réalisent une conjonction sont ceux qui codent un couple formé d'un réalisateur de chaque terme de la conjonction ;
  • on a n ⊪ (φψ) lorsque n=⟨0,p⟩ où p ⊪ φ ou bien n=⟨1,q⟩ où q ⊪ ψ : autrement dit, les entiers naturels qui réalisent une disjonction sont ceux qui codent un couple dont le premier membre indique quel terme de la disjonction est réalisé et le second le réalise ;
  • on a n ⊪ (φψ) lorsque, pour tout p tel que p ⊪ φ, on a (np↓) ⊪ ψ : autrement dit, les entiers naturels qui réalisent une implication sont ceux qui codent un programme (une fonction calculable (partielle)) qui, quand on lui fournit en entrée un entier p réalisant l'antécédent de l'implication, termine en temps fini et renvoie un entier réalisant la conclusion ;
  • en particulier (puisque ¬φ signifie φ⇒⊥ et qu'aucun entier ne réalise ⊥), on a n ⊪ ¬φ (pour n'importe quel n) lorsqu'il n'existe aucun entier p tel que p ⊪ φ ;
  • on a n ⊪ ∀x.φ(x) lorsque, pour tout k, on a (nk↓) ⊪ φ(k) (ou, pour être tout à fait précis, (nk↓) ⊪ φ(‘k’) où ‘k’ désigne la constante qui désigne l'entier naturel k, et φ(‘k’) la substitution de cette constante pour x dans la formule φ(x)) : autrement dit, les entiers naturels qui réalisent une quantification universelle sont ceux qui codent une fonction calculable (totale) qui, quand on lui fournit en entrée un entier k, renvoie un entier réalisant l'instance φ(k) en question de la formule universellement quantifiée ;
  • on a n ⊪ ∃x.φ(x) lorsque n=⟨k,p⟩ où pφ(k) (ou, pour être tout à fait précis, pφ(‘k’) comme dans le point précédent) : autrement dit, les entiers naturels qui réalisent une quantification existentielle sont ceux qui codent un couple dont le second membre un entier réalisant l'instance de la formule existentiellement quantifiée donnée par le premier membre.

Pour être tout à fait exact, je définis par les mêmes clauses énumérées ci-dessus deux variantes de la réalisabilité : il y a une notion dans l'univers mathématique, si j'ose dire, ambiant (n étant un entier naturel et φ une formule de l'arithmétique du premier ordre, n ⊪ φ a le sens défini par les clauses ci-dessus), et il y a une réalisabilité formalisée qui consiste à voir les clauses ci-dessus elles-mêmes dans l'arithmétique du premier ordre, c'est-à-dire qu'on va définir, par exemple, n ⊪ (φψ) comme la formule ∃n.(n=⟨p,q⟩ ∧ (pφ) ∧ (qψ)) et ainsi de suite. Il n'y a pas forcément besoin de distinguer ces deux notions, mais il est important de noter que la réalisabilité peut être formalisée, et donc que n ⊪ φ est elle-même une formule de l'arithmétique du premier ordre (pour n une variable libre n'apparaissant pas libre dans φ). (Ce n'est pas tellement différent de la formalisation de la notion de démonstration : l'affirmation φ est démontrable dans l'arithmétique de Peano, par exemple, est un énoncé arithmétique lorsque φ en est un : cf. ici pour plus d'explications.)

On lit n ⊪ φ en disant que n réalise φ ou que φ est réalisée par n, et n.(nφ) en disant que φ est réalisable.

↑Entry #2675 [older| permalink|newer] / ↑Entrée #2675 [précédente| permalien|suivante] ↑

↓Entry #2664 [older| permalink|newer] / ↓Entrée #2664 [précédente| permalien|suivante] ↓

(lundi)

Petite note technique sur la différence entre seuil d'immunité grégaire et taux d'attaque final

Je veux écrire ici une petite note sur un point que je pense avoir mal expliqué par le passé, et autour duquel il existe une certaine confusion. Il s'agit du rapport entre le seuil d'immunité grégaire d'une part et le taux d'attaque final d'une épidémie d'autre part : quelle est la différence entre ces deux concepts, que penser de l'écart, ou overshoot entre ces deux quantités, et laquelle est la plus pertinente en pratique. Il faudra bien distinguer le cas du modèle théorique SIR et le cas d'une épidémie réelle (et pour faire la transition de l'un à l'autre, j'évoquerai rapidement un modèle SIR « en deux phases »).

Le seuil d'immunité grégaire (ou …collective, peut-être un meilleur terme parce qu'il évoque moins l'image déplaisante d'un troupeau, mais comme j'ai commencé avec grégaire je préfère maintenant rester dessus) est la proportion d'immuns qu'il faut atteindre dans la population pour que le nombre de reproduction effectif de l'épidémie devienne <1. Autrement dit :

  • soit on considère une épidémie naissante, c'est-à-dire qu'il n'y a que très peu d'infectés, mais pour laquelle un certain nombre d'individus sont préalablement immunisés (par exemple par un vaccin) : alors le seuil d'immunité grégaire est le nombre d'immunisés nécessaires pour que l'épidémie ne démarre pas (son nombre de reproduction est <1 donc le petit nombre d'infectés disparaît simplement) ;
  • soit on considère une épidémie déjà en cours, et dans ce cas le fait qu'on atteigne le seuil d'immunité grégaire se voit au fait que le nombre d'infectés commence à décroître, c'est-à-dire qu'il y en a de moins en moins (le nombre de reproduction est <1 donc ce nombre diminue progressivement).

Par contraste, le taux d'attaque final (qui n'a de sens qu'en considérant le cours d'une épidémie particulière) est la proportion d'individus qui seront atteints par l'épidémie pendant toute sa durée.

Même si on suppose que l'immunité est parfaitement stérilisante et dure indéfiniment (ce que je ferai pour simplifier, ce n'est pas le propos ici de discuter de ces questions), ces deux quantités sont différentes : la raison est simple, c'est que même une fois que le seuil d'immunité grégaire est atteint, le nombre d'infectés commence certes à diminuer, mais il n'est pas nul pour autant, donc il y a de l'inertie : ce nombre d'infectés en infecte un plus petit nombre, qui en infecte à son tour un encore plus petit nombre, et ainsi de suite, mais la somme de tout ça n'est pas nulle.

La différence entre ces deux quantités s'appelle l'overshoot de l'épidémie : i.e., l'overshoot est la proportion qu'elle attaque en plus du seuil d'immunité grégaire.

[Graphes des courbes de taux d'attaque et de seuil d'immunité grégaire]Dans le cas du modèle théorique (ultra-simpliste) SIR, on peut calculer explicitement ces deux quantités. Je l'ai fait dans l'entrée que j'ai écrite à ce sujet, mais je n'ai pas été clair parce que je n'avais pas la bonne terminologie (j'ai parlé de modèle extrêmement simpliste pour une description qui calcule, en fait, le seuil d'immunité grégaire, ce qui a pu augmenter la confusion), je redis donc les choses un peu autrement : en notant κ le nombre basique de reproduction (lettre que je préfère à R parce que le R de SIR a un sens différent), c'est-à-dire le nombre de reproduction pour une population immunologiquement (et sociologiquement) naïve :

  • le seuil d'immunité grégaire rherd dans le modèle SIR se calcule à partir du nombre basique de reproduction κ par la formule rherd = 1 − 1/κ (la démonstration est facile : lorsqu'une proportion s des individus est susceptible, le nombre de reproduction effectif tombe de κ à κ·s simplement parce que chaque contact possiblement infectieux a cette probabilité de donner effectivement une infection, du coup pour avoir κ·s = 1 on doit avoir s = 1/κ et cela correspond à la proportion complémentaire r = 1 − 1/κ d'immuns au final) ;
  • le taux d'attaque final r dans le modèle SIR se calcule à partir du nombre basique de reproduction κ par la formule r = 1 + W(−κ·exp(−κ))/κ où W désigne la fonction transcendante W de Lambert (j'ai déjà démontré cette formule dans mon entrée passée sur le sujet).

J'ai tracé ces deux courbes ci-contre en fonction du nombre de reproduction κ : en rouge le seuil d'immunité grégaire, et en bleu le taux d'attaque final. Rappelons en outre, pour ce qui est du comportement asymptotique que [encore une fois, tout ça je l'ai déjà dit, mais avec une terminologie qui n'était pas claire] :

  • si le nombre basique de reproduction κ est juste un peu au-delà de 1, disons 1 + h avec h>0 petit, alors le seuil d'immunité grégaire rherd vaut hh² + O(h³) tandis que le taux d'attaque final r vaut 2·h − (8/3)·h² + O(h³), donc en gros le double (ce qui se conçoit grosso modo par le fait que les deux périodes de l'épidémie sont alors symétriques, celle où elle est croissante jusqu'au seuil d'immunité grégaire, et celle d'overshoot où elle est décroissante jusqu'à tendre vers son taux d'attaque final),
  • si le nombre basique de reproduction κ est grand, alors le seuil d'immunité grégaire rherd vaut 1 − 1/κ (il n'y a rien à simplifier) tandis que le taux d'attaque final r vaut 1 − exp(−κ) − κ·exp(−2κ) + O(κ²·exp(−3κ)), qui devient vite extrêmement proche de 1.

↑Entry #2664 [older| permalink|newer] / ↑Entrée #2664 [précédente| permalien|suivante] ↑

↓Entry #2656 [older| permalink|newer] / ↓Entrée #2656 [précédente| permalien|suivante] ↓

(samedi)

Quelques sujets mathématiques en vrac sous forme de MIGHTDO

Pendant que j'étais occupé à ne pas écrire dans ce blog, les sujets sur lesquels j'aurais pu écrire quelque chose se sont accumulés. Je veux dire, les sujets sur lesquels soit j'ai appris quelque chose et j'aurais pu/dû le braindumper ici pour me simplifier la vie quand j'aurais plus tard oublié et voulu réapprendre, soit je me suis simplement dit que c'était quelque chose de potentiellement intéressant dans quoi je devrais me plonger si j'avais le temps. Bref, voici une liste de quelques choses sur lesquels je n'ai rien écrit, et il n'est pas impossible que j'y revienne, mais il ne faut pas compter dessus non plus. Pas un TODO, mais un MIGHTDO, si on veut.

(Les différentes parties qui suivent n'ont généralement aucun rapport entre elles. C'est bien le problème, si j'ose dire, de trop aimer l'éclectisme. Par ailleurs, elles mélangent des sujets où j'ai quelques trucs à expliquer (mais je ne le fais pas vraiment ici) et d'autres où j'ai simplement des questions à poser, ou encore où je n'ai rien à dire mais que j'utilise simplement comme memento pour me rappeler que c'est quelque chose d'intéressant à visiter ou revisiter un jour. Les explications, ou les absences d'explications, qui suivent, se placent aussi à des niveaux très variés de prérequis mathématiques.)

Liens vers les sous-parties de cette entrée : • Mandelbrot • inégalités de Bell • topologie sans points • axiome de Scott • topos effectif • complexité au-delà du calculable • algèbres de Jordan • problème du secrétaire • mécanique sphérique • surface de Bring • Cayley-Bacharach • le spectre des polynômes à valeurs entières

↑Entry #2656 [older| permalink|newer] / ↑Entrée #2656 [précédente| permalien|suivante] ↑

↓Entry #2654 [older| permalink|newer] / ↓Entrée #2654 [précédente| permalien|suivante] ↓

(samedi)

Quelques considérations de graphes aléatoires pour l'épidémiologie

Même si mon moral est moins mauvais, je continue à avoir beaucoup de mal à faire autre chose que de l'épidémiologie. Du coup, je vais en parler encore une fois, pour présenter une approche différente du calcul du taux d'attaque, qui permet cette fois-ci d'illustrer (par des considérations théoriques plutôt que des simulations numériques) certains effets d'hétérogénéité. (Il s'agit d'une traduction+développement de ce que j'ai écrit dans ce fil Twitter [lien direct Twitter] ainsi que celui-ci [lien direct Twitter], et secondairement, de ce fil [lien direct Twitter] plus ancien.) Mais je commence par quelques remarques d'ordre méta sur ces effets d'hétérogénéité et les épidémiologistes de fauteuil (si ça ne vous intéresse pas, sautez après).

On (un des auteurs !) a enfin fini par me pointer du doigt un livre (et donc une référence citable !) où étaient traitées les probématiques épidémiologiques qui me préoccupaient : il s'agit de Mathematics of Epidemics on Networks (From Exact to Approximate Models) d'István Z. Kiss, Joel C. Miller et Péter L. Simon (Springer 2017). Non seulement il traite exactement tout ce que je voulais voir traité, mais la présentation est vraiment très agréable pour le mathématicien que je suis : les énoncés sont précis, les approximations sont expliquées avec soin, les notations ne sont pas trop pénibles, bref, je le recommande très vivement. (Quel dommage que toutes les bibliothèques soient fermées… Si seulement il y avait un site web — qui pourrait par exemple porter le nom en anglais d'une bibliothèque et du premier livre de la Bible — où on pourrait trouver les PDF de ce genre de choses. Ah non, zut, ce serait illégal, parce qu'on a des lois à la con qui empêchent la diffusion des connaissances. Mais pardon, je digresse.)

Il y aurait peut-être à analyser la raison pour laquelle j'ai réussi à passer à côté de cet excellent ouvrage jusqu'à tout récemment. (Il est possible qu'on me l'ait déjà suggéré et que je sois quand même passé à côté de la suggestion, parce que le mot networks ne m'inspirait pas : en fait, il s'agit de graphes, il y a apparemment des gens qui, parce qu'ils ont une approche un peu différente, parlent de réseaux pour parler de graphes, et notamment de graphes aléatoires, ce qui est leur droit mais ça ne facilite pas la communication. J'aimerais quand même bien comprendre, par exemple, pourquoi si on recherche Galton-Watson "attack rate" dans Google, les deux premières réponses sont de moi, alors que ça a quand même l'air d'être des termes très naturels à rechercher dans le contexte de la propagation des épidémies, et d'ailleurs le livre que je viens de mentionner devrait être dans les résultats, et beaucoup plus haut qu'un tweet à moi.) Mais je ne vais pas m'étendre là-dessus, en tout cas pas maintenant.

Bref, toujours est-il que j'ai été soulagé de voir que tout un tas de phénomènes que je voulais voir étudiés, et que j'avais au moins en partie redécouverts, comme ce que je vais décrire ci-dessous, étaient effectivement étudiés quelque part, et que j'aurai des références citables à montrer. J'ai l'habitude de redécouvrir des résultats connus, je dirais même que ça fait partie du fonctionnement normal de la science, et quand je l'apprends je suis plutôt content que mon intuition ne soit pas complètement à côté de la plaque.

En revanche, je demeure perplexe quant au fait que ces phénomènes soient bien connus ou non des épidémiologistes. Il y a deux prépublications qui sont sorties récemment, une sur l'arXiv (par des matheux) et une autre sur medRxiv (par des épidémiologistes plus médecins, ça se voit au fait qu'ils déposent sur medRxiv et n'utilisent pas TeX ☺️), qui font tous les deux la même observation, évidemment formulée et argumentée de façon plus précise, que j'écrivais dans cette entrée de blog ou de façon concise dans ce tweet (en mars) : l'épidémie va atteindre, et donc immuniser, les personnes les plus connectées en premier, ce qui fait que l'hétérogénéité des contacts contribue à réduire le seuil d'immunité à partir duquel elle se met à régresser (le premier de ces documents calcule 43%, ce qu'il ne faut pas, à mon avis, prendre comme une prédiction mais comme un ordre de grandeur grossier de l'effet qu'on peut attendre). D'un côté, il semble que ce type d'effet ait été étudié depuis 1980 (au plus tard). Mais de l'autre, un épidémiologiste renommé (Marc Lipsitch) semble considérer que c'est intéressant et vaguement nouveau, et il y en a qui n'ont pas reçu le message (et ce n'est qu'un exemple parmi d'autres où j'ai vu affirmer, y compris de la part de personnes qui sont des épidémiologistes ou qui ont une formation proche, que puisque R₀~3 on doit atteindre ~70% d'immunisés pour que l'épidémie régresse). Donc il y a, au minimum, un problème de communication. Ce n'est pas très grave, maintenant j'ai au moins quelque chose d'un peu plus crédible (un PDF !) à citer pour contester cette idée (et le fait que Marc Lipsitch prenne ça au sérieux est bien puisque c'est lui qui est à l'origine, d'avoir popularisé le chiffre de 70% comme taux d'attaque, même s'il l'a immédiatement nuancé). Mais ça reste un peu pénible d'avoir l'impression d'être le crackpot qui vient contredire les experts qui ont dit que c'était 70%. (Un peu quand comme l'OMS a fait une communication un peu hâtive en affirmant qu'il n'y avait aucun signe que l'infection par le Covid-19 confère une quelconque forme d'immunité, alors que quand même, si, il y a des raisons de le penser : ce n'est vraiment pas une position confortable que de tenir le discours je ne suis pas du tout médecin, mais je vais quand même remettre l'OMS à sa place sur une question de médecine. Bon, je digresse encore.)

PS : D'ailleurs, on me souffle que j'ai peut-être contribué à diffuser ces idées. Tant mieux si c'est le cas.

[Taux d'attaque d'une épidémie avec R₀=2.5 en fonction de l'écart-type du nombre de contacts]J'en viens à ce dont je voulais vraiment parler : un modèle basé sur la percolation dans des graphes aléatoires et permettant de modéliser (de façon simpliste !) la manière dont la variance du nombre de contacts infectieux modifie le taux d'attaque d'une épidémie à nombre de reproduction R₀ donné. C'est ce que représentent les courbes ci-contre, en l'occurrence pour R₀=2.5 (contacts infectieux par individu en moyenne), avec l'écart-type σ du nombre de contacts infectieux en abscisse, et en ordonnée le taux d'attaque prédit (en bleu par un modèle basé sur un graphe orienté, en rouge par un modèle symétrique) : je veux expliquer un peu comment lire ces courbes et comment elles ont été calculées.

↑Entry #2654 [older| permalink|newer] / ↑Entrée #2654 [précédente| permalien|suivante] ↑

↓Entry #2648 [older| permalink|newer] / ↓Entrée #2648 [précédente| permalien|suivante] ↓

(jeudi)

Sur une variante à temps de rétablissement constant du modèle épidémiologique SIR

Il y a quatre éternités semaines, quand nous n'étions pas encore maintenus prisonniers chez nous, j'ai parlé ici du modèle épidémiologique SIR, le plus basique qui soit. Je rappelle brièvement les principes qui le définissent :

  • l'immunité acquise est permanente, les individus sont successivement S (susceptibles, c'est-à-dire jamais infectés donc susceptibles de l'être), I (infectés et infectieux) et R (rétablis, c'est-à-dire guéris ou morts) (il existe toutes sortes de variantes, par exemple le modèle SEIR ajoutant un état E (exposé) pour les individus infectés mais non encore infectieux) ;
  • la population est homogène (fongible) avec mélange parfait dans les contacts (j'ai parlé ici de l'effet de modifier cette hypothèse) ;
  • la contamination et le rétablissement se font selon une cinétique d'ordre 1, c'est-à-dire que la contamination se fait proportionnellement aux proportions d'infectés et de susceptibles (avec une constante cinétique β), et que le rétablissement se fait proportionnellement à la proportion d'infectés (avec une constante cinétique γ).

Rappelons brièvement ce que j'ai exposé la dernière fois. Les équations de ce modèle SIR basique, que j'appellerai (*) pour m'y référer plus tard, sont les suivantes (il s'agit d'un système d'équations différentielles ordinaires non-linéaire, du premier ordre et autonomes) :

  • s′ = −β·i·s
  • i′ = β·i·sγ·i
  • r′ = γ·i
  • (s+i+r=1)

s,i,r≥0 sont les proportions de susceptibles, d'infectieux et de rétablis dans la population ; les solutions de ces équations ne semblent pas pouvoir s'exprimer en forme close, mais on peut exprimer s en fonction de r (à savoir s = exp(−κ·r) dans les conditions exposées ci-dessous).

Je rappelle les principales conclusions que j'avais exposées dans mon entrée sur ce modèle (*), en supposant qu'on parte d'une population presque entièrement susceptible avec une proportion infinitésimale d'infectés (plus exactement, on s'intéresse aux solutions pour lesquelles s→1 quand t→−∞) ; on notera κ := β/γ le nombre de reproduction, que je suppose >1 :

  • tant que s reste très proche de 1 (si on veut, t→−∞), les proportions i et r croissent comme des exponentielles de pente logarithmique βγ = β·((κ−1)/κ), avec un rapport 1/(κ−1) entre les deux, autrement dit comme i = c·exp((βγt) = c·exp(β·((κ−1)/κt) et r = c·(γ/(βγ))·exp((βγt) = c·(1/(κ−1))·exp(β·((κ−1)/κt) (ergotage : dans l'entrée sur le sujet, j'avais mis un −1 aux exponentielles pour r, parce que je voulais partir de r=0, mais je me rends compte maintenant qu'il est plus logique de partir d'une solution où i/r tend vers une constante en −∞, cette constante étant κ−1) ;
  • au moment du pic épidémique (maximum de la proportion i d'infectés), on a s = 1/κ et i = (κ−log(κ)−1)/κ et r = log(κ)/κ ;
  • quand t→+∞, la proportion i tend vers 0 (bien sûr) et s tend vers Γ := −W(−κ·exp(−κ))/κ (en notant W la fonction de Lambert) l'unique solution strictement comprise entre 0 et 1 de l'équation Γ = exp(−κ·(1−Γ)) (qui vaut 1 − 2·(κ−1) + O((κ−1)²) pour κ proche de 1, et exp(−κ) + O(κ·exp(−2κ)) pour κ grand), tandis qu'évidemment r, lui, tend vers 1−Γ.

Je veux ici explorer la modification d'une hypothèse de ce modèle (*), celle qui concerne le rétablissement. Quand j'écris ci-dessus que le rétablissement se fait proportionnellement à la proportion d'infectés (avec une constante cinétique γ), au niveau individuel, cela signifie la chose suivante :

Pendant chaque intervalle de temps de longueur (durée) dt très courte, la probabilité qu'un individu infecté (I) se rétablisse (I→R) vaut γ·dt et ce, indépendamment d'un individu à l'autre et d'un instant à l'autre.

Autrement dit, le temps de rétablissement d'un individu infecté donné suit une distribution de probabilité exponentielle d'espérance 1/γ.

Autant l'hypothèse analogue sur la cinétique de la contamination est relativement plausible (si on admet le principe éminemment discutable d'une population homogène et du mélange parfait !), autant l'hypothèse sur le temps de rétablissement est médicalement insensé : on est en train de dire que si vous êtes malade, votre probabilité de guérir (ou d'ailleurs, de mourir) ne dépend pas de l'avancement de votre maladie mais est la même pendant la première heure que pendant la 1729e (si tant est que vous soyez encore malade à ce stade-là). Une maladie ne se comporte pas comme ça !

Cherchons donc à remplacer cette hypothèse par une autre, tout aussi simpliste, mais néanmoins un peu plus proche de la réalité médicale, celle du rétablissement en temps constant.

Un individu infecté (I) se rétablit toujours au bout du même temps T après son moment d'infection.

Autrement dit, le temps de rétablissement d'un individu infecté donné suit une distribution de Dirac concentrée en T (qui est, du coup, son espérance).

↑Entry #2648 [older| permalink|newer] / ↑Entrée #2648 [précédente| permalien|suivante] ↑

↓Entry #2645 [older| permalink|newer] / ↓Entrée #2645 [précédente| permalien|suivante] ↓

(mardi)

Sur l'impact de la structure du graphe social dans le taux d'attaque des épidémies

Je suis vraiment débordé (le temps que je passe à me documenter sur l'épidémie et à répondre aux présentations biaisées et autres conneries sur Twitter représente une surcharge de travail absolument énorme qui s'ajoute au fait que tout est devenu tellement plus long et compliqué dans ma vie, je ne vais pas pouvoir tenir longtemps comme ça), donc je me contente ici de reproduire en français ce que j'ai écrit dans un fil Twitter :

Il s'agit d'expériences numériques sur l'influence de la structure du graphe social sur le taux d'attaque des épidémies (taux d'attaque = le nombre de personnes infectées cumulé pendant l'épidémie).

Rappelons la situation basique : j'ai déjà écrit ici sur mon blog (et ici en anglais sur Twitter) sur ce que prédit le modèle épidémiologique SIR au sujet du taux d'attaque. En bref, il prédit un taux d'attaque énorme : 89% (de la population touchée) pour un nombre de reproduction de 2.5. (La formule, comme je l'ai expliqué, est 1 + W(−κ·exp(−κ))/κ = 1 − exp(−κ) + O(κ·exp(−2κ)) où κ est le nombre de reproduction. Par ailleurs, il faut bien différencier ce taux d'attaque du seuil d'immunité grégaire qui, lui, vaut, 1 − 1/κ, et qui est le taux d'infectés à partir duquel l'épidémie commence à régresser, c'est-à-dire le taux d'attaque au pic épidémiologique.)

Or les épidémies réelles ne semblent pas avoir des taux d'attaque aussi énormes, même avec des nombres de reproduction de l'ordre de ce que je viens de dire. Bien sûr, on connaît mal le taux d'attaque même a posteriori, mais (malgré une absence d'immunité préalable aux souches) il semble que les grippes de 1918 et 1957 aient infecté autour de 30% de la population à différents endroits, pas franchement autour de 90%.

Alors que se passe-t-il ? Mon explication est que SIR, étant un modèle basé sur des équations différentielles, ne connaît qu'une seule chose, c'est la proportion de la population qui est susceptible, infectée et rétablie, et pas où ces personnes sont ni comment elles interagissent socialement.

Autrement dit, un tel modèle suppose un « mélange parfait » : tout individu a la même probabilité d'infecter n'importe quel autre individu. Ce n'est bien sûr pas du tout le cas dans la réalité. En réalité, une bonne proportion des contaminations suit un graphe social (famille, amis, collègues).

Même les modèles plus sophistiqués qui stratifient la population par catégories d'âge (disons) supposent toujours un mélange parfait dans chaque catégorie. Je soupçonne que c'est la raison pour laquelle le papier d'Imperial obtient un taux d'attaque si élevé (j'en ai déjà parlé dans cette entrée, voir aussi ce fil Twitter).

Alors, comment peut-on prendre en compte le fait que les contaminations suivent des graphes sociaux, et que doit-on en attendre ? Je m'attendais, et je voulais tester, deux effets apparentés mais distincts :

Le premier effet est que si l'épidémie doit suivre les liens d'un graphe social de connectivité relativement modeste (chacun n'ayant qu'un petit nombre de parents/amis/collègues par rapport à toute la population), elle va s'étouffer plus rapidement, même pour un nombre de reproduction donné, par rapport au cas de mélange aléatoire : c'est ce que j'ai essayé de dire ici sur Twitter ainsi que dans cette entrée dans la phrase la première [sous-raison] c'est (a) que quand on retire une proportion suffisamment élevées de sommets d'un graphe (en l'occurrence celui des contacts humains), il cesse de « percoler », c'est-à-dire qu'on ne peut plus passer d'un sommet à un autre. Ce phénomène est, en effet, lié à des questions de seuil de percolation dans les graphes (qui est, en gros, la proportion des sommets, ou des arêtes selon la définition, qu'il faut retirer aléatoirement à un graphe pour qu'il cesse d'avoir une composante connexe géante) : l'idée est que quand suffisamment de personnes (=sommets, =nœuds) sont immunisées, l'épidémie ne peut plus se propager d'un point à un autre : même avec l'hypothèse de mélange parfait le nombre d'immunisés ralentit l'épidémie, mais le seuil de percolation suggère qu'une proportion plus faible d'immunisés peut arrêter complètement la propagation (et, probablement, on la ralentit plus vite avant de l'arrêter complètement).

Le second phénomène est différent : non seulement il doit suffire de retirer relativement peu de nœuds pour arrêter l'épidémie (comme je viens de l'expliquer), mais en plus l'épidémie va retirer (c'est-à-dire infecter et rendre immuns) en premier les nœuds les plus « précieux » à sa propre propagation, parce que ce sont les nœuds les plus connectés, les « célébrités ». C'est ce que j'ai essayé d'exprimer ici et  (+ tweet suivant) sur Twitter, ainsi que dans la même entrée que mentionée dans la phrase (b) les infections ont tendance à infecter en premier les personnes qui sont hautement connectées dans le graphe, et en les rendant immunes, elle neutralise en premier les liens qui lui permettaient le plus facilement de se propager.

Tout ça n'est que mon intuition ! Maintenant, voyons si je peux modéliser ces phénomènes, pour au moins montrer qu'ils existent. Je ne vais pas chercher à quantifier les effets (il y a tout simplement trop de paramètres avec lesquels jouer), seulement d'illustrer qu'ils peuvent exister et semblent jouer dans la direction que je pensais.

J'ai donc écrit un petit programme Perl qui simule un modèle épidémique SEIR stochastique. SEIR, ça signifie que les nœuds (les individus) passent entre quatre états, S = susceptible = non-infecté, puis E = exposé = en incubation, puis I = infectieux et enfin R = rétabli = immunisé ou mort. Stochastique, ça signifie que plutôt que modéliser les choses avec des équations différentielles, je prends un grand nombre de nœuds (300 000 dans mes expériences) et les contaminations ont lieu au hasard. Ça rend les calculs non-reproductibles, mais cela permet de gérer des situations bien plus complexes qu'avec des équations différentielles.

↑Entry #2645 [older| permalink|newer] / ↑Entrée #2645 [précédente| permalien|suivante] ↑

↓Entry #2639 [older| permalink|newer] / ↓Entrée #2639 [précédente| permalien|suivante] ↓

(samedi)

Un tout petit peu d'épidémiologie mathématique

Dans l'entrée précédente, je soulevais entre autres la question de comment calculer (et de comment appeler !) le nombre, que j'y appelais r, de personnes qui sont finalement infectés par une épidémie (quelle que soit l'issue de cette infection) puisque c'est un des facteurs du produit f·r qui donnera le taux de mortalité due à l'infection (l'autre étant la proportion f des cas qui conduisent à un décès) ou de tout autre calcul analogue (comme g·r pour le nombre de cas graves où g est la proportion correspondante). Dans plusieurs mises à jour ultérieures de cette entrée, j'ai signalé que j'ai fini par apprendre que r s'appelle le taux d'attaque et un raisonnement simpliste pour l'estimer, que je reproduis ici parce que je vais vouloir le comparer à une estimation donnée par un modèle différent :

[Essentiellement recopié de ce fil Twitter :] Une amie m'a expliqué le rapport que je cherchais à comprendre entre le taux de reproduction de base R₀ (= nombre de personnes que chaque personne infectée infecte à son tour) et le taux d'attaque final r (= proportion de la population qui sera infectée à terme pendant l'épidémie) : dans le modèle le plus simpliste, c'est r = 1 − 1/R₀ ; en effet, tant que le taux de reproduction est >1, l'épidémie croît exponentiellement ; mais si une proportion r a déjà été infectée, le taux effectif de reproduction est ramené à R₀·(1−r) parce que, en supposant que les personnes déjà infectées sont immunisées et sont également réparties dans la population (j'ai bien dit, modèle simpliste !), seule une proportion 1−r est encore susceptible d'être contaminée ; donc l'épidémie cesse de progresser lorsque R₀·(1−r) redescend à 1, c'est-à-dire r = 1 − 1/R₀. C'est probablement la raison pour laquelle certains ont prédit r ~ 70% en l'absence de contre-mesures efficaces pour réduire R₀ qui a été initialement mesuré à R₀ ~ 3. Encore une fois, ceci est un modèle extrêmement simpliste.

Dans la suite, je vais noter plutôt κ que R₀ ce nombre de reproduction, parce que même si R₀ est la notation standard elle serait source de confusion dans le modèle SIR où la lettre R désigne les cas rétablis (guéris, recovered en anglais ; enfin, avec une drôle de définition de rétablis puisque dans le modèle qui va suivre on ne cherche pas à compter les décès et on les compte avec les guérisons). Par ailleurs, plutôt que le taux d'attaque final noté r ci-dessus (ce qui, par chance, colle bien, à la limite, avec l'usage de la lettre R que je viens d'évoquer), je vais m'intéresser plutôt à la proportion complémentaire s = 1−r, i.e., la proportion de la population qui échappe à l'épidémie, et dont le raisonnement simpliste que je viens de recopier prédit donc qu'il s'agit de 1/κ.

Maintenant, en suivant de près ce fil Twitter (ou ici sur Thread Reader), que je développe un peu un peu, je vais essayer d'expliquer la prédiction que fait un modèle basique en épidémiologie, le modèle SIR :

Le modèle SIR modélise une infection en traduisant l'évolution dans le temps de trois variables : s (susceptible) la proportion de la population qui n'a pas encore contracté l'infection (et qui est donc susceptible de l'attraper), i (infectée) la poportion de la population qui est actuellement infectée, et r (rétablie) la proportion de la population qui n'est plus infectée, que ce soit suite à une guérison ou un décès (cf. ci-dessus : on ne s'intéresse pas à la différence ici). On a s + i + r = 1 puisqu'il s'agit de trois parties exclusives et exhaustives : il y a donc seulement deux variables indépendantes. Le modèle fait toutes sortes d'hypothèses simplificatrices : notamment, que la population est constante (puisque les décès comptent parmi les guéris, ce n'est pas idiot), et surtout, que les personnes ayant contracté l'infection ne peuvent pas la contracter une seconde fois (soit parce qu'elles sont immunisées soit parce qu'elles sont décédées).

Il s'agit d'écrire une équation différentielle (non-linéaire, du premier ordre) portant sur ces variables. L'idée est d'écrire le type d'équations utilisées en cinétique chimique : imaginez qu'on aurait deux réactions chimiques, la réaction d'infection S + I → I + I (une personne infectée en infecte une autre) et la réaction de rétablissement, I → R (les personnes infectées se rétablissent toutes seules avec le temps, je rappelle une fois de plus que rétablir ici compte les décès, tout ce qui m'intéresse est que ces personnes ne puissent plus en contaminer d'autres). Ce qu'on fait en cinétique chimie (de façon ultra-simplifiée…) pour modéliser des réactions de type X + Y → Z est qu'on va écrire que l'occurrence d'une telle réaction, i.e., la variation de concentration due à cette réaction (qui va compter positivement dans la concentration de Z et négativement pour X et Y) est proportionnelle à une certaine constante cinétique (positive) fois le produit des concentrations de X et de Y à des puissances appelées l'ordre de la cinétique dans chacun de ces réactifs, typiquement 1. Dans le modèle épidémiologique SIR, les deux réactions d'infection et de rétablissement seront supposées d'ordre 1. On va appeler β et γ leurs constantes cinétiques respectives : les termes de vitesse de l'infection et du rétablissement seront donc β·i·s et γ·i respectivement. Autrement dit :

Si je note x′ la dérivée dx/dt par rapport au temps (t) de la variable x, les équations du modèle SIR seront :

  • s′ = −β·i·s
  • i′ = β·i·sγ·i
  • r′ = γ·i

(La somme de ces trois quantités fait évidemment zéro, comme il se doit puisqu'on doit conserver s+i+r=1 : comme en chimie, rien ne se crée, rien ne se perd, mais tout se transforme.) La première équation, donc, modélise le fait que la population non encore infectée décroît par la vitesse infection dans le temps β·i·s qui est proportionnelle à une constante β fois les proportions de personnes infectées i et susceptibles de l'être s : si l'on préfère, cela signifie qu'une personne susceptible a une probabilité β·i de devenir infectée par unité de temps (très petite) ; la troisième modélise le fait que les personnes infectées deviennent rétablies avec la vitesse γ·i : si l'on préfère, cela signifie qu'une personne infectée a une probabilité γ de devenir rétablie par unité de temps (très petite) ; et l'équation du milieu, donc, assure l'équilibre s+i+r=1.

La nouvelle (et énorme !) hypothèse simplificatice qu'on a faite en écrivant ces équations, c'est de supposer que le comportement « local » de l'épidémie et de la population ne change ni avec le temps ni avec le progrès de l'épidémie : la probabilité d'infection par rencontre S+I, ou de guérison, ne changent pas : ceci exclut, par exemple, le fait que la population changerait ses habitudes avec la progression de l'épidémie (prendrait des mesures prophylactique), que le système de santé soit débordé (ce qui jouerait possiblement sur le temps de guérison), que le pathogène mute pour devenir plus ou moins virulent, et toutes sortes d'autres scénarios sortant de notre modèle extrêmement basique.

Les constantes cinétiques β et γ ont pour grandeur l'inverse d'un temps : il s'agit essentiellement de l'inverse du temps espéré d'infection si toute la population est infectée et du temps espéré de guérison. Remarquons donc qu'en changeant l'échelle de temps on multiplie β et γ par la même constante : le seul paramètre sans dimension dans le modèle est le rapport κ := β/γ, qu'on interprète comme le nombre de personnes qu'une personne infectée infectera en moyenne dans une population entièrement susceptible avant d'être elle-même rétablie. Comme il s'agit du seul paramètre sans dimension, toute discussion doit se faire sur κ. C'est ce κ = β/γ qu'on appelle nombre de reproduction et qui est souvent noté R₀, mais que je préfère noter κ ici pour éviter la confusion avec la variable r.

↑Entry #2639 [older| permalink|newer] / ↑Entrée #2639 [précédente| permalien|suivante] ↑

↓Entry #2616 [older| permalink|newer] / ↓Entrée #2616 [précédente| permalien|suivante] ↓

(mardi)

Merci de ne pas faire dire n'importe quoi à Gödel !

Je m'étonne de ne pas trouver d'endroit où je me serais déjà plaint à ce sujet sur ce blog. Peut-être que je sais mal chercher et qu'un petit gnome serviable va me déterrer ça, mais même si j'ai déjà ranté à ce sujet, ça ne fait pas de mal de me répéter, après tout, radoter est un de mes super-pouvoirs :

Le théorème d'incomplétude de Gödel est sans doute le théorème mathématique le plus abusé par les non-mathématiciens. Cela tient certainement au fait qu'on peut en donner des versions dangereusement approximatives et alléchamment sensationnelles comme on ne pourra jamais tout prouver à partir desquelles il est tentant de faire un pas vers la métaphysique pour tirer des conclusions encore plus fantabuleuses. Je crois avoir vu passer des tentatives d'invoquer ce théorème pour prouver :

  • l'inexistence de Dieu (sur l'air de Gödel assure qu'on ne peut jamais tout savoir, or Dieu est censé être omniscient, donc Dieu n'existe pas),
  • l'existence de Dieu (sur l'air de Gödel assure que la logique et le raisonnement humains ne peuvent pas arriver à toute vérité, donc la vérité est au-delà de l'humain, et c'est qu'elle est divine ; ça me fait penser à cet extrait du film Ridicule)[#],
  • la supériorité de l'humain sur la machine (sur l'air de Gödel montre qu'on ne peut pas mécaniquement arriver à la vérité, mais l'intuition humaine arrive à voir que l'énoncé de Gödel est vrai, c'est donc qu'elle est supérieure à la machine),
  • l'existence de la conscience (je ne sais plus les détails, mais ça devait recouper le raisonnement précédent),
  • l'inexistence de la conscience,
  • que la vérité est inaccessible au seul raisonnement, ou inaccessible tout court,
  • que la quête d'une théorie ultime de la physique est futile,
  • etc.

[#] Une ironie supplémentaire dans l'invocation du théorème d'incomplétude de Gödel pour argumenter pour l'existence de Dieu, c'est que Gödel lui-même a inventé une « preuve » de l'existence de Dieu (ou plus exactement, une formalisation en logique modale de l'argument ontologique de Saint Anselme). Cette preuve ressemble plus à une blague qu'à un argument sérieux, en fait (Gödel introduit une série d'axiomes plus hasardeux les uns que les autres, et dont on sait maintenant qu'ils sont, en fait, sinon contradictoires, au moins amenant des conclusions complètement délirantes, et il en déduit l'existence d'un truc vérifiant la définition de Dieu), et il n'est pas clair si Gödel lui-même la prenait au sérieux. Enfin, bref.

Tous ces raisonnements sont bien sûr du pur pipo. Plus généralement, toute tentative pour donner un sens philosophique (au-delà de la philosophie des mathématiques, bien sûr : métaphysique, théologique, ou même épistémologique si on s'éloigne des mathématiques) au théorème d'incomplétude de Gödel doit être considérée comme hautement suspecte.

Ce que dit le théorème précisément, je ne vais pas le rappeler ici, je l'ai expliqué notamment ici et , avec quel succès je ne sais pas, mais en tout cas ce n'est pas mon propos ici : mon propos est que ce théorème est un énoncé technique sur la logique du premier ordre, et que toute tentative pour le faire sortir de son cadre technique est certainement une arnaque.

Même si on ne comprend pas ce que ceci signifie, peu importe : le théorème d'incomplétude affirme que

  • tout système formel en logique du premier ordre
  • qui soit récursivement (= calculablement) axiomatisé
  • et qui contient (un fragment suffisant de) l'arithmétique

ne peut pas être à la fois consistant [← anglicisme pratique pour cohérent] et complet, i.e., s'il ne prouve jamais simultanément P et ¬P (:= la négation de P), alors il y a un P pour lequel il ne prouve aucun des deux.

Ce que je veux souligner là, c'est qu'il y a des hypothèses techniques (essentiellement trois, celles que je viens de lister), et que si on omet ces hypothèses, on est probablement en train de dire des bêtises.

Plus exactement, ce que j'ai cité est plutôt le théorème d'incomplétude de Gödel-Rosser. Le théorème d'incomplétude de Gödel, ce serait que tout système formel en logique du premier ordre qui soit récursivement axiomatisé et qui contient (un fragment suffisant de) l'arithmétique ne peut pas être à la fois ω-consistant et complet, mais l'ω-consistance est une hypothèse pénible à expliquer (autant supposer le système arithmétiquement vrai, à ce compte-là) et je ne veux pas chercher des noises à ceux qui ne feraient pas la différence entre Gödel et Gödel-Rosser. (Enfin, si on veut ergoter, le théorème d'incomplétude de Gödel, il dit : Zu jeder ω-widerspruchsfreien rekursiven Klasse ϰ von Formeln gibt es rekursive Klassenzeichen r, so daß weder v Gen r noch Neg (v Gen r) zu Flg(ϰ) gehört (wobei v die freie Variable aus r ist) — et j'avoue que j'ai beau connaître l'allemand, avoir lu l'article par le passé, et avoir une bonne idée de ce que c'est censé vouloir dire, ce n'est pas super clair pour autant pour moi. Mais je pense qu'il est raisonnable de qualifier l'énoncé ci-dessus de théorème d'incomplétude de Gödel.)

L'absence de mention de ces trois hypothèses doit être un drapeau rouge à double titre. D'abord, que le raisonnement est suspect (si on invoque un théorème sans vérifier ses hypothèses, alors que celles-ci sont indispensables, c'est sans doute que le raisonnement est incorrect — bien sûr il peut arriver qu'on ne le dise pas explicitement parce que la satisfaction de telle ou telle hypothèse est évidente et se passe de commentaire, mais dans le cas présent, j'ai du mal à imaginer que ce soit possible). Ensuite, que la personne qui tient le raisonnement ne comprend probablement pas bien le théorème qu'elle prétend appliquer si elle n'en connaît pas les hypothèses exact et le sens de celles-ci. Un autre signe à cet égard est d'ailleurs quand on parle du théorème de Gödel comme s'il n'y en avait qu'un (alors que, sans aller chercher loin, Gödel a aussi pondu un théorème de complétude qui très superficiellement et mal interprété pourrait avoir l'air de dire exactement le contraire du théorème d'incomplétude) ; ceci dit, il ne faut pas non plus accorder trop de valeur à ce signe parce que beaucoup de mathématiciens tout à fait sérieux sont susceptibles de parler du théorème de Gödel (ou d'autres auteurs : je parle régulièrement du théorème d'Euler — pour l'affirmation que aφ(m)≡1 (mod m) si a est premier à m — alors qu'Euler a démontré des milliers de théorèmes).

↑Entry #2616 [older| permalink|newer] / ↑Entrée #2616 [précédente| permalien|suivante] ↑

↓Entry #2601 [older| permalink|newer] / ↓Entrée #2601 [précédente| permalien|suivante] ↓

(samedi)

Comment répondre à la question à quoi ça sert ? en maths et sciences fondamentales

Je me livre ici à quelques réflexions (un peu décousues, je dis toujours ça), autour de la question à quoi servent les maths pures, et les sciences fondamentales en générales ?, sur la notion d'utilité et d'applications pratiques. Pas sûr que tout ce que je dise soit très cohérent (je passe sans véritable transition du rapport entre lettres et sciences au rapport entre enseignement et recherche sans développer adéquatement ni l'un ni l'autre), mais j'espère au moins arriver à faire passer l'idée qu'il ne faut pas accepter sans broncher les préjugés les plus banals à ce sujet.

J'ai l'impression que dans l'esprit de beaucoup de gens[#], il y a une dichotomie (assez claire même si elle n'est pas forcément clairement énoncée) entre : d'un côté les sciences et techniques, dont l'importance dans la société et notamment dans l'enseignement est justifiée par leur utilité pratique, et d'autre part les arts et aux lettres et autres humanités, dont l'importance est justifiée par leur rôle culturel. Si je reformule cette idée dans des termes qui sentent bon la fin du 19e siècle et que je mets quelques majuscules d'emphase : on aurait d'un côté ce qui meut l'Humanité sur le chemin du Progrès, et de l'autre ce qui Éclaire ce chemin en montrant la voie vers le Progrès et le distinguant des Ténèbres alentours. Ou quelque chose comme ça. Ce que je veux dire, c'est que dans cette vision des choses, on a d'un côté des domaines comme la médecine ou la physique qui apportent des bienfaits à l'Homme, et de l'autre, ceux comme la philosophie et l'Histoire qui doivent en quelque sorte alimenter son sens moral.

[#] J'accepte bien volontiers que j'énonce peut-être ici un métapréjugé (i.e., un préjugé sur les préjugés que peuvent avoir les gens) : mais ce n'est pas bien grave si je dénonce une idée qui, en fait, n'existe pas vraiment.

Peut-être que je caricature un peu, mais je pense au moins que l'idée est assez répandue que la raison pour laquelle on doit enseigner l'Histoire et la géographie au lycée est que ces disciplines feraient partie de la « culture générale » que tout bon citoyen doit avoir, tandis que la raison pour laquelle on doit enseigner les mathématiques est qu'elles seraient utiles pour toutes sortes de choses.

Bref, je pourrais m'appesantir à dénoncer le stéréotype du littéraire qui considère que la culture générale se limite aux choses qu'il connaît ; qui pense qu'il est indispensable que tous les lycéens français sachent que Le Cid est une pièce de Corneille, que la cinquième république a été établie en 1958 et que les Pyrénées sont à la frontière entre la France et l'Espagne ; mais qui ne sait pas citer une loi de Newton ou de la thermodynamique, ignore si les plantes sont des eucaryotes ainsi que la différence entre une bactérie et un virus, n'a absolument aucune idée du fonctionnement d'Internet ou du Web, et ne sait peut-être même pas dire combien il y a de millimètres cubes dans un mètre cube ; et si on lui montre du doigt ces incohérences, répondra qu'il a fait des études littéraires et que ces questions techniques sont bien plus pointues et d'ailleurs ne lui servent à rien puisqu'il n'est pas scientifique ; et consentira peut-être à donner comme exemple de culture générale scientifique à peu près la seule chose qu'il sait, disons, que la Terre tourne autour du Soleil et pas le contraire. Je caricature ? En fait, non : j'en ai rencontré plus d'un, comme ça, qui se plaignaient que les jeunes ne savaient plus rien de nos jours, et qui démontraient immédiatement après une ignorance crasse et assumée dans tout domaine scientifique (j'ai le souvenir, par exemple, de quelqu'un qui ne savait pas de quoi était fait un atome, et qui avait l'air de trouver totalement fantaisiste la suggestion que cela pouvait faire partie de la culture générale de le savoir). Mais j'ai déjà parlé de ça dans cette entrée passée, et je ne veux pas la répéter ici. J'écrirai Un Jour® une entrée sur la culture générale et l'effet de perspective dont tout le monde est victime — et je m'inclus dans le tout le monde — qui fait qu'on croit toujours indispensables les savoirs qu'on a soi-même et superflus ceux que l'on n'a pas[#2]. [Mise à jour : c'est ici.] Nous avons tous des trous énormes dans notre « culture générale », et c'est normal : ce qui me dérange plus, en fait, est qu'à une époque où nous avons tous tout le savoir du monde à la portée de nos doigts, l'attitude consistant à ne pas se précipiter sur Wikipédia quand on découvre l'existence d'un de ces trous. Mais tout ça est une digression par rapport au sujet général de cette entrée, est je la referme maintenant.

[#2] Ceci vaut d'ailleurs encore au sein d'un domaine : les mathématiciens, par exemple, croient toujours que les outils et théorèmes mathématiques qu'ils connaissent et manipulent sont centraux dans les mathématiques et qu'il est indispensable de les connaître, alors que tout ce qui sort de leur domaine de prédilection est quelque chose d'arcane.

Toujours est-il que cette attitude consistant à imaginer que les sciences doivent être jugées à l'aune de leur utilité déteint au sein des sciences elles-mêmes, et que les scientifiques se retrouvent à justifier leur travail, notamment leur recherche pour ceux qui sont chercheurs, en expliquant que ça peut servir à quelque chose (et disons-le franchement, la plupart de ces justifications sont bidon, ce qui est normal parce que quand on découvre des choses nouvelles, on ne peut pas encore savoir où elles nous mèneront). Et ce n'est pas tout : la notion d'utilité est elle-même insidieusement réduite à celle d'applications.

Dans ces conditions, les sciences pures sont dans une situation très inconfortable d'apparence paradoxale : puisqu'elles sont des sciences, elles sont censées servir à quelque chose, mais puisqu'elles sont pures, elles n'ont pas d'applications ; donc à force d'accepter les différentes idées stupides que j'ai énoncées plus haut, on en revient à devoir trouver des justifications comme ah, mais on ne peut pas encore savoir si ceci aura un jour des applications. Avec comme exemple représentatif la théorie des nombres, que Gauß ou je ne sais qui considérait comme la reine des mathématiques parce qu'elle n'avait pas d'applications et qui finit par en avoir, et d'importance économique absolument capitale, à travers la cryptographie. Cet exemple est juste mais il est trompeur : je veux dire qu'au lieu d'essayer de trouver des justifications dans des exemples pareils, on ferait mieux de rejeter les prémisses idiotes que les sciences sont justifiées par leur utilité et que la seule forme d'utilité est dans les applications pratiques.

Au lieu de ça, la comparaison que j'aime donner est la suivante : imaginer qu'on puisse se passer des sciences pures pour se focaliser sur les applications est comme imaginer qu'on puisse couper les racines d'un pommier parce qu'il n'y a pas de pommes qui poussent dessus. Ce que je veux dire par là est que les sciences, et le savoir humain en général, est comme un être vivant : les différentes parties s'irriguent conceptuellement les unes les autres ; certaines produisent des applications directes, d'autres non, mais s'imaginer qu'on peut amputer des parties sans ruiner la santé de l'ensemble est tout simplement stupide.

↑Entry #2601 [older| permalink|newer] / ↑Entrée #2601 [précédente| permalien|suivante] ↑

↓Entry #2598 [older| permalink|newer] / ↓Entrée #2598 [précédente| permalien|suivante] ↓

(dimanche)

Sur les fonctions réelles continues et le compactifié de Stone-Čech

Le contenu de cette entrée est presque complètement inclus dans le très classique, et remarquablement bien écrit, livre de Leonard Gillman & Meyer Jerison, Rings of Continuous Functions (1960), qui contient d'ailleurs bien d'autres choses intéressantes. Mais j'en avais assez de perdre du temps à retrouver des choses contenues dans ce livre à chaque fois que je les oublie, donc je voulais me faire un aide-mémoire, et à ce moment-là autant le mettre en ligne sur mon blog, d'autant plus qu'il s'agit là de culture générale mathématique (que, selon moi, tout mathématicien devrait avoir, — au moins pour les grandes lignes de ce que je raconte, évidemment, disons les « spoilers » ci-dessous, pas les détails un peu arcanes sur les espaces d'Urysohn et les réelcompacts). Mais mon exposition est assez différente de celle de Gillman & Jerison, et pas seulement parce que le fait de ne pas donner de preuves permet de réorganiser les résultats dans un ordre parfois plus satisfaisant, mais aussi parce que j'ai cherché à développer autant que possible les parallèles entre les faits annoncés, et j'ai une approche un tout petit peu plus « catégorique ».

Comme j'ai écrit énormément de choses très rapidement, y compris des choses qui ne sont pas verbatim dans la littérature (je ne m'en éloigne guère, mais parfois je change un peu les hypothèses ou les conventions, et il faut donc adapter les énoncés : par exemple, Gillman & Jerison supposent les espaces complètement réguliers quand il s'agit de décrire le compactifié de Stone-Čech, ce qui me déplaît énormément ; parfois aussi, quand j'interpole des résultats, je fais une démonstration dans ma tête, mais je n'ai pas tout vérifié avec le soin le plus absolu), il est probable que j'aie fait un certain nombre d'erreurs. On va dire que le but du jeu est de les retrouver !

Je suppose que le lecteur sait déjà ce qu'est un espace topologique et une fonction continue entre espaces topologiques (ainsi que les autres termes de base de la topologie générale : ouverts, fermés, voisinages, intérieur, adhérence, homéomorphisme, sous-espace / topologie induite, topologie produit, espace compact [:= compact séparé], ce genre de choses — cf. par exemple ce glossaire ou différentes pages de ce wiki). Mais je ne suppose pas que le lecteur sait ce qu'est, par exemple, un espace complètement régulier. Je suppose aussi connues les notions d'anneau [sous-entendu : commutatif], ou plutôt de ℝ-algèbre [commutative], et d'idéal d'un tel anneau (et je rappelle qu'un idéal maximal d'un anneau est un idéal ≠(1) et maximal pour l'inclusion parmi les idéaux ≠(1), ou, ce qui revient au même, un idéal tel que quand on quotiente l'anneau par lui on obtient un corps).

Remarque informatique : J'utilise dans ce qui suit les caractères ‘𝔪’, ‘𝔬’, ‘𝔭’, ‘’ et ‘𝒰’ pour, respectivement, un ‘m’ gothique minuscule, un ‘o’ gothique minuscule, un ‘p’ gothique minuscule, un ‘F’ cursif et un ‘U’ cursif. Comme ces caractères peuvent parfois manquer dans des polices j'ai prévu un peu de magie en JavaScript qui remplacera en un seul clic tous ces symboles par des lettres latines toutes bêtes : donc, si vous ne voyez pas les caractères que je viens de nommer, cliquez ici pour activer ce remplacement.

✱ Si X est un espace topologique, on note C(X) l'ensemble des fonctions réelles continues X→ℝ, avec l'addition et la multiplication point à point (c'est-à-dire que f+g est la fonction xf(x)+g(x) et que fg est la fonction xf(x)⁢g(x)) ; chaque réel c est identifié à la fonction constante xc dans C(X). (C'est donc un anneau commutatif et même une ℝ-algèbre commutative. On peut le munir d'autres structures, notamment un ordre partiel défini par fg lorsque f(x)≤g(x) pour tout xX, qui est d'ailleurs un treillis avec fg la fonction xf(x)∨g(x) := max(f(x),g(x)) et fg la fonction xf(x)∧g(x) := min(f(x),g(x)), et une valeur absolue |f| = f∨(−f). On peut éventuellement aussi introduire une ou plusieurs topologies sur C(X), mais ce n'est pas ce qui va m'intéresser ici ; en revanche, je souligne qu'on n'a pas de norme intéressante sur C(X).)

À côté de C(X), on a C*(X) qui est formé des fonctions réelles continues bornées c'est-à-dire les f∈C(X) telles qu'il existe un B∈ℝ tel que pour tout xX on ait |f(x)|≤B. Il est évident que la somme et le produit de deux fonctions bornées sont bornés, si bien que C*(X) est un sous-anneau de C(X).

On peut par ailleurs noter que C et C* sont des foncteurs contravariants des espaces topologiques vers les ℝ-algèbres commutatives, ce qui signifie que donnée une application continue h:XY on fabrique de façon évidente des morphismes C(Y)→C(X) et C*(Y)→C*(X) (remarquer le sens des flèches !), simplement par composition à droite par h, c'est-à-dire qu'elles envoient une fonction continue f:Y→ℝ [éventuellement bornée] sur la composée fh:X→ℝ. On peut noter C(h):C(Y)→C(X) et C*(h):C*(Y)→C*(X) pour ces deux morphismes de « composition à droite par h ». La « fonctorialité » signifie simplement que (i) si id:XX est l'identité alors C(id) et C*(id) sont aussi l'identité, et (ii) si h:XY et k:YZ alors C(kh)=C(h)∘C(k) et C*(kh)=C*(h)∘C*(k).

✱ La problématique qui m'intéresse est de décrire le rapport entre l'espace X et son C(X) et son C*(X), comment on peut retrouver l'un à partir de l'autre, ce genre de choses.

Plus précisément, parmi les questions qu'il est naturel de se poser :

  • La ℝ-algèbre C(X) caractérise-t-elle l'espace topologique X ? Permet-elle de le retrouver (autrement dit, si X₁ et X₂ ont « le même » C(X), c'est-à-dire que C(X₁) et C(X₂) sont isomorphes en tant que, disons, ℝ-algèbres, alors X₁ et X₂ sont-ils homéomorphes) ? Dans les cas où c'est possible, comment peut-on reconstruire X à partir de C(X) ? Par ailleurs, peut-on identifier les ℝ-algèbres qui apparaissent comme des C(X) ? Peut-on, d'ailleurs, retrouver la structure de ℝ-algèbre (i.e., les fonctions constantes et leur isomorphisme avec ℝ) à partir de celle d'anneau ?
  • Mêmes questions pour C*(X).
  • Quel est le rapport entre C(X) et C*(X) ? Peut-on identifier les fonctions bornées de façon purement algébrique ? Pour quel genre d'espace a-t-on C(X) = C*(X) (toutes les fonctions continues sont bornées) ? Comment les ℝ-algèbres qui apparaissent comme des C(X) se situent-elles parmi ceux qui apparaissent comme des C*(X) ? Peut-on notamment trouver un espace Xˆ (en fonction de X) pour lequel on aurait C(Xˆ) = C*(X) (ou le contraire) ?

(Digression : J'ai essayé d'écrire là les questions qu'il me semble qu'on « devrait » vraiment spontanément se poser — et donc chercher à résoudre — dès qu'on introduit ce genre de constructions, sans préjuger de celles qui ont une réponse plus ou moins intéressante. Je trouve toujours agaçants les livres qui traitent d'un sujet mathématique et qui omettent une question qui me semble « évidemment naturelle », ne serait-ce que pour dire qu'on ne connaît pas de réponse satisfaisante ou que les auteurs n'en connaissent pas.)

Quelques spoilers :

  • La ℝ-algèbre C*(X) caractérise l'espace X pour les espaces compacts [séparés]. On pourra alors reconstruire X comme l'ensemble des idéaux maximaux de C*(X). Je crois qu'on ne sait pas caractériser de façon algébrique satisfaisante les ℝ-algèbres C*(X). En revanche, donné un espace topologique X, il y a un unique espace compact βX pour lequel C*(βX) = C*(X) (c'est donc un choix canonique d'espace X′ ayant ce C*(X)) : on l'appelle le « compactifié de Stone-Čech » de X. En général, dire que C*(X₁) et C*(X₂) sont isomorphes va signifier que les espaces ont le même compactifié de Stone-Čech.
  • La ℝ-algèbre C(X) caractérise l'espace X pour tous les espaces dits « réelscompacts » (ce qui inclut énormément de choses, par exemple tous les espaces métriques). On pourra alors reconstruire X comme l'ensemble des idéaux maximaux de C(X) tel que le quotient soit ℝ. Je crois qu'on ne sait pas caractériser de façon algébrique satisfaisante les ℝ-algèbres C(X). En revanche, donné un espace topologique X, il y a un unique espace réelcompact υX pour lequel C(υX) = C(X) (c'est donc un choix canonique d'espace X′ ayant ce C(X)) : on l'appelle le « réelcompactifié [de Hewitt-Nachbin] » de X. En général, dire que C(X₁) et C(X₂) sont isomorphes va signifier que les espaces ont le même réelcompactifié.
  • Les C*(X) sont des cas particuliers des C(X), par cela je veux dire que pour tout espace topologique X il existe un espace Xˆ pour lequel on a C(Xˆ) = C*(X), et (d'après ce qui précède) ceci caractérise complètement Xˆ si on lui impose de plus d'être compact : c'est là aussi le compactifié de Stone-Čech de X (noté βX). On peut caractériser algébriquement les fonctions bornées au sein de C(X) puisqu'on peut même caractériser l'image de f∈C(X), à savoir l'ensemble des c∈ℝ tels que fc ne soit pas inversible dans C(X). Les espaces pour lesquels C(X) = C*(X), ou simplement pour lesquels C(X) est un C*(X′), sont les espaces dits « weierstrassiens » ou « pseudocompacts » (et c'est notamment le cas des espaces compacts).

↑Entry #2598 [older| permalink|newer] / ↑Entrée #2598 [précédente| permalien|suivante] ↑

↓Entry #2592 [older| permalink|newer] / ↓Entrée #2592 [précédente| permalien|suivante] ↓

(lundi)

Quelques mots (essentiellement méta) sur l'intuitionnisme et les mathématiques constructives

Cela fait un certain temps que je me dis que je devrais écrire une ou plusieurs entrées sur ce blog sur des sujets tournant autour de la logique intuitionniste et des mathématiques constructives. La présente entrée est une sorte de TODO étendu où je mélange, de façon malheureusement confuse et désorganisée, des remarques introductives voire vulgarisatrices sur le sujet (et expliquant de quoi il est question), des remarques d'ordre méta (où je dis que je devrais parler de ceci ou de cela, ou bien me demande comment je pourrais le faire, ou encore me dis que je ne comprends pas bien telle ou telle chose) et des explications de fond assez disparates (et faites à des niveaux de prérequis, il faut bien le dire, complètement incohérents). Même l'ordre dans lequel je dis les choses est assez bizarre (à la limite, je me demande s'il ne vaut pas mieux lire cette entrée en commençant par la fin). Je pense que ça vaut quand même la peine de publier tout ça, en conseillant au lecteur de simplement sauter les passages qui lui semblent obscurs (puisque de toute façon il y a très peu de dépendances dans ce que je vais raconter).

En fait, au départ, je me suis surtout dit que ce serait intéressant d'écrire quelque chose sur le topos effectif et la réalisabilité de Kleene. Je peux au moins recopier l'introduction informelle que j'ai commencé à rédiger à ce sujet :

Pour dire très très très sommairement et très très très vaguement de quoi il est question,

  • un topos est une sorte de « monde mathématique alternatif » régi par les lois de la logique intuitionniste (une logique plus faible que la logique usuelle, ou classique, dans laquelle on a essentiellement supprimé la loi du tiers exclu qui affirme que toute formule logique est soit vraie soit fausse, ou, de façon équivalente, que tout ce qui n'est pas faux est vrai) ; de façon un peu plus précise, un topos est une catégorie (peu importe ce qu'est exactement une « catégorie ») qui possède un certain nombre de propriétés communes avec la catégorie des ensembles, ce qui permet d'y mener un certain nombre de constructions mathématiques usuelles, mais dont le comportement va néanmoins être différent sur un certain nombre de choses, et notamment la logique ; et
  • le topos effectif est un topos particulier qui présente un intérêt particulier en calculabilité : il présente un monde alternatif dans lequel toutes les fonctions ℕ→ℕ sont calculables, ce qui nous offre un regard neuf sur la calculabilité par rapport à sa présentation classique, où la logique intuitionniste éclaire ce que sont les raisonnements effectifs (voir à ce sujet cet article introductif d'Andrej Bauer, qui ne parle pas du topos effectif mais explique en quoi travailler en logique intuitionniste peut rendre plus facile ou plus naturelle la calculabilité).

Mais reculons d'un cran : pour parler du topos effectif, je me suis dit qu'il fallait d'abord que j'écrive quelque chose sur la réalisabilité de Kleene, qui en est en quelque sorte le prélude. (Si vous voulez savoir très très très sommairement et très très très vaguement de quoi il est question, S. C. Kleene a introduit dès 1945 une notion appelée « réalisabilité », en tentant de donner un sens précis à l'intuitionnisme, et — en très très très gros — le fait qu'une formule arithmétique φ soit « réalisable » [par un entier naturel n] signifie que n en apporte une sorte de « témoignage algorithmique », par exemple, si φ est une formule du type ∀x.ψ(x) affirmant que ψ(x) est vraie pour tout x, alors « réaliser » cette formule va se faire en apportant un programme qui prend en entrée un entier k et en sortie calcule un entier qui réalise ψ(k) ; pour un peu plus de détails, voir le début de cette question que j'ai posée sur MathOverflow, ou pour encore plus, voir le texte Realizability: An [sic] Historical Essay de Jaap van Oosten.) Le lien entre la réalisabilité de Kleene et le topos effectif est très fort : disons que le topos effectif est défini en cherchant une généralisation assez directe de la réalisabilité à des formules plus complexes que celles de l'arithmétique (et les formules arithmétiques qui sont réalisables sont exactement celles qui sont vraies dans le topos effectif).

Ajout : voir cette entrée ultérieure au sujet de la réalisabilité de Kleene, où je la définis proprement ; en voici depuis un sur la réalisabilité propositionnelle. Nouvel ajout : le billet sur le topos effectif est ici.

Bon, mais je me suis alors rendu compte que ce serait commencer in media res de parler du topos effectif ou même de réalisabilité si je ne commençais pas par parler d'intuitionnisme ou de mathématiques constructives. Et là, je suis embêté par le fait que j'ai à la fois trop de choses à dire et pas assez (trop, parce que ça touche à énormément de sujets ; pas assez, parce qu'en ce faisant ça touche aussi à trop de questions sur lesquelles je ne sais pas grand-chose).

Ajout : ce nouveau billet est une tentative de réécriture complète de celui qui suit, et il est peut-être plus clair (ou peut-être que les deux s'éclairent l'un l'autre, je ne sais pas).

La logique intuitionniste est une logique plus faible que la logique classique dans laquelle on s'interdit la loi du tiers exclu (qui dit que tout énoncé P est soit vrai soit faux ; c'est-à-dire : P∨¬P) ; cela revient essentiellement à s'interdire le raisonnement par l'absurde (ou plus exactement, le raisonnement par l'absurde de la forme je veux montrer P : supposons par l'absurde que P soit faux <…>, j'arrive à une contradiction, donc P ne peut pas être faux, c'est-à-dire qu'il est vrai — c'est la dernière partie qui coince en logique intuitionniste ; en revanche, on peut toujours dire je veux montrer que P est faux : supposons par l'absurde que P soit vrai <…>, j'arrive à une contradiction, c'est-à-dire que P est faux, parce que P est faux signifie précisément que la vérité de P est absurde). Faire des maths sans le tiers exclu peut ressembler à un exercice aussi futile qu'essayer de boxer avec les deux mains attachées derrière le dos, et dans une certaine mesure ça y ressemble effectivement, mais cela présente néanmoins un certain intérêt : non seulement c'est intéressant du point de vue de la pure logique de se demander ce qu'on peut faire sans cet axiome (et cette question a des connexions inattendue avec toutes sortes d'autres parties des mathématiques) ; mais par ailleurs, une démonstration en logique intuitionniste apporte véritablement plus de contenu qu'une démonstration classique : pour commencer, elle est valable de façon plus large (et notamment dans les topoï [pluriel de topos]), mais aussi, elle est constructive (au moins si on part de certains axiomes), c'est-à-dire qu'elle exhibe les objets dont elle affirme l'existence.

Une des idées centrales des maths constructives (qui sont à peu près, quoique pas forcément exactement, la même chose que les mathématiques exercées dans le cadre de la logique intuitionniste) est que si on veut prouver P ou Q (en symboles, PQ), il devrait être suffisant mais aussi nécessaire de prouver soit P, soit Q (et notamment, de savoir lequel des deux est vrai !) : ceci va manifestement complètement à l'encontre du tiers exclu qui postule que pour tout énoncé P, soit P est vrai soit ¬P l'est, sans qu'on puisse forcément trancher lequel (et parfois, effectivement, on ne peut prouver ni l'un ni l'autre). De même, si on veut prouver il existe un x tel que P(x) (en symboles, ∃x.P(x)), il devrait être suffisant mais aussi nécessaire d'exhiber un x pour lequel on peut prouver P(x) : de nouveau, ceci va à l'encontre des raisonnements par l'absurde qui ressemblent à supposons qu'aucun tel x n'existe <…>, j'arrive à une contradiction, donc un tel x doit exister (mais au final ne donnent aucune information pour en construire un).

Ajout () : Juste après avoir publié cette entrée je me rends compte que j'ai oublié d'insérer un paragraphe que je comptais écrire sur l'interprétation de Brouwer-Heyting-Kolmogorov qui tente d'expliquer au moins informellement le sens des connecteurs de la logique intuitionniste. Spécifiquement, il s'agit de variations autour des explications suivantes :

  • une preuve de PQ (conjonction) est un couple formé d'une preuve de P et d'une de Q,
  • une preuve de PQ (disjonction) est la donnée d'une preuve de P ou d'une preuve de Q (avec, bien sûr, l'information de laquelle des deux),
  • une preuve de PQ (implication) est une manière de transformer une preuve de P en une preuve de Q,
  • une preuve de ⊤ (le vrai) est triviale,
  • une preuve de ⊥ (le faux) n'existe pas,
  • une preuve de ∀x.P(x) est une manière de transformer un x en une preuve de P(x),
  • une preuve de ∃x.P(x) est la donnée d'un t particulier et d'une preuve de P(t).

↑Entry #2592 [older| permalink|newer] / ↑Entrée #2592 [précédente| permalien|suivante] ↑

↓Entry #2590 [older| permalink|newer] / ↓Entrée #2590 [précédente| permalien|suivante] ↓

(mardi)

Encore une tentative pour expliquer Gödel

Méta : J'ai écrit un fil un peu long sur Twitter pour tenter d'expliquer le théorème de Gödel, qui reprend grosso modo des idées de cette entrée passée (au moins la partie sur Gödel de celle-ci) mais en mettant l'accent un peu différemment et donnant plus de détails sur les conditions de prouvabilité. Comme ça peut être un complément intéressant et que tout le monde n'aime pas le format Twitter, je reproduis ici ce que j'y ai dit, en reformatant un minimum (en revanche, mon style sur Twitter est sans doute un peu différent de mon style sur ce blog, et je n'ai pas le courage de reformuler plus qu'a minima) :

Je fais d'abord une tentative pour lever la confusion au sujet formalisme. Quand on formalise les [raisonnements] mathématiques, on les décrit sous forme de manipulations de suites de symboles (« syntaxe ») qui obéissent à des règles bien précises. Il est évident que si on demande que les règles de raisonnement elles-mêmes soient formalisées en mathématiques, on a une régression infinie : si quelqu'un prétend ne comprendre que ce qui est formel, c'est turtles all the way down, on ne peut rien démarrer… Pour que le bootstrap soit possible, il faut bien accepter l'idée de décrire les règles de manipulation de la logique en français, ou en faisant appel à des notions mathématiques elles-mêmes pas formalisées (mais néanmoins précises) ! (Refuser cette idée ce serait comme refuser qu'on puisse jouer aux échecs sous prétexte que les règles des échecs n'ont pas été formalisées dans ZFC. Or personne ne pense qu'on a besoin de ZFC pour jouer aux échecs !)

En revanche, ce qu'on peut faire, c'est une fois qu'on a accepté ces règles et construit un système formel avec, utiliser ce système formel pour revisiter (« refléter ») les règles, cette fois-ci formellement : autrement dit, on reconstruit tout le système qu'on a déjà construit, mais on le fait cette fois-ci à l'intérieur du système « externe » qui a été construit informellement. Il y a une mise en abyme. Je parlerai de système interne pour celui qu'on construit ainsi.

Typiquement, ça se fait avec un truc appelé codage de Gödel : si le système externe contient l'arithmétique, on dit qu'on peut refléter toutes les règles formelles comme des manipulations arithmétiques pour fabriquer le système informel. Le code de Gödel d'une formule, c'est l'entier qui représente cette formule (et qui devient, du coup, manipulable par le système externe). Du coup les énoncés comme P est prouvable (qui sont, à la base, informels, parlant du système externe) deviennent des énoncés formels du système externe et qui parlent du système interne. Des énoncés arithmétiques. (Je noterai P plus bas pour P est prouvable.)

Et là, il y a une sorte de postulat épistémologique, qui est que ce que :

Les règles du système interne (formalisées dans le système externe) reflètent correctement les règles du système externe lui-même.

Donc si on croit que le système externe ne dit pas de conneries, et s'il dit que le système interne ne peut pas prouver <ceci-cela>, alors effectivement le système externe ne peut pas prouver <ceci-cela>. (Un ultra-formaliste pourrait rejeter ce postulat et dire : pour moi, les maths formelles sont juste un jeu typographique dénué de sens, le système externe ce sont les règles du jeu, le prétendu système interne n'a pas de sens, pas plus qu'aucun énoncé du jeu. Mais en vrai, les gens font des maths parce qu'ils croient qu'une preuve du fait que 2+2=4 apporte quelque information sur le monde réel, donc il y a bien une connexion entre le système externe, qui vit dans le monde réel, et le système interne, formalisé.)

Accessoirement, les systèmes externe et interne n'ont pas vraiment besoin d'être les mêmes : en fait on a besoin de très peu d'axiomes pour faire fonctionner Gödel. Mais je ne sais pas si ça aide de dire ça. Donc restons dans l'idée que ce sont « les mêmes ».

Maintenant, de quels ingrédients a-t-on besoin pour prouver Gödel ? On a besoin des trois « conditions de prouvabilité de Hilbert-Bernays » (que je noterai (A), (B), (C)), et de l'astuce de Quine. Expliquons ça successivement :

Les conditions de prouvabilité de Hilbert-Bernays remplacent le postulat épistémologique dont j'ai parlé plus haut par quelque chose de précis et de formel. En gros, elles font le lien entre les niveaux externe et interne (et interne², cf. plus bas).

Première chose : (A) si le système externe prouve un énoncé P, alors il prouve que le système interne prouve P [ou plutôt, le code de Gödel de P]. Pourquoi ? Parce que si on a une preuve de P, on peut « refléter » cette preuve : la réécrire comme une preuve formelle dans le système interne, et ceci fournit une preuve de son existence dans le système externe. Bien sûr, tout ce que je viens de dire est informel, puisque (A) est par essence informel ! Mais si on applique ça à un P bien précis et une preuve de P formelle explicite, la recette que je viens de dire donne une preuve tout à fait explicite et formelle (dans le système formel externe) de l'existence d'une preuve dans le système interne. Donc, si on veut, (A) est un métathéorème informel : en soi il n'est pas formel, mais il s'instancie en des théorèmes formels (du système externe) dont on a la recette de construction.

Maintenant on peut refaire tout ça avec un niveau de plus (on a alors trois systèmes : l'externe est informel, l'interne est formalisé dans le système externe, et l'interne² dans le système interne — ça s'arrêtera là) : Tout ce que j'ai dit sur le (A) vaut de nouveau et donne, cette fois, une preuve formelle (dans le système externe) du fait (B) suivant : si le système interne prouve P alors il prouve que le système interne² prouve P. Cette fois c'est un vrai théorème du système externe, pas juste un métathéorème informel.

Enfin, (C) dit que si le système interne prouve [le code de Gödel] de PQ et [celui de] P, alors il prouve [celui de] Q. Ça c'est juste le fait qu'on a la règle de modus ponens dans le système interne (c'est presque une définition).

Bref, si je note □P l'énoncé (du système externe) qui dit qu'il existe une preuve de P dans le système interne, mes conditions de prouvabilité sont :

  • (A) si P est un théorème alors □P en est un,
  • (B) □P⇒□□P est un théorème,
  • (C) □(PQ)⇒□P⇒□Q est un théorème

(tous ces théorèmes dans le système externe ! et ce, quels que soient les énoncés P et Q).

Maintenant, l'astuce de Quine (en fait, le procédé diagonal), c'est quelque chose qui permet de fabriquer un énoncé G tel que G⇔¬□G (démontrablement dans le système externe !), autrement dit un énoncé qui dit je ne suis pas un théorème.

L'astuce fonctionne exactement comme la manière dont on écrit des programmes qui écrivent leur propre code, chose que j'explique en détails dans cette page web consacrée aux quines. On fabrique une formule R(x) (du système externe) qui dit si x est une formule du système interne ayant une variable libre, et qu'on remplace cette variable par le code de Gödel de x elle-même, alors le résultat n'est pas un théorème (du système interne), ou de façon encore plus informelle, R(x) signifie x(‘x’) n'est pas un théorème (soit ¬□x(‘x’)) en notant ‘x’ le code de Gödel de x. Mais du coup, R(‘R’) équivaut à R(‘R’) n'est pas un théorème (soit ¬□R(‘R’)), et c'est ça que je note G.

[Ajout par rapport au fil Twitter:] Dans la présentation informelle proposée par Hofstadter, G est en gros la phrase suivante : Si on prend le morceau de phrase suivant et qu'on le fait suivre (après deux points) de lui-même entre guillemets, on obtient quelque chose qui n'est pas un théorème : Si on prend le morceau de phrase suivant et qu'on le fait suivre (après deux points) de lui-même entre guillemets, on obtient quelque chose qui n'est pas un théorème — l'astuce est de dire cet énoncé n'est pas un théorème sans passer par une référence à cet énoncé que notre système formel ne permet pas de faire ; ceci est exactement parallèle au mécanisme des quines qui permettent de coder imprimer ce programme dans un langage qui ne permet pas une référence à ce programme.

Une fois qu'on a ces ingrédients, la preuve de Gödel est pure manipulation formelle. Tout le raisonnement est tenu dans le système externe (mais parle du système interne — voire interne² quand il y a des □□) :

Supposons □G. Alors □□G d'après (B). Mais la preuve (explicite !) de G⇒¬□G donne □(G⇒¬□G) d'après (A). Or □G et □(G⇒¬□G) donnent □¬□G par (C). Or □□G et □¬□G (i.e. □(□G⇒⊥)) donnent □⊥ d'après (C). Bref, on a prouvé □G⇒□⊥ soit ¬□⊥⇒¬□G. C'est-à-dire (qu'on a prouvé dans le système externe) que si le système interne prouve G alors il prouve ⊥ (c'est-à-dire 0=1). I.e., si le système interne est consistant (¬□⊥), il ne peut pas prouver G (soit : ¬□G)… …donc G, qui équivaut à ¬□G, est vrai ! (c'est-à-dire, est un théorème du système externe, prouvé sous l'hypothèse (¬□⊥) que le système interne est consistant. C'est le premier théorème d'incomplétude.

Maintenant, en appliquant (A) à cette preuve (explicite !) de ¬□⊥⇒¬□G, on obtient □(¬□⊥⇒¬□G) donc □¬□⊥⇒□¬□G (par (C)). Comme ¬□GG donne □(¬□GG) par (A) donc □¬□G⇒□G par (C), les implications □¬□⊥⇒□¬□G, □¬□G⇒□G et □G⇒□⊥ mises bout à bout donnent finalement □¬□⊥⇒□⊥, ou encore ¬□⊥⇒¬□¬□⊥. C'est le second théorème d'incomplétude : si le système (interne) est consistant (¬□⊥) alors il ne prouve pas la consistance du système interne² (¬□¬□⊥). Et comme en fait tous ces systèmes sont le même, le postulat épistémologique évoqué ci-dessus permet de lire ce théorème formel ¬□⊥⇒¬□¬□⊥ sous la forme si Peano est consistant, il ne prouve pas sa propre consistance (idem pour ZFC).

Évidemment, l'ultra-formaliste dont j'ai parlé plus haut objectera et dira que j'ai juste prouvé un énoncé cabalistique ¬□⊥⇒¬□¬□⊥ sans aucun sens dans le monde réel et qui ne dit rien sur mon système externe (lequel vit dans le monde réel). Mais si on croit que les « vrais » entiers naturels ont un sens et que Peano en dit des choses vraies, on est forcé de conclure que Peano est incomplet et ne sait pas prouver certaines choses vraies (essentiellement, il ne sait pas qu'il dit lui-même la vérité).

↑Entry #2590 [older| permalink|newer] / ↑Entrée #2590 [précédente| permalien|suivante] ↑

↓Entry #2587 [older| permalink|newer] / ↓Entrée #2587 [précédente| permalien|suivante] ↓

(lundi)

Un joli problème paradoxal de théorie de l'information

(Je n'ai pas de poisson d'avril à proposer cette année, mais je vous donne deux entrées pour le prix d'une.)

Même s'il a déjà été signalé hier en commentaire à cette vieille entrée, je ne peux pas ne pas écrire une petite entrée sur ce magnifique problème qui continue à narguer mon intuition — il s'agit d'un de ces cas où les mathématiques font quelque chose qui devrait être très sérieusement impossible, et j'ai beau arriver à prouver (et à comprendre la preuve intellectuellement) que c'est possible, je reste incapable de me faire intuitivement à l'idée que ça l'est. (Je sais, je sais : In mathematics you don't understand things. You just get used to them.)

Le problème est très simple (mais je l'écris de façon un peu longue pour qu'il n'y ait aucune ambiguïté sur les règles du jeu) :

Le cruel Docteur No a capturé deux mathématiciens, que nous appellerons Alice et Bob. Après avoir permis à ceux-ci de se concerter sur leur stratégie, il va les soumettre à son épreuve dont il leur communique les termes : chacun des deux pourra observer une suite binaire infinie aléatoire uniformément distribuée (c'est-à-dire une suite de 0 et 1 dont chaque terme vaut 0 ou 1 avec probabilité ½, indépendamment les uns des autres ; on peut imaginer une suite de résultats de tirages de pile ou face), les deux suites (celle qu'Alice observe et celle que Bob observe) étant indépendantes. Pour fixer les idées, mettons que les termes de chaque suite sont numérotés par les entiers naturels (0, 1, 2, 3, etc.) : appelons (An) la suite observée par Alice et (Bn) la suite observée par Bob. Alice et Bob ne peuvent pas communiquer entre eux (une fois finie la concertation initiale sur leur stratégie commune, mais celle-ci est antérieure à l'observation des suites). Alice, après avoir observé sa suite choisit un entier naturel a qui sera le numéro d'un terme dans la suite de Bob ; de même, Bob, après avoir observé sa suite, choisit un entier naturel b qui sera le numéro d'un terme dans la suite d'Alice. Alice et Bob gagnent (un gentil petit cadeau de la part du Docteur No) si chacun a choisi un terme valant 1 dans la suite de l'autre, c'est-à-dire, si la valeur Ab du terme de la suite d'Alice numéroté par l'entier b choisi par Bob et la valeur Ba du terme de la suite de Bob numéroté par l'entier a choisi par Alice valent tous les deux 1.

Quelle stratégie Alice et Bob peuvent-ils employer pour maximiser leur chance de gain ?

(Formellement : une stratégie d'Alice est une fonction borélienne α:{0,1}→ℕ et une stratégie de Bob est une fonction borélienne β:{0,1}→ℕ. La probabilité de succès est la mesure de l'ensemble des couples (A,B) ∈ {0,1} × {0,1} tels que A(β(B)) = B(α(A)) = 1, pour la mesure de probabilité uniforme sur {0,1} × {0,1} (les valeurs α(A) et β(B) sont les entiers noté a et b). Et on demande de maximiser cette probabilité.)

Remarque : Si on est gêné par les suites infinies (et c'est vrai que c'est agaçant de parler de l'observation d'une suite infinie), on peut ramener le problème à niveau fini : soit N un entier, Alice verra le résultat de N tirages de pile ou face uniformes et indépendant, Bob verra le résultat de N autres tels tirages (indépendants entre eux et indépendants de ceux d'Alice), et chacun devra choisir un entier entre 0 inclus et N exclu faisant référence à un tirage de l'autre, les deux gagnant s'ils ont choisi un tirage valant 1 chez l'autre. La situation paradoxale décrite ci-dessous est la même pour N fini que pour des suites infinies, c'est juste que les nombres sont moins ronds. (Formellement, dans le cas fini, une stratégie d'Alice est une fonction α:{0,1}N→{0,…,N−1} et une stratégie de Bob est une fonction β:{0,1}N→{0,…,N−1}. La probabilité de succès est la mesure de l'ensemble des couples (A,B) ∈ {0,1}N × {0,1}N tels que A(β(B)) = B(α(A)) = 1, pour la mesure de probabilité uniforme sur {0,1}N × {0,1}N.)

Une stratégie évidente consiste à ce qu'Alice et Bob choisissent tous les deux l'entier 0 (faisant référence au premier terme de la suite de l'autre). Dans ce cas, ils gagnent avec probabilité ¼, puisque chacun de A₀ et de B₀ vaut 1 avec probabilité ½, indépendamment l'un de l'autre. Choisir d'autres entiers constants (par exemple si Alice choisit 42 et Bob choisit 1729) donne toujours exactement la même chose.

L'idée intuitive qu'on a spontanément (en tout cas que j'ai eue, et dont je n'arrive toujours pas vraiment à me défaire) est qu'on ne peut pas faire mieux que ¼ :

Raisonnement incorrect : Alice et Bob ne peuvent pas communiquer, n'ont aucune information sur la suite de l'autre, et leurs deux suites sont indépendantes, donc il est impossible que l'observation de sa propre suite puisse aider Alice à faire quoi que ce soit d'utile sur la suite de Bob. Donc il n'y a rien de mieux à faire que de choisir des a et b constants (dont la valeur n'a pas d'importance).

Raisonnement incorrect (variante) : Quelle que soit la manière dont Alice choisit a, la valeur Ba vaudra 0 avec probabilité ½ et 1 avec probabilité ½ (puisque le choix de a ne peut dépendre que de A, qui est indépendant de B), et de même Ab vaudra 0 avec probabilité ½ et 1 avec probabilité ½. Puisque les suites A et B sont indépendantes et que (du coup) les variables a et b le sont, on a Ab = Ba = 1 avec probabilité ¼, et on ne peut pas faire mieux.

Et pourtant, c'est faux.

Et ce n'est même pas très compliqué de faire mieux que 1/4. Voici une stratégie simple qui donne une probabilité 1/3 de succès à Alice et Bob : Alice choisit pour a l'indice du premier 1 dans sa propre suite, et Bob choisit de même pour b l'indice du premier 1 dans la sienne. La probabilité d'avoir a=b est la somme de 1/4 (probabilité d'avoir a=b=0) plus 1/4² (probabilité d'avoir a=b=1) plus 1/4³ (probabilité d'avoir a=b=2), etc., c'est-à-dire la somme des 1/4k, qui vaut 1/3 ; et lorsque c'est le cas, par construction, Ab = Ba = 1 ; par ailleurs, si ab, alors Alice et Bob perdent (par exemple, si a<b, on a Ba = 0 puisque b est l'indice du premier 1 dans la suite B). Donc la probabilité de succès de cette stratégie est exactement 1/3.

J'ai beau avoir écrit cette preuve. Je n'arrive vraiment pas à me faire une idée intuitive de comment il est possible que cette stratégie fonctionne.

Mais je peux quand même dire ceci : la raison pour laquelle les raisonnements ci-dessus (tendant à « prouver » l'impossibilité) sont incorrects, c'est que s'il est bien vrai que chacun de Ab et Ba vaut 0 ou 1 avec probabilité ½, ils ne sont pas indépendants (puisque a dépend de A et b de B), et plus exactement, Alice et Bob peuvent s'arranger (et c'est ce qu'ils font dans la stratégie ci-dessus) pour que les deux événements Ab = 1 et Ba = 1 soient corrélés. Autrement dit, si on ne peut pas améliorer la chance d'avoir Ab = 1, on peut au moins s'arranger pour que, lorsque c'est le cas, ceci apporte des informations sur a ou sur B qui font que Ba = 1 a plus de chances de se produire. Je continue à trouver ça peu clair intuitivement, mais c'est déjà ça.

Maintenant, ce qui est amusant (et presque un peu décevant ?), c'est que cette jolie stratégie donnant 1/3 n'est toujours pas optimale : comme il est expliqué sur le fil MathOverflow lié au début de cette entrée (dans la réponse de mihaild), on peut faire 7/20 (c'est-à-dire 35%). Mais on ne sait pas si c'est optimum, et on n'a pas (au moment où j'écris, d'après de fil de discussion) de borne supérieure autre que le ½ évident.

Référence croisée : ce fil Twitter.

Mise à jour () : La borne supérieure a été améliorée à 3/8 dans le fil MathOverflow avec un argument très simple (quand j'aurai le temps, j'essaierai de mettre à jour ce paragraphe pour le donner). Par ailleurs, il apparaît que problème était déjà discuté (de façon un peu généralisée) dans ce papier, qui prouve une borne supérieure de 3/8, et annonce mais sans preuve une borne supérieure de 81/224.

Complément () : Pour la complétude de cette entrée, je reproduis en la paraphrasant la preuve de la borne supérieure par 3/8 de la probabilité de succès. Si on note a l'entier choisi par Alice et a′ l'entier qu'elle choisirait avec la même stratégie si elle observait la suite (1−An) au lieu de (An) (i.e., si on échange les 0 et 1 dans ce qu'Alice observe), et de même b et b′ pour Bob, alors on peut remarquer que l'espérance E(AbBa) de AbBa (qui est la probabilité de succès p qu'on cherche à maximiser) est aussi égale à l'espérance de (1−Ab) Ba (puisque 1−A est une variable distribuée comme A et toujours indépendante de B) ou de Ab (1−Ba) ou encore de (1−Ab) (1−Ba). La somme de ces quatre espérances (qui est 4p) est donc l'espérance de (Ab+1−Ab) (Ba+1−Ba), soit 4pE((Ab+1−Ab) (Ba+1−Ba)) = 1 + E(AbAb) + E(BaBa) + E((AbAb) (BaBa)) soit encore 1 + E((AbAb) (BaBa)) puisque E(Ab) = E(Ab) et E(Ba) = E(Ba) (en fait, chacune de ces quatre espérances vaut ½). Enfin, comme AbAb vaut +1 ou −1, l'espérance E((AbAb) (BaBa)) est majorée par E(|BaBa|), elle-même majorée par ½ (si ij alors E(|BiBj|)=½). Au final, on a prouvé 4p≤1+½, soit p≤3/8 comme annoncé.

↑Entry #2587 [older| permalink|newer] / ↑Entrée #2587 [précédente| permalien|suivante] ↑

↓Entry #2578 [older| permalink|newer] / ↓Entrée #2578 [précédente| permalien|suivante] ↓

(dimanche)

Et maintenant, un peu de logique linéaire

Je traîne depuis longtemps l'idée de vulgariser quelques notions de logique linéaire. Du point de vue de la vulgarisation, la logique linéaire a ceci de sympathique que c'est quelque chose mathématiquement à tellement « bas niveau » que je n'ai besoin de présupposer aucune sorte de connaissance mathématique préalable pour en parler : en principe, on peut la considérer comme un pur petit jeu syntactique dont les règles ne sont pas très compliquées — même si, présenté sous cette forme, il risque de ne pas apparaître comme très intéressant, et même s'il est bon d'avoir du recul pour avoir une idée de quelles règles appliquer à quel moment, la compréhension des règles elles-mêmes ne nécessite pas de savoir particulier. Du point de vue personnel, la logique linéaire est quelque chose qui me frustre beaucoup parce que, d'un côté, je la trouve extrêmement élégante et joliment symétrique, de l'autre, à chaque fois qu'elle semble avoir une application ou une interprétation quelque part, on se rend compte qu'il y a une note en bas de page qui fait que ce n'est pas vraiment la logique linéaire (il y a par exemple un axiome en plus, ou un connecteur en moins, ou seulement un fragment du système, ou quelque autre variation), et l'élégance est rompue ; et aussi, pour cette raison, l'intuition qu'on peut se former est brouillée.

De quoi s'agit-il ? D'un système formel inventé par le logicien français Jean-Yves Girard en 1987. J'avoue ne guère avoir d'idée de ce qu'il voulait faire avec, parce que les textes de Girard sont… un peu inhabituels… bourrés de mots qu'il ne définit pas, de références cryptiques, et de blagues dont on se demande si ce sont des blagues (comme l'intervention insistante du brocoli dans beaucoup de ses papiers). Mais depuis, elle a trouvé diverses applications et connexions : en logique, en informatique théorique ou plus appliquée, en algèbre et théorie des catégories, en théorie des jeux et même en physique quantique (sauf qu'à chaque fois, comme je le dis ci-dessus, il y a quelque chose en plus ou en moins) ; mais je ne compte pas essayer de décrire ces applications et connexions, qui sont pourtant sans doute ce qu'il y a de plus intéressant dans l'histoire, parce que je n'ai pas l'espace ni le temps pour ça.

Bref. Avant d'expliquer quelles sont les règles du jeu, il faut que j'essaye de donner une idée de ce dont il s'agit (en agitant les mains). On parle de logique linéaire, et il s'agit effectivement d'une généralisation de la logique classique, mais ce terme risque de donner une impression tout à fait fausse, et on devrait peut-être plutôt s'imaginer que ça s'appelle formalisme d'échanges ou synallagologie universelle ou quelque chose de ce genre (le seul problème du mot synallagologie est que personne ne sait ce qu'il veut dire puisque je viens de l'inventer… mais à part ça, il est parfait). La différence essentielle est la suivante : en logique usuelle, si on fait un raisonnement tendant à démontrer une conclusion X à partir d'hypothèses A, B et C, disons, on peut utiliser librement A, B et C dans le cours du raisonnement, chacune aussi souvent qu'on veut (on peut aussi, d'ailleurs, ne pas du tout utiliser une hypothèse) ; la logique linéaire, pour sa part, exige que chacune des « hypothèses » (qu'il vaut mieux, du coup, ne pas considérer comme des hypothèses) soit utilisée une et une seule fois : on ne peut ni les multiplier ni les faire disparaître (évidemment, il y aura des moyens de marquer des hypothèses spéciales qu'on peut multiplier et/ou faire disparaître, mais ce n'est pas le cas par défaut) ; dans ces conditions, il vaut mieux, donc, considérer qu'on n'a pas du tout à faire à une logique, à des raisonnements et à des hypothèses et conclusions, mais à des échanges (gestion de ressources abstraites, transactions économiques, réactions chimiques, que sais-je encore) qui ont des entrées (réactifs) et des sorties (produits), ou quelque chose comme ça. Par exemple, la logique linéaire pourrait concevablement servir à formaliser des contrats financiers (j'avais déjà évoqué quelque chose de ce genre), mais il ne faut pas s'imaginer que la logique linéaire elle-même dira grand-chose d'intéressant : de même que la logique classique ne fournit que le langage le plus basique au-dessus duquel on peut bâtir des raisonnements (il faut ajouter des axiomes intéressants pour obtenir quelque chose d'intéressant), la logique linéaire n'est qu'un cadre, en lui-même extrêmement primitif pour possiblement décrire des échanges.

↑Entry #2578 [older| permalink|newer] / ↑Entrée #2578 [précédente| permalien|suivante] ↑

↓Entry #2568 [older| permalink|newer] / ↓Entrée #2568 [précédente| permalien|suivante] ↓

(jeudi)

Le retour du cruel Docteur No

Le Docteur No est de retour ! Il était plutôt occupé ces derniers temps à résoudre des problèmes informatiques, mais le voilà revenu et qui s'adonne à son passe-temps favori qui consiste à capturer des mathématiciens pour les soumettre à des énigmes idiotes (voir ici pour des épisodes précédents) :

Le cruel Docteur No a capturé 100 mathématiciens pour les soumettre à une épreuve démoniaque. Après avoir permis aux mathématiciens de se concerter initialement, il va placer sur la tête de chacun d'entre eux un chapeau portant un nombre entier entre 1 et 100 (inclus) de façon que chacun puisse voir le nombre porté par les chapeaux de tous les autres mais pas le sien. Les mathématiciens n'ont aucune information sur la manière dont les numéros seront attribués et il peut parfaitement y avoir des répétitions. Les mathématiciens n'auront plus le droit de communiquer à partir du moment où la distribution des chapeaux commence. Chacun devra émettre un avis sur le numéro qu'il pense que son propre chapeau porte : ces avis seront émis par pli secret et connus du seul Docteur No (i.e., les mathématiciens ne connaissent pas les réponses fournis les les autres). Le Docteur No est plutôt clément aujourd'hui : il libérera les mathématiciens si au moins l'un d'entre eux a fourni une réponse correcte (i.e., deviné le numéro que porte son chapeau) ; dans le cas contraire, il tuera tous les mathématiciens avec ses tortures particulièrement raffinées.

Les mathématiciens pourraient évidemment tous répondre au hasard (auquel cas ils auraient 63% de chances d'être libérés ; je laisse ça aussi en exercice mais ça n'a pas vraiment de rapport avec le problème). Mais en se concertant, ils peuvent s'arranger pour être certains d'être libérés : comment font-ils ?

La solution est simple, mais on peut perdre beaucoup de temps en cherchant dans la mauvaise direction. Je tire ça d'ici (la réponse est indiquée en rot13 dans un commentaire en-dessous) ; ce fil contient d'ailleurs un certain nombre d'autres devinettes rigolotes.

Sinon, voici un autre problème (pas vraiment une énigme), qui n'a absolument aucun rapport avec celui qui précède si ce n'est qu'il est venu à ma connaissance autour du même moment (il est inspiré de cette question mais c'est une variante assez différente du même genre d'idées) :

On considère n+1 objets et deux joueurs (Alice et Bob). Chacun des deux joueurs a un ordre de préférence (strict) sur les objets, et ces ordres de préférence sont connus de l'un comme de l'autre. Ils vont jouer au jeu suivant : chacun, tour à tour, va éliminer un objet, jusqu'à ce qu'il n'en reste plus qu'un (après n tours, donc). Ce dernier objet est gagné par les deux joueurs (i.e., ils se le partagent, ou si on préfère on peut dire qu'il y en a deux copies et que chacun en reçoit une, bref, chacun cherche à maximiser la valeur qu'il accorde à l'objet restant). Le jeu est à information parfaite (les deux joueurs savent tout : ce que l'autre joueur veut, et ce qu'il fait). Quelle stratégie vont-ils appliquer (en fonction des deux ordres de préférence) ? Et comment peut-on prédire efficacement l'objet final ?

On peut le formaliser plus précisément ainsi : soient 1,…,n les objets, triés dans l'ordre de préférence d'Alice (du moins préféré au plus préféré), et soient σ(1),…,σ(n) les valeurs de préférence de Bob pour les objets dans cet ordre, où σ est une permutation de {1,…,n} (c'est-à-dire qu'il aime le moins l'objet σ−1(1) et qu'il préfère σ−1(n)). Alice va jouer la stratégie qui cherche à maximiser la valeur i du dernier objet restant, tandis que Bob va jouer la stratégie qui cherche à maximiser σ(i) : on demande comment calculer i en fonction de σ et comment Alice doit calculer son premier coup. (On peut évidemment procéder de façon inductive : chaque coup possible d'Alice se ramène à un jeu de même nature avec un objet de moins et les rôles des joueurs échangés, mais ce que je demande c'est si on peut faire mieux ou plus simple.)

À titre d'exemple, si n=2 et que les préférences d'Alice sont 1<2<3, selon les valeurs (σ(1),σ(2),σ(3)) des préférences de Bob pour ces trois objets l'objet choisi est 3 (le préféré d'Alice) dans tous les cas sauf si (σ(1),σ(2),σ(3)) vaut (2,3,1) ou (3,2,1) (i.e., si l'objet préféré d'Alice est celui que Bob aime le moins), auquel cas l'objet choisi est 2 (le deuxième préféré d'Alice). C'est assez intuitif.

Il se peut que la réponse soit très facile : je n'ai pas pris le temps d'y réfléchir (trop occupé que j'étais à coudre des numéros sur des chapeaux).

On pourrait aussi demander ce qui se passe si l'un des joueurs joue la stratégie « gloutonne » (consistant à éliminer à chaque coup son objet le moins préféré), selon que l'autre joueur le sait ou selon que l'autre joueur croit toujours qu'il jouera désormais de façon rationnelle. On pourrait aussi jouer à changer l'alternance des coups entre Alice et Bob (plutôt que de les faire alterner mécaniquement) et chercher l'ordre le « plus équitable » dans un sens qu'il faudrait formaliser. Bon, bref, je trouve l'idée générale du jeu intéressante, mais je ne sais pas quelle est la bonne question à poser (c'est peut-être ça la question, en fait : trouver la question la plus intéressante à poser sur ce jeu).

↑Entry #2568 [older| permalink|newer] / ↑Entrée #2568 [précédente| permalien|suivante] ↑

↓Entry #2562 [older| permalink|newer] / ↓Entrée #2562 [précédente| permalien|suivante] ↓

(mercredi)

Sur la rédaction des maths et la recherche de l'inambiguïté

L'an dernier, j'ai eu l'honneur de déjeuner avec Jean-Pierre Serre, et nous avons discuté entre autres de la rédaction des mathématiques. (Comme Serre est à mon avis — et je suis loin d'être le seul à le penser — un des mathématiciens dont le style de rédaction est le plus parfait qui soit, c'était évidemment très intéressant pour moi d'entendre ce qu'il avait à dire. Je recommande d'ailleurs de regarder cet exposé où il dénonce beaucoup de mauvaises habitudes dans ce domaine.)

Il a beaucoup insisté sur l'importance d'écrire des énoncés justes : c'est-à-dire notamment, si un énoncé P(n) est vrai pour tout n≥1, de bien écrire pour tout n≥1 et de ne pas laisser le lecteur penser que P(0) puisse être vrai lorsqu'il ne l'est pas (et c'est encore pire quand l'énoncé commence à être vrai à 2 ou 3, voire au-delà). Je savais déjà qu'il accordait beaucoup d'importance à ça[#]. Mais comme je mentionnais les codes correcteurs d'erreurs, il a fait cette autre remarque que je trouve tout à fait digne d'être érigée en maxime, à savoir qu'il essayait d'écrire les mathématiques comme un code correcteur d'erreurs (je n'ai malheureusement pas noté la phrase exact qu'il a employée, mais ça fait peut-être justement partie du phénomène souligné). Ce qu'il voulait dire est qu'inévitablement, dans une rédaction mathématiques, il y aura des choses qui seront mal lues : soit que l'auteur lui-même fasse un lapsus, soit que le manuscrit soit mal retapé, soit que l'imprimeur change certains symboles, soit que le lecteur lise mal ou ait une convention différente sur certaines choses : il faut essayer d'écrire de manière à rendre le texte relativement robuste par rapport à ces erreurs (pour qu'elles soient détectables ou, encore mieux, corrigeables).

[#] Plusieurs fois j'ai assisté à un séminaire où Serre était dans l'assistance[#2], où l'orateur commence énonce un théorème et où Serre s'exclame mais c'est complètement faux ! ce n'est pas possible, ça ! — alors là, l'orateur, visiblement paniqué, se demande si Serre vient de trouver en direct un contre-exemple au théorème principal, et au bout d'un moment de confusion on comprend que Serre protestait contre le fait que l'énoncé était trivialement faux pour n=0. Moment sans doute très désagréable pour l'orateur, mais je pense qu'après ça on apprend très vite à se demander si pour tout n veut vraiment dire tout n.

[#2] Tiens, puisque j'en suis à raconter des anecdotes à son sujet[#3], un jour j'ai assisté à un séminaire où l'orateur a commencé à parler du groupe de Serre comme si tout le monde savait évidemment de quoi il s'agissait (je sais que c'était dans un contexte de représentations galoisiennes, mais moi-même je n'avais aucune idée de ce que c'était censé être). L'éponyme a levé la main pour demander qu'est-ce que le groupe de Serre ?. La morale, là, et je pense aussi que l'orateur l'aura bien retenue, c'est que même quand on parle à une assistance de gens très distingués, il faut être très conservateur dans ce qu'on suppose que tout le monde connaît.

[#3] Allez, une troisième pour la route. Quand j'ai écrit cet article avec mon directeur de thèse, ce dernier l'a envoyé à Serre pour lui demander son avis avant publication. Entre autres remarques, il a relevé le bout de phrase par récurrence sur le naturel k et a commenté ce n'est sûrement pas vous [Colliot-Thélène] qui avez écrit ça : de fait, c'est moi qui l'avais rédigé ce passage. (L'objection est que Serre n'aime pas qu'on écrive un naturel pour un entier naturel. Je vous rassure, ses autres remarques sur l'article étaient beaucoup plus intéressantes.) Colliot-Thélène a regardé dans le petit Robert, qui recense bien quelque part naturel comme nom masculin dans le sens de entier naturel, et nous avons décidé de laisser la phrase comme ça. Mais depuis, je fais plus attention à écrire généralement un entier naturel plutôt que juste un naturel (sauf quand j'ai l'impression que le second allège vraiment la phrase).

↑Entry #2562 [older| permalink|newer] / ↑Entrée #2562 [précédente| permalien|suivante] ↑

↓Entry #2557 [older| permalink|newer] / ↓Entrée #2557 [précédente| permalien|suivante] ↓

(dimanche)

Une méditation sur le nombre 24 et la causalité en mathématiques

Dans cette entrée, je vais commencer par parler de maths, mais ensuite je veux me servir de ce que j'aurai raconté pour soulever une question de philosophie (ou peut-être, de psychologie) des maths. Ces deux parties n'ont pas vraiment de rapport sauf que la première sert d'illustration pour la seconde : on doit pouvoir sauter la première partie (ou la lire en diagonale) et quand même comprendre quelque chose à la seconde, enfin, j'espère. (Mais bon, je ne suis pas content de ce que j'ai écrit dans la seconde partie, donc ça n'a peut-être pas d'intérêt.)

*

Je racontais récemment que le nombre 24 était particulièrement magique à cause de l'existence de certains objets exceptionnels, notamment le réseau de Leech en dimension 24 (défini dans l'entrée en question). Maintenant, considérons le fait suivant (problème des boulets de canon, conjecturé par Édouard Lucas autour de 1875 et démontré par George Watson en 1918) :

L'équation 0² + 1² + ⋯ + n² = m² a exactement trois solutions, à savoir quand (n,m) vaut (0,0), (1,1) ou (24,70). Autrement dit, à part les deux cas triviaux (0²=0² et 0²+1²=1²), la seule situation où la somme des carrés des premiers entiers naturels est encore un carré est donnée par 0² + 1² + ⋯ + 24² = 70².

(La somme 0²+1²+⋯+n² vaut encore n·(n+1)·(2n+1)/6, mais si on écrit l'équation comme n·(n+1)·(2n+1) = 6m², on ne voit pas vraiment pourquoi elle est intéressante.)

Ce n'est pas très facile à montrer, mais ce n'est pas ça qui m'intéresse.

On pourrait dire que le fait que la somme des carrés des entiers naturels jusqu'à 24 est un carré (et qu'à part les cas triviaux c'est le seul) est une propriété remarquable du nombre 24. Pas franchement passionnante, mais bon. Mais a priori, on se dit que cette propriété n'a aucun rapport particulier avec les propriétés magiques du nombre 24 que j'ai évoquées dans mon autre entrée.

Sauf qu'en fait, si.

↑Entry #2557 [older| permalink|newer] / ↑Entrée #2557 [précédente| permalien|suivante] ↑

↓Entry #2555 [older| permalink|newer] / ↓Entrée #2555 [précédente| permalien|suivante] ↓

(jeudi)

Les trois magiciens du nombre 24 : le code de Golay, le réseau de Leech et le module de Moonshine

S'il y a un nombre magique en mathématiques, c'est bien 24. (Je pense que Douglas Adams a juste inversé les chiffres.) Le nombre 8 vient presque à égalité, et 12, peut-être 6 et 16 ont aussi quelques propriétés magiques (qui, globalement, sont toujours liées à celles de 24), mais celui qui est vraiment farabuleux (pardonnez le néologisme), c'est 24.

Je voudrais dans cette entrée essayer de témoigner de la magie de 24 en définissant deux et en évoquant le troisième de trois objets exceptionnels qui font que 24 est si spécial. On pourrait aussi les appeler les trois générations de « magiciens » qui tirent leur pouvoir magique du nombre 24. Ces objets sont : le code de Golay binaire (première génération), le réseau de Leech (deuxième génération), et le module de Moonshine (troisième génération). Mon but est donc d'en parler un peu, en définissant proprement les deux premières générations, en essayant que ce que je dis sur la première soit accessible à un très large public, et en disant quelques mots de la troisième. Ou du moins, mon but était tout ça, parce que je me suis pas mal embourbé et je ne suis pas du tout content de ce que j'ai écrit : je donne certes une définition du code de Golay binaire et du réseau de Leech, mais je crois ne pas avoir du tout réussi à passer l'idée de pourquoi ils sont intéressants au fond. Et comme souvent, je crois que je me retrouve à présupposer de mon lecteur un niveau de connaissances mathématiques préalables qui varie de façon assez incohérente d'un endroit à l'autre (au début je m'efforce vraiment de ne rien supposer, et à la fin, il sera certainement nécessaire d'avoir au moins une intuition de ce qu'est un groupe). Néanmoins, maintenant que tout ça est écrit, je ne vais pas ne pas le publier, donc prenez-le pour ce que ça vaut.

Comme par ailleurs, le nombre 8 est aussi magique (quoiqu'un peu moins que 24), je peux aussi parler de deux des trois[#] générations de magiciens qui tirent leur pouvoir magique de celui-ci : le code de Hamming de longueur 8 et le réseau E₈, parce qu'ils sont utiles pour approcher leurs analogues du nombre 24.

[#] Je crois que le troisième qui complète la série serait l'algèbre d'opérateurs de sommets dont Griess parle dans son article A vertex operator algebra related to E₈ with automorphism group O⁺(10,2), mais je ne comprends décidément pas bien tout ça.

Bref, le tableau à garder en tête (juste pour le plan : je vais expliquer ce que tout ça veut dire) est quelque chose comme :

Nombre magique1re génération2e génération3e génération
24Code de Golay binaireRéseau de LeechModule de Moonshine
8Code de Hamming de longueur 8Réseau E₈[Voir note #]

Le terme de génération évoque l'idée que les objets de la deuxième génération se définissent en termes de ceux de la première, et ceux de la troisième en termes de ceux de la deuxième, et qui plus est, il y a une certaine similarité entre la manière dont ces objets s'enfantent les uns les autres (je ne prétends pas que c'est rigoureusement la même, ni entre les colonnes, ni entre les lignes : notemment, il n'y a pas de « foncteur » dans l'affaire, juste une certaine analogie).

Je vais aussi évoquer, à chaque fois, les groupes de symétrie de ces différents objets, qui ressembleront à la numérologie suivante (là aussi, je dois expliquer ce que sont ces machins, mais à chaque fois, je donne le nom du groupe de symétrie et son ordre, c'est-à-dire le nombre de symétries) :

Nombre magique1re génération2e génération3e génération
24
M24
244 823 040
Co₀
8 315 553 613 086 720 000
Monstre (F₁)
808 017 424 794 512 875 886 459 904 961 710 757 005 754 368 000 000 000
8
C₂³⋊PSL(3,2)
1 344
W(E₈)
696 729 600
O(10,2,+) [???]
46 998 591 897 600

Plan de la suite :

↑Entry #2555 [older| permalink|newer] / ↑Entrée #2555 [précédente| permalien|suivante] ↑

↓Entry #2551 [older| permalink|newer] / ↓Entrée #2551 [précédente| permalien|suivante] ↓

(jeudi)

Le corps à un élément, et autres licornes mathématiques

Les chasseurs-prouveurs se rassemblaient comme chaque soir autour de l'équation de la chaleur et se racontaient les histoires de leurs aventures. Joueur-Atlas, qui était célèbre pour avoir autrefois attrapé un groupe parfait à 8 315 553 613 086 720 000 éléments évoqua le fils de « son » groupe, dont il avait aperçu la silhouette monstrueuse, à la lumière de la lune, en train de remuer près du nombre 196 883, et qu'il espérait voir un jour capturé. Mais ce soir, c'était au tour du vieux Bâtisseur-Alternatif de prendre la parole.

— Un jour, j'ai vu un corps comme je n'en avais jamais vu auparavant.

Il désigna une figure rupestre qu'il avait exécutée il y a longtemps, à la craie sur le tableau noir du Hilbertraum : un F pas tout à fait gras finissant par un 1 plutôt bas. Et il conclut théâtralement :

— Figurez-vous que ce corps n'avait qu'un seul élément.

Certains soupiraient d'entendre Bâtisseur-Atlernatif raconter toujours la même histoire à dormir debout, mais les jeunes chasseurs-prouveurs étaient fascinés :

— Un corps à un seul élément ? Mais ce n'est pas possible, grand-père !

— Pourtant je l'ai bien vu. Et attendez, ce n'est pas le plus incroyable… il était… sous l'anneau des entiers !

Cette révélation fit place à un silence choqué de la part de ceux qui n'avaient pas encore entendu cette légende. Un corps caché sous l'anneau des entiers ! Cela semblait si impossible — et en même temps si prometteur !

Bon, trêve d'humour à 1/1728 zorkmids.

Ce que j'appelle licorne mathématique, c'est un objet mathématique dont on aimerait croire à l'existence, un objet dont on a une certaine intuition et même des indices suggérant sa présence, qui, naïvement envisagé tel quel, n'existe pas, n'est pas possible, conduit à des paradoxes et des contradictions. On peut démontrer qu'il n'existe pas, que les propriétés qu'on lui attribue sont impossibles, et pourtant, on cherche quand même un moyen de le faire exister.

Ce qui fait que les licornes sont des licornes, c'est qu'on n'a pas trouvé la bonne définition ou la bonne théorie-cadre. Chasser la licorne, c'est donc chasser la définition ou la théorie qui lui permettra d'exister et de faire disparaître les paradoxes. Cela peut sembler bizarre : si on s'imagine qu'on donne naissance à un objet mathématique en le définissant, comment peut-il y avoir des objets qu'on poursuive sans parvenir à les définir ? Pourtant, cela se produit assez souvent (et je prends même ça pour un indice — certes pas terriblement concluant — dans le sens que les mathématiques existent indépendamment de l'homme).

*

L'exemple le plus simple est sans doute celui des nombres complexes. La manière dont je vais l'évoquer prend des libertés avec l'Histoire, qu'on m'en pardonne, mais mon but n'est past de raconter l'histoire des maths mais d'expliquer le concept d'une licorne. La racine carrée de −1, donc, était une licorne : un nombre qui, multiplié par lui-même, donne −1, c'est impossible a priori. Et on a une preuve de cette impossibilité : à savoir, que x soit positif ou négatif, son carré x² = x·x est forcément positif, donc ne peut jamais valoir −1. Bref, √(−1) est une licorne. Pourtant, quelqu'un prétend avoir vu des traces de la licorne : si on fait comme si elle existait, si on oublie cette impossibilité, si on mène les calculs comme si la racine carrée des nombres négatifs avait un sens, on arrive à résoudre des équations du troisième degré qu'on ne savait pas résoudre autrement (celles qui ont trois racines). Comment expliquer que quelque chose d'impossible conduise à une conclusion heureuse ? C'est cela qui fait soupçonner que la licorne existe vraiment, et qui donne envie de la capturer.

Maintenant on ne voit plus du tout que cette histoire a été une licorne : maintenant, √(−1) est un nombre complexe, quelque chose de tellement banal qu'on en oublie trop facilement que cela a pu représenter un paradoxe, une licorne. Pourtant, pour capturer cette licorne, il a fallu faire un saut conceptuel : abandonner l'idée que les nombres soient ordonnés, c'est un saut conceptuel gigantesque (les nombres ont été faits pour être ordonnés, pourrait-on dire ; les opérations algébriques sont une sophistication ajoutée sur le concept de comparaison). Mais une fois fait le saut conceptuel, une fois définie la notion de nombre complexe, la licorne est capturée, elle perd tout son mystère, on s'aperçoit que la définition antérieure de nombre était restrictive (ce qui ne signifie pas qu'elle n'ait pas de valeur !, il n'est pas question de remplacer systématiquement les nombres réels par des nombres complexes en mathématiques ou ailleurs).

Ce qui m'intéresse dans cette histoire, c'est la démarche où d'abord on aperçoit des traces de pas qui semblent paradoxales (cette bestiole marche comme un cheval, pourtant elle semble avoir une corne !), on traque le concept, et on finit par capturer la licorne, c'est-à-dire résoudre le paradoxe, rendre possible ce qu'on avait démontré impossible, en contournant l'impossibilité par une définition élargie. La licorne se capture par la définition. C'est inhabituel par rapport à la pratique générale des mathématiques qui consiste à chasser les preuves, pas les définitions (ni les licornes).

Méta : Dans la suite, je vais évoquer quelques autres licornes. Ne sachant pas à quel niveau de vulgarisation me placer, je n'ai pas vraiment pris de décision cohérente à ce sujet, et je suppose donc de la part de mon lecteur des connaissances variables de paragraphe en paragraphe : j'espère néanmoins avoir fait en sorte qu'on puisse comprendre un petit peu l'idée générale même si on ne comprend pas tel ou tel passage. D'autre part, comme mon but était de raconter une histoire plus que d'exposer des maths, il se peut que je dise des choses un peu abusées ici ou là (j'espère quand même avoir toujours été assez vague pour qu'on ne puisse pas m'accuser d'avoir écrit un énoncé indiscutablement faux, mais si c'est le cas, je mettrai la faute sur les licornes qui m'ont poussé).

↑Entry #2551 [older| permalink|newer] / ↑Entrée #2551 [précédente| permalien|suivante] ↑

↓Entry #2549 [older| permalink|newer] / ↓Entrée #2549 [précédente| permalien|suivante] ↓

(samedi)

Quelques conseils pour les étudiants en maths

À l'approche de la rentrée, je me dis qu'il peut être utile que je publie quelques conseils pour les étudiants en maths. Ceux-ci sont inspirés à la fois de ce que j'ai écrit dans ce fil Twitter et de ce que j'ai expliqué de vive voix à un élève de prépa qui me demandait de tels conseils : ayant ainsi un peu réfléchi à ce que j'avais à dire, autant le mettre sur ce blog.

Il s'agit là de conseils généraux (et sans doute d'une bonne dose de proverbial enfonçage de portes ouvertes à ma fidèle hache bénie +2 trempée dans la potion de banalités), s'adressant plutôt à des étudiants entre approximativement ce qui correspond, dans le système éducatif français, aux niveaux bac à bac+5 (disons) : grosso modo, avant ça, on ne fait pas tellement de maths au sens « raisonnement déductif » (ayant la démonstration comme méthode essentielle) ; et après, si vous en êtes arrivé là, vous avez assez de familiarité avec les mathématiques pour ne pas avoir besoin de mes conseils. Certaines des choses que je vais dire s'appliquent à d'autres disciplines adjacentes, comme la physique ou l'informatique (pour ce qui est de l'informatique théorique, mon avis est qu'il s'agit de toute façon d'une branche des mathématiques, même si elle ne s'assume pas toujours comme telle) ; quelques uns s'appliquent sans doute à n'importe quelle discipline, mais je me focalise quand même sur les maths.

On doit pouvoir tirer de ces conseils aux étudiants quelques conseils pour les enseignants (en appliquant la dualité étudiant-enseignant et le foncteur de réduction des platitudes), mais comme je n'aime pas donner des leçons à ce sujet, je vais laisser ça en exercice au lecteur.

✱ Conseil nº1 : aimer ce que l'on fait. C'est peut-être un peu idiot de dire ça, mais je suis persuadé qu'on ne peut correctement faire des maths que si on les trouve un minimum belles et intéressantes. Si on les conçoit comme une corvée, elles le resteront. Si on les conçoit comme (la métaphore que j'aime bien utiliser) l'exploration d'un palais magnifique et incompréhensiblement gigantesque, à la structure à la fois labyrinthique et élégante, on peut arriver à comprendre que ce soit à la fois excitant et séduisant, et en tirer la motivation nécessaire à leur étude.

Je ne peux évidemment pas donner de recette magique pour comprendre que les maths sont belles. C'est quelque chose que j'essaie de communiquer, mais il est évident que je ne vais pas transformer tout le monde en matheux. Mais, même si on a un a priori négatif (et certaines formes d'enseignement des mathématiques laissent hélas place à bien peu d'autre que la corvée rébarbative), il est au moins essentiel de garder l'esprit ouvert à cette possibilité, que les maths puissent être fascinantes. Je pense qu'il est au moins utile, même si on est réfractaire, de chercher les sous-domaines sur lesquels on accroche un peu plus, et de peut-être chercher à se renseigner sur l'allure générale du paysage mathématique, méditer sur la question de pourquoi certaines personnes y trouvent goût (est-ce qu'on a reçu une image déformée par un enseignement rébarbatif ou est-ce qu'on est véritablement hostile aux mathématiques ? dans ce dernier cas, il vaut certainement mieux arrêter de les étudier le plus rapidement possible et ne pas céder aux sirènes qui promettent une meilleure carrière ou quelque chose de ce genre). L'histoire des sciences peut aussi être une passerelle vers un intérêt pour les mathématiques elles-mêmes.

✱ Conseil nº1b : faire preuve de curiosité intellectuelle, et questionner ce que l'on fait. Apprendre le cours pour le cours est la meilleure garantie d'en rester là. Pour comprendre un cours de maths, il faut plutôt le questionner[#], le décortiquer, essayer de prendre du recul. Pour ça, le mieux est de garder à l'esprit toutes sortes de questions (pourquoi fait-on ça ?, où veut-on en venir ?, comment fonctionne cet objet ?) ; je vais donner des exemples plus précis de telles questions (à se poser à soi-même ou à poser à l'enseignant) dans les conseils suivants, mais le message plus général est que tout questionnement est bienvenu (voir aussi les conseils nº6 et 6b ci-dessous).

[#] Dans un cours de langue, si un étudiant demande pourquoi 95 en français de France se dit-il quatre-vingt-quinze ?, on ne peut pas vraiment lui donner de réponse sauf des choses comme c'est comme ça ou c'est un accident historique, peut-être accompagnées d'une histoire du phénomène (mais c'est déjà empiéter des langues sur la linguistique, et ça n'aidera pas tellement à l'apprentissage du français). L'enseignant en maths, lui, doit être prêt à se justifier de plus près que ça.

↑Entry #2549 [older| permalink|newer] / ↑Entrée #2549 [précédente| permalien|suivante] ↑

↓Entry #2548 [older| permalink|newer] / ↓Entrée #2548 [précédente| permalien|suivante] ↓

(jeudi)

Des figures que j'en ai marre de refaire, et des histoires de kaléidoscopes

[Dessins des systèmes de racines de rang 2]Il y a des des figures que je me retrouve à refaire encore et toujours, à chaque fois que je veux réfléchir à un certain sujet. Parmi ceux que je reproduis avec une fréquence qui finit par devenir vraiment pénible, il y a ceux qui apparaissent ci-contre à droite, et que je me suis enfin de sorti les doigts du c** pour produire en PDF avec TikZ (suivez le lien pour le PDF). Comme je ne suis certainement pas le seul trouver ces figures utiles pour réfléchir, je les mets en ligne. Et du coup, je peux en profiter pour faire un peu de vulgarisation sur ce qu'ils représentent.

Je vais essayer d'expliquer ça sous l'angle de la géométrie euclidienne élémentaire, à travers la question de classifier et de comprendre les kaléidoscopes (simpliciaux). L'intérêt, outre que c'est peut-être plus parlant, est ne pas supposer que qui que ce soit ait lu mon récent rant interminable sur les groupes de Lie (mais en même temps, essayer de dire les choses de manière à quand même éclairer le rant en question). En fait, après coup, je ne suis rendu compte que ce n'était pas forcément une très bonne approche, et que cette entrée ressemble beaucoup à une accumulation de faits qui partent dans tous les sens et qui ne reflètent pas bien (pun unintended) l'élégance du sujet. En plus de ça, comme c'est un sujet que j'ai l'habitude de voir abordé autrement que comme de la géométrie euclidienne, je ne suis pas très sûr de l'ordre dans lequel les faits s'agencent logiquement, et je n'ai pas toujours une idée très claire de la difficulté qu'il y aurait à les démontrer dans une telle approche. Et aussi à cause de ça, il faut que j'avertisse que je n'ai pas vérifié très soigneusement (je veux dire, encore moins que d'habitude…) tous les résultats que j'énonce dans cette entrée, et qu'il est fort possible que j'aie oublié une hypothèse ou une autre pour me raccrocher à là où je veux en venir ; notamment, j'ai failli complètement négliger la « condition supplémentaire » que j'ai finalement trouvé utile d'introduire plus bas dans la définition d'un kaléidoscope. Malgré tout ça, j'espère que ce que je raconte est au moins un peu intéressant.

↑Entry #2548 [older| permalink|newer] / ↑Entrée #2548 [précédente| permalien|suivante] ↑

↓Entry #2546 [older| permalink|newer] / ↓Entrée #2546 [précédente| permalien|suivante] ↓

(mercredi)

Quelques points de vue (de matheux) sur les grandeurs physiques et unités de mesure

Dans cette entrée, je voudrais évoquer la question des grandeurs physiques (longueur, durée, vitesse, masse, courant électrique…) et des unités de ces grandeurs. Je vais jeter un regard de matheux sur ce que ces choses sont, proposer quelques points de vue ou (esquisses de) définitions formelles possibles, et m'interroger sur l'utilité et la pertinence de ces points de vue, notamment pédagogiques, mais aussi du point de vue de la question de l'incertitude des mesures.

Je précise que cette entrée part un peu dans tous les sens, parce que j'ai commencé par écrire de la façon dont les idées me venaient (ou me revenaient, parce que ce sont des idées que je rumine depuis longtemps), et j'ai voulu raconter trop de choses à la fois, donc il y a plein de digressions. En plus de ça, j'ai un peu permuté les bouts que j'avais écrits (il en reste certainement des incohérences comme des je vais y revenir alors que les choses sont dans un autre ordre), puis repermuté, puis re-repermuté au fur et à mesure que j'ajoutais des digressions, et finalement je ne sais plus du tout dans quel ordre je dis les choses. Heureusement, il n'y a pas trop de lien logique clair ni de dépendance entre les différents morceaux ce que je raconte, donc on doit pouvoir lire cette entrée dans le désordre puisque c'est comme ça qu'elle a été écrite ! J'ai essayé de marquer par des triples accolades {{{…}}} (cf. ici) les digressions les plus identifiables, dans l'espoir que ça aide à s'y retrouver un peu.

À l'origine je voulais parler de la manière dont un mathématicien peut définir ce que sont les grandeurs physiques et leurs unités. Mais je n'ai pas résisté à parler d'autres choses, à faire un tableau de plein de grandeurs (ci-dessous) et à entrer dans des discussions sur ce que sont les grandeurs dans la pratique, sur les incertitudes et les échelles de masse. J'ai commencé à écrire des choses sur la réforme du SI qui doit avoir lieu d'ici quelques mois, puis je me suis dit que non, ça faisait vraiment trop, mais il en reste quand même des bouts… (Je garde donc pour une entrée ultérieure les explications précises sur la réforme du SI, même si j'y fais allusion à diverses reprises ici.) Bref, voilà pourquoi cette entrée est encore plus désordonnée que d'habitude. J'espère qu'il y a quand même des choses à en tirer !

Pour essayer de fixer la terminologie, j'appellerai grandeur (plutôt que dimension qui peut causer confusion) quelque chose comme « la masse » de façon abstraite ; et j'appellerai quantité [de cette grandeur] une masse particulière (par exemple 70kg), mesurée, donc, dans une unité. Si on veut parler comme un informaticien, donc, la grandeur sera, pour moi, le type (« la masse »), tandis que la quantité sera l'instance de ce type (70kg). Et l'unité est une quantité particulière (de la grandeur) qu'on a choisie pour exprimer toutes les autres. Comme n'importe quelle quantité non nulle (disons peut-être strictement positive) peut servir d'unité, la différence entre « quantité » et « unité » est juste une question de regard qu'on porte dessus.

Je ne sais pas si ce choix terminologique était le meilleur, je conviens que c'est un peu contre-intuitif de dire que la grandeur de [la quantité] 70kg est la masse, mais je ne suis pas certain qu'il existe de choix vraiment bon (et puis, maintenant que c'est fait, je n'ai plus envie de tout rééditer). J'ai essayé de m'y tenir systématiquement, de toujours utiliser le mot grandeur pour le type et quantité pour la valeur dans le type, mais je ne peux pas exclure quelques lapsus occasionnels.

Ajout () : En fait, je ne distingue pas vraiment la grandeur et la dimensionnalité de cette grandeur (définie formellement ci-dessous), par exemple je ne distingue pas les grandeurs « énergie » et « moment d'une force » (tous les deux ayant l'unité SI de kg·m²/s², même si dans un cas on l'appelle plutôt le joule et dans un autre cas plutôt le newton·mètre, la distinction est plus mnémotechnique que fondamentale) ; de même, pour moi, le watt et le volt·ampère sont bien la même chose, nonobstant le fait qu'on ne les utilise pas exactement de la même manière ; je vais faire occasionnellement allusion à ce problème.

Bref, qu'est-ce que c'est que toute cette histoire ?

Pour commencer, une des propriétés des grandeurs et des unités est qu'on peut les multiplier et les inverser (donc, les diviser) ; alors qu'on ne peut ajouter ou soustraire que des quantités de même grandeur, mais ça j'y reviendrai plus loin. Par exemple, une unité de longueur divisée par une unité de durée (=temps) donne une unité de vitesse (mètre par seconde, kilomètre par heure) : et il s'agit bien d'une division des quantités correspondantes (1km=1000m, 1h=3600s donc 1km/h = 1000m/3600s = (1000/3600)m/s = 0.2777…m/s). On peut dire que, indépendamment des unités, la grandeur « vitesse » est le quotient de la grandeur « longueur » par la grandeur « durée ». De même, la grandeur « surface » est le carré de la grandeur « longueur » (son produit par elle-même). Et la grandeur « fréquence » est l'inverse de la grandeur « durée » (l'unité SI de fréquence, le hertz, est l'inverse de l'unité SI de temps, la seconde).

↑Entry #2546 [older| permalink|newer] / ↑Entrée #2546 [précédente| permalien|suivante] ↑

↓Entry #2537 [older| permalink|newer] / ↓Entrée #2537 [précédente| permalien|suivante] ↓

(dimanche)

Un problème d'algorithmique (en lien secret avec la formule de Weyl)

Méta : Régulièrement je tombe sur des problèmes mathématiques qui me paraissent tellement simples, tellement naturels et/ou tellement évidents (je veux dire évidents à poser, pas forcément évidents à résoudre !) que c'est inconcevable qu'il n'existe pas déjà une littérature abondante à leur sujet. Mais faute de connaître les bons mots-clés ou la bonne façon de formuler le problème (car souvent un même problème admet mille et une reformulations ou réinterprétations), je peux galérer pour mettre le doigt sur cette littérature. C'est extrêmement frustrant. Pour digresser sur ce problème en général, cf. par exemple cette vidéo où le YouTubeur Tom Scott passe la moitié du temps à raconter combien il a eu du mal à trouver le terme Inogon light pour en savoir plus sur un type de signal nautique utilisant intelligemment des effets de moiré pour montrer aux bateaux où aller en fonction de leur position. L'Internet a quelque chose de la Kabbale : quand on connaît le Vrai Nom de quelque chose, on acquiert du pouvoir sur cette chose — en l'occurrence, le pouvoir d'en savoir plus. Le problème que je veux évoquer ici fait partie de ces problèmes qui me semblent tellement « s'imposer » que je suis sûr qu'il a un nom et qu'il y a des chapitres entiers de bouquins d'algorithmiques qui lui sont consacrés ; mais comme je ne le formule pas forcément sous le bon angle, je ne trouve pas.

Il s'agit, donc, de quelque chose que je comprends raisonnablement bien du côté mathématique, mais dont l'algorithmique me laisse passablement perplexe. Ce qui veut dire que j'ai beaucoup de choses à raconter, dont beaucoup ne sont sans doute pas pertinentes pour le problème algorithmique, mais je ne sais pas au juste ce qui l'est et ce qui ne l'est pas.

Voici la première variante du problème algorithmique, qui est la plus simple et élémentaire à énoncer : je vais l'appeler la variante (AS), parce que je vais vouloir en formuler un certain nombre, ce sera plus commode si je leur donne des noms. (Le S signifie symétrique ; le A est là comme dans la classification de Killing-Cartan, mais pour l'instant peu importe.)

(AS) On se donne x et y deux vecteurs (de longueur, disons, n≥1), à coordonnées entières. Je suppose que la somme des coordonnées de x est nulle, et pareil pour y (je ne sais pas si ça sert vraiment à quelque chose).

Problème : trouver tous les produits scalaires possibles σ(xy entre y et un vecteur σ(x) obtenu en permutant les coordonnées de x, avec, pour chacun, son nombre d'occurrences, c'est-à-dire le nombre de permutations σ des coordonnées de x qui conduisent à ce produit scalaire.

Exemple : si x=(−2,−1,0,1,2) et y=(−2,0,0,1,1), la réponse attendue est {−7: 4 fois, −6: 4 fois, −5: 12 fois, −4: 8 fois, −3: 12 fois, −2: 4 fois, −1: 8 fois, 0: 16 fois, 1: 8 fois, 2: 4 fois, 3: 12 fois, 4: 8 fois, 5: 12 fois, 6: 4 fois, 7: 4 fois} (chaque produit scalaire possible σ(xy étant suivi de son nombre d'occurrences : notamment, il y a 16 permutations des coordonnées de x qui donnent un produit scalaire nul avec y). • Autre exemple : si x=y=(−2,−1,0,1,2), la réponse attendue est {−10: 1 fois, −9: 4 fois, −8: 3 fois, −7: 6 fois, −6: 7 fois, −5: 6 fois, −4: 4 fois, −3: 10 fois, −2: 6 fois, −1: 10 fois, 0: 6 fois, 1: 10 fois, 2: 6 fois, 3: 10 fois, 4: 4 fois, 5: 6 fois, 6: 7 fois, 7: 6 fois, 8: 3 fois, 9: 4 fois, 10: 1 fois}.

Il y a évidemment plein de façons de reformuler ça et plein de remarques évidentes à faire. Par exemple, je peux dire qu'il s'agit de considérer toutes les façons d'apparier (bijectivement) les coordonnées de x avec celles de y et de sommer les produits des coordonnées appariées entre elles : sous cette forme, il est évident que le résultat est symétrique entre x et y ; par ailleurs, il est clair que ça ne change rien de permuter les coordonnées de x ou celles de y, donc on peut les supposer triées au départ. Si on veut, je me donne deux paquets (deux « multiensembles ») x et y de nombres, de même taille, mais sans ordre, et je cherche toutes les façons de faire un produit scalaire.

↑Entry #2537 [older| permalink|newer] / ↑Entrée #2537 [précédente| permalien|suivante] ↑

↓Entry #2532 [older| permalink|newer] / ↓Entrée #2532 [précédente| permalien|suivante] ↓

(mercredi)

Petite animation (merdique) d'ondes sur un tore plat

En lien avec l'entrée précédente (que personne n'a lue mais c'est normal), j'ai produit ce petit gadget JavaScript qui (s'il n'est pas complètement cassé) représente une animation de l'équation des ondes sur un tore plat, en l'occurrence le tore plat E/L quotient du plan euclidien E=ℝ² par un réseau L triangulaire équilatéral (i.e., la fonction est périodique par L), à partir d'une condition initiale gaussienne assez piquée (censée donnée une idée d'approximation d'une distribution δ). Si on préfère, cela revient à faire l'équation des ondes dans le plan à partir d'une condition initiale qui est la somme d'une gaussienne centrée sur chaque point de L. Concrètement, il s'agit juste de calculer (la fonction du temps t et du point xE/L) :

αL* cα exp ( 2 i π (αx) ) cos ( 2 π |α| t ) cα = exp ( |α|2 / U2 )

— soit, en plus moche pour les navigateurs cassés qui ne gèrent pas le MathML —

αL* cα·exp(2iπ(α·x))·cos(2π|αt) cα = exp(−|α|²/U²)

U est un paramètre d'étroitesse de la condition initiale, et, histoire de faire le lien avec les notations de l'entrée précédente, Λ(α)=|α|² et m(α)=1 pour le paramétrage par tous les éléments αL*. Ce que fait mon programme est uniquement de calculer cette somme (pour les α pas trop loin de l'origine dans L* ; pour alléger les calculs, il précalcule les fonctions de α et x et se contente ensuite de les sommer).

Commentaire mathématique : Si la dimension d'espace était impaire, l'évolution de l'équation des ondes à partir d'un δ initial se ferait uniquement sur des fronts sphériques centrés sur les points du réseau (imaginez que vous superposez une sphère centrée en chaque point de L, dont le rayon croit linéairement avec le temps, et dont l'amplitude décroît proportionnellement à la surface de façon que la quantité totale reste constante), et l'évolution à partir d'une gaussienne donne la même chose avec des sphères un peu épaissies ; en dimension paire, ce qui est le cas ici, ce « principe de Huygens » ne vaut pas, la fonction de Green de l'équation des ondes n'est pas concentrée sur une sphère[#], il y a une « queue » (négative par rapport au front d'onde, et qui apparaît en bleu sur mon animation). • Par ailleurs, même si le réseau L* ici est le réseau des poids de SU₃ et même si on a symétrie par le groupe de Weyl, il ne s'agit pas de l'équation des ondes sur SU₃ (pour ça il faudrait corriger Λ(α) et m(α)), c'est en gros ce qu'essaie d'expliquer l'interminable entrée qui précède.

[#] Ceci dit, ça doit être aussi assez joli comme dessin, une superposition de cercles de rayon croissant linéairement avec le temps et centrés sur chacun des points d'un réseau L triangulaire équilatéral.

Bon, tout ça fait des images pas trop moches, je dois l'avouer, et l'aspect « kaléidoscopique » apparaît assez clairement. Je pourrais mettre une animation de ce genre sur YouTube.

Ajout () : Voici les vidéos YouTube : pour un réseau triangulaire équilatéral et pour un réseau carré (j'ai eu la folie, dans les deux cas, de calculer ça en 1920×1080, 25fps, pour une vidéo de 3′=180s ; ça m'a pris deux fois 40 minutes de calcul, mais il faut reconnaître que le résultat n'est pas mal).

Ajout 2 : Je recopie le lien fourni dans le commentaire de Benoit qui a écrit une version bien plus efficace de mon animation en utilisant WebGL.

Mais le calcul en direct est péniblement lent. Je pensais que sur un ordinateur moderne je n'aurais même pas besoin d'optimiser et je pouvais calculer la somme de quelques centaines de cosinus par pixel d'une image de taille raisonnable à une vitesse d'animation qui dépasse la perception de l'œil humain, mais apparemment calculer des centaines de millions de cosinus par seconde ça ne se fait pas sur un simple ordinateur de bureau, en tout cas pas en JavaScript.

Comme je déteste optimiser par-dessus tout, et que JavaScript commence à me sortir par les oreilles, je ne touche plus à ce code. Si quelqu'un veut l'améliorer (rendre le truc interactif en ajoutant un bouton pause ou quelque chose comme ça, permettre de bouger, zoomer, ou ce que vous voudrez, ou encore changer le réseau — il y a juste quelques lignes à commenter/décommenter pour faire un réseau carré), envoyez-moi des patchs, mon code est lisible et commenté, mais je ne veux pas de suggestions non codées. Parce que, là, pour le moment, les incantations propitiatoires du JavaScript servant à conjurer des petites crottes de ragondin, elles me gonflent prodigieusement.

Si vous voulez savoir ce que ça donne comme son, voici la conversion directe en onde sonore de la valeur mesurée au point central (l'origine de E/L, celle où est centrée la gaussienne initiale, i.e., mettre x=0 dans les formules ci-dessus), avec exactement les paramètres de l'animation, juste accélérée d'un facteur 8800 par rapport à l'animation affichée par le JavaScript. Mais ça donne juste un bruit strident atroce (moralité, une jolie image ne correspond pas forcément à un joli son, et si je veux transformer des spectres en sons un peu harmonieux, une dissipation dans le temps, dépendant de la fréquence, est indispensable).

↑Entry #2532 [older| permalink|newer] / ↑Entrée #2532 [précédente| permalien|suivante] ↑

↓Entry #2531 [older| permalink|newer] / ↓Entrée #2531 [précédente| permalien|suivante] ↓

(lundi)

Où je commence par penser tout haut toujours à propos de spectres, et finis par ranter de façon interminable sur les caractères des groupes de Lie

Avant-propos : Cette entrée est une sorte de brain-dump, qui finit assez différemment de ce qu'elle commence. J'étais parti sur l'idée, sans avoir forcément pour but d'être compréhensible (en tout cas pas complètement, en tout cas pas par tout le monde), de jeter des réflexions surtout pour moi-même (comme une sorte de sauvegarde de mon état mental) sur des questions autour du spectre du laplacien. Sachant que je n'avais pas les idées complètement claires sur certaines des choses qui suivent, donc je ne peux pas expliquer tout ça parfaitement, encore moins le vulgariser au niveau où j'aimerais idéalement le faire : le but était plutôt de retrouver mes idées éventuellement plus tard, quitte à produire quelque chose d'un peu abscons et pas forcément bien correct mathématiquement ; et je me disais que ça ne ferait pas de mal de les mettre en ligne. Mais en pondant tout ça, je me suis laissé emporter par mon sujet, et la section sur les groupes de Lie compacts a pris une place démesurée, et s'est écartée du point de vue initial (finalement, pour ce que je raconte sur les groupes de Lie, on n'a pas vraiment besoin de savoir ce qu'est un laplacien ni de prononcer son nom, et d'ailleurs comme je prends l'exemple du groupe des rotations, on n'a pas vraiment non plus besoin de savoir ce qu'est un groupe de Lie compact) ; et j'en ai écrit des pages sur l'analyse de Fourier sur un groupe de Lie compact. Chose que je comprends quand même nettement mieux que le problème du spectre du laplacien en général, mais ça ne veut pas forcément dire que je l'explique mieux. Et finalement, je ne sais plus bien de quoi parle cette entrée, il y a plusieurs sujets assez indépendants, et le niveau auquel je place mes explications varie d'un endroit à l'autre. Bref, je ne sais pas ce que tout ça vaut, mais maintenant que c'est écrit, ce serait quand même idiot de ne pas le mettre en ligne. C'est dommage que, comme j'ai fait une énorme moussaka, tout le monde va être rebuté, mais tant pis, je n'ai plus le courage d'essayer de démêler les ingrédients de la moussaka.

Je commence en reprenant la ligne de pensées commencée dans l'entrée précédente (et inspirée par un roman de Connes, Chéreau et Dixmier, donc) : je cherche à produire des sons mathématiques intéressants (et pas déplaisants) à écouter, et une des façons d'y arriver semble être de considérer un spectre, notamment le spectre du laplacien (et donc en pratique, de l'équation des ondes) sur une variété riemannienne (compacte, parce que je ne suis pas analyste ni géomètre, moi, je ne sais pas gérer le cas non-compact[#]) ; plusieurs questions soulevées incidemment : quels objets choisir pour lesquels on sait calculer explicitement le spectre du laplacien (et qu'est-ce que ça signifie au juste) ?, quelles données sont associées au spectre en question ?, comment précisément convertir ce spectre en un son ?, d'ailleurs, comment mener le calcul sur ordinateur ? ; et aussi : comment vulgariser la notion de spectre du laplacien (notamment sur un groupe de Lie, espace riemannien symétrique, etc.) ? (Je ne compte pas tant essayer de faire cette vulgarisation ici et maintenant, mais peut-être donner les pistes par lesquelles je l'aborderais pour pouvoir les retrouver si je devais le faire plus tard.) Je vais évoquer le cas des tores plats (quotients de l'espace euclidien par un réseau) puis, comme expliqué au paragraphe précédent, je vais dévier sur la théorie de Weyl de l'analyse harmonique sur les groupes de Lie compacts, ce qui est largement indépendant de ce que je raconte au début. Et à la fin, je serai trop fatigué pour parler des espaces riemanniens symétriques autrement que pour dire que suis trop fatigué.

[#] Une blague, qui est d'ailleurs peut-être une histoire vraie, qu'on m'avait racontée il y a longtemps, concerne un mathématicien dont la femme… — non, ne soyons pas sexiste comme ceux qui m'ont raconté cette histoire, je vais plutôt dire : — une mathématicienne dont le mari ne connaît absolument rien aux maths ; mais elle lui a donné l'astuce suivante permettant presque à tous les coups de poser une question pertinente lors d'un échange entre matheux : il suffit d'attendre qu'il y ait une petite pause dans la conversation, de prendre un air pensif, et de demander et est-ce que vous avez considéré le cas non-compact ?.

Plan

↑Entry #2531 [older| permalink|newer] / ↑Entrée #2531 [précédente| permalien|suivante] ↑

↓Entry #2528 [older| permalink|newer] / ↓Entrée #2528 [précédente| permalien|suivante] ↓

(lundi)

Le Spectre d'Atacama — et quelques spectres de groupes de Lie à écouter

(La première partie de cette entrée parle d'un roman qui parle de maths, la second parle de maths vaguement inspirées par le roman en question : à part cette proximité d'idées, il n'y a pas vraiment de rapport entre elles. Si les maths vous ennuient, à la fin, il y a des sons bizarres à écouter.)

Je viens de finir de lire le livre Le Spectre d'Atacama d'Alain Connes, Danye Chéreau et Jacques Dixmier, et j'avoue que je ne sais pas bien ce que j'en ai pensé. Pour commencer, c'est un livre assez difficilement classable : une sorte de mélange entre roman de science-fiction, fantaisie poétique, vulgarisation scientifique, plaidoyer sur l'intelligence artificielle, conte philosophique, récit picaresque et transposition en fiction de cet essai sur l'hypothèse de Riemann. Chacun des ingrédients me plaît a priori, et j'aime beaucoup l'idée de faire de la fiction à partir de la science, y compris de façon un peu poétique ; mais je trouve le mélange trop peu homogène… disons qu'il y a des grumeaux.

Le style est souvent un peu faible, mais ça ne me gêne pas tant que ça ; ce qui me gêne nettement plus, en revanche, c'est que l'intrigue part tellement dans tous les sens, accumule tellement d'invraisemblances et de rebondissements en apparence gratuits que ma suspension d'incrédulité, à force d'être tellement secouée, finit par lâcher complètement le coup. Parfois le roman devient didactique, parfois il est humoristique, parfois encore onirique, mais il y a trop de moments où on ne sait pas vraiment à quel degré le lire. L'idée de départ est bonne : un astrophysicien travaillant au réseau d'antennes de l'Atacama détecte un spectre d'absorption qui l'intrigue et fait appel à un ami mathématicien (de l'IHÉS…) pour essayer de le comprendre. Il y a aussi quelques tableaux du milieu académique qui sont plutôt réussis. Mais rapidement, et quitte à divulgâcher jusqu'à la fin de ce paragraphe, il est question d'une physicienne qui a volontairement passé son cerveau dans le rayon du LHC et qui a acquis la conscience quantique de vivre dans un espace de Hilbert et des capacités transhumaines mais seulement quand elle est à proximité d'un certain ordinateur : et là, je trouve que c'est vraiment un peu trop ; en plus de ça, le mathématicien part dans un périple dont on ne comprend pas vraiment le sens, qui l'emmène à Valparaiso puis sur une île perdue au milieu de nulle part puis à Sainte-Hélène, et tout ça ne sert pas vraiment l'intrigue. Et quand il est question d'ordinateurs, on sent que les auteurs ne sont pas du tout dans leur élément.

Ceci étant, je pense que c'est un ouvrage intéressant sur le plan de la communication scientifique : pas tellement d'idées scientifiques (il y a un peu de vulgarisation, mais ce n'est certainement pas l'objet principal du livre, et elle est plutôt light), mais de l'amour de la science et — et c'est important — des liens qui relient mathématiques, physique et informatique, et aussi du fait que la science « dure » peut avoir des aspects poétiques. Sur ce plan-là, je dirais que c'est plutôt une réussite. Peut-être finalement que ce roman, qui ne présuppose pas de connaissances scientifiques ou mathématiques, plaira plus à ceux qui justement l'abordent sans a priori.

J'en viens à des maths : la lecture du roman décrit ci-dessus m'a au moins convaincu (ou rappelé) que « les spectres » c'est important et intéressant. Je sais bien, pour avoir souvent entendu des gens le dire, que le spectre du laplacien (sur une variété riemannienne, disons), par exemple, c'est archi-super-important, mais j'avoue que je ne sais essentiellement rien de ce qu'il y a à dire, justement, sur ce spectre du laplacien, même dans des cas idiots (compacts, agréablement symétriques, tout ça tout ça).

En guise d'exercice, je me suis dit que j'allais calculer le spectre du laplacien pour des groupes de Lie compacts G (ou éventuellement des espaces homogènes G/H, par exemple des espaces riemanniens symétriques ou bien des R-espaces (variétés de drapeaux réelles), choses que je confonds d'ailleurs trop facilement[#]).

[#] Digression : Les espaces riemanniens symétriques irréductibles de type compact et simplement connexes sont (les groupes de Lie compacts simples simplement connexes eux-mêmes ainsi que) les quotients G/KG est un groupe de Lie compact simple simplement connexe et K le sous-groupe compact connexe maximal d'une forme réelle G₀ de G (par exemple, la sphère de dimension n est Spin(n+1)/Spin(n) où Spin(n) est le compact connexe maximal de la forme Spin(n,1) de Spin(n+1)), et on peut aussi voir K comme les points fixes d'une involution de G qui correspond à l'involution de Cartan définissant G₀ ; j'ai certainement commis quelques erreurs en disant ça (notamment dans la connexité et la simple connexité), mais l'idée générale doit être à peu près ce que j'ai dit. Les R-espaces, eux, s'obtiennent sous la forme G₁/PP est un parabolique d'un groupe de Lie réel semisimple G₁, qu'on peut aussi voir comme G/(GP) où G est un sous-groupe compact connexe maximal de G₁ et GP un sous-groupe compact maximal (du facteur de Levi) de P (par exemple, l'espace projectif réel dimension n est défini par le quotient de SL(n+1,ℝ) par son parabolique maximal associé à la première racine simple, i.e., les matrices dont la première colonne n'a que des zéros à partir de la deuxième ligne, et on peut le voir comme le quotient SO(n+1)/S(O(n)×O(1)) du sous-groupe compact connexe maximal SO(n+1) de SL(n+1,ℝ)) ; de nouveau, j'ai certainement commis quelques erreurs en disant ça, mais l'idée générale doit être ça. Je n'ai jamais vraiment compris « pourquoi » il y avait ces deux types de quotients très importants des groupes de Lie réels compacts, comment il faut y penser, par exemple du point de vue de l'analyse harmonique, et, de façon encore plus perturbante, pourquoi certains espaces peuvent se voir à la fois comme un espace riemannien symétrique et comme un R-espace (ou presque : cf. l'exemple que je viens de donner de la sphère et de l'espace projectif réel). Si quelqu'un a des éléments de réponse à m'apporter ou simplement des références où ces deux types de quotients sont discutés côte à côte de manière à me désembrouiller, ça m'intéresse ! (J'ai regardé l'article Geometry of Symmetric R-spaces de Tanaka, et j'ai eu l'impression de comprendre encore moins bien et de confondre encore plus après sa lecture.)

Mais aussi, j'avais (peut-être même que j'ai encore) vaguement l'espoir que des spectres intéressants, comme le spectre du laplacien sur tel ou tel espace bien sympathique, pourrait conduire à des sons harmonieux et donc répondre à ma question de trouver un objet mathématique qui s'« auditorise » de façon intéressante et agréable (plutôt que de se « visualiser ») ; dans cet ordre d'idées j'avais bien produit ceci, mais ce n'était pas du tout agréable à écouter et la construction de ces sons n'était pas franchement des plus naturelles.

L'idée générale, cette fois-ci, est qu'une fois connu le spectre du laplacien on peut s'en servir pour résoudre l'équation des ondes et obtenir les fréquences des vibrations propres de l'objet considéré (comme les racines carrées des opposées des valeurs propres du laplacien). Et donc produire des sons qui correspondraient à la manière dont « vibre » l'objet considéré — un groupe de Lie compact G ou un espace homogène G/H — quand, par exemple, on donne un coup dessus.

J'avoue que l'idée de taper un groupe de Lie pour voir comment il résonne me plaît énormément. (Et si j'en crois la lecture du Spectre d'Atacama, ça a aussi des chances de plaire à Connes et/ou Dixmier.)

Bref. Du peu que je sais de l'analyse harmonique sur les groupes de Lie et du théorème de Peter-Weyl, et si je comprends bien que le Casimir fournit la valeur du laplacien sur ce qui correspond à chaque représentation irréductible, le spectre du laplacien sur un groupe de Lie compact G est donné, à un facteur multiplicatif près (essentiellement arbitraire(?), mais négatif), par l'ensemble des valeurs C(v) := ⟨v,v+2ρ⟩ où v parcourt le réseau des poids dominants pour G. (Si tout ceci est du chinois pour vous, ce n'est pas très important, mais l'idée est qu'à G est associé un réseau euclidien appelé le « réseau des poids » et un cône polyédral de sommet l'origine dans cet espace euclidien appelé la « chambre de Weyl », auquel appartient le vecteur ρ dit « vecteur de Weyl », et les poids dominants sont les éléments de la chambre de Weyl ; chaque tel v, ou plus exactement le « caractère » χv associé, peut se concevoir comme un mode propre — un mode de vibration, si on veut — du groupe G, et la valeur du Casimir C(v) := ⟨v,v+2ρ⟩, est essentiellement l'opposé de la valeur propre du laplacien dont le vecteur propre est le caractère : Δχv = −C(vχv pour une certaine normalisation de Δ. S'il y a dans l'assistance des gens qui s'y connaissent en analyse harmonique et qui pourraient confirmer que j'ai bien compris, et peut-être même recommander un endroit où ce que je viens de dire serait écrit noir sur blanc sous cette forme y compris avec la valeur du Casimir, je leur serais reconnaissant.) Par exemple, pour les groupes de rang 2 : pour A₂ (i.e., SU₃), je trouve des valeurs (proportionnelles à) 8/3, 6, 20/3, 32/3, 12, 16, 50/3, 56/3, 68/3, 24, 80/3, 30… (où seules celles qui sont entières sont possibles pour la forme adjointe PSU₃) ; pour B₂ (i.e., Spin₅), je trouve 5/2, 4, 6, 15/2, 10, 21/2, 12, 29/2, 16, 35/2, 18, 20… (où seules celles que j'ai soulignées sont possibles pour la forme adjointe SO₅) ; et pour G₂, je trouve 12, 24, 28, 42, 48, 60, 64, 72, 84, 90, 100, 108… ; et sinon, pour F₄ : 12, 18, 24, 26, 32, 36, 39, 40, 42, 46, 48, 52… ; et vous devinez évidemment j'ai fait le calcul pour E₈ : 60, 96, 120, 124, 144, 160, 180, 186, 192, 196, 200, 210…

Et pour les espaces homogènes G/H, il doit s'agir de se limiter aux plus hauts poids v qui définissent des représentations de G dont la restriction à H a des points fixes (ou, ce qui revient au même par réciprocité de Frobenius, des représentations qui apparaissent dans l'induite à G de la représentation triviale de H, mais je ne suis pas si ça aide de le dire comme ça). J'arrive (mais laborieusement) à faire les calculs sur des cas particuliers en utilisant l'implémentation des règles de branchement dans Sage. Par exemple, le spectre de G₂/SO₄ (l'espace des sous-algèbres de quaternions dans les octonions) semble être : 28, 60, 72, 112, 132, 168, 180, 208, 244, 264, 300, 324… Mais je comprends trop mal les règles de branchement pour savoir s'il faut chercher une logique d'ensemble ou ce à quoi elle ressemblerait (sur les coordonnées de v dans la base des poids fondamentaux ; ce n'est même pas clair pour moi les v en question forment un sous-réseau du réseau des poids ou quel est son rang). Ajout () : À la réflexion, pour les espaces riemanniens symétriques, je crois que je comprends au moins à peu près la situation (tout est dans la notion de système de racines restreintes) ; je crois même que tout est dit dans le chapitre V (par ex., théorème V.4.1) du livre de 1984 de Sigurður Helgason (Groups and Geometric Analysis), même si j'ai vraiment du mal à le lire ; je crois bien que le rang du réseau des poids v tels que la restriction à H ait des points fixes non triviaux coïncide avec le rang de l'espace symétrique G/H, même si j'aimerais bien voir ça écrit noir sur blanc.

Une chose qui m'étonne beaucoup est que ces suites ne semblent pas être dans l'OEIS. Tout le monde parle de l'importance du spectre du laplacien et personne n'a pris la peine de mettre le résultat, pour les cas les plus évidents que sont les groupes de Lie compacts, dans l'OEIS ‽ Comment est-ce possible ‽ J'hésite cependant à les soumettre moi-même parce que, à vrai dire, je ne suis pas très sûr de bien comprendre ce que je fais. (Et, entre autres choses, je ne sais pas du tout si les valeurs que j'ai listées ci-dessus ont un sens dans l'absolu ou seulement à proportionalité près. La valeur du Casimir semble dépendre d'une normalisation un peu arbitraire sur la longueur des racines ou quelque chose comme ça, et du coup je ne sais pas bien quoi prendre ou quoi soumettre.)

Pour ce qui est de produire des sons à partir de ça, il y a un autre truc sur lequel je n'ai pas des idées claires, c'est quelles amplitudes relatives il serait logique d'utiliser pour ces différentes harmoniques. Si on donne un coup de marteau sur le groupe de Lie G₂ (mais pas assez fort pour le casser !), il va peut-être résonner à des fréquences proportionnelles aux racines carrées de 12, 24, 28, 42, 48, 60, etc., mais avec quelles amplitudes ? Le problème se pose déjà sur une sphère de dimension 2 (SO₃/SO₂, si on veut) : les valeurs propres du laplacien sphérique sont (proportionnelles à) (+1), donc si on fait vibrer une sphère, elle produit des fréquences proportionnelles à 1, √3, √6, √10, etc., mais une fois ce spectre connu, ça ne donne pas pour autant un son (même si ça peut faire de jolies animations). Un bout de la réponse est fourni par la multiplicité des valeurs propres en question (sur la sphère, par exemple, (+1) a la multiplicité +1 parce qu'il y a ce nombre-là d'harmoniques sphériques de niveau  indépendantes) ; s'agissant d'un groupe de Lie G, les multiplicités sont les carrés N(v)² des dimensions N(v) = χv(1) des représentations irréductibles correspondantes (par exemple, s'agisant de G₂, les valeurs propres avec multiplicité sont (12,7²), (24,14²), (28,27²), (42,64²), (48,77²), (60,77²), etc.). Mais ensuite ? Il me semble que, pour parler abusivement, les « coefficients » de la distribution δ (centrée en 1∈G) sur la base des caractères χv sont les N(v) = χv(1) et qu'il serait donc logique de donner à la fréquence √C(v) une amplitude proportionnelle à N(v)² (si on tape un coup sec et très localisé sur notre groupe de Lie), mais évidemment ceci diverge très méchamment. Je peux régulariser en remplaçant δ par une gaussienne, ce qui doit revenir à multiplier les coefficients par exp(−C(vσ²) avec σ une sorte d'écart-type de la gaussienne, mais le choix de σ est complètement arbitraire dans l'histoire. Bref, je peux produire des sons en superposant des fréquences proportionnelles aux √C(v) avec des amplitudes proportionnelles aux N(v)²·exp(−C(vσ²), mais le son en question dépend de façon énorme de σ. Une autre idée est de faire varier l'amplitude avec le temps pour donner une dissipation aux modes de vibration, par exemple en exp(−C(vt) (inspiré de l'équation de la chaleur).

Pour faire quand même des essais, de façon assez arbitraire, j'ai décidé de faire que l'intensité de la fréquence √C(v) décroisse en exp(−(C(v)/C(v₀))·(1+t/3s)) où v₀ est le poids qui correspond à la représentation adjointe de G (c'est-à-dire, la plus haute racine), et j'ai de même normalisé les fréquences pour que la fréquence de v₀ soit à 440Hz. C'est-à-dire que j'ai superposé des sin(2π·440Hz·(C(v)/C(v₀))·t) · N(v)² · exp(−(C(v)/C(v₀))·(1+t/3s)) où t est le temps et v parcourt les poids de G. Je n'aime pas le côté assez arbitraire de tout ça (et en particulier de mon 1+), donc je suis preneur d'idées plus naturelles, mais au moins les sons sont intéressants et, pour une fois, pas du tout désagréables à écouter.

Ceci n'est qu'une première expérience : j'en ferai sans doute d'autres quand j'aurai des idées plus claires sur ce que je veux faire et ce qui est intéressant, mais en attendant, voici quelques essais de ce que ça peut donner comme son de frapper différents groupes de Lie compacts (calibrés pour que leur représentation adjointe sonne le la à 440Hz) : en rang 1 : A₁ (c'est-à-dire SU₂, qui est vraiment une 3-sphère, je voulais vérifier que ça avait un son de cloche plausible et ça a effectivement un son de cloche vaguement plausible, c'est déjà ça) ; en rang 2 : A₂ (c'est-à-dire SU₃), B₂ (c'est-à-dire Spin₅) et G₂ ; en rang 4 : A₄ (c'est-à-dire SU₅), B₄ (c'est-à-dire Spin₉), C₄ (c'est-à-dire Sp₄), D₄ (c'est-à-dire Spin₈) et F₄ ; et bien sûr : E₆ et E₈. Tous ces fichiers sont du FLAC et chacun dure 6 secondes, si votre navigateur ne les ouvre pas spontanément, téléchargez-les et vous trouverez certainement un truc qui les lit. Tous les groupes que je viens de donner sont la forme simplement connexe, mais j'ai aussi produit des essais pour comparer le son de la forme simplement connexe avec la forme adjointe (laquelle a moins d'harmoniques) : Spin₅ versus SO₅ d'une part, et SU₃ versus PSU₃ de l'autre.

Ajout () : voir ce fil Twitter et/ou cette version sur YouTube pour les sons de quelques grassmanniennes réelles, complexes et quaternioniques, ainsi que le plan projectif octonionique.

↑Entry #2528 [older| permalink|newer] / ↑Entrée #2528 [précédente| permalien|suivante] ↑

↓Entry #2527 [older| permalink|newer] / ↓Entrée #2527 [précédente| permalien|suivante] ↓

(jeudi)

Une conjecture « du dimanche » sur les nombres premiers

Je racontais ici que les « mathématiciens du dimanche » étaient souvent fascinés par les nombres premiers et capables de produire toutes sortes de conjectures fantaisistes à leur sujet ; et aussi, ils sont fascinés par l'écriture en base 10. Voici que je vois passer sur MathOverflow (et précédemment sur Math.StackExchange) la conjecture suivante, qui ressemble beaucoup à la caricature de la « conjecture du mathématicien du dimanche », à ceci près qu'elle conjecture que des nombres ne sont pas premiers :

Soit j≥1 un entier naturel, et Nj le nombre formé de la concaténation des écritures en base 10 des nombres (« de Mersenne » consécutifs) 2j+1−1 et 2j−1 ; c'est-à-dire : Nj = 10m·(2j+1−1) + (2j−1) où m := ⌊log(2j−1)/log(10)⌋+1 est le nombre de chiffres de l'écriture décimale de 2j−1.

(Par exemple, N₁=31 (concaténation de 3 et 1), N₂=73 (concaténation de 7 et 3), N₃=157 (concaténation de 15 et 7), N₄=3115 (concaténation de 31 et 15), etc.)

Conjecture d'Enzo Creti : si Nj≡6 (mod 7), alors Nj n'est pas premier.

(Par exemple : pour j=9, on a N9=1023511, qui est congru à 6 modulo 7, et il n'est pas premier : il vaut 19×103×523 ; pour j=10, on a N10=20471023, qui est congru à 6 modulo 7, et il n'est pas premier : il vaut 479×42737.)

(Je ne sais pas si l'auteur de cette conjecture est un mathématicien « du dimanche », je ne sais rien sur lui, mais l'énoncé, en tout cas, ressemble exactement au type de spéculations sur les nombres premiers et les écritures en base 10 dont je voulais parler.)

Ce genre de problèmes est à la fois agaçant et passablement intéressant au niveau méta.

Expérimentalement, la conjecture est vérifiée jusqu'à des valeurs passablement grandes de j (l'auteur prétend être allé jusqu'à 4×10⁵ ; moi je me suis arrêté à 10⁴) ; et de plus, elle n'est pas vide, c'est-à-dire qu'il y a effectivement une densité significative (en fait, 1 sur 9) de j pour lesquels la prémisse Nj≡6 (mod 7) est vérifiée.

(On peut accessoirement remarquer que dans chacune des autres classes de congruence de Nj modulo 7, exceptée bien sûr la classe 0, on trouve des nombres premiers. C'est la classe 6 qui semble éviter les nombres premiers. À toutes fins utiles, en distinguant les cas de congruence de m modulo 6 et de j modulo 3, on peut remarquer que 10m·(2j+1−1) + (2j−1) est congru à 6 modulo 7 lorsque soit (m≡3 (mod 6) et j≡0 (mod 3)) soit (m≡4 (mod 6) et j≡1 (mod 3)).)

Pourtant, je pense que n'importe quel théoricien des nombres sera d'accord avec moi pour dire qu'il ne croit pas une seule seconde à une telle conjecture. Pourquoi ?

D'abord, on se rappelle que le théorème des nombres premiers peut s'interpréter en disant que la « probabilité d'être premier » empirique d'un entier x tiré au hasard vaut environ 1/log(x) ; ou si le nombre est impair par construction, disons plutôt 2/log(x). En l'occurrence, on a log(Nj) = 2·log(2)·j + O(1), si bien que Nj a empiriquement une « probabilité d'être premier » qui décroît comme une fonction harmonique de j (quelque chose comme 1/(log(2)·j), en tenant compte du fait qu'il est forcément impair). Or la série harmonique diverge, donc il n'est pas vraisemblable que les Nj échouent tous à être premiers « par hasard ». En revanche, comme la série harmonique diverge très lentement (logarithmiquement), cela veut bien dire qu'il peut être nécessaire de pousser très très loin pour trouver un contre-exemple, donc avoir vérifié 10⁴ ou 10⁵ valeurs ne vaut pas grand-chose, et il n'est pas du tout invraisemblable que 10⁴ ou 10⁵ valeurs échouent toutes à être premières « par hasard » (expliquant ainsi la constatation expérimentale).

Il est donc invraisemblable que la conjecture soit vraie « par hasard », mais vraisemblable qu'elle le paraisse quand même jusqu'à 10⁴ ou 10⁵. Maintenant, se peut-il que la conjecture soit vraie autrement que « par hasard » ? Cela voudrait dire qu'il y aurait une « raison » expliquant une factorisation de 10m·(2j+1−1) + (2j−1) à tous les coups (par exemple une identité algébrique, ou une conguence à 0 qui vaut à tous les coups, enfin, une « raison » qui fait qu'il n'est jamais premier). Or, si on met de côté la donnée que m est le nombre de chiffres décimaux de 2j−1, ce n'est pas vrai que 10m·(2j+1−1) + (2j−1) n'est jamais premier. En effet, en changeant un petit peu m, j'ai le contre-exemple de 1070·(2230−1) + (2229−1) (où le nombre 2229−1 a 69 chiffres décimaux, j'ai inséré juste un 0 de plus dans la concaténation) : ce nombre est bien congru à 6 modulo 7, et il est premier (il a 140 chiffres, alors vous m'épargnerez de l'écrire complètement). Bref, si la conjecture était vraie autrement que par hasard, il faudrait avoir une factorisation de 10m·(2j+1−1) + (2j−1) qui dépende du fait que m est précisément le nombre de chiffres décimaux de 2j−1, et ça, ça semble complètement abracadabrant. (Tout ce que je raconte est complètement empirique, bien sûr, je n'ai pas de contre-exemple à la conjecture énoncée plus haut, mais j'explique pourquoi je n'y crois pas.)

Bref, je suis complètement convaincu qu'il y a un contre-exemple, et que ce contre-exemple a un j très grand (donc un Nj gigantesque), et ce n'est pas très surprenant qu'il soit difficile à trouver. Pour être un peu plus précis dans la quantification de la vraisemblance, numériquement, le produit des 1−(2/log(Nj)) (i.e., leur probabilité empirique de ne pas être premiers, le 2 étant là parce qu'ils sont impairs) parcourant ceux des Nj qui sont congrus à 6 modulo 7 vaut environ 0.25 pour j allant jusqu'à 10⁴, c'est-à-dire qu'il y avait a priori environ 25% de chances pour qu'aucun de ces nombres ne soit premier compte tenu de leurs tailles (et du fait qu'ils sont impairs) ; si on monte jusqu'à 4×10⁵, cela doit tomber à environ 18%. Bref, ce n'est pas du tout invraisemblable que la conjecture soit vraie jusqu'à ce point-là « par hasard ». Il suffit qu'il y ait une dizaine de mathématiciens du dimanche qui essaient des conjectures de ce genre, et il y en aura bien un qui tombera sur une qui marche sur toutes les valeurs que sa patience lui permettra de tester ; en fait, il suffit même qu'un seul mathématicien du dimanche ait testé la restriction des Nj à suffisamment de classes de congruence modulo des petits nombres pour tomber sur une qui semble ne contenir que des nombres composés.

Il n'est cependant pas exclu à mes yeux qu'il y ait une « raison » un peu plus précise que le hasard pour laquelle la conjecture soit vraie pour des « assez petites » valeurs de j, et c'est un problème possiblement intéressant. Il est par exemple possible que plein de cas de congruence de j et de m excluent la primalité. (Un exemple idiot est que si j est congru à 0 modulo 4, sans aucune discussion sur m, alors Nj est multiple de 5 — parce que 2j−1 l'est — et donc Nj n'est certainement pas premier ; donc déjà il n'y a plus que les quatre cinquièmes des j qui jouent vraiment, et cela contribue à rendre encore moins invraisemblable que la conjecture soit vraie « par hasard » pour des petites valeurs de j. Mais il y a peut-être des choses plus intelligentes à dire.)

Il y a notamment une chose qu'on peut voir, c'est que m := ⌊log(2j−1)/log(10)⌋+1 (le nombre de chiffres décimaux de 2j−1) vaut en fait ⌊j·ξ⌋+1 où ξ := log(2)/log(10) ≈ 0.301. Les réduites du développement en fraction continue de ξ sont 1/3, 3/10, 28/93, 59/196, etc. Si je remplace m=⌊j·ξ⌋+1 par m=⌊j·x⌋+1 où x est une de ces réduites, j'obtiens d'autres suites de nombres Nj (dépendant de x que j'omets abusivement dans la notation), à savoir Nj := 10(⌊j·x⌋+1)·(2j+1−1) + (2j−1), qui coïncident avec Nj au début (et d'autant plus loin que la réduite est bonne), et je peux poser la question de la conjecture analogue pour ces suites-là. Pour x=1/3, la conjecture sur les N′ ne vaut pas, car pour j=330, le nombre N330 = 10111·(2331−1) + (2330−1) est congru à 6 modulo 7 et est premier ; pour x=3/10, la conjecture sur les N′ ne vaut pas non plus, car pour j=849, le nombre N849 = 10255·(2850−1) + (2849−1) est congru à 6 modulo 7 et est premier. Mais pour x=28/93 (autrement dit, avec Nj := 10(⌊j·28/93⌋+1)·(2j+1−1) + (2j−1)), je n'ai pas trouvé de contre-exemple : au moins jusqu'à j=10⁴, les Nj qui sont congrus à 6 modulo 7 ne sont jamais premiers. C'est déjà moins invraisembable d'imaginer que tous ces Nj-là soient premiers que pour les Nj de la conjecture de départ : on peut tout à fait imaginer qu'il y ait une distinction de quelque chose comme 93 cas selon la valeur de j qui permette, dans chacun des cas (ou simplement dans un grand nombre de ces cas, diminuant d'autant le hasard !), de montrer que Nj serait divisible par quelque chose. Du coup, si Nj n'est jamais premier, cela expliquerait que plein de petites valeurs de Nj soient composées, et il est encore moins surprenant qu'ensuite on tombe par hasard sur des nombres composés.

Mise à jour (avant publication) : bon, en fait, pour j=14058, le nombre N14058 = 104233·(214059−1) + (214058−1) est congru à 6 modulo 7 et semble être premier (en tout cas il passe des tests de pseudo-primalité), donc mon explication n'est pas la bonne, mais je la laisse parce qu'on voit que ce genre de choses est tout à fait imaginable.

Laissant de côté la question mathématique proprement dite, il reste la question de savoir comment un mathématicien (au hasard, féru de vulgarisation) doit réagir face à ce genre de conjectures. C'est toujours un peu délicat d'expliquer je n'y crois pas du tout malgré vos constatations expérimentales, et même si on peut expliquer tout ce que je viens d'expliquer sur les probabilités, il reste quand même un certain acte de foi, quand je dis qu'il est « complètement abracadabrant » qu'il y ait un phénomène de ce genre sur les nombres premiers qui fasse intervenir de façon cruciale le nombre de chiffres décimaux du nombre 2j−1 (même si on le revoit comme ⌊j·ξ⌋+1 avec ξ = log(2)/log(10)).

↑Entry #2527 [older| permalink|newer] / ↑Entrée #2527 [précédente| permalien|suivante] ↑

↓Entry #2522 [older| permalink|newer] / ↓Entrée #2522 [précédente| permalien|suivante] ↓

(mercredi)

Les revêtements doubles du groupe symétrique sont pénibles

Écrire l'entrée récente sur la vulgarisation des mathématiques m'a motivé a essayer d'écrire un morceau de vulgarisation sur la symétrie, les groupes finis et (l'histoire de) la classification des groupes simples finis. Comme c'était évidemment prévisible, ce texte est en train de grandir jusqu'à une taille démesurée, et comme d'habitude le risque commence à devenir sérieux que je finisse par en avoir marre et que je laisse tomber ; j'essaierai, le cas échéant, de m'efforcer de publier ce que j'aurai déjà écrit même si c'est inachevé plutôt que le garder indéfiniment dans mes cartons en pensant je finirai peut-être un jour. Ceci n'est pas le texte en question : c'est une tangente qui est déjà insupportablement longue en elle-même. Mais ceci est une illustration de ce que je disais dans l'entrée récente liée ci-dessus : on apprend toujours quelque chose en faisant de la vulgarisation, même quand on croit se placer à un niveau où on sait déjà tout ; et aussi que ça peut être un problème mathématiquement difficile de trouver comment bien expliquer ceci ou cela.

Puisqu'il s'agit de raconter mes difficultés, je m'adresse dans ce qui suit à des lecteurs qui sont déjà familiers avec la notion de groupe (et de sous-groupe, de quotient, de permutations, de signature (=parité) d'une permutation, et quelques choses à peu près à ce niveau-là). Normalement le contenu de l'entrée interminable que je viens de promettre pour un lendemain rieur devrait suffire à comprendre celle-ci (mais bon, c'est la théorie ; pour la pratique, je ne sais pas bien). Bref.

Remarque informatique : J'utilise la notation 𝔖 pour le groupe symétrique et 𝔄 pour le groupe alterné. Vous devriez voir une ‘S’ gothique (enfin, fraktur) pour le premier et un ‘A’ gothique pour le second. Mais on me souffle que dans certaines contrées reculées où la totalité d'Unicode ne baigne pas encore le monde de sa lumière bienfaisante et où les polices sont incomplètes, ces deux symboles pourraient apparaître comme des simples carrés (sans même un numéro hexadécimal à l'intérieur permettant de les distinguer simplement), ce qui est un peu fâcheux si je cherche à dire que 𝔄n est simple (pour n≥5) alors que 𝔖n n'est que « presque simple », par exemple. Pour toucher aussi ces provinces reculées (ainsi que les gens qui font une allergie à l'écriture gothique), j'ai prévu un peu de magie en JavaScript qui remplacera en un seul clic tous ces symboles par des identifiants plus explicites Sym et Alt : cliquez ici pour activer ce remplacement.

Je commence par expliquer le contexte (même si ce n'est pas vraiment important pour ce que je veux raconter ci-dessous, et c'est un peu plus technique, donc on peut ignorer), une des idées que je veux évoquer, au moins rapidement et en petits caractères, même si c'est un peu technique, est le fait qu'un groupe simple fini non abélien G apparaît souvent, dans la nature, « étendu » par des petits groupes (résolubles, souvent cycliques), de l'une ou l'autre, ou les deux, de manières (que, à ma grande honte, j'ai beaucoup tendance à confondre). À savoir : (1) « par la droite » par des automorphismes extérieurs, c'est-à-dire sous la forme d'un groupe E (dit presque simple) intermédiaire entre G et le groupe Aut(G) des automorphismes de G, si bien que G est un sous-groupe distingué de E avec un « petit » quotient (le plus gros possible étant le groupe Out(G)=Aut(G)/Int(G) des automorphismes extérieurs de G) ; ou bien (2) « par la gauche » par un sous-groupe central, c'est-à-dire sous la forme d'un groupe G˜ (dit quasisimple), parfait (= sans quotient abélien), ayant cette fois G comme quotient par un noyau contenu dans le centre de G˜ (et de nouveau il y a un plus gros revêtement possible, donné par le multiplicateur de Schur) ; et on peut avoir les deux à la fois, ce qui complique encore les définitions (je n'en connais d'ailleurs pas qui ne soient pas passablement pénibles à donner, donc si quelqu'un a ça, ça m'intéresse), et en plus on se perd dans les marais de l'« isoclinisme ». Je voudrais donner des exemples des deux phénomènes, voire des deux à la fois. Ne voulant pas supposer que mon lecteur est familier avec l'algèbre linéaire, je voudrais donner l'exemple du groupe alterné G = 𝔄n des permutations paires sur n objets. À ce moment-là, l'exemple de la situation (1) est facile à donner, c'est le groupe symétrique E = G:2 = 𝔖n de toutes les permutations sur n objets (et il n'est pas difficile d'expliquer que l'automorphisme intérieur défini par une permutation impaire devient, quand on le restreint au groupe alterné G = 𝔄n, un automorphisme extérieur). La situation (2) se produit aussi, et il existe un revêtement double G˜ = 2·G = 2·𝔄n, et deux revêtements doubles (« isoclinaux ») 2·𝔖n⁺ et 2·𝔖n⁻. J'ai donc bien envie d'essayer de décrire à quoi ressemblent ces groupes. L'ennui, c'est qu'ils ne sont vraiment pas commodes à décrire.

Ce dont il est question, ce sont deux groupes 2·𝔖n⁺ et 2·𝔖n⁻ qui sont des « revêtements doubles » du groupe symétrique 𝔖n sur n objets, et qu'on peut considérer comme des sortes de « permutations avec un signe »[#].

[#] (Ajout)Il vaut mieux éviter de dire permutations signées, parce que le groupe des permutations signées est encore autre chose (que les quatre groupes de permutations-avec-un-signé décrits ci-dessous, et qui ont tous 2·n! éléments) : le groupe des permutations signés, ou « produit en couronne » {±1} ≀ 𝔖n, lui, a 2n·n! éléments : on peut le décrire comme les permutations de l'ensemble {±1}×{1,…,n} qui, si elles envoient (+1,x) sur (±1,y) doivent alors nécessairement envoyer (−1,x) sur (∓1,y) (autrement dit, changer la première coordonnée de la source change la première coordonnée de la cible) ; on peut aussi voir ça comme des matrices dont toutes les entrées sont nulles sauf qu'il y a des ±1 sur le graphe d'une permutation (entre lignes et colonnes). Ce groupe {±1} ≀ 𝔖n, bien que plus gros, est assez simple à manipuler, et malheureusement il ne contient pas (sauf pour n très petit) les groupes 2·𝔖n⁺ et 2·𝔖n⁻ dont je veux parler ici. Je vais y revenir.

L'idée est la suivante : je vais chercher des groupes G˜ ayant 2n! éléments, à savoir deux pour chaque permutation σ dans G := 𝔖n ; disons qu'on va noter +[σ] (ou simplement [σ]) et −[σ] les deux éléments de G˜ correspondant à une permutation σ, mais attention, le choix de qui est +[σ] et qui est −[σ] est dans une certaine mesure arbitraire, c'est bien ça qui va poser problème. Je vais maintenant imposer plusieurs choses : d'abord, si 1 désigne la permutation triviale (l'identité : celle qui envoie chaque objet sur lui-même), alors +[1], qu'on va juste noter +1 ou 1 sera l'élément neutre de mon groupe ; quant à −[1], qu'on va simplement noter −1, il aura la propriété que le produit (−1)·[σ] sera −[σ] et le produit (−1)·(−[σ]) sera +[σ] comme on s'y attend, autrement dit, −1 est « central » (il commute à tout) et échange +[σ] et −[σ] ; enfin, je vais vouloir que [σ]·[τ] soit ±[σ·τ] où σ·τ désigne le produit dans 𝔖n et ± signifie qu'il y a peut-être un signe (cela dépend de σ et τ : on pourrait le noter c(σ,τ)) mais je n'impose rien à son sujet (c'est-à-dire, rien que ce qui est nécessaire pour obtenir un groupe).

Il se trouve qu'il y a (pour n≥4) exactement quatre groupes qui répondent aux contraintes que je viens d'énoncer : deux sont sans intérêt (mais il est pertinent de les décrire pour expliquer un peu comment les choses peuvent fonctionner) et les deux autres sont ces fameux revêtements doubles 2·𝔖n⁺ et 2·𝔖n⁻ dus à Issai Schur :

  1. Le plus évident est le groupe produit direct {±1}×𝔖n (ou 2×𝔖n étant entendu que 2 désigne abusivement le groupe cyclique Z₂={+1,−1} à deux éléments) ; c'est-à-dire qu'ici le signe et la permutation n'interagissent pas du tout. Autrement dit, dans ce groupe-là, on a [σ]·[τ] = [σ·τ] (toujours avec un signe ‘+’), et il n'y a vraiment rien d'intéressant à en dire. Remarquons que si σ est une transposition (= permutation d'ordre 2 échangeant deux éléments et laissant fixes tous les autres), alors ±[σ] est d'ordre 2 dans ce groupe, et que si σ est le produit de deux transpositions de support disjoints (= permutation d'ordre 2 échangeant deux paires d'éléments et laissant fixes tous les autres), alors ±[σ] est encore d'ordre 2.
  2. Un groupe un petit peu moins évident est celui dans lequel [σ]·[τ] = [σ·τ] sauf lorsque σ et τ sont toutes les deux impaires, auquel cas [σ]·[τ] = −[σ·τ]. Faute d'idée de meilleure notation, je vais le noter 2⊙𝔖n pour y faire référence plus tard. En fait, il est peut-être plus parlant pour y penser de modifier la notation et, lorsque σ est une permutation impaire, de noter (ou en tout cas de penser comme) +i[σ] et −i[σ] plutôt que +[σ] et −[σ] les deux éléments du groupe qui relèvent σ, où i est la racine carrée complexe standard de −1, auquel cas la règle des signes que je viens de donner est assez logique. (Je répète que je ne change pas du tout le groupe, là, je change juste la manière de noter ses éléments ou simplement d'y penser.) Ce groupe a la propriété que si σ est une transposition, alors ±[σ] est d'ordre 4 dans ce groupe (puisque son carré va être −1 d'après la règle de signe), et que si σ est le produit de deux transpositions de support disjoints, alors ±[σ] est d'ordre 2.
  3. On a le groupe 2·𝔖n⁺ que je vais essayer (sans grand succès) de décrire : il a la propriété que si σ est une transposition, alors ±[σ] est d'ordre 2 dans ce groupe (son carré vaut 1), et que si σ est le produit de deux transpositions de support disjoints, alors ±[σ] est d'ordre 4 (son carré vaut −1).
  4. Enfin, on a le groupe 2·𝔖n⁻ : il a la propriété que si σ est une transposition, alors ±[σ] est d'ordre 4 dans ce groupe (son carré vaut −1), et que si σ est le produit de deux transpositions de support disjoints, alors ±[σ] est également d'ordre 4 (son carré vaut −1).

Les deux premiers groupes dont je viens de parler (2×𝔖n et 2⊙𝔖n) deviennent identiques si on se limite aux permutations paires (et c'est toujours aussi inintéressant : c'est {±1}×𝔄n qu'on peut aussi noter 2×𝔄n) ; il en va de même des deux derniers : on note 2·𝔄n (groupe d'ordre n!) la restriction de l'un ou l'autre de 2·𝔖n⁺ ou 2·𝔖n⁻ aux permutations ±[σ] avec σ paire.

↑Entry #2522 [older| permalink|newer] / ↑Entrée #2522 [précédente| permalien|suivante] ↑

↓Entry #2520 [older| permalink|newer] / ↓Entrée #2520 [précédente| permalien|suivante] ↓

(mardi)

Mathématiques discrètes et continues

(Pour l'explication du titre, voir cette vieille entrée.)

J'assistais tout à l'heure à une séance de présentation, pour les élèves de Télécom ParisPloum où j'enseigne, des différentes filières (=spécialisations) entre lesquelles ils doivent piocher pour leur deuxième année. (La première année est généraliste, et en seconde année ils doivent choisir essentiellement deux-parmi-N spécialisations.) À vrai dire, j'étais plus là pour écouter les questions des élèves et les réponses faites par mes collègues, qui s'en sortaient très bien et n'avaient pas trop besoin de mon aide ; mais c'est intéressant, ne serait-ce que sociologiquement, de savoir ce que nos élèves ont comme questions à poser, et éventuellement comme préconceptions, sur les enseignements qu'on leur propose.

L'une des filières où j'enseigne s'appelle MITRO comme Mathématiques, Informatique Théorique, et Recherche Opérationnelle : c'est un rassemblement légèrement hétéroclite de cours à dominance plus théorique ayant pour but de donner une culture générale utile, soit en complément d'autres filières, soit pour entrer dans un master en informatique ou en recherche opérationnelle ; j'y fais un cours de théorie des jeux dont j'ai déjà parlé. (J'enseigne aussi un cours sur les courbes algébriques dans une filière AC2Q comme Algèbre, Codage, Crypto, Quantique, et les deux filières ont une intersection assez importante dans leur population d'élèves.)

Et une des questions qui m'a frappée à laquelle mon collègue présentant MITRO a dû répondre à un bon nombre de reprises, portait sur le contenu des mathématiques. Ça ne m'avait pas tellement frappé les années précédentes, ou peut-être que je n'avais juste pas fait attention :

En fait, nos élèves ont une vision très étroite de ce que sont les mathématiques. Et on ne peut pas leur en vouloir : ils sortent (pour l'essentiel) des classes prépa françaises, où on leur a enseigné, au moins sous l'étiquette mathématiques, des maths qui se limitent essentiellement à deux choses, (1) de l'algèbre linéaire, (2) de l'analyse réelle classique, et depuis récemment un peu de (3) probabilités. En première année à Télécom, ils ont des cours de maths qui couvrent les probabilités et encore plus d'analyse (un peu d'analyse fonctionnelle, cette fois ; j'enseigne aussi dans le cadre de ce cours-là). Donc au final, pour eux, les maths, c'est des espaces vectoriels (réels ou complexes), des intégrales et des probas (essentiellement). Et ils nous demandent, soit en l'espérant soit en le craignant, s'il y a des choses comme ça dans la filière MITRO. La notion de maths discrètes leur est largement inconnue.

Mais ce qui est un peu ironique, c'est qu'en fait ils ont déjà fait des maths discrètes (par exemple, ils savent ce que c'est qu'un graphe, un arbre, ce genre de choses) : simplement, ils en ont fait, en prépa ou après, dans des cours étiquetés informatique. Et j'enseigne moi-même un cours sur les langages formels (cf. ici) qui, dans mon esprit, est clairement un cours de maths, mais qui est étiqueté informatique (ceci provoque d'ailleurs des malentendus dans l'autre sens, parce que j'en ai qui se plaignent qu'on ait besoin de raisonner).

Je suis de l'avis que l'informatique théorique, ainsi qu'une bonne partie de la physique théorique, fait partie des mathématiques. En fait, pour moi, les mathématiques ne se définissent pas par leur objet d'étude mais par leur méthode, c'est-à-dire le fait qu'on arrive à la vérité par un raisonnement déductif dont la rigueur se cherche dans l'aspect formel ou du moins formalisable ; par opposition, essentiellement, aux sciences expérimentales dont la méthode est inductive et la rigueur se cherche dans l'application méticuleuse d'un protocole expérimental. Il se trouve que cette distinction — qui n'exclut pas qu'il y ait des régions intermédiaires où on combine un raisonnement partiellement heuristique et des constatations expérimentales — est largement transverse à un domaine comme l'informatique, la physique ou l'astronomie, et je classifie donc l'informatique théorique comme étant à la fois des maths (pour la méthode) et de l'informatique (pour la finalité).

Mais peu importent les classifications. (Si vous trouvez que je dis des conneries ci-dessus, je n'ai pas vraiment l'intention de défendre ma position, je dis comment je pense spontanément les choses, mais fondamentalement je me fous un peu de savoir comment on place les frontières entre les domaines d'investigation du savoir humain.) Ce qui m'inquiète, c'est l'effet de myopie disciplinaire.

Que les classes prépa françaises n'enseignent essentiellement que de l'algèbre linéaire, de l'analyse réelle classique et des probabilités, je ne me sens pas spécialement fondé à le critiquer. À un certain niveau, j'aimerais bien qu'on y rencontre la notion de corps fini, mais je comprends qu'il y a plein de choix à faire, que tout le monde tire la couverture à soi, que c'est très politique, etc.

Mais ce que je trouve vraiment regrettable, quand je repense à l'entrée que je viens d'écrire où j'évoque l'idée que le grand public se fait des mathématiques (manipuler des gros nombres ou manipuler des grosses formules), c'est que des élèves qui en ont quand même avalé nettement plus que le grand public aient toujours une idée finalement toujours aussi étroite de ce que sont les mathématiques. C'est-à-dire que je trouve que, même si on n'a pas le temps d'enseigner ceci ou cela de précis, et même si « ça ne sert à rien » (or je ne crois pas que ça ne serve à rien), on doit quand même pouvoir trouver le moyen de faire un survol de ce que sont les branches, et comment elles se nomment, des mathématiques, toutes les mathématiques. (Disons au moins en se donnant comme but que ce ne soit pas une surprise d'apprendre qu'il y a des mathématiciens qui étudient les graphes et qui n'ont qu'un rapport extrêmement lointain avec l'informatique. Mais aussi pour pouvoir leur dire, voyez, ce qu'on va vous enseigner, c'est les parties anciennes de ce tout petit bout-là.) Je pense bien sûr la même chose des autres sciences qu'ils peuvent être amenés à étudier, même si j'ai l'impression — peut-être effet de ma propre myopie — que la « cartographie » des mathématiques est particulièrement mal connue.

↑Entry #2520 [older| permalink|newer] / ↑Entrée #2520 [précédente| permalien|suivante] ↑

↓Entry #2519 [older| permalink|newer] / ↓Entrée #2519 [précédente| permalien|suivante] ↓

(dimanche)

Réflexions décousues sur la vulgarisation mathématique

Bon, il faut peut-être que j'arrête d'intituler mes entrées quelques réflexions sur… ou réflexions décousues sur…, parce qu'à peu près tout ce que j'écris finit par rentrer dans cette forme. Mais j'aime bien me retrancher derrière cette sorte d'excuse quand je ne sais pas très bien à l'avance ce que je vais raconter et/ou que je n'ai pas envie d'essayer d'élaborer un plan. [Ajout : J'ai essayé de faire un plan a posteriori en insérant des intertitres à certains points dans cette entrée, peut-être que ça aide à la lire.]

☞ Vulgarisation à différents niveaux

La vulgarisation mathématique (et occasionnellement, physique) occupe une grande place dans ce blog. Enfin, déjà, il faut se demander ce que le terme vulgarisation recouvre au juste, vu que je parle rarement en faisant l'effort d'être compréhensible par un public complètement non-initié (i.e., Madame Michu — parce que ma maman en a marre d'être prise en exemple de la-personne-qui-ne-connaît-rien-aux-maths), mais je pense qu'il y a justement une place intéressante, et trop peu exploitée, pour toute forme de communication qui s'adresse à un public plus large que les spécialistes mais néanmoins plus étroit que le vulgum pecus, par exemple un scientifique d'un autre domaine, ou un enseignant du secondaire. (Le monde scientifique est tellement cloisonné[#] que les initiatives par lesquelles les biologistes et les informaticiens se tiendraient mutuellement au courant de leurs recherches, hors d'un cadre d'applications directes, sont extrêmement rares, et c'est même le cas entre algébristes et analystes ; et il en va semblablement entre enseignants-chercheurs dans le supérieur et enseignants du secondaire. Tout cela est vraiment triste.) Convenons d'appeler encore ça de la vulgarisation. Je ne sais pas si c'est exactement ça que j'essaie de faire, le niveau auquel je place mon exposition de tel ou tel concept mathématique dépend plus de mon inspiration du moment et de la difficulté du concept lui-même que de l'intention de viser tel ou tel public que je cerne, de toute façon, assez mal. Mais il est certain que j'écris des explications à ces niveaux assez variés[#2], et j'ose espérer qu'au moins une partie de ce que j'ai pu écrire au chapitre vulgarisation mathématique a été compréhensible par le très grand public et qu'au moins une partie a pu être intéressante pour d'autres matheux (et peut-être même que ces parties ont une intersection non-triviale, ce qui serait formidable). Bref.

[#] J'ai déjà plusieurs fois cité Giancarlo Rota à ce sujet : A leader in the theory of pseudo-parabolic partial differential equations in quasi-convex domains will not stoop to being understood by specialists in quasi-parabolic partial differential equations in pseudo-convex domains.

[#2] Enfin, j'ai toujours considéré ça comme évident, mais au moins une personne lisant mon blog (et que je ne dénoncerai pas) ne s'en était pas aperçu. Dès qu'il est question de maths, je ne comprends plus rien… — D'accord, mais est-ce que tu avais bien compris que parfois quand je parle de maths ce n'est pas censé être compréhensible par le grand public et parfois si ? — Hum… Là on peut vraiment considérer que c'est un échec.

☞ Mon intérêt pour la vulgarisation

Bref, je fais souvent de la vulgarisation mathématique, mais je n'ai jamais vraiment parlé de vulgarisation mathématique : pourquoi ça m'intéresse, pourquoi j'en lis, pourquoi j'en fais, etc.

Je suis tombé dans la marmite de la vulgarisation scientifique quand j'étais petit (avouons que mon papa m'a un peu poussé dans la marmite en question), par exemple à travers le livre Cosmos de Carl Sagan (tiré de la série du même nom), ou de One, Two, Three… Infinity de George Gamow (ça fait plus de trente ans que je ne l'ai pas lu, celui-là, je devrais sans doute y jeter à nouveau un œil pour voir ce qu'il contenait), ou encore The Emperor's New Mind de Penrose ainsi que (plus tard) Gödel, Escher, Bach de Hofstadter auquel le livre de Penrose est plus ou moins une réponse, ou enfin Les Trous noirs de Jean-Pierre Luminet.

Et je continue à apprécier la vulgarisation scientifique (en tout cas quand elle est bonne) à différents niveaux. Même quand je n'apprends rien sur le fond, ce qui est rarement le cas ne serait-ce que parce que les vulgarisateurs racontent de l'histoire des sciences en même temps que la science elle-même, j'apprends quelque chose de très important, qui est comment communiquer, justement, avec le grand public, ce qui est loin d'être évident, et d'autant moins évident qu'on parle d'un sujet abstrait comme la physique théorique ou les mathématiques. Une des difficultés de l'exercice est de trouver des analogies ou des images qui respectent le double impératif largement contradictoire d'être parlantes (c'est-à-dire compréhensibles mais aussi éclairantes) et correctes (c'est-à-dire qui évitent de simplifier tellement les choses que ça devient une bouillie de mots qui ne veulent plus rien dire) : c'est quelque chose de véritablement difficile, et j'essaie de retenir les bonnes analogies que je trouve pour pouvoir les resservir éventuellement. Et même quand il s'agit de quelque chose que je connais très bien, il y a toujours quelque chose à apprendre sur comment bien le résumer, comment souligner ce qui est le plus important, quoi mettre en lumière et quoi passer sous silence, etc. À titre d'exemple, le cosmologiste Sean Carroll est, à mon avis, un vulgarisateur extraordinaire, et cette petite série de cinq épisodes de trois ou quatre minutes chacun sur la direction du temps (s'adressant à des gens qui, quand même, ont une certaine culture scientifique générale) est un modèle à suivre de comment expliquer les choses clairement bien que rapidement (ou cet exposé, plus long et sans doute plus élémentaire, sur le même sujet).

Inversement, quand on écrit de la vulgarisation, on apprend toujours quelque chose sur ce sur quoi on écrit. Même quand on pense exposer quelque chose qu'on connaît parfaitement, et quel que soit le niveau auquel on se place, il y aura toujours quelque chose à apprendre, ou au moins à mieux comprendre, dans le processus d'explication. C'est une des raisons qui me pousse à me prêter à l'exercice (et plus généralement, à aimer enseigner), et je pense que cela devrait faire partie de n'importe quel travail de recherche.

☞ Pourquoi j'aime parler de trucs « vieux »

Il y a quand même une chose qui m'agace dans la vulgarisation, en tout cas comme certains la pratiquent, c'est la tendance à surreprésenter les progrès récents (dans le domaine scientifique considéré), voire, la recherche personnelle du vulgarisateur. Je comprends évidemment les raisons qui poussent à ça : il est gratifiant de parler de ce qu'on fait soi-même, et on a envie de montrer au grand public qu'on fait avancer la science, et ce qui se passe « sur le front ». Et inversement, le grand public a sans doute plus envie qu'on lui parle de la physique toute récente que de celle de Newton. L'ennui, c'est que pour bien faire comprendre la physique toute récente, il faut sans doute commencer par bien faire comprendre celle de Newton (puis celle de Maxwell, puis celle d'Einstein et celle de Schrödinger et Heisenberg… enfin, vous voyez l'idée). Forcément, dans le cadre de la vulgarisation, on va sauter des étapes, commettre des approximations, passer des choses sous silence, et peut-être ne faire qu'évoquer Newton pour dire directement des choses sur le boson de Higgs ou les ondes gravitationnelles ou la théorie des cordes ou que sais-je encore. C'est bien, et c'est normal. Mais il est quand même utile qu'il y ait aussi des gens qui vulgarisent Newton, et ce n'est pas forcément si évident que ça, et c'est vraiment utile parce que Newton est quand même bigrement pertinent dans la vie de tous les jours (certainement plus que les ondes gravitationnelles), et d'ailleurs ce serait sacrément utile dans le débat politique si le grand public connaissait un peu mieux la physique, disons, de Boltzmann (par exemple ce que j'en racontais ici). Mais je m'écarte un peu de la question de la vulgarisation pour m'aventurer dans celle de la culture générale scientifique (question sur laquelle j'aurais beaucoup à dire, mais je vais essayer de garder ça pour une autre fois).

Je ne suis pas spécialement tenté, moi, de vulgariser ma propre recherche[#3] (même en mettant de côté le fait que ma propre recherche papillonne dans tous les sens plutôt qu'elle ne progresse dans une direction bien définie). J'en ai déjà déçu plus d'un, comme ça, qui m'invitait à parler devant telle ou telle assistance (par exemple ici) et qui espérait plus ou moins que je parlerais de quelque chose d'un peu actuel : non, j'ai plutôt envie de parler d'objets ou de théories mathématiques qui sont bien connues depuis des dizaines et des dizaines d'années. Ne serait-ce que parce que plus c'est vieux, mieux c'est compris, et mieux on sait, entre autres, quelle est la bonne façon de voir et de présenter les choses. J'aime comparer les maths à un palais magnifique et incompréhensiblement gigantesque, à la structure à la fois labyrinthique et extraordinairement belle, — palais qu'on visite en étant totalement aveugle, si bien qu'on ne peut que tâtonner pour comprendre comment les salles sont agencées et quels bibelots précieux elles contiennent : si je dois emmener un groupe de touristes faire un tout petit tour du palais, je vais plutôt les emmener visiter les salles bien cartographiées que celles qu'on ne sait atteindre que par un chemin compliqué et qui sont peut-être encore en train d'être déterrées par les archéologues (hum, mes métaphores sont un peu mélangées, mais vous voyez l'idée).

[#3] Plus généralement, d'ailleurs, je constate empiriquement que les exposés scientifiques sont d'autant plus intéressants et agréables à écouter (à mon avis personnel à moi que j'ai) que l'orateur ne parle pas de ses propres travaux (c'est la règle au séminaire Bourbaki, mais j'aimerais que plus de séminaires adoptassent le même principe).

☞ Comment communiquer la beauté des mathématiques ?

C'est indiscutablement la beauté des mathématiques, et plus précisément la beauté de certains objets mathématiques, qui me motive à la fois pour faire des maths et pour communiquer autour des maths. La physique m'intéresse mais les maths font bien plus, elles m'émerveillent. J'ai déjà parlé ici et de deux de mes fascinations mathématiques les plus profondes (la symétrie et la « grandeur »), j'ai déjà plein de fois fait références à ces entrées, donc je ne vais pas revenir dessus. Mais étant moi-même envoûté par l'élégance de telle ou telle structure mathématique, j'ai envie de partager cette fascination, pas seulement à mes collègues mais aussi au grand public.

↑Entry #2519 [older| permalink|newer] / ↑Entrée #2519 [précédente| permalien|suivante] ↑

↓Entry #2518 [older| permalink|newer] / ↓Entrée #2518 [précédente| permalien|suivante] ↓

(vendredi)

Quelques notes sur la factorisation des entiers

J'ai donné mardi un exposé à des professeurs de classes préparatoires, dans le cadre d'un journée Télécom-UPS, sur la factorisation des entiers (l'idée était que je fisse un exposé général introductif sur le problème, qu'un de mes collègues donnât un exposé sur les courbes elliptiques et qu'un autre organisât un TP sur l'algorithme de Lenstra). Mes transparents ne sont sans doute pas très intéressants parce que je les ai écrits à la quatrième vitesse (quoi, le 15 mai c'est demain ? mais j'étais persuadé que c'était mercredi !), ils contiennent d'ailleurs du coup sans doute beaucoup d'erreurs ou d'approximations, et je les ai accompagnés d'énormément d'explications à l'oral ; mais à tout hasard, les voici.

↑Entry #2518 [older| permalink|newer] / ↑Entrée #2518 [précédente| permalien|suivante] ↑

↓Entry #2512 [older| permalink|newer] / ↓Entrée #2512 [précédente| permalien|suivante] ↓

(mardi)

Encore de jolies images quasipériodiques

Encore un peu d'art mathématique construit autour de l'élégance du nombre 7 et de la quasipériodicité. Cette fois-ci, je vais faire travailler votre navigateur plutôt que calculer les images moi-même (l'image qui suit, normalement, est animée et change de temps en temps ; sa périodicité est d'une semaine de 10 minutes et 04.8 secondes [correction () j'avais fait une erreur d'un facteur 1000 parce que JavaScript renvoie le temps en millisecondes et pas en secondes]) :

Jeu de couleurs : Échelle :

M'étant fatigué à programmer ça, j'avoue que j'ai maintenant un peu la flemme d'expliquer de quoi il s'agit (surtout que je ne suis pas sûr d'en avoir une idée si précise moi-même), et je suis un peu tenté de dire vous n'avez qu'à lire le source JavaScript, il n'est pas obfusqué. Mais pour dire quand même un peu d'où ça sort, je suis parti d'une jolie construction de pavages de Penrose décrite dans un article de Nicolaas Govert de Bruijn, Algebraic theory of Penrose's non-periodic tilings of the plane, I, Nederl. Akad. Wetensch. (=Indag. Math.) 43 (1981), 39–42 (notamment §4), et j'ai remplacé 5 par 7 un peu partout (on peut d'ailleurs changer seven = 7 par d'autres valeurs dans mon code et voir ce que ça fait, ça devrait marcher ou au moins marchouiller) et supprimé une hypothèse qui a sans doute un intérêt pour cet article mais pas vraiment s'il s'agit juste de faire de « jolies images ». (Cet article m'avait été présenté par un candidat au moment où j'étais examinateur aux TIPE à l'ENS. J'avais écrit du code à ce moment-là, mais je n'avais pas bien compris comment fabriquer quelque chose de symétrique, et par ailleurs je coloriais les morceaux de façon bizarre, donc ça ne donnait pas un résultat très beau ; j'y ai repensé en écrivant l'entrée précédente.)

Très sommairement, la construction est la suivante : on part de sept familles de droites parallèles régulièrement espacées dont les directions sont séparées des multiples de 2π/7 (dans un premier temps, on pourra imaginer que l'origine du plan est à mi-chemin entre deux droites dans chaque famille) : appelons ça une heptagrille. On fait l'hypothèse qu'il n'y a pas de points où trois droites différentes de l'heptagrille se coupent. Le pavage sera en quelque sorte dual de l'heptagrille, au sens où à chaque intersection de deux droites de l'heptagrille on va associer un losange du pavage (et chaque sommet du pavage est associé à une composante connexe du complémentaire de la réunion des droites de l'heptagrille). Pour calculer les coordonnées d'un point du pavage, on commence par attribuer des valeurs entières aux bandes délimitées par les droites de chaque famille de l'heptagrille, disons de façon que l'origine ait la valeur 0 : pour un point P « général » du plan où vit l'heptagrille (« général » c'est-à-dire non situé sur une droite) on a ainsi sept valeurs entières k₀,…,k₆, selon les bandes où il se situe, et on associe à P le point Φ(P) du plan complexe somme des kj·ζjζ=exp(2iπ/7) est une racine septième de l'unité ; si le point P est sur une droite, l'un des kj va prendre deux valeurs entières adjacentes au voisinage de P, et s'il est sur deux droites à la fois, on va avoir deux des kj qui prennent deux valeurs adjacentes : les quatre points associés par Φ (i.e., sommes des kj·ζj) sont alors les quatre sommets d'un losange du pavage. Ceci définit le pavage, qu'il est facile de construire en énumérant tous les points de croisement de droites de deux familles de l'heptagrille. (La forme du losange est déterminée par l'écart entre les angles des deux droites qui s'intersectent au point auquel il est associé.) Pour muter le pavage, on peut décaler les différentes familles de droites constituant l'heptagrille (si le décalage est le même pour chaque famille, la symétrie est conservée).

Bon, la description ci-dessus est certainement assez obscure, mais je n'ai pas le temps d'expliquer mieux. Par ailleurs, il y a certainement quelque chose d'intelligent à dire qui fait intervenir les mots système de racines de type A et plan de Coxeter, mais là, tout de suite, comme ça, je ne vois pas bien.

Ajout () : J'ai ajouté un sélecteur pour afficher ça en couleurs (les couleurs sont choisies d'après l'orientation des losanges). Mais je continue à préférer nettement la version en teintes de gris (choisies d'après la forme des losanges). Nouveaux ajouts : J'ai aussi ajouté de quoi changer l'échelle, et de quoi se déplacer (cliquer+déplacer la souris), voir aussi l'entrée suivante.

↑Entry #2512 [older| permalink|newer] / ↑Entrée #2512 [précédente| permalien|suivante] ↑

↓Entry #2511 [older| permalink|newer] / ↓Entrée #2511 [précédente| permalien|suivante] ↓

(lundi)

Je fais de jolies images avec la transformée de Fourier

[Transformée de Fourier d'un carré]

[Transformée de Fourier d'un hexagone]

[Transformée de Fourier d'un octogone]

[Transformée de Fourier d'un décagone]

[Transformée de Fourier d'un dodécagone]

[Transformée de Fourier d'un tétradécagone]

[Transformée de Fourier d'un hexadécagone]

[Transformée de Fourier d'un octadécagone]

[Transformée de Fourier d'un icosagone]

[Transformée de Fourier d'un doicosagone]

[Transformée de Fourier d'un tétraicosagone]

Pour une fois, cette entrée mathématique n'a aucun autre but que de « faire joli ». Il y a peut-être des choses intéressantes à dire à ce sujet (et ces choses intéressantes font peut-être intervenir des mots comme quasi-cristal ou pavage de Penrose), mais je n'ai pas vraiment envie d'y réfléchir.

Les images ci-contre à droite (faites défiler vers le haut et vers le bas, ou voyez ici sur Imgur) représentent les transformée de Fourier de polygones réguliers, et plus exactement des n-gones réguliers pour n pair allant de 4 à 24. Elles sont représentées par des nuances de gris pour les valeurs positives (où 0=noir et n=blanc) et des nuances de bleu pour les valeurs négatives (où 0=noir et −n=bleu intense). Ce que j'appelle transformée de Fourier d'un n-gone régulier (ou plus exactement, des sommets du polygone — je ne trouve pas de tournure qui ne soit pas invraisemblablement lourde), c'est la transformée de Fourier d'une somme de n distributions δ, l'une centrée en chaque sommet du n-gone (le n-gone étant lui-même centré à l'origine). Plus concrètement, la fonction tracée est donc la somme de n ondes planes (toutes en phase à l'origine) partant dans chacune des n directions régulièrement espacées autour de l'origine :

k=0n1 exp ( 2iπ ( cos(2kπn) x + sin(2kπn) y ) )

(Ou, pour les navigateurs pourris qui ne gèrent pas le MathML : ∑k=0n−1exp(2iπ·(cos(2kπ/nx+sin(2kπ/ny)).) Pour n pair, ceci est bien une fonction réelle, et elle possède une symétrie de rotation d'ordre n autour de l'origine. Contrairement à l'impression qu'on peut avoir, elle n'est pas périodique (sauf dans les cas « cristallographiques » n=4 et n=6, qui ne sont pas franchement passionnants), seulement quasi-périodique. Il n'est pas concevable une seule seconde que je sois le premier à mettre de telles images en ligne mais, bizarrement, je ne trouve pas comment d'autres gens ont pu les appeler.

On pourra noter que quand n tend vers l'infini, la fonction (correctement renormalisée) tend (en un certain sens, que je n'ai vraiment pas envie de chercher à préciser) vers une fonction de Bessel de première espèce J de la distance à l'origine : c'est ce qu'on commence à voir par le jeu d'anneaux concentriques autour de l'origine pour n grands.

Bon, enfin, ce qui importe surtout c'est que ce soit visuellement plaisant, et je trouve que ça l'est.

Comme la fonction n'est pas périodique, ça pourrait être intéressant (surtout pour n modérément grand) d'en faire un « explorateur » interactif en JavaScript, où on pourrait se déplacer dessus, zoomer ou dézoomer, et ce serait calculé en temps réel. Mais j'avoue que je n'ai pas la patience de programmer ça.

En revanche, pour ceux qui trouvent que mes images 2D ci-dessus sont trop statiques, je peux vous proposer une version 3D, qui consiste à faire la transformée de Fourier d'un polyèdre régulier et de la « trancher » en tranches 2D (c'est-à-dire, d'afficher des valeurs dans des plans parallèles les uns aux autres) selon une direction de plan qui présente une symétrie maximale (plan de Coxeter) : j'ai mis ça sur YouTube, et vous pouvez voir la transformée de Fourier d'un icosaèdre régulier et celle d'un dodécaèdre régulier (le plan de Coxeter fournit une symétrie d'ordre 10 : c'est la direction de plan parallèle à deux faces opposées quelconques du dodécaèdre). Là aussi, j'ai du mal à comprendre pourquoi une recherche Google de Fourier transform of dodecahedron ou variantes ne donne essentiellement rien (à part des choses que j'ai moi-même calculées, dont une vieille version des mêmes vidéos) : même si ça ne doit servir qu'à « faire joli », c'est pourtant quelque chose d'éminemment naturel à regarder, il me semble.

(J'ai vaguement imaginé, aussi, calculer la transformée de Fourier de polygones et polyèdres pleins, et aussi de leurs facettes et arêtes, mais outre que ce serait excessivement pénible à calculer, je pense que ce serait très décevant, en fait : ça ressemblerait sans doute à peu près la même chose mais en s'atténuant très vite quand on s'écarte de l'origine.)

↑Entry #2511 [older| permalink|newer] / ↑Entrée #2511 [précédente| permalien|suivante] ↑

↓Entry #2507 [older| permalink|newer] / ↓Entrée #2507 [précédente| permalien|suivante] ↓

(mercredi)

Le progrès récent sur le problème de Hadwiger-Nelson

J'ai déjà parlé à plusieurs reprises du problème de Hadwiger-Nelson sur ce blog (ici en général, et ici pour mes malheurs personnels liés à ce problème), et il faut que j'en reparle puisqu'il y a eu un progrès considérable. Le problème de Hadwiger-Nelson a ceci de sympathique que c'est un problème de mathématique de niveau recherche (au sens empirique où il y a, effectivement, des mathématiciens professionnels qui ont fait de la recherche dessus et publié des choses à son sujet) dont un bon élève de primaire peut comprendre l'énoncé, un bon collégien peut comprendre les meilleures bornes connues jusqu'à la semaine dernière, et un bon lycéen peut les trouver lui-même. (Enfin, quelque chose comme ça.) Je rappelle l'énoncé :

Trouver le plus petit nombre χ de couleurs nécessaires pour colorier le plan de manière à ce qu'il n'y ait jamais deux points situés à distance 1 l'un de l'autre et qui aient la même couleur.

Ce χ s'appelle le nombre chromatique du plan ou nombre [chromatique] de Hadwiger-Nelson. Jusqu'à la semaine dernière, tout ce qu'on savait était que 4≤χ≤7.

Le fait que χ≤7, c'est-à-dire que sept couleurs suffisent, est montré par un coloriage explicite (d'un pavage du plan par des hexagones) avec 7 couleurs, coloriage qui est représenté par le dessin ci-contre à droite que je recopie de ma précédente entrée sur le sujet ; l'unité de longueur est figurée par le trait noir dans le coin en haut à gauche de la figure : quel que soit l'endroit où on le place et la manière dont on le tourne, les deux extrémités tombent toujours sur deux couleurs différentes ; et le problème est, donc, de savoir si on peut faire ça avec strictement moins de sept couleurs.

La minoration χ≥4 (c'est-à-dire qu'au moins quatre couleurs sont nécessaires), elle, est démontrée par un graphe fini tout à fait explicite, appelé Moser's spindle (fuseau de Moser ?) : je le recopie lui aussi de mon entrée précédente (ci-contre à gauche), toutes les arêtes représentées ont la même longueur (l'unité de longueur), et il n'est pas possible de colorier ses sommets avec seulement trois couleurs de façon que deux sommets reliés par une arête ne soient jamais de la même couleur. (En effet, si on ne dispose que de trois couleurs, chaque triangle équilatéral de côté 1 [du graphe] doit avoir un sommet de chaque couleur, du coup, dans le graphe représenté à gauche, chacun des deux sommets en haut à droite a la même couleur que celui en bas à gauche, donc ils ont la même couleur l'un que l'autre, or ils sont reliés par une arête.) Bref, dans tout coloriage du plan avec 3 couleurs, il y en a deux situés à distance 1 qui ont la même couleur.

Si vous n'aimez pas le fuseau de Moser, vous pouvez aussi utiliser le graphe de Golomb, représenté ci-contre à gauche (lui n'était pas dans l'entrée précédente, il faut bien que je m'embête un peu à faire du SVG et à calculer que les coordonnées d'un point valent (1,√11)/6), qui est plus joli et plus symétrique. Comme le fuseau de Moser, il n'est pas coloriable avec trois couleurs : si on a seulement trois couleurs, une fois qu'on en donne une au point central, les six points à distance 1 de lui doivent partager les deux autres couleurs en alternance, et notamment les trois qui sont reliés au triangle « oblique » sont de la même couleur, ce qui ne laisse que deux couleurs pour colorier ce dernier.

Bref, la minoration vient de graphes finis tout à fait explicites.

En fait, on sait à cause d'un théorème de compacité (que les théoriciens des graphes appellent le théorème d'Erdős et de Bruijn, et que les logiciens considèrent comme une conséquence immédiate du théorème de compacité du calcul propositionnel) que toute minoration sur χ s'obtient par un graphe fini, c'est-à-dire que χ est aussi la plus grande valeur possible du nombre de couleurs d'un graphe de ce genre. Donc on peut reformuler le problème de Hadwiger-Nelson de la façon suivante :

Trouver le plus petit nombre χ de couleurs nécessaires pour colorier un nombre fini quelconque de points du plan de manière à ce qu'il n'y ait jamais deux points situés à distance 1 l'un de l'autre et qui aient la même couleur.

(Le « fuseau de Moser » ci-dessus étant à comprendre comme l'ensemble de sept points qui sont les sommets tracés : on ne peut pas colorier cet ensemble de sept points avec trois couleurs donc χ≥4.)

Jusqu'à la semaine dernière, donc, c'est tout ce qu'on savait. Toute recherche sur ce problème a porté sur des analogues ou des généralisations (nombre chromatique de l'espace, nombre chromatique du plan à coordonnées dans ceci-cela, nombre chromatique fractionnaire, ce genre de choses).

Voilà que, dimanche, un certain Aubrey (David Nicholas Jasper) de Grey a mis un papier sur l'arXiv prouvant que χ≥5 : i.e., dans tout coloriage du plan avec 4 couleurs, il y en a deux situés à distance 1 qui ont la même couleur. (Je l'ai appris par un commentaire sur ma première entrée au sujet du problème.)

C'est assez sidérant pour plusieurs raisons. D'abord parce que c'est quand même un problème sur lequel on est restés coincés pendant environ 50 ou 60 ans (l'histoire du problème est elle-même assez tarabiscotée, mais il semble que Nelson l'ait imaginé dans les années '50 et qu'il — le problème — soit devenu célèbre une petite dizaine d'années plus tard). Mais aussi parce le de Grey auteur du papier n'est pas mathématicien (ou en tout cas, pour éviter de se mouiller sur ce que mathématicien veut dire, il n'est pas mathématicien de profession, et ne semble pas avoir fait de contributions aux mathématiques avant ça) ; il est « biogérontologue », connu pour ses positions contre le vieillissement, et considéré par certains comme un gourou voire un crackpot (le fait qu'il ressemble à Gandalf doit aider ce genre de préjugés). Il ne faut pas croire sur parole n'importe quel papier mis sur l'arXiv surtout quand il annonce un résultat « spectaculaire », mais, en l'occurrence, (1) le papier est bien écrit (les arguments sont rapides mais clairs et écrits dans le style habituel dans lequel on écrit les mathématiques), et de toute façon (2) une fois connu le graphe construit, il est modérément facile de vérifier le résultat par ordinateur, des gens ont déjà vérifié qu'un des graphes décrits par de Grey est réalisable avec distance 1[#] et (au moyen d'un SAT-solver) n'est pas 4-coloriable[#2], donc le résultat principal est certifié valable (nonobstant d'éventuelles erreurs très mineures dans la description).

[#] Ici et dans la suite, j'emploie le terme réalisable avec distance 1 pour dire que le graphe est réalisable comme un ensemble de points dans le plan de sorte que toutes les arêtes aient longueur 1. (On peut éventuellement demander que, réciproquement, chaque paire de points à distance 1 donne effectivement une arête du graphe ça ne changera rien puisque ajouter des arêtes ne peut qu'augmenter le nombre chromatique.)

[#2] Ici et dans la suite, j'emploie le terme k-coloriage pour signifier, bien sûr, un coloriage avec k couleurs de façon que deux sommets reliés par une arête (i.e. à distance 1) ne soient jamais de la même couleur ; et k-coloriable pour dire qu'un k-coloriage existe.

Mais pour être épatant, ce résultat est aussi un peu frustrant, je vais essayer de dire pourquoi.

Quand j'avais commencé à réfléchir au problème de Hadwiger-Nelson, ma première intuition était que χ=7 était sans l'ombre d'un doute la bonne valeur, et qu'il s'agissait juste de trouver de bons graphes, et que, si on ne les connaissait pas, c'était juste qu'on n'avait pas cherché assez fort, notamment avec des ordinateurs. (Cette intuition initiale est donc confirmée par le résultat de de Grey, mais je ne vais pas dire ha ha, j'avais raison, puisque, comme je vais l'expliquer, j'ai ensuite changé d'avis.) En gros, ce qui fait « marcher » le « fuseau de Moser » représenté ci-dessus est qu'on a le triangle équilatéral dont les 3-coloriages sont très peu nombreux, donc suffisamment rigides pour qu'on arrive à les combiner pour fabriquer un graphe plus gros qui n'est pas 3-coloriable. L'espoir, ensuite, serait que les 4-coloriages du fuseau (ou du graphe de Golomb) soient assez rigides pour qu'on arrive à combiner plusieurs fuseaux pour former un graphe qui ne soit pas 4-coloriable. Et qu'on puisse monter encore un coup pour former un graphe qui ne soit pas 5-coloriable, puis un qui ne soit pas 6-coloriable, ce qui démontrerait χ=7.

Plus tard, j'étais beaucoup moins convaincu de χ=7 : la raison est que j'ai essayé de réfléchir à comment on pourrait construire des graphes réalisable avec distance 1 et qui ne soient pas 4-coloriables, et j'ai eu l'impression de buter contre des problèmes insurmontables. Comme je le dis au paragraphe précédent, on peut essayer de combiner des fuseaux de Moser (ou des graphes de Golomb) et essayer de limiter leurs possibilités de 4-coloriages jusqu'à toutes les tuer. Mais j'ai un peu essayé et je m'y suis salement cassé les dents : tout me semblait suggérer que plus on augmente le nombre de sommets plus les possibilités de 4-coloriages se multiplient, plus vite qu'on arrive à les tuer en ajoutant des arêtes. Pour être un peu moins vague, j'ai eu l'impression que la seule façon exploitable de fabriquer des graphes réalisables avec distance 1 dans le plan est de prendre deux graphes G₁,G₂ déjà réalisés avec distance 1 et utiliser une isométrie plane sur G₂ (en faixant G₁) pour imposer des identifications de sommets ou fabriquer des arêtes, mais pour ça, on n'a que très peu de degrés de liberté (le groupe des isométries planes est de dimension 3), donc, sauf coïncidences, on ne peut ajouter essentiellement que trois arêtes (ou une identification de sommet et une arête) ; j'ai eu l'impression que « sauf coïncidence », tout ceci devrait conduire à une borne sévère sur la dégénérescence des graphes réalisables avec distance 1, donc sur leur nombre chromatique ; en fait, qu'ils devaient être des graphes de Laman — « sauf coïncidence », donc, mais je ne voyais pas comment fabriquer des « coïncidences » intéressantes. Bref, tout ça pour dire que j'ai essayé justement l'approche que de Grey fait marcher, que je n'ai pas du tout réussi à en faire quoi que ce soit, et que je me suis même mis à penser que ça ne pouvait pas marcher « sauf coïncidence » mais que ce serait extraordinairement difficile de prouver l'inexistence de telles « coïncidences » ou, a contrario, d'en fabriquer. Du coup, j'ai commencé à douter que χ=7 soit la bonne valeur (je ne prétends pas que j'étais convaincu que χ=4, mais que ma foi dans le fait que χ=7 s'était envolée jusqu'à ce qu'on me signale la trouvaille de de Grey).

Ajout : Un autre de mes espoirs était qu'on puisse chercher à extraire un graphe de nombre chromatique au moins 5 (voire 6, voire 7) comme un sous-graphe de l'analogue de Hadwiger-Nelson sur un corps fini, disons le graphe (ℤ/pℤ)² avec une arête entre (x₁,y₁) et (x₂,y₂) lorsque (x₂−x₁)² − (y₂−y₁)² = 1. (La motivation étant que si un graphe plan est réalisable avec distance 1, il est aussi réalisable à coordonnées algébriques, ces coordonnées de scindent modulo un ensemble de densité >0 de nombres premiers p, donc imposent la même inégalité sur les nombres chromatiques des (ℤ/pℤ)² pour la relation que je viens de dire.) Évidemment, cet espoir était naïf — mais vu que les coordonnées du graphe calculé par de Grey sont dans des extensions assez petites de ℚ comme je le soupçonnais, ce n'était pas complètement stupide non plus.

C'est dire que je suis surpris par le tour de force. La lecture du papier lui-même est un peu décevante, cependant : il y a un mélange de raisonnements « à la main » sur les 4-coloriages de graphes de plus en plus grands réalisables avec distance 1, et de vérifications par ordinateur (avec différentes astuces pour rendre la vérification plus gérable), mais au final je ne suis pas vraiment Éclairé sur la manière dont il arrive à obtenir suffisamment d'arêtes eu égart au nombre de sommets (le graphe réalisable avec distance 1 et non 4-coloriable dont Dustin Mixon publie le fichier de données sur son blog — revoici le lien — a 1585 sommets et 7909 arêtes), ou sur la raison pour laquelle je m'étais trompé en pensant qu'il était très difficile d'obtenir une grande dégénérescence.

Ce qui est frustrant, c'est que ce progrès rend le problème de Hadwiger-Nelson beaucoup moins intéressant. Peut-être que la presse généraliste va s'en emparer (et raconter des conneries), et il va sans doute y avoir des efforts renouvelés pour construire des graphes plus simples prouvant χ≥5 (cf. ici) ou pour pousser jusqu'à χ≥6 voire χ=7, mais mathématiquement, le problème a un peu perdu de sa beauté. Pourquoi ? Déjà parce qu'on ne peut plus donner ça comme un exemple de problème où l'état de l'art correspond à ce qu'un lycéen peut trouver tout seul. Mais surtout je suis maintenant revenu à mon intuition première, et complètement convaincu d'une part que χ=7 et d'autre part que des graphes le démontrant peuvent se construire avec des techniques de type « dupliquer et identifier » et des recherches sur ordinateur (à supposer qu'ils ne soient pas trop grands). Il aurait été beaucoup plus intéressant de chercher à montrer que certains graphes ne peuvent pas exister que de chercher à les exhiber.

Après, on peut s'intéresser à toutes sortes de problèmes adjacents. Je soumets notamment la question suivante, ou problème de Hadwiger-Nelson lorentzien (que j'espérais rendre publique via cette note, mais comme cette dernière est partie à la poubelle, personne n'est au courant de ce problème) [ajout : cf. cette question sur MathOverflow] :

Trouver le plus petit nombre χL de couleurs nécessaires (ou bien ∞ si aucun nombre fini ne suffit) pour colorier le plan de manière à ce qu'il n'y ait jamais deux points (t₁,x₁) et (t₂,x₂) situés à intervalle de Lorentz 1 l'un de l'autre, c'est-à-dire (t₂−t₁)² − (x₂−x₁)² = 1, et qui aient la même couleur.

(Autrement dit, on remplace les cercles de rayon 1 — translatés de {x²+y²=1} — dans le problème de Hadwiger-Nelson par des hyperboles translatées de {t²−x²=1}, représentant, si on veut, un intervalle d'espace-temps. Il y a beaucoup de similarités, parce que le groupe des isométries lorentziennes, comme le groupe des isométries euclidiennes, et de dimension 3. À la différence du problème de Hadwiger-Nelson euclidien, dans le problème lorentzien les graphes réalisables avec intervalle 1 sont naturellement orientés, par la valeur de la coordonnée t ; et on peut se convaincre qu'il n'existe pas de triangle ; comme il existe néanmoins des cycles d'ordre impair, on a quand même χL≥3.)

Je conjecture que χL=∞ (en tout cas, je ne sais montrer aucune borne supérieure sur χL). Le problème semble plus dur que Hadwiger-Nelson euclidien, car il ne semble pas exister de coloriage évident avec un nombre fini de couleurs, mais a contrario, si on veut prouver χL=∞, il faudra construire toute une famille de graphes finis.

Ajout : Je devrais mentionner qu'une des raisons de s'intéresser à χL est que l'analogue complexe du nombre de Hadwiger-Nelson, c'est-à-dire le nombre chromatique χC du graphe ℂ² avec une arête entre (x₁,y₁) et (x₂,y₂) lorsque (x₂−x₁)² − (y₂−y₁)² = 1, majore à la fois χ (euclidien) et χL (lorentzien), et qu'il est lui-même majoré par le χ de ℝ⁴ pour la métrique de signature indéfinie (++−−) (c'est-à-dire le nombre chromatique du graphe ℝ⁴ avec des arêtes définies par des hyperboloïdes translatés de {t²+u²−v²−w²=1}). Je conjecture à plus forte raison que χC=∞, et en fait c'est surtout ça que je trouve intéressant (parce que c'est un problème purement algébrique).

↑Entry #2507 [older| permalink|newer] / ↑Entrée #2507 [précédente| permalien|suivante] ↑

↓Entry #2503 [older| permalink|newer] / ↓Entrée #2503 [précédente| permalien|suivante] ↓

(vendredi)

Jouons maintenant avec le groupe de Weyl de F₄ parce que c'est plus facile

Cette entrée est la petite sœur de la précédente : après avoir écrit cette dernière, je me suis rendu compte (et on me l'a par ailleurs fait remarquer dans les commentaires) qu'il y a une version plus simple de ce dont j'y parlais et que j'aurais pu évoquer. Du coup, je vais essayer de le faire ici, en utilisant massivement le copier-coller et le recherche-remplacement. Ce que je ne sais pas, c'est s'il vaut mieux lire cette entrée-ci, ou la précédente, ou les deux en parallèle ou dans un certain ordre (bon, la réponse est peut-être bien « aucune des deux »).

Note : Principales modifications systématiques par rapport à l'entrée précédente : 8→4, E₈→F₄, D₈→B₄, 696 729 600 → 1152, et (0,1,2,3,4,5,6,23) → (½,3⁄2,5⁄2,11/2) ; il n'y a que trois vecteurs dans ma liste finale au lieu de 135 ; les contraintes de parité de changements de signes disparaissent (et du coup trouver un représentant dominaint pour W(B₄) consiste juste à passer aux valeurs absolues et à trier) ; l'opération de soustraire à chacune des huit composantes le quart de la somme de toutes devient soustraire à chacune des quatre composantes la moitié de la somme de toutes. Mais il y a quelques autres différences par ci par là, comme le fait que le système de racines est un tout petit peu plus compliqué à définir (c'est bien la seule chose qui se complique). ⁂ Ah, et puis sinon j'ai un problème typographique, qui est de savoir comment représenter agréablement des demi-entiers : il y a un symbole magique ½ pour un demi, qui est bien pratique parce que ça apparaît souvent, pour trois demis et cinq demis on peut utiliser le U+2044 FRACTION SLASH et écrire 3⁄2 et 5⁄2 ce qui si vous avez la bonne police apparaîtra peut-être comme une jolie fraction ; mais pour 11/2 je ne peux pas vraiment faire mieux qu'avec un bête U+002F SOLIDUS, parce que si je mets U+2044 FRACTION SLASH à la place, la sémantique est celle de (et ça apparaîtra exactement comme ça sous certaines polices), soit un-et-demi. Du coup, j'ai le choix entre cette écriture (½, 3⁄2, 5⁄2, 11/2) qui est bien moche, ou bien écrire (1/2, 3/2, 5/2, 11/2) mais alors il y a à la fois du ½ et du 1/2 pour le même nombre, c'est bizarre ; et si j'écris 1/2 partout, le vecteur (1/2, 1/2, 1/2, 1/2) est quand même moins lisible que (½, ½, ½, ½). Remarquez, je pourrais écrire 1½ pour trois demis et 2½ pour cinq demis, mais les matheux détestent ça en général (vu que 2·½ c'est 1 et pas 5/2). Pfff, que c'est pénible, les petites crottes de ragondin.

Partons de quatre nombres (= un élément de ℝ⁴) ; pour que ce que je raconte ne suppose aucune connaissance mathématique particulière, je précise que j'appellerai ça un vecteur et j'appellerai composantes du vecteur les quatre nombres en question. Par exemple (1, 0, 0, 0), ou bien (½, 3⁄2, 5⁄2, 11/2) sont des vecteurs avec lesquels on va pouvoir jouer (ces exemples vont être intéressants pour la suite ; et oui, c'est bien un 11/2 que j'ai écrit à la fin, bear with me, ce n'est pas une blague dans le style quel est le quatrième nombre qui complète la suite : ½, 3⁄2, 5⁄2… ?c'est évidemment 11/2). Maintenant, à partir de ce vecteur, imaginons qu'on ait le droit de faire, autant de fois qu'on veut, et dans n'importe quel ordre, les opérations très simples suivantes :

  • permuter ses composantes — c'est-à-dire les réordonner — de n'importe quelle manière (par exemple, on peut transformer (½, 3⁄2, 5⁄2, 11/2) en (3⁄2, 11/2, 5⁄2, ½), ce sont les mêmes nombres écrits dans un ordre différent),
  • changer le signe — c'est-à-dire transformer en leur opposé, remplacer moins par plus et vice versa — d'un nombre quelconque des composantes (par exemple, on peut transformer (½, 3⁄2, 5⁄2, 11/2) en (½, −3⁄2, −5⁄2, −11/2), j'ai changé le signe de trois composantes),
  • soustraire à chacune des quatre composantes la moitié de la somme de toutes (par exemple, ceci transforme (½, 3⁄2, 5⁄2, 11/2) en (−9⁄2, −7⁄2, −5⁄2, ½) : la somme des nombres était (½)+(3⁄2)+(5⁄2)+(11/2)=10 donc j'ai soustrait 5 à chacun).

Voilà qui n'est pas bien compliqué. Pour fixer la terminologie les opérations des deux premiers types que je viens de dire seront appelées opérations de W(B₄) tandis que les opérations des trois types seront dites opérations de W(F₄) (je n'essaye pas du tout de définir ce que c'est que W(B₄) ou W(F₄), en tout cas pas pour le moment, ce sont juste des termes à considérer comme un bloc).

Les opérations de W(B₄) sont assez faciles à comprendre, en réfléchissant un peu on arrive assez facilement à voir ce qu'on peut faire avec (une description plus précise sera donnée plus bas, notamment, de quand on peut passer d'un vecteur à un autre par ces opérations). Celles de W(F₄), c'est-à-dire si on permet la troisième opération que j'ai dite, sont déjà plus mystérieuses mystérieuses : je vais donner quelques exemples ci-dessous ce qu'on peut faire avec.

La question générale est, que peut-on atteindre en appliquant les règles qui viennent d'être dites ? Autrement dit, partant d'un certain vecteur initial, quels vecteurs va-t-on pouvoir fabriquer avec les opérations qui viennent d'être dites (et combien y en a-t-il) ?

Pour prendre un exemple vraiment idiot, si le vecteur d'origine était (0, 0, 0, 0), on ne va pas très loin, il reste identique à lui-même sous l'effet de n'importe laquelle des opérations que j'ai décrites, et donc c'est la seule chose qu'on pourra atteindre.

Si le vecteur de départ est (1, 0, 0, 0), les opérations de W(B₄) (i.e., celles les deux premiers types) permettent de le transformer en n'importe quel vecteur ayant une composante égale à +1 ou −1 et les trois autres nulles, ou en abrégé un vecteur du type (±1, 0, 0, 0) (cela fait 4×2=8 vecteurs si on compte bien) ; la troisième opération transforme (1, 0, 0, 0) en (½, −½, −½, −½), et de là avec les opérations de W(B₄) on peut fabriquer les différents vecteurs (±½, ±½, ±½, ±½) dont toutes les composantes valent +½ ou −½ ; cela fait 2⁴=16 vecteurs de cette forme, soit 8+16=24 vecteurs : il se trouve (il faut le vérifier mais ce n'est pas difficile) que c'est tout ce qu'on obtient de la sorte : 24 vecteurs et pas plus. Ces 24 vecteurs portent le nom de racines courtes de F₄ (là aussi, je ne vais pas chercher à définir ce que ça veut dire, en tout cas pas aujourd'hui).

Pour donner un autre exemple, si le vecteur de départ est (1, 1, 0, 0), les opérations de W(B₄) permettent de le transformer en n'importe quel vecteur du type (±1, ±1, 0, 0) (deux composantes égales à +1 ou −1, les deux autres nulles : cela fait 6×4=24 vecteurs), et la troisième opération ne fait, cette fois, rien de nouveau. Ces 24 vecteurs portent le nom de racines longues de F₄ ; et réunies aux 24 vecteurs définis au paragraphe précédent, on obtient 48 vecteurs appelés système de racines de F₄ (c'est là essentiellement le seul point sur lequel F₄ est plus compliqué que E₈ défini à l'entrée précédente : il y a des racines courtes et longues alors que dans E₈ il n'y a qu'une seule longueur).

Je peux donner d'autres exemples. Si on part de (1, 1, 1, 0), on va pouvoir atteindre 96 vecteurs différents par les opérations de W(F₄) : il y a les 32 vecteurs du type (±1, ±1, ±1, 0) avec des signes quelconques (et un emplacement quelconque du 0), et les 64 vecteurs du type (±3⁄2, ±½, ±½, ±½) avec des signes quelconques (et un emplacement quelconque du 3⁄2), ce qui fait 32+64=96 vecteurs au total. Si on part de (2, 1, 1, 0), on peut aussi atteindre 96 vecteurs différents (ce sont juste ceux qui s'obtiennent déjà par les opérations de W(B₄), c'est-à-dire (±2, ±1, ±1, 0) avec des signes quelconques et une permutation quelconque des composantes). Si on part de (2, 1, 0, 0), on peut atteindre 144 vecteurs différents (les 48 du type (±2, ±1, 0, 0) et les 96 du type (±3⁄2, ±3⁄2, ±½, ±½)).

Mais dans le « cas général » (disons, celui qui se produit avec probabilité 1 si notre vecteur initial a été tiré au hasard, ou bien si on est parti de (½, 3⁄2, 5⁄2, 11/2)), on va atteindre exactement 1152 vecteurs. (En fait, la condition pour que ça soit le cas n'est pas très compliqué : il est nécessaire et suffisant, pour que cela se produise, que les quatre composantes du vecteur initial soient toutes non nulles, deux à deux distinctes, qu'il n'y en ait pas deux qui soient opposées, et qu'il n'y en ait pas non plus un certain nombre dont la somme soit égale à la somme des autres.) Et dans absolument tous les cas, le nombre de vecteurs qu'on peut atteindre sera fini, et sera même un diviseur de ce nombre maximal qu'est 1152.

(Il y a d'ailleurs exactement 16 cas possibles entre le cas le plus spécial qu'est (0, 0, 0, 0) et qui donne un seul vecteur atteignable et le cas le plus général qui en donne 1152. Mais je préfère rester vague sur ce que j'entends par un cas possible, parce que ce n'est pas vrai que chacun de ces cas donne forcément un nombre de vecteurs atteints différents. Les nombres de vecteurs atteignables possibles sont : 1, 24, 96, 144, 192, 288, 576 et 1152)

↑Entry #2503 [older| permalink|newer] / ↑Entrée #2503 [précédente| permalien|suivante] ↑

↓Entry #2502 [older| permalink|newer] / ↓Entrée #2502 [précédente| permalien|suivante] ↓

(mercredi)

Jouons avec le groupe de Weyl de E₈ et cherchons la logique

J'ai raconté plein de fois dans ce blog (généralement je fais référence à cette entrée-là, mais c'est un thème récurrent, et de toute façon je radote) à quel point je suis fasciné par la symétrie et les structures combinatoires et toujours à la recherche de nouvelles façons de faire apparaître ou de représenter des objets mathématiques que je trouve remarquables. (Tiens, je n'ai pas encore parlé de mon jeu de cartes faussement divinatoires basé sur la combinatoire des 27 droites sur une surface cubique ? Faites-moi penser à vous montrer ça, un jour.) Je voudrais essayer ici de parler de façon extrêmement élémentaire un de mes objets préférés (il s'agit du groupe de Weyl de E₈, mais chut ! je veux éviter les mots barbares) pour arriver à une sorte de petite devinette, dont je n'ai pas la réponse, sur le mode « quelle est la logique dans les nombres suivants ? ».

Avertissement : La présentation qui suit risque d'être un peu irritante pour les mathématiciens — ou d'ailleurs pour des non-mathématiciens — parce que je vais faire tout un tas d'affirmations sans aucune sorte de justification, ce qui est normal pour de la vulgarisation, mais, pire, de façon peut-être gratuitement mystifiante ou à l'encontre de l'ordre et de la présentation logiques des choses. Désolé pour ceux que ça agacera, mais cette approche a un certain mérite pour là où je veux en venir. • Pour ceux qui veulent jouer, vous pouvez sauter toutes les explications, aller voir directement la liste de nombres donnée ci-dessus, et chercher une logique élémentaire : je pense qu'il y en a une, mais je ne la trouve pas.

Ajout : Voir aussi l'entrée suivante (qui est en bonne partie un copier-coller de celle-ci) pour le cas de F₄, qui est plus simple et donc peut-être pédagogiquement préférable.

Partons de huit nombres (= un élément de ℝ⁸) ; pour que ce que je raconte ne suppose aucune connaissance mathématique particulière, je précise que j'appellerai ça un vecteur et j'appellerai composantes du vecteur les huit nombres en question. Par exemple (1, 1, 0, 0, 0, 0, 0, 0), ou bien (0, 1, 2, 3, 4, 5, 6, 23) sont des vecteurs avec lesquels on va pouvoir jouer (ces exemples vont être intéressants pour la suite ; et oui, c'est bien un 23 que j'ai écrit à la fin, bear with me, ce n'est pas une blague dans le style quel est le huitième nombre qui complète la suite : 0, 1, 2, 3, 4, 5, 6… ?c'est évidemment 23). Maintenant, à partir de ce vecteur, imaginons qu'on ait le droit de faire, autant de fois qu'on veut, et dans n'importe quel ordre, les opérations très simples suivantes :

  • permuter ses composantes — c'est-à-dire les réordonner — de n'importe quelle manière (par exemple, on peut transformer (0, 1, 2, 3, 4, 5, 6, 23) en (0, 4, 3, 6, 1, 23, 5, 2), ce sont les mêmes nombres écrits dans un ordre différent),
  • changer le signe — c'est-à-dire transformer en leur opposé, remplacer moins par plus et vice versa — d'un nombre pair quelconque des composantes (par exemple, on peut transformer (5, 6, 7, 8, 9, 10, 11, −12) en (−5, −6, 7, −8, 9, −10, −11, 12), j'ai changé le signe de six composantes, et six est bien pair),
  • soustraire à chacune des huit composantes le quart de la somme de toutes (par exemple, ceci transforme (0, 1, 2, 3, 4, 5, 6, 23) en (−11, −10, −9, −8, −7, −6, −5, 12) : la somme des nombres était 0+1+2+3+4+5+6+23=44 donc j'ai soustrait 11 à chacun).

Voilà qui n'est pas bien compliqué. Pour fixer la terminologie les opérations des deux premiers types que je viens de dire seront appelées opérations de W(D₈) tandis que les opérations des trois types seront dites opérations de W(E₈) (je n'essaye pas du tout de définir ce que c'est que W(D₈) ou W(E₈), en tout cas pas pour le moment, ce sont juste des termes à considérer comme un bloc).

Les opérations de W(D₈) sont assez faciles à comprendre, en réfléchissant un peu on arrive assez facilement à voir ce qu'on peut faire avec (une description plus précise sera donnée plus bas, notamment, de quand on peut passer d'un vecteur à un autre par ces opérations). Celles de W(E₈), c'est-à-dire si on permet la troisième opération que j'ai dite, sont déjà plus mystérieuses mystérieuses : je vais donner quelques exemples ci-dessous ce qu'on peut faire avec.

La question générale est, que peut-on atteindre en appliquant les règles qui viennent d'être dites ? Autrement dit, partant d'un certain vecteur initial, quels vecteurs va-t-on pouvoir fabriquer avec les opérations qui viennent d'être dites (et combien y en a-t-il) ?

Pour prendre un exemple vraiment idiot, si le vecteur d'origine était (0, 0, 0, 0, 0, 0, 0, 0), on ne va pas très loin, il reste identique à lui-même sous l'effet de n'importe laquelle des opérations que j'ai décrites, et donc c'est la seule chose qu'on pourra atteindre.

Si le vecteur de départ est (1, 1, 0, 0, 0, 0, 0, 0), les opérations de W(D₈) (i.e., celles les deux premiers types) permettent de le transformer en n'importe quel vecteur ayant deux composantes égales à +1 ou −1 et les six autres nulles, ou en abrégé un vecteur du type (±1, ±1, 0, 0, 0, 0, 0, 0) (cela fait 8×7×2=112 vecteurs si on compte bien) ; la troisième opération transforme (1, 1, 0, 0, 0, 0, 0, 0) en (½, ½, −½, −½, −½, −½, −½, −½), et de là avec les opérations de W(D₈) on peut fabriquer les différents vecteurs (±½, ±½, ±½, ±½, ±½, ±½, ±½, ±½) dont toutes les composantes valent ±½ avec un nombre pair de signes moins (ou, ce qui revient au même, de signes plus ; cela fait 2⁷=128 vecteurs de cette forme), soit 112+128=240 vecteurs : il se trouve (il faut le vérifier mais ce n'est pas très difficile) que c'est tout ce qu'on obtient de la sorte : 240 vecteurs et pas plus. Ces 240 vecteurs forment d'ailleurs ce qui s'appelle le système de racines de E₈ (là aussi, je ne vais pas chercher à définir ce que ça veut dire, en tout cas pas aujourd'hui).

Je peux donner d'autres exemples. Si on part de (2, 0, 0, 0, 0, 0, 0, 0) (ou de (1, 0, 0, 0, 0, 0, 0, 0), cela revient évidemment au même quitte à tout diviser par deux, mais j'ai des raisons de préférer (2, 0, 0, 0, 0, 0, 0, 0)), on va pouvoir atteindre 2160 vecteurs différents par les opérations de W(E₈) ; c'est un peu plus fastidieux à compter : pour ceux qui veulent les détails, il y a les 16 vecteurs du type (±2, 0, 0, 0, 0, 0, 0, 0), les 1024 du type (∓3⁄2, ±½, ±½, ±½, ±½, ±½, ±½, ±½) avec un nombre pair de signes d'en bas, et les 1120 du type (±1, ±1, ±1, ±1, 0, 0, 0, 0) avec des signes quelconques. Si on part de (2, 1, 1, 0, 0, 0, 0, 0), on peut atteindre 6720 vecteurs différents (c'est encore plus pénible à compter). Si on part de (5⁄2, ½, ½, ½, ½, ½, ½, ½), on peut atteindre 17 280 vecteurs différents. Si on part de (3, 1, 0, 0, 0, 0, 0, 0) on peut atteindre 30 240 vecteurs différents.

Mais dans le « cas général » (disons, celui qui se produit avec probabilité 1 si notre vecteur initial a été tiré au hasard, ou bien si on est parti de (0, 1, 2, 3, 4, 5, 6, 23)), on va atteindre exactement 696 729 600 vecteurs. (En fait, la condition pour que ça soit le cas n'est pas très compliqué : il est nécessaire et suffisant, pour que cela se produise, que les huit composantes du vecteur initial soient deux à deux distinctes, qu'il n'y en ait pas deux qui soient opposées, et qu'il n'y ait pas non plus un nombre pair d'entre elles dont la somme soit égale à la somme des autres.) Et dans absolument tous les cas, le nombre de vecteurs qu'on peut atteindre sera fini, et sera même un diviseur de ce nombre maximal qu'est 696 729 600.

(Il y a d'ailleurs exactement 256 cas possibles entre le cas le plus spécial qu'est (0, 0, 0, 0, 0, 0, 0, 0) et qui donne un seul vecteur atteignable et le cas le plus général qui en donne 696 729 600. Mais je préfère rester vague sur ce que j'entends par un cas possible, parce que je ne crois pas que chacun de ces cas donne forcément un nombre de vecteurs atteints différents. En tout cas, les plus petits nombres possibles de vecteurs qu'on peut atteindre à partir d'un vecteur donné sont essentiellement ceux que j'ai listés ci-dessus : 1, 240, 2160, 6720, 13 440 et 17 280.)

☞ Il faut que je souligne que le fait qu'on obtienne un nombre fini de vecteurs est tout à fait remarquable. Si je faisais juste une toute petite modification à mes règles ci-dessus en autorisant, dans la deuxième opération, de changer le signe d'un nombre quelconque de composantes (au lieu d'exiger un nombre pair), alors n'importe quel vecteur non nul permettrait d'atteindre un nombre infini d'autres vecteurs avec les règles ainsi modifiées. La situation que je décris est véritablement exceptionnelle au sens où les « choses de ce genre » (en fait, les groupes finis de réflexions dans un espace euclidien) se rangent en un certain nombre de familles infinies plus une poignée d'exceptions, et W(E₈) fait partie de ces exceptions. Mais revenons à la situation bien particulière que j'ai considérée.

↑Entry #2502 [older| permalink|newer] / ↑Entrée #2502 [précédente| permalien|suivante] ↑

↓Entry #2492 [older| permalink|newer] / ↓Entrée #2492 [précédente| permalien|suivante] ↓

(samedi)

Approximation diophantienne ; et une bizarrerie mathématique : la constante de Freiman

Il est bien connu que l'ensemble ℚ des rationnels, que je noterai ici p/q sous forme irréductible, est dense dans les réels ℝ, c'est-à-dire que si x∈ℝ, on peut trouver p/q aussi proche qu'on veut de x, ou encore : (pour tout ε>0, il existe p/q tel que) |xp/q| < ε. Là où les choses deviennent plus intéressantes, c'est quand on commence à se demander, donné x∈ℝ, combien il faut payer pour l'approcher par p/q rationnel : autrement dit, si je veux une approximation de qualité ε>0, combien je dois le payer en utilisant un rationnel compliqué, le « compliqué » en question se mesurant par le dénominateur q>0 utilisé (on pourrait prendre la « hauteur » max(|p|,q), ou peut-être |p|+q, mais ça ne changerait pas grand-chose). Le sujet général s'appelle l'approximation diophantienne, et je n'y connais pas grand-chose, mais rappelons quand même les résultats les plus standards à ce sujet.

Si h est une fonction croissante des entiers naturels non nuls vers les réels strictement positifs, je peux dire qu'un réel x est h-approchable par les rationnels (ou simplement h-approchable) lorsqu'il existe des rationnels p/q de dénominateur q arbitrairement élevé tels que |xp/q| < 1/h(q) (formellement : pour tout n entier naturel non nul, il existe p et q entiers premiers entre eux avec qn tels que |xp/q| < 1/h(q)). Il faut y penser comme : en payant avec un dénominateur q j'obtiens une qualité d'approximation h(q). Plus la fonction h grandit vite, plus je demande une bonne approximation, donc plus il est difficile de trouver de tels x. Si h′≥h, ou même simplement si cette inégalité vaut à partir d'un certain rang, alors tout réel h′-approchable est, en particulier, h-approchable. Si h est constante (je demande une qualité d'approximation constante, et je suis prêt à payer arbitrairement cher pour l'avoir) ou simplement bornée, tout réel x est approchable, c'est ce que j'ai rappelé ci-dessus, mais on va voir ci-dessous qu'on peut faire mieux. Dans la pratique, on prendra donc une fonction h de limite ∞ en ∞, sinon la définition n'a guère d'intérêt.

Si h est quelconque (croissante des entiers naturels non nuls vers les réels strictement positifs), il existe toujours des réels h-approchables au sens ci-dessus : c'est une conséquence du théorème de Baire : quel que soit n>0, l'ensemble des x pour lesquels il existe p/q avec qn vérifiant |xp/q| < 1/h(q) est ouvert (puisque c'est une réunion d'intervalles ouverts de largeur 2/h(q) centrés en les p/q) et dense (puisqu'il contient l'ensemble dense des rationnels p/q de dénominateur qn) ; donc (le théorème de Baire assure que) leur intersection est non vide, c'est-à-dire qu'il existe des réels x, et même qu'il existe un ensemble dense, pour lesquels il existent des p/q avec q arbitrairement grand vérifiant |xp/q| < 1/h(q), ce qui signifie exactement qu'ils (les x en question) sont h-approchables. Bref, on peut trouver des réels approchés arbitrairement bien par des rationnels, quelle que soit la qualité h de l'approximation qu'on demande pour un dénominateur donné.

Un autre résultat, dit théorème d'approximation de Dirichlet, est que quel que soit x irrationnel, il existe des p/q de dénominateur q arbitrairement élevé tels que |xp/q| < 1/q² (c'est-à-dire que x est q²-approchable, ceci étant une écriture abusive pour dire h-approchable pour h(q)=q²). La démonstration est vraiment facile mais astucieuse : on considère les parties fractionnaires zk := yk−⌊yk⌋ (entre 0 inclus et 1 exclu) des réels yk := k·x pour 0≤kN entier ; ceci fait N+1 nombres zk, qu'on répartit en les N intervalles de largeur 1/N partitionnant [0;1[ (je veux dire : l'intervalle entre 0 inclus et 1/N exclu, l'intervalle entre 1/N inclus et 2/N exclu, et ainsi de suite jusqu'à l'intervalle entre (N−1)/N inclus et 1 exclu) ; comme il y a plus de réels que d'intervalles, deux d'entre eux, disons zk et z avec k<, qui tombent dans le même intervalle de largeur 1/N, donc ils vérifient |zzk| < 1/N, c'est-à-dire |·x − ⌊·x⌋ − k·x + ⌊k·x⌋| < 1/N, ce qui donne |q·xp| < 1/Nq = k et p = ⌊·x⌋−⌊k·x⌋, et comme 0<q<N (puisque 0≤k<N), on a du coup |xp/q| < 1/(N·q) < 1/q² comme annoncé ; quant au fait qu'on puisse trouver des q arbitrairement grands vérifiant ça, c'est simplement parce que (tant que x est irrationnel !, ce qui n'a pas encore été utilisé), chaque q donné ne peut vérifier |xp/q| < 1/(N·q) que jusqu'à un certain N (à savoir la partie entière de |q·xp|), et donc en prenant un N plus grand que ça, on obtient un p/q forcément différent (je laisse le lecteur remplir les détails).

↑Entry #2492 [older| permalink|newer] / ↑Entrée #2492 [précédente| permalien|suivante] ↑

↓Entry #2477 [older| permalink|newer] / ↓Entrée #2477 [précédente| permalien|suivante] ↓

(vendredi)

Notes de cours de théorie des langages formels

Un des cours (de première année) dont je suis responsable à l'ENST Télécom ParisTech ParisSaclay NewUni l'école où j'enseigne concerne la théorie des langages [formels], c'est-à-dire les langages rationnels, expressions rationnelles et automates finis, les langages algébriques et grammaires hors-contexte, et pour finir une toute petite introduction à la calculabilité (sujet dont je me suis déjà plaint, et plus d'une fois, de la difficulté à l'enseigner proprement). J'ai tout juste fini d'en réécrire le poly, complètement en retard puisque le cours a déjà commencé et qu'il va falloir du temps pour l'impression.

Comme je suis partisan de l'ouverture et de la disponibilité des documents d'enseignement, voici les notes en question. Si certains de mes lecteurs sont intéressés par ce sujet, ou veulent m'aider à traquer les erreurs qui demeurent certainement nombreuses, n'hésitez pas à me faire parvenir vos commentaires (mais comme je mets à jour ce lien régulièrement, pensez à recopier la ligne Git de la première page pour que je sache à quelle version vous faites référence).

(Il va de soi que le contenu lui-même, qui est le résultat de divers compromis, que ce soit sur le temps imparti ou sur l'équilibre entre mathématiques et informatique pratique, est souvent boiteux. Ce n'est pas la peine de me faire des remarques à ce sujet ; enfin, ce n'est pas qu'elles soient mal venues, c'est juste qu'elles ne seront pas suivies d'effets.)

↑Entry #2477 [older| permalink|newer] / ↑Entrée #2477 [précédente| permalien|suivante] ↑

↓Entry #2462 [older| permalink|newer] / ↓Entrée #2462 [précédente| permalien|suivante] ↓

(jeudi)

Petit guide bordélique de quelques ordinaux intéressants

Méta / avant-propos

L'écriture de cette entrée aura été assez chaotique, et un peu un échec : j'ai changé plusieurs fois d'avis sur ce que je voulais y mettre, et du coup le résultat est parti un peu dans tous les sens. Cela faisait longtemps que je me disais que je devrais écrire quelque chose sur des ordinaux remarquables (comme une suite de l'entrée d'introduction à leur sujet), j'y ai repensé en écrivant l'entrée sur la programmation transfinie, je m'y suis remis en reprenant (et en copiant-collant) des bouts de choses que j'avais écrites antérieurement et laissées de côté, mais ça s'est enlisé. Je commence par expliquer pourquoi — et dans une certaine mesure, comment lire cette entrée.

Ajout : j'aurais sans doute ajouter quelque part un lien vers ce billet passé où je parle de l'aspect psychologique de pourquoi les ordinaux me fascinent.

Mon idée initiale était d'aider le lecteur à situer un certain nombre d'ordinaux intéressants (dont j'ai pu parler par le passé ou dont je pourrais parler ultérieurement) en les classant dans l'ordre (ce qui est bien avec les ordinaux, c'est qu'ils sont, justement, bien ordonnés) : j'ai déjà écrit cet autre texte à ce sujet (lié depuis l'entrée précédente), mais il est un plutôt technique, son but étant surtout de rassembler des pointeurs vers la littérature mathématique publiée, alors qu'ici je voulais donner un aperçu plus intuitif de (certains de) ces ordinaux intéressants.

Je me suis dit que j'allais faire un plan en trois parties, que j'appellerai domaines : (1) les ordinaux calculables (et a fortiori dénombrables), c'est-à-dire les ordinaux strictement inférieurs à l'ordinal de Church-Kleene ω₁CK, (2) les ordinaux non calculables mais néanmoins dénombrables, c'est-à-dire ≥ω₁CK mais néanmoins <ω₁ (qui, en gros, ne sont intéressants que s'ils sont « admissibles »), et (3) les ordinaux non dénombrables (qui, en gros, ne sont intéressants que s'ils sont des cardinaux). Ce plan a le bon goût de permettre d'insister sur le fait que, par exemple, certains ordinaux, bien que monstrueusement grands et complexes à définir, sont néanmoins encore calculables (domaine (1), c'est-à-dire <ω₁CK), ce qui donne une petite idée de combien ω₁CK est gigantesque.

Mais ce plan a aussi l'inconvénient que l'ordre naturel sur les ordinaux (la taille, quoi) n'est pas du tout la même chose que l'ordre d'importance, d'intérêt, ou de difficulté à les définir (je peux définir ω₁ en disant que c'est le plus petit ordinal indénombrable, ou que c'est l'ensemble des ordinaux dénombrables triés par ordre de taille : ça ne laisse peut-être pas comprendre à quel point il est riche et complexe, mais au moins, c'est une définition nette et précise, alors que certains ordinaux beaucoup plus petits, quoique structuralement moins riches, sont beaucoup plus subtils à définir, puisqu'on veut les définir, justement, de façon beaucoup plus précise et complète). Plus subtilement, d'ailleurs, mon plan par taille des ordinaux a aussi l'inconvénient que l'ordre de taille n'est même pas l'ordre de dépendance logique des ordinaux : c'est ce phénomène qu'on appelle imprédicativité qui veut qu'on fasse appel, pour construire certains ordinaux, à des ordinaux encore plus grands ; ainsi, la construction de l'ordinal de Bachmann-Howard (qui est <ω₁CK, donc dans le domaine (1) de mon plan) fait appel à une « fonction d'écrasement », qui présuppose de savoir ce que c'est que ω₁CK ou peut-être ω₁ (l'un ou l'autre peut servir, et on lui donne le nom de Ω dans les notations), et c'est encore pire dans la construction d'ordinaux calculables encore plus grands, qui nécessitent d'invoquer des ordinaux récursivement grands ou de grands cardinaux.

Je le savais, bien sûr, mais je pensais pouvoir contourner ces difficultés en fournissant au fur et à mesure des informations minimales sur les grands ordinaux des domaines (2) et (3) alors que je décrivais le domaine (1), quitte à y revenir plus tard. Finalement, c'est une très mauvaise idée, et cette partie (1) a beaucoup trop gonflé et est devenue, du même coup, assez illisible. (Un autre problème est que ce qui rend les ordinaux calculables vraiment intéressants est leur lien avec certaines théories logiques, et il faudrait vraiment beaucoup de place pour expliquer ce que sont exactement des théories telles que la « théorie des ensembles de Kripke-Platek », l'« arithmétique du second ordre limitée à la Δ¹₂-compréhension », la « théorie des définitions inductives ».) En même temps que ça, j'ai commencé à en avoir vraiment marre d'écrire sur des ordinaux de plus en plus techniques à expliquer. Du coup, j'ai calé sur la partie (1), ce qui casse vraiment l'intention initiale, puisque j'avais surtout envie (pour rester sur la lancée de la programmation transfinie) d'essayer de dire des choses sur les ordinaux nonprojectibles, stables et compagnie, qui sont résolument dans la partie (2).

Au final, c'est un peu n'importe quoi : cette entrée me fait l'effet d'une moussaka géante où on ne comprend plus rien. Mais je pense qu'il y a quand même un certain intérêt à ce que je publie ce « n'importe quoi » plutôt que de le ranger dans mes cartons, c'est-à-dire dans le vaste cimetière des entrées que j'ai commencées et jamais publiées. Car après tout, ce que j'écris est correct (enfin, je crois), et même si vers la fin je lance dans l'air de plus en plus de termes non définis faute de patience pour les définir, ou que je pars complètement dans l'agitage de mains, certains en tireront quand même quelque chose.

Finalement, les différentes sous-parties de cette entrée sont, je l'espère, assez indépendantes les unes des autres, donc comme d'habitude, et même plus encore que d'habitude, j'encourage à sauter les passages qu'on trouve incompréhensibles ou trop techniques (beaucoup d'entre eux ne servent, finalement, à rien).

Comme expliqué ci-dessus, je vais d'abord faire quelques remarques générales sur les ordinaux intéressants, expliquer plus précisément le plan que j'avais en tête, puis parler d'ordinaux calculables (i.e., <ω₁CK, le domaine (1)), et m'arrêter en queue de poisson.

↑Entry #2462 [older| permalink|newer] / ↑Entrée #2462 [précédente| permalien|suivante] ↑

↓Entry #2460 [older| permalink|newer] / ↓Entrée #2460 [précédente| permalien|suivante] ↓

(vendredi)

Un peu de programmation transfinie

Ça fait très longtemps que j'ai envie d'écrire cette entrée, parce que je trouve le sujet extrêmement rigolo : en gros, ce dont je veux parler, c'est comment définir et programmer un ordinateur transfini ? (comment concevoir un langage de programmation considérablement plus puissant qu'une machine de Turing parce qu'il est capable de manipuler directement des — certains — ordinaux ?). Techniquement, ce dont je veux parler ici, c'est de la théorie de la α-récursion (une branche de la calculabilité supérieure qui a fleuri dans les années '70 et qui semble un peu moribonde depuis) ; sauf que la α-récursion n'est jamais présentée comme je le fais ici, c'est-à-dire en décrivant vraiment un langage assez précis dans lequel on peut écrire des programmes pour certains ordinateurs transfinis. Ces ordinateurs ont le malheur de ne pas pouvoir exister dans notre Univers (encore que, si on croit certaines théories complètement fumeuses que j'avais imaginées… ?) ; mais même s'ils n'existent pas, je pense que le fait d'écrire les choses dans un style « informatique » aide à rendre la théorie mathématique plus palpable et plus compréhensible (en tout cas, c'est comme ça que, personnellement, j'aime m'en faire une intuition).

Bref, ce que je voudrais, c'est que cette entrée puisse plaire à la fois à ceux qui aiment la programmation et à ceux qui aiment les ordinaux ; ce que je crains, c'est qu'en fait elle déplaise à la fois à ceux qui n'aiment pas la programmation et à ceux qui n'aiment pas les ordinaux — ce qui est logiquement différent. On verra bien.

Il faut que je précise que tout ce que je raconte est un territoire relativement mal couvert par la littérature mathématique (il y a certainement des gens qui trouveraient tout ça complètement évident, mais je n'en fais pas partie, et comme je le disais, je soupçonne que la plupart étaient surtout actifs vers '70 et sont maintenant un peu âgés ou sont passés à autre chose), et jamais de la manière dont je le fais (comme un vrai langage de programmation : il y a des gens qui ont « redécouvert » des domaines proches comme avec les machines de Turing infinies ou les machines ordinales de Koepke, mais c'est un peu différent). Du coup, il faut prendre tout ce que je raconte avec un grain de sel : je n'ai pas vérifié chaque affirmation avec le soin que j'aurais fait si j'étais en train d'écrire un article à publier dans un journal de recherche.

Une autre remarque : cette entrée contient un certain nombre de digressions, notamment parce que je pars dans plusieurs directions un peu orthogonales. Je n'ai pas voulu les mettre en petits caractères comme je le fais souvent, pour ne pas préjuger de ce qui est important et ce qui ne l'est pas, et je n'ai pas eu le courage de tracer un leitfaden, mais tout ne dépend pas de tout : donc, si on trouve un passage particulièrement obscur ou inintéressant, on peut raisonnablement espérer(!) qu'il ne soit pas vraiment important pour la suite.

*

Pour faire une sorte de plan ce dont je veux parler, je vais décrire un langage de programmation assez simple (dont la syntaxe sera imitée de celle du C/JavaScript) et différentes variantes autour de ce langage. Plus exactement, je vais définir quatre langages : un langage (0) « de base » et deux extensions qu'on peut appliquer à ce langage (les extensions « forward » et « uloop », qui seront définies après), de sorte qu'à côté du langage (0) de base, il y aura le langage (1) avec extension « forward », le langage (2) avec extension « uloop », et le langage (3) avec les deux extensions à la fois ; tout ça peut encore être multiplié par deux si j'autorise les tableaux dans le langage, ce qui, finalement, ne changera rien à son pouvoir d'expression, et c'est peut-être surprenant.

Chacun de ces langages pourra servir dans le « cas fini » (le langage manipule des entiers naturels, et chacun des langages (0)–(3) peut être implémenté sur un vrai ordinateur et servir de vrai langage de programmation) ou dans le « cas transfini » (le langage manipule des ordinaux). J'expliquerai plus précisément en quoi consiste ce cas transfini, mais je veux insister dès à présent sur le fait que les langages de programmation (0)–(3) seront exactement les mêmes dans ce cas transfini que dans le cas fini (plus exactement, leur syntaxe sera exactement la même ; la sémantique pour les langages (0)&(1) sera prolongée, tandis que pour les langages (2)&(3) elle sera raffinée et dépendra d'un « ordinal de boucle » λ).

↑Entry #2460 [older| permalink|newer] / ↑Entrée #2460 [précédente| permalien|suivante] ↑

↓Entry #2447 [older| permalink|newer] / ↓Entrée #2447 [précédente| permalien|suivante] ↓

(lundi)

Sons et graphes de caractères de groupes de Lie

Il y a quelque temps, je me désolais de ne jamais avoir réussi à trouver un objet mathématique dont je pourrais faire une représentation sous forme auditive — plutôt que visuelle — et qui serait mélodieux à entendre.

Or ces derniers temps, je réfléchissais à des problèmes — et globalement, à essayer de comprendre plus précisément des choses — autour de caractères de groupes de Lie, et j'ai été amené à tracer des fonctions qui ressemblent à ceci (cliquez pour agrandir) :

[Caractères fondamentaux du groupe de Lie F₄ restreintes au tore du SU₂ principal de Kostant]

Là, je devrais essayer de dire de quoi il s'agit. L'ennui, c'est que ce n'est pas facile. Je peux donner une explication pour les experts, mais elle n'éclairera pas du tout le grand public (ni même le public moyennement averti) ; je l'écris surtout pour m'en souvenir moi-même :

(Pour les experts, donc.)

Il s'agit des caractères fondamentaux d'un groupe de Lie (réel compact) simple (dans la figure ci-dessus, il s'agit de F₄), restreints au tore du SU₂ principal de Kostant, c'est-à-dire, plus concrètement, le groupe à un paramètre engendré par la demi-somme des coracines positives. Autrement dit, si ρ# est la demi-somme des coracines positives (ou somme des copoids fondamentaux), donnée une représentation définie par son système de poids, on applique ρ# aux poids en question, ce qui donne des demi-entiers (les multiplicités étant sommées), à interpréter comme les poids d'une représentation de SU₂, ou comme définissant un polynôme trigonométrique. Une façon de calculer en pratique consiste à appliquer la formule de caractère de Weyl avec une petite astuce (cf. §3.1 de cet article) : si ρ est la demi-somme des racines positives et λ un poids dominant, on calcule le produit des tλ+ρ,α#⟩−1 où t est une indéterminée et α# parcourt les coracines positives, et on divise ce polynôme par le produit des tρ,α#⟩−1 ; ceci donne un polynôme en t (dont la valeur en 1 est précisément la dimension de la représentation de poids dominant λ, c'est la formule de dimension de Weyl ; quant au degré, il vaut 2⟨λ,ρ#⟩, c'est-à-dire la somme des coefficients de λ sur la base des racines simples) : les coefficients de ce polynôme sont ceux recherchés : si on les décale (i.e. on divise encore par tλ,ρ#⟩) et qu'on lit comme un polynôme trigonométrique, c'est la fonction recherchée. Voici par exemple le calcul en Sage dans le cas de F₄ :

sage: WCR = WeylCharacterRing("F4", style="coroots")
sage: weylvec = sum([rt for rt in WCR.positive_roots()])/2
sage: R.<t> = PolynomialRing(QQ,1)
sage: weyldenom = prod([t^weylvec.scalar(rt.associated_coroot())-1 for rt in WCR.positive_roots()])
sage: weylnumer1 = prod([t^(weylvec+WCR.fundamental_weights()[1]).scalar(rt.associated_coroot())-1 for rt in WCR.positive_roots()])
sage: weylnumer2 = prod([t^(weylvec+WCR.fundamental_weights()[2]).scalar(rt.associated_coroot())-1 for rt in WCR.positive_roots()])
sage: weylnumer3 = prod([t^(weylvec+WCR.fundamental_weights()[3]).scalar(rt.associated_coroot())-1 for rt in WCR.positive_roots()])
sage: weylnumer4 = prod([t^(weylvec+WCR.fundamental_weights()[4]).scalar(rt.associated_coroot())-1 for rt in WCR.positive_roots()])
sage: weylnumer1/weyldenom
t^22 + t^21 + t^20 + t^19 + 2*t^18 + 2*t^17 + 3*t^16 + 3*t^15 + 3*t^14 + 3*t^13 + 4*t^12 + 4*t^11 + 4*t^10 + 3*t^9 + 3*t^8 + 3*t^7 + 3*t^6 + 2*t^5 + 2*t^4 + t^3 + t^2 + t + 1
sage: weylnumer2/weyldenom
t^42 + t^41 + 2*t^40 + 3*t^39 + 5*t^38 + 7*t^37 + 10*t^36 + 12*t^35 + 16*t^34 + 20*t^33 + 25*t^32 + 29*t^31 + 35*t^30 + 39*t^29 + 45*t^28 + 50*t^27 + 55*t^26 + 58*t^25 + 62*t^24 + 63*t^23 + 66*t^22 + 66*t^21 + 66*t^20 + 63*t^19 + 62*t^18 + 58*t^17 + 55*t^16 + 50*t^15 + 45*t^14 + 39*t^13 + 35*t^12 + 29*t^11 + 25*t^10 + 20*t^9 + 16*t^8 + 12*t^7 + 10*t^6 + 7*t^5 + 5*t^4 + 3*t^3 + 2*t^2 + t + 1
sage: weylnumer3/weyldenom
t^30 + t^29 + 2*t^28 + 3*t^27 + 4*t^26 + 5*t^25 + 7*t^24 + 8*t^23 + 10*t^22 + 11*t^21 + 13*t^20 + 14*t^19 + 16*t^18 + 16*t^17 + 17*t^16 + 17*t^15 + 17*t^14 + 16*t^13 + 16*t^12 + 14*t^11 + 13*t^10 + 11*t^9 + 10*t^8 + 8*t^7 + 7*t^6 + 5*t^5 + 4*t^4 + 3*t^3 + 2*t^2 + t + 1
sage: weylnumer4/weyldenom
t^16 + t^15 + t^14 + t^13 + 2*t^12 + 2*t^11 + 2*t^10 + 2*t^9 + 2*t^8 + 2*t^7 + 2*t^6 + 2*t^5 + 2*t^4 + t^3 + t^2 + t + 1

Le polynôme en question doit d'ailleurs avoir un rapport très fort avec les crystal graphs de Kashiwara et Littelmann (les coefficients énumèrent le nombre de nœuds à chaque hauteur du graphe) ; et sans doute avec les groupes quantiques : je n'y connais rien, mais dans le cas de Ar, on obtient exactement le coefficient binomial gaussien (r+1,i) pour la i-ième représentation fondamentale. • Par ailleurs, il y a une grande similarité avec un autre polynôme important, à savoir le produit des tα,ρ#⟩+1−1 où t est une indéterminée et α parcourt les racines positives, divisé par le produit des tα,ρ#⟩−1 : ce polynôme-là énumère les éléments du groupe de Weyl par leur longueur (Carter, Simple Groups of Lie Type (1972/1989), théorème 10.2.2 page 153), par exemple pour F₄ on trouve t^24 + 4*t^23 + 9*t^22 + 16*t^21 + 25*t^20 + 36*t^19 + 48*t^18 + 60*t^17 + 71*t^16 + 80*t^15 + 87*t^14 + 92*t^13 + 94*t^12 + 92*t^11 + 87*t^10 + 80*t^9 + 71*t^8 + 60*t^7 + 48*t^6 + 36*t^5 + 25*t^4 + 16*t^3 + 9*t^2 + 4*t + 1, il est en lien avec les exposants du groupe de Weyl (id, théorème 10.2.3 page 155), et à très peu de choses près donne la fonction zêta du groupe algébrique, c'est-à-dire compte ses points sur les corps fini (id, proposition 8.6.1 page 122), ou de façon sans doute plus pertinente, les points de la variété de drapeau associée. Je ne comprends pas bien le rapport précis entre tous ces polynômes (notons que j'ai écrit le dernier pour coller avec ce que je trouve dans Carter, mais si je ne m'abuse, c'est aussi le produit des tρ,α#⟩+1−1 où t est une indéterminée et α parcourt les racines positives, divisé par le produit des tρ,α#⟩−1, ce qui le fait ressembler encore plus à ce que j'ai écrit ci-dessus). [Ajout : ce dernier polynôme est appelé q-polynomial ici. Je devrais ajouter, pour reproduire ce qui est mentionné sur cette page, que pour obtenir le polynôme donnant nombre de points de la variété de drapeau partielle définie par un ensemble S de nœuds du diagramme de Dynkin, on fait le produit des tα,ρ#⟩+1−1 divisé par le produit des tα,ρ#⟩−1, où cette fois α parcourt seulement les racines ayant au moins un coefficient strictement positif devant une racine simple omise de S.]

Il faudrait essayer de vulgariser tout ça, mais ce n'est pas évident : pas tellement parce que les objets en question sont compliqués (fondamentalement, le calcul final est un petit calcul combinatoire, assez facile, même si évidemment le présenter comme tel ne fournit aucune motivation), mais surtout parce que, comme c'est souvent le cas dans ce domaine entre la théorie des groupes algébriques, la théorie de la représentation, et la combinatoire algébrique, chaque objet peut se voir d'une multitude de manières différentes (ce qui est d'ailleurs la source d'incompréhensions diverses et variées). J'avais commencé à essayer d'écrire quelque chose, non pas vraiment pour expliquer mais juste pour donner une idée de ce dont il est question (en agitant énormément les mains), mais même comme ça, ça partait tellement dans tous les sens que c'est incompréhensible : je le recopie quand même ici (comme un gros bloc de texte), mais je ne recommande de le lire que pour rigoler :

↑Entry #2447 [older| permalink|newer] / ↑Entrée #2447 [précédente| permalien|suivante] ↑

↓Entry #2433 [older| permalink|newer] / ↓Entrée #2433 [précédente| permalien|suivante] ↓

(dimanche)

Sections du diagramme de Voronoï du réseau E₈

Je ne savais pas bien à quoi m'attendre quand j'ai calculé cette image, mais probablement pas à ça :

[Section plane aléatoire du diagramme de Voronoï de E₈]

(Cliquez pour une vue plus large.)

De quoi s'agit-il ? C'est une section plane aléatoire du diagramme de Voronoï du réseau E₈ : il faut que j'explique ces termes (mais is ça ne vous intéresse pas, il y a d'autres images, et des liens vers des vidéos, plus bas).

Le réseau E₈ est un arrangement régulier de points en dimension 8, qui a toutes sortes de propriétés remarquables. En fait, il n'est pas difficile de le définir concrètement : il s'agit des octuplets (x₀,x₁,…,x₇) de nombres réels tels que :

  • les coordonnées x₀,x₁,…,x₇ sont soit toutes entières soit toutes entières-et-demi (par entier-et-demi je veux évidemment dire un nombre qui vaut un entier plus ½, par exemple 5/2),
  • la somme x₀+x₁+⋯+x₇ de toutes les coordonnées (qui est forcément un entier d'après le point précédent) est paire.

À titre d'exemple, (0, 0, 0, −1, 2, −1, 1, −1) et (−1.5, 2.5, −0.5, 1.5, −1.5, −0.5, −2.5, 0.5) sont dans le réseau E₈ ; en revanche, (0, 0, 0, −1, 2, −1, 1.5, −1.5) n'y sont pas (les coordonnées ne sont ni toutes entières ni toutes entières-et-demi), et (−1.5, 2.5, −0.5, 1.5, −1.5, −0.5, −2.5, 0.5) non plus (la somme n'est pas paire).

La somme ou différence de deux points du réseau E₈ est encore dedans : c'est là la propriété essentielle d'être un réseau (et ce qu'un non-mathématicien qualifierait de points régulièrement espacés). Les points du réseau E₈ les plus proches de l'origine (0,0,0,0,0,0,0,0) sont d'une part ceux de la forme (±1,±1,0,0,0,0,0,0) (où exactement deux coordonnées, quelconques, valent soit 1 soit −1 : ceci fait 28×4=112 possibilités — 28 choix de deux coordonnées et 4 choix de leurs signes), et d'autre part ceux de la forme (±½,±½,±½,±½,±½,±½,±½,±½) (où chaque coordonnée vaut ½ ou −½, et où il y a un nombre pair de valeurs −½ : ceci fait 2⁸/2=128 possibilités) : au total, 112+128=240 points tous à distance √2 de l'origine ; ces 240 points sont ce qu'on appelle les racines du système E₈ et ils engendrent le réseau, mais ici c'est le réseau plus que ses racines qui m'intéresse. Entre autres propriétés remarquables, c'est le réseau E₈ qui réalise l'empilement optimal de boules identiques en dimension 8 (mettre une boule de rayon (√2)/2 autour de chaque point du réseau : elles se touchent sans se chevaucher et remplissent 25.367% de l'espace, ce qui ne paraît peut-être pas impressionnant, mais en dimension 8 on ne peut pas faire mieux).

Donné un ensemble (discret) de points dans l'espace euclidien, le diagramme de Voronoï associé est la division de l'espace en cellules de Voronoï, la cellule de Voronoï d'un point étant la région des points de l'espace qui sont plus proches de ce point-là que de tout autre point de l'ensemble. En général, un diagramme de Voronoï ressemble à ce que Google images vous montrera (il est formé de cellules qui sont des polytopes convexes dont les facettes sont hyperplans médiateurs entre le point définissant la cellule et un autre point). Lorsque l'ensemble des points est un réseau, toutes les cellules ont la même forme : la cellule de Voronoï de l'origine est l'ensemble des points plus proches de l'origine que de tout autre point du réseau, elle est d'ailleurs symétrique, et toutes les autres cellules sont identiques autour d'un autre point, elles sont translatées les unes des autres. S'agissant du réseau E₈ précisément, la cellule de Voronoï de l'origine est un polytope convexe ayant 240 facettes[#], une par racine du système de racines, chaque facette étant un morceau de l'hyperplan médiateur entre l'origine et la racine en question. (Il n'est pas vrai dans un réseau en général que les facettes de la cellule de Voronoï de l'origine soient ainsi définies uniquement par les points les plus proches de l'origine. Mais c'est vrai pour ce qu'on appelle un réseau de racines, et notamment E₈.)

[#] Il a aussi 19440 sommets : 2160 sont les points à distance 1 de l'origine ainsi que de quinze autres points du réseau, on les appelle les trous profonds du réseau E₈ (un exemple d'un tel point est (1,0,0,0,0,0,0,0)), et 17280 sont les points à distance (2√2)/3≈0.943 de l'origine ainsi que de sept autres et ce sont les trous superficiels (un exemple d'un tel point est (−5/6, 1/6, 1/6, 1/6, 1/6, 1/6, 1/6, 1/6)).

Bref, le diagramme de Voronoï du réseau E₈ est un pavage de l'espace de dimension 8 par des copies (translatées) de ce polytope à 240 facettes, chacune étant centrée sur un point du réseau. Il y a un algorithme assez simple[#2] pour décider, quand on se donne un point de l'espace, à quelle cellule de Voronoï il appartient, c'est-à-dire, trouver le point du réseau le plus proche (on parle aussi d'algorithme de décodage pour ce réseau).

[#2] En voici une description. Commençons par expliquer comment trouver le point du réseau D₈ le plus proche d'un point donné, où le réseau D₈ est le réseau formé des points de coordonnées toutes entières de somme paire (c'est-à-dire les points du réseau E₈ dont toutes les coordonnées sont entièrs). Donné (z₀,z₁,…,z₇) un point à approcher, on appelle x₀ l'entier le plus proche de z₀ et de même pour les autres : ceci fournit le point (x₀,x₁,…,x₇) à coordonnées entières le plus proche de (z₀,z₁,…,z₇). Si la somme x₀+x₁+⋯+x₇ des coordonnées est paire, c'est le point de D₈ recherché. Sinon, l'astuce suivante permet de le trouver : parmi les coordonnées x, prendre celle qui est le plus loin du z correspondant, et la remplacer par l'arrondi de ce z dans l'autre sens. À titre d'exemple, si on part du point (0.3, −0.1, 0.1, −1.0, 2.0, −0.4, 0.9, −0.7), l'arrondi des coordonnées à l'entier le plus proche donne (0, 0, 0, −1, 2, 0, 1, −1), la somme est impaire, donc on corrige le plus mauvais arrondi, à savoir −0.4 transformé en 0, en prenant l'entier de l'autre côté, donc −1, ce qui donne le point (0, 0, 0, −1, 2, −1, 1, −1) qui est le point du réseau D₈ le plus proche du point initial. S'agissant du réseau E₈, maintenant, on peut faire ce calcul une fois pour trouver le point de D₈ le plus proche, puis soustraire ½ toutes les coordonnées, refaire le calcul pour trouver le point de D₈ le plus proche du point ainsi modifié et rajouter ½ à toutes les coordonnées : on obtient ainsi deux points de E₈ (l'un dans D₈ et l'autre dans D₈+(½,½,½,½,½,½,½,½)) ; il n'y a plus qu'à comparer la distance de ces deux points au point d'origine et choisir le plus proche (soit en comparant les distances soit en calculant l'équation de l'hyperplan médiateur, ce qui revient essentiellement au même). Il existe des algorithmes légèrement plus efficaces que ce que je viens de décrire, mais en contrepartie ils sont plus fastidieux à implémenter et je pense que ça n'en vaut pas la peine.

↑Entry #2433 [older| permalink|newer] / ↑Entrée #2433 [précédente| permalien|suivante] ↑

↓Entry #2430 [older| permalink|newer] / ↓Entrée #2430 [précédente| permalien|suivante] ↓

(mercredi)

Exposé pour Math en Jeans : les slides

J'ai mis en ligne ici le support que je compte utiliser pour mon exposé devant des lycéens samedi après-midi à Math en Jeans, intitulé Le jeu de nim : thème et variations.

Soit dit en passant, je ne suis pas spécialement hostile aux anglicismes, mais celui-là m'agace — en fait, le terme anglais n'est pas terrible pour commencer : qu'est-ce qu'on peut dire en français, plutôt que slide, pour parler d'une image projetée, de nos jours, par vidéoprojecteur, et servant à illustrer un exposé ?

Il manque, évidemment, l'accompagnement audio (si je suis très motivé, je ferai une vidéo sur YouTube), mais je me dis que si je n'ai pas trop mal réussi mon coup, on doit pouvoir à peu près comprendre même sans les explications orales. (Évidemment, il y a des endroits où elles sont quand même utiles à la clarté des choses ! Je pense par exemple au calcul des valeurs de Grundy dans l'exemple slide 18, qui est très facile à expliquer de vive voix avec un pointeur laser mais franchement laborieux si on veut l'écrire.)

Je précise que je n'ai pas l'intention de tout présenter : il y en a sans toute trop, peut-être même beaucoup trop (combien n'est pas clair). J'essaierai de m'adapter en fonction de la manière dont mon auditoire réagit. Disons que le minimum est le contenu des slides 3 à 14, ce qui suit contient plusieurs sujets de difficulté inégale, donc j'en traiterai un sous-ensemble, quelque part entre « rien » et « tout », selon le temps disponible et la manière dont j'ai l'impression qu'ils comprennent. (Exemple de parcours possible : 1–16,20–22,29.)

Les commentaires sont bienvenus ; mais ce n'est pas la peine de me dire que j'aurais dû m'y prendre complètement autrement, ou traiter un autre sujet : il est trop tard pour ça ; et ce n'est pas non plus la peine de me suggérer d'ajouter une figure, j'ai suffisamment souffert avec TikZ comme ça. Les suggestions locales d'amélioration/reformulation (surtout en nombre de mots constant !) seront appréciées. Mais ce qui est particulièrement bienvenu est un avis sur la difficulté relative des différentes slides pour des lycéens (motivés), ainsi que leur attrait, ou le temps qu'il faudrait y passer pour les expliquer : relatif, parce que si ça ne sert pas à grand-chose de dire que tout est trop dur, ça a un intérêt de se demander si la slide 30 est plus ou moins difficile à comprendre que la 23 (par exemple), dans la mesure où je devrai certainement faire des choix sur quoi présenter (modulo un hypothétique director's cut sur YouTube).

Bilan : voir l'entrée suivante.

↑Entry #2430 [older| permalink|newer] / ↑Entrée #2430 [précédente| permalien|suivante] ↑

↓Entry #2429 [older| permalink|newer] / ↓Entrée #2429 [précédente| permalien|suivante] ↓

(dimanche)

Hidden Figures

Mon poussinet et moi sommes allés voir le film Hidden Figures (le titre français — Les Figures de l'ombre — ne rend pas vraiment le jeu de mot le jeu de mot entre une personne et un chiffre dans un calcul), et je voudrais vraiment le recommander.

Il s'agit de l'histoire, vraie mais bien sûr partiellement romancée, de trois femmes noires « calculatrices » à la NASA au début des années 1960 (plus exactement, au centre de recherches Langley en Virginie, entre le premier vol dans l'espace de Ûrij [=Yuri] Gagarin en 1961 et celui de John Glenn en 1962). La manière dont elles sont confrontées à la fois à la discrimination raciale et au sexisme, et leurs différentes façons d'y faire face, sont montrées avec une certaine subtilité, de même que l'atmosphère côté américain de la « course à l'espace ». L'histoire suit une trame hollywoodienne bien formatée et qu'on peut trouver un peu trop schématique, mais les actrices jouent très bien (Taraji Henson, qui interprète Katherine Goble, Janelle Monáe qui joue Mary Jackson, et surtout Octavia Spencer — que je connaissais par un autre film remarquable, The Help — dans le rôle de Dorothy Vaughan), et pour une fois qu'on voit un film dont les personnages principaux sont des femmes noires, et mathématiciennes qui plus est, ne boudons pas notre plaisir. (Et puis j'ai un faible pour l'ambiance course à l'espace, l'ambiance « atompunk », ici illustrée avec une certaine sympathie sans excès.)

Scientifiquement, le film ne commet pas de bourde majeure, en tout cas pas que j'en aie repérée : le moment le plus faux sur ce plan-là est celui où l'héroïne principale, Katherine Goble, effectue au tableau, devant une salle de généraux un peu médusés, un calcul de paramètres de réentrée orbitale avec une précision dont il devrait être à peu près évident pour n'importe qui ayant un chouïa de culture scientifique, qu'il n'est pas atteignable de tête, en tout cas pas un temps tel que présenté ; je suis prêt à ne pas faire mon grincheux pour quelque chose du genre. Il y a aussi un certain nombre de modifications du tempo par rapport à la réalité, imposées pour s'adapter au rythme cinématographique, que je suis également prêt à pardonner.

Il est vrai que j'aurais aimé voir un peu de considération pour la différence entre la notion de calcul symbolique et celle de calcul numérique, choses que le grand public ne doit pas vraiment apprécier, mais qui n'est certainement pas impossible à faire passer. Les équations qu'on entr'aperçoit dans différents plans ont l'air superficiellement sensées, mais mélangent inexplicablement des valeurs numériques à virgules dans des expressions par ailleurs symboliques ; et de façon plus profonde, je n'ai pas vraiment idée de quel genre de calculs on faisait faire à ces « calculatrices », soit en général, soit précisément celles qui sont les héroïnes de ce film.

Et on ne peut pas dire que les répliques m'aident à deviner. À un moment, le chef d'équipe joué par Kevin Costner demande à Katherine Goble si elle sait calculer un repère de Frénet — et elle complète : par le procédé d'orthogonalisation de Schmidt. C'est vraiment amusant comme effet Zahir, parce que je discutais du repère de Frénet avec mon poussinet un quart d'heure avant d'aller voir le film (à propos du tome 5, particulièrement poussiéreux, du Cours de Mathématiques spéciales de MM. Ramis-Deschamps-Odoux), et je mentionnais justement qu'il s'agissait précisément du résultat d'un Gram-Schmidt sur les dérivées successives du mouvement : j'ai eu du mal à ne pas éclater de rire à la coïncidence. Mais même si vois le lien avec des trajectoires dans l'espace, je ne sais vraiment pas précisément dans quel genre de calcul, symbolique ou numérique, on utilise le repère de Frénet.

En vérité, même si je connais ma mécanique orbitale et lagrangienne, je n'ai aucune idée précise du genre de calculs qu'il faut réellement mener pour envoyer un homme dans l'espace. (Bon, je dois dire, je n'ai même pas d'idée précise sur le genre de calculs qu'il faut mener pour construire un pont ou un moteur à explosion. Je suis un peu comme le matheux d'une blague générique sur les ingénieurs, physiciens et mathématiciens, qui démontrerait que le pont, le moteur à explosion ou le vol orbital sont possibles — par une démonstration non-constructibe qui ferait appel à l'axiome du choix.)

Sur la précision scientifique des films hollywoodiens de façon plus générale, j'étais tombé il y a un certain temps sur cette vidéo qui explique que des gens ont mis en place une hotline permettant à l'industrie du cinéma d'être mis en contact avec des scientifiques de tel ou tel domaine quand ils veulent des conseils ou des éléments (phrases, équations à mettre sur un tableau, etc.) pour rendre leurs films scientiquement plus crédibles. Ça expliquerait un certain progrès que j'ai cru constater dans le domaine depuis les années '90 (même si ce progrès est souvent bien superficiel, il faut l'admettre : le fait de prononcer une phrase techniquement sensée à tel ou tel moment ne va pas compenser une absurdité fondamentale de principe ; il y a toujours très peu de films qui, comme The Martian, se donnent pour mission d'être véritablement réalistes scientifiquement, d'un bout à l'autre, ce qui implique d'aller plus loin qu'appeler une hotline de temps à autre).

À part ça, je me rends compte que je ne remplis pas vraiment consciencieusement la catégorie cinema de ce blog : ces derniers temps, j'ai vu en salles, entre autres, Manchester by the Sea et 君の名は (traduit en « français »(?!) par Your Name), et j'ai trouvé que les deux étaient vraiment des chefs d'œuvre. Je n'ai pas le temps d'en faire une critique maintenant (et ce serait un peu du réchauffé), mais je les recommande tous les deux très vivement, ce sont des films d'une très grande subtilité humaine et psychologique.

↑Entry #2429 [older| permalink|newer] / ↑Entrée #2429 [précédente| permalien|suivante] ↑

↓Entry #2427 [older| permalink|newer] / ↓Entrée #2427 [précédente| permalien|suivante] ↓

(mercredi)

Sujet d'exposé pour Math en Jeans

Je me suis engagé à donner un exposé (quelque part entre le 24 et le 27 mars) dans le cadre de l'événement Math en Jeans : c'est-à-dire qu'il s'agit de vulgarisation adressée à des lycéens motivés (a priori de seconde).

J'ai toute latitude pour choisir le sujet, donc je vais sans doute choisir un des trucs sur lesquels j'ai déjà fait de la vulgarisation, soit sur ce blog soit ailleurs : la contrainte est que je dois pouvoir raconter ça en une heure (en prévoyant des probables interruptions par des questions) et que ça soit accessible à des lycéens. Et, bien sûr, que ce soit susceptible de les intéresser.

Je n'ai pas une idée très précise de ce qu'un lycéen (motivé !) connaît en maths ni de ce qui l'intéressera : peut-être que certains lecteurs (par exemple s'il y en a qui enseignent en lycée ou qui sont ou out été lycéens il n'y a pas trop longtemps) peuvent m'éclairer un peu.

Globalement, j'ai plutôt trop d'idées que pas assez, donc je me demande si vous avez des conseils sur ce qui passerait plus ou moins bien parmi les thèmes suivants (j'essaie de mettre à chaque fois un lien vers une entrée de ce blog qui raconte de quoi il s'agit, mais il ne s'agit pas forcément de raconter exactement la même chose, notamment quand il s'agit de choses un peu techniques : c'est plus pour donner une idée) :

[Ajout : quelques arguments pour/contre ces différents sujets.]

  • Les (très très) grands nombres et/ou les ordinaux infinis. (On peut donner un côté ludique à la chose avec le jeu de l'hydre. Pour : ça intéresse facilement, voire, ça impressionne ; ça ne dépend pas trop de connaissances qu'ils pourraient avoir ou ne pas avoir. Contre : ça peut donner l'impression d'être peu rigoureux, et on peut facilement larguer les gens dans les définitions sans leur donner de moyen de se rattraper ; certains risquent d'avoir déjà entendu de la vulgarisation à ce sujet.)
  • La géométrie sphérique et la géométrie hyperbolique (voir cette entrée et les quelques suivantes). (On peut donner un côté ludique à la chose en montrant mes différents labyrinthes hyperboliques. Pour : c'est visuel et ça accroche facilement. Contre : ils ne connaissent pas forcément grand-chose en trigonométrie, donc difficile d'introduire la formule fondamentale qui permet de faire plein de calculs réels. Autre problème pratique : les illustrations sont très fastidieuses à réaliser pour moi.)
  • Quelques notions de théorie combinatoire des jeux et notamment comment gagner au jeu de nim (un peu comme ici mais sans les trucs infinis). (Pour : ils ressortent avec quelque chose de vraiment utilisable — à savoir la stratégie gagnante de jeux comme nim, des jeux de retournement de pièces, voir nim⊗nim ; sur les jeux de retournement de pièces, je peux introduire des codes correcteurs ; le tout serait sans doute facile à comprendre et ils n'auront sans doute pas vu avant. Contre : ça peut donner l'impression d'être très anecdotique.)
  • Quelques notions de géométrie finie (voir ici et pour des illustrations). (Contre : n'ayant pas vu de géométrie projective avant, l'élégance de l'idée de construire des structures combinatoires à partir de notions géométriques risque de leur échapper complètement.)
  • …et sans doute plein d'autres choses dont j'ai parlé à l'occasion sur mon blog, comme le problème de Hadwiger-Nelson (pas sûr qu'on puisse tenir une heure avec ça), le lemme de Higman (ça fait une démonstration complète et très accessible, mais c'est sans doute très peu vendeur), l'automorphisme exceptionnel de 𝔖₆ (peut-être pas très motivant).
  • Les cardinaux infinis. (Pour : ça a l'avantage de permettre de faire des vraies démonstrations : argument diagonal de Cantor et/ou théorème de Cantor-Bernstein. Contre : c'est peut-être aride ; et comme pour les ordinaux, ça peut donner l'impression d'être peu rigoureux.)
  • Les groupes finis, vus comme des groupes de permutations, et présentés comme des puzzles (cf. ceci).
  • Une introduction à la géométrie projective.
  • …et encore plein d'autres choses.

(Sujets triés par ordre approximatif d'intérêt/faisabilité a priori.)

PS : Je dois fournir un titre rapidement, donc c'est plutôt pressé !

PPS : Idéalement, j'aimerais arriver à faire au moins une « vraie » démonstration pendant mon exposé, mais je me rends compte que c'est mal parti. Certains sujets le permettent quand même mieux que d'autres.

Fin : Finalement, j'ai choisi de faire un exposé sur la théorie des jeux, dont le titre sera Jeu de nim : thème et variations. (Comme je l'explique en commentaires, les géométries sphérique et hyperbolique m'ont paru trop difficiles à présenter à des élèves qui connaissent a priori très peu de trigonométrie et pas la fonction exponentielle — ni à plus forte raison les lignes trigonométriques hyperboliques. Quant aux grands nombres et ordinaux, c'est sans doute plus facile de trouver en ligne de la vulgarisation à ce sujet, et j'avais peur par ailleurs que ça puisse en perdre rapidement plus d'un, et/ou que ça donne l'impression d'être peu rigoureux, foire fumeux. Les jeux dont je vais parler, au contraire, sont quelque chose de bien concret et sur quoi on peut « mettre les mains ».) • Je parlerai au moins du jeu de nim, de ses différentes variations et déguisements, et de jeux de retournement de pièces (ce que Berlekamp, Conway et Guy appellent, avec leur terminologie inimitablement baroque, Moebius, Mogul et Gold Moidores, et peut-être leurs liens avec les codes correcteurs ; ou de façon générale, de certaines choses qu'on trouve au tout début du volume ♣ de Winning Ways).

Ajout : voir une entrée ultérieure.

↑Entry #2427 [older| permalink|newer] / ↑Entrée #2427 [précédente| permalien|suivante] ↑

↓Entry #2425 [older| permalink|newer] / ↓Entrée #2425 [précédente| permalien|suivante] ↓

(mercredi)

La magie du nombre six redessinée sous forme pentagonale

L'avant-dernière entrée était consacrée au commentaire mathématique d'un dessin illustrant une propriété magique du nombre six : l'existence de six « pentades » (c'est-à-dire six façons de regrouper trois par trois les doublets sur six objets de manière que deux doublets regroupés ne partagent jamais un objet) ; ce dessin était présenté sous forme « hexagonale », c'est-à-dire que chacune des pentades montrait les six objets sous la forme des six sommets d'un hexagone régulier, ce qui à son tour suggérait une certaine disposition des pentades elles-mêmes (comme la permutation cyclique de l'hexagone fixe une pentade, en échange deux, et permute cycliquement les trois dernières, j'avais choisi une disposition et un coloriage qui mettait en évidence ces transformations). On m'a convaincu de refaire le même dessin sous forme « pentagonale », c'est-à-dire en disposant les six objets sous la forme des cinq sommets d'un pentagone régulier plus son centre. Voici le résultat (il s'agit donc, conceptuellement, du même dessin, mais où les objets ont été disposés différemment, les pentades aussi, et les couleurs sont différentes) :

Cette fois, la disposition pentagonale suggère de s'intéresser à la permutation cyclique des cinq objets disposés selon les sommets du pentagone : ce 5-cycle permute aussi les pentades selon un 5-cycle, ce qui suggère de les disposer elles aussi de façon pentagonale, avec au centre celle qui est fixée par le cycle, et en pentagone autour celles qui sont permutées cycliquement. J'ai donc choisi comme couleurs le noir et cinq couleurs maximalement saturées disposées régulièrement sur le cercle chromatique (bon, c'est plutôt un hexagone chromatique, mais peu importe). Du coup, tout le dessin est laissé invariant si on effectue une rotation de 2π/5 (=un cinquième de tour) en permutant aussi cycliquement les couleurs.

En plus de cela, le choix de la disposition définit ce que j'aime appeler une polarité symétrique sur l'ensemble à six objets : cela signifie que si on met en correspondance chaque objet avec la pentade qui occupe « la même place » dans la disposition graphique, alors l'automorphisme qui en résulte est involutif, au sens où une pentade de pentades va reprendre la place de l'objet qui lui correspond naturellement (on pourrait, du coup, se figurer ce dessin comme une structure fractale où le petit disque représentant chaque objet est remplacé par le dessin de la pentade correspondante, et ainsi de suite à l'infini). J'ai essayé de donner aux objets les mêmes couleur que les pentades, mais j'ai trouvé que ça embrouillait plutôt qu'autre chose.

Je n'arrive pas vraiment à décider, mais je crois quand même que je préfère la forme hexagonale du dessin. La forme pentagonale est peut-être un chouïa plus symétrique, mais c'est une symétrie moins bonne, parce qu'elle donne un rôle particulier à un des objets (en le plaçant au centre du pentagone) ; et, de façon plus grave, elle donne l'impression que la correspondance objets↔pentades que j'appelle polarité symétrique ci-dessus est naturelle alors qu'elle résulte de la disposition pentagonale (or tout l'intérêt de l'automorphisme extérieur de 𝔖₆ est justement que les pentades ne sont pas en correspondance naturelle avec les objets). Mais ça a certainement un intérêt de voir ces deux dessins (et d'essayer de se convaincre que c'est bien la même chose).

(Pour aller un cran plus loin, ça peut être intéressant de se convaincre que quelle que soit la manière dont on décide d'identifier les objets du dessin « pentagonal » avec les objets du dessin « hexagonal », il en découle une identification des pentades, et inversement, quelle que soit la manière dont on décide d'identifier les pentades, il en découle une identification des objets.)

Ajout () :

On me fait la remarque suivante : plutôt que disposer mes six objets selon un pentagone régulier plus son centre, ce qui en distingue un, j'aurais pu les disposer selon les sommets d'un icosaèdre régulier modulo antipodie (c'est-à-dire, en identifiant deux sommets opposés ; ou si on préfère, selon les six diagonales centrales d'un icosaèdre régulier). Je ne vais pas faire la représentation graphique parce que ce serait trop pénible, mais en fait c'est très intéressant : cette disposition icosaédrale évite de distinguer un objet, mais elle distingue toujours une pentade privilégiée, et c'est presque exactement ce qu'elle fait.

Plus exactement : le groupe des isométries directes de l'icosaèdre est isomorphe au groupe alterné (=groupe des permutations paires) 𝔄₅ sur cinq objets, et l'automorphisme extérieur de 𝔖₆ est justement une façon de se représenter les choses. Placer les six objets aux sommets d'un icosaèdre modulo antipodie définit une pentade privilégiée (à savoir, l'unique pentade laissée fixée par la rotation d'angle 2π/5 autour d'un sommet quelconque de l'icosaèdre) ; et les isométries directes de l'icosaèdre sont précisément les permutations paires sur les 5 pentades restantes (i.e., fixant cette pentade privilégiée). Les 5 synthèmes de la pentade privilégiée peuvent se voir comme 5 sextuplets d'arêtes de l'icosaèdre (sextuplets parce que ce sont des triplets d'arêtes opposées) dont les milieux forment un octaèdre, ce qui permet de retrouver une description classique du groupe des isométries de l'icosaèdre comme les permutations paires sur cinq octaèdres inscrits dans l'icosaèdre. (Il est pertinent de remarquer au passage qu'un permutation sur six objets est paire si et seulement si la permutation correspondante sur les pentades l'est.)

On doit aussi pouvoir faire le lien avec des structures de droite projective sur le corps à cinq éléments : comme les pentades sur six objets sont aussi en bijection avec toutes les façons de voir les six objets comme la droite projective sur 𝔽₅, ça veut dire qu'il y a une structure de droite projective sur 𝔽₅ « naturelle » (privilégiée) sur les sommets d'un icosaèdre modulo antipodie. Je soupçonne qu'il y a une jolie façon de la voir en réduisant modulo 5 les birapports des sommets de l'icosaèdre dans quelque chose, mais les détails m'échappent.

↑Entry #2425 [older| permalink|newer] / ↑Entrée #2425 [précédente| permalien|suivante] ↑

↓Entry #2423 [older| permalink|newer] / ↓Entrée #2423 [précédente| permalien|suivante] ↓

(lundi)

Sur la magie du nombre six (l'automorphisme exceptionnel de 𝔖₆)

J'ai posté dans une entrée récente le dessin suivant, avec la devinette d'essayer de trouver ce qu'il représente et ce qu'il nous apprend :

Les réponses dans les commentaires ont été intéressantes (et j'ai bien fait de proposer cette devinette), parce que plusieurs personnes ont remarqué des aspects différents du dessin, et ont fait des observations justes et pertinentes. La réponse mathématique que je vais tenter d'expliquer tourne autour du fait que les matheux énoncent classiquement en disant que le groupe des permutations sur six objets (et uniquement sur six objets) possède un « automorphisme extérieur non-trivial » ; mais cette formulation n'a aucun sens pour les non matheux, et même pour les matheux je trouve qu'elle ne fait pas vraiment ressortir pourquoi ce fait est remarquable et exceptionnel. Donc le mieux est peut-être de formuler le fait remarquable sous la forme suivante (qui est certes un peu de l'agitage de mains, mais qu'on peut rendre rigoureux, et que je trouve en tout cas plus parlant), et c'est ça que je vais essayer d'expliquer :

À partir de six objets, il est possible de construire, de façon systématique, de nouvelles « choses », également au nombre de six, tout aussi interchangeables que les objets de départ, mais qui ne peuvent pas être mis en correspondance systématique avec eux.

De plus, ceci n'est possible pour aucun autre nombre que six.

Pour les mathématiciens qui aiment la théorie des catégories, ce qui précède est censé signifier la chose suivante : le groupoïde formé des ensembles de cardinal 6 avec les bijections pour morphismes admet un endofoncteur fidèle (donc automatiquement une autoéquivalence) mais qui n'est pas naturellement isomorphe à l'identité ; et ce n'est vrai pour aucun autre entier naturel que 6.

C'est un exemple d'un de ces phénomènes exceptionnels en mathématiques, comme on nomme des structures intéressantes qui apparaissent uniquement dans un petit nombre de cas : en l'occurrence, cet « automorphisme exceptionnel de 𝔖₆ » fait partie d'une sorte de chemin magique d'objets exceptionnels, qui le relie aussi aux groupes de Mathieu ou au système de racines de E₆ et aux vingt-sept droites sur la surface cubique. Mais celui-ci a l'intérêt d'être raisonnablement facile à expliquer, surtout avec mon (j'espère) zouli dessin (censé représenter ces six « choses » qui, plus bas, s'appellent des pentades).

Au passage : la notation 𝔖₆ (vous devriez voir une S gothique avec un 6 en indice) désigne le groupe des permutations sur 6 objets, c'est-à-dire l'ensemble des façons de leur faire changer de place (ou pas) ; voir aussi cette entrée antérieure et cette vidéo YouTube pour une description animée des différents sous-groupes transitifs de 𝔖₆ (c'est-à-dire, toutes les façons de permuter six objets qui sont capables de placer n'importe quel objet à n'importe quel endroit).

Après, je dois avertir que, si je suis parti pour expliquer ça, mon enthousiasme s'est un peu atténué en chemin, et la fin de cette entrée est sans doute un peu bâclée (j'avoue que j'ai passé tellement de temps à trouver le bon chemin pour expliquer proprement la combinatoire des synthèmes et pentades ci-dessous qu'à la fin j'en avais marre, et j'ai plutôt traîné des pieds pour la finir). Je la publie telle quelle en espérant qu'elle ait un certain intérêt, même si je me rends compte qu'elle est bancale et un peu décousue. (Par ailleurs, si on n'est pas intéressé par les détails, ne pas hésiter à sauter les démonstrations, qui ne sont pas franchement indispensables pour la compréhension de l'ensemble.)

Partons, donc de six objets. On pourra imaginer si on veut qu'ils sont placés aux six sommets d'un hexagone, comme dans chacun des hexagrammes ci-dessus ; ou bien qu'ils sont numérotés 0,1,2,3,4,5 : ça n'a aucune importance (et je vais tâcher de préciser cette absence d'importance plus loin). Je vais introduire quatre termes désignant des structures de complexité croissante fabriqués sur ces six objets : outre les 6 objets eux-mêmes, je vais définir les 15 doublets, les 15 synthèmes et les 6 pentades (ces dernières étant, essentiellement, ce que j'ai représenté ci-dessus). Précisément :

  • Les objets sont ces six choses dont je suis parti. Il y a donc 6 objets.
  • Les doublets sont les paires d'objets : par « paire » j'entends la donnée de deux objets (différents) sans qu'il y ait un ordre particulier entre les deux. Ainsi, si mes objets sont représentés comme les six sommets d'un hexagone, les doublets sont toutes les arêtes et diagonales de l'hexagone (tous les segments représentés sur l'un des dessins ci-dessus). Si les objets sont numérotés 0,1,2,3,4,5, alors les doublets peuvent être numérotés 01,02,03,04,05,12,13,14,15,23,24,25,34,35,45 : remarquez qu'il n'y a pas de 21, par exemple, dans ma liste, parce que c'est la même chose que 12 (c'est en ce sens que je dis qu'il s'agit de paires sans ordre ou non ordonnées).

    Il y a 15 doublets : ceci peut se voir soit en comptant l'énumération que je viens de faire (et en se convainquant qu'il n'y a ni omission ni répétition), soit en faisant le raisonnement que pour choisir un doublet, on choisit un premier objet parmi 6, puis un second parmi 5, et on doit ensuite diviser par deux parce qu'on a obtenu chaque doublet deux fois (selon que l'un ou l'autre objet a été choisi en premier) ; bref, il y a 6×5÷2=15 doublets.

    Je dirai par ailleurs que deux doublets distincts sont enlacés (c'est moi qui invente le mot, il n'est pas standard) lorsqu'ils ont un objet en commun : par exemple, si j'ai numéroté les objets, les doublets 02 et 23 sont enlacés (ils ont l'objet 2 en commun), tandis que 02 et 13 ne sont pas enlacés.

  • Maintenant, ça se complique. Un synthème est la donnée de trois doublets (distincts, sans ordre) dont aucun n'est enlacé avec un autre, c'est-à-dire, ne faisant intervenir aucun objet en commun ; autrement dit, il s'agit d'une façon de regrouper mes six objets en trois doublets, l'ordre n'ayant pas d'importance. Si on préfère, c'est une façon d'apparier (« marier ») les objets deux par deux. Par exemple, si je numérote mes objets, 01/23/45 est un synthème (formé des doublets 01, 23 et 45 : on apparie 0 avec 1, et 2 avec 3, et 4 avec 5) ; de même, 03/14/25 est un synthème. Sur les dessins ci-dessus, si vous regardez un quelconque des hexagones et une couleur particulière, il y trois segments de cette couleur, c'est-à-dire trois doublets, qui constituent un synthème (autrement dit, ils n'ont aucun objet/sommet en commun).

    Combien y a-t-il de synthèmes ? On peut faire le raisonnement suivant : pour construire un synthème, je choisis un parmi les 15 doublets ; puis je dois en choisir un autre qui ne fait intervenir aucun des objets du premier doublet, ce qui me laisse 4×3÷2=6 possibilités pour le second doublet ; puis je choisis le troisième, et là, je n'ai plus du tout de possibilité ; et en faisant tout ça, j'ai compté six fois chaque synthème puisque j'ai pu prendre ses trois doublets dans n'importe quel ordre, et il y a six ordres possibles : je me retrouve donc avec 15×6÷6=15 synthèmes. Voici un raisonnement peut-être plus simple : pour construire un synthème, je choisis l'objet que je vais apparier avec l'objet 0, j'ai donc 5 possibilités de choix (tous les objets sauf 0), puis je considère le premier objet non encore apparié et je choisis avec quel objet je vais l'apparier, ce qui me laisse 3 choix possibles (à savoir, n'importe quel objet autre que les 2 déjà appariés et l'objet que je cherche à apparier), et une fois ces choix faits, le synthème est complètement déterminé (car il ne reste que deux objets à apparier, et on ne peut donc que les mettre ensemble), donc j'ai 5×3=15 synthèmes.

    On peut aussi les énumérer exhaustivement : visuellement, cela se fait très bien, et voici les 15 synthèmes représentés graphiquement (faites défiler horizontalement) :

    Ou si on préfère numéroter les objets, ils sont (dans l'ordre utilisé ci-dessus si les objets sont numérotés de 0 à 5 dans le sens contraire des aiguilles d'une montre à partir de celui qui est à droite) : 03/14/25, 01/23/45, 05/12/34, 03/15/24, 02/14/35, 04/13/25, 03/12/45, 05/14/23, 01/25/34, 04/12/35, 04/15/23, 02/15/34, 02/13/45, 05/13/24, 01/24/35.

    Je dirai par ailleurs que deux synthèmes distincts sont enlacés lorsqu'ils n'ont pas de doublet en commun. (Je sais, ça peut sembler inversé : j'ai défini deux doublets comme enlacés lorsqu'ils ont un objet en commun ; mais on va voir que c'est logique.) Par exemple, 03/14/25 et 01/23/45 sont enlacés, tandis que 03/14/25 et 03/15/24 ne le sont pas (ils ont le doublet 03 en commun).

  • Quatrième et dernière définition : une pentade (également appelée pentade synthématique ou total synthématique) est formée de cinq synthèmes (distincts, sans ordre) qui sont tous enlacés les uns avec les autres : autrement dit, c'est une façon de répartir les quinze doublets trois par trois pour former cinq synthèmes.

    Pour dire les choses de façon un peu différente : une pentade est une manière de colorier les quinze doublets avec cinq couleurs de façon que deux doublets distincts enlacés (=ayant un objet commun) ne soient jamais de la même couleur (il est facile de se convaincre qu'il y aura alors forcément trois doublets, donc un synthème, de chaque couleur) ; je souligne que l'identité des couleurs n'a aucune importance (si on échange deux couleurs, la pentade reste la même), seul compte le fait que deux doublets aient ou n'aient pas la même couleur.

    Chacun des six hexagones de mon dessin initial représente une pentade, figurée par un coloriage des segments : si on se concentre sur un des hexagones, chacune des couleurs représente un synthème de la pentade, et la pentade est la répartition des doublets en ces cinq synthèmes. On peut se convaincre que les six pentades dessinées sont toutes distinctes (j'insiste : il ne s'agit pas simplement de voir que les couleurs sont différentes, mais que la répartition des doublets entre les synthèmes est différente).

    On pourrait s'imaginer qu'il y a beaucoup de pentades, mais en fait, il y en a a exactement six (i.e., je les ai toutes dessinées, chacune une seule fois, ci-dessus). Je démontrerai plus loin ce fait qui rend toute l'histoire intéressante.

Pour résumer tout ce qui précède, les 6 objets définissent 15 doublets (chacun formé de 2 objets distincts) ; on a aussi défini 15 synthèmes (chacun formé de 3 doublets distincts mutuellement non enlacés), et enfin des pentades (au nombre de 6 mais on ne le sait pas encore, chacune formée de 5 synthèmes distincts mutuellement enlacés). Mon but est d'expliquer qu'il y a une forme de « symétrie » qui échange objets et pentades en même temps qu'elle échange doublets et synthèmes.

↑Entry #2423 [older| permalink|newer] / ↑Entrée #2423 [précédente| permalien|suivante] ↑

↓Entry #2418 [older| permalink|newer] / ↓Entrée #2418 [précédente| permalien|suivante] ↓

(dimanche)

Sur les adjectifs qui élargissent le nom qu'ils qualifient

Le point de grammaire(?) que je veux évoquer ici concerne surtout la terminologie scientifique, notamment mathématique, même s'il est a priori complètement général.

Normalement, quand on accole une épithète à un nom, ou en fait n'importe quelle sorte de complément, le sens devrait être de préciser, c'est-à-dire de restreindre, l'ensemble des entités possiblement désignées. Par exemple, même si vous ne savez pas ce que c'est qu'un foobar (c'est normal !), ni ce que signifie l'adjectif cromulent (idem), si je parle d'un foobar cromulent, vous pouvez conclure qu'il s'agit d'une sorte particulière de foobar, qui a une propriété additionnelle (être cromulent) par rapport à celle d'être un foobar. De même, un bazqux roncible frobnicable devrait être un type spécial de bazqux roncible, qui est lui-même une sorte de bazqux ; et le groupe des ptérodoncles mouffetés de Linné devrait être un ensemble (d'animaux ?) plus restreint que celui des ptérodoncles.

Je suis sûr que les grammairiens ou les linguistes ont un terme précis pour ce phénomène, mais je ne le connais pas ; ou peut-être, au contraire, un terme pour les exceptions. Car il y a bien sûr des exceptions. Dans le langage courant, elles abondent. Un secrétaire général n'est pas vraiment un secrétaire (et pas du tout un général, mais ça c'est plutôt une blague). Un procureur adjoint n'est pas un procureur, puisqu'il n'est qu'adjoint (et il en va de même d'adjectifs comme délégué). Un faux bourdon n'est évidemment pas un bourdon, comme un faux acacia n'est pas un acacia : on peut s'attendre à ce qu'un faux foobar ne soit pas un foobar, d'un autre côté, une fausse bonne idée est quand même une idée, même si elle n'est pas une bonne idée. Il y a aussi tout ce qui est nommé par métonymie ou par métaphore : un blouson noir n'est pas une sorte de blouson et un visage pâle n'est pas une sorte de visage ; une peau de chagrin était bien ce que ça dit jusqu'à ce qu'un roman de Balzac donne un sens très particulier à cette expression. Et ainsi de suite. Évidemment, les frontières des mots dans le langage non-technique ne sont pas rigoureusement définies, donc il n'est pas toujours possible de décider avec certitude si un adjectif est ou n'est pas restrictif au sens du paragraphe précédent : un tableau noir est-il un type particulier de tableau, par exemple ? certainement si on prend tableau au sens le plus large, mais ce n'est pas ce qu'on entend normalement par ce mot. Un hôtel de ville est un hôtel pour une certaine définition d'hôtel, mais ce n'est plus vraiment le sens courant de ce mot. Et je ne saurais pas vraiment dire si un coup de soleil est une sorte de coup, ou si le clair de lune est une sorte de clair (whatever that may be).

Dans le vocabulaire technique, on pourrait espérer que les mots aient un sens suffisamment précis pour pouvoir éviter ces gags, mais ce n'est pas le cas. En mathématiques, un faisceau pervers n'est pas un faisceau et en physique, un champ quantique n'est pas un type particulier de champ [classique] mais un concept parallèle dans un cadre adjacent (la théorie quantique des champs), et il est discutable qu'une étoile à neutrons soit une étoile. Sans compter, bien sûr, les cas où le terme technique est une locution indivisible : un trou noir (terme technique) n'est pas une sorte particulière de trou (terme non technique). La situation reste beaucoup plus rare que dans le langage courant.

Il y a cependant une situation importante où un foobar cromulent n'est pas une sorte particulière de foobar, et dont les matheux ont assez souvent besoin, et peut-être aussi d'autres sciences (les exemples ne me viennent pas trop à l'esprit, mais je suppose qu'ils doivent exister), ce sont les cas où on veut au contraire élargir le sens d'un mot. Autant la situation normale est que l'adjectif restreint le sens d'un mot, et les diverses situations évoquées jusqu'ici sont des cas où il déplace (comme faux, adjoint, etc.) ou bien le transforme de façon complètement imprévisible et figée par l'usage (blouson noir), la situation d'élargissement est encore un peu autre chose.

Le cas d'usage typique pour les maths est qu'un foobar est défini par différentes propriétés, et on veut désigner un objet qui vérifie toutes les propriétés du foobar sauf une. On peut bien sûr appeler ça un quasi-foobar ou un pseudo-foobar ou un presque foobar (near foobar en anglais ; certains grammairiens grincheux pourraient râler de voir un adverbe — presque — qualifier un nom), ou ce genre de choses, mais on aura peut-être envie de parler de foobar généralisé, et là, l'adjectif généralisé élargit le sens du mot.

Mais je pense que la situation la plus fréquente est celle, très proche, où on fait tout un traité sur les foobars bleutés, alors par flemme d'écrire bleuté à chaque fois, on convient dans l'en-tête du traité : le terme foobar désignera ci-après, sauf précision du contraire, un foobar bleuté. Une fois cette convention faite, pour parler d'un foobar en général, on doit écrire foobar non nécessairement bleuté, et non nécessairement bleuté est une locution adjectivale qui a cette propriété d'élargir le sens du mot foobar (en retirant la restriction bleuté). Et comme le mot nécessairement est lui-même long à dire, on écrit le plus souvent foobar non bleuté, ce qui est un abus de langage ou de logique parce qu'on veut, en fait, dire non nécessairement bleuté (i.e., foobar dans le sens où on retire la convention faite initialement qu'il est sous-entendu bleuté, mais il se pourrait qu'il soit quand même bleuté quand même). Il faut admettre que cela cause une certaine confusion, mais je ne connais aucune façon agréable de se sortir de ce problème de rédaction.

Le cas d'école est celui de la commutativité (et éventuellement de l'unitarité ou de l'associativité) des anneaux : en algèbre, un anneau est défini comme un ensemble muni d'opérations (l'addition et la multiplication) vérifiant un certain nombre de propriétés (l'associativité de l'addition, la commutativité de celle-ci, l'existence d'un neutre et de symétriques pour l'addition, la distributivité de la multiplication sur l'addition, l'associativité de la multiplication et l'existence d'un neutre pour la multiplication ; la dernière, voire les deux dernières n'étant pas systématiquement incluses dans la définition) ; et les gens qui font de l'algèbre commutative vont avoir envie d'ajouter une propriété supplémentaire, la commutativité de la multiplication, ce qui donne la notion d'anneau commutatif (commutatif étant ici un adjectif régulier, c'est-à-dire restrictif). C'est pénible d'écrire anneau commutatif trente-six fois par page, alors on fait souvent la convention que anneau signifiera désormais anneau commutatif (typiquement sous la forme : tous les anneaux considérés ici seront, sauf précision du contraire, supposés commutatifs, et peut-être, pour qu'il n'y ait aucun doute sur la définition utilisée, unitaires [i.e., possédant un élément neutre pour la multiplication] et associatifs). Mais on a quand même envie de temps en temps de dire quelque chose sur les anneaux plus généraux, alors on devrait écrire anneau non nécessairement commutatif en utilisant un adjectif qui élargit le sens du mot. Sauf qu'en fait, il n'est quasiment jamais intéressant de parler spécifiquement d'anneaux non nécessairement commutatifs qui ne sont effectivement pas commutatifs (au sens où il existe vraiment x et y tels que x·yy·x), donc on dit simplement non commutatif pour non nécessairement commutatif ; ce qui conduit à la situation absurde qu'un anneau commutatif est un cas particulier d'un anneau non commutatif (puisque ce dernier terme signifie en fait non nécessairement commutatif). C'est agaçant, j'en conviens, mais je ne connais pas de façon agréable de s'en sortir.

En fait, c'est très souvent le cas avec les adjectifs en non en mathématiques : de la même manière, un automate fini déterministe est un cas particulier d'un automate fini non déterministe (puisque ce dernier terme signifie en fait non nécessairement déterministe).

Le terme d'algèbre est particulièrement merdique parce qu'il signifie plein de choses selon le contexte : la multiplication peut être commutative et associative, ou seulement associative, ou même pas ; si on la suppose associative par défaut (ce qui est quand même le plus courant), ça n'empêchera pas d'écrire algèbre de Lie alors que le crochet de Lie n'est pas associatif (on a une autre hypothèse à la place, l'identité de Jacobi) ; de même, si on écrit algèbre alternative, il faut comprendre que l'hypothèse d'associativité a été remplacée par quelque chose de plus faible (l'hypothèse d'alternativité / de Moufang) ; et c'est pareil pour les algèbres de Jordan. Donc une algèbre de Lie, une algèbre alternative et une algèbre de Jordan ne sont (en général) pas des algèbres [associatives], ce sont des algèbres non [nécessairement] associatives, en revanche toute algèbre [associative] est une algèbre alternative. Et c'est sans compter la notion très générale d'algèbre sur une monade ! Pour le mathématicien habitué, tout ça ne pose pas trop de problème, à part un énervement certain quand on tient à la logique, mais quand il s'agit d'enseigner, c'est vraiment embêtant.

Certains proposent parfois des adjectifs différents pour rendre la terminologie moins incohérente : par exemple, si on convient qu'un corps est nécessairement commutatif (ce qui, n'en déplaise à Bourbaki, est quasiment universellement admis), lorsqu'on veut parler de corps non nécessairement commutatif, plutôt que d'écrire la longue expression corps non nécessairement commutatif ou l'abus de langage corps non commutatif, certains aiment écrire algèbre à division (avantage : c'est bien une algèbre ; inconvénient : personne ne sait au juste ce que c'est qu'une algèbre), ou corps gauche (avantage : c'est relativement court et agréable à écrire ; mais il reste que ce n'est pas un corps, et le terme n'est pas ultra standard), voire corps-gauche (le trait d'union permet de faire comme si ce n'était pas un adjectif et de prétendre qu'il est complètement normal qu'un corps-gauche ne soit pas un corps). Ça peut marcher pour des cas précis, mais ce n'est pas une solution universelle.

On pourrait aussi se demander ce qu'un adverbe est censé avoir comme effet général sur un adjectif (qui lui-même qualifie un nom) : si les foobars orgnesquement cromulents sont censés être des foobars, comment se situent-ils par rapport aux foobars cromulents ? Je ne crois pas vraiment qu'il y ait de convention absolue en mathématiques : parfois localement cromulent implique cromulent, parfois c'est la réciproque qui vaut, parfois ni l'un ni l'autre.

↑Entry #2418 [older| permalink|newer] / ↑Entrée #2418 [précédente| permalien|suivante] ↑

↓Entry #2410 [older| permalink|newer] / ↓Entrée #2410 [précédente| permalien|suivante] ↓

(mardi)

Une version de Gödel sur l'inséparabilité des théorèmes et antithéorèmes

(Le mot antithéorème, dans le titre et dans ce qui suit, désigne un énoncé P dont la négation logique, que je note ¬P, est un théorème, i.e., un énoncé réfutable alors qu'un théorème désigne un énoncé démontrable. Si vous avez du mal à distinguer vrai/faux de théorème/antithéorème, vous pouvez réviser ici.)

Je fais de temps en temps des remarques sur le théorème de Gödel (par exemple ici), il semble que ce soit un sujet dont on n'arrête pas d'extraire du jus. J'ai fait une remarque à ce sujet récemment sur MathOverflow, je me dis qu'elle pourrait intéresser mes lecteurs, donc je vais tenter de l'expliquer. Je vais essayer de reléguer les détails ou les complément un peu plus techniques à plein de notes : ceux qui veulent juste the big picture peuvent ignorer ces notes (et, dans tous les cas, il vaut peut-être mieux les garder pour une seconde lecture). Pour ceux qui veulent vraiment juste the bottom line, j'explique ici, en utilisant un tout petit peu de calculabilité, pourquoi il existe non seulement des énoncés indémontrables et irréfutables (i.e., « logiquement indécidables »), mais même de tels énoncés dont l'indémontrabilité et l'irréfutabilité sont elles-mêmes indémontrables (i.e., « logiquement indécidablement indécidables »). J'avoue qu'il y a un peu plus de subtilités dans tous les sens que ce que je pensais (i.e., beaucoup de notes), mais j'espère qu'on peut quand même en retenir quelque chose sans comprendre tous les détails.

La clé de tout ça, c'est de méditer sur la manière dont un algorithme (i.e., une machine de Turing) peut séparer les théorèmes et les antithéorèmes, ou le vrai et le faux — en gros, montrer qu'il ne peut pas, même pas en un sens assez faible.

Voici un premier fait : il est possible de produire un algorithme (i.e., une machine de Turing) qui, quand on lui donne un énoncé mathématique P, termine en répondant oui lorsque P est un théorème, et termine en répondant non lorsque P est un antithéorème (i.e., ¬P est un théorème). Il suffit, pour cela, d'énumérer toutes les démonstrations mathématiques possibles (par exemple en énumérant toutes les suites de symboles possibles, en vérifiant pour chacune s'il s'agit d'une démonstration conforme aux règles de la logique, tout ceci étant faisable algorithmiquement), et si on tombe sur une démonstration de P, on s'arrête et on répond oui, tandis que si on tombe sur une démonstration de ¬P, on s'arrête et on répond non. Je n'ai pas précisé dans quel système axiomatique je me place, cela pourrait être, par exemple, l'arithmétique de Peano [du premier ordre] PA ou la théorie des ensembles ZFC (mais dans ce cas, il faudra la supposer cohérente, ce que ZFC lui-même ne peut pas prouver, sans quoi tout énoncé serait à la fois théorème et antithéorème ce qui n'est pas bien intéressant). Bien sûr, tout cela est complètement théorique (dans la vraie vie, la démonstration automatisée ne sert que dans des théories extrêmement étroites, pas pour des énoncés mathématiques « généraux »). Mais le point théorique à souligner, c'est que l'algorithme que je viens de décrire ne termine pas si P n'est ni un théorème ni un antithéorème (i.e., s'il est logiquement indécidable dans la théorie considérée) : la contrainte est seulement que si P est un théorème, l'algorithme termine en répondant oui, et si ¬P est un théorème, l'algorithme termine en répondant non.

Voici un deuxième fait : il n'est pas possible de faire un algorithme (i.e., une machine de Turing) qui, quand on lui donne un énoncé mathématique P, termine en répondant oui lorsque P est vrai, et termine en répondant non lorsque P est faux (i.e., ¬P est vrai). En fait, ce n'est même pas possible si on se limite[#] à ce que P soit un énoncé arithmétique (c'est-à-dire, qui ne parle que d'entiers : voir ici pour une petite discussion) ; ni même si on se limite encore plus à ce que P soit un énoncé arithmétique Π₁ (c'est-à-dire un énoncé de la forme pour tout entier naturel n, on a Q(n), où Q, lui, est arithmétique et algorithmiquement testable en temps fini pour chaque n donné ; voir ici pour une discussion). La démonstration de ce deuxième fait est facile si on connaît un tout petit peu de calculabilité, plus exactement, l'indécidabilité algorithmique du problème de l'arrêt : si un algorithme comme je décrit ci-dessus (i.e., capable de dire si un énoncé est vrai ou faux) existait, il serait notamment capable de dire si l'énoncé <tel algorithme> ne termine pas quand on le lance sur <telle entrée> est vrai ou faux (ceci est bien un énoncé arithmétique, et il est même arithmétique Π₁), et du coup, de résoudre algorithmiquement le problème de l'arrêt.

[#] À vrai dire, si je ne mets pas une restriction de ce genre, c'est encore pire : on ne peut même pas énoncer formellement ce que ça voudrait dire d'avoir un algorithme qui répond oui ou non selon que l'énoncé est vrai ou faux.

Quand on met ensemble les deux faits que je viens de dire, on obtient le théorème de Gödel : en effet, s'il est possible de faire un algorithme qui répond oui sur les théorèmes et non sur les antithéorème, et impossible de faire un algorithme qui répond oui sur les énoncés vrais et non sur les énoncés faux, c'est forcément que les deux concepts ne sont pas identiques !, et donc, si tant est que tous les théorèmes de la théorie sont bien vrais (ou au moins les théorèmes arithmétiques, ou au moins[#2] les théorèmes arithmétiques Σ₁), il y a forcément des énoncés vrais, et même forcément des énoncés arithmétiques Π₁ vrais[#3], mais qui ne sont pas des théorèmes. C'est le théorème de Gödel, et c'est d'ailleurs peut-être la manière la plus simple de le voir. La construction peut être rendue explicite (car l'indécidabilité du problème de l'arrêt l'est). Je crois que cette façon de démontrer le théorème de Gödel était une motivation importante pour Turing dans l'étude du problème de l'arrêt.

↑Entry #2410 [older| permalink|newer] / ↑Entrée #2410 [précédente| permalien|suivante] ↑

↓Entry #2408 [older| permalink|newer] / ↓Entrée #2408 [précédente| permalien|suivante] ↓

(dimanche)

La forme élégante du plan projectif complexe

Je ressors ici de mes cartons une vieille entrée commencée il y a très longtemps, et plusieurs fois reprises, abandonnée, re-reprise, re-abandonnée, etc. Il s'agit d'essayer d'expliquer ce que c'est, et dans une certaine mesure comment visualiser, le plan projectif complexe[#] et sa géométrie. (Sauf qu'à cause de l'histoire compliquée de la rédaction de ce texte, qui s'étale sur des années, j'ai changé plusieurs fois d'avis sur ce que je voulais raconter, et il ne faut pas s'attendre à une grande cohérence. Mais j'espère au moins que les différents bouts seront intéressants.)

Le plan projectif complexe est intéressant parce qu'il appartient à la liste des espaces homogènes et isotropes (ou : deux points homogènes), ce que j'avais évoqué dans mon entrée sur les octonions (plus précisément, ici ; je voulais en parler depuis longtemps), et il est le plus simple/petit parmi eux qui ne soit pas maximalement symétrique, c'est-à-dire, qui ne soit pas un espace euclidien, une sphère (ou espace projectif réel) ou un espace hyperbolique : si on veut essayer d'imaginer ce que la notion d'espace homogène et isotrope signifie, et pourquoi ce n'est pas pareil que maximalement symétrique, il est donc bon de commencer par là ; d'autant plus qu'il n'est que de dimension (réelle) 4, ce qui n'est pas totalement hors de portée de l'imagination, et de toute façon tous ceux qui sont plus compliqués vont le contenir (ou bien contenir son dual, le plan hyperbolique complexe).

Mais il y a une raison supplémentaire d'en parler, c'est que le plan projectif complexe est une sorte d'amalgame entre le plan projectif réel (qui n'est autre que la sphère ordinaire, après identification des points antipodaux) et la droite projective complexe (a.k.a., sphère de Riemann, qui est elle aussi la sphère ordinaire, cette fois sans identification des antipodes, mais qu'il sera pertinent d'imaginer de rayon deux fois plus petit) : ces deux espaces-là sont faciles à comprendre, et sont aussi l'occasion de parler de deux projections particulières de la sphère, à savoir la projection gnomonique et la projection stéréographique. Car le plan projectif réel est fortement lié à la projection gnomonique de la sphère, et la droite projective complexe à la projection stéréographique. • Toutes les deux fonctionnent en projetant la sphère sur un plan tangent à elle et en projetant depuis un point appelé centre de projection (c'est-à-dire que pour projeter un point de la sphère, on trace la droite ou demi-droite partant de ce centre de projetant et reliant le point à projeter, et son intersection avec le plan choisi définit la projection) : la différence est que dans le cas de la projection gnomonique on projette depuis le centre de la sphère tandis que dans le cas de la stéréographique on projette depuis le point antipodal du point de tangence du plan choisi. La projection gnomonique préserve l'alignement (i.e., envoie les grands cercles sur des droites) et c'est d'ailleurs la seule à le faire, tandis que la stéréographique préserve les angles. (Voir aussi mes explications sur les projections de la sphère et l'application au cas de la Terre, ou encore le texte que j'avais écrit il y a bien longtemps sur le sujet de la cartographie.)

[#] Plus exactement : le plan projectif complexe muni de sa métrique/distance de Fubini-Study, qui est alors une variété riemannienne de dimension 4 ; peut-être que je devrais dire plan elliptique complexe (ou plan projectif hermitien ?) — la terminologie n'est pas totalement claire.

Table des matières

Définition rapide et résumé pour les gens pressés

Pour les lecteurs qui veulent tout de suite une définition, le plan projectif complexe est l'ensemble des triplets (u,v,w) de nombres complexes non tous les trois nuls, dans lesquels on identifie (u′,v′,w′) avec (u,v,w) lorsqu'il existe λ complexe non nul tel que (u′,v′,w′) = λ·(u,v,w) (et pour marquer cette identification, on note (u:v:w) la classe de (u,v,w), c'est-à-dire l'ensemble {(λu,λv,λw) | λ∈ℂ×}). Autrement dit, on identifie (u,v,w) et (u′,v′,w′) lorsque les trois rapports u/u′, v/v′ et w/w′ sont tous les trois égaux (plus exactement, les coordonnées nulles doivent être les mêmes d'un côté et de l'autre, et les rapports entre coordonnées non nulles de part et d'autres doivent être les mêmes). On dit que u, v, w sont les coordonnées homogènes du point (définies à un facteur multiplicatif λ commun, donc). Souvent on les prendra normalisées, c'est-à-dire que |u|²+|v|²+|w|²=1 (mais ceci ne définit toujours pas les coordonnées uniquement, car on peut encore multiplier par un complexe λ de module 1).

Pour définir le plan projectif réel, on imposera bien sûr à u,v,w d'être réels (non tous nuls) ; et pour la droite projective réelle, on imposera à w d'être nul (i.e., on n'utilise que deux coordonnées). On pourrait bien sûr définir l'espace projectif de dimension n quelconque en utilisant n+1 coordonnées homogènes. Et on peut faire la même définition avec les quaternions qu'avec les réels ou les complexes (il faut juste faire attention dans ce cas à bien fixer le sens de la multiplication : disons qu'on identifie (u,v,w) avec (λu,λv,λw) pour λ un quaternion non nul : cela revient à identifier (u,v,w) et (u′,v′,w′) lorsque u·u−1, v·v−1 et w·w−1 sont égaux ou, ce qui revient au même, que u−1·v=u−1·v′ et v−1·w=v−1·w′ et w−1·u=w−1·u′, avec les conventions évidentes lorsque des coordonnées sont nulles). Pour les octonions, en revanche, on ne peut fabriquer que la droite et le plan projectifs, et les définitions sont plus délicates.

Mais ce dont je veux surtout parler, ce n'est pas juste le plan projectif complexe, c'est aussi la distance qu'on met dessus (et que je vais motiver en commençant par le cas du plan projectif réel et de la droite projective complexe), qu'on appelle la métrique de Fubini-Study, et qui vaut dist((u:v:w), (u′:v′:w′)) = Arccos(|u·u*+v·v*+w·w*| / √((|u|²+|v|²+|w|²)·(|u′|²+|v′|²+|w′|²))) où z* désigne le conjugué complexe de z ; donc, pour des coordonnées normalisées, c'est dist((u:v:w), (u′:v′:w′)) = Arccos(|u·u*+v·v*+w·w*|), autrement dit l'arc-cosinus du module du produit scalaire hermitien entre les coordonnées normalisées. Il est facile de vérifier que cette distance ne dépend pas des coordonnées homogènes choisies.

Cette distance fait du plan projectif réel une sphère de dimension 2 et rayon 1 où les points antipodaux sont identifiés (l'identification étant par la projection gnomonique), et de la droite projective complexe une sphère de dimension 2 et rayon ½ (l'identification étant par la projection stéréographique) dite « sphère de Riemann ». Quant au plan projectif complexe, de dimension 4, il a une forme où ces deux sortes de sphères jouent un rôle important, et que j'ai tendance à décrire intuitivement comme un « tissu de sphères » (les sphères en question sont les droites projectives complexes du plan projectif complexe : il en passe exactement une par deux points distincts quelconques, et deux d'entre elles se coupent toujours en un point unique). Ce plan projectif complexe, par ailleurs, possède énormément de symétrie, puisqu'elle est homogène et isotrope (« tous les points sont interchangeables, ainsi que toutes les directions à partir d'un point »).

Je dirai encore un mot sur les plans projectifs réels contenus dans le plan projectif complexe, sur les symétries de ce dernier, et sur différentes sortes d'angles qu'on peut définir (car si tous les points se valent et que toutes les distances égales se valent, en revanche, la situation des angles est plus compliquée).

↑Entry #2408 [older| permalink|newer] / ↑Entrée #2408 [précédente| permalien|suivante] ↑

↓Entry #2386 [older| permalink|newer] / ↓Entrée #2386 [précédente| permalien|suivante] ↓

(mercredi)

Hadwiger-Nelson et autres malheurs

Les oulipiens ont inventé le concept du plagiat par anticipation, il faut peut-être que j'explore la manière dont il s'applique aux mathématiques. Pour une fois je vais raconter mes malheurs à ce sujet. Mais il faut d'abord que je donne le contexte.

J'ai déjà parlé du problème de Hadwiger-Nelson, cette question ouverte célèbre qui consiste à déterminer le nombre minimum de couleurs qu'il faut pour colorier le plan de façon que deux points situés à distance 1 (unité fixée quelconque) n'aient jamais la même couleur : on sait seulement que la réponse (i.e., le nombre chromatique du plan pour la relation être-à-distance-un) est entre 4 et 7 ; et je qualifie volontiers ça de problème ouvert le plus embarrassant des mathématiques, parce que vraiment tout le monde peut comprendre l'énoncé, un lycéen peut retrouver les bornes que je viens de donner et on n'a pas fait de progrès par rapport à ça. On peut, en revanche, essayer de changer un peu la question pour faire du progrès sur un terrain adjacent.

Vers avril 2012, j'ai réfléchi avec quelques collègues à de telles questions adjacentes (par exemple, savoir si on peut calculer d'autres invariants intéressants du graphe des points du plan avec la relation être-à-distance-un, comme sa capacité de Shannon — enfin, celle de son complémentaire, parce qu'un des collègues en question a des conventions opposées à tout le monde, et des bons arguments pour les défendre), mais nous n'avons pas trouvé grand-chose d'intéressant. • Comme je parlais du problème en question à mon poussinet, il m'a demandé ce qu'on savait du nombre chromatique pour des points à coordonnées rationnelles (i.e., le nombre minimum de couleurs qu'il faut pour colorier l'ensemble ℚ² des points à coordonnées rationnelles du plan, de façon que deux points situés à distance 1 n'aient jamais la même couleur). J'ai trouvé la solution à cette question-là (2 couleurs sont suffisantes — et évidemment nécessaires), et je l'ai exposée à mes collègues ; l'un d'eux a rapidement repéré que ce fait était déjà bien connu (le résultat est dû à un Douglas Woodall, en 1973). J'ai fait remarquer que les mêmes techniques permettaient de montrer des choses sur d'autres corps, par exemple ℚ(√3) (le corps des nombres de la forme a+b√3, où a et b sont rationnels) pour lesquel le nombre chromatique du plan vaut exactement 3, et cela a suscité un intérêt modéré.

Je suis alors tombé sur le livre d'Alexander Soifer, The Mathematical Coloring Book (publié en 2009), presque entièrement consacré au problème de Hadwiger-Nelson. Ce livre signale le résultat de Woodall (le nombre chromatique du plan à coordonnées dans ℚ vaut 2) et quelques unes de ses variations, et mentionne explicitement comme problème ouvert de trouver des nombres chromatiques d'autres corps, par exemple ℚ(√2). Je me suis rendu compte que je savais aussi calculer la réponse pour ℚ(√2) (c'est un peu plus compliqué que pour ℚ(√3)), et du coup que ça valait peut-être la peine de rédiger tout ça.

Les choses ont un peu traîné, mais j'ai mis sur l'arXiv une petite note contenant ces résultats et quelques faits liés que j'ai trouvé à dire sur le problème. Je pense qu'elle est facile à lire.

Je pense que les trois angoisses majeures du mathématicien quand il a obtenu son résultat sont : (1) de trouver une erreur dans sa démonstration, voire un contre-exemple à l'énoncé, (2) de trouver que le résultat est, en fait, quasiment trivial (i.e., au contraire du (1), trouver une démonstration « trop simple » de l'énoncé), et (3) d'apprendre que tout a déjà été fait avant. S'agissant du (1), j'ai passé (je passe toujours) un temps fou à relire, re-relire, et re-re-relire mes démonstrations, et j'ai atteint un niveau raisonnable de certitude qu'elles étaient correctes, même si je n'ai pas pu persuader qui que ce soit d'y jeter un coup d'œil. S'agissant du (2), l'angoisse est largement neutralisée quand il s'agit d'un problème ouvert répertorié (c'est notamment à ça qu'il sert de répertorier les problèmes ouverts). Restait l'angoisse numéro (3). J'ai écrit à Soifer (l'auteur du bouquin sur le sujet) pour lui demander si la question était toujours ouverte depuis 2009, mais il ne m'a pas répondu (je ne peux pas lui en tenir rigueur, je suis le premier à ne pas répondre à mes mails). J'ai cherché comme j'ai pu dans les bases de données de publications mathématiques et dans Google tout ce qui pouvait tourner autour de Hadwiger-Nelson ou tout ce qui citait le livre de Soifer ou quelques publications-clés, et je n'ai rien trouvé. En fait, presque personne ne semble faire quoi que ce soit au sujet du problème de Hadwiger-Nelson, donc je me suis dit que c'était certainement bon.

Finalement, j'ai soumis ma note à un journal en octobre dernier. Ils l'ont gardé plutôt longtemps (octobre à juillet), et je me suis dit que c'était sans doute un bon signe : si on rejette un article par manque d'intérêt, d'habitude, on le fait rapidement, alors que si on prend le temps de rentrer dans les détails mathématiques, c'est certainement que l'article est jugé assez intéressant, or je ne craignais pas trop qu'on y trouvât des fautes.

J'ai reçu hier le rapport : il commence plutôt bien, mais in cauda venenum : il m'apprend à la fin que l'immense majorité des résultats que je croyais avoir obtenus figurent déjà dans une note non publiée (et pas non plus mise sur l'arXiv, seulement sur la page personnelle de son auteur) d'un certain Eric Moorhouse de l'Université du Wyoming. Et ce Moorhouse a une très nette antériorité, puisque la version actuelle de sa note est datée de 2010 et qu'on trouve même des traces d'une version de 1999 qui contient aussi les résultats essentiels. Cette note m'avait échappé sans doute parce qu'elle n'utilise nulle part le terme Hadwiger-Nelson, et apparemment elle (ou en tout cas, sa version de 1999) avait aussi échappé à Soifer quand il a écrit son livre.

Et il n'y a pas que les résultats qui sont proches : les techniques que j'ai mises en œuvre sont quasiment identiques à celles de Moorhouse (je ne peux même pas espérer parler de démonstrations alternatives). Même la question que je soulève de savoir si le nombre chromatique de ℂ² pour la relation (xx′)² + (yy′)² = 1 est finie, est déjà dans l'article antérieur. J'ai bel et bien été « plagié par anticipation » ! Plus sérieusement, je suis dans une situation vraiment embarrassante, parce qu'on pourrait m'accuser de plagiat ; le rapporteur qui a lu ma note a eu l'intelligence de deviner que ce n'était pas le cas (et il l'écrit clairement à l'éditeur), mais je me méfierai à l'avenir avant d'accuser qui que ce soit de plagiat, parce que je me rends compte à quel point ça peut arriver facilement.

Il y a bien quelques bouts restants dans ma note qui ne sont pas contenus dans ce qu'a fait Moorhouse (pour ceux qui veulent regarder, les §2–4 sont essentiellement incluses dans son travail, sauf peut-être la borne inférieure de la proposition 4.6, mais ce n'est pas franchement passionnant, et les §5–7 partent un peu dans une autre direction), mais je vois mal comment ils pourraient être publiés, ne serait-ce que par manque de cohérence : ce sont des petites remarques éparses qui n'ont plus aucun fil conducteur. (La réponse de l'éditeur du journal auquel j'avais soumis l'article ne ferme pas complètement la porte à cette possibilité, mais il demande des révisions substantielles qui ont l'air difficiles à mener.) À vrai dire, j'espérais beaucoup pouvoir profiter de la publication de cette note pour attirer l'attention sur le problème de Hadwiger-Nelson minkowskien (=lorentzien), i.e., pour la métrique de Minkowski (ℝ² pour la relation (tt′)² − (zz′)² = 1), et sur le fait que je ne sais même pas si le nombre chromatique est fini. Mais ça ne se fait pas de publier un article avec des questions, il faut qu'il y ait des résultats nouveaux pour servir de prétexte à poser des questions. C'est vraiment triste.

En fait, je suis même assez effondré, parce que j'avais investi pas mal de temps, pas tant dans les résultats eux-mêmes mais dans la rédaction de cette note, que j'espérais rendre aussi jolie que possible.

J'ai écrit à Moorhouse pour lui faire part de mon embarras, lui présenter mes excuses d'avoir mis sur l'arXiv comme mien des résultats qu'il avait obtenus avant, et demander s'il accepterait de faire une publication jointe, mais je ne vois pas vraiment pourquoi il accepterait (par ailleurs, je ne sais pas s'il est encore actif, ou s'il lit son mail, ou s'il y répond).

Ce n'est pas la première fois que ça m'arrive de retomber sur des résultats déjà connus, en fait, ou quelque mésaventure du genre — même si c'est la première fois que c'est aussi flagrant. Deux fois pendant ma thèse, d'autres mathématiciens ont obtenu des résultats beaucoup plus forts que les miens et quasiment simultanément (là, j'avais techniquement l'antériorité, mais quand elle se joue à très très peu, ce n'est pas forcément évident pour les journaux et relecteurs, et ça a quelque chose d'un peu absurde de se retrouver à citer un article postérieur qui fait que l'article qu'on écrit n'a déjà plus aucun intérêt). Et je ne compte pas le nombre de concepts que j'ai « découverts » pour apprendre que j'étais né trop tard dans un monde déjà trop vieux : par exemple, en 2001, j'ai « découvert » les séries de Hahn, j'étais tout excité de comprendre qu'elles formaient un corps algébriquement clos, et on m'a fait savoir que j'arrivais à peu près un siècle trop tard. J'ai aussi trouvé plein de choses sur la multiplication de nim avant de découvrir que Lenstra était passé avant, etc. Ce genre de choses arrive à tout mathématicien, mais la multiplicité des cas qui m'ont touché commence à me rendre parano. Pourtant, je cherche à m'écarter des sentiers battus.

↑Entry #2386 [older| permalink|newer] / ↑Entrée #2386 [précédente| permalien|suivante] ↑

↓Entry #2368 [older| permalink|newer] / ↓Entrée #2368 [précédente| permalien|suivante] ↓

(mardi)

Le lemme de Higman expliqué aux enfants

Ceci est un peu une expérience de vulgarisation scientifique : je voudrais essayer d'expliquer et de démontrer un résultat mathématique non-trivial en m'adressant aux gens n'ayant aucune connaissance mathématique particulière (même pas, en principe, ce qu'est un nombre), mais seulement un peu de patience pour lire des explications plutôt verbeuses (bon, OK, si je demande de la patience, ce n'est pas vraiment pour les enfants, mais je ne sais pas quoi dire d'autre). Je pense que cela peut servir d'exemple pour illustrer ce à quoi peut ressembler le travail d'un mathématicien et les raisonnements qu'il fait, et surtout, pourquoi il peut s'agir de tout autre chose que de formules et de calculs. (Ceci étant, la vulgarisation mathématique est quelque chose de difficile parce qu'en plus de chercher à expliquer les concepts ou les outils eux-mêmes, il faut trouver quelque chose à répondre aux gens qui demanderont des choses comme à quoi ça sert de se poser ce genre de question ? de façon plus ou moins agressive.) Ai-je réussi à rendre les choses compréhensibles ? À vous de me le dire — enfin, à ceux d'entre vous qui ne sont pas déjà mathématiciens.

C'est aussi un petit exercice un peu oulipien : expliquer une démonstration mathématique sans utiliser de « variables » (je veux dire des choses comme le nombre n, le mot w, le langage L, l'ensemble S, etc., ou a fortiori la suite (vi)) pour désigner les objets, puisque je ne suppose pas mon lecteur familier avec cette façon de désigner les choses. (Ce petit exercice est peut-être complètement stupide, d'ailleurs, parce qu'il n'est pas clair que m'obliger à utiliser des périphrases comme le mot qu'on considérait ou le langage dont on était parti aide vraiment à comprendre, et je pense même le contraire : mais cet exercice à l'intérêt de m'obliger à limiter le nombre d'objets manipulés dans une phrase donnée, à donner des exemples, etc., donc je pense qu'il a du bon.) J'ai quand même réécrit la démonstration une deuxième fois avec ce genre de langage, pour comparer (là aussi, aux non-mathématiciens de me dire si c'est plus ou moins clair).

J'ai choisi pour l'exercice un théorème de combinatoire : le lemme de Higman. Pourquoi précisément le lemme de Higman ? Parce que c'est un résultat important, relativement récent (1952), que je trouve très joli, et dont la démonstration, simple, élégante et pas trop longue, ne fait appel à aucun concept sophistiqué, mais est un bon exemple de raisonnement pas du tout trivial aboutissant à une conclusion peut-être surprenante. Mais aussi parce que cette démonstration contient des idées mathématiques importantes (un raisonnement par l'absurde qui est une forme de descente infinie), et parce que le résultat lui-même admet des myriades d'applications et de généralisations dans toutes sortes de directions, dont certaines sont des sujets de recherche actifs, et dont certaines utilisent une démonstration relativement proche de celle que je vais présenter.

Alors, de quoi s'agit-il ?

Je commence par présenter le contexte.

On va d'abord parler de mots, et je vais expliquer exactement ce que j'entends par là. Un mot est une succession (finie) de lettres de l'alphabet. Par exemple : abracadabra est un mot (d'une longueur de 11 lettres, mais peu importe, j'ai dit qu'il n'était pas nécessaire de savoir compter). Un mot n'est pas obligé d'avoir un sens en français ou dans une quelconque autre langue : kvtyeohegwnfth est un mot valable. Un mot peut être arbitrairement long : anticonstitutionnellementologiepouettruc est un mot valable. Il peut aussi être arbitrairement court : a est un mot. On va même autoriser le mot, appelé mot vide, qui n'a aucune lettre dedans (de longueur zéro) : il y a juste un petit problème pour l'écrire parce qu'il ne se voit pas, d'où l'intérêt de mettre des guillemets autour pour qu'on le voie quand même : (est le mot vide). Une lettre peut être répétée autant de fois qu'on veut : aaaaaaaaaaaaaa est un mot parfaitement valable (et différent de aaaaaaaaaaaaa).

En revanche, on n'a pas le droit à autre chose que des lettres : pouet42truc n'est pas autorisé. Ou du moins il ne l'est pas si on est convenu à l'avance que l'alphabet est formé des lettres ‘a’, ‘b’, ‘c’, ‘d’, ‘e’, ‘f’, ‘g’, ‘h’, ‘i’, ‘j’, ‘k’, ‘l’, ‘m’, ‘n’, ‘o’, ‘p’, ‘q’, ‘r’, ‘s’, ‘t’, ‘u’, ‘v’, ‘w’, ‘x’, ‘y’ et ‘z’ à l'exclusion de toute autre : en fait, le lemme de Higman marchera tout aussi bien si je veux ajouter les chiffres dans l'alphabet, ou les caractères accentués, ou les majuscules ; ou si je prends l'alphabet grec, ou russe, ou sanskrit, ou tous les caractères chinois : la seule chose qui importe est que l'alphabet soit fini et décidé à l'avance et qu'on n'y touche plus (et on pourra toujours appeler lettres les choses qu'on a mises dans l'alphabet) ; mais pour fixer les idées dans cette explication, on va dire qu'il s'agit de l'alphabet latin minuscule, c'est-à-dire exactement des — 26 mais peu importe — caractères que je viens d'énumérer.

Ce concept étant (j'espère) clair, on va jouer à un petit jeu (à seul ou à plusieurs) consistant à écrire des mots les uns à la suite des autres.

La seule règle du jeu est la suivante : une fois qu'un mot a été écrit, il n'est plus autorisé d'écrire un mot qui s'obtient en ajoutant des lettres dans le mot en question (au début, à la fin, n'importe où au milieu, ou tout ça à la fois). Par exemple, si le mot truc a été joué, on ne peut plus jouer trucage, mais pas non plus trouc ni structure ni autruche ni tirebouchon (eh oui, dans tirebouchon il y a truc, voyez : tirebouchon) ni introductif (idem : introductif), ni cturtutrcu (cherchez bien, il y a moyen de retrouver truc dans cet ordre en retirant les bonnes lettres : cturtutrcu). Et, bien sûr, on ne peut pas rejouer truc lui-même. Si le mot a a été joué, on ne peut plus jouer aucun mot comportant un ‘a’ n'importe où. (Et si le mot vide a été joué, plus aucun mot n'est jouable et le jeu doit s'arrêter.) • Pour parler de façon plus concise, un mot qui s'obtient à partir d'un autre en ajoutant des lettres s'appellera un sur-mot, et inversement, l'autre (qui s'obtient en retirant des lettres n'importe où) s'appellera un sous-mot : donc truc est un sous-mot de tirebouchon et tirebouchon est un sur-mot de truc (et tout mot contenant la lettre ‘a’ est un sur-mot de a, et tout mot est un sur-mot du mot vide). On convient que tout mot est un sur-mot et un sous-mot de lui-même. La règle du jeu est donc : on ne peut pas jouer un mot dont un sous-mot a déjà été joué, ou encore, jouer un mot « grille » (consomme, interdit, bannit) définitivement tous ses sur-mots. C'est là la seule règle.

Évidemment, si on veut vraiment faire un jeu intéressant à partir de l'histoire, il faudra ajouter des règles décidant qui gagne (par exemple, en disant que celui qui joue le mot vide perd — si on décide qu'il gagne, le jeu n'est vraiment pas bien palpitant ; en fait, même si on décide qu'il perd, il y a une stratégie gagnante très facile). Mais ce n'est pas tellement ça qui va m'intéresser.

↑Entry #2368 [older| permalink|newer] / ↑Entrée #2368 [précédente| permalien|suivante] ↑

↓Entry #2356 [older| permalink|newer] / ↓Entrée #2356 [précédente| permalien|suivante] ↓

(mardi)

Une question d'Analyse (moyenner une fonction), et de pourquoi elle m'intéresse

Commençons tout de suite par la question qui m'intéresse (je précise que je n'en connais pas la réponse), que je vais faire suivre de commentaires mathématiques, puis métamathématico-psychologiques :

Soit f une fonction réelle 1-périodique, et L¹ sur une période (ou, si ça ne suffit pas : mesurable et bornée). Est-il vrai que pour presque tout x, la moyenne arithmétique de f(x), f(x+1/n), f(x+2/n), f(x+3/n), …, f(x−1/n), converge vers l'intégrale de f (sur une période) ?

Cette question peut se voir comme la suite d'une question que j'avais proposée en exercice : si j'appelle (n(f))(x) la moyenne dont il est question ci-dessus, je sais montrer un certain nombre de choses, par exemple que n(f) tend dans Lp vers (la fonction constante égale à) l'intégrale de f si f est Lp et p<∞, ou qu'il y a convergence uniforme si f est Riemann-intégrable. Je signale quelques autres faits apparentés (ainsi qu'une esquisse de démonstration de ce que je viens de dire) dans cette question sur math.stackexchange, où je pose la question recopiée ci-dessus et je demande aussi s'il y a convergence dans L (lorsque f est L). Au moment où j'écris, je n'ai pas eu de réponse (et la question n'a suscité que très peu d'intérêt, ouin ☹️).

Mise à jour () : Comme on me le signale en commentaire, la réponse est non : même pour f mesurable et bornée (en fait, même pour la fonction indicatrice d'une partie de ℝ/ℤ), il n'y a pas forcément convergence presque partout, ni même « quelque part », de n(f) vers f. C'est l'objet de l'article de Walter Rudin, An Arithmetic Property of Riemann Sums, Proc. Amer. Math. Soc. 15 (1964), 321–324. La démonstration de Rudin est courte et a l'air assez jolie et arithmétique. • Par ailleurs, auparavant, Marcinkiewicz et Zygmund, dans Mean values of trigonometrical polynomials, Fund. Math. 28 (1937), chapitre II, théorème 3 p. 157, avaient déjà montré que pour la fonction précise −log(|x|)/√|x| sur [−½,½], prolongée par périodicité, qui est L¹ sur une période mais non bornée, on n'a convergence nulle part. • Par ailleurs, ces articles montrent que d'autres que moi ont pensé que la question était naturelle, et d'autre part, qu'elle n'était pas triviale. (Le terme qui me manquait pour chercher était somme de Riemann : je pensais qu'une somme de Riemann était le cas associé à une subdivision quelconque, pas spécialement régulière, et qu'on n'allait donc pas trouver grand-chose de plus en cherchant ce terme que la construction de l'intégrale de Riemann.)

Mais une méta-question que je trouve aussi intéressante, c'est : pourquoi est-ce que je trouve la question ci-dessus extrêmement intéressante, importante et naturelle ? (Peut-être que je ne serai plus de cet avis si j'obtiens la réponse, mais au minimum je la trouve intéressante au sens où j'ai vraiment envie d'avoir la réponse.) Ce n'est pas juste que moyenner une fonction comme ça est une opération qui me semble très naturelle (et assez élégante) et qu'on a envie de savoir si ça converge vers l'intégrale voire, si ça donnerait une « définition » de l'intégrale de Lebesgue. L'Analyse n'est pas un sujet dont je suis un grand fan, mais à partir du moment où on me présente une « situation » mathématique (ici, le fait de moyenner une fonction 1-périodique par ses n translatés par 1/n, et de considérer la limite quand n→+∞) sur laquelle j'arrive à dire des choses, j'ai naturellement envie de me poser toutes les questions « adjacentes » à la situation : si j'ai un résultat de convergence dans Lp pour p<∞, j'ai naturellement envie de poser la question de la convergence L et de la convergence presque partout. (D'ailleurs, le mystère c'est pourquoi j'ai mis plus d'un an à me rendre compte que ces questions étaient naturelles et que je ne savais pas les résoudre !) En plus de cela, il y a toujours un degré de frustration à penser : bon sang, mais une question aussi simple et naturelle que ça, je devrais savoir y répondre !, ou au moins, trouver la réponse dans un livre/article.

J'ai souligné le mot naturel dans le paragraphe précédent, parce que c'est un aspect psychologique fondamental dans la manière dont je conçois les mathématiques : il n'y a pas que le fait que les objets soient élégamment symétriques et beaux par leur grandeur qui me motive, il y aussi le caractère naturel des questions qu'on se pose. Je me considère comme un mathématicien pur non pas parce que je ferais des choses qui ne servent à rien, mais parce que ce qui me motive quand je me pose une question de maths n'est pas qu'elle serve à quelque chose (même à l'intérieur des mathématiques), mais qu'elle soit naturelle dans le contexte. Et c'est une qualité que je ne sais pas définir (même si cela a certainement un rapport avec la simplicité) et dont je me demande à quel point elle est personnelle, voire complètement illusoire. Un autre mathématicien sera-t-il convaincu que la question ci-dessus est intéressante ? Je ne sais pas. (Pas plus que pour les questions de l'entrée précédente. En revanche, une question telle que est-il vraie que pour toute fonction réelle f il existe une partie dense à laquelle la restriction de f est continue ? est probablement « naturelle » si j'en crois les réactions que j'ai eues.)

Toujours est-il que je n'ai pas le temps d'y réfléchir sérieusement (et je ne suis pas sûr d'y connaître assez en Analyse pour avoir une chance sérieuse de savoir résoudre le problème), donc j'essaie insidieusement de convaincre d'autres gens d'y faire attention et d'y réfléchir à ma place. Wir müssen wissen — wir werden wissen! 😉

↑Entry #2356 [older| permalink|newer] / ↑Entrée #2356 [précédente| permalien|suivante] ↑

↓Entry #2355 [older| permalink|newer] / ↓Entrée #2355 [précédente| permalien|suivante] ↓

(dimanche)

Quelques théorèmes de points fixes

Je suis un peu débordé en ce moment par la préparation de deux cours[#] qui commencent dans deux semaines et dont je n'ai pour l'instant que des notes très éparses et inachevées, d'autant plus que j'enseigne autre chose en ce moment. Mais pendant la préparation d'un de ces cours, je suis tombé sur une difficulté mathématique au sujet de laquelle j'aimerais l'avis de mes lecteurs mathématiciens (il doit bien y en avoir) ou amateurs de mathématiques : ce n'est pas que je ne sache pas démontrer quelque chose, mais que je m'étonne de la façon dont je le démontre, et je trouve qu'il y a quelque chose de surprenant dans toute l'histoire. Bref, je vais commenter les ressemblances et différences entre quelques énoncés apparemment très semblables et surtout différentes démonstrations des énoncés en question.

[#] L'un de ces cours concerne la théorie des jeux ; ou plutôt les théories des jeux, parce qu'il y a plusieurs domaines que leurs spécialistes appellent théorie des jeux, selon le type de jeux étudiés, et dont l'intersection est relativement faible : pensez à celle (que je ne sais pas nommer plus précisément) qui cherche des équilibres de Nash et celle (en gros, la théorie combinatoire des jeux) qui cherche à calculer des valeurs de Sprague-Grundy, par exemple, chacune a tendance à se définir comme « la » théorie des jeux, et d'ailleurs ça m'énerve, en tout cas je voudrais parler des deux et de quelques autres encore. Mes notes en cours d'écriture sont ici. L'autre cours concerne les courbes algébriques, pour lequel il va s'agir de remanier profondément un cours de géométrie algébrique (anciennes notes ici) que je donnais déjà.

Voici quatre énoncés mathématiques très simples, en théorie élémentaire des ensembles, que je pourrais regrouper sous le label général de théorèmes de points fixes, et que je vais appeler successivement (P), (P$), (F) et (F$) :

(P) Soit X un ensemble : on note 𝒫(X) son ensemble des parties. Soit Ψ:𝒫(X)→𝒫(X) une application vérifiant les deux propriétés suivantes : (i) Ψ est progressive, c'est-à-dire que Ψ(A)⊇A pour tout A∈𝒫(X), et (ii) Ψ est croissante, c'est-à-dire que si AB alors Ψ(A)⊇Ψ(B). Alors il existe un plus petit A∈𝒫(X) tel que Ψ(A)=A (c'est-à-dire un A tel que Ψ(A)=A et que si A′ vérifie aussi Ψ(A′)=A′ alors AA′).

(P$) [Exactement le même énoncé que (P) sans supposer (i).] Soit X un ensemble : on note 𝒫(X) son ensemble des parties. Soit Ψ:𝒫(X)→𝒫(X) une application vérifiant la propriété suivante : Ψ est croissante, c'est-à-dire que si AB alors Ψ(A)⊇Ψ(B). Alors il existe un plus petit A∈𝒫(X) tel que Ψ(A)=A. [Un peu mieux : il existe un plus petit A tel que Ψ(A)⊆A, et ce A vérifie Ψ(A)=A.]

Pour les deux énoncés suivants, j'ai besoin de rappeler la notion de fonction partielle : si X et Z sont deux ensembles, une fonction partielle XZ est une fonction définie sur une partie de X et à valeurs dans Z ; on peut aussi la voir comme une partie de X×Z (à savoir, le graphe de la fonction) qui soit fonctionnelle au sens où si elle contient à la fois (x,z₁) et (x,z₂) pour le même xX alors forcément z₁=z₂. La relation fg entre fonctions partielles signifie alors que la fonction f prolonge la fonction g (i.e., que f est définie partout où g l'est, et qu'alors leurs valeurs coïncident).

(F) [Exactement le même énoncé que (P) avec des fonctions partielles XZ au lieu de parties de X.] Soient X et Z deux ensembles : on note 𝒟 l'ensemble des fonctions partielles XZ. Soit Ψ:𝒟→𝒟 une application vérifiant les deux propriétés suivantes : (i) Ψ est progressive, c'est-à-dire que Ψ(f)⊇f pour tout f∈𝒟, et (ii) Ψ est croissante, c'est-à-dire que si fg alors Ψ(f)⊇Ψ(g). Alors il existe une plus petite f∈𝒟 telle que Ψ(f)=f (c'est-à-dire un f tel que Ψ(f)=f et que si f′ vérifie aussi Ψ(f′)=f′ alors ff′). [Précision : on me fait remarquer à juste titre que cet énoncé est en fait totalement creux (cf. la mise à jour ci-dessous).]

(F$) [Exactement le même énoncé que (F) sans supposer (i), donc exactement le même que (P$) avec des fonctions partielles au lieu de parties.] Soient X et Z deux ensembles : on note 𝒟 l'ensemble des fonctions partielles XZ. Soit Ψ:𝒟→𝒟 une application vérifiant la propriété suivante : Ψ est croissante, c'est-à-dire que si fg alors Ψ(f)⊇Ψ(g). Alors il existe une plus petite f∈𝒟 telle que Ψ(f)=f. [Un peu mieux : il existe un plus petit f tel que Ψ(f)⊆f, et ce f vérifie Ψ(f)=f.]

(Nomenclature : j'appelle (P) et (P$) les énoncés sur les Parties, (F) et (F$) ceux sur les Fonctions partielles, et (P$) et (F$) les énoncés qui vous en donnent plus pour votre argent.) J'espère que j'ai écrit ces énoncés de façon à ce qu'il n'y ait pas le moindre doute sur leur signification formelle. L'objet dont chacun de ces énoncés affirme l'existence peut être qualifié de plus petit point fixe de Ψ.

Commentaires : Le sens intuitif de ces résultats est quelque chose comme le suivant : on a une opération Ψ qui, pour prendre l'exemple de l'énoncé (F), prend une fonction f et l'étend en une fonction peut-être définie sur un peu plus de points, et par ailleurs, Ψ possède une propriété de cohérence, à savoir que si on étend f, on étend aussi le résultat de l'opération Ψ(f) ; alors il existe une « clôture du vide » pour l'opération Ψ, c'est-à-dire qu'en partant de rien, l'opération Ψ vous permet d'arriver à une certaine fonction f à partir de laquelle l'opération Ψ ne la fait plus grandir. Pour donner un exemple d'application de (P$), considérer l'ensemble X=ℕ des entiers naturels, et l'opération Ψ qui à un ensemble A de naturels associe l'ensemble formé des entiers 2, 3 et tous les produits de deux éléments de A : le plus petit point fixe sera alors l'ensemble de tous les entiers qu'on peut fabriquer en multipliant 2 et 3 autant qu'on veut ensemble (à savoir l'ensemble des 2i·3j avec au moins un de i et j non-nul, mais peu importe) ; plus généralement, (P) ou (P$) peut servir à montrer l'existence de toutes sortes de « clôtures » sous des opérations variées. Généralement parlant, le concept de plus petit point fixe (ou de point fixe en général) apparaît très souvent en mathématiques, et il existe tout un labyrinthe — mais je crois vraiment que les énoncés que j'ai cités ci-dessus sont parmi les plus naturels.

↑Entry #2355 [older| permalink|newer] / ↑Entrée #2355 [précédente| permalien|suivante] ↑

↓Entry #2353 [older| permalink|newer] / ↓Entrée #2353 [précédente| permalien|suivante] ↓

(samedi)

Petites notes sur la calculabilité, et quelques remarques à ce sujet

Je donnais jeudi matin une très courte[#] introduction à la calculabilité, dans le cadre d'un cours intitulé Théorie des Langages (donc un sujet plutôt connexe que contenant) dont j'enseigne à un groupe ; des circonstances anecdotiques (des feutres manquants[#2] au début de la séance, les élèves qui filent pour aller à un partiel à la fin) ont fait que je n'ai pas pu la finir correctement. J'ai donc envoyé des notes écrites[#3] aux élèves, auxquelles je n'ai pas résisté à la tentation d'ajouter quelques compléments en petits caractères. Comme ces notes (qui sont très basiques et passablement informelles même par rapport à ce que j'ai pu raconter sur le sujet sur ce blog) peuvent peut-être intéresser d'autres gens, je les mets en ligne ici. L'approche choisie consiste à ne pas chercher à définir formellement ce qu'est un algorithme (que ce soit par une machine de Turing ou autrement), vu que de toute façon on ne demandera à personne de programmer une machine de Turing, et pédagogiquement il semble que si on formalise un modèle de calcul, cela paralyse les étudiants au point qu'ils ne comprennent plus la notion d'algorithme alors qu'en entrant ils savaient.

[#] Et je trouve véritablement triste que dans une grande école dont l'informatique est une des spécialités, le seul contact que tous les élèves auront avec des notions aussi fondamentales que le problème de l'arrêt ou la notion de problèmes décidable et semi-décidable, c'est une séance d'une heure et demie dans le cadre d'un cours plutôt consacré à autre chose (et sur laquelle il est donc difficile de les interroger à l'examen).

[#2] Obtenir des feutres qui marchent au début de chaque cours peut être une véritable quête du graal.

[#3] Ils ont aussi un poly de cours (il n'a pas l'air d'être disponible publiquement), mais j'ai suivi une présentation différente dans mon exposé, suivant le principe qu'on comprend parfois mieux quand les choses sont expliquées deux fois de façon différente, et du coup j'ai repris mes notations dans ces notes.

Mais même en racontant des choses très basiques, on peut apprendre des choses ou s'éclaircir les idées. Notamment sur deux points, tous deux plus ou moins liés à l'énumération φ0,φ1,φ2,… des fonctions calculables partielles ℕ⇢ℕ. Il faut comprendre qu'on numéroté les programmes, par exemple par taille puis par ordre lexicographique, et que φe(n1,…,nk) est le résultat de l'exécution du e-ième programme auquel on fournit les arguments n1,…,nk, la valeur étant indéfinie si le programme ne (s'exécute pas correctement ou) ne termine pas. Un point important est qu'il existe un programme universel, c'est-à-dire que la fonction (e,n) ↦ φe(n) est elle-même calculable (informatiquement, cela signifie qu'on peut écrire un « interpréteur », qui prend un programme e et un paramètre n et exécute le programme sur cette entrée ; philosophiquement, cela signifie que le fait d'exécuter un algorithme est lui-même algorithmique). Les deux points qui m'avaient un peu échappés sont les suivants :

✱ Le premier point concerne le théorème s-m-n de Kleene. Si h(m,n)=φe(m,n) est une fonction calculable des deux variables m,n, alors pour chaque valeur de m elle est calculable dans la variable n : ça c'est plus ou moins une évidence ; mais ce qui l'est moins, c'est qu'on peut algorithmiquement fabriquer un indice s(e,m) pour cette fonction, au sens où φs(e,m)(n) = φe(m,n) avec s une fonction calculable — c'est ça que dit le théorème s-m-n. Informatiquement, cela signifie qu'il y a une transformation algorithmique (le s en question) qui prend un programme e prenant deux arguments m et n (ou en fait, deux jeux d'arguments), et une valeur à donner au premier, et qui renvoie un nouveau programme s(e,m) où ces arguments ont été fixés à cette valeur. Dans toute formalisme de calcul précis (que ce soit les machines de Turing, ou un langage de programmation réel), c'est plus ou moins évident — dans un langage de programmation fonctionnel, par exemple, cela signifie curryfier la fonction et appliquer à une constante — et la fonction s sera mieux que calculable (elle sera primitive récursive, et certainement beaucoup mieux que ça, parce que ce n'est pas un problème algorithmiquement difficile de substituer une valeur dans un programme !). Mais comme je n'introduisais pas de modèle de calcul précis, je me suis demandé si ça pouvait se démontrer in abstracto, à partir de la simple existence de l'énumération des fonctions calculables partielles et l'existence d'un programme universel.

La réponse est non, il existe des numérotations des fonctions calculables partielles qui vérifient le théorème d'universalité mais pas le théorème s-m-n. Un contre-exemple est fourni en définissant à partir d'une numérotation standard φe une nouvelle numérotation ψv+1,e(0)=v (et ψv,e(0) non définie), et sinon, ψv,e(n)=φe(n) (dans tout ça, ‹x,y› désigne un codage quelconque des couples d'entiers naturels par des entiers naturels) : autrement dit, dans la numérotation ψ, on précise séparément la valeur en 0 de la fonction (y compris « non définie ») et ses autres valeurs via une numérotation standard. Sur cet exemple, toute fonction calculable partielle apparaît bien dans les ψ, mais on ne peut pas calculer, à partir de d'un indice e d'une fonction calculable partielle h parmi les ψ, un tel indice pour la fonction constante de valeur h(1), car il faudrait pour cela déterminer si h(1) est défini (i.e., termine), donc résoudre le problème de l'arrêt. Donc on ne peut pas faire de substitution dans les ψ de façon algorithmique.

Pour raconter ce contre-exemple dans des termes informatiques, imaginons un langage de programmation permettant de coder des fonctions ℕ⇢ℕ (ou ℕk⇢ℕ, enfin peu importe) et qui est un langage tout à fait banal à une particularité près : la valeur en 0 de la fonction (qu'il s'agisse d'un entier ou du fait de partir en boucle infinie) doit être précisée par une instruction spéciale au début du programme, la seule instruction qui sera lue pour calculer cette valeur en 0, les autres valeurs étant calculées par un programme « normal » (par ailleurs, cette bizarrerie ne s'applique qu'à la fonction main, si j'ose dire, du programme). Interpréter ce langage, ou le compiler vers un autre, ne pose pas de problème particulier, et ce langage permet de représenter toutes les fonctions calculables partielles, ou d'ailleurs d'écrire un interpréteur pour un langage standard (une machine de Turing, disons) ou quelque chose comme ça. Mais il ne vérifie pas le théorème s-m-n, et ceci cause des bizarreries : on ne peut pas, par exemple, compiler un programme vers ce langage sauf à calculer à la compilation la valeur de la fonction en 0, ce qui risque de provoquer une boucle infinie ; et on ne peut pas algorithmiquement remplacer un programme dans ce langage par le programme qui calcule la (fonction constante égale à la) valeur en 1 de cette fonction. Ceci suggère que le terme Turing-complet est défini de façon un peu trop vague : à mon avis, ce qui importe est que l'énumération des fonctions partielles calculées par le langage considéré soit non seulement l'ensemble de toutes les fonctions calculables partielles, mais aussi que la numérotation soit acceptable au sens où on peut de façon calculable convertir une machine de Turing en le langage en question, et on peut montrer que cela revient exactement à vérifier le théorème s-m-n (avec une fonction s calculable).

(Référence pour tout ça : Soare, Recursively Enumerable Sets and Degrees, 1987, chapitre I, exercices 5.9 à 5.11. C'est de là que je tire le contre-exemple au théorème s-m-n.)

✱ Le second point concerne la fonction « castor affairé », qui à n associe le plus long temps d'exécution possible d'une machine de Turing à ≤n états et qui termine effectivement (en partant d'un ruban vide). Il est facile de voir que fonction, appelons-la h, dépasse infiniment souvent n'importe quelle fonction calculable [totale] f, au sens où, quelle que soit f calculable, il existe une infinité de n tels que h(n)≥f(n). (En effet si ce n'est pas le cas pour une certaine fonction f, quitte à modifier un nombre fini de valeurs de celle-ci, on a h(n)≤f(n) pour tout n, et on peut alors résoudre le problème de l'arrêt pour une machine de Turing — partant d'un ruban vide — en attendant f(n) étapes où n est son nombre d'états : si la machine ne s'est pas arrêtée au bout de ce temps-là, elle ne s'arrêtera jamais.) Mais le résultat classique dû à Tibor Radó est plus fort : la fonction h du « castor affairé » finit par dominer n'importe quelle fonction calculable f, au sens où, quelle que soit f calculable, l'inégalité h(n)≥f(n) est toujours vraie à partir d'un certain point, et je n'avais pas vraiment fait attention au fait que ce n'est pas trivial de passer de l'un à l'autre.

La démonstration d'origine de ce résultat (trouvable ici) est d'une part assez peu lisible (j'arrive à la suivre pas à pas, mais l'idée générale m'échappait) et d'autre part très spécifique au cas de la fonction « castor affairé » sur les machines de Turing en comptant leurs états. Par exemple, si on définit la fonction h en appelant h(n) la plus grande des valeurs φe(0) (ou φe(e), peu importe) qui soient définies pour 0≤en (l'argument montrant qu'elle dépasse infiniment souvent toute fonction calculable marche essentiellement pareil), alors est-il encore vrai que h finit par dominer n'importe quelle fonction calculable ? La réponse est oui, comme il résulte d'un échange sur math.stackexchange (je n'ai pas osé aller sur MathOverflow pour cette question), où on a pu m'expliquer beaucoup plus clairement l'argument de Radó, ce qui m'a permis de le généraliser facilement.

(J'en ai profité pour apprendre ce qu'est un degré de Turing hyperimmune, à savoir qu'il calcule une fonction qui dépasse infiniment souvent n'importe quelle fonction calculable, ce qui n'implique pas automatiquement qu'il calcule une fonction qui finit par dominer n'importe quelle fonction calculable.)

✱ Sinon, de fil en aiguille, je suis tombé par accident sur la relation suivante : pour A et B deux ensembles d'entiers naturels, notons AB lorsqu'il existe deux fonctions calculables partielles ℕ⇢ℕ qui se restreignent en des bijections réciproques entre ces deux ensembles. C'est une notion qui me semble extrêmement naturelle, mais qui n'est pas ce qu'on appelle de façon standard un isomorphisme calculable entre les deux ensembles. Mais ce qui me frappe, c'est que je n'ai réussi à en trouver aucune mention dans la littérature. [Mise à jour : il s'agit de la relation d'équivalence calculable (ou équivalence récursive), dont les types ont été, en fait, largement étudiés, notamment ceux qui s'appellent les isols ; voir pour commencer le livre de Dekker et Myhill de 1960, Recursive Equivalence Types, ainsi que le survey par Dekker et Ellentuck, Myhill's work in recursion theory, Ann. Pure Appl. Logic 56 (1992), 43–71, et les références qu'il contient.]

↑Entry #2353 [older| permalink|newer] / ↑Entrée #2353 [précédente| permalien|suivante] ↑

↓Entry #2349 [older| permalink|newer] / ↓Entrée #2349 [précédente| permalien|suivante] ↓

(mercredi)

Quelques clarifications sur l'intuitionnisme et l'ultrafinitisme

En relisant l'entrée précédente que j'ai écrite et un ou deux commentaires qui ont été postés dessus, j'ai peur d'avoir pu laisser imaginer que je considérais les mathématiques intuitionnistes/constructives comme aussi farfelues que l'existence d'un entier strictement compris entre 3 et 4, ou même, qu'un nombre non-négligeable de mathématiciens pourraient le considérer. Ce n'est certainement pas le cas : la seule chose que je compare, c'est la frustration que peut ressentir (superficiellement) un mathématicien classique devant ces mondes étranges (comment ça, il n'est pas toujours vrai que tout nombre réel x vérifie x≥0 ou x≤0 ???). Mais il vaut la peine de se demander pourquoi, au juste, parmi les trois « abandons » suivants,

  • abandonner l'idée que toute affirmation soit vraie ou fausse (le principe du tiers exclu),
  • abandonner l'idée qu'un nombre comme 10↑(10↑100) ait un sens,
  • abandonner l'idée que 4 soit le plus petit entier après 3,

la première donne indiscutablement lieu à des mathématiques sérieuses, la seconde peut-être mais peut-être pas, et la troisième certainement pas.

Ce que veut avant tout le mathématicien, c'est que les règles du jeu soient claires. Même si on ne prend pas la position formaliste extrême qui considère les maths comme un jeu typographique formel consistant à manipuler des successions de symboles dénués de sens selon des règles arbitraires mais relativement simples[#], les mathématiciens seront sans doute unanimes pour dire qu'il est essentiel dans la pratique des mathématiques qu'il existe des règles objectives et inambiguës sur les manipulations autorisées dans l'écriture d'une démonstration, suffisamment claires pour qu'on puisse toujours, avec assez de patience, trancher un différend sur la validité d'une démonstration en détaillant n'importe quel passage incriminé jusqu'à l'application mécanique de ces règles.

Or les mathématiques intuitionnistes/constructives ont des règles claires : ce ne sont pas les mêmes que les mathématiques classiques (plus exactement ce sont un sous-ensemble, ou une restriction, selon la présentation exacte choisie ; mais du coup, on peut ajouter des axiomes supplémentaires pour compenser qui contrediraient les mathématiques classiques), mais au moins — dans leur formulation moderne[#2] — ce sont des règles indiscutablement bien formulées et objectives. Plus exactement, le mathématicien classique peut comprendre les règles des mathématiques intuitionnistes/constructives par plusieurs mécanismes :

[ajout : voir cette entrée ultérieure un petit plus précise sur l'intuitionnisme]

  • syntaxiquement : même si les démonstrations intuitionnistes ne sont pas les mêmes que les démonstrations classiques, l'objet « démonstration » (obéissant aux règles intuitionnistes) peut lui-même être considéré comme un objet des mathématiques classiques (que ce soit comme un entier par un codage de Gödel ou comme une flèche dans une catégorie, ou autre chose du genre), étudié et analysé par elles ;
  • sémantiquement : le(s) monde(s) des mathématiques intuitionnistes peuvent se « plonger » dans le monde des mathématiques classiques, c'est-à-dire que toute affirmation des mathématiques intuitionnistes peut se décoder comme une affirmation classique portant sur des objets particuliers (vivant dans un « modèle de Kripke », un topos, une structure de réalisabilité, un univers à valeurs dans une algèbre de Heyting ou quelque chose comme ça).

(Ces deux approches sont elles-mêmes reliées par des théorèmes de validité et de complétude : je ne rentre pas dans les détails.) On peut par ailleurs relier la logique intuitionniste à d'autres logiques alternatives mais classiques et bien comprises (par des procédés comme ci-dessus), par exemple la logique modale S4.

[Ajout ] Je peux au moins donner une idée de ce dont je parle sous la forme suivante. En mathématiques classiques, si on décide d'interpréter les connecteurs logiques PQ, PQ et ¬P comme décrivant l'intersection, la réunion, et le complémentaire de parties P et Q d'un ensemble T fixé, alors certainement on a ¬¬P=P (le complémentaire du complémentaire d'une partie est la partie elle-même, justement parce qu'on travaille en logique classique) et ¬(PQ)=(¬P)∨(¬Q) ; maintenant, changeons un peu le contexte, et considérons T un espace topologique, imaginons que P et Q sont des ouverts de T, que PQ et PQ désignent l'intersection et la réunion de deux ouverts, mais maintenant ¬P désigne l'intérieur du complémentaire de P (=le plus grand ouvert disjoint de P ; et plus généralement, on peut noter PQ pour l'intérieur de la réunion de Q avec le complémentaire de P, c'est-à-dire l'ouvert des points au voisinage desquels P est inclus dans Q) : alors ¬¬P ne coïncide plus forcément avec P, c'est le « régularisé » de P (=l'intérieur de son adhérence), et de même ¬(PQ) ne coïncide plus forcément avec (¬P)∨(¬Q) (alors que ¬(PQ), lui, coïncide toujours avec (¬P)∧(¬Q)) ; en fait, les règles valables en général dans cette interprétation sont précisément celles du calcul propositionnel intuitionniste, et sont une manière dont le mathématicien classique peut les comprendre (sémantiquement) : comme des affirmations sur les ouverts d'un espace topologique (classique).

D'autre part, les mêmes choses sont valables dans l'autre sens, c'est-à-dire que si on peut « expliquer » les mathématiques intuitionnistes aux mathématiciens classiques comme ci-dessus, on peut aussi « expliquer » les mathématiques classiques aux mathématiciens intuitionnistes (par exemple par l'insertion de doubles négations à des endroits stratégiques). Du coup, les mathématiciens classiques et intuitionnistes ne seront peut-être pas d'accord sur l'intérêt ou la signification des énoncés qu'ils démontrent, mais au moins chacun peut-il expliquer son travail aux autres. (Dans la pratique, bien entendu, les « mathématiciens classiques » et à plus forte raison les « mathématiciens intuitionnistes » ne sont que des archétypes idéalisés : tout le monde est capable de faire sa traduction mentale dans un sens ou dans l'autre, quelle que soit sa représentation préférée de l'Univers.)

Pour dire les choses de façon plus concise : les mathématiques classiques et intuitionnistes sont peut-être différentes, mais leur métamathématique est compatible.

Il en va tout autrement de l'idée qu'il existerait un entier strictement entre 3 et 4 : cette idée fictionnelle est présentée sans être accompagnée de règles permettant de travailler avec et de lui donner un sens. Il n'est pas exclu que de telles règles puissent exister (par exemple : en fait, ce qu'on appelle entier ici est un élément de ℕ[√13] = {u+v·√13 : u,v∈ℕ} (approche sémantique), et il faudrait remplacer les axiomes de Peano par une axiomatisation des faits les plus évidents de la théorie du premier ordre de ℕ[√13] (approche syntaxique)), et qui du coup ferait disparaître le mystère de cette idée (à défaut de lui donner un intérêt…). Mais telle quelle, l'idée est dépourvue de sens aux yeux des mathématiciens parce qu'elle est dépourvue de règles précises.

L'idée intermédiaire (l'ultrafinitisme, j'en ai déjà parlé) occupe une position intermédiaire : on peut peut-être donner un sens à l'ultrafinitisme, mais l'idée est radicale en ce sens qu'elle nécessite de changer non seulement les mathématiques mais aussi les métamathématiques. Notamment, pour refuser l'existence du nombre 10↑(10↑100), il faut refuser l'idée qu'une démonstration puisse occuper un tel nombre de symboles — or les métamathématiques classiques l'admettent (certes, on ne va pas l'écrire explicitement, mais les métamathématiques classiques admettent de considérer comme démonstrations valables des objets qui ne pourraient pas être écrits en pratique, au moins si on en a une description raisonnablement (méta)manipulable) ; pire, il faut probablement refuser l'idée qu'une démonstration puisse occuper seulement 10↑100 symboles (parce qu'en environ ce nombre là de symboles, je peux démontrer l'existence de 10↑(10↑100) à quelqu'un qui admet que la multiplication sur les entiers est totale, ce que de nombreux ultrafinitistes admettent, ce qui permet d'écrire des choses comme 10×10×10×⋯×10), et il faut donc probablement refuser l'idée même d'utiliser « librement » l'arithmétique pour faire des métamathématiques. Je ne suis moi-même pas à l'aise avec l'ultrafinitisme (j'ai vraiment du mal à ne pas considérer la position comme simplement ridicule), mais voici ce qu'écrivent Cherubin & Mannucci dans A very short history of ultrafinitism (in : Kennedy & Kossak (eds.), Set Theory, Arithmetic, and Foundations of Mathematics (Cambridge 2011)) :

First, the rejection of infinitary methods, even the ones based on the so-called potential infinite, must be applied at all levels, including that of the meta-mathematics and that of the logical rules. Both syntax and semantics must fit the ultrafinitistic paradigm. Approaches such as Finite Model Theory are simply not radical enough for the task at hand, as they are still grounded in a semantics and syntax that are saturated with infinite concepts.

Second, barring one term in the dichotomy finite-infinite, is, paradoxically, an admission of guilt: the denier implicitly agrees that the dichotomy itself is valid. But is it? Perhaps what is here black and white should be replaced with various shades of grey.

Bref, même si le programme ultrafinitiste peut sembler à quelqu'un comme moi aussi fantaisiste que l'idée qu'il y aurait peut-être un entier à découvrir strictement entre 3 et 4, il faut avoir la modestie d'admettre que peut-être des règles du jeu précises peuvent en être données, fussent-elles des règles qui imposent de réévaluer aussi les métamathématiques : peut-être le programme peut-il être éclairci comme l'intuitionnisme l'a été, et peut-être sera-t-il possible aux mathématiciens « idéalistes » de comprendre précisément les ultrafinitistes (à défaut d'être d'accord avec eux).

[#] Je ne vais pas faire l'exercice ici et maintenant, mais il est parfaitement possible de présenter un ensemble des « règles du jeu » qui soit compréhensible par à peu près n'importe qui (disons, pas plus compliqué que les règles des échecs ou du tarot) et qui, appliquées mécaniquement, permette de démontrer tous les théorèmes des mathématiques « standard » (ZFC) et uniquement ceux-ci. En ce sens, donc, n'importe qui peut faire des maths formelles : la difficulté du travail du mathématicien est de se faire une idée d'où on va dans ce jeu et comment on peut atteindre un but, et communiquer à d'autres le fait qu'on l'a atteint, sans écrire toutes les étapes intermédiaires.

[#2] Dans leur formulation moderne, c'est-à-dire, je crois, depuis les travaux de Gödel, Heyting, Kolmogorov et d'autres. Lorsque Brouwer a initialement introduit ses idées, il n'était probablement pas clair qu'elles pouvaient être rigoureusement formalisées, d'autant qu'il était lui-même profondément hostile à l'idée de formaliser les mathématiques, de les priver de leur aspect créatif/intuitif ou de les réduire à un jeu typographique ; et c'est peut-être pour ça que ces idées ont d'abord suscité une telle hostilité (non seulement elles étaient radicales, mais en outre elles n'étaient sans doute pas bien définies aux yeux de mathématiciens comme Hilbert).

↑Entry #2349 [older| permalink|newer] / ↑Entrée #2349 [précédente| permalien|suivante] ↑

↓Entry #2347 [older| permalink|newer] / ↓Entrée #2347 [précédente| permalien|suivante] ↓

(lundi)

Comment utiliser les points comme parenthèses ?

Dans une expression mathématique comme

(2+2+2)×(3+4)

les parenthèses servent à indiquer quelles sous-expressions doivent être calculées en premier (la convention, en leur absence, étant qu'on évalue les multiplications avant les additions, si bien que 2+2+2×3+4 sans parenthèses se comprend comme 2+2+(2×3)+4). Mais il existe d'autres manières possibles d'indiquer l'ordre des opérations sans utiliser de parenthèses — ou en tout cas pas sous cette forme. Une possibilité consisterait à utiliser la notation préfixe (où le symbole d'une opération binaire précède les deux quantités sur lesquelles elles s'applique, ce qui donne dans ce cas : × + + 2 2 2 + 3 4) ou bien postfixe (où l'opération binaire suit les deux quantités sur lesquelles elle s'applique, donc 2 2 + 2 + 3 4 + × comme on le taperait sur une calculatrice à notation polonaise inversée), mais ces conventions sont extrêmement peu lisibles pour un humain.

Une autre façon de noter les choses, qui me semble assez intéressante ou en tout cas instructive, même si elle n'a jamais vraiment été utilisée en-dehors de la logique, consiste à utiliser les points comme parenthèses, que je veux présenter et discuter un peu. Sur mon exemple, cette notation donnerait :

2+2+2.×.3+4

avec des points autour du symbole de multiplication pour marquer qu'il doit être effectué après les additions. (On va supposer que le point n'est pas utilisé comme séparateur décimal, ou qu'il y a quelque magie typographique qui évite l'ambiguïté : ni ici ni ailleurs dans cette entrée il n'y a de nombres fractionnaires.)

La manière dont on lit une telle expression est la suivante : on commence par la séparer aux endroits où se trouve des points, on évalue tous les morceaux qui ont un sens en tant qu'expression (en l'occurrence, 2+2+2 et 3+4), puis on réattache les morceaux remplacés par leur valeur (ce qui donne 6×7).

Lorsqu'il y a plusieurs niveaux d'imbrications, on utilise des groupes formés d'un nombre de points croissant pour séparer les niveaux : la règle est alors qu'on commence par regrouper les morceaux séparés par un seul point, puis par un groupe de deux, puis de trois, et ainsi de suite. (Ainsi, un groupe d'un plus grand nombre de points correspond à un niveau de parenthésage plus « extérieur ».) Par exemple,

(14/(1+1))×(6+7)×(30−(6+5))

peut se réécrire dans la notation « ponctuée » comme

14/.1+1:×.6+7.×:30−.6+5

et pour l'évaluer, on commence par calculer les morceaux séparés par des points qui ont un sens tout seuls (1+1, 6+7 et 6+5), puis on regroupe les morceaux séparés par de simples points (14/.1+1 soit 14/2, et 30−.6+5 soit 30−11), et enfin on regroupe les morceaux séparés par deux points. Pour plus de symétrie quant au niveau d'opération × dans le facteur central, on peut préférer écrire

14/.1+1:×:6+7:×:30−.6+5

ce qui est peut-être plus lisible, surtout si on reflète le nombre de points dans l'espacement de la formule :

14/.1+1 :×: 6+7 :×: 30−.6+5

On peut bien sûr utiliser des symboles pour les groupes de deux, trois, quatre points et ainsi de suite : si je récupère des symboles Unicode pas vraiment fait pour, l'expression 6−(5−(4−(3−(2−1)))) peut se ponctuer en 6−∷5−∴4−:3−.2−1, mais généralement on se contente de mettre plusieurs caractères ‘.’ ou ‘:’ d'affilée pour représenter un groupe, comme 6−::5−:.4−:3−.2−1 (il faut traiter ces deux écritures comme parfaitement synonymes).

Les points servent donc à la fois de parenthèses ouvrantes et fermantes : il n'y a en fait pas d'ambiguïté car la directionalité est indiquée par la position par rapport aux symboles d'opérations (si je vois 20−.1+1, cela ne peut signifier que 20−(1+1) car (20−)1+1 n'a pas de sens) ; plus exactement, chaque groupe de points doit être adjacent à un symbole d'opération (sauf si on omet la multiplication, cf. ci-dessous), et correspond à une parenthèse soit ouvrante soit fermante selon qu'il est immédiatement après ou avant l'opération. Et la parenthèse court jusqu'au prochain groupe de points (vers la droite ou vers la gauche, selon le cas évoqué) dont le nombre de points est supérieur ou égal à celui considéré, ou à l'extrémité de l'expression (où se sous-entend un nombre infini de points, si on veut ; ainsi, sur mon premier exemple, on écrit 2+2+2.×.3+4 et non .2+2+2.×.3+4.).

Pour ceux qui veulent des règles plus formelles, je propose les suivantes. En écriture, si on a un arbre d'analyse formé d'opérations possiblement associatives, disons x1x2⋆…⋆xk (pour une certaine opération ici notée ⋆, et avec k=2 si l'opération ⋆ n'est pas supposée avoir d'association par défaut), pour la transformer en « expression ponctuée », on écrit de façon récursive chacun des sous-arbres x1,x2,…,xk comme expression ponctuée, et on concatène ces écritures en plaçant à gauche de chaque symbole ⋆ un groupe de points dont le nombre est strictement supérieur au nombre de points de n'importe quel groupe apparaissant dans l'écriture de la sous-expression gauche (si celle-ci est un atome = une feuille de l'arbre, c'est-à-dire un nombre ou une variable, on peut ne mettre aucun point) ; et de même à droite. Il est admissible de mettre plus de points que nécessaire, par exemple si on veut mettre le même nombre à gauche et à droite de chaque ⋆ intervenant à un niveau donné. On peut, bien sûr, avoir des règles supplémentaires lorsqu'on suppose une certaine priorité des opérations (par exemple, (3×2)+1 peut être noté 3×2+1 si on admet que la multiplication est prioritaire sur l'addition ; toutefois, ceci ne s'applique essentiellement qu'au niveau le plus bas : (3×(1+1))+1 devra certainement être noté 3×.1+1:+1, parce qu'on ne gagnerait rien que de la confusion à le noter 3×.1+1.+1). • Inversement, pour décoder une telle expression, on va, pour n allant de 0 au nombre maximum de points dans un groupe, remplacer chaque expression maximale de la forme x1x2⋆…⋆xk avec les xi des sous-arbres déjà constitués (ou des atomes), en ignorant les groupes de ≤n points pouvant intervenir à gauche ou à droite de l'opération ⋆, par un sous-arbre (ou un bloc parenthésé, si on préfère).

Ce système de notations ne recouvre pas tous les cas possibles d'usage des parenthèses. Disons qu'il nécessite plus ou moins qu'il y ait des symboles d'opérations dans l'histoire : si on a affaire à un contexte mathématique dans lequel on donne un sens différent aux notations u(v) et (u)v (ce qui, honnêtement, ressemble à une très mauvaise idée), ou à u et (u) (même remarque), alors on ne peut pas utiliser des points à la place des parenthèses.

Néanmoins, il marche dans des situations un peu plus générales que ce que j'ai présenté ci-dessus. Par exemple, il continue de fonctionner même si on décide de ne pas écrire le symbole × de multiplication : notamment, si dans la version parenthésée, au lieu de (14/(1+1))×(6+7)×(30−(6+5)) je décide d'écrire (14/(1+1))(6+7)(30−(6+5)), alors de même dans la version ponctuée, au lieu de 14/.1+1:×.6+7.×:30−.6+5 j'écris 14/.1+1:6+7:30−.6+5 et il n'y a pas d'ambiguïté dans le fait que quand un groupe de points apparaît directement entre deux atomes (nombres ou variables), il représente une multiplication (et comme 6.7 représente 6×7, de même 2+2+2.3+4 représente (2+2+2)×(3+4) ; tandis que 2+2+(2×3)+4 s'écrira 2+2+:2.3:+4 ou même, un peu audacieusement, 2.+.2.+.2.3.+.4 si on décide que la multiplication est prioritaire sur l'addition). Ceci fonctionne encore même si on suppose que la multiplication omise n'est pas associative : on distingue bien u(vw) de (uv)w comme u.vw et uv.w respectivement.

Par rapport aux règles formelles que j'ai proposées ci-dessus, l'omission du symbole de multiplication se traite ainsi lors de l'écriture : (a) on écrit toujours au moins un point pour la multiplication quand elle est entre deux chiffres, et (b) au lieu de mettre un groupe de points à gauche et à droite du symbole ⋆ (qui doit être omis), on en met un seul, avec un nombre de points commun, supérieur à celui de tout groupe intervenant dans n'importe quelle sous-expression parmi les x1,x2,…,xk (avec cette règle, 2(x+y)(t⋆(u+v)) s'écrit 2:x+y:t⋆.u+v plutôt que 2.x+y:t⋆.u+v si on veut vraiment placer les trois facteurs 2, x+y et t⋆(u+v) au même niveau).

Il n'y a pas non plus de problème avec les opérations unaires, qu'elles soient écrites de façon préfixe ou postfixe. Il y a, cependant, un problème si on a une opération qui peut être aussi bien unaire que binaire et que le symbole de multiplication est omis : c'est le cas avec le signe moins si on veut pouvoir écrire (2/3)(−3) (qui vaudrait −2 par multiplication implicite) et le distinguer de (2/3)−3 (qui vaut −7/3), les deux étant a priori ponctués comme 2/3.−3 ; on peut résoudre ce problème de différentes façons, par exemple en imposant que pour les opérations binaires qui peuvent aussi être unaires, le nombre de points à gauche et à droite soit égal quand elles fonctionnent comme opérations binaires (donc (2/3)−3 se ponctuerait comme 2/3.−.3, qui se lit sans ambiguïté), et/ou que le signe de multiplication ne peut pas être omis devant une opération unaire (donc (2/3)(−3) devrait s'écrire 2/3.×.−3).

Il me semble par ailleurs qu'il n'y a pas de problème particulier avec une opération ternaire (par exemple si je décide que t?u!v signifie si t=0 alors v et sinon u — je change légèrement la notation du C parce que les deux points sont pris par le sujet de cette entrée — alors il n'y a pas de problème à écrire de façon ponctuée des expressions contenant cette expression imbriquée en elle-même de façon arbitraire). Ceci étant, je n'ai pas forcément pensé à toutes les bizarreries des notations mathématiques, peut-être qu'il y a des cas où le système de points ne fonctionnera pas alors que les parenthèses fonctionnent (outre ceux que j'ai déjà mentionnés).

Il faut que j'en profite pour signaler qu'il y a toutes sortes de petites variations possibles dans le système, j'en ai déjà implicitement signalé quelques unes. Je mentionne notamment la suivante, qui est plus économique dans le nombre de points utilisés, au détriment de la lisibilité de l'ensemble, et qui me semble plutôt une mauvaise idée. Plus haut j'ai signalé que 6−(5−(4−(3−(2−1)))) s'écrit 6−::5−:.4−:3−.2−1 (et c'est ce qui résulte des règles formelles que j'ai proposées), mais on peut aussi imaginer l'écrire simplement come 6−.5−.4−.3−.2−1 ce qui est après tout inambigu vu que chaque ‘.’ suivant immédiatement un symbole d'opération doit représenter une parenthèse ouvrante. (La modification des règles formelles que j'ai proposées doit être quelque chose comme ceci. En écriture, on place à gauche de chaque symbole ⋆ un groupe de points dont le nombre est immédiatement strictement supérieur au plus grand nombre de points de n'importe quel groupe qui apparaît, dans l'écriture de la sous-expression gauche, immédiatement à droite d'un symbole d'opération — ou comme symbole de multiplication omis — en ignorant donc les groupes de points qui apparaissent immédiatement à gauche d'un symbole d'opération ; et symétriquement pour la droite. Et en lecture, pour chaque niveau n de points, on doit grosso modo répéter tant que possible la recherche d'une expression x1x2⋆…⋆xk avec les xi des sous-arbres déjà constitués, la remplacer par un sous-arbre, et retirer les éventuels groupes de n points — mais pas plus — qui seraient adjacents à l'expression.)

Comme je l'ai dit plus haut, je crois que les points comme parenthèses n'ont été véritablement employés que dans des textes de logique (et uniquement entre les connecteurs logiques, pas dans les expressions arithmétiques comme sur les exemples que j'ai pris), même s'il n'y a pas de raison de la lier à ce contexte précis. Je ne sais pas exactement qui a inventé cette notation : peut-être Peano dans ses Arithmetices principia: nova methodo ; mais je sais surtout qu'elle est utilisée dans les Principia Mathematica de Russell et Whitehead dont elle contribue à la réputation d'illisibilité même si je crois que c'est loin d'être ce qui les rend le plus difficile (on pourra jeter un coup d'œil à la page des Principia que j'ai déjà évoquée sur ce blog, et utiliser cette page pour quelques indications sur comment décoder tout ça). J'ai d'ailleurs l'impression que les philosophes qui s'intéressent à la logique mathématique ont, plus que les logiciens vraiment matheux, tendance à utiliser des notations vieillotes (il y a peut-être une raison sociologique à creuser), et en particulier ces points-comme-parenthèses. Il y a aussi l'épouvantable symbole ‘⊃’ utilisé à la place de ‘⇒’ pour l'implication, que la grande majorité des matheux ont abandonné il y a belle lurette, et que des philosophes s'obstinent, Apollon sait pourquoi, à utiliser.

Mais l'autre question à se poser, bien sûr, c'est : ce système de notation avec des points à la place des parenthèses a-t-il des avantages ? Je sais qu'a priori il semble plus compliqué que les parenthèses. Peut-être l'est-il intrinsèquement, mais je crois que c'est essentiellement une question d'habitude (c'est difficile d'être sûr vu que je n'en ai moi-même guère la pratique). Je vois trois principaux arguments qu'on peut avancer pour défendre le système de points : (1) il est légèrement plus compact (quand on discute une opération non associative, il est plus léger d'écrire uv.w que (uv)w, par exemple), (2) on repère plus rapidement le niveau d'imbrication des choses (qui n'a jamais peiné, dans une expression parenthésée, à retrouver où chaque parenthèse se ferme ?), et (3) il est, finalement, relativement analogue à la ponctuation d'un texte en langage naturel (où, grossièrement parlant, on regroupe d'abord les mots non séparés par une ponctuation, puis les groupes séparés par des virgules, puis ceux séparés par des points-virgules, et enfin ceux séparés par des points), rendu plus logique. Le principal inconvénient que je lui vois, c'est que si on veut remplacer, dans une expression, une valeur par une autre expression, on va possiblement devoir incrémenter le nombre de points partout dans l'expression, alors que les parenthèses assurent que tout se passe forcément bien.

Bien entendu, je ne propose pas de changer une notation mathématique bien établie (les parenthèses sont quand même pratiques, finalement), mais il peut être intéressant de se rappeler qu'il y a, ou qu'il y avait a priori, d'autres notations possibles et pas forcément idiotes. Se le rappeler peut aider à mieux comprendre l'analyse syntaxique, à la fois des expressions mathématiques et des phrases ponctuées en langage naturel (cf. mon point (3) ci-dessus) ; et cela peut aussi suggérer comment faciliter la lecture d'une expression mathématique par des enrichissements typographiques (typiquement : mettre à chaque endroit possible un espacement proportionnel au nombre de points qu'on aurait dans la notation avec les points comme parenthèses).

↑Entry #2347 [older| permalink|newer] / ↑Entrée #2347 [précédente| permalien|suivante] ↑

↓Entry #2345 [older| permalink|newer] / ↓Entrée #2345 [précédente| permalien|suivante] ↓

(mardi)

Deux remarques sur l'intuition du théorème de Gödel

C'est un théorème bien connu, et que j'ai expliqué il y a quelques années dans cette longue entrée, que ZFC (:= le système d'axiomes standard de la théorie des ensembles), s'il est consistant, ne peut pas démontrer que ZFC est consistant. C'est là le « second » théorème d'incomplétude de Gödel dans le cas particulier de ZFC. De même, PA (:= l'arithmétique de Peano du premier ordre) ne peut pas démontrer que PA est consistant. (Dans les deux cas, l'affirmation que le système est consistant signifie qu'il n'existe pas de suite finie de symboles partant des axiomes et suivant les règles de la logique pour arriver à la conclusion absurde 0=1 : et on a le droit de parler de suites finies de symboles parce qu'elles peuvent se remplacer par des entiers grâce à ce qu'on appelle le codage de Gödel. Je ne rentre pas dans les détails puisque j'ai déjà expliqué ça et qu'il y a déjà quantité de bonne vulgarisation sur le sujet.)

Du coup, on peut être tenté d'ajouter à ZFC un nouvel axiome Consis(ZFC), qui affirme ZFC est consistant, formant un nouveau système ZFC₁ ; puis, comme le théorème de Gödel s'applique aussi à lui, on peut encore ajouter un nouvel axiome Consis(ZFC₁) qui affirme que celui-là est consistant, formant un nouveau système ZFC₂ ; « et ainsi de suite ». (En réalité, il y a beaucoup de subtilités ici dans le ainsi de suite, et de toute façon ce n'est pas une bonne façon d'enrichir ZFC, ces axiomes étant à la fois beaucoup moins forts, moins maniables et moins intéressants, que les axiomes de grands cardinaux par lesquels on l'étend usuellement. S'agissant de PA, on peut aussi faire cette construction, en gardant à l'esprit que PA, PA₁, PA₂, etc., et leurs consistance, sont de toute façon des conséquences (théorèmes) de ZFC.)

Ce point est bien connu, donc, et peut-être même trop connu, à tel point qu'on fait dire à ce théorème de Gödel un peu n'importe quoi. Les deux faits suivants, en revanche, sont bien moins connus, et mériteraient pourtant de l'être autant, parce qu'ils invitent à reconsidérer la manière dont on interprète (au moins sur le plan intuitif ou philosophique) ce théorème d'incomplétude. J'ai mentionné ces faits en passant lors de l'entrée passée vers laquelle je viens de faire un lien, mais je pense que je n'ai pas assez attiré l'attention dessus, ce qui est dommage.

(Les deux points suivants sont indépendants l'un de l'autre.)

✱ Le premier fait, c'est qu'on peut tout à fait fabriquer une théorie ZFC† dont les axiomes sont ceux de ZFC plus un axiome supplémentaire qui dit ZFC† est consistant. Oui, c'est circulaire (la théorie affirme sa propre consistance), mais ce n'est pas très difficile d'arriver à formaliser ça en utilisant les astuces de points fixes habituelles. Et de même, on peut former PA† dont les axiomes sont ceux de PA (Peano) plus un axiome supplémentaire qui dit que PA† est consistant. Il s'agit d'une façon assez naturelle d'essayer de contourner le théorème d'incomplétude (au moins quand on a mal compris celui-ci), en se disant puisque je ne peux pas démontrer que mon système formel est consistant, je vais l'ajouter comme axiome (et affirmer directement que l'ensemble est consistant plutôt qu'ajouter un axiome qui dit que la théorie de départ est consistante, puis un autre qui dit que cette nouvelle théorie est encore consistante, et encore un autre qui dit que celle-ci est consistante « et ainsi de suite »).

Bref, on peut fabriquer cette théorie ZFC† ou PA†, mais le problème c'est elle est inconsistante (elle démontre 0=1). Parce que le théorème de Gödel s'applique à elle aussi, et comme il affirme que si la théorie est consistante elle ne peut pas démontrer sa consistance, et qu'elle démontre effectivement sa consistance (puisque c'est un axiome, et qu'un axiome compte bien comme une démonstration), du coup, elle n'est pas consistante.

Alors voilà, ce n'est pas bien passionnant, certes : j'ai construit une théorie et j'ai expliqué qu'elle ne marchait pas — mais je pense que c'est quand même instructif, au moins sur le plan de l'intuition. Quand on présente le théorème d'incomplétude de Gödel, que ce soit au grand public, à des mathématiciens non-spécialistes, ou à des débutants en logique, l'idée qui en résulte typiquement — et je ne prétends pas qu'elle soit fausse — est qu'un système formel consistant T (récursivement axiomatisable, et contenant un fragment suffisant de l'arithmétique) n'est jamais assez « puissant » pour démontrer sa propre consistance, mais que (a) il s'agit d'une notion un peu constructive de démonstration, et (b) la raison pour laquelle on est conduit à ajouter des axiomes qui disent T est consistant et cette théorie-là est consistance et cette théorie- est consistante, « et ainsi de suite », est qu'on ne peut jamais tout faire d'un coup. Or l'exemple de la construction que je viens de donner montre qu'il faut se méfier de cette intuition : (b) on peut tout à fait écrire une théorie qui affirme sa propre consistance, et (a) cette théorie est forcément inconsistante parce que le théorème de Gödel interdit à une théorie consistante (récursivement axiomatisable, et contenant un fragment suffisant de l'arithmétique) non seulement démontre sa propre consistance, mais même simplement qu'il l'affirme (un axiome compte bien comme une démonstration). Je vais citer la présentation de Torkel Franzén (Inexhaustibility, 2004, chap. 12) parce que je trouve qu'il est particulièrement clair :

It is often emphasized that the resources of a theory T do not themselves suffice to enable a proof of the consistency of T. Again it is only by “going outside the system” than one can prove that T is consistent.

A weakness of this emphasis is that it doesn't take into account that the relevant concept of proof is a very liberal one. The consistency of T is provable in the theory T+Consis(T). This is not because any new fundamental principle has been introduced or because the theory T+Consis(T) incorporates any new insight that goes beyond those expressed in T, but simply because the consistency of T has been postulated. We don't require any more of a proof, as the term is used in logic. Accordingly, the second incompleteness theorem makes a stronger statement than one might naturally suppose. The consistency of T not only cannot be derived from the basic principles embodied in T, it cannot even be consistently asserted in T. A theory cannot consistently postulate its own consistency. By the diagonal lemma, we can produce a formula φ formalizing This sentence is consistent with T, but since T+φ then proves its own consistency, we know that in fact it is inconsistent.

Why is it impossible for T to consistently postulate Consis(T)? Because a paradox results from such a postulate, or so Gödel's proof of the second theorem suggests. If T asserts its own consistency, it must both assert and deny the provability of the sentence formalizing This sentence is not provable in T. It's not just a matter of T lacking the resources to establish a particular truth (that T is consistent) but of it being impossible to consistently sneak in this truth as an assertion or postulate in the theory itself. Saying that one must go outside the system to prove the consistency of T conveys the suggestion that T metaphorically speaking has a kind of “blind spot”, that it cannot reflect on or understand or inspect itself sufficiently to establish its own consistency—and indeed in extrapolations from the incompleteness theorem to other fields (religion, physics, psychology) this suggestion is frequently made explicit. The fact that T cannot even consistently assert its own consistency, without attempting any inspection or justification whatever, would seem to indicate that this suggestion is a bit of a red herring.

Je trouve que cela illustre très bien la manière dont on a tendance à mal se représenter le théorème d'incomplétude comme traduisant un problème profond de « manque de force » — alors qu'il s'agit de quelque chose d'à la fois plus trivial et plus profond. (Bien sûr, tout ceci est juste une question d'interprétation intuitive : il n'y a aucune difficulté ou subtilité mathématique dans tout ce que j'ai écrit.)

Mais si ce point est un peu trivial et en quelque sorte négatif, le suivant est beaucoup plus intéressant mathématiquement, et il est plutôt positif. Par ailleurs, il concerne spécifiquement ZFC et PA (pas que ce soient les seules théories auxquelles il s'applique, mais il ne s'applique pas à « à peu près n'importe quoi » comme le point que je viens de faire).

✱ J'en viens donc au second fait que je voulais signaler. Il faut d'abord que je rappelle que ZFC et PA ont un nombre infini d'axiomes : ils comportent en effet des schémas d'axiomes (le principe de récurrence dans le cas de PA, et pour ce qui est de ZFC, les schémas de séparation (=compréhension, =sélection) et ceux de remplacement). Ces axiomes veulent affirmer certains faits pour toute propriété P (des entiers naturels dans le cas de PA, ou des ensembles dans le cas de ZFC) : comme la logique du premier ordre ne permet pas de quantifier sur les propriétés, on s'en tire en postulant tous les énoncés dans lesquels P est remplacé par n'importe quelle formule explicitement écrite dans le langage où on se place — ce qui fait donc une infinité d'axiomes.

(Digression : Il y a d'autres façons de faire, consistant plus ou moins à faire de la logique du second ordre, et qui permettent de ramener cette infinité d'axiomes à un nombre fini au prix d'une complication de la logique, et parfois un renforcement du système : ce sont par exemple la théorie des ensembles de Gödel-Bernays, essentiellement aussi forte que ZFC, ou celle, strictement plus forte, de Morse-Kelley, les deux permettant de parler de classes, ce qui revient à permettre de quantifier sur les propriétés, et, s'agissant de l'arithmétique, le système ACA qui est exactement parallèle de Gödel-Bernays et l'arithmétique du second ordre Z₂=PA² qui est exactement parallèle de Morse-Kelley. Mais je vais m'abstenir de plus parler de toutes ces théories, d'autant que ça devient vite technique quand il s'agit de distinguer la vraie logique du second ordre de la logique du second ordre « réifiée » au premier ordre au sens où on a une logique du premier ordre à deux types d'objets qui fait semblant d'être une logique du second ordre en décrétant que l'un de ces types est le type des « classes » ou « propriétés » de l'autre type, ce qui revient finalement au même sauf que la notion de modèle et toute la sémantique qui va avec est différente.)

Un point qui me semble très important, et qui est rarement suffisamment souligné dans les cours élémentaires de logique, est le suivant :

Chacun de ZFC et de PA prouve la consistance de tous ses sous-ensembles finis d'axiomes.

Autrement dit, ZFC ne prouve pas la consistance de ZFC (c'est ce par quoi j'ai commencé : le second théorème d'incomplétude), mais ZFC prouve la consistance de n'importe quel ensemble fini d'axiomes de ZFC. Et la même chose vaut pour PA. On dit que ce sont des théories réflexives. En fait, il y a mieux : n'importe quelle extension de l'une ou l'autre de ces théories, écrite dans le même langage, est elle-même réflexive (on dit que ZFC et PA sont essentiellement réflexives : dans le cas de PA, c'est un théorème de 1952 dû à Andrzej Mostowski, et dans le cas de ZFC, je crois que le résultat est dû à Richard Montague et/ou Azriel Lévy vers 1960).

Une des conséquences de ce théorème est que ni ZFC ni PA, s'ils sont consistants, ne peut pas être axiomatisé par un nombre fini d'axiomes (si un ensemble fini T₀ de théorèmes de ZFC, ou du coup, d'axiomes de ZFC, suffisait à impliquer tous les axiomes de ZFC, alors ZFC prouverait la consistance de T₀, donc T₀ prouverait la consistance de T₀, et en prenant T₀ assez fort pour faire de l'arithmétique basique — je ne rentre pas dans les détails — ceci contredit le théorème de Gödel appliqué à la théorie T₀ ; et exactement le même raisonnement vaut pour PA). Mieux : comme ZFC et PA sont essentiellement réflexifs, aucune théorie consistante contenant ZFC ou PA et écrite dans le même langage ne peut être axiomatisée par un nombre fini d'axiomes. Mais ce n'est pas vraiment de ça que je veux parler.

Le résultat ci-dessus doit surprendre, parce qu'il paraît contredire le théorème de Gödel. L'argument serait le suivant : s'il y avait une contradiction dans ZFC, la démonstration de cette contradiction n'utiliserait qu'un nombre fini d'axiomes de ZFC (si on veut, c'est le théorème de compacité syntaxique, mais c'est une trivialité : une démonstration, étant de longueur finie, ne peut faire appel qu'à un nombre fini d'axiomes !) ; mais d'après ce que j'ai dit, ZFC prouve que ceci ne peut pas se produire (tout ensemble fini d'axiomes de ZFC est consistant) — du coup, ZFC est consistant, et on semble avoir prouvé ce fait dans ZFC ! Quelle est l'arnaque ?

L'arnaque est que le théorème de réflexivité ci-dessus est un métathéorème ; plus exactement, donné un ensemble T₀ quelconque d'axiomes de ZFC, on a une recette tout à fait explicite qui fabrique une démonstration à partir des axiomes de ZFC dont la conclusion est T₀ est consistant, et c'est un théorème (de ZFC, PA ou de systèmes encore plus faibles) que cette recette marche, i.e., l'énoncé encadré ci-dessus est bien un théorème. Mais, s'il est vrai que pour tout T₀ fini ⊆ZFC, T₀ est consistant est un théorème de ZFC, et que ceci est aussi un théorème de ZFC ou PA (i.e., pour tout T₀ fini ⊆ZFC, T₀ est consistant est un théorème de ZFC), en revanche, l'affirmation pour tout T₀ fini ⊆ZFC, T₀ est consistant, elle, n'est pas un théorème de ZFC (si ce dernier est consistant), car elle implique la consistance de ZFC d'après le raisonnement que j'ai fait au paragraphe ci-dessus.

Je répète : pour tout ensemble fini T₀ d'axiomes de ZFC, on sait fabriquer une démonstration dans ZFC que cet ensemble T₀ est consistant, et on sait montrer dans ZFC (ou PA ou moins) que ce procédé marche bien, mais on ne peut pas en conclure dans ZFC que tout ensemble fini T₀ d'axiomes de ZFC est consistant. On peut résumer cette situation ainsi : il est vrai que pour tout ensemble fini T₀ d'axiomes de ZFC, ZFC démontre la consistance de T₀, mais il ne le fait pas uniformément en T₀. C'est un cas du phénomène appelé la ω-incomplétude : pour tout n on démontre P(n) selon une recette générale et explicite, mais on ne peut pas démontrer ∀n.P(n) (ici, s'imaginer que n est un codage de T₀ et P(n) est l'affirmation que ce T₀ est consistant).

Absolument tout ceci vaut en remplaçant ZFC par PA partout (i.e., pour tout sous-système fini T₀ de PA, PA démontre que T₀ est consistant, mais ne le fait pas de façon uniforme). Ce fait est, d'ailleurs, étonnamment difficile à trouver écrit dans des bouquins de logique arithmétique.

Pour autant, pour tout usage philosophique ou épistémologique, je suis tenté de dire que ce qui précède (je veux dire, le résultat encadré ci-dessus) est exactement aussi bien qu'une démonstration de la consistance de ZFC dans ZFC, resp. de PA dans PA. Je ne sais pas au juste ce qu'on espérerait accomplir à avoir une démonstration de la consistance de ZFC dans ZFC ou de celle de PA dans PA (le projet de Hilbert était plutôt d'avoir une démonstration de la consistance d'un système fort dans un système faible, donc disons quelque chose comme celle de ZFC dans PA, or ça c'est vraiment hors de question). Mais je suppose que l'idée serait quelque chose comme je suis prêt à admettre comme mathématiquement vrais et certains les résultats — au moins arithmétiques — dont j'ai une démonstration dans ZFC, et je me sentirais plus rassuré si j'étais certain qu'il n'y a pas de démonstration de résultats absurdes dans ZFC, ce qui n'est pas si idiot que ça même si c'est circulaire (admettre que ZFC est vrai — ne serait-ce qu'arithmétiquement — est beaucoup plus fort qu'admettre qu'il est consistant, donc à partir du moment où on l'admet comme vrai, l'étape épistémologique à l'admettre comme consistant devrait être gratuite). Le principe de réflexion que j'ai encadré ci-dessus rend la réticence à admettre que ZFC est consistant encore plus bizarre dans ce contexte : si je suis prêt à admettre la consistance de tous ses sous-systèmes finis, je devrais bien admettre la consistance de la théorie tout entière ; plus exactement, si on me fournit un modèle simple permettant de construire, pour tout ensemble fini T₀ d'axiomes de ZFC, une preuve du fait que T₀ est consistant (et en outre, une méta-preuve du fait, d'ailleurs plus ou moins évident, que ce procédé fonctionne bien), il serait extrêmement bizarre de ne pas en admettre la conclusion, à savoir que tout ensemble fini T₀ d'axiomes de ZFC est consistant.

↑Entry #2345 [older| permalink|newer] / ↑Entrée #2345 [précédente| permalien|suivante] ↑

↓Entry #2341 [older| permalink|newer] / ↓Entrée #2341 [précédente| permalien|suivante] ↓

(vendredi)

Pourquoi les ordinaux me fascinent (une introspection psychologico-mathématique)

J'ai déjà récemment écrit une entrée sur mon obsession pour la symétrie, qui est certainement responsable d'une bonne partie de l'attrait que les mathématiques ont pour moi, et qui déborde sur ma fascination pour certaines formes de mysticisme assez visible notamment dans les œuvres littéraires que j'ai tenté d'écrire quand j'étais plus jeune. Mais il y un autre aspect des mathématiques qui me hante et sur lequel je n'arrive pas vraiment à placer un nom : disons, faute de mieux, la grandeur. Je ne sais pas non plus expliquer exactement en quoi cela consiste (pour être clair, je ne parle pas d'un concept mathématique, mais du ressenti commun que j'ai de certaines parties des mathématiques) ; j'ai tendance à penser que c'est le contrepoids de la symétrie, donc peut-être que hiérarchie serait un meilleur terme. Si je devais écrire sérieusement une œuvre dont j'ai déjà publié certains fragments aléatoires, et imaginer un monde allant avec, où les mathématiques donneraient des pouvoirs arcanes, il y aurait sans doute deux types de pouvoirs et d'utilisateurs de ceux-ci : les magiciens, qui utiliseraient la symétrie, et les clercs (pour reprendre la terminologie rôliste) qui utiliseraient la hiérarchie. Maintenant, c'est peut-être mon obsession pour la symétrie qui me fait proposer cette classification : néanmoins, il est certain que, si je dois faire le chemin des mathématiques au mysticisme, la symétrie m'évoque clairement une forme de magie pour les raisons que j'ai déjà expliquées dans l'entrée que je lui ai consacrée, tandis que ce dont je veux parler ici a une saveur, disons, plus religieuse, et cela transparaît notamment dans la Théorie de la Totalité Transfinie de Turing que je décrivais dans cette entrée.

Je ne sais pas quel est le phénomène mathématique sous-jacent à cette impression mentale de « grandeur » ou « hiérarchie », donc, mais je sais quel est le concept qui la réalise le plus parfaitement : il s'agit des ordinaux. J'ai déjà écrit ici une vulgarisation de ce concept (et j'ai même fait un visualisateur permettant de naviguer parmi les plus petits d'entre eux, quoiqu'il faille admettre qu'on n'y voit rien), donc mon but ici n'est pas de parler de mathématiques (même s'il est à prévoir que je n'y résisterai pas : j'invite alors le lecteur non intéressé par les questions techniques à ignorer ces passages, ou de lire en diagonale). [Ajout : pour des descriptions de quelques grands ordinaux et comment on peut les manipuler, je renvoie à ce billet ultérieur.] Ce que je veux présenter aujourd'hui est l'effet psychologique qu'ont sur moi les ordinaux — une sorte de psychanalyse de l'infini, si on veut, si ce n'est que je ne prétends pas vraiment être sérieux.

C'est une réalisation fascinante pour beaucoup d'enfants, lorsqu'ils apprennent à compter, qu'il n'y a pas de plus grand nombre : j'ai déjà écrit des choses à ce sujet, je ne vais pas revenir sur cette fascination du fait que dans le jeu qui peut dire le nombre le plus grand, quand quelqu'un dit N, quelqu'un d'autre peut dire N+1 (ou N×2, ou N², ce qui en langage d'enfants signifie transformer un zilliard en un zilliard de zilliards ; je note qu'une compréhension des opérations les plus importantes des fonctions arithmétiques « élémentaires » vient assez tôt). La grande-cousine de mon poussinet nous racontait récemment, en parlant de son petit-fils, que ce dernier était passionné par savoir qui compte le plus loin. Et j'ai le souvenir assez net d'avoir joué à ces jeux (qui peut dire le nombre le plus grand, et qui peut compter le plus loin) quand j'étais petit. Je pense que c'est un signe que les mathématiques peuvent exercer un véritable attrait sur les enfants — avant qu'on les rende chiantes pour eux en leur faisant faire des calculs pénibles et mécaniques à en dégoûter n'importe qui. Et je me souviens aussi d'avoir eu des discussions, quand je jouais à ces jeux, pour savoir si quelqu'un avait le droit de dire l'infini, et si l'infini plus un (ou l'infini plus l'infini ou l'infini d'infinis) était alors une réponse légitime, entre ceux qui pensaient que ça n'existait pas, ceux qui pensaient que c'était de toute façon pareil que l'infini, et ceux qui pensaient que c'était encore plus grand. (Et en un certain sens, tous ont raison : il y a différentes sortes d'infinis mathématiques selon l'usage qu'on veut en faire ; mais les ordinaux vont plutôt donner raison aux derniers, et pousser la logique.)

Mais il n'y a pas que cette fascination pour les nombres à laquelle je pense chez les petits enfants. Il y en a une autre, que je pourrais traduire comme l'idée que beaucoup de choses doivent être totalement ordonnées : l'autre jour, dans une brasserie où je déjeunais, j'entendis un enfant demander à son père qui était le plus fort entre Darth Vader (enfin, Dark Vador, vu qu'il parlait français) et je n'ai pas entendu le deuxième terme mais j'imagine volontiers que c'était un super-héros quelconque. Par coïncidence (coïncidence certes un peu limitée vue la sortie prochaine d'un film très attendu), j'ai entendu, le même jour dans la rue, un autre gamin poser exactement la même question entre Darth Vader et un camion lancé à toute allure contre lui (question bizarre, mais je crois bien que c'est ça). Je ne sais pas si cette croyance que la puissance des super-héros ou des choses est totalement ordonnée (par la relation gagne un combat contre ?) est enracinée dans le développement de notre cerveau ou si c'est culturel[#] (par exemple, à force de se faire entrendre dire que Foo est plus Zippyesque que Bar dans la publicité, dans la fiction, etc.), mais il est certain que, petits, nous avons un certain goût pour les relations d'ordre total et que ce goût ne disparaît pas totalement, en tout cas pas chez moi, quand nous découvrons qu'en fait le monde n'est pas si simple, et que deux choses ne sont pas toujours comparables.

[#] Spontanément, j'aurais plutôt tendance à imaginer que c'est culturel ; mais d'un autre côté, beaucoup du Mahābhārata, de ce que j'en ai retenu, est consacré à comparaison de personnages guerriers chaque fois plus puissants et moralement plus droits que tous ceux qui ont précédé, et à la vérification de ces comparaisons au cours de combats. Donc même si les auteurs épiques prennent ensuite plaisir à trahir les attentes qu'on peut avoir sur les résultats de ces comparaisons, et à introduire des ordres cycliques ou autrement paradoxaux, je soupçonne que la présupposition de l'ordre total se trouve bien dans la culture qui a engendré cette épopée.

Les ordinaux sont en quelque sorte la sublimation de ces jeux d'enfants : de deux ordinaux, il y a toujours un plus grand (plus fort, plus puissant, plus infini), d'ailleurs dans quasiment tous les cas qu'on rencontre, le plus grand est tellement monstrueusement plus grand que le plus petit, que le plus petit pourrait essentiellement être le nombre 1 ; et à chaque fois qu'on a un ensemble d'ordinaux, il y en a un qui est le plus petit de l'ensemble, et il y a un ordinal qui est plus grand que tous ceux qu'on s'est donné. Si on appelle ω la réponse que fait l'enfant qui dit l'infini en réponse aux milliards et milliards de milliards que les autres ont proposés (techniquement, donc, le plus petit ordinal supérieur à tous les ordinaux finis), alors il y aura des ordinaux ω+1 (l'infini plus un), ω·2 (l'infini plus l'infini) et ω² (l'infini d'infinis) et encore d'autres choses plus grandes.

Mais je ne vais pas expliquer mon obsession pour les ordinaux uniquement à partir de ces jeux d'enfants. Il y a aussi une élégance intellectuelle dans la manière dont les ordinaux sont construits qui ne peut pas ne pas susciter l'admiration : le même genre d'élégance qui fait qu'on comprend que compter à partir de zéro est la bonne façon de faire (et d'ailleurs les ordinaux commencent à zéro). J'ai tenté d'expliquer ça dans l'entrée de vulgarisation que je leur ai consacrée, mais je pourrais résumer la construction des ordinaux en :

À chaque fois qu'on a construit les ordinaux jusqu'à un certain point, on crée un nouvel ordinal qui vient juste après tous ceux-là.

Cette idée est tellement génialement simple qu'on a du mal à se rendre compte de sa puissance. Initialement, je ne sais pas du tout ce que c'est qu'un ordinal, donc je n'en ai aucun : selon le principe que je viens d'énoncer, je crée donc un premier ordinal qui vient après rien du tout (i.e., c'est le plus petit de tous les ordinaux), et que j'appelle 0. Je connais maintenant donc un seul ordinal, qui s'appelle 0, et selon le principe de construction que j'ai énoncé, j'en crée donc un autre qui vient juste après 0, et je l'appelle 1. À ce stade-là, je connais donc 0 et 1, et je crée donc un nouvel ordinal qui vient juste après eux, et je l'appelle 2. En procédant de la sorte, je crée des ordinaux correspondant aux entiers naturels (0, 1, 2, 3, 4, 5… 42, 43, 44… 1000… 10↑42…), qui n'ont déjà pas de fin. Mais contrairement à la fabrication des entiers naturels (en gros, je fabrique 0, puis si je fabrique n, alors je fabrique aussi n+1), le principe que j'ai énoncé ci-dessus continue de s'appliquer : maintenant que je connais les entiers naturels, je crée un nouvel ordinal qui vient juste après tous ceux-là et je l'appelle ω, ce qui donne à son tour naissance à ω+1 et ainsi de suite.

Ce principe de construction est merveilleux parce que c'est exactement le même qui s'applique à chaque fois, et qui ne cesse jamais de s'appliquer, et malgré cela il donne naissance à une richesse et une diversité extraordinaires, mais je vais y revenir. Et les choses se déroulent ex nihilo, à partir du rien (comme je viens de le dire, le principe marche dès le début : on n'a pas d'ordinaux pour commencer, donc on crée un ordinal 0, il n'y a pas de règle spéciale pour 0 comme il y a pour les entiers naturels). On comprend que Cantor, quand il a découvert le concept, ait été ébloui par ce sur quoi il venait de mettre le doigt. (Et on comprend aussi que cette idée ait suscité la réticence, pour ne pas dire l'hostilité, de la communauté mathématique de l'époque, selon un modèle assez bien résumé par ce webcomic ; David Hilbert a bien eu raison en parlant — au sujet de la théorie des ensembles, mais certainement des ordinaux et cardinaux en particulier — d'un paradis que Cantor a créé et dont il ne faut pas que les mathématiciens nous chassions nous-mêmes.) Mais la construction a été rendue encore plus éblouissante par von Neumann, qui a proposé (Zur Einführung der transfiniten Zahlen, Acta Litt. ac Scient. Univ. Hung. 1, 199–208, en 1923 — l'auteur était âgé de même pas vingt ans !) la réalisation suivante des ordinaux, maintenant complètement standard :

Un ordinal est l'ensemble des ordinaux plus petits que lui.

Ainsi 0 est l'ensemble vide (∅) puisqu'il n'y a pas d'ordinaux plus petits, tandis que 1 est l'ensemble {0} = {∅} ayant pour seul élément 0=∅ puisque ce dernier est le seul ordinal plus petit, et 2 est l'ensemble {0,1} = {∅,{∅}} ayant les éléments 0 et 1, et ainsi de suite ; et ω = {0,1,2,3,…} est l'ensemble des entiers naturels. Ce n'est pas terriblement important pour la nature des ordinaux qu'ils soient « réalisés » comme par la définition de von Neumann, mais cela ajoute encore énormément à l'élégance fascinante de la construction, et au sentiment qu'elle se fabrique toute seule à partir de rien.

Il est certain que cette idée ne pouvait que susciter lors de son introduction des réactions assez vives, centrées sur le caractère légitime ou non d'admettre l'infini (l'infini actuel, c'est-à-dire réalisé, et non seulement potentiel) comme objet mathématique légitime, et le fait que la construction soit littéralement ex nihilo n'améliorait certainement pas les choses. J'ai mentionné ci-dessus qu'elle a suscité une très vive opposition, y compris de la part de certains des esprits les plus brillants du monde mathématique, comme Poincaré ou Weyl (et pas seulement des mathématiciens : Wittgenstein était profondément hostile à la théorie des ensembles), tandis que d'autres, non moins brillants, comme Hilbert ou von Neumann, l'ont accueillie avec enthousiasme. Pour la défense des sceptiques et détracteurs de la théorie des ensembles en général, et des ordinaux en particulier, il faut dire que toutes sortes de paradoxes (celui de Burali-Forti qui invite à considérer l'ensemble de tous les ordinaux, je vais y revenir, et celui de Russell qui invite à considérer l'ensemble de tous les ensembles qui n'appartiennent pas à eux-mêmes) ont été découverts dans la première formulation, pré-axiomatique, de la théorie des ensembles, et ces paradoxes pouvaient s'interpréter comme une impossibilité absolue de traiter directement des quantités infinies sans arriver à des contradictions. De nos jours, tout le monde ou presque admet que la difficulté était simplement de codifier rigoureusement les règles par lesquelles on a le droit de manipuler des ensembles et des infinis, et qu'une fois ces règles bien fixées (comme elles l'ont été par Ernst Zermelo et Abraham Fraenkel), la contradiction disparaît sûrement. (« Sûrement », même si on sait depuis Gödel qu'on ne peut jamais être totalement certain que la contradiction n'existe pas, et à chaque fois qu'on ajoute un infini plus grand, on ne peut qu'en être moins certain.) Ce qui est certain, c'est que la théorie des ensembles, et notamment la notion d'ordinal, fait partie des mathématiques maintenant considérées comme standard, même si cela n'empêche pas les crackpots de se concentrer dessus et de chercher à contredire différents passages de ce qu'ils ne comprennent pas, et spécifiquement les résultats de Cantor (typiquement, l'argument diagonal, même si celui-ci ne parle pas vraiment d'ordinaux). Et s'il y a une contradiction dans la théorie des ensembles, elle ne doit pas être si évidente que ça, vu le plaisir avec lequel des raisonnements d'une sophistication incroyable sont menés dedans et ne sont jamais encore tombés dans une contradiction[#2].

[#2] Digression technique : Des gens savants peuvent ici me rétorquer : mais si, on est tombé dans une contradiction !, en postulant l'existence d'un cardinal Reinhardt. Je ne sais pas si ça mérite vraiment une réponse autre que oui, à force de chercher vraiment très fort à introduire des axiomes aussi forts que possible et aussi proches que possible d'une contradiction, on a fini par en trouver une (repassez quand vous aurez une contradiction dans ZFC), i.e., je ne sais pas si l'existence d'un cardinal Reinhardt a sérieusement été avancée comme « vraie ». Cela n'a pas empêché de très grands théoriciens des ensembles (je pense à Hugh Woodin) de se livrer à une analyse mathématique et philosophique approfondie de pourquoi cette contradiction et ce qu'elle signifie sur la taille de l'infini — voir par exemple l'article de Woodin intitulé The Realm of the Infinite — et voir d'ailleurs aussi les commentaires et explications de Peter Koellner sur ce texte. En tout état de cause, cette histoire va plutôt dans le sens que quand on trouve une contradiction dans la théorie des ensembles, elle n'est pas spécialement difficile à exhiber (la preuve de la contradiction de ZFC + « il existe un cardinal Reinhardt » a été trouvée rapidement, et elle n'est ni longue ni très subtile), donc j'ai tendance à croire que savoir jusqu'où on a pu aller trop loin donne plutôt confiance en la solidité de l'édifice. (Alternativement, on peut défendre la thèse que la contradiction ne vient pas du grand cardinal supposé mais de l'axiome du choix qui limiterait la taille possible de l'infini, comme l'axiome de constructibilité place la limite beaucoup plus bas — en-dessous d'un cardinal mesurable.)

Mais il est vrai que tous ces concepts fleurent bon la contradiction tant ils semblent jouer avec elle et tourner autour d'elle en l'évitant tout juste. C'est le cas de façon générale de la logique mathématique (le théorème de Gödel s'approche dangereusement du paradoxe de la phrase qui dit cette phrase est fausse, et nous invite à contempler attentivement la différence entre vérité et démontrabilité — j'ai écrit de la vulgarisation à ce sujet ici), mais quand on ajoute les infinis dans l'histoire et la difficulté à se les imaginer intuitivement, on comprend que certains ne se sentent pas du tout à l'aise. Pour en revenir à mon propos initial, je pense que c'est justement pour cela que, une fois admis que le sujet n'est pas contradictoire, on le trouve d'autant plus stimulant. Arriver à suivre certains raisonnements en logique ou théorie des ensembles, bien plus que dans d'autres branches des mathématiques, peut être comparable à un roman policier où tout le monde réussit à tromper le détective tout en disant la plus stricte vérité.

Une conséquence de la construction des ordinaux (que ce soit celle de Cantor ou celle, plus précise, de von Neumann), est qu'on ne peut jamais en contempler la totalité : parler de la totalité des ordinaux contredit immédiatement le principe même de construction des ordinaux, puisque cette totalité devrait s'exposer immédiatement à définir un nouvel ordinal plus grand qu'eux (dans la construction de von Neumann : l'ensemble de tous les ordinaux), ce qui définit un nouvel ordinal, contredisant la totalité de la totalité supposée ! C'est là essentiellement le paradoxe de Burali-Forti. La résolution moderne technique est que les ordinaux ne forment en effet pas un ensemble, ils ne sont pas regroupables en ensemble — si on veut donner un nom à tous les ordinaux, ce sera une classe — et la construction des ordinaux ne s'applique qu'aux ensembles. Mais le contenu intuitif de cette explication technique est le suivant : on ne peut en effet jamais contempler la totalité des ordinaux, il faut donc décider d'arrêter de les construire à un certain moment (mais il vaut mieux choisir un moment raisonnablement « robuste », et c'est essentiellement cela que permettent les axiomes de la théorie des ensembles), après quoi l'ordinal qui vient immédiatement après s'appelle la classe des ordinaux et on fait semblant que ce n'est pas un ordinal, pas plus que ceux qui viendraient après ; des propriétés plus ou moins compliquées, ajoutées sous forme d'axiomes, permettent de décider jusqu'où on impose d'aller dans la construction des ordinaux. Bref, on a effectivement affaire à quelque chose qui n'est jamais achevé, qui ne peut jamais l'être par sa définition même, il faut juste décider à quel moment on a quelque chose de « suffisamment achevé », i.e., robuste, pour ce qu'on veut en faire. Là aussi, il s'agit d'une perspective extrêmement dérangeante, et donc d'autant plus fascinante quand on arrive à l'accepter mentalement : quand on dit les ordinaux, il y a toujours quelque chose d'inachevé dans l'histoire. Et en fait, quand on examine de plus près la logique, on se rend compte, et je vais essayer d'en dire un mot plus bas même si c'est un peu complexe, que cet « inachèvement » ne concerne pas que la notion de tous les ordinaux mais même des ordinaux bien précis, disons ω₁ ou même ωCK (voire ω ?), ce qui rend tout le panorama encore plus mystérieux.

Dans ces conditions, il ne me surprend pas que Cantor soit devenu mystique. Une rumeur persistante veut qu'il ait été fou, ce qui va certainement dans le sens de démolir ad hominem ses théories : la vérité est surtout qu'il a souffert de dépression, notamment à cause du rejet de ses idées. Mais il est vrai qu'il a été habité d'idées tout à fait mystiques, comme l'affirmation que c'était Dieu qui lui avait inspiré l'idée des nombres transfinis, et que leur existence (actuelle et non simplement potentielle) apportait une lumière sur l'existence ou sur la pensée de Dieu. Et il a écrit plusieurs lettres à des prêtres catholiques, dont le pape Léon XIII, au sujet de théologie et de rapports entre théologie et mathématiques. Même Hilbert utilise le terme de paradis (Paradies) pour décrire le monde de la théorie des ensembles ouvert par Cantor, avec ses infinis et ses raisonnements non-constructifs, et ce n'est certainement pas un hasard. Moi-même je ne peux que plaider coupable en ce qui concerne le mysticisme (même si, je l'ai déjà expliqué, pour moi le mysticisme est avant tout intéressant artistiquement, par exemple comme prémisse pour une œuvre littéraire) : ma fascination pour les ordinaux a indiscutablement une origine à peu près mystique, et quand je propose une interprétation eschatologique du paradis cantorien (encore une fois, la Théorie de la Totalité Transfinie de Turing), c'est du mysticisme à 0.02¤ (voire à (1/ω).

Mais c'est là que je veux, pour expliquer le mécanisme psychologique qui joue, rejoindre le mot que j'ai utilisé plus haut : grandeur. J'ai déjà dit que je fais souvent des rêves de vastes labyrinthes à explorer et j'ai déjà comparé les mathématiques à un palais magnifique et extraordinairement beau en même temps que labyrinthique : si la symétrie est ce qui fait la beauté du palais, la grandeur joue beaucoup pour qu'on ait envie de l'explorer. Les ordinaux sont le terrain de jeu ultime pour ce qui est de la grandeur, et comme un gosse qui découvre un nouveau terrain d'aventure, j'ai envie de m'y lancer.

↑Entry #2341 [older| permalink|newer] / ↑Entrée #2341 [précédente| permalien|suivante] ↑

↓Entry #2337 [older| permalink|newer] / ↓Entrée #2337 [précédente| permalien|suivante] ↓

(lundi)

Qu'est-ce qu'une machine hyperarithmétique ?

Voici un concept mathématique (voire, informatique ?) dont je suis tout étonné de découvrir que je ne l'ai jamais encore proprement défini sur ce blog, alors même que ça aurait été logique et pertinent de le faire dans différentes entrées que j'ai déjà écrites. (Par exemple, j'y fais explicitement référence dans cette entrée, et il aurait été logique d'en parler dans celle-ci ; et au sujet de cette entrée récente, je pourrais dire qu'il s'agit exactement de la puissance de calcul du niveau ωCK de la « Théorie de la Totalité Transfinie de Turing ».) Je voudrais donc réparer ce manque, d'autant plus que je trouve que le sujet devrait être standard, et connu, notamment, de tous les informaticiens théoriciens vaguement préoccupés de calculabilité ou de complexité (or je suis sûr que ce n'est pas le cas[#]) : une machine hyperarithmétique est un type d'ordinateur théorique strictement plus puissant que les machines de Turing, et il me semble qu'avoir en tête à la fois la notion de fonctions hyperarithmétiques (plus générales que les fonctions calculables au sens de Church-Turing, donc) et la notion de fonctions primitives récursives (plus restreintes) aide à mieux comprendre les contours de la calculabilité (y compris si on ne s'intéresse, in fine, qu'aux machines de Turing). Il me semble par ailleurs qu'il s'agit d'une notion relativement intuitive (je vais donc essayer de la présenter comme telle), qu'il est donc dommage de laisser cachée dans des textes de calculabilité supérieure un peu oubliés et au formalisme souvent obscur.

Je commence par rappeler[#2] ce que c'est que la calculabilité au sens habituel, i.e., de Church-Turing : les lecteurs pour lesquels ce concept est familier peuvent sauter jusqu'au symbole ♠ plus bas.

En bref, [une fonction] calculable (sous-entendu : au sens de Church-Turing) signifie [une fonction] qui pourrait être calculé(e), en principe, par un algorithme tournant sur un ordinateur — sachant que cet ordinateur n'a aucune limite sur la quantité de mémoire qu'il peut utiliser, ni sur le temps qu'il peut prendre, à part que le temps doit être fini (et la mémoire, du coup, automatiquement aussi).

Pour donner une définition plus précise, il y a plein de possibilités : la première qui ait été introduite historiquement, vers 1930, est le lambda-calcul de Church, mais même si elle est utile pour modéliser les langages de programmation fonctionnels, elle n'est pas très parlante intuitivement ; la seconde définition est venue par les fonctions générales récursives (je n'ai pas réussi à comprendre exactement quelle en était l'histoire, mais elles doivent être associées à un ensemble intersectant les noms suivants : Herbrand, Gödel, et Kleene) ; mais la définition de la calculabilité qui a vraiment achevé de convaincre le monde des mathématiciens qu'il s'agissait de la bonne notion est venue en 1936 quand Turing a défini la machine qui porte maintenant son nom. Quantité d'autres définitions ont été données depuis (par exemple avec des machines à registres). J'en donnerai moi-même une (illisible) ci-dessous comme produit dérivé d'une définition rigoureuse du sujet principal de cette entrée (pour les fonctions calculables, retirer la clause (vii) qui me sert à définir les fonctions hyperarithmétiques). Le point important est que toutes ces définitions sont équivalentes au sens où elles conduisent à la même classe de fonctions « calculables » : la fameuse thèse de Church-Turing affirme que n'importe quelle tentative pour définir la notion de « fonction calculable par un algorithme » aboutira, in fine, à cette même classe des fonctions calculables (au sens de Church-Turing, donc), étant bien entendu que l'« algorithme » doit manipuler à tout instant des données finies, et terminer en temps fini (et, par ailleurs, ne peut pas faire appel au hasard, ou en tout cas le résultat final ne doit pas en dépendre).

↑Entry #2337 [older| permalink|newer] / ↑Entrée #2337 [précédente| permalien|suivante] ↑

↓Entry #2335 [older| permalink|newer] / ↓Entrée #2335 [précédente| permalien|suivante] ↓

(lundi)

Comment écrire les nombres en base 5×6

Nous écrivons les nombres en base 10 (c'est-à-dire que pour compter des billes, nous faisons des tas de 10, puis des tas de 10 de ces tas, puis des tas de 10 de ceux-là, etc., et nous indiquons par un chiffre le nombre de chaque type de tas) : heureusement, de la Chine à la Patagonie, tout le monde est d'accord là-dessus, y compris les pays reculés qui continuent à diviser leurs unités de longueur en 1760 et leurs unités de poids en 16. On voit parfois avancée çà ou là l'idée qu'on ferait mieux de compter en une autre base (typiquement 12). Le choix de 10 n'est peut-être pas idéal, mais l'intérêt d'avoir un standard commun à tout le monde est infiniment supérieur à l'avantage d'avoir telle ou telle autre base peut-être préférable dans l'absolu : même si nous utilisions une base franchement merdique, comme 11, il vaudrait mieux rester sur un standard merdique mais commun que de chercher à créer de la confusion en en changeant (c'est d'ailleurs pour le même genre de raison que je ne pense pas qu'il soit une bonne idée d'essayer de changer d'autres choses qui ont été adoptées universellement, comme le calendrier grégorien et ses bizarreries bêtement baroques). Tout ça pour dire que je ne propose certainement pas une seule seconde de changer de système d'écriture des nombres (même si j'avais le pouvoir de motiver des gens à initier un tel changement, je ne voudrais en aucun cas m'en servir). J'espère que j'ai bien enfoncé la porte ouverte, et que je peux maintenant aborder la question purement théorique de ce que pourrait être une bonne base si on devait repartir de zéro.

L'intérêt d'avoir une base b divisible par des petits nombres (premiers) est principalement que les fractions simples vont pouvoir s'écrire en base b de façon simple : le fait que 10=2×5 fait que les rationnels 1/2 et 1/5 s'écrivent respectivement 0.5 et 0.2 en cette base, tandis que le fait que 3 ne divise aucune puissance de 10 est responsable du fait que 1/3 s'écrit 0.333333…, ce qui est un peu agaçant dès qu'on veut manipuler des tiers (notamment à cause des arrondis : si on arrondi 1/3 à 0.333, alors dès qu'on en met trois, on tombe sur 0.999 et il y a un millième qui est tombé à l'eau). L'argument en faveur de la base b=12 est que comme il est divisible par 2, 3 et 4, il simplifie l'écriture des fractions de petit dénominateur (1/2 s'y écrit 0.6, 1/3 s'y écrit 0.4, et 1/4 s'y écrit 0.3), mais évidemment, on perd le 1/5, qui devient 0.24972497…, ce qui n'est pas franchement plaisant. • L'intérêt d'avoir une base b petite est, quant à lui, que les tables d'addition et de multiplication sont d'autant plus courtes à apprendre : la base 2 est bien sûr particulièrement simple de ce point de vue-là, et il est naturel qu'on s'en serve dans circuits électroniques (je veux dire : outre le fait qu'il est naturel de représenter 0 et 1 par l'absence et la présence d'un signal, l'addition et la multiplication se calculent de façon particulièrement simple), même si elle est peu appropriée au calcul humain à cause de la longueur de la représentation des nombres.

D'un autre côté, les choix sont apparemment limités : si la base est trop petite, les nombres sont trop longs à écrire, si elle est trop grande, les tables d'opération sont trop complexes à mémoriser, et si on cherche à avoir autant de divisibilités que possible, il semble que 6 ou 12 soient peut-être les choix les plus sensés, et en tout cas 10 n'est pas du tout mauvais.

(À ce propos, j'espère enfoncer de nouveau des portes grandes ouvertes, mais quand j'écris par exemple la base 12, il va de soi que ce 12 est lui-même écrit de la manière dont nous écrivons habituellement les nombres, c'est-à-dire dans la base dont la valeur est [le nombre de ‘I’ dans ce qui suit] IIIIIIIIII. C'est complètement idiot, mais si on n'éclaircit pas ce point, certains sont capables de s'imaginer que le nombre 10 est magique.)

La discussion ci-dessus, cependant, néglige le fait qu'il y a toutes sortes de variations possibles sur l'écriture en base b, qui peuvent être utiles dans différents sens, ou qui pourraient arriver pour des raisons essentiellement historiques. Les mayas, et les aztèques à leur suite, par exemple, pour autant que je comprenne, écrivaient les nombres en base 20, sauf que le chiffre des vingtaines était exceptionnel et n'allait que jusqu'à 18 : i.e., ils faisaient des paquets de 20 unités, puis des paquets de 18 paquets, puis des paquets de 20 de ces paquets, et de même de 20 à tous les niveaux suivants ; ceci fournissait une correspondance avec leur calendrier de 18 mois de 20 jours. Par ailleurs, même l'écriture des chiffres de 0 à 19 était plus ou moins faite en base 5 (ils utilisaient un bâton pour le nombre 5, un point pour le nombre 1, et donc par exemple trois bâtons et deux points pour le chiffre 17 — je dis bien chiffre, parce que 17 était un chiffre de leur écriture en base à-peu-près-20 ; le zéro était noté spécialement, pour ne pas laisser un vide disgracieux dans l'écriture).

Pour donner un exemple d'écriture qui n'est pas tout à fait une base b entière mais qui s'y rapproche beaucoup, on peut écrire les entiers en « base Fibonacci » : cette représentation n'utilise que les chiffres 0 et 1 et interdit à deux ‘1’ d'être consécutifs, la valeur des positions étant donnée par les termes de la suite de Fibonacci ((1,)1,2,3,5,8,13,21… chacun étant la somme des deux précédents). Ainsi, comme 17=13+3+1, le nombre 17 s'écrira 100101 : et les premiers entiers s'écrivent 0, 1, 10, 100, 101, 1000, 1001, 1010, 10000, 10001, 10010, 10100, 10101, 100000, etc. Ce mécanisme d'écriture (dont il existe d'ailleurs un certain nombre de variations) peut avoir un intérêt dans certaines circonstances, et il est possible d'y mener des calculs, mais évidemment, il est encore plus encombrant que la base 2 (et l'écriture fractionnaire n'est pas du tout claire). Je l'évoque surtout pour montrer qu'il n'y a pas que les écritures en base b qui peuvent avoir un sens ou un intérêt. (D'ailleurs, mon voisin de bureau est spécialiste de ce genre de questions.)

⁂ Bon, alors, si je devais absolument choisir un système d'écriture des nombres de novo, qui soit relativement aisément manipulable à la main si on oublie l'héritage de la base 10, je crois que je choisirais la base 30 écrite sous la forme 5×6, c'est-à-dire une base alternée 5 et 6.

Autrement dit, l'idée est de faire des paquets de 6, puis de faire des paquets de 5 de ces paquets, puis des paquets de 6 de ces paquets-là, puis des paquets de 5 de ceux-là, et ainsi de suite en alternant 6 et 5 : comme les paquets de paquets sont toujours de 30, on peut dire qu'on travaille en base 30, mais on le fait en n'utilisant que des paquets de 6 ou 5, ce qui garde des chiffres petits et manipulables, et des tables d'opérations facilement mémorisables.

Concrètement, on utiliserait deux séries de chiffres, disons 0,1,2,3,4,5 pour les chiffres en base 6, et Z,A,B,C,D pour ceux en base 5 ; ces deux séries alterneraient systématiquement (en terminant par la série 0…5 pour le chiffre des unités). Le fait d'avoir deux séries de chiffres qui alternent peut d'ailleurs avoir un intérêt en lui-même : il évite certaines erreurs de décalage d'une colonne (à la fois à la lecture, et lorsqu'on effectue les opérations). • Les premiers entiers s'écrivent donc 0, 1, 2, 3, 4, 5, A0, A1, A2, A3, A4, A5, B0, B1, B2, B3, B4, B5, C0, C1, C2, C3, C4, C5, D0, D1, D2, D3, D4, D5, 1Z0, 1Z1, 1Z2, 1Z3, 1Z4, 1Z5, 1A0, etc. Le nombre décimal 1760 s'écrirait, par exemple, 1D4C2 dans ce système, parce qu'il vaut 1×30² + 4×6×30 + 4×30 + 3×6 + 2 (le 2 est le chiffre des unités, le C est le chiffre des sixaines, le 4 est le chiffre des groupes de 5×6=30, le D est le chiffre des groupes de 6×5×6 = 6×30 = 180, et le 1 est le chiffre des groupes de 5×6×5×6 = 30² = 900) : cette conversion est, bien sûr, fastidieuse, mais ça ne dit rien sur cette base spécialement parce que la conversion d'une base à une autre est toujours fastidieuse (enfin, sauf entre puissances d'un même nombre).

↑Entry #2335 [older| permalink|newer] / ↑Entrée #2335 [précédente| permalien|suivante] ↑

↓Entry #2323 [older| permalink|newer] / ↓Entrée #2323 [précédente| permalien|suivante] ↓

(jeudi)

Analysons le mécanisme de vote du Conseil de l'UE

Le Conseil de l'Union européenne, dont le nom officiel est juste le Conseil, et qu'on appelle parfois aussi informellement Conseil des ministres parce qu'il réunit les ministres des 28 états membres sur un sujet donné, est en quelque sorte la chambre haute de la législature de l'Union européenne (dont le Parlement européen serait la chambre basse), représentant les intérêts des États membres tandis que le Parlement européen représente la population de l'Union : il est donc vaguement analogue au Sénat des États-Unis ou au Bundesrat allemand (représentant, dans les deux cas, les entités fédérées). Si je simplifie en passant sous silence un nombre incroyable de cas particuliers, subtilités, astérisques et autres exceptions, une directive européenne (l'équivalent d'une loi) doit, pour être adoptée (selon la procédure législative ordinaire) être proposée par la Commission, et adoptée dans les mêmes termes par le Parlement et le Conseil. Je me propose d'analyser un peu la manière dont ce Conseil vote.

Les gens qui n'aiment pas lire des logorrhées (mais que faites-vous sur mon blog, aussi ?) peuvent sauter plus bas où il y a des jolis graphiques.

La petite minute nécessaire du Club Contexte : il y a aussi un Conseil européen, terminologie épouvantablement idiote parce qu'il n'est pas plus européen que l'autre, qui ressemble beaucoup au Conseil [des ministres] en ce qu'il est formé des représentants des 28 États membres, mais qui diffère en ce qu'il est formé des chefs d'État ou de gouvernement au lieu des ministres, et dont les fonctions ne sont pas tout à fait claires au niveau institutionnel (il « dirige », donne des « impulsions », etc.). Du coup, le Conseil européen a très rarement l'occasion de procéder à des votes, à part pour des cas très précis comme quand il s'agit de nommer le président de la Commission et qu'il n'y a pas de consensus. Les deux conseils (Conseil européen et Conseil [des ministres]) se ressemblent par certains points : dans les rares cas où le Conseil européen effectue un vote, c'est le même mécanisme de vote que pour le Conseil, et les deux Conseils ont, par exemple, le même logo représentant le futur bâtiment qu'ils auront aussi en commun (parfois l'un des deux ajoute au logo le mot latin Consilium, mais je n'ai pas compris lequel, ça a l'air de changer, et c'est peut-être obsolète), et ils ont le même site Web. Il y a aussi des différences : notamment, contrairement au Conseil [des ministres], qui est présidé par un État tournant tous les six mois [subtilité : sauf quand il est en formation affaires étrangères], le Conseil européen est présidé par une personne stable, en l'occurrence l'ancien Premier ministre polonais Donald Tusk. Je pense que l'idée est que si on considère l'UE comme un État fédéral ou confédéral, le Conseil européen en est une sorte de chef d'État collégial : il nomme le chef du gouvernement, c'est-à-dire de la Commission, et il a la main sur les grandes lignes de la politique étrangère. (Il n'est pas rare dans les dispositions constitutionnelles qu'il y ait une certaine porosité ou proximité entre le chef de l'État et la chambre haute du parlement : par exemple, le vice-président des États-Unis est ex officio président du Sénat, tandis que le président du Sénat français devient président par intérim si le président décède, et on peut certainement citer d'autres exemples ; la confusion entre les deux Conseils se comprend donc un peu dans cette logique.) • Par ailleurs, il ne faut pas confondre l'un ou l'autre de ces Conseils, qui sont des institutions de l'Union européenne, avec le Conseil de l'Europe, qui est une autre institution internationale, strictement plus grande que l'Union européenne (et dont, par exemple, la Norvège, la Suisse et la Russie sont membres). Pour tout arranger au niveau confusion, le Conseil de l'Europe a le même drapeau que l'Union européenne (c'est même lui qui l'a utilisé en premier), et aussi le même hymne.

Généralités : La plupart des décisions du Conseil [de l'UE, i.e., Conseil des ministres] se prennent, dans la pratique, sur la base du consensus : un vote a lieu formellement, mais il est précédé de beaucoup de négociations, voire de marchandages, menées informellement (par courrier électronique, par l'intermédiaire des représentants permanents à Bruxelles, ou au cours de réunion officieuses du Conseil), surtout par la présidence tournante du Conseil : lorsque la présidence annonce qu'elle dispose d'une majorité suffisante pour approuver la proposition, les éventuels pays minoritaires préfèrent négocier leur ralliement au vote en échange de quelques concessions plutôt que d'enregistrer une « contestation publique », i.e., de figurer sur le papier final comme votant contre (ce qui peut être embarrassant, diplomatiquement ou politiquement, sauf s'il s'agit d'enregistrer un point vis-à-vis de leur opinion publique nationale). Ce n'est pas pour autant que les détails du mécanisme de vote n'ont pas d'importance ! Car ce sont tout de même eux qui définissent le pouvoir des différents pays dans les négociations informelles, et même si le vote formel apparaît comme unanime — même si on cherche le compromis pour arriver à l'unanimité — l'avis d'un petit pays sera évidemment d'autant plus écouté s'il a le moyen de tout bloquer que si on sait qu'on peut toujours se passer de son accord. (Une analyse précise de la dynamique de vote pour ce qui est de la contestation publique, sur la période 1995–2010, est menée dans ce rapport de Wim van Aken, Voting in the Council of the European Union.)

Le mécanisme de vote dans toute sa subtilité juridique est assez complexe. D'abord, il y a plusieurs mécanismes différents selon le type de motion soumise au vote, et qui exigent des majorités différentes : majorité simple (principalement pour des questions de procédure ou des résolutions sans valeur légale), majorité qualifiée (la procédure ordinaire), ou unanimité (essentiellement pour tout ce qui est conçu comme une coopération intergouvernementale : par exemple, en matière fiscale). Même au sein de la majorité qualifiée, une des conditions demandées est différente selon que le Conseil vote sur une proposition de la Commission ou non (il y a donc, en quelque sorte, deux majorités qualifiées différentes : la normale, pour voter sur une proposition de la Commission, et la renforcée, pour les cas où le Conseil agit de sa propre initiative, essentiellement en matière de politique étrangère). • Pour compliquer encore les choses, pendant une période transitoire qui dure de novembre 2014 à mars 2017, les règles de vote actuelles, entérinées dans le traité de Lisbonne de 2007 (qu'on appellera donc en abrégé règles de Lisbonne, en gros : 55% des états membres représentant 65% de la population), peuvent parfois — à la demande d'un membre du Conseil — être remplacées par les règles antérieures, contenues dans le traité de Nice de 2001 (règles de Nice, en gros : >50% des états membres, et 73.8% des voix pondérées). • Pour compliquer encore un peu plus les choses, une déclaration annexée aux traités (parfois appelée « compromis de Ioannina », ) veut que si un groupe d'états n'est pas suffisant pour constituer une minorité de blocage (c'est-à-dire, une minorité capable d'empêcher un vote de passer, donc, avec les règles de Lisbonne, 45% des états membres ou représentant 35% de la population de l'Union) mais n'est « pas trop loin » d'en constituer une, alors la présidence du Conseil et l'ensemble de ses membres s'engagent à faire des efforts pour trouver une solution tenant compte de leurs objections. • Pour compliquer la complication, la définition de pas trop loin dans la phrase précédente sera abaissée en avril 2017 (pour compenser le fait qu'on ne pourra plus invoquer les règles de Nice ; jusqu'à mars 2017, il suffit de représenter 3/4 du nombre de membres ou de la population nécessaires à constituer une minorité de blocage, tandis qu'à partir d'avril 2017, elle est abaissée à 55% sur ces deux critères). Ouf ! On comprend que les choses ne soient pas aisées à décrire.

Mon but est ici, en oubliant un peu les subtilités de la négociation et de la culture du compromis, de faire quelques points plutôt d'ordre mathématique, mais à un niveau assez simple, sur le mécanisme de vote du Conseil à la majorité qualifiée (« normale »), à la fois dans les règles de Lisbonne et dans les règles de Nice. Et d'en profiter pour faire quelques remarques plus générales sur l'analyse du pouvoir dans un système de vote de ce genre.

[J'avais déjà écrit un billet sur le sujet ici, au moment où le mécanisme de vote était en train d'être débattu (et en écrivant par erreur Conseil européen au lieu de Conseil [de l'Union européenne ou des ministres]). J'y proposais un mécanisme de vote particulier. Ici, je vais plutôt me pencher sur la question de comment analyser un mécanisme de vote existant.]

↑Entry #2323 [older| permalink|newer] / ↑Entrée #2323 [précédente| permalien|suivante] ↑

↓Entry #2321 [older| permalink|newer] / ↓Entrée #2321 [précédente| permalien|suivante] ↓

(mercredi)

Jouons à analyser la forme des continents

[Sommes partielles d'harmoniques sphériques pour la forme des continents] [Niveau 0] [Niveau 1] [Niveau 2] [Niveau 3] [Niveau 4] [Niveau 5] [Niveau 6] [Niveau 7] [Niveau 8] [Niveau 9] [Niveau 10] [Niveau 11] [Niveau 12] [Niveau 13] [Niveau 14] [Niveau 15] [Niveau 16] [Niveau 17] [Niveau 18] [Niveau 19] [Niveau 20] [Niveau 21] [Niveau 22] [Niveau 23] [Niveau 24] [Niveau 25] [Niveau 26] [Niveau 27] [Niveau 28] [Niveau 29] [Niveau 30] [Niveau 31] [Niveau 32] [Niveau 33] [Niveau 34] [Niveau 35] [Niveau 36] [Harmoniques sphériques pour la forme des continents] [Niveau 0] [Niveau 1] [Niveau 2] [Niveau 3] [Niveau 4] [Niveau 5] [Niveau 6] [Niveau 7] [Niveau 8] [Niveau 9] [Niveau 10] [Niveau 11] [Niveau 12] [Niveau 13] [Niveau 14] [Niveau 15] [Niveau 16] [Niveau 17] [Niveau 18] [Niveau 19] [Niveau 20] [Niveau 21] [Niveau 22] [Niveau 23] [Niveau 24] [Niveau 25] [Niveau 26] [Niveau 27] [Niveau 28] [Niveau 29] [Niveau 30] [Niveau 31] [Niveau 32] [Niveau 33] [Niveau 34] [Niveau 35] [Niveau 36]

Je cherchais à me faire une idée intuitive un peu plus claire de la notion mathématique de décomposition en harmoniques sphériques (voir ici pour une explication très sommaire) : or la meilleure façon de comprendre une notion mathématique est probablement de s'amuser avec — je me suis dit que pour avoir une fonction raisonnablement « parlante » sur la sphère avec laquelle faire joujou, un candidat assez naturel est la forme des continents. J'ai donc analysé cette fonction en harmoniques sphériques ; plus exactement, j'ai pris la fonction qui vaut −1 sur la terre et +1 sur la mer, histoire d'être mieux centré vers 0, mais c'est peu important (ça va juste introduire des facteurs ½ pénibles un peu partout dans la suite), et en faisant semblant que la Terre est une sphère. Ce calcul n'a, bien sûr, rien d'original, même si le genre de fonction qu'on analyse pour des applications plus sérieuses seraient plutôt l'altitude, le champ de gravité ou quelque chose de ce goût. Je tire mes données géographiques de cette page (Earth Specular Map 8K). J'ai utilisé la bibliothèque SHTns pour faire les calculs (après une tentative pitoyable pour les faire moi-même, cf. ci-dessous).

L'image à gauche de ce texte montre les sommes partielles de cette décomposition en harmoniques sphériques : en haut, le niveau =0, en-dessous la somme des niveaux =0 et =1, puis la somme des niveaux ≤2, et ainsi de suite (à chaque fois, toutes les valeurs de m, c'est-à-dire −m, sont mises pour chaque , donc si on veut, la première ligne montre 1 terme, le suivant la somme de 4 termes, puis la somme de 9 et ainsi de suite). La Terre est vue en double projection orthographique, c'est-à-dire comme si elle était vue de l'infini : hémisphère nord à gauche, hémisphère sud à droite, le pôle correspondant au centre de chaque disque, le méridien de Greenwich comme le segment horizontal reliant les pôles — tout ceci devrait être assez clair sur les dernières images où on commence vraiment à voir la forme des continents ; mais bien sûr, cette façon de projeter n'a vraiment rien à voir avec le calcul lui-même, qui est porte sur la sphère. L'image de droite montre chaque niveau d'harmoniques séparément (si on veut, chaque ligne de l'image de droite est donc la différence entre la ligne correspondante de l'image de gauche et la précédente : elle montre donc ce qui a changé ; de nouveau, à chaque fois, toutes les valeurs de m, c'est-à-dire −m, sont sommées pour le correspondant). On peut cliquer sur chacune des lignes de l'image pour la voir en plus gros. Sur l'image de gauche (sommes partielles), même si j'ai tronqué la fonction à −1 et +1, on voit assez nettement les artefacts classiques qui résultent d'une troncature de la transformée de Fourier (ici sphérique mais peu importe).

L'intérêt de cette décomposition en harmoniques sphériques est qu'elle est naturelle pour la sphère : ce que je veux dire, c'est qu'elle ne dépend pas du choix des coordonnées — de la position des pôles. Pour dire les choses autrement, si on fait tourner la sphère n'importe comment, chacun des niveaux de la décomposition (et, a fortiori, la somme des niveaux ≤) tourne de la même façon. (Il est essentiel ici de sommer tous les m : si on ne prenait que les termes avec m=0, par exemple, on obtiendrait une moyenne selon les cercles de latitude, et ça, ça dépend du choix des pôles.) Pour dire les choses encore autrement, et de façon un peu plus savante, quand on applique une rotation de la sphère, chaque harmonique sphérique Y[,m] est transformé en une combinaison linéaire des Y[,m′] pour le même (mais pour l'ensemble des −m′≤) : l'espace vectoriel engendré par les Y de niveau (exactement) est stable par rotations (c'est une représentation de SO(3), et c'est même, pour ceux qui savent ce que ça veut dire, la représentation irréductible de plus haut poids ).

En fait, pour un algébriste, la meilleure façon de présenter les choses est certainement la suivante : l'espace vectoriel engendré par les Y de niveau ≤ est tout simplement l'espace vectoriel des polynômes sur la sphère de degré ≤. (Attention cependant, comme x²+y²+z²=1 sur la sphère, le degré d'un polynôme y est mal défini ; je parle ici de l'espace, qui est de dimension (+1)², des restrictions à la sphère de l'espace — lui-même de dimension (+1)(+2)(+3)/6 — des polynômes de degré ≤ en x,y,z. On peut aussi préférer utiliser les polynômes harmoniques, c'est-à-dire dont le laplacien 3D est nul : pour ceux-là, la restriction à la sphère est une bijection, le degré est bien défini et coïncide avec la graduation par .) On peut même dire mieux : si on introduit le produit scalaire défini par l'intégration sur la sphère (normalisée pour avoir surface 1), alors la composante en harmoniques de niveau ≤ d'une fonction f est la projection orthogonale, pour ce produit scalaire, de f sur l'espace vectoriel des polynômes sur la sphère de degré ≤. Quant aux harmoniques sphériques réelles Y elles-mêmes, si je ne m'abuse, on peut dire que Y[0,0], Y[1,0], Y[1,1], Y[1,−1], Y[2,0], Y[2,1], Y[2,2], Y[2,−1], Y[2,−2], Y[3,0], etc. (ordonnées par puis par m en mettant les valeurs négatives après les positives), s'obtiennent par orthonormalisation de Gram-Schmidt à partir des polynômes 1, z, x, y, z², xz, x², yz, xy, z³, xz², x²z, x³, yz², xyz, x²y, etc. (ordonnés par degré total, puis par degré ≤1 en y, puis par degré en x). On obtient ainsi : Y[0,0] = 1 ; Y[1,0] = √3·z ; Y[1,1] = √3·x ; Y[1,−1] = √3·y ; Y[2,0] = √5·(z²−½x²−½y²) ; Y[2,1] = √15·xz ; Y[2,2] = √15·(½x²−½y²) ; Y[2,−1] = √15·yz ; Y[2,−2] = √15·xy ; Y[3,0] = √7·(z³−(3/2)x²z−(3/2)y²z) ; Y[3,1] = √42·(xz²−¼x³−¼xy²) ; etc.

Encore une autre façon de voir le niveau de la décomposition en harmoniques sphériques d'une fonction f est, peut-être à une constante près dont je ne suis pas très sûr, comme la convolée de cette fonction avec Y[,0] (j'insiste : convoler avec Y[,0] donne la projection sur tous les Y[,m] de ce niveau) : en général, la convolution de deux fonctions sur la sphère n'a pas de sens (on ne peut pas ajouter deux points sur la sphère), mais elle en a quand l'une des fonctions convolées est zonale, c'est-à-dire qu'elle ne dépend que de la latitude. En l'occurrence, Y[,0] vaut, à un coefficient de normalisation près, P[](cos(θ)) où P[] est un polynôme de Legendre et θ désigne la colatitude (=π/2 moins la latitude).

Du coup, les niveaux de la décomposition en harmoniques sphériques ont donc une vraie signification par rapport à la fonction sommée.

Le terme =0, ou ce que les physiciens appellent le terme monopôle, est simplement la moyenne de la fonction : dans l'exemple que j'ai pris, il nous renseigne donc sur la proportion de terre et de mer. Je trouve une moyenne de 0.4283, ce qui, compte tenu du fait que j'ai mis la terre à −1 et la mer à +1, signifie qu'il y aurait (1+0.4283)/2 soit 71.41% de mer, et 28.59% de terre ferme, sur la Terre. Je suppose que les mesures peuvent varier selon ce qu'on compte exactement comme terre et mer, notamment dans les régions polaires — je donne ici simplement ce qui résulte de l'image dont je suis partie, et je ne sais pas vraiment quelle est sa source — et peut-être quand on tient compte de l'aplatissement de la Terre, mais cette valeur est au moins réaliste. Pour dire les choses autrement, si on imagine que les terres émergées ont une densité surfacique constante égale à 1 sur la surface de la sphère (et que la mer a une densité nulle), ce qu'on mesure ici est la masse totale (c'est une façon bizarre de formuler les choses, mais la comparaison à la masse va être utile pour comprendre les deux termes suivants comme un terme de barycentre et un terme de moment d'inertie).

Le terme =1, ou terme dipôle, calcule la somme (ou la moyenne) des coordonnées x, y et z contre la fonction, donc donne aussi une information sur la Terre qui a un sens intuitif assez clair : sa direction correspond au barycentre des terres émergées, ce qui se rapporte au genre de problème dont je parlais ici. Mon calcul place ce barycentre à 44.4° de latitude (nord) et 29.0° de longitude (est), du côté de Constanța en Roumanie. Ceci colle au moins grossièrement avec ce qu'on trouve sur Wikipédia, mais celle-ci a l'air surtout de citer des crackpots qui veulent plus ou moins que ce centre ait un rapport avec la Grande Pyramide, et je ne vois pas de raison de penser que mon calcul serait moins bon que le leur (de nouveau, ça dépend sans doute surtout de ce qu'on compte comme terres émergées dans les régions arctiques).

Maintenant, il faut souligner ceci : ce dont je parle ci-dessus est la notion bien définie (en général) de barycentre sphérique, qui est tout simplement la projection sur la sphère (depuis son centre) du barycentre calculé en 3D (j'ai déjà dû citer le joli article de Galperin, A concept of the mass center of a system of material points in the constant curvature spaces, Comm. Math. Phys. 154 (1993) 63–84) ; mais dans le terme dipôle, il a bien trois composantes réelles (puisqu'il y a trois harmoniques sphériques au niveau 1, Y[1,0], Y[1,1] et Y[1,−1]), i.e., ce terme dipôle a une amplitude et pas juste une direction. Il donne donc aussi la profondeur du barycentre 3D. Mon calcul donne un moment dipolaire de la terre émergée de norme 0.0996, c'est-à-dire 34.83% du moment monopolaire (0.2859, la proportion de terre émergée, cf. ci-dessus), c'est-à-dire qu'il place le barycentre des terres émergées à 34.83% du rayon de la Terre à partir de son centre (soit à (x,y,z)=(0.2176,0.1205,0.2439) si z est orienté du centre vers le pôle nord, et x du centre vers le point de longitude 0 sur l'équateur).

(J'espère ne pas avoir mal placé un √3 ou ½ quelque part dans ce calcul : les harmoniques sphériques de niveau 1 avec la convention de normalisation que j'utilise sont Y[1,0]=√3·z, Y[1,1]=√3·x et Y[1,−1]=√3·y, du coup il y a des √3 qui se promènent ; il y a aussi un −2 à cause de ma convention sur les valeurs de la fonction, et il faut encore diviser par la valeur 0.2859 du terme monopôle si on veut obtenir la position du barycentre 3D.)

↑Entry #2321 [older| permalink|newer] / ↑Entrée #2321 [précédente| permalien|suivante] ↑

↓Entry #2320 [older| permalink|newer] / ↓Entrée #2320 [précédente| permalien|suivante] ↓

(mardi)

X+Y (=Le Monde de Nathan =A Brilliant Young Mind)

Je ne sais pas pourquoi ce film a plusieurs noms en anglais, et je ne sais pas non plus pourquoi ils ont décidé de l'appeler Le Monde de Nathan pour sa sortie en France (le 10 juin dernier ; sortie DVD le 21 octobre prochain), alors que X+Y passe très bien dans beaucoup de langues (en contrepartie du fait qu'il est pénible à rechercher sur Internet).

Je racontais il y a quelques mois que j'avais trouvé un peu agaçant que les scénaristes de The Imitation Game fassent passer Alan Turing pour un autiste alors qu'il ne l'était pas, et alimentent ainsi le cliché qui veut que les mathématiciens dans la fiction soient toujours au minimum socialement incompétents quand ils ne sont pas carrément mentalement atteints. Ici, le héros est un jeune autiste anglais doué pour les mathématiques et qui participe aux olympiades internationales de cette discipline. Comme les exercices des olympiades de mathématiques m'agacent[#] autant que le cliché dont je viens de parler, on peut dire que le film ne partait pas avec un a priori très favorable de ma part.

Pourtant, il m'a assez plu pour que je le recommande. D'abord, parce qu'il a réussi à éviter le cliché que je craignais : le héros est autiste et doué pour les mathématiques, et c'est clairement et pas donc ou car, et il y a d'autres personnages qui montrent assez nettement que les scénaristes ne confondent pas les deux. Ils évitent aussi le cliché apparenté (I'm looking at you, Good Will Hunting) du jeune prodige qui est forcément tellement fort en maths qu'il résout tout immédiatement[#2] et fait passer tous les autres pour des nuls — ici, sans vouloir spoiler, le héros est doué, mais il l'est de façon réaliste. C'est sans doute parce que le film est basé sur un documentaire, donc sur des faits réels, qu'il réussit à éviter l'hyperbole, mais c'est assez rare pour être souligné.

(Je ne dis pas que le film évite tous les clichés ou invraisemblances. Par exemple, on laisse beaucoup trop peu de temps à ceux qui préparent les olympiades pour réfléchir sur un problème donné : or absolument personne ne résout ce genre de problème en quelques secondes ; mais on peut justifier ce choix pour des raisons de rythme.)

Ensuite, je trouve assez rare de voir un film qui montre des mathématiques, fussent-elles des mathématiques d'olympiades (voir ma note ci-dessous pour la nuance), sans faire n'importe quoi : on ne nous montre pas seulement des gribouillis ressemblant vaguement à des formules et qui ne veulent rien dire : plusieurs problèmes d'olympiades (ou en tout cas tout à fait dans le genre des problèmes d'olympiades) sont posés, les réflexions sont plausibles, et il y a même une question pour laquelle la démonstration est faite au tableau, de façon correcte et complète (bon, c'est une question à mon avis trop facile pour être d'olympiades, et ce n'est pas très réaliste qu'on applaudisse le héros pour l'avoir trouvée, mais au moins un nombre non négligeable de spectateurs pourra comprendre).

Enfin, l'acteur principal, Asa Butterfield, est remarquable de justesse, dans un rôle pourtant difficile. (On l'avait déjà vu dans Hugo Cabret et Ender's Game, où il était également bon, mais le scénario de ces deux films à gros budget laissait à mon avis moins place à la subtilité des émotions.) L'actrice qui joue sa mère, en revanche, m'a semblé beaucoup moins bonne, mais peut-être que je me laisse influencer par le fait que le personnage m'agaçait.

Sinon, je trouve amusante la coïncidence suivante : j'ai fait référence à l'entrée de blog que j'ai écrite sur le biopic de Turing, qui y est présenté à tort comme autiste, et dans cette même entrée j'évoquais aussi le film, sorti au même moment, sur la vie de Hawking, qui lui a (vraiment) une maladie neurodégénerative. Or le film dont je parle ici met en scène à la fois un personnage autiste et un autre qui a une maladie neurodégénerative (et il est explicitement comparé à Hawking, d'ailleurs). Enfin, peut-être que ce n'est pas une coïncidence mais une sorte de référence.

[#] Pour essentiellement deux raisons. Primo, je trouve que ça a peu de rapport avec les mathématiques : il s'agit de problèmes généralement atrocement astucieux et ne faisant appel à aucune théorie générale, alors que, à mon sens, les mathématiques consistent justement à trouver des théories générales pour éviter les astuces. Bon, pour leur défense, certains problèmes d'olympiades sont au moins assez jolis, ce qui est aussi une caractéristique importante des bonnes mathématiques à mes yeux — mais seulement certains, parce qu'il y en a beaucoup qui sont non seulement difficiles et astucieux mais aussi fondamentalement moches et sans intérêt. (Je précise que je ne suis pas vexé d'y être mauvais : je crois même que je m'en sors honorablement, ou en tout cas que je m'en sortais honorablement quand j'avais l'âge. On m'a d'ailleurs demandé, comme j'avais eu un prix au Concours général de maths, de participer à l'équipe française de la 35e olympiade à Hong Kong — mais comme j'avais aussi un autre prix en physique pour lequel j'étais invité aux États-Unis au même moment, je n'y suis pas allé.) Secundo, et sans doute le plus important : je trouve que l'idée de compétition, que ce soit entre les individus ou les pays, va complètement à l'encontre de l'esprit de la science qui est — ou devrait être — collaboratif et non compétitif.

[#2] Hint : dans la réalité, les maths sont dures pour tout le monde. Si elles ne l'étaient pas, l'hypothèse de Riemann serait décidée à l'heure qu'il est. (En fait, on peut même défendre l'idée que c'est une conséquence d'un théorème et d'un postulat physico-philosophique de Church et Turing que : les mathématiques ne peuvent pas être triviales pour aucun habitant de cet Univers, humain, extra-terrestre ou ordinateur.)

↑Entry #2320 [older| permalink|newer] / ↑Entrée #2320 [précédente| permalien|suivante] ↑

↓Entry #2317 [older| permalink|newer] / ↓Entrée #2317 [précédente| permalien|suivante] ↓

(mercredi)

Petit supplément à ma page d'ondes sur la sphère

J'ai présenté avant-hier une page en JavaScript (enfin, deux : avec WebGL ou sans) qui affiche une animation d'ondes sur une sphère (un peu plus précisément, une solution de l'équation des ondes (∂²/∂t²−c²Δ)φ=0, où Δ est le laplacien sphérique ; ou en fait, trois solutions à la fois, une pour chaque composante de couleur RGB). J'ai ajouté un bouton pause, mais ce n'est pas le plus intéressant : j'ai surtout ajouté toutes sortes de modes spéciaux.

(Si la description qui suit ne vous intéresse pas, sautez directement jusqu'au dernier paragraphe.)

On m'avait demandé si je pouvais permettre un choix de la condition initiale (pour les non-mathématiciens : la configuration à partir de laquelle l'onde évolue) : ce serait assez compliqué de fournir une façon de faire ça en général, mais on peut quand même permettre de choisir une configuration qui a des symétries particulières (qui se conserveront avec l'évolution dans le temps). C'est ce que ma page JavaScript permet maintenant. Un exemple de tel cas est la situation où il y a symétrie par rapport au centre de la sphère : l'état est en permanence le même en deux points antipodaux l'un de l'autre (i.e., φ(−x,−y,−z) = φ(x,y,z)) ; si on veut, on peut considérer qu'il s'agit alors d'une équation des ondes sur le plan projectif réel (qui est la sphère où on a identifié les paires de points antipodaux) ; ceci a l'intérêt qu'on voit alors la totalité de la configuration (puisque le programme n'affiche qu'un hémisphère, mais l'autre s'en déduit par symétrie). On peut imaginer d'autres symétries de ce genre, évidemment : par rapport à un plan (si c'est le plan parallèle au plan de projection — que j'appelle z=0 — alors on voit de nouveau toute la configuration, puisque de nouveau l'autre hémisphère est symétrique, mais cette fois par rapport à un plan, ce qui est donc subtilement différent) ; ou par rapport à un axe, et dans ce cas, à différents niveaux. (Je me suis limité à une symétrie d'ordre 2 ou 3 par rapport à l'axe de vision, parce que je ne calcule pas assez d'harmoniques pour qu'une symétrie d'ordre supérieur puisse être intéressante à voir, déjà 3 est limite. C'est dommage, parce qu'en général on pouvait demander des groupes de symétrie plus intéressants, à savoir les symétries d'un des solides réguliers. Mais bon, même dans le cas de la symétrie cubique/octaédrale, je n'ai pas le courage de calculer l'action sur les harmoniques sphériques.)

Mais j'ai un autre type de configuration particulière à proposer : il s'agit des cas où l'équation des ondes conserve la « masse totale », c'est-à-dire techniquement la norme L² (en l'occurrence, sur chacun des canaux de couleur) : pour parler grossièrement, des creux et des bosses peuvent se déplacer, mais leur quantité totale doit rester inchangée (note : la moyenne reste de toute façon constante — dans mon cas, à 0 que je représente par le gris intermédiaire qui sert aussi de fond — et c'est ici de la moyenne quadratique que je parle). Je ne sais pas quel est le terme standard (il y en a probablement un) pour désigner ce genre de configurations de l'équation des ondes. La situation complètement opposée est celle d'une onde stationnaire : très grossièrement parlant, dans une onde stationnaire, les creux et les bosses apparaissent et disparaissent, mais ne changent pas de place. Comme ce n'est pas terriblement intéressant, j'ai défini les configurations « stationnaires par niveau », qui sont celles où chaque niveau d'harmoniques sphériques (et chaque canal RGB) définit une onde stationnaire. Ces deux conditions se combinent d'ailleurs agréablement avec la condition d'être symétrique par rapport au centre de la sphère (« projectif », cf. ci-dessus), donc j'ai aussi mis les conjonctions en question.

Pour ceux qui connaissent un peu plus de maths, voici une explication plus claire sur ces deux conditions de conserver la masse L² et d'être stationnaire par niveau : en général, on peut écrire φ = ∑u,m(tY[,m] (pour −m, et parcourant les entiers naturels — même si mon JavaScript ne monte que jusqu'à 8), où les Y[,m] sont les harmoniques sphériques (réelles), qui vérifient (ΔY[,m] = −(+1)·Y[,m]) et sont orthogonaux au sens L² et u,m(t) est une sinusoïde de fréquence (c/2π)·√((+1)) (c'est ça qui assure qu'on vérifie l'équation des ondes). Cette dernière condition peut s'écrire u,m(t) = Re(Z,m·exp[i·c·√((+1))·t]) avec Z,m un nombre complexe (dont le module et l'argument déterminent l'amplitude et la phase de cette sinsuoïde). La condition de conserver la masse L² signifie que la somme des carrés de ces parties rélles ne dépend pas de t, ce qui revient en fait à ce que la somme des carrés des complexes Z,m (pour −m) s'annulle pour chaque . La condition d'être stationnaire par niveau, elle, signifie que pour chaque , les Z,m ont tous la même phase à π près (i.e., ils sont proportionnels par des nombres réels).

La première condition m'a d'ailleurs conduit au problème suivant, qui est assez perturbant : comment tirer au hasard de façon « naturelle » des nombres complexes Z1,…,Zk tels que Z1² + ⋯ + Zk² = 0 ? (Il revient au même de chercher des réels A1,…,Ak et B1,…,Bk tels que la somme des Ai² soit égale à la somme des Bi², et que la somme des Ai·Bi soit nulle, i.e., deux vecteurs de même norme et orthogonaux.) En l'absence de condition, je choisis les Zi en tirant leur partie réelle Ai et leur partie imaginaire Bi indépendamment selon une distribution gaussienne (dont l'écart-type décroît avec , mais ce n'est pas la question ici) ; pour une distribution stationnaire par niveau, je choisis un complexe de module 1 une fois pour toutes, et je le multiplie par des réels tirés selon une distribution gaussienne ; mais pour Z1,…,Zk tels que Z1² + ⋯ + Zk² = 0, ce n'est pas clair ce qu'il vaut mieux faire. Je pensais prendre une distribution gaussienne conditionnée par cette condition, mais je suis tombé sur le paradoxe de Borel, et du coup je ne sais pas exactement quoi faire. Au final, je tire Ai et Bi selon des distributions gaussiennes, je projette B sur l'orthogonal à A, et je le renormalise pour avoir la même norme que A (on se convaincra, au moins, que c'est en fait symétrique entre A et B), mais peut-être que la distribution que je donne à la norme carrée de A n'a pas le bon nombre de degrés de liberté (si tant est qu'il y en ait un « bon »).

Toutes ces choses étant dites, je serais curieux de savoir quelle impression font ces différents « modes », même (et surtout) sur ceux qui n'ont pas lu ou compris les explications ci-dessus. Y en a-t-il qui vous semblent plus jolis ? Et pensez-vous pouvoir les reconnaître (à part ceux qui présentent des symétries vraiment évidentes, c'est-à-dire les axialement 2-symétrique et 3-symétrique et les symétriques par rapport aux plans x=0 et y=0) ? Il y a une façon de reconnaître les modes projectifs (même si elle n'est pas évidente quand on laisse tourner l'animation ; je laisse en exercice de deviner de quoi il s'agit). Mais pour ce qui est des modes L²-conservatif et stationnaire par niveau, je n'arrive pas à savoir si j'arrive vraiment à les reconnaître ou si c'est une sorte d'effet placébo (je devrais écrire de quoi faire des tests à l'aveugle) ; et de même pour le mode symétrique par rapport à z=0.

↑Entry #2317 [older| permalink|newer] / ↑Entrée #2317 [précédente| permalien|suivante] ↑

↓Entry #2316 [older| permalink|newer] / ↓Entrée #2316 [précédente| permalien|suivante] ↓

(lundi)

Une envoûtante animation d'ondes sur la sphère

Comme je me suis décidé relativement récemment (j'y ai fait allusion au passage) à apprendre des choses que j'aurais sans doute dû savoir depuis longtemps sur l'analyse harmonique élémentaire sous les groupes compacts, j'ai voulu faire joujou avec les harmoniques sphériques.

Comme il faut bien que je dise au moins approximativement de quoi il est question, voici une petite digression à ce sujet.

Beaucoup de gens, même non-mathématiciens, ont probablement entendu parler des séries de Fourier, qui sont une façon d'analyser un signal périodique en le décomposant en fréquences harmoniques : grossièrement parlant, il s'agit d'écrire une fonction de fréquence f comme comme de sinusoïdes de fréquences n·f (la « n-ième harmonique ») dont l'amplitude et la phase dépendent de n. Par ailleurs, un signal périodique, c'est la même chose qu'une fonction sur le cercle (un temps circulaire, c'est la même chose qu'un temps qui se répète périodiquement : voyez l'article Wikipédia que je viens de lier, il y a plein d'animations très cool dessus) : les séries de Fourier sont donc, si on préfère, une façon d'analyser les fonctions sur le cercle.

Mais il existe aussi une façon très analogue de décomposer un signal sur la sphère (il existe des choses analogues en toute dimension, et même sur des espaces nettement plus généraux, mais limitons-nous à la sphère de dimension 2) : au lieu de développer en sinusoïdes de fréquences multiples de la fondamentale comme on le fait pour les séries de Fourier (séries trigonométriques), on développe en termes de ce qu'on appelle des harmoniques sphériques, et le développement correspondant s'appelle l'expansion multipolaire (note : ces deux articles Wikipédia parlent en gros de la même chose, et il faudrait sans doute les fusionner, ou au moins les harmoniser — mais c'est un bon exemple de la manière dont des gens peuvent faire la même chose sans vraiment se parler). Ce développement a beaucoup de propriétés communes avec le développement de Fourier. Une différence notable est qu'au lieu d'avoir juste le multiple n de la fréquence fondamentale (si on veut, le nombre de bosses sur le cercle), les harmoniques sphériques Y[,m] dépendent de deux entiers, étant en quelque sorte le niveau de fréquence totale et m la fréquence en longitude. (Elles dépendent, du coup, du choix d'un pôle nord : si on effectue une rotation quelconque de la sphère, la fonction Y[,m] se transforme en une combinaison linéaire des Y[,m′] pour ce même et l'ensemble de tous les m′. Une recherche sur Google images donnera au moins une idée de ce à quoi ces fonctions ressemblent, sous différentes représentations graphiques — et on voit vaguement en quoi ce sont des analogues sur la sphère des sinusoïdes sur le cercle.) Du coup, au lieu d'avoir une série simple de coefficients de Fourier, on a une série à deux indices ,m de coefficients harmoniques.

Cette décomposition a des applications dans beaucoup de domaines : elle est très souvent pertinente pour étudier une fonction sur la sphère. Par exemple, une quantité définie sur la Terre tout entière, ça aurait un sens, et souvent un intérêt, d'essayer de la décomposer en harmoniques sphériques (je ne sais pas, moi, la forme du géoïde, la température de surface à un instant donné, la densité de population humaine, ce genre de choses ; les physiciens font ça aussi, entre autres, pour analyser le rayonnement cosmologique fossile et confronter son spectre aux théories).

Mais un intérêt particulier de la décomposition en séries de Fourier (ou de la transformée de Fourier), c'est aussi de résoudre des équations aux dérivées partielles linéaires ; d'ailleurs, Fourier cherchait, historiquement, à résoudre l'équation de la chaleur sur un cercle. C'est aussi le cas pour la décomposition en harmoniques sphériques, essentiellement à cause du fait que les harmoniques sphériques Y[,m] sont des vecteurs propres du laplacien shérique (ΔY[,m] = −(+1)·Y[,m]), ce qui les rend très confortables dans des équations faisant intervenir cet opérateur, comme l'équation de la chaleur ou l'équation des ondes. C'est pour cette raison que les harmoniques sphériques apparaissent dans la résolution de l'équation de Schrödinger pour le calcul des orbitales des électrons dans un atome.

Bref, j'ai fait une petite page en JavaScript qui représente l'évolution — linéaire — d'une onde sur une sphère (ou peut-être plutôt trois ondes, une par composante de couleur). En fait, j'ai fait deux versions de cette page :

ici en WebGL et ici sans

(la version WebGL est généralement beaucoup plus rapide que celle sans — cette dernière pourrait mettre plusieurs secondes, voire dizaines de secondes, à se charger, et affichera certainement moins d'images par seconde — mais la version WebGL a aussi plus de chances de ne pas marcher, ou de marcher bizarrement, ou dans de rares cas de crasher le navigateur ; à part ça, elles sont censées afficher exactement la même chose, aux choix aléatoires initiaux près).

Je trouve ça extrêmement joli et vraiment fascinant à regarder. J'ai passé un temps fou à regarder cette sphère opalescer jusqu'à me laisser hypnotiser par elle. (Mais pourquoi on ne m'a jamais dit ça, quand on m'a parlé de l'équation des ondes, que ça pouvait être aussi joli ?)

Après, je suis un peu déçu par les navigateurs. La version sans WebGL est lente, ce qui est peut-être normal parce qu'elle doit effectuer en gros 81 multiplications+additions par pixel et par rafraîchissement, mais je pensais quand même que les ordinateurs arriveraient à faire un ordre de grandeur plus vite que ça, surtout qu'on m'a tellement vanté que JavaScript était maintenant un langage ultra-rapide. La version avec WebGL est d'une rapidité acceptable, mais j'ai horriblement souffert pour l'écrire, à me cogner contre une limitation après une autre de ce truc (par exemple, j'avais voulu faire ça avec des textures flottantes, mais déjà c'est une extension pas garantie et ensuite de toute façon, on ne peut pas demander 81 textures flottantes, quelle que soit leur taille, c'est trop). Dans tous les cas, je n'ai pas vraiment pu aller au-delà de 9 niveaux d'harmoniques sphériques (c'est-à-dire <9 ; c'est pour ça que la sphère est aussi lisse) : c'est dommage, parce que je pense que ça peut être intéressant avec beaucoup plus, mais je ne vois pas vraiment comment améliorer l'efficacité.

(Les téméraires peuvent reprendre le fichier et modifier la ligne var degree_cut = 9 pour remplacer 9 par le nombre qu'ils voudront, mais déjà pour 12, la version sans WebGL est inacceptablement lente chez moi — bon, il est vrai qu'on peut baisser la résolution pour compenser, en changeant les attributs width="300" height="300" de l'élément canvas — et la version WebGL ne marche tout simplement plus puisque le « fragment shader » devient trop long et bute contre une autre limitation du machin.)

J'essaierai sans doute de calculer une animation en haute résolution et avec beaucoup d'harmoniques (j'aimerais bien arriver à mettre quelque chose comme 30 niveaux), et la mettre sur YouTube. Qui, bien sûr, massacrera impitoyablement la qualité de ce que j'aurai calculé (surtout que les formats vidéo sont très mauvais avec les couleurs qui bougent), mais bon, je ne sais pas bien quoi faire de mieux.

Mise à jour : Voici un lien vers la version sur YouTube, où j'ai calculé 31 niveaux d'harmoniques ce qui donne plus de détails sur la sphère (détails malheureusement en partie obscurcis par la compression vidéo).

Ajout/suite : Voir l'entrée suivante.

↑Entry #2316 [older| permalink|newer] / ↑Entrée #2316 [précédente| permalien|suivante] ↑

↓Entry #2310 [older| permalink|newer] / ↓Entrée #2310 [précédente| permalien|suivante] ↓

(lundi)

Comment faire un jeu de Tribble

Je continue sur les idées développées dans cette entrée (et dans une moindre mesure la suivante) : ma métaphorique petite sœur se plaint qu'un quadrangle généralisé ce n'est pas, nonobstant mes explications fumeuses, une structure très convaincante pour inventer des jeux de cartes, alors que le jeu de Dobble a au moins réussi à convaincre des gens de l'éditer. Si ce dernier est basé sur le principe que deux cartes quelconques ont toujours un symbole en commun, peut-on faire un paquet où trois cartes quelconques auraient toujours un symbole en commun ?

Réponse : oui, on peut, mais je crois qu'il va falloir admettre un nombre de symboles par carte un peu désagréablement élevé (ou un nombre total de cartes bien bas) :

[Carte à quatre symboles] [Carte à quatre symboles] [Carte à quatre symboles] [Carte à quatre symboles] [Carte à quatre symboles] [Carte à quatre symboles] [Carte à quatre symboles] [Carte à quatre symboles] [Carte à quatre symboles] [Carte à quatre symboles] [Carte à quatre symboles] [Carte à quatre symboles] [Carte à quatre symboles] [Carte à quatre symboles] [Carte à quatre symboles] [Carte à quatre symboles] [Carte à quatre symboles] [Carte à quatre symboles] [Carte à quatre symboles] [Carte à quatre symboles] [Carte à quatre symboles] [Carte à quatre symboles] [Carte à quatre symboles] [Carte à quatre symboles] [Carte à quatre symboles] [Carte à quatre symboles]

J'ai créé ici 26 cartes portant chacune 30 symboles choisis parmi un répertoire de 130, chaque symbole apparaissant sur 6 cartes différentes, deux cartes distinctes ayant toujours exactement 6 symboles en commun, et trois cartes distinctes ayant toujours exactement 1 symbole en commun. On peut donc imaginer toutes sortes de jeux de rapidité (ou en fait, plutôt de patience) consistant à chercher le symbole en commun à trois cartes, selon des règles inspirées de celles qui servent pour Dobble. Maintenant, à vrai dire, je trouve ça surtout excessivement fastidieux : il m'a fallu plus de deux minutes pour trouver le symbole commun entre les trois premières cartes (notons que l'ordre des cartes affiché ci-dessus n'est pas aléatoire, et ce symbole est en fait commun aux cinq premières cartes et à la dernière, mais ce n'est pas un bug), et je ne trouve pas ça spécialement ludique. Mais bon, il y a plein de choses que je ne trouve pas ludique et que d'autres gens aiment, alors peut-être que ce jeu peut quand même trouver des adeptes (si quelqu'un veut un tirage physique, qu'il me fasse signe).

Ajout : Un jeu qu'on pourrait jouer avec ces cartes consiste à distribuer à chaque joueur le même nombre de cartes (le plus élevé possible) en en laissant deux face retournée sur la table ; quiconque peut montrer du doigt un symbole en commun entre une carte quelconque de sa main et les deux cartes sur la table pose sa carte sur la table et défausse l'une des deux qui y étaient déjà (de façon qu'il y en ait toujours deux) ; le jeu se continue jusqu'à ce que quelqu'un se soit ainsi débarrassé de toutes ses cartes. La particularité de cette procédure est que celui qui arrive à poser une de ses cartes gagne un avantage pour le coup suivant vu qu'il a pu déjà rechercher l'intersection entre les deux cartes sur la table.

Pour répondre à des questions naturelles : l'ordre de disposition des symboles sur une carte donnée est totalement aléatoire (j'ai commencé par essayer de trouver une logique qui me convienne, mais j'ai vite craqué et opté pour un tirage au hasard — enfin, au hasard déterministe —, au prétexte qu'il vaut mieux un chaos garanti qu'un ordre basé sur une logique douteuse) ; et la permutation des symboles à l'intérieur du répertoire l'est aussi. L'ordre des cartes affiché ci-dessus n'est pas aléatoire, mais ça n'a pas d'importance puisqu'un vrai jeu de cartes serait de toute façon mélangé avant usage. Et sinon, je sais que mon choix de symboles est certainement merdique, mais je n'accepterai de critiques que de la part de gens qui peuvent en suggérer un meilleur ; j'ai cherché à avoir une proportion raisonnable de signes d'écriture (lettres ou caractères chinois) et de dessins, j'ai voulu éviter les symboles qui se ressemblent trop (par exemple, je n'ai pas mis le ‘C’ parce qu'il est trop semblable au ‘G’, je n'ai pas mis le ‘Ш’ parce qu'il est trop semblable au ‘Щ’, etc.) même si je sais qu'il en reste, et globalement il n'y a pas trop de logique mais c'est un peu l'idée.

J'explique maintenant comment construire la chose, parce que je trouve ça assez joli : pour résumer très brièvement, on peut dire que si le jeu de Dobble est basé sur l'idée que deux points distincts dans le plan (projectif, mais peu importe) déterminent une unique droite, celui-ci est basé sur l'idée que trois points distincts sur la sphère déterminent un unique cercle (cercle signifiant petit ou grand cercle, i.e., l'intersection de la sphère avec un plan ; en l'occurrence, le plan passant par ces trois points) : on imaginera les cartes du jeu comme les points de la sphère, et les symboles sur une carte comme les cercles passant par ce point. Il ne reste plus qu'à transformer ça en une structure finie en passant sur un corps fini, donc à expliquer ce que sphère et cercle veulent dire dans ce contexte. En gros, je dois parler un peu de géométrie de Möbius.

↑Entry #2310 [older| permalink|newer] / ↑Entrée #2310 [précédente| permalien|suivante] ↑

↓Entry #2308 [older| permalink|newer] / ↓Entrée #2308 [précédente| permalien|suivante] ↓

(mercredi)

Comment faire un jeu de cartes à partir d'un quadrangle généralisé

L'entrée précédente m'a donné envie de concevoir des jeux de cartes avec des structures combinatoires mathématiques remarquables. Je vais déjà en tirer un avec une structure liée à celle des 27 droites sur une surface cubique (à savoir, l'ensemble des 36 double six de telles droites)[#], mais ce serait plutôt pour faire de la cartomancie oulipienne. Je me demandais ce que je pourrais inventer de plus jouable. Et d'un autre côté, parmi les structures combinatoires que j'avais vaguement à l'esprit, il y avait (je les ai mentionnées dans l'entrée précédente, et je vais dire ci-dessous de quoi il s'agit) les quadrangles généralisés.

((Ceux de mes lecteurs qui ne sont pas intéressés par les aspects mathématiques peuvent directement sauter au dessin des cartes ci-dessous, après quoi je pose quelques questions de design, si j'ose dire.))

Pour essayer d'imaginer quelque chose de jouable, j'ai médité sur la structure d'un jeu ordinaire de 52 cartes. Tout le monde sait qu'il s'agit des 13×4 cartes constituant chacune des combinaisons, des couples si on veut, entre un symbole de {A,2,3,4,5,6,7,8,9,X,V,D,R} (la « valeur » de la carte) et un symbole de {♣,♢,♡,♠} (la « couleur » de la carte, le terme français était d'ailleurs épouvantablement ambigu parce qu'il recouvre à la fois ce que les Anglais appellent suit, c'est-à-dire le symbole que je viens de dire, et ce que les Anglais appellent colour, c'est-à-dire noir pour ♣,♠ ou rouge pour ♢,♡ — mais passons). Mathématiquement, on a donc affaire au produit cartésien {A,2,3,4,5,6,7,8,9,X,V,D,R} × {♣,♢,♡,♠}, qui n'est pas une structure combinatoire très intéressante. Si on considère les cartes comme des points et les symboles comme des droites (verticales ou horizontales : voir le dessin ci-dessous), on a affaire à une simple grille. Maintenant, voici quelques propriétés de cette « géométrie », qui peuvent paraître bizarrement compliquées, mais dont on va voir le sens à les énoncer ainsi :

A 2 3 4 5 6 7 8 9 X V D R
  1. Sur chaque carte figurent exactement 2 symboles (distincts) [à savoir, l'indication de sa valeur et l'indication de sa couleur].
  2. Chaque symbole figure sur exactement 4 ou 13 cartes (distinctes) [4 dans le cas d'une valeur, 13 dans le cas d'une couleur].
  3. Deux cartes ayant deux symboles en commun coïncident [il n'y a pas de cartes différentes ayant la même valeur et la même couleur]. Diverses reformulations équivalentes : deux cartes distinctes ont au plus un symbole en commun ; deux symboles distincts figurent sur au plus une carte ; deux symboles figurant tous les deux sur deux cartes distinctes coïncident.
  4. Si C est une carte et σ est un symbole qui ne figure pas sur C, alors il existe exactement une carte D et un symbole τ tels que σ figure sur D et τ figure à la fois sur C et sur D. [Explication ci-dessous.]

La propriété (4) peut sembler bizarre, mais concrètement, elle signifie simplement que si C est une carte et σ est soit une valeur différente soit une couleur différente de celle de C, alors il existe une carte D qui a cette valeur ou couleur et qui pour l'autre symbole (couleur ou valeur respectivement) τ a la même que celle de C.

Cette dernière propriété, d'ailleurs, est en quelque sorte celle utilisée dans un nombre essentiellement infini de jeux de cartes (par exemple le jeu commercial Uno, le « huit américain » ou « maou maou », le « Tschau Sepp » suisse, etc.) qui sont des variantes mineures autour du principe suivant : chaque joueur a des cartes dans sa main dont il doit se débarrasser, ils jouent tour à tour et chacun peut poser une carte ayant un symbole commun avec la carte précédemment jouée (c'est-à-dire concrètement : ayant la même valeur ou la même couleur — le plus souvent la même couleur, bien sûr, puisqu'il y a plus de telles cartes). La propriété signifie alors que si la carte C a été jouée et que je veux passer le jeu à σ qui n'est pas actuellement jouable (i.e., changer la valeur ou la couleur), il y a une unique carte jouable D qui permettra de faire ce changement.

Si j'ai écrit les propriétés sous la forme bizarre ci-dessus, c'est pour pouvoir amener la définition d'un quadrangle généralisé, ou plus exactement, un quadrangle généralisé fini de paramètres (s,t) (deux entiers), définition que je vais formuler ici avec des cartes et des symboles (mais les termes classiques seraient points et droites, sachant que la définition est symétrique entre les deux, à permutation près des paramètres s et t ; je fais ici la convention que les cartes sont les points et les symboles les droites, mais le contraire irait tout aussi bien) :

  1. Sur chaque carte figurent exactement t+1 symboles (distincts).
  2. Chaque symbole figure sur exactement s+1 cartes (distinctes).
  3. Deux cartes ayant deux symboles en commun coïncident. Diverses reformulations équivalentes : une carte est complètement déterminée par la donnée de deux quelconques de ses symboles ; deux cartes distinctes ont au plus un symbole en commun ; deux symboles distincts figurent sur au plus une carte ; deux symboles figurant tous les deux sur deux cartes distinctes coïncident ; un symbole est complètement déterminé par la donnée de deux cartes sur lequel il figure.
  4. C σ D τ Si C est une carte et σ est un symbole qui ne figure pas sur C, alors il existe exactement une carte D et un symbole τ tels que σ figure sur D et τ figure à la fois sur C et sur D. (Cf. dessin ci-contre.)

Les propriétés (3)&(4) sont donc exactement les mêmes que ce que j'ai énoncé pour un jeu de cartes usuelles. La (1) est une généralisation de ce qu'elle était ci-dessus pour autoriser plus que 2 symboles par carte (par contre, on notera bien que la troisième propriété continue à parler de deux symboles : une carte est complètement déterminée par deux quelconques de ses symboles). La propriété (2), en revanche, diffère de ce qu'on avait pour un jeu de cartes ordinaires, en ce sens que chaque symbole figure maintenant sur le même nombre de cartes, au lieu qu'il y ait des types de symboles figurant sur un nombre plus ou moins grand de cartes.

Dans la propriété (4), on dit parfois que τ est le perpendiculaire de σ à travers C : cette terminologie a l'avantage de bien faire ressortir l'unicité, et elle est raisonnable quand on pense à l'exemple d'une grille (par exemple, le perpendiculaire à ♠ passant par 8♡ est 8 : c'est bien le cas sur le dessin de la grille que j'ai fait plus haut). Néanmoins, cette terminologie suggère une notion métrique (des angles), qui n'existent pas ici : on demande simplement une condition d'incidence entre σ et τ (à savoir, qu'ils figurent sur la carte D). D'autre part, comme cartes et symboles jouent des rôles totalement symétriques dans les propriétés (j'ai fait mes dessins avec les cartes pour points et les symboles pour droites, mais je pouvais faire le contraire), on pourrait tout aussi bien dire que D est la perpendiculaire de C à travers σ (et pour le coup, dans le cas d'une grille, c'est beaucoup moins intuitif : la perpendiculaire à 8♡ par ♠ est 8♠). Passons.

↑Entry #2308 [older| permalink|newer] / ↑Entrée #2308 [précédente| permalien|suivante] ↑

↓Entry #2307 [older| permalink|newer] / ↓Entrée #2307 [précédente| permalien|suivante] ↓

(mercredi)

Le jeu de cartes Dobble et la géométrie projective expliquée aux enfants

[Arrangement des cartes de Dobble]J'avais déjà entendu parler du jeu de cartes Dobble (appelé Spot it! aux États-Unis). Il s'agit d'un jeu de 55 cartes circulaires (logiquement il devrait y en avoir 57, mais il en manque deux pour une raison que seul l'éditeur du jeu connaît), chacune portant 8 symboles différents parmi 57 symboles possibles (un peu façon émojis : cœur, clé, cadenas, flocon de neige, sens interdit, coccinelle, vous voyez le genre). La propriété sur laquelle se base le jeu est que deux cartes quelconque du jeu ont toujours un et un seul symbole en commun, et le jeu est un jeu de rapidité consistant à identifier le plus rapidement possible ce symbole (selon les variantes : entre une carte qu'on a en main et une carte au sommet d'une pioche, ou quelque chose comme ça). Le jeu est assez distrayant et intéressant en ce que c'est un jeu auquel des adultes et des très jeunes enfants peuvent jouer ensemble et trouver également rigolo, ce qui n'est pas une contrainte évidente.

Mais son intérêt est également mathématique, car il s'agit d'une structure combinatoire classique et remarquable : pour les mathématiciens qui me lisent, disons brièvement qu'il s'agit du plan projectif sur le corps fini à 7 éléments (les cartes étant, disons, les points, et les symboles les droites — ou le contraire si on préfère — et le fait pour un symbole de figurer sur une carte étant la relation d'incidence). Pour les non-mathématiciens, on peut mentionner une autre propriété, duale de la précédente, qu'ont les cartes : deux symboles quelconques figurent toujours sur une et une seule carte — sauf s'il s'agit d'une des deux cartes « manquantes ». Mais le jeu n'exploite pas cette autre propriété, ce qui est vraiment dommage, parce que c'est la combinaison des deux qui rend la structure mathématiquement vraiment intéressante (voir ici par exemple). Voir aussi cet article de vulgarisation sur le site Images des mathématiques qui tente d'expliquer un peu les choses pour les non-mathématiciens. Comme son auteur (que je salue au passage si par hasard il me lit), je trouve vraiment dommage que les éditeurs n'aient pas eu de meilleure idée pour exploiter la structure combinatoire remarquable qu'ils ont concrétisée que de faire un simple jeu de rapidité (et n'utilisant qu'une seule des deux propriétés duales que j'ai mentionnées), et j'appelle à ce qu'on invente d'autres jeux amusants avec ce jeu de cartes. On pourrait par exemple jouer à choisir deux symboles (i.e. : deux joueurs en choisissent chacun un, le notent sur un papier, et le révèlent simultanément), et essayer de trouver le plus rapidement possible, toutes les cartes étant étalées simultanément, quelle est celle qui contient les deux symboles choisis — mais il y a certainement plus intelligent à faire.

J'avais entendu parler de Dobble, disais-je, parce que plusieurs personnes m'avaient indépendamment proposé, comme une énigme, d'imaginer comment je concevrais un tel jeu (ce qui n'est pas vraiment une énigme, parce que pour un matheux un peu algébriste, un peu géomètre et/ou un peu combinatoricien, la structure d'un plan projectif sur un corps fini est tellement naturelle que j'avais donné la réponse avant d'avoir compris la question). Toujours est-il que je n'avais pas vu les cartes ni retenu le nom. Mais ce week-end, en passant chez des amis à Lyon, j'ai vu le jeu en question. (Il s'agit, d'ailleurs, des mêmes amis qui m'avaient fait découvrir le jeu de Set, un autre jeu de cartes basé sur une géométrie finie — en l'occurrence l'espace affine de dimension 4 sur le corps à 3 éléments.)

Et il y a assurément quelque chose de fascinant pour un matheux (surtout fasciné par les jolies structures combinatoires) d'avoir un plan projectif fini entre les mains. Ceci permet d'expliquer de façon visuelle et interactive comment fonctionne la géométrie projective finie bien mieux que je ne saurais le faire avec un tableau. Avec toutes sortes de questions qui se soulèvent naturellement, par exemple : comment trouver, le plus efficacement possible, quelles sont les deux cartes manquantes ? (imaginons que j'aie un jeu complet de 57 cartes, avec un ensemble de symboles inconnu a priori, et que j'en retire deux au hasard, comment trouver le plus rapidement l'ensemble des symboles de ces deux cartes retirées ?). Et comment disposer efficacement les cartes pour exhiber la structure géométrique ? Sur la photo ci-dessus, même si elle n'est pas terrible, on voit un tel arrangement possible : le carré 7×7 principal (celui où il manque une carte dans le coin en bas à gauche) a la propriété que chaque ligne de cartes a un symbole en commun, chaque colonne en a un, mais aussi chaque diagonale (prolongée cycliquement), chaque antidiagonale, et en fait, les diagonales de pas quelconques (cherchez les cartes ayant un cactus, par exemple) — un matheux dira qu'il s'agit du plan affine sur le corps à 7 éléments, et les cartes restantes (où il en manque aussi une) sont la droite à l'infini. Avec cette disposition, il n'est pas difficile de trouver quels sont les symboles des deux cartes manquantes ; reste que c'est un chouïa fastidieux d'y parvenir. Je me suis aussi amusé à calculer la disposition (duale) des symboles, ce qui permet de faire des petits tours de magie, du genre : choisis une carte, ne me la montre pas, dis-moi deux symboles qu'elle porte, et je te dirai quels sont les autres.

Je me serais précipité pour acheter le jeu s'il n'y avait pas ce gag des deux cartes manquantes, ce qui pour un obsessif-compulsif comme moi est aussi frustrant que l'idée d'avoir un beau rayonnage de livres tous identiques sauf un qui dépasserait les autres de 1cm. (Il existe aussi un Dobble Kids, dont les images laissent suggérer qu'il doit être basé sur un plan projectif d'ordre 5 au lieu de 7, et au lieu d'avoir les 31 cartes qu'il est alors censé avoir, les descriptions que je lis çà et là suggèrent qu'il n'en a que 30 — décidément, cet éditeur cherche à tuer les mathématiciens obsessifs.) Je pourrais aussi concevoir et faire imprimer mes propres cartes. (Je ne sais pas ce que valent les sites Web qui proposent l'impression de cartes personnalisées, mais je tombe par exemple sur celui-ci, qui proposent des tarifs raisonnables, même s'ils le deviendront certainement moins après frais de port depuis les États-Unis — je ne trouve pas grand-chose basé en France ou en Europe, et le problème c'est que les jeux de cartes personnalisés font référence à la personnalisation des dos, pas des faces.) En revanche, si je fais ça, je passerai sans doute une éternité à me torturer sur la manière la plus logique, symétrique et élégante de choisir les symboles et de les disposer sur les cartes (dans le cas de Dobble, c'est visiblement fait au hasard, y compris pour la forme et l'orientation, ce qui participe justement à la difficulté du jeu).

On pourrait aussi chercher à faire des jeux de cartes avec d'autres structures mathématiques (après tout, un plan projectif, c'est un immeuble de Bruhat-Tits classique sphérique de type A₂ : je peux regarder par exemple le type B₂ [ajout : voir l'entrée suivante], et ainsi fabriquer un jeu de 40 cartes avec 4 symboles parmi 40 sur chacune, telles que deux cartes aient toujours au plus un symbole en commun, et que si un symbole ne figure pas sur une carte donnée, alors il existe exactement une autre carte ayant ce symbole et ayant un symbole en commun avec la carte donnée). Mais bon, avant de trouver un jeu à faire avec une structure plus compliquée, il serait déjà intéressant d'en trouver avec les plans projectifs.

↑Entry #2307 [older| permalink|newer] / ↑Entrée #2307 [précédente| permalien|suivante] ↑

↓Entry #2303 [older| permalink|newer] / ↓Entrée #2303 [précédente| permalien|suivante] ↓

(vendredi)

Je passe plusieurs jours à paramétrer une surface cubique

Un des paradoxes de la manière dont je gère (mal !) mon temps est que quand je n'ai pas de choses importantes et urgentes qui m'occupent de façon pressante, toutes sortes de petites choses moins importantes ou moins urgentes que j'ai laissé de côté pendant d'autres périodes percolent alors à la surface, et j'ai l'impression d'être presque plus débordé. D'autant plus que le temps que prennent ces choses n'est pas forcément évident à évaluer. Ainsi l'exemple d'un calcul que j'ai commencé de façon très accessoire suite à une question d'un collègue, que je pensais pouvoir traiter assez rapidement, et qui m'a finalement obsédé pendant à peu près dix jours, à m'énerver de ne pas arriver à faire ce que je voulais et de croire N fois avoir trouvé le bon bout pour tomber en fait dans un cul-de-sac, au point que j'en ai perdu le sommeil pendant une nuit.

D'autant plus que ce n'était pas tellement le résultat du calcul qui m'intéressait, et dont je suis totalement certain qu'il est connu depuis Klein, Cayley, Clebsch ou, au pire, Segre, et qu'il figure dans quantité de livres ou d'articles, mais d'y arriver moi-même, et de façon systématique, sans essayer de « deviner » le résultat (qui, a posteriori, était éminemment devinable), bref, de vérifier que je savais mener ce calcul à bien. Apparemment, la réponse est : oui, j'y arrive, mais très difficilement (et je ne suis pas certain d'avoir été complètement systématique, au final).

Mais je crois qu'il est important pour un mathématicien, en tout cas pour un géomètre algébriste, d'essayer de faire des calculs parfois. Même, ou plutôt surtout, en utilisant un ordinateur : comme l'a écrit Knuth, Science is knowledge which we understand so well that we can teach it to a computer, et l'intérêt d'essayer d'expliquer quelque chose à un ordinateur est de vérifier qu'on le comprend soi-même bien (à défaut d'ordinateur, un étudiant neuneu peut être utile, ou un post de blog 😉). Donc, vérifier qu'on sait passer d'une incantation magique comme une surface cubique est, géométriquement, l'éclaté du plan projectif en six points en position générale (et ces 6 points, les 15 droites passant par deux d'entre eux, et les 6 coniques par cinq d'entre eux, forment les 6+15+6 = 27 droites de la surface cubique) à une suite de calculs qui donnent le paramétrage d'une surface donnée, c'est vérifier qu'on a compris l'incantation.

Bon, j'avoue, je dis ça pour essayer de me convaincre que mon calcul était difficile, or il ne l'était pas, ou du moins, il n'aurait pas dû l'être vu que j'ai passé trois quatre cinq ans à faire une thèse sur les (hyper)surfaces cubiques et que j'en ai même fait un DVD.

Bref.

Le but, si on veut, est de décrire (paramétrer) toutes les solutions rationnelles de l'équation z₁³ + z₂³ + z₃³ = 1, autrement dit, toutes les façons d'écrire 1 comme somme des cubes de trois rationnels (en fait, ce serait plutôt −1, mais ça n'a pas d'importance, il suffit de changer les signes). Pour donner un peu de contexte sur ces sortes d'équations diophantiennes, il faut que j'explique ce qui se passe pour les problèmes analogues s'agissant de la somme de deux carrés, de trois carrés, et de deux cubes.

[Figure géométrique] Je devrais donc commencer par parler des solutions rationnelles de l'équation z₁² + z₂² = 1 (les points rationnels sur le cercle unité si on considère que z₁ représente l'abscisse et z₂ l'ordonnée) et de leur paramétrage. Les solutions rationnelles de z₁² + z₂² = 1 sont données par z₁ = (1−t²)/(1+t²) et z₂ = 2t/(1+t²) pour t parcourant les rationnels (on obtient exactement toutes les solutions comme ça si on convient en outre que t=∞ donne (z₁,z₂)=(−1,0) ; la réciproque est donnée par t = z₂/(1+z₁) = (1−z₁)/z₂). Ces formules peuvent se relier aux formules donnant le cosinus et le sinus d'un angle θ en fonction de la tangente de l'angle moitié (attention !, je ne prétends pas que l'angle θ lui-même soit rationnel, ni même que sa valeur ait un intérêt quelconque dans le problème). La figure ci-contre (si votre navigateur vous la montre et que vous arrivez à la déchiffrer) est censée illustrer ce paramétrage, figure sur laquelle j'ai pris t=1/3, qui donne la solution z₁=4/5 et z₂=3/5 (on a (4/5)² + (3/5)² = 1, c'est-à-dire que le point (4/5,3/5) est sur le cercle unité, ou, si on préfère chasser les dénominateurs, 4² + 3² = 5²). Ces formules (le « paramétrage rationnel d'une conique par une droite de pente variable par un de ses points ») sont une sorte de pons asinorum de la géométrie arithmétique, et avec un tout petit peu de mauvaise foi on peut les attribuer à Pythagore ou à Euclide (dans la recherche des « triplets pythagoriciens », c'est-à-dire des solutions entières de l'équation Z₁² + Z₂² = Z₀² : le fait que le 4² + 3² = 5², c'est-à-dire que le triangle de côtés entiers 4,3,5 est rectangle, est connu depuis très longtemps, et la recherche de solutions analogues intéressait les mathématiciens dès l'antiquité). Il est donc assez naturel de se demander ce qui se passe si on change un petit peu l'équation.

La même technique que ci-dessus marche mutatis mutandis si on cherche les solutions rationnelles de z₁² + z₂² + z₃² = 1 (les points rationnels sur la sphère unité) ou même pour n'importe quel nombre de variables : on s'inspirera de la projection stéréographique de la sphère pour arriver à quelque chose comme z₁ = (1−v²−w²)/(1+v²+w²) avec z₂ = 2v/(1+v²+w²) et avec z₃ = 2w/(1+v²+w²) pour v et w rationnels (je passe sous silence des petites subtilités notamment sur ce qui arrive « à l'infini »).

Si on remplace les carrés par des cubes, en revanche, les choses sont très différentes : l'équation z₁³ + z₂³ = 1 n'a pas de solution rationnelle autre que les deux évidentes (1,0) et (0,1), cela a été démontré par Euler en 1770 (en montrant le cas particulier n=3 du théorème de Fermat, c'est-à-dire que Z₁³ + Z₂³ = Z₀³ n'a pas de solution entière). • Mais en ajoutant une variable, l'équation z₁³ + z₂³ + z₃³ = 1 a de nouveau quantité de solutions rationnelles, et mon calcul consistait essentiellement à en trouver le paramétrage :

z₁ = (9 − 9v + 3v² − 3v³ − 3w − 6v·w − 3v²·w + 3w² − v·w² − w³)/(9 − 9v + 3v² − 3v³ + 3w + 6v·w + 3v²·w + 3w² − v·w² + w³)

z₂ = (−9 − 9v − 3v² − 3v³ + 3w − 6v·w + 3v²·w − 3w² − v·w² + w³)/(9 − 9v + 3v² − 3v³ + 3w + 6v·w + 3v²·w + 3w² − v·w² + w³)

z₃ = (9 + 9v + 3v² + 3v³ + 3w − 6v·w + 3v²·w + 3w² + v·w² + w³)/(9 − 9v + 3v² − 3v³ + 3w + 6v·w + 3v²·w + 3w² − v·w² + w³)

vérifient z₁³ + z₂³ + z₃³ = 1 quels que soient v,w,

avec pour réciproque (« presque partout »)

v = (−1 + z₁² − z₂ − z₂² − z₁·z₃ + z₃²)/(z₁·z₂ + z₃)

w = (1 − 2z₁ + z₁² + z₂ − z₁·z₂ + z₂² + z₃ − z₁·z₃ + 2z₂·z₃ + z₃²)/(z₁·z₂ + z₃)

(Je vais expliquer qu'on peut écrire ces formules de façon un peu plus jolie !) Par exemple, v=2 et w=3 donnent la solution z₁=−5/4, z₂=−3/4 et z₃=3/2, et on a bien (−5/4)³ + (−3/4)³ + (3/2)³ = 1, ou, si on préfère chasser les dénominateurs, (−5)³ + (−3)³ + 6³ = 4³, ou encore, si on est resté un peu en retard sur les derniers progrès mathématiques et qu'on n'aime pas les nombres négatifs, 6³ = 5³ + 3³ + 4³ (au niveau des entiers naturels, les formules ci-dessus produisent donc plein de cubes égaux à la somme de trois autres cubes, ou, selon les signes, de sommes de deux cubes égaux à une autre telle somme). Remarquer que ces formules, comme celles que j'ai données plus haut pour le paramétrage rationnel du cercle ou de la sphère, permettent non seulement de trouver des solutions rationnelles, mais aussi d'approcher une solution réelle par une solution rationnelle (il suffit d'appliquer la « réciproque » sur les réels, d'approcher les paramètres, et d'appliquer la formule directe). Par exemple, si je veux trois entiers « assez proches » dont la somme des cubes est encore un cube, je pars de la solution réelle où z₁, z₂, z₃ valent 3−1/3 ≈ 0.6933612744, pour laquelle les formules réciproques me donnent v ≈ −1.4422495703 et w ≈ 2.0800838231, qui sont proches de −450/312 et 649/312 respectivement, et en appliquant les formules directes avec ces deux rationnels, on trouve, après avoir chassé les dénominateurs, 1403846621³ + 1403905879³ + 1403840755³ = 2024722855³, et ce n'est pas évident de trouver des choses comme ça autrement qu'en utilisant ce genre de techniques.

(Évidemment, c'est plus impressionnant avec le paramétrage rationnel du cercle : si vous cherchez des triangles rectangles à côtés entiers dont les angles non-droits soient proches de 45°, on remplacera t dans les formules donnant le paramétrage rationnel du cercle par les approximants successifs de √2 − 1, et on obtient ainsi successivement 3²+4²=5², 21²+20²=29², 119²+120²=169², 697²+696²=985², 4059²+4060²=5741², etc., où à chaque fois les deux carrés sommés sont non seulement proches mais même consécutifs — je ne sais pas si cette suite était connue des anciens Grecs.)

↑Entry #2303 [older| permalink|newer] / ↑Entrée #2303 [précédente| permalien|suivante] ↑

↓Entry #2301 [older| permalink|newer] / ↓Entrée #2301 [précédente| permalien|suivante] ↓

(jeudi)

Volumes intrinsèques (quermaß) des convexes

Considérons les questions suivantes, dont l'énoncé ne fait pratiquement appel qu'à des notions de niveau collège (quitte à les reformuler ou spécialiser un tout petit peu : par exemple, j'ai écrit convexe dans la seconde, mais si on veut, on peut considérer des cas particuliers comme un triangle, rectangle ou ellipse, pour simplifier) :

  • Une valise est contenue dans une autre : montrer que la somme des trois dimensions de la valise contenue est inférieure à la somme des trois dimensions de la valise contenante. Ou, plus formellement : si un parallélépipède rectangle (dans l'espace euclidien de dimension 3) est inclus dans un autre (on ne demande pas que les côtés des deux parallélépipèdes soient parallèles), alors la somme des trois côtés du contenu est inférieure [i.e., inférieure ou égale] à celle des trois côtés du contenant.
  • Si deux courbes fermées sans intersection (i.e., des courbes de Jordan) dans le plan délimitent des domaines convexes, l'une étant complètement contenue dans l'autre (i.e., le domaine convexe délimité est inclus), montrer que la longueur de courbe contenue est inférieure à la longueur de la courbe contenante. Idem dans l'espace : si deux convexes de l'espace sont inclus l'un dans l'autre, alors la surface du bord du convexe contenu est inférieure à la surface du bord du convexe contenant. Idem en n'importe quelle dimension.
  • On projette un cube orthogonalement sur un plan choisi aléatoirement (uniformément) : quelle est l'espérance de la surface de la projection ? (I.e., quelle est l'« ombre moyenne » d'un cube ?) On pince un cube entre deux plans parallèles dont la direction est choisie aléatoirement : quel est l'écart entre ces deux plans ? (I.e., quel est la largeur moyenne d'un cube selon une direction tirée au hasard ?)

Ces questions ont ceci en commun que, selon le niveau de réflexion qu'on leur accorde, elles semblent faciles (leur énoncé est tout à fait élémentaire), puis difficiles (on ne sait pas par quel bout les aborder), puis faciles (quand on les prend bien) : elles ont aussi ceci en commun qu'elles sont toutes résolubles grâce à la même notion mathématique, celle de volume intrinsèque d'un convexe (ou intégrale de quermaß, c'est la même chose à une constante et une renumérotation près) : c'est une notion que je trouve très jolie et naturelle, pas du tout compliquée à expliquer, et qui semble bizarrement peu connue même des mathématiciens en-dehors des spécialistes de la convexité ou de la géométrie intégrale/stochastique, alors qu'on peut en tirer des choses très simples (comme l'illustrent les problèmes ci-dessus). Bon, peut-être qu'en fait tout le monde connaît, et que j'étais le dernier à être mis au courant (il y a environ quatre ans, quand j'ai entendu parler de ces choses-là pour la première fois), mais ma réaction a été pourquoi aucun cours de maths que j'ai suivi ne m'a présenté ce concept vraiment naturel et intéressant ?!. Il y a toutes sortes de façon de l'approcher, je vais me contenter de donner les résultats basiques qui me semblent les plus importants.

Très grossièrement, l'idée est qu'à côté du volume (de dimension n) et de la surface (de dimension n−1, où n est la dimension ambiante — en fait, on prendra plutôt la demi-surface pour une raison de cohérence d'ensemble), on peut définir (pour un convexe compact) une sorte de « mesure » en chaque dimension entre 1 et n ; dans le cas d'un parallélotope (pas forcément rectangle, mais imaginons-le rectangle pour fixer les idées), le i-ième volume intrinsèque est égal, à une constante près (1/2i) à la somme des volumes i-dimensionnels (longueur, surface, volume, etc.) de toutes les faces de dimension i du parallélotope.

Voici une façon d'approcher cette notion. Si K est un convexe compact dans l'espace euclidien de dimension n, on peut considérer K+B(ρ) (où B(ρ) désigne la boule fermée centrée en l'origine et de rayon ρ, c'est-à-dire) l'ensemble des points situés à distance ≤ρ de K, autrement dit l'épaississement de K jusqu'à distance ρ, ou simplement la « boule » (mais j'éviterai ce terme) centrée sur K et de rayon ρ. On s'intéresse au volume [i.e., à la mesure de Lebesgue] V(K+B(ρ)) de cet ensemble de points : on peut montrer que c'est un polynôme en ρ (pour ρ≥0), et ce sont les coefficients de ce polynôme qui vont m'intéresser. Il est évident, en considérant séparément les cas ρ=0 et ρ très grand, que le coefficient constant (donc la valeur pour ρ=0) est simplement le volume V(K) de K, et que le terme dominant est le volume V(B(ρ)) de la n-boule de rayon ρ, que je vais noter 𝒱n·ρn avec 𝒱n le volume de la n-boule unité (qui vaut πn/2/(n/2)!, mais ce ne sera pas très important). On peut aussi se convaincre, en considérant le comportement pour ρ très petit mais non nul (disons, la dérivée en ρ=0), que le coefficient de degré 1 est la surface de K (c'est-à-dire la mesure (n−1)-dimensionnelle de son bord).

↑Entry #2301 [older| permalink|newer] / ↑Entrée #2301 [précédente| permalien|suivante] ↑

↓Entry #2299 [older| permalink|newer] / ↓Entrée #2299 [précédente| permalien|suivante] ↓

(lundi)

Notes sur les réseaux euclidiens, et le réseau de Leech

Je mets ici les transparents d'un exposé que j'ai donné vendredi matin dans le cadre d'une journée Télécom-UPS (Le Numérique pour tous) s'adressant aux professeurs de classes préparatoires : le sujet que j'ai évoqué était celui des réseaux euclidiens[#] et de leurs applications en cryptographie. Comme j'ai moi-même appris plein de choses en préparant cet exposé (entre autres en me plongeant un peu plus que je ne l'avais fait jusqu'alors dans le célèbre livre Sphere Packings, Lattices and Groups des deux mathémagiciens John Conway et Neil Sloane), je n'ai pas résisté à partir un peu dans tous les sens, et forcément j'avais beaucoup plus de choses sur mes planches que je ne pouvais en exposer en une heure : inversement, j'espère que leur lecture peut être intéressante sans l'exposé oral pour les accompagner.

Je n'ai notamment pas pu m'empêcher d'évoquer (le réseau) E₈, même s'il n'a aucun rapport avec la crypto dont j'étais censé parler. Ce qui me fait penser que si j'ai beaucoup parlé de E₈ sur ce blog, soit de l'algèbre ou du groupe de Lie de ce nom, soit du système de racines qui le définit, je n'ai pas vraiment parlé du réseau E₈ (celui engendré par le système de racines), qui est pourtant un objet plus simple (dans sa définition sans doute la plus compacte, c'est l'ensemble {(x₁,…,x₈) ∈ (ℤ⁸∪(ℤ+½)⁸) : x₁+⋯+x₈ ∈ 2ℤ} des octuplets de réels soit tous entiers soit tous ½+entiers, et dont la somme est un entier pair) ; et je n'ai jamais parlé du réseau de Leech de dimension 24 (qui est pourtant presque aussi ubiquiste dans les mathématiques que E₈, et peut-être encore plus exceptionnel). Voici une façon concise (mais peu constructive) de caractériser ces deux objets : si vous vivez dans un espace de dimension 8 (resp. 24) et que vous cherchez à empiler des boules toutes identiques, vous remarquerez qu'il y a une unique façon de mettre le nombre maximum de boules autour d'une boule centrale de façon à ce qu'elle la touchent toutes, à savoir 240 d'entre elles (resp. 196560), et de plus, une fois réalisé ce motif, il se continue de façon périodique (chaque boule ayant toujours ce même nombre maximum de voisines) ; en regardant le centre des boules, vous avez ainsi réalisé le réseau E₈ (resp. le réseau de Leech ou son symétrique). À part en dimension 2 où on obtient facilement le réseau hexagonal par la même construction (en disposant six cercles identiques autour d'un septième qu'ils touchent tous), les dimensions 8 et 24 sont exceptionnelles, au moins parmi celles qu'on connaît (j'ignore si on sait dire quelque chose sur les dimensions telles que l'arrangement maximal de boules identiques autour d'une boule centrale soit unique et engendre de plus un réseau, mais il n'y en a pas d'autre que 2,8,24 en dimension ≤24, et pas d'autre connue : dans les autres dimensions, les boules ne sont pas du tout rigides — par exemple, en dimension 3, on peut placer au maximum 12 boules identiques touchant une autre donnée, mais il y a beaucoup de façons de le faire, et elles peuvent se déplacer tout en gardant le contact avec la boule centrale).

Ceci étant, si les questions d'empilement de sphère sont frappantes, elles ne permettent pas vraiment de travailler avec le réseau de Leech. Sur le modèle de la définition que j'ai donnée ci-dessus du réseau E₈ (les octuplets de réels, soit tous entiers soit tous ½+entiers, dont la somme est un entier pair), voici la façon la plus simple et constructive que je connaisse de définir le réseau de Leech. Comme il vit en 24 dimensions, il y a 24 coordonnées à donner, et je disposerai ces 24 coordonnées sur les sommets d'un icosaèdre régulier (rappelons qu'un icosaèdre régulier a 12 sommets), deux par sommet, que j'appellerai arbitrairement la coordonnéee rouge et la coordonnée bleue (pour ce sommet). Le réseau de Leech est formé des points dont les coordonnées multipliées par √8 sont 24-uplet d'entiers vérifiant les conditions suivantes : (0) les bits 0 (=bits de poids faible) de ces 24 entiers sont tous les mêmes (i.e., ils sont soit tous pairs, soit tous impairs), (1) le bit 1 de l'entier rouge sur chaque sommet de l'icosaèdre est égal au XOR des bits 1 des entiers bleus des sommets qui ne sont pas adjacents à lui [la même chose est alors automatiquement vraie en échangeant bleue et rouge, et cette condition est une façon de dire que les bits 1 forment un mot du code de Golay binaire (24,12,8)], et enfin (2) le XOR des bits 2 de tous les entiers est égal à leur bit 0 commun [on a déjà dit que les bits 0 sont tous les mêmes]. (Note : le facteur √8 est un simple facteur de normalisation. Il a pour but d'assurer que le réseau de Leech a un covolume — c'est-à-dire la valeur absolue du déterminant d'une base — égal à 1, et alors les produits scalaires de deux vecteurs quelconques sont toujours entiers.)

±?/√8±?/√8±?/√8±?/√8±?/√8±?/√8
±?/√8±?/√8±?/√8±?/√8±?/√8±?/√8
±?/√8±?/√8±?/√8±?/√8±?/√8±?/√8
±?/√8±?/√8±?/√8±?/√8±?/√8±?/√8

Le tableau ci-contre, si mon JavaScript est bien fait, est censé afficher des vecteurs aléatoires de la plus petite longueur non nulle (à savoir 2) uniformément choisis parmi les 196560 possibles dans le réseau de Leech (qui est engendré par eux, c'est-à-dire, est l'ensemble de toutes les combinaisons entières de ces vecteurs) ; j'ai laissé non simplifiées des expressions comme 2/√8 (ou 4/√8, qui apparaît très rarement) pour mieux coller avec la présentation que je viens de donner. Ici, les coordonnées ont été disposées en tableau 6×4 parce que c'est plus commode à mettre sur une page Web qu'un icosaèdre avec deux coordonnées par sommet : si on veut faire le lien entre ces deux présentations, on peut reprendre l'étiquetage des cases que j'avais utilisée dans une entrée récente, et qui est rappelée en attributs title (i.e., si on passe la souris au-dessus d'une case), et les disposer sur un icosaèdre de la façon suivante : en appelant ♈︎ un premier sommet, les cinq sommets adjacents s'appelleront cycliquement ♑︎♒︎♏︎♓︎♊︎, et les six sommets opposés aux six que je viens de nommer seront ♎︎ et ♋︎♌︎♉︎♍︎♐︎ respectivement (à chaque fois, les deux étiquettes que je donne servent à définir la coordonnée « rouge » et la coordonnée « bleue » au sommet en question de l'icosaèdre).

Mais bon, il y a quantité de manières de décrire ou de construire le réseau de Leech (dans un seul chapitre du livre précédemment mentionné — le chapitre 24, et je soupçonne d'ailleurs que le numéro n'est pas un hasard —, Conway et Sloane donnent d'ailleurs 23 constructions différentes, une pour chacun des types de trous profonds [sic] du réseau). C'est un des signes qu'il s'agit d'un objet mathématique riche et extraordinaire qu'il y ait tellement de façons de le décrire. En voici une autre : on considère d'abord le réseau appelé II25,1 (dans l'espace Minkowskien de dimension 25+1) dont les points sont (exactement comme pour ma description de E₈ ci-dessus) les 26-uplets de réels, soit tous entiers soit tous ½+entiers, dont la somme est un entier pair ; dans ce réseau, on considère le vecteur v = (0,1,2,3,…,24|70), qui, vu que 70² = 0² + 1² + ⋯ + 24², est orthogonal à lui-même pour le produit scalaire Minkowskien ; on considère alors les vecteurs de II25,1 qui sont orthogonaux à v (c'est-à-dire que la somme des 25 premières coordonnées multipliées par 0,1,2,3,…,24 respectivement, est égale à la dernière multipliée par 70), modulo v lui-même : le réseau ainsi formé est isométrique au réseau de Leech. Ou, pour parler en physicien, on se place dans un espace-temps de relativité restreinte avec 25 dimensions d'espace et 1 de temps, on considère un photon qui se déplace à la vitesse (0/70, 1/70, …, 24/70), et le réseau très simple II25,1, vu par ce photon (dans l'espace perpendiculaire à son déplacement) est le réseau de Leech. Le passage entre cette description et la précédente, cependant, n'est pas évident.

[#] La terminologie prête vraiment à confusion, parce que le mot français réseau correspond à la fois à l'anglais network et lattice, et c'est du second qu'il est question. Mais l'anglais n'est pas moins ambigu, puisque lattice correspond à la fois au français réseau et treillis. Il ne reste plus qu'à inventer une quatrième sorte d'objet, qui s'appellerait treillis en français et network en anglais, et on aura un beau graphe bipartite complet K(2,2) dans les traductions.

↑Entry #2299 [older| permalink|newer] / ↑Entrée #2299 [précédente| permalien|suivante] ↑

↓Entry #2294 [older| permalink|newer] / ↓Entrée #2294 [précédente| permalien|suivante] ↓

(jeudi)

Exposé au séminaire Codes sources sur mon labyrinthe hyperbolique

Pour les ~7×10⁹ d'entre vous qui n'ont pas pu assister à mon exposé tout à l'heure au séminaire Codes sources (dont j'ai déjà parlé) consacré à l'explication de mon labyrinthe hyperbolique (toujours le même), les transparents sont ici — ou du moins, les transparents de la première partie de mon exposée, dédiée à l'exposition des idées mathématiques sous-jacentes ; ensuite j'ai commenté le code directement dans un éditeur, donc je ne peux que renvoyer vers les commentaires de celui-ci. Il y a évidemment beaucoup de choses que j'ai dites qui ne sont pas sur les transparents, mais ils donneront au moins une idée de ce dont j'ai parlé.

↑Entry #2294 [older| permalink|newer] / ↑Entrée #2294 [précédente| permalien|suivante] ↑

↓Entry #2293 [older| permalink|newer] / ↓Entrée #2293 [précédente| permalien|suivante] ↓

(samedi)

Déformation continue d'une rotation de 2 tours en rien du tout

Dans l'entrée que j'ai postée hier je mentionnais le groupe Spin(3), revêtement double du groupe SO(3) des rotations de la sphère, c'est-à-dire qu'il distingue une rotation par un tour complet de pas de rotation du tout ; et je mentionnais que le groupe Spin(3), lui, est simplement connexe (on ne peut pas le revêtir à son tour) : tout lacet, i.e., tout chemin qui revient à son point de départ, dans Spin(3), et notamment celui qui fait faire deux tours complets à la sphère, peut être contracté en rien du tout. J'ai essayé d'illustrer ce fait par une vidéo que je viens de mettre sur YouTube :

La sphère en haut à gauche (celle numérotée 0) fait deux tours complets pendant une période (=8 secondes) de la vidéo ; celle en bas à droite (numérotée 27) ne bouge pas. Chacune des sphères intermédiaires effectue un mouvement qui part et arrive à la même position de référence, et chacun de ces mouvements est très proche des mouvements de la sphère précédente et suivante. Ceci illustre le fait qu'on peut passer continûment de deux tours complets à zéro. Chose qui ne serait pas possible pour un seul tour (ou si on avait affaire à un cercle, quel que soit le nombre non-nul de tours).

Ceci étant, je n'y vois toujours pas grand-chose à la manière dont cette déformation se fait ou pourquoi elle n'est pas possible pour un seul tour (mon espoir était d'acquérir une intuition visuelle sur le groupe spin, pour le comprendre autrement que juste intellectuellement, et ce n'est pas franchement un succès). J'ai aussi produit une version séquentielle de la vidéo, où la sphère fait des mouvements successifs au lieu qu'on les voie tous simultanément, je ne sais pas si c'est plus clair :

OK, je vois bien que l'idée très grossière est que l'axe qui sert d'axe de rotation dans le premier mouvement (suivre des yeux le point de rencontre des trois pentagones verts) se met, au cours des différents mouvements, à faire des tours, si bien que la sphère n'a plus vraiment besoin de tourner autour de lui, puis ce tour qu'il décrit est lui-même recontracté à rien du tout, mais cette description est vraiment vague, et ne me fournit pas une explication visuelle intuitive de pourquoi on a besoin de faire deux tours pour contracter.

↑Entry #2293 [older| permalink|newer] / ↑Entrée #2293 [précédente| permalien|suivante] ↑

↓Entry #2292 [older| permalink|newer] / ↓Entrée #2292 [précédente| permalien|suivante] ↓

(vendredi)

Racontons des choses autour de la notion de groupe de Lie

Puisque j'ai publié une première entrée sur les octonions, je me dis qu'il faudrait que je fasse un peu de vulgarisation sur la notion de groupe de Lie et sur leur classification — et pourquoi c'est un résultat mathématique majeur. Voici une tentative pour raconter quelques choses dans cette direction.

Comme d'habitude quand je fais de la vulgarisation mathématique, (1) je ne sais pas bien à quel niveau de public je m'adresse (et ce niveau va d'ailleurs varier de façon incohérente au cours du texte, même pas forcément de façon monotone vu qu'il m'arrive de faire des digressions pour revenir ensuite à des choses plus basiques), et (2) je vais chercher à « raconter » les maths plus qu'énoncer des définitions et des résultats précis (j'essaie très fort de ne rien dire de faux, mais je dois souvent me réfugier dans un certain niveau de flou quand je veux cacher quelques détails techniques) : mon but est de donner un petit aperçu de ce à quoi ressemble cette théorie classique, certainement pas de l'enseigner précisément (pour ça, il y a toutes sortes de livres, d'ailleurs j'en suggère quelques uns). L'idée est que — qu'on me corrige si ce que je pense est en fait assez stupide — ça peut intéresser des gens de lire des choses à ce sujet, et de regarder les petits dessins que sont les diagrammes de Dynkin et de Satake, sans avoir envie d'apprendre (et/ou le temps de comprendre) ce qu'est précisément, par exemple, un système de racines, une involution de Cartan, ou en fait, un groupe de Lie.

Après, je peux aussi en profiter pour parler à un public plus averti pour lui dire, par exemple regardez le groupe SO*(2n) comme il est tout gentil et tout mimine, pourquoi est-ce que personne n'en parle jamais, de ce pauvre petit groupe ?, ou pour partager mon agacement qu'il soit si difficile de trouver des informations fiables et précises sur certaines choses (celui qui veut traverser le pont de la mort doit répondre aux questions suivantes : quel est le sous-groupe compact maximal de la forme déployée algébriquement simplement connexe de E₇ ? combien sa forme déployée adjointe algébriquement connexe a-t-elle de composantes réelles ? quelle est sa couleur préférée ?).

Table des matières

La notion de groupe et de groupe de Lie

Symétries discrètes

Pour commencer, si je devais m'adresser à un public qui n'a aucune connaissances mathématiques particulières, je présenterais un groupe comme les formes de symétries que peut posséder un objet mathématique (en étant délibérément vague sur ce que objet mathématique peut recouvrir, et en recouvrant sous le terme symétrie tout ce qui « ne change pas » cet objet, cf. les exemples et commentaires ci-dessous). Cette définition est assez floue, mais elle a le mérite de permettre de comprendre pourquoi il s'agit d'un concept extrêmement central en mathématiques (alors que si on prend la vraie définition comme un ensemble muni d'une loi de composition binaire vérifiant les axiomes gnagnagna, ça ne saute pas forcément aux yeux pourquoi cette définition est la bonne et pourquoi le concept est essentiel).

Par exemple, si je considère un pentagone régulier (ou de façon équivalente, une étoile à cinq branches comme ceci), cette figure a dix symétries : quatre rotations autour du centre du pentagone (de façon à amener un sommet sur un des quatre autres, ce qui donne des angles de ±72° ou ±144° mais peu importe), cinq symétries axiales (les réflexions par rapport à des axes passant par un des cinq sommets du pentagone), et la « symétrie » consistant à ne rien faire, qu'on appelle symétrie identité, ou élément neutre du groupe, et que les mathématiciens incluent toujours parce que cela rend la notion de groupe bien plus commode. L'ensemble de ces dix symétries s'appelle le groupe diédral du pentagone (et on dit qu'il est d'ordre 10, parce qu'il y a dix éléments dedans). Soit dit en passant, si on considère une étoile à cinq branches entrelacée (c'est-à-dire où on voit dans quel sens une branche passe au-dessus d'une autre, comme sur cette version du drapeau marocain), la figure n'a plus que cinq symétries (les cinq rotations de ±72° et ±144°, ou plus exactement, les quatre rotations et l'élément neutre / identité qui est une rotation de 0°), parce qu'une symétrie axiale changerait le sens d'entrelacement de l'étoile : ce groupe s'appelle alors le groupe cyclique à cinq éléments (et c'est un exemple d'un sous-groupe, en l'occurrence un sous-groupe du groupe diédral du pentagone : en ajoutant une structure à un objet mathématique, on restreint ses symétries). Remarquons que la plupart des figures géométriques (prenez un triangle quelconque, par exemple) n'ont pas du tout de symétrie, ou plutôt, ils n'ont que la symétrie idiote consistant à ne rien faire (l'identité ou élément neutre, comme je l'ai appelée ci-dessus), et leur groupe de symétrie est appelé le groupe trivial, ou groupe à un seul élément.

↑Entry #2292 [older| permalink|newer] / ↑Entrée #2292 [précédente| permalien|suivante] ↑

↓Entry #2276 [older| permalink|newer] / ↓Entrée #2276 [précédente| permalien|suivante] ↓

(mardi)

Les octonions sont-ils intéressants ? (première partie)

J'ai promis depuis une éternité de parler d'octonions, et cette entrée a été commencée à ce moment-là, puis laissée de côté, puis remaniée complètement suite à une réflexion que j'ai entreprise sur la notion de géométrie, puis laissée de nouveau de côté, puis reprise, etc. Le résultat, écrit par bribes, manque donc certainement de cohérence globale, j'espère qu'on ne m'en voudra pas. Je reprends la formulation du titre d'une entrée passée pour m'interroger de nouveau sur l'intérêt d'un concept mathématique parmi ceux qui fascinent beaucoup, notamment les mathématiciens amateurs, et ceux qui aiment se demander voyons jusqu'où on peut généraliser les choses : en l'occurrence, les octonions, dont je vais tâcher d'expliquer de quoi il s'agit. Mais, quitte à spoiler la suite, je peux d'ores et déjà révéler que ma conclusion générale sera plus positive que pour les nombres surréels : je prétends que les octonions sont un objet naturel, même si les raisons de leur existence ont quelque chose d'un peu étonnant et mystérieux ; en revanche, les tentatives pour les généraliser encore sont idiotes parce qu'elles passent complètement à côté de la raison profonde pour laquelle les octonions sont intéressants (en se concentrant sur des phénomènes superficiels).

Introduction

Dans cette première partie d'une série d'entrées consacrées aux octonions (mais qui, comme tout ce que j'entreprends, présente un risque sérieux de ne jamais être finie), je n'arriverai pas encore à répondre à la question du titre, puisque je ne ferai essentiellement que définir et présenter les objets en question. Après une présentation et un petit historique censés être lisibles par absolument tout le monde, je veux commencer par rappeler ce que sont les nombres complexes et les quaternions, pour ensuite aborder les octonions. J'expliquerai pourquoi les quaternions sont intéressants et utiles notamment pour calculer avec les rotations dans l'espace, et j'essaierai de présenter ensuite de façon analogue des liens des octonions avec les rotations en sept ou huit dimensions. Je parlerai ensuite un peu des automorphismes des octonions, qui constituent le groupe de Lie exceptionnel G2 (il faudra donc dire un peu ce que cela signifie), et j'évoquerai enfin quelques pistes pour la suite.

Je prévois de continuer avec encore deux entrées sur le sujet : l'une (déjà essentiellement écrite) contiendra un microscopique aperçu du sujet des octonions entiers et notamment leur lien avec mon E8 préféré, et une autre (largement à écrire ou à réécrire, donc probablement pour jamais) doit expliquer ce qu'est le carré magique de Freudenthal-Tits, qui permet vraiment de répondre (positivement !) à la question du titre — oui, les octonions sont intéressants à cause de leur lien profond avec les groupes de Lie exceptionnels G2, F4, E6, E7 et (de nouveau !) E8.

Table des matières

Présentation sans mathématiques, et petit historique

Disons immédiatement la chose suivante : les octonions (𝕆) sont une sorte de « nombres » qui s'inscrit logiquement après les nombres réels ℝ, les nombres complexes ℂ et les quaternions ℍ. Les nombres complexes sont un objet de dimension réelle 2, c'est-à-dire qu'un nombre complexe renferme essentiellement la donnée de deux nombres réels (sa partie réelle et sa partie imaginaire) ; les quaternions sont de dimension réelle 4, c'est-à-dire qu'ils ont quatre coordonnées réelles, et les octonions sont de dimension réelle 8. Ceci donne naturellement envie de prolonger la suite des puissances de 2 et d'inventer des sortes de nombres qui soient de dimension réelle 16, 32 et ainsi de suite, mais le caractère véritablement exceptionnel des octonions offre toutes sortes de raisons de comprendre, au contraire, qu'elle doit s'arrêter (et que c'est justement le fait qu'elle s'arrête qui rend les octonions intéressants !), c'est-à-dire que tout objet qu'on peut inventer pour la prolonger est soit entièrement dénué d'intérêt soit complètement délirant.

Il m'est impossible de faire l'historique des nombres réels puisque la progression historique, à ce sujet, est trop éloignée de la progression mathématique : la géométrie grecque utilise implicitement une notion de mesure, mais la mesure d'une longueur ou d'une aire ne sont pas véritablement unifiées et le concept de nombre négatif n'existe pas ; a contrario, il serait absurde de dater les nombres réels de leur première construction véritablement rigoureuse (peut-être par Cauchy ou Dedekind) car ce serait suggérer qu'Euler, Lagrange ou Gauß ne comprenaient pas ce concept, ce qui est manifestement faux parce que les questions algébriques qui m'intéressent ici sont assez peu liées aux questions (quasi fondationnelles) sur la complétude des nombres réels. Je passe donc sur les nombres réels.

Les nombres complexes ont commencé à apparaître avec la résolution des équations du troisième degré notamment par Jérôme Cardan (vers 1545) : la raison en est que même si une équation réelle du troisième degré a toujours une solution réelle, il peut être nécessaire d'introduire des racines carrées de nombres négatifs, c'est-à-dire de passer par les nombres complexes, pour exprimer ce qui sera finalement une quantité réelle (on sait maintenant, grâce à la théorie de Galois, que le cas où les trois racines d'une équation cubique réelle sont toutes réelles, le fameux casus irreducibilis, lié au problème de la trissection de l'angle, ne peut se résoudre en radicaux que si on accepte des radicaux non réels). Mais même si Cardan fait intervenir, presque malgré lui, des nombres complexes, c'est Bombelli qui en développe une première théorie un peu sérieuse dans son livre d'algèbre publié en 1572. Curieusement, ce n'est que tardivement, peut-être avec Argand en 1806, et avec la recherche de démonstrations du théorème fondamental de l'algèbre (une équation algébrique de degré n dans les nombres complexes a toujours n solutions comptées avec multiplicités), qu'on a acquis la représentation claire des nombres complexes comme les points d'un plan (donc de dimension 2 sur les nombres réels) dont la partie réelle et la partie imaginaire seraient les deux coordonnées.

Les nombres complexes ayant ainsi deux coordonnées réelles, et étant liés de façon agréable à la géométrie plane, il est naturel de chercher si on peut construire des sortes de nombres avec trois coordonnées, qu'on pourrait lier à la géométrie dans l'espace. William Hamilton a passé des années de sa vie, vers 1830–1840, à chercher de tels nombres (sans avoir, bien sûr, une définition exacte de ce qu'il cherchait). C'est en 1843 qu'il a découvert les quaternions, de dimension 4 réelle, en même temps qu'il a compris la raison pour laquelle la dimension 3 ne pouvait pas répondre à ses attentes, à savoir l'inexistence d'une « identité des trois carrés » analogue à l'« identité des deux carrés » ((a²+b²) · (a′²+b′²) = (a·a′−b·b′)² + (a·b′+b·a′)²) qui exprime la multiplicativité de la norme complexe et celle « des quatre carrés » ((a²+b²+c²+d²) · (a′²+b′²+c′²+d′²) = (a·a′−b·b′−c·c′−d·d′)² + (a·b′+b·a′+c·d′−d·c′)² + (a·c′−b·d′+c·a′+d·b′)² + (a·d′+b·c′−c·b′+d·a′)²) liée à l'existence des quaternions mais qui était déjà connue d'Euler et de Lagrange.

Malgré le fait qu'ils soient de dimension 4, les quaternions ont, comme je l'expliquerai, des applications naturelles à la géométrie euclidienne de dimension 3 (pour le calcul des rotations dans l'espace). C'est sans doute la raison pour laquelle ils ont eu un certain succès, et ont valu une grande renommée à leur inventeur. (En fait, comme souvent en mathématiques, les découvertes avaient été préfigurées par d'autres : en l'occurrence, Gauß avait essentiellement découvert les quaternions dans un texte de 1819 sur les rotations de la sphère, qu'il n'a pas jugé bon de publier.) Toujours est-il que dans la deuxième moitié du XIXe siècle ont fleuri des textes, des chaires et des cours sur la « science des quaternions ». (Une anecdote que je n'ai pas réussi à confirmer veut que quand Charles Dodgson, plus connu sous le pseudonyme de Lewis Carroll, a publié Alice in Wonderland, la reine Victoria lui a fait promettre de lui envoyer une copie du prochain livre qu'il écrirait : le livre en question était un traité sur les quaternions, et l'histoire ne dit pas si Victoria l'a autant apprécié.) Les quaternions continuent d'avoir une certaine utilité pour représenter informatiquement des orientations dans l'espace (de façon compacte et efficace).

Les octonions, en revanche, n'ont pas eu une telle popularité, et n'ont guère d'utilité pratique. Découverts (sous le nom d'octaves), à peine quelques mois après les quaternions, par un ami de Hamilton, John Graves, celui-ci s'est fait voler la vedette par Arthur Cayley qui a publié l'existence des octonions en 1845.

Il existe une façon systématique (la construction de Cayley-Dickson) pour passer des nombres réels aux complexes, des complexes aux quaternions, et des quaternions aux octonions : mais à chaque fois qu'on applique cette construction, on perd quelque chose. Quand on passe des réels aux complexes, on perd la propriété d'être un corps ordonné (ou ordonnable) ; quand on passe des complexes aux quaternions, on perd la commutativité de la multiplication, c'est-à-dire que x·y et y·x ne seront plus égaux en général dans les quaternions ; quand on passe des quaternions aux octonions, on perd l'associativité de la multiplication, c'est-à-dire que x·(y·z) et (x·yz ne seront plus égaux en général dans les octonions (ce qui doit faire frémir d'horreur tout mathématicien qui se respecte, mais heureusement on garde au moins une forme faible de l'associativité appelée alternativité) ; et si on cherche à continuer la construction, on perd la seule raison pour laquelle les choses avaient encore un intérêt, à savoir la multiplicativité des normes ou le fait que x·y=0 ne se produit que pour x=0 ou y=0. Même avec ces propriétés, il n'est pas du tout évident que les octonions aient le moindre intérêt autrement que comme une petite curiosité algébrique : il se trouve qu'ils en ont, mais il me semble que la seule explication convaincante de ce fait passe par la théorie des groupes de Lie exceptionnels, et je reporterai à plus tard ces explications.

Quelques lectures : Une excellente référence (souvent citée) concernant les octonions en général est l'introduction de John Baez à leur sujet [edit : lien cassé (en ce moment ?), mais le même texte est disponible sur l'arXiv] ; une autre est le livre de J. H. Conway et D. Smith, On Quaternions and Octonions (their Geometry, Arithmetic and Symmetry). Beaucoup de ce que je vais dire est contenu dans ces sources, mais je vais essayer de dire certaines choses de façon plus élémentaire, ou au moins d'arriver plus rapidement à ce qui est amusant. Une autre référence est les chapitre 9 et 10 par Koecher et Remmert dans le livre Numbers de Ebbinghaus &al. Pour une présentation élégante de la multiplication sur les octonions sans passer par la construction de Cayley-Dickson, je conseille cet article de Bruno Sévennec. Enfin, pour une description claire et approfondie du « carré magique » de Freudenthal (dont je devrai parler plus tard), je recommande ce survey par Barton et Sudbery, qui est le seul que j'aie trouvé vraiment satisfaisant sur le sujet (on pourra aussi consulter cet article de Freudenthal lui-même, en allemand, qui reprend les choses à zéro, de façon assez claire et efficace). Je tire la plupart des informations de mon aperçu historique du livre Mathematics and its History de John Stillwell (notamment les chapitres 14 et 20).

Définition rapide pour les gens pressés

Pour les lecteurs qui n'auraient pas la patience de lire tout ce qui suit, voici une définition ultra-rapide des algèbres à divisions des complexes, quaternions et octonions (on peut aussi l'ignorer sachant que tout va être redit ci-dessous). Il s'agit respectivement des expressions de la forme x(0) + x(1)·i pour les complexes, x(0) + x(1)·i + x(2)·j + x(3)·k pour les quaternions et x(0) + x(1)·i + x(2)·j + x(3)·k + x(4)· + x(5)·i· + x(6)·j· + x(7)·k· pour les octonions (il faudrait traiter i·, j· et k· comme trois lettres supplémentaires, même si je les ai écrites comme des produits pour économiser les lettres de l'alphabet) ; l'addition se fait terme à terme, et la multiplication se fait en développant complètement l'écriture et en utilisant la table qui suit :

×1ijki·j·k·
11ijki·j·k·
ii−1kji·k·j·
jjk−1ij·k·i·
kkji−1k·j·i·
i·j·k·−1ijk
i·i·k·j·i−1kj
j·j·k·i·jk−1i
k·k·j·i·kji−1

(La ligne de la table donne le symbole de gauche à multiplier et la colonne donne le symbole de droite : ainsi, i·j=k tandis que j·i=−k. Pour les complexes, seules les deux premières lignes et colonnes servent, et pour les quaternions, seules les quatre premières lignes et colonnes. Il y a toutes sortes de conventions différentes pour nommer la base des octonions, mais celle que j'ai choisie a l'avantage que — je pense — tous les mathématiciens seront d'accord sur le contenu de la table de multiplication une fois qu'on a choisi les noms.)

La multiplication des complexes est commutative et associative, celle des quaternions est associative mais non commutative, et celle des octonions n'est même pas associative ((i·j=k· tandis que i·(j·)=−k·) ; elle vérifie cependant des conditions plus faibles, dites d'alternativité, à savoir que x·(x·y)=(x·xy, x·(y·x)=(x·yx et y·(x·x)=(y·xx (ce qui revient à dire que l'associateur {x,y,z} := (x·yzx·(y·z) est complètement antisymétrique en ses trois variables).

Si on préfère, on peut aussi définir les octonions à l'aide des formules suivantes (où q,q′,r,r′ désignent des quaternions) : (1) q·(r′·) = (r′·q, (2) (r·q′ = (r·q* et (3) (r·)·(r′·) = −r*·r, où ici, x* désigne le quaternion conjugué de x (cf. ci-dessous). Les mêmes formules en mettant j à la place de peuvent servir à définir les quaternions à partir des complexes, et avec i à définir les complexes à partir des réels. (On parle du procédé de Cayley-Dickson. Pour aider à retenir ces formules, on peut notamment retenir le fait que si w est un quaternion de module 1 quelconque, alors l'application ℝ-linéaire qui fixe les quaternions et envoie un octonions de la forme q′· sur (w·q′)·, constitue un automorphisme des octonions : ceci contraint énormément les formules.)

On peut aussi retenir que i, j, k s'associent et vérifient i² = j² = k² = i·j·k = −1, que la même chose vaut aussi pour n'importe lequel des trois avec (par exemple, i² = ² = (i·)² = −1), et enfin que si on prend deux distincts de i, j, k, avec , alors cette fois ils s'anti-associent toujours, par exemple i·(j·) = −(i·j = −k·. Ceci suffit à reconstruire la table.

Le conjugué d'un complexe, quaternion ou octonion, s'obtient en changeant le signe de toutes les composantes x(p) sauf la partie réelle x(0) (i.e., les conjugués de 1,i,j,k,,i·,j·,k· valent respectivement 1,−i,−j,−k,−,−i·,−j·,−k·). On a (x·y)* = y*·x*, et par ailleurs N(x) := x·x* est la somme des carrés des composantes de x, donc c'est un nombre réel, qui ne peut être nul que si x l'est. En mettant ces deux propriétés ensemble, on voit que tout complexe, quaternion ou octonion x non nul a un inverse de même type, donné par x*/N(x). (Il est utile de savoir que, dans les octonions, le parenthésage n'a pas d'importance dans tout produit faisant intervenir uniquement deux octonions, x, y, ainsi qu'éventuellement leurs conjugués x* et y*, et bien sûr les nombres réels, ce qui permet de conclure que x·y multiplié à gauche par l'inverse de x ou à droite par l'inverse de y donne bien ce qu'on espérait.) On définit par ailleurs |x| = √N(x) (le module, ou la valeur absolue, de x), et aussi Re(x) = ½(x+x*) = x(0) (la partie réelle de x) : cette dernière vérifie notamment Re(x·y)=Re(y·x) et aussi Re(x·(y·z))=Re((x·yz).

↑Entry #2276 [older| permalink|newer] / ↑Entrée #2276 [précédente| permalien|suivante] ↑

↓Entry #2256 [older| permalink|newer] / ↓Entrée #2256 [précédente| permalien|suivante] ↓

(dimanche)

Un nouveau jeu de labyrinthe(?) hyperbolique

Je viens de produire un nouveau jeu de labyrinthe hyperbolique. Je n'étais pas vraiment satisfait du précédent (introduit ici) parce que je trouvais qu'il y a quelque chose d'insatisfaisant à plaquer un labyrinthe au sens traditionnel (i.e., des murs infranchissables) sur l'espace hyperbolique : l'espace hyperbolique est labyrinthique en lui-même (au sens où, par exemple, si on se trompe de direction quelque part, on doit essentiellement revenir à son point de départ pour aller à l'endroit où on voulait aller), je trouvais qu'il faudrait exploiter ce fait — et c'est ce que j'ai tenté de faire dans cette nouvelle version.

[Navigation : pour plus d'explications sur la géométrie hyperbolique, voir cette série de trois entrées passées : 1, 2, 3 (plus ou moins indépendantes) ; voir aussi celle-ci pour des illustrations de différentes projections ; voir aussi les jeux de labyrinthe hyperbolique que j'introduis ici et et , et celui-ci dont j'explique le fonctionnement dans des transparents disponibles ici.]

Le monde, « périodisé » du plan hyperbolique, est exactement le même que dans la version précédente (88110 carrés formant une surface de genre 8812, et pavé par des carrés selon mon pavage préféré), de même type que le monde « jouet » dont je bassine régulièrement mes lecteurs depuis quelques jours, si ce n'est que ce dernier n'a que 30 carrés formant une surface de genre 4, ce qui le rend plus facile à analyser. J'ai repris le monde à 88110 carrés (et qui est un déguisement du graphe de Cayley du groupe PSL(2,89)) parce qu'il est facile à construire, et d'une taille suffisamment raisonnable.

Cette fois, donc, il n'y a aucun obstacle : juste 24 orbes de couleur cachés (quoique placés de façon régulière) dans ce monde, et qu'il s'agit de collecter, mais c'est surtout un prétexte pour explorer ce à quoi ce monde peut ressembler. Pour aider à l'exploration, chaque orbe fait apparaître un domaine de couleur proche autour de lui, tous connexes et approximativement de même taille (c'est-à-dire dans les 3700 cases). J'ai donné des noms aux orbes pour décorer et surtout pour éviter qu'on s'arrache les cheveux à savoir quand deux couleurs sont identiques.

Le monde n'est pas très grand en diamètre : on peut aller de n'importe quelle carré à n'importe quel autre en au plus 17 mouvements (consistant à passer à une case adjacente). Ce qui n'empêche que ces 17 mouvements, dans un pavage hyperbolique, permettent d'aller à beaucoup plus d'autres cases que ce que ce serait dans un pavage euclidien. On retombe donc assez difficilement sur ses pas (sauf évidemment à suivre une boucle — par exemple en allant tout droit selon un des axes du quadrillage on boucle en 11 mouvements).

Globalement, ce n'est pas très difficile une fois qu'on a un peu compris comment fonctionnent les choses.

Pour aider à savoir par où on est passé, j'ai mis une fonction « petit poucet » qui est amusante en elle-même.

Bref, dans l'ensemble je trouve que c'est plus réussi que le jeu de labyrinthe précédent. Mais j'aimerais surtout trouver comment motiver des gens plus doués que moi pour écrire des jeux informatiques à explorer plus les possibilités intéressantes offertes par la géométrie hyperbolique.

Petit changement () : Je garantis maintenant l'existence d'au moins un orbe à distance de vue du point de départ (mais ça peut être délicat de le repérer).

Amélioration () : J'ai ajouté un système de balises qu'on peut déposer dans le labyrinthe (et rappeler à tout moment) et qui indiquent la direction dans laquelle elles se trouvent (ou du moins une direction, puisqu'il y a souvent plusieurs chemins menant d'un point à un autre selon la façon dont on tourne dans le monde).

↑Entry #2256 [older| permalink|newer] / ↑Entrée #2256 [précédente| permalien|suivante] ↑

↓Entry #2254 [older| permalink|newer] / ↓Entrée #2254 [précédente| permalien|suivante] ↓

(mercredi)

Je m'amuse avec les groupes de Coxeter et je continue à disséquer la surface de Bring

Puisque j'étais parti pour manipuler des polygones hyperboliques, j'ai glissé, suivant le fil conducteur de simplement chercher à apprendre des maths belles et amusantes, vers la combinatoire des groupes de Coxeter (et des dessins qui auraient plu à Escher).

[Un pavage hyperbolique étiqueté][Un pavage hyperbolique étiqueté]Sans me proposer d'expliquer la situation en général, je peux facilement en parler sur le cas particulier (mais représentatif) illustré par les images ci-contre à gauche et à droite (peu importent pour l'instant les différences, qui ne sautent d'ailleurs probablement pas aux yeux). Si on ignore les étiquettes, il s'agit d'un pavage du plan hyperbolique par des triangles tous identiques, caractérisés de façon unique par le fait qu'ils ont aux sommets les angles π/4, π/2 et π/5 (lus dans le sens des aiguilles d'une montre pour les triangles blancs, et dans le sens trigonométrique pour les triangles noirs). De façon équivalente, on obtient cette figure en partant de mon pavage préféré de l'espace hyperbolique par des « carrés » dont cinq se rejoignent en chaque sommet, et en divisant chaque carré en huit selon ses quatre axes de symétrie (deux diagonales et deux médianes). On peut donc regrouper les triangles huit par huit pour retrouver le pavage hyperbolique par des « carrés » d'angle 2π/5 en chaque sommet (chercher les bords teintés en gris sur ma figure), et c'est bien sûr cette parenté qui me fait utiliser cet exemple particulier ; on peut aussi, au contraire, regrouper les triangles dix par dix (chercher les bords teintés en rouge sur ma figure) pour obtenir le pavage dual par des pentagones à angles droits. Le pavage triangulaire s'obtient en partant d'un triangle quelconque le constituant, et en effectuant de façon répétée des symétries par rapport à ses trois côtés (je les ai, à chaque fois, légèrement teintés en gris, vert et rouge).

L'ensemble des transformations en question, c'est-à-dire l'ensemble des compositions de symétries par rapport aux côtés des triangles, est appelé le groupe de Coxeter Δ(2,4,5), ou groupe de Coxeter engendré par trois réflexions x, y, z vérifiant x²=y²=z²=1 avec (x·y)⁵=1, (y·z)⁴=1 et (x·z)²=1 (l'opération · étant la composition des transformations). De plus, donnés deux triangles, il existe une et une seule transformation dans le groupe de Coxeter qui transforme l'un en l'autre. Une fois fixé un triangle de référence (disons, celui étiqueté ε sur mes figures), tous les triangles peuvent s'identifier aux éléments du groupe de Coxeter (via la transformation qui envoie le triangle de référence dans le triangle considéré) : on peut donc associer à toute suite de x, y et z un triangle, qui est celui obtenu en partant du triangle de référence (ε) et en effectuant les transformations indiquées par ces lettres. Concrètement, soit on lit le mot de droite à gauche, auquel cas x, y et z désignent les symétries par rapport aux trois côtés fixés du triangle de référence, soit on le lit de gauche à droite, auquel cas x, y et z désignent les côtés qu'on doit traverser, z étant le petit côté de l'angle droit (teinté en rouge sur ma figure), y l'hypoténuse (teintée en vert), et x le grand côté de l'angle droit (teinté en gris).

Ceci fournit donc (une fois fixé le triangle de référence) une façon de désigner n'importe quel triangle du pavage par une suite de x, y et z (les triangles blancs, dont l'orientation est la même que le triangle de référence, sont ceux ayant un nombre pair de lettres, correspondant à une transformation qui préserve l'orientation, tandis que les noirs, dont l'orientation est opposée, sont ceux ayant un nombre impair de lettres). Mais il existe plusieurs suites pouvant désigner le même triangle : pour commencer, comme x² (c'est-à-dire x·x) est l'identité, on peut supprimer ou insérer un nombre pair quelconque de x consécutifs dans un mot, et de même pour les y et les z : mais ce ne sont pas là les seules simplifications possibles, puisqu'on a aussi (xz)²=1, c'est-à-dire xzxz=1, ce qui se traduit plus concrètement par zx=xz (cette exemple prouve qu'il n'y a pas unicité de l'écriture, même si on impose à celle-ci d'être de longueur minimale). On appelle mot réduit sur x, y et z une écriture de longueur minimale conduisant à un élément/triangle donné ; et même parmi les mots réduits, on peut par exemple s'intéresser à celui qui est lexicographiquement le plus petit (ce qui conduit à préférer l'écriture xz à zx). Ma figure de gauche ci-dessus montre chaque triangle étiqueté par le mot réduit lexicographiquement le plus petit : ceci fournit bien une étiquette unique pour chaque triangle. D'autres variations sont possibles : le mot réduit lexicographiquement le plus grand, le mot réduit lexicographiquement le plus petit lu à l'envers (remarquons que lire un mot à l'envers revient à prendre son inverse dans le groupe de Coxeter), ou le mot réduit lexicographiquement le plus grand lu à l'envers. (La figure de droite ci-dessus montre les mots réduits lexicographiquement les plus grands lus à l'envers : si la différence avec la figure de gauche ne vous frappe pas, cherchez le mot xyxyx d'un côté, qui est yxyxy de l'autre.)

↑Entry #2254 [older| permalink|newer] / ↑Entrée #2254 [précédente| permalien|suivante] ↑

↓Entry #2251 [older| permalink|newer] / ↓Entrée #2251 [précédente| permalien|suivante] ↓

(samedi)

Où je résous une équation

L'équation[#] a₁·b₁·a−1·b−1 · a₂·b₂·a−1·b−1 · a₃·b₃·a−1·b−1 · a₄·b₄·a−1·b−1 conjugué à u₁·v₂·u−1·v−1 · u₂·v₃·u−1·v−1 · u₃·v₄·u−1·v−1 · u₄·v₁·u−1·v−1 dans le groupe libre a (entre autres) comme solution :

  • a₁ = v₁·u₄·v−1
  • b₁ = v₂·u
  • a₂ = v₂·u₁·v−1
  • b₂ = v₃·u
  • a₃ = v₃·u₂·v−1
  • b₃ = v₄·u
  • a₄ = v₄·u₃·v−1
  • b₄ = v₁·u

ou réciproquement

  • u₁ = b−1·a₁·b
  • v₁ = a−1·b
  • u₂ = b−1·a₂·b
  • v₂ = a−1·b
  • u₃ = b−1·a₃·b
  • v₃ = a−1·b
  • u₄ = b−1·a₄·b
  • v₄ = a−1·b

(Et la conjugaison se fait par v₁·u₄.)

[#] Les inconnues sont a₁,b₂,a₃,b₄,a₁,b₂,a₃,b₄ tandis que u₁,u₂,u₃,u₄,v₁,v₂,v₃,v₄ sont les générateurs du groupe libre — mais ça ne change rien si on fait le contraire : c'est pour ça que je donne à la fois une solution et une réciproque.

Cela pouvait effectivement peut-être se trouver de tête en regardant assez longuement les équations et en ayant foi dans le fait (douteux) qu'une équation aussi symétrique devait pouvoir admettre une solution symétrique. En l'occurrence, j'ai trouvé ces valeurs en appliquant l'algorithme de Whitehead déguisé sous forme d'un problème combinatoire, et finalement en appliquant un Dijkstra sur le graphe des 127072 façons de tracer 8 cordes disjointes entre 16 points cycliquement ordonnées. Je n'ai pas du tout d'idée claire sur la question de savoir si cette solution est vaguement unique[#2] (et si oui, en quel sens).

[#2] Enfin, je sais qu'elle n'est pas unique, puisque la première version que j'ai trouvée (en minimisant le nombre de chiasmes plutôt qu'une certaine forme de longueur) était beaucoup plus désagréable : a₁=v−1·u−1·v₁·u₄·u₁·u₃·v₄, b₁=v−1·u−1·v₂·u−1·v−1·u₃·v₄, a₂=v−1·u−1·v₂·u₁·u₂, b₂=v₃·u−1·v−1·u₃·v₄, a₃=v−1·u−1·u−1·v−1·u₃·v₄, b₃=v−1·u−1·v₁·u₄·u₃·v₄·v−1·u₃·v₄, a₄=v−1·u−1·v₁·u₄·u₃·v₄·u−1·v−1·u−1·u−1·v−1·u₃·v₄, b₄=v−1·u−1·v₁·u₄·u₃·v−1·u−1·u−1·v−1·u₃·v₄, dont la réciproque est donnée par u₁=a₃·a−1·b−1·a₁·a₃·b₄·a₄·a−1, v₁=a₃·a−1·b−1·a−1·b−1·a−1, u₂=a₃·a−1·b−1·a−1·a−1·b−1·a₂, v₂=a₃·a−1·b−1·a−1·b₁·b₄·a₄·a−1, u₃=a₃·a−1·a−1, v₃=b₂·b₁·a₁·a₃·b₄·a₄·a−1, u₄=a₃·b₃·b₄·a₄·a−1, v₄=a₃·b−1·a−1 (et la conjugaison se fait par v−1·u−1·v₁·u₄). Est-ce pourtant, en un certain sens, « la même » solution ?

L'ennui, c'est qu'arrivé à ce stade-là, je ne sais plus très bien ce que je dois faire de cette solution, parce que je ne me rappelle plus vraiment ce que je voulais faire au début : je suis parti de questions sur le revêtement hyperbolique d'une surface de Riemann pour arriver, de fil en aiguille, à quelque chose de sérieusement différent, et maintenant que j'ai la réponse, j'ai oublié quelle était la question. Ça fait penser à une vieille blague avec un père jésuite, ça (quand on a la réponse, on ne comprend plus la question).

Suite : voir ici.

↑Entry #2251 [older| permalink|newer] / ↑Entrée #2251 [précédente| permalien|suivante] ↑

↓Entry #2250 [older| permalink|newer] / ↓Entrée #2250 [précédente| permalien|suivante] ↓

(jeudi)

Où je pose un problème combinatoire

Le problème qui suit vient d'une suite de réflexions sur le thème des deux dernières entrées, mais peu importe : la question est compréhensible et intéressante en elle-même, elle me semble même très jolie, et elle ne dépend pas de la lecture des entrées en question — je ne vais d'ailleurs quasiment pas expliquer comment je suis arrivé à ce problème (seulement en note en bas).

Je suppose que j'ai 2m symboles, pour un certain entier m≥1, que je noterai X1,X2,X3,…,Xm et X1′,X2′,X3′,…,Xm′ ; la correspondance entre Xi et Xi′ est essentielle, et je dirai que Xi′ est le symbole complémentaire de Xi et réciproquement. Je m'intéresse à des cycles de longueur 2m sur ces symboles faisant intervenir chaque symbole une et une seule fois (le terme cycle signifie qu'on identifie deux suites qui s'obtiennent l'un à partir de l'autre par une permutation cyclique, par exemple X1,X2,X1′,X2′ s'identifie à X2,X1′,X2′,X1). Il existe bien sûr (2m)!/(2m) = (2m−1)! tels cycles.

On va définir sur les cycles des opérations qui porteront le nom de chiasme de Whitehead (le terme est de moi). Pour définir un chiasme de Whitehead, on commence par choisir un des symboles Z (qui peut être un Xi ou un Xi′) qu'on appellera la base du chiasme ; puis on considère la suite des symboles strictement comprise entre Z et le symbole complémentaire Z′ (c'est-à-dire Xi′ ou Xi si Z vaut Xi ou Xi′ respectivement) qui le suit dans le cycle ; on découpe ce segment de façon quelconque en deux segments consécutifs (non vides si on veut que le chiasme soit non-trivial) et on échange ceux-ci. Voici un exemple : X1,X2,X3,X4,X1,X2,X3′,X4′ peut devenir X1,X2,X4,X1,X3,X2,X3′,X4′ par un chiasme de Whitehead si on choisit pour base Z=X2 et qu'on découpe le segment X3,X4,X1′ en X3 et X4,X1 ; en prenant pour base Z=X4 (et en se souvenant que tout est cylique !), le même cycle X1,X2,X3,X4,X1′,X2′,X3′,X4 peut devenir X3,X1,X2,X4,X1′,X2′,X3′,X4.

Remarquons que tout symbole Z peut servir à définir un chiasme de Whitehead (fût-il trivial), puisqu'on peut toujours lire cycliquement jusqu'à tomber jusqu'au symbole complémentaire Z′ qui suit ; si ce dernier survient k symboles plus loin, on pourra faire k−2 chiasmes de Whitehead non triviaux ; et comme le Z′ est lui-même suivi d'un Z (le même qu'au départ) 2mk symboles plus loin, on peut faire finalement 2m−4 chiasmes de Whitehead non triviaux partant de Z ou Z′ (du moins si ceux-ci ne sont pas immédiatement adjacents, i.e., k≠1,2m−1). Au final, on peut donc faire exactement m(2m−4) chiasmes de Whitehead non-triviaux sur n'importe quel cycle (n'ayant nulle part deux symboles complémentaire adjacents).

Ma question est la suivante : comment peut-on détecter si deux cycles peuvent se déduire l'un de l'autre par une suite de chiasmes de Whitehead, et le cas échéant, comment les produire ? (Il est évident que le problème est décidable puisqu'il est, après tout, fini, mais je demande quelque chose de plus utilisable que l'énumération exhaustive.) Une variante de cette question autorise aussi d'effectuer une permutation des symboles préservant la complémentarité (cf. ci-dessous).

Un point de vue possible, qui simplifie peut-être le problème, ou au contraire le complique, je ne sais pas, consiste à se limiter aux cycles ayant la propriété suivante. Donné un cycle, je peux considérer la fonction qui à un symbole U associe le complémentaire V′ du symbole V qui suit immédiatement U dans le cycle (autrement dit, la fonction composée du cycle considérée interprétée comme une permutation cyclique, et de la fonction « symbole complémentaire ») ; si cette fonction est elle-même un 2m-cycle (c'est-à-dire, si, en partant d'un symbole quelconque et en appliquant successivement cette fonction, on retombe sur le symbole en 2m étapes et pas avant), je dirai que le cycle de départ est parfait, et que l'autre cycle obtenu par cette construction est son dual de Whitehead. Il est clair que le dual de Whitehead est alors lui-même parfait et que son dual est le cycle de départ. Par exemple, le cycle X1,X2,X1′,X2′ est parfait et son dual est X1,X2′,X1′,X2 (i.e., dans ce cas particulier, le cycle inversé), tandis que X1,X2,X3,X1′,X2′,X3′ n'est pas pas parfaite (on tombe sur deux 3-cycle X1,X2′,X3 et X2,X3′,X1′). En fait, si m est impair, aucun cycle (de longueur 2m) n'est jamais parfait (en effet, un 2m-cycle est une permutation impaire, et la fonction « symbole complémentaire » l'est aussi si m est impair, donc la composée ne peut pas être un 2m-cycle). Notons d'ailleurs qu'un cycle parfait ne peut jamais avoir deux symboles complémentaires adjacents.

Il est facile de se convaincre que l'effet d'un chiasme de Whitehead sur le dual d'un cycle parfait est de déplacer le symbole de base d'un endroit à un autre sans changer aucun autre symbole — et notamment, un cycle parfait demeure parfait après application d'un chiasme de Whitehead (ou de façon plus générale, le nombre de cycles du dual ne changerait pas sous l'effet d'un chiasme de Whitehead si on prenait la peine de définir de façon évidente le dual d'un cycle imparfait). On peut se limiter à regarder l'effet des chiasmes de Whitehead sur les cycles parfaits. Ou, si on préfère, sur leur dual (l'effet du chiasme est alors très simple puisqu'on ne déplace qu'un symbole, mais la difficulté est qu'on ne peut pas le déplacer n'importe où).

Si j'ai bien réussi à dérouler[#] et à reformuler une série de résultats autour de la théorie des surfaces qui débutent par un théorème de Whitehead de 1936 (le neveu, pas l'oncle), on peut toujours passer d'un cycle parfait à un autre par une suite de chiasmes de Whitehead, quitte à effectuer, de plus, une permutation des variables préservant la complémentarité (c'est-à-dire qu'on peut renommer Xi en Xj ou Xj′, mais on doit alors renommer Xi′ en Xj′ ou Xj respectivement). Ce que je ne sais pas, c'est par exemple comment produire concrètement une telle suite d'opérations, ou quelle liberté on a dans le processus, ou comment détecter si on a vraiment besoin d'une permutation des variables. Ou si on a vraiment besoin de ces résultats assez compliqués (dont il s'agit d'un cas extrêmement spécial et particulier). En fait, à peu près tout est obscur pour moi dans cette histoire, à commencer par le meilleur point de vue à adopter (entre un cycle et son cycle dual, savoir s'il faut les voir comme des mots du groupe libre ou des permutations, etc.). Il faut peut-être que je me plonge dans les détails de la démonstration de la classification des surfaces topologiques pour y voir plus clair.

En attendant, ceci ferait peut-être un casse-tête amusant, que je pourrais essayer de programmer en JavaScript : quitte à tracer le cycle comme autant de points sur un cercle, entre lesquels on relierait les symboles complémentaires (c'est la seule donnée qui survit si on s'autorise, comme je le suggère ci-dessus, une permutation des variables préservant la complémentarité), essayer de transformer une configuration donnée en une autre par des chiasmes de Whitehead. Ceux-ci se voient assez bien graphiquement (comme illustré par les figures en SVG ci-contre à gauche : en rouge, la base d'un chiasme, qui échange les segments vert et bleu ; à droite, une cible possible à atteindre).

Bref, si quelqu'un a quelque chose à dire sur le sujet, ça m'intéresse. (Ou même sur les données d'un appariement sur 2m points cycliquement ordonnés, c'est-à-dire d'un 2m-cycle et d'une involution sans point fixe.)

[#] Plutôt pour m'en souvenir moi-même qu'à l'intention de mes lecteurs, je note ici rapidement le raisonnement. Si on note la suite cyclique des symboles donnée par le dual d'un cycle parfait comme je l'ai défini, on obtient un mot cyclique dans le groupe libre sur autant de générateurs ; l'algorithme de Whitehead (voir notamment Lyndon & Schupp, Combinatorial Group Theory (1977), proposition 4.19 et la discussion précédente) assure que deux mots cycliques sont transformables l'un en l'autre par un automorphisme du groupe libre exactement quand ils le sont par des transformations de Whitehead (n'allongeant pas le mot) qui, sur le cycle dual, se voient comme les chiasmes de Whitehead que j'ai définis. Mais d'autre part les cycles parfaits définissent des surfaces orientables (à un trou) de genre m/2, cf. l'article de Marc Culler auquel je faisais référence dans la dernière entrée.

↑Entry #2250 [older| permalink|newer] / ↑Entrée #2250 [précédente| permalien|suivante] ↑

↓Entry #2249 [older| permalink|newer] / ↓Entrée #2249 [précédente| permalien|suivante] ↓

(vendredi)

Où je comprends un peu mieux comment recoller les surfaces

Cette entrée fait logiquement suite à la précédente, même si je vais essayer de redire en partie (pour, j'espère, éclaircir) ce que j'y disais. Sinon, ce n'est pas grave, c'est au moins un nouveau prétexte pour faire des zoulis dessins.

[Un domaine fondamental dans le plan hyperbolique]Je me posais (dans l'entrée précédente) la question de comprendre la forme — et mathématiquement, la réalisation du groupe fondamental — d'une surface obtenue en recollant un polygone par identification de certaines arêtes du bord. (En l'occurrence, un polygone hyperbolique, mais peu importe si on veut juste se poser des questions de topologie.)

J'ai tracé ci-contre à gauche (cliquez pour agrandir) une version beaucoup plus symétrique, quitte à découper à l'intérieur des carrés du pavage, du « domaine fondamental » de mon labyrinthe hyperbolique jouet, ce sera beaucoup plus satisfaisant d'expliquer sur cet exemple-là, même si les explications que je vais donner sont tout à fait générales. J'ai aussi choisi un code de couleurs plus logique (et, j'espère, un peu moins difficile à repérer visuellement), et surtout, j'ai choisi de marquer les identifications par des pastilles au niveau des sommets plutôt que par des languettes au niveau des arêtes. Il faut donc comprendre qu'on a identifié les deux arêtes portant chaque paire de couleurs consécutives possible (par exemple les deux arêtes vert-clair–bleu-foncé sont identifiées, en identifiant bien sûr les extrémités de la même couleur ; ainsi, dans mon jeu de labyrinthe, si on sort du domaine fondamental par une arête vert-clair–bleu-foncé, on y rentre par l'autre arête ayant le même code de couleurs). Le but est de comprendre, de façon aussi explicite que possible, ce qu'on obtient en faisant ces identifications (et pourquoi, sur mon exemple, on obtient un tore à 4 poignées, et comment voir ces poignées).

Pour ceux qui auraient du mal à voir la figure ou qui voudraient plus de détails, la description complète du polygone hyperbolique ci-contre est la suivante : gris [X] bleu-clair [Y] vert-foncé* [X] rouge-clair [Y] jaune-foncé [Y] vert-clair [X] rouge-foncé* [Y] bleu-clair [X] gris [X] rouge-clair [Y] bleu-foncé* [X] jaune-clair [Y] vert-foncé [Y] bleu-clair [X] jaune-foncé* [Y] rouge-clair [X] gris [X] jaune-clair [Y] rouge-foncé* [X] vert-clair [Y] bleu-foncé [Y] rouge-clair [X] vert-foncé* [Y] jaune-clair [X] gris [X] vert-clair [Y] jaune-foncé* [X] bleu-clair [Y] rouge-foncé [Y] jaune-clair [X] bleu-foncé* [Y] vert-clair [X] : ici, les couleurs étiquettent les sommets (c'est la seule chose qui importe topologiquement), les astérisques qui suivent certains d'entre eux signifient qu'à cet endroit le polygone a un angle de 3π/4 (i.e., en suivant le périmètre, on fait un tournant de π/4 vers la gauche) alors que partout ailleurs c'est un angle droit (i.e., on fait un tournant de π/2 vers la gauche), et les lettres X et Y font référence aux longueurs des cotés, à savoir X≈1.4693517444 et Y≈1.5919125929 unités naturelles de longueur du plan hyperbolique. (Mon polygone hyperbolique a donc seulement deux angles différents et deux longueurs différentes de côtés ; par ailleurs, il est symétrique par rapport à quatre axes. Mais je répète que, pour ce qui est de la topologie, les longueurs des côtés et angles aux sommets n'ont pas d'importance.)

Le fait de colorier les sommets et pas les arêtes, pour l'identification, aide à y voir plus clair : on est naturellement amené à tracer le graphe des sommets du polygone en reliant deux sommets lorsqu'il y a une arête qui les joint sur le polygone. Sur mon exemple, bien que le polygone ait prima facie 32 sommets (et donc 32 arêtes), il n'a en fait, compte tenu des identifications, que 9 sommets distincts (i.e., 9 couleurs), et 32/2 = 16 arêtes. Le graphe d'adjacence des sommets est tracé ci-contre pour ceux dont le navigateur supporte le SVG.

↑Entry #2249 [older| permalink|newer] / ↑Entrée #2249 [précédente| permalien|suivante] ↑

↓Entry #2248 [older| permalink|newer] / ↓Entrée #2248 [précédente| permalien|suivante] ↓

(lundi)

J'essaie de recoller une surface de Riemann

[Un domaine fondamental dans le plan hyperbolique]Le hasard a fait que j'ai repensé à mon petit jeu HTML de labyrinthe hyperbolique, mais en fait surtout à sa version jouet (voir ici et pour les de mon blog où j'en discute), en même temps que je me posais des questions sur les surfaces de Riemann. En fait, je ne suis pas terriblement content de mon jeu : l'espace hyperbolique étant en lui-même labyrinthique, il est dommage de plaquer dessus un dédale avec des murs infranchissables — je voudrais refaire un jeu où les mouvements ne sont jamais bloqués, pour montrer qu'il est quand même difficile de trouver son chemin dedans. Mais pour cela, il faut que je comprenne un peu mieux des choses sur lesquelles j'ai encore des idées vagues, même sur la version jouet de mon labyrinthe.

☞ J'invite mon lecteur à essayer d'y jouer, ou plus exactement, à s'y déplacer en essayant de se faire une idée de la géographie du labyrinthe — je ne parle pas de la forme des murs eux-mêmes, mais de la périodicité de l'espace. On peut par exemple jouer à se rendre de la case de départ à la case d'arrivée en utilisant le nombre exact de déplacements optimal (indiqué à droite), et à prévoir le chemin avant de faire le moindre déplacement. On devrait au moins réussir à se convaincre qu'il n'y a en fait, dans ce labyrinthe-jouet, que 30 cellules différentes (les cellules étant des « carrés » de murs possibles, au centre desquels se trouve un cercle de référence). Les carrés blancs dans l'image ci-contre à gauche représentent un choix possible des 30 cellules du labyrinthe (c'est-à-dire que tout carré en-dehors de ceux-ci est, en fait, identique à l'un de ceux-ci) : on dit qu'il s'agit d'un domaine fondamental (pour le groupe fondamental de mon labyrinthe).

Il faut souligner qu'il y a beaucoup d'arbitraire dans le choix d'un tel domaine fondamental : j'aurais très bien pu faire passer tel ou tel carré d'un côté à l'autre en le remplaçant par un autre qui lui est équivalent. Mais bon, cela n'a pas beaucoup d'importance, et on n'aurait pas pu obtenir une figure symétrique — malgré le fait que la surface recollée que je décris ci-dessous soit réellement très symétrique.

La figure ci-contre à gauche peut être considérée comme une sorte de patron géométrique : les arêtes du bord ont été décorées par des petites languettes figurées en couleur ; si on recolle chaque languette sortante sur le bord des cases blanches avec la languette entrante qui lui correspond (par exemple, les deux languettes rouges qui forment une sorte d'entaille un tout petit peu à gauche du bas de la figure sont à recoller avec les deux languettes rouges qui forment une saillie en haut à gauche), on obtient la forme de l'espace de mon labyrinthe-jouet — ou pour dire les choses autrement, dans ce labyrinthe, si on sort du « domaine fondamental » par une des languettes colorées, on y rentre par la languette qui correspond.

↑Entry #2248 [older| permalink|newer] / ↑Entrée #2248 [précédente| permalien|suivante] ↑

↓Entry #2247 [older| permalink|newer] / ↓Entrée #2247 [précédente| permalien|suivante] ↓

(vendredi)

Grothendieck, la propriété intellectuelle, et le testament de Virgile

Bon alors il faut vraiment que je publie quelque chose sur la mort d'Alexander Grothendieck, ne serait-ce que pour faire cesser le flux de gens qui m'envoient un mail — ou postent un commentaire sur une entrée qui n'a rien à voir — pour me demander si j'en avais connaissance (oui, la preuve) ou s'étonner que je n'aie rien à dire à ce sujet (eh, calmez-vous un peu !, il est mort depuis même pas 48h, la presse people n'a pas encore sorti un numéro spécial à son sujet — permettez que je n'écrive pas instantanément des textes au kilomètre).

Je n'essaierai pas, en tout cas pas aujourd'hui, de parler du contenu de ses travaux mathématiques, parce que rien que pour vulgariser — et pas au niveau le plus élémentaire qui soit — le concept très basique de schéma j'avais écrit une des entrées les plus longues de ce blog, alors je ne sais pas ce que ça donnerait si je me lançais dans une explication de ce que sont les champs, les topos (topoï ?) ou les motifs (sur ce dernier concept au moins, ce serait en outre présumer de mes connaissances mathématiques que de tenter d'en parler).

Je n'essaierai certainement pas non plus de parler de la polémique autour de la paternité de telle ou telle idée mathématique, qui l'a conduit à se fâcher avec une partie de la communauté mathématique, notamment nombre de ses anciens élèves, et à publier sa version des faits dans un très long texte, Récoltes et Semailles, où il règle ses comptes avec beaucoup de gens. Je sais que je suis un peu extrémiste quand je pense que les articles scientifiques devraient être anonymes (pas forcément au sens où le nom des auteurs serait tenu secret mais au sens où il ne devrait pas être une donnée importante), et que les objets et théorèmes mathématiques ne devraient pas être nommés d'après des personnes vivantes ou mortes mais d'après des idées (i.e., pas comme la conjecture de Poincaré, le théorème des zéros de Hilbert ou les conjectures de Weil, mais comme l'hypothèse du continu, le théorème de la boule chevelue ou la conjecture de pureté cohomologique absolue) : toujours est-il que les questions de paternité m'intéressent très peu — le monde des idées, et en tout cas des idées mathématiques, n'est la propriété de personne. (Et de toute façon, quand les élèves de Grothendieck étaient mes enseignants ou les enseignants de mes enseignants, je me vois mal me prononcer sur qui à fait quoi.)

Je n'essaierai enfin pas de parler de la vie de Grothendieck ou de ses idées politiques (et plus généralement non-mathématiques), parce que je ne prétends pas en savoir assez, ou comprendre ce personnage complexe et énigmatique, et d'autres s'en chargeront certainement mieux que moi.

Mais il y a un point sur lequel je voudrais dire un mot, c'est sur la question des écrits de Grothendieck et de leur propriété intellectuelle (au sens juridique du droit d'auteur). Parce que la communauté mathématique, du moins, ceux qui s'intéressent à la géométrie algébrique, a un problème pratique : la référence incontournable qui fonde la lecture moderne de cette discipline est une série de textes (écrits en français), les Éléments de Géométrie algébrique (ÉGA), soit environ 1800 pages, plus les Séminaires de Géométrie algébrique [du Bois Marie] (SGA), numérotés de SGA1 à SGA7, soit environ 5700 pages au total (sans compter SGA4½), dont Grothendieck est soit l'auteur soit un coauteur (comme je l'explique ci-dessus, la question de la paternité intellectuelle m'intéresse peu — même si ici il n'y a aucune contestation — mais je parle au sens juridique). Malgré des efforts de divers auteurs pour écrire des introductions plus ou moins complètes à la géométrie algébrique (et dont le plus sérieux est sans doute le monumental Stacks Project que je salue très bas au passage ainsi que son grand coordinateur, A. Johan de Jong), aucun n'a réussi à couvrir tout ce que couvrent les ÉGA et encore moins les SGA (et quand bien même on y arriverait, il resterait encore que toutes les références numériques à ces textes n'ont de sens que si on y a accès).

Or voici le problème pratique : Alexander Grothendieck s'est toujours opposé à ce qu'on réédite ces textes. (Je ne prétends pas comprendre, encore moins expliquer, quelles étaient ses motivations pour le refuser.)

↑Entry #2247 [older| permalink|newer] / ↑Entrée #2247 [précédente| permalien|suivante] ↑

↓Entry #2240 [older| permalink|newer] / ↓Entrée #2240 [précédente| permalien|suivante] ↓

(vendredi)

Un petit exercice d'Analyse (moyenner une fonction)

Un petit exercice d'Analyse pas très difficile (j'en ai traité un bout avec mes élèves à Télécom Paris), mais que je trouve amusant :

Soit f:ℝ/ℤ→ℝ (c'est-à-dire : une fonction réelle de la variable réelle qui soit 1-périodique) ; on note N(f) la fonction définie comme la moyenne arithmétique des N translatées de f par des multiples de 1/N, c'est-à-dire : (N(f))(x) = (1/N) · ∑k∈{0,…,N−1} f(x+k/N) ; et, si f est intégrable (c'est-à-dire, intégrable sur une période), soit (f) la fonction constante égale à l'intégrale ∫ℝ/ℤ f de f (sur une période). On se demande dans quelle mesure N(f) tend vers (f) quand N tend vers +∞ :

  • Si f est Lp (c'est-à-dire, de puissance p-ième intégrable sur une période) pour 1≤p<+∞, alors N(f) tend vers (f) dans Lp (c'est-à-dire : l'intégrale de la puissance p-ième de la différence tend vers 0).
  • Si f est Riemann-intégrable, alors N(f) tend vers (f) uniformément.

(Indication : montrer la convergence uniforme — qui entraîne donc la convergence Lp pour un p quelconque — pour une fonction en escalier ; il suffit pour ça de la montrer pour la fonction indicatrice d'un intervalle [0;c[ de ℝ/ℤ.)

Dans le cas p=2, il y a une jolie démonstration en regardant les séries de Fourier (l'effet de N est de décimer la série de Fourier).

On pourra aussi montrer que N ne tend pas vers en tant qu'opérateur (i.e., pour la norme).

Bref, je sais faire ça, mais j'ai quand même l'impression de manquer de recul sur la question : qu'il doit y avoir une façon plus élégante et plus générale d'inscrire ces résultats dans un contexte plus éclairant. D'ailleurs, la deuxième partie me surprend beaucoup, j'étais tellement persuadé que le résultat aurait dû être l'affirmation plus faible si f est Riemann-intégrable, alors N(f) tend vers (f) ponctuellement, et si f est réglée, alors la convergence est uniforme que j'ai cherché en vain à trouver une erreur dans mon raisonnement (j'ai fini par me convaincre qu'il était bien correct, mais j'ai toujours la sensation déplaisante d'avoir mal compris quelque chose d'important).

Pour mémoire, une fonction f est dite réglée (ou parfois Dieudonné-intégrable) lorsque pour tout ε>0 il existe une fonction en escalier h telle qu'on ait partout |fh|≤ε (i.e., f est uniformément approchable par les fonctions en escalier) ; cela équivaut à dire qu'elle admet en tout point une limite à gauche et une limite à droite (finies). • Une fonction f est dite Riemann-intégrable lorsque pour tout ε>0 il existe des fonctions en escalier h et ψ telles qu'on ait partout |fh|≤ψ, avec ∫ψε (i.e., f est approchable par les fonctions en escalier avec une erreur uniformément contrôlée par une fonction en escalier elle-même de norme 1 arbitrairement petite) ; cela équivaut à dire que f est bornée et que son ensemble de points de discontinuités est Lebesgue-négligeable. • Pour comparaison, si fonction f est Lebesgue-intégrable, pour tout ε>0 il existe une fonction en escalier h telle que ∫|fh|≤ε (i.e., f est approchable au sens L¹ par les fonctions en escalier). • J'aime bien présenter ces trois propriétés côte à côte, cela aide à situer la notion d'intégrabilité au sens de Riemann entre celle de fonction réglée et celle d'intégrabilité au sens de Lebesgue. Cela devrait peut-être expliquer pourquoi j'avais l'intuition qu'on aurait besoin de f réglée pour pouvoir conclure à la convergence uniforme de N(f) vers (f).

Ajout : voir aussi cette question pour une « suite » de cet exercice.

↑Entry #2240 [older| permalink|newer] / ↑Entrée #2240 [précédente| permalien|suivante] ↑

↓Entry #2237 [older| permalink|newer] / ↓Entrée #2237 [précédente| permalien|suivante] ↓

(dimanche)

Les entrées de blog que je n'arrive jamais à écrire

Il y a une éternité j'ai promis d'écrire une entrée de ce blog sur les octonions. Je n'ai jamais réussi à la publier. Ce n'est pas que je n'aie rien à écrire, au contraire, ni même que je n'aie rien écrit : j'ai commencé (au hasard des moments où j'ai du temps pour le faire) à écrire des choses sur le sujet, et de plus en plus, et je me suis rendu compte que ça débordait dans tous les sens, et qu'à la fin non seulement ça devenait trop long et indigeste, mais en plus que ça manquait de structure et de cohérence thématique. Alors j'ai décidé de diviser cette entrée en trois parties : des généralités sur les octonions d'abord, puis une petite partie sur les octonions entiers et les réseaux dans les octonions, et enfin une troisième (indépendante de la seconde) sur la géométrie octonionique et le carré magique de Freudenthal-Tits — qui est à mon avis la principale raison pour laquelle les octonions sont intéressants. Puis j'ai commencé à développer la première partie séparément, et je me suis rendu compte que je devais parler d'automorphismes des octonions, et à me poser plein de questions qui débordaient dans tous les sens, et finalement cette partie-là, même prise seule, devient aussi trop longue, et il faudrait la couper à son tour. [Mise à jour () : cette partie a fini par être publiée.] Pareil pour la troisième partie : là j'ai pensé, je vais changer de point de vue et écrire une petite entrée sur les espaces deux-points-homogènes (dits aussi : homogènes et isotropes), c'est-à-dire, pour parler grossièrement, les espaces (au sens : variétés riemanniennes) qui sont identiques en tous les points et dans toutes les directions. C'est une question très naturelle et intéressante que de classifier ces espaces homogènees et isotropes, la réponse a été apportée par Tits et Wang (très rapidement : outre l'espace euclidien et les sphères, ce sont les espaces projectifs et hyperboliques sur les réels, complexes, quaternions, et octonions, sachant que sur les octonions il n'y a que la droite et le plan projectif, et la droite et le plan hyperbolique, pas de dimension plus élevée) ; ce sont des espaces très beaux par leur pure symétrie, et élégants dans leur description, et ils amènent naturellement à parler des octonions et des groupes de Lie exceptionnels (et cela apporte une réponse possible à la question qui m'avait tracassé, qu'est-ce qu'une géométrie). Et bien sûr, en essayant de parler de ça, j'ai de nouveau eu trop de choses à dire et de nouveau ça débordait dans tous les sens.

Il n'y a pas que les octonions qui m'aient causé ce souci. C'est un peu quelque chose qui m'arrive à chaque fois que je parle de n'importe quoi : je ne sais pas sélectionner ce dont je veux parler, ma logorrhée s'étale sans structure dans toutes les directions, et à la fin tout est trop long et indigeste. Mais bien sûr, c'est pire quand je parle de maths. J'ai par exemple aussi voulu écrire une entrée sur les notations ordinales, essentiellement, pour faire suite à cette entrée déjà très longue, et expliquer comment on peut « fabriquer » (décrire, expliciter, travailler avec, calculer sur) des ordinaux récursifs très grands — essentiellement, vulgariser la notion de fonction d'écrasement pour fabriquer de grands ordinaux à partir de cardinaux de plus en plus sophistiqués (inaccessibles, Mahlo), cette question étant du coup naturellement liée à celle des grands nombres que j'ai abordée à plusieurs reprises. Bref, j'ai travaillé là-dessus au hasard de ma motivation et de mon temps disponible. Et puis je suis rentré dans les explications sur les fonctions d'écrasement forcément d'autant plus compliquées qu'on fabrique des ordinaux plus grands, c'est devenu long, très long, très très très long, et il me reste sur les bras une montagne d'explications que je ne sais pas bien comment structurer ou diviser pour la rendre un peu digeste. (Cela n'arrange pas les choses que j'ai cru comprendre quelque chose, que je me suis trompé, que j'ai compris autrement, que je me suis retrompé, que j'ai compris que j'avais bien compris initialement, et qu'au bout du compte je me suis beaucoup embrouillé sur les différentes variantes qu'on peut construire autour des fonctions d'écrasement — par exemple, il y en a qui sont croissantes, d'autres qui ne le sont pas, et il y a toutes sortes de conventions possibles sur comment organiser et définir les valeurs.) [Mise à jour : j'ai quand même fini par publier une entrée sur ce sujet, même si je ne sais pas bien si c'est celle dont je parlais ci-dessus.] J'ai encore d'autres entrées dans le même genre, commencées parce que je pensais que je n'aurais pas tant de choses que ça à dire, et finalement dans des limbes où les choses sont à moitié écrites et peut-être ne seront jamais achevées.

C'est un peu l'histoire de ma vie, de commencer plein de choses, et de n'en finir que très peu.

Alors qu'est-ce que je devrais faire ? Publier des choses inachevées ? En publier le début, quitte à nuire à la cohérence de la suite ? Laisser tomber ? Attendre un temps potentiellement infini que j'arrive à terminer ce que j'ai commencé ? Je ne sais vraiment pas.

Il faut dire aussi que je ne sais pas bien qui lit mon blog, et notamment qui lit les entrées mathématiques (et quel est son niveau en maths et quels sont ses centres d'intérêt). En vérité, le principal lecteur pour lequel j'écris, c'est mon moi futur : j'écris parce que j'ai compris quelque chose, que j'ai envie de pouvoir le recomprendre à l'avenir quand j'aurai un peu oublié, alors je me l'explique à moi-même pour savoir que je pourrai relire telle ou telle entrée et resavoir ce que j'aurai su. À titre d'exemple, en réfléchissant à des questions de bases de données SQL, je suis récemment retombé sur cette entrée, et il y a un peu plus longtemps, en me posant des questions de physique, sur celle-ci : dans les deux cas, j'avais totalement oublié les subtilités de ce que j'y raconte, et je n'étais pas fâché que la personne qui sait le mieux m'expliquer les choses — c'est-à-dire, moi — me prenne par la main pour me redire comment tout ça fonctionne. Idem concernant les séries de Fourier (que je m'oblige à réapprendre à chaque fois que je dois enseigner le sujet, sachant très bien que j'oublierai dans le mois qui suit). En termes informatiques, on pourrait dire que mon blog est mon espace de swap : c'est là que je consigne les choses quand ma mémoire déborde (ou que je veux changer de contexte). Mais le temps de swap est lui-même long !

↑Entry #2237 [older| permalink|newer] / ↑Entrée #2237 [précédente| permalien|suivante] ↑

↓Entry #2223 [older| permalink|newer] / ↓Entrée #2223 [précédente| permalien|suivante] ↓

(lundi)

La fonction de Fabius

[Graphe de la fonction de Fabius sur [0;24]]

J'ai déjà raconté trente-douze fois sur ce blog, et je continuerai à radoter à ce sujet, qu'une de mes motivations à être mathématicien, c'est de pouvoir visiter le petit musée des objets mathématiques remarquables, admirables, ou simplement curieux. L'objet dont je veux parler ici est plus amusant que fascinant, ce qui ne l'empêche pas d'être élégant. Il s'agit d'une fonction réelle définie par un certain J. Fabius (j'ignore le prénom ou s'il y a un lien de parenté avec l'actuel ministre français des affaires étrangères) dans une publication de 1966.

[Graphe de la fonction de Fabius sur [0;1]]Voici une première définition possible : on appelle Un (pour n≥1 entier) une suite de variables aléatoires indépendantes et uniformément réparties dans [0;1], et on pose Z = ∑n Un/2n (remarquer que Z est aussi une variable aléatoire à valeurs dans [0;1]). Alors la fonction de Fabius f sur [0;1] est la distribution de probabilité de la variable Z (c'est-à-dire que f(t), pour t∈[0;1], est la probabilité que Zt). (Le graphe en est tracé ci-contre à gauche.) Pour souligner que cette loi de probabilité n'est pas complètement bizarre ou surgie de l'espace, on peut remarquer que si au lieu d'une variable Un uniformément répartie dans [0;1] on utilisait des variables Vn (toujours indépendantes et) uniformément réparties dans {0;1} (c'est-à-dire valant 0 avec probabilité ½ et 1 avec probabilité ½), alors ∑n Vn/2n serait uniforément répartie dans [0;1] puisque la construction revient à tirer chaque bit de cette variable à pile ou face indépendamment.

Pour compléter f aux réels positifs, on décrète alors que f(t+1) = 1−f(t) pour t∈[0;1], et que f(t+2k) = −f(t) pour t∈[0;2k] (lorsque k≥1 est entier). Le caractère naturel de ce prolongement va apparaître dans ce qui suit. (Le graphe de la fonction ainsi prolongée est tracé en haut de cette entrée.)

La fonction possède un lien très fort avec la suite de Morse-Thue, qui peut être définie comme la suite dont le n-ième terme vaut 0 ou 1 selon que le nombre de 1 dans l'écriture binaire de n (=le poids de Hamming H(n) de n) est pair ou impair ; si on préfère, la suite de Morse-Thue est celle qui s'obtient en partant d'un 0 et en effectuant un nombre infini de fois le remplacement simultané de 0 par 01 et de 1 par 10 (cf. l'article Wikipédia). Comme on peut le voir sur le graphe ci-dessus (et ce n'est pas difficile avec la définition que j'ai donnée), le signe de f sur l'intervalle [2n;2n+1] vaut + ou − selon que la valeur du n-ième terme de la suite de Morse-Thue est 0 ou 1.

[Construction de la fonction de Fabius]Voici une autre définition possible de la fonction de Fabius, dans cet esprit : on appelle d'abord f0 la fonction qui sur l'intervalle [2n;2(n+1)[ vaut +½ ou −½ selon que la valeur du n-ième terme de la suite de Morse-Thue est 0 ou 1 ; puis par récurrence sur k, on appelle fk+1(x) l'intégrale de fk(t) pour t allant de 0 à 2x. La suite de fonctions en question converge alors uniformément vers f. (Ci-contre, j'ai tracé f0 en vert, f1 en un vert subtilement différent, f2 en caca d'oie et f en bleu.)

[Graphe de la fonction de Fabius sur [0;24] et de sa dérivée]Qu'on utilise l'une ou l'autre construction, il n'est pas très difficile de voir que la fonction de Fabius vérifie l'équation fonctionnelle surprenante suivante : f′(t) = 2f(2t) pour tout t≥0. (Pour l'illustrer, j'ai tracé la fonction de Fabius accompagnée de sa dérivée f′.) Notons qu'il ne s'agit pas d'une équation différentielle, puisque la valeur de la dérivée est reliée à la valeur de la fonction en un autre point ; d'ailleurs, il n'y a pas unicité de la solution de cette équation : la fonction nulle la vérifie aussi (avec, d'ailleurs, la même « condition initiale » f(0)=0) ; néanmoins, l'équation en question et la connaissance de la fonction f sur [0;1] définit de façon unique sa valeur sur tous les réels positifs.

Puisque f(0)=0, l'équation fonctionnelle f′(t) = 2f(2t) implique de façon assez évidente que toutes les dérivées de f en 0 valent 0 : la fonction de Fabius est infiniment plate en 0. En fait, il aussi facile de voir qu'en n'importe quel rationnel dyadique, toutes ses dérivées s'annulent à partir d'un certain rang : par conséquent, f n'est pas égale à la somme de sa série de Taylor développée en un dyadique, et comme les dyadiques sont denses dans les réels, f est une fonction C mais nulle part analytique (c'était la motivation de son introduction). On peut aussi se persuader que, en une valeur comme 1/3, la série de Taylor de f a un rayon de convergence nul (parce que la dérivée r-ième de f est donnée par f(r)(t) = 2r(r+1)/2·f(2r·t)).

Mais il y a quantité de choses que j'ignore. Par exemple, expérimentalement, il semble que f(1/4) = 5/72, et je ne sais pas le prouver (pas que j'aie essayé, du reste). Il serait tentant de penser, pour l'expliquer, qu'il y ait une relation entre f(t) et f(t+½) comme il y en a entre f(t+2s) pour tout s≥0 entier, mais une telle relation ne peut pas, il me semble, être algébrique, donc je ne sais pas ce qu'on est en droit d'espérer. Je ne sais pas non plus si on peut dire quelque chose d'intelligent de la valeur f(1/3) ≈ 0.180165114801 (cette valeur n'évoque rien à Google, mais Google n'est pas très bon pour faire du calcul symbolique inverse ; elle n'évoque rien non plus à ce site ni à Wolfram Alpha). Pourtant, l'écriture binaire du nombre 1/3 est suffisamment remarquable pour qu'on soit en droit de penser que la fonction f devrait en faire quelque chose d'intéressant.

Enfin, je dois signaler que je ne sais pas vraiment calculer les valeurs de f de façon efficace. Le mieux que je connaisse est une expression de fk que j'écris en MathML pour ceux dont le navigateur le supporte :

f k ( x ) = 2 k(k+1)/2 k! 0j2k1x cj ( x j2k1 ) k c0 = 12 et cj = 12 ( (1) H(j) (1) H(j1) ) si j>0

(j'ai noté H(j) pour le poids de Hamming de j). Mais cette équation n'est pas vraiment satisfaisante parce qu'elle ne relie pas fk+1 à fk, par exemple par une sorte de dichotomie. Bref, la fonction de Fabius reste assez mystérieuse à mes yeux.

Ajout : dans un même esprit, voir aussi la fonction ‘?’ [point d'interrogation] de Minkowski dont j'avais déjà parlé autrefois.

Ajout : voir aussi la réponse de François Guéritaud en commentaire () où il explique comment calculer la valeur et les dérivées successives de f aux dyadiques en comparant des intégrales de la fonction et de ses polynômes osculateurs, ce qui répond en partie à mes interrogations (notamment sur le fait que f(1/4)=5/72). • Surajout : voir ce petit texte de Haugland qui explique essentiellement la même chose.

↑Entry #2223 [older| permalink|newer] / ↑Entrée #2223 [précédente| permalien|suivante] ↑

↓Entry #2219 [older| permalink|newer] / ↓Entrée #2219 [précédente| permalien|suivante] ↓

(mercredi)

Quelques horreurs mathématiques (autour du 17e problème de Hilbert)

J'ai passé un certain nombre de jours en ermite à me reposer et à apprendre des maths amusantes. Je ne vais pas faire un compte-rendu (j'exposerai sans doute des bouts un peu au hasard d'entrées à venir), mais je viens de retomber sur les faits suivants, qui sont vraiment trop horribles pour ne pas les raconter.

(Attention, éloignez les enfants, c'est vraiment épouvantable.)

Commençons par ceci :

Il existe une fonction f qui est C sur ℝ, positive, ne s'annulant qu'en 0 où elle est infiniment plate (i.e., sa dérivée s'annule à tout ordre) et dont la racine carrée n'est même pas C2. Ou, comme il est facile de voir qu'il est équivalent : f ne peut pas s'écrire comme le carré d'une fonction C2.

Pourquoi c'est horrible ? Parce que la racine carrée d'une fonction C strictement positive est évidemment C (puisque la racine carrée est C sur les réels strictement positifs), et on se dit qu'une fonction positive présentant un unique point d'annulation devrait être gérable : si la fonction f est simplement supposée positive partout et nulle à l'origine, la dérivée doit y être nulle, donc f a un développement limité à tout ordre en 0 commençant par c·x², et il est possible d'extraire une racine carrée de ce développement, donc on imagine facilement qu'on devrait pouvoir mettre ensemble cette racine carrée du développement avec la racine carrée de la fonction hors de l'origine pour obtenir une racine carrée qui soit C. Et si la fonction est infiniment plate à l'origine, ça devrait être encore plus facile : le développement est nul, il n'y a donc aucune difficulté à prendre sa racine carrée, et on aimerait donc croire que cette fonction √f, qui a un développement limité à tout ordre en 0 et qui est C ailleurs qu'en 0, devrait bien être C partout !

Eh bien non. Non seulement la racine carré n'est pas forcément C, mais elle n'est même pas forcément C2, ce qui est tout de même très vexant. Le contre-exemple figure dans l'article Glaeser, Racine carrée d'une fonction différentiable, Ann. Inst. Fourier Grenoble 13 (1963) 203–210 (partie III ; le début montre que, quand même, la racine carrée est C1), et il n'est même pas difficile (il tient en deux petites pages, et c'est essentiellement des maths de classes préparatoires).

Bon, passons à la suite des horreurs. On se dit que quand même, à défaut de pouvoir écrire une fonction C positive comme carré d'une fonction bien régulière, on devrait au moins pouvoir l'écrire comme somme de carrés de fonctions bien régulières, ou quelque chose du genre. (Ce genre de questions consistant à écrire des choses positives comme somme de carrés s'appelle le 17e problème de Hilbert, sauf que normalement on s'intéresse plutôt à des fonctions rationnelles.)

↑Entry #2219 [older| permalink|newer] / ↑Entrée #2219 [précédente| permalien|suivante] ↑

↓Entry #2217 [older| permalink|newer] / ↓Entrée #2217 [précédente| permalien|suivante] ↓

(mardi)

Pourquoi e et π paraissent-ils plus aléatoires que génériques ?

Je veux discuter ici non d'une question de maths mais d'une question de philosophie des maths (et qui, pour une fois, n'est pas de la logique mais plutôt la théorie des nombres !). Néanmoins, pour l'expliquer, il faut bien que je parle de maths.

Un fait empirique est le suivant : quand on fait des études statistiques sur les décimales, disons, du nombre e ou du nombre π, celles-ci se comportent empiriquement comme une suite aléatoire (comme si elles avaient été tirées au hasard par un grand dé cosmique). Par exemple, les décimales en base 10 semblent équidistribuées (il y a autant de 0 que de 1 que de 2… que de 9) ; mieux, les suites de 2 chiffres semblent équidistribuées (il y a autant de 00 que de 01… que de 99), et pareil pour les suites de 3 chiffres et plus, tant qu'on a assez de données pour faire des statistiques significatives (or, s'agissant des décimales de π, on en a beaucoup). Autrement dit, on conjecture que π est un nombre « normal », ce qui regroupe ces différentes affirmations sur la fréquence des décimales. Et ce n'est pas vrai qu'en base 10, qui n'a aucune raison d'être spéciale : on conjecture que π est normal en toute base (entre autres, écrit en base 2, on conjecture qu'il devrait contenir quelque part le contenu de ce blog jusqu'à sa fin, codé en binaire ; ceci n'a, évidemment, rien de remarquable : il faudrait aller si loin dans les décimales pour le trouver qu'indiquer l'endroit où on le trouve est essentiellement aussi long que donner le contenu lui-même).

Pour motiver cette conjecture on donne typiquement l'explication suivante : « presque tous » les nombres réels sont normaux en toute base. C'est-à-dire que si on tire un nombre réel aléatoirement (uniformément entre 0 et 1), la probabilité qu'il ait les propriétés que j'esquisse ci-dessus vaut exactement 1. Ceci est un énoncé mathématique clair et pas très difficile (pas du tout conjectural) : l'ensemble des nombres réels qui n'ont pas la propriété d'être normaux en toute base est un ensemble dit négligeable (=de mesure de Lebesgue nulle, ce qui signifie techniquement qu'on peut le recouvrir par une suite d'intervalles dont la somme des longueurs converge et a une somme arbitrairement petite, cf. ci-dessous), correspondant à un événement de probabilité 0. On reformule aussi ce fait en disant que presque tous les nombres réels sont normaux en toute base (presque tous veut dire précisément que l'ensemble de ceux qui ne le sont pas est négligeable). Dès lors (dit-on), il n'est pas surprenant, si presque tous les nombres réels ont la propriété d'être normaux en toute base, de conjecturer que π en particulier l'est. Je ne prétends pas que cette justification soit insensée, mais elle glisse de la poussière sous le tapis, à savoir la raison pour laquelle presque tous est une bonne notion.

↑Entry #2217 [older| permalink|newer] / ↑Entrée #2217 [précédente| permalien|suivante] ↑

↓Entry #2211 [older| permalink|newer] / ↓Entrée #2211 [précédente| permalien|suivante] ↓

(vendredi)

Petit tour de magie 2-adique

Je me demande régulièrement s'il serait possible de trouver une application des nombres p-adiques ailleurs qu'en mathématiques ; par exemple, une application des 2-adiques en informatique (ce qui semble le plus plausible, parce que les ordinateurs, manipulant des nombres binaires, manipulent en fait des entiers 2-adiques approchés). Je n'ai pour l'instant rien trouvé de bien convaincant. Voici cependant un exemple qui pourrait servir avec un peu d'imagination, et qui en tout cas fait un « tour de magie » rigolo :

Soit a un entier impair écrit en binaire, disons, sur 64 bits, dont on suppose qu'il est le carré d'un entier : on cherche à retrouver cette racine carrée (exacte). Voici une façon de s'y prendre : (1) itérer, en partant de y=1, la fonction y ↦ 2ya·y², jusqu'à tomber sur un point fixe qu'on notera b (note : tous les calculs sont faits en binaire sur la même largeur, disons 64 bits ; comme il est habituel en informatique, on jette les bits supérieurs) ; (2) itérer, en partant de x=1, la fonction xx·(3−b·x²)/2. Autrement dit, en C :

unsigned long
exact_odd_square_root (unsigned long a) {
  unsigned long y = 1;
  for ( unsigned long yn = 0 ; y != (yn = 2*y - a*y*y) ; y = yn );
  unsigned long x = 1;
  for ( unsigned long xn = 0 ; x != (xn = x*((3-y*x*x)>>1)) ; x = xn );
  if ( x & ((((unsigned long)(-1))>>2)+1) )
    x = -x;
  return x & ((unsigned long)(-1))>>1;
}

(Les dernières lignes servent à corriger le nombre : il y a quatre valeurs de x sur vérifiant x²=a, différant par le bit de poids fort et/ou par un changement de signe global — la fonction renvoie donc celui dont les deux bits de poids fort valent 0. L'écriture ((((unsigned long)(-1))>>2)+1) sert à représenter le nombre ayant 1 juste au-dessous du poids fort sans avoir à faire d'hypothèse sur la taille des unsigned long.)

La fonction est évidemment limitée (on pourrait calculer une fonction exact_square_root() en décalant le nombre du nombre de bits adéquat — forcément pair — jusqu'à trouver un nombre impair, en appliquant la fonction exact_odd_square_root() ci-dessus, puis en refaisant le décalage vers la gauche de la moitié du nombre de bits, mais la gestion des bits de poids fort serait encore un peu plus pénible). Il y a cependant un truc rigolo, c'est qu'elle retrouve la racine carrée même si le calcul du carré a débordé (par exemple, sur 64 bits, si on fait 1000000000001*1000000000001, on trouve 2003766205206896641 et pas 1000000000002000000000001, mais la fonction ci-dessus retrouve bien 1000000000001 comme racine carrée pour cette valeur), du moins si les deux bits de poids fort valent 1 (on ne peut pas faire mieux). Par ailleurs, le nombre d'itérations est très petit (quelque chose comme 6 au pire dans chaque boucle pour un nombre de 64 bits), donc on pourrait dérouler les boucles.

L'explication 2-adique est vraiment facile : la première itération calcule l'inverse 2-adique b de a par une méthode de Newton, la seconde calcule la racine carrée par une méthode du même genre (on peut peut-être la présenter comme une méthode de Newton, en tout cas j'ai cherché un polynôme ayant un point fixe superattractif où on veut). J'imagine que je ne suis pas le premier à écrire un truc de ce genre, je n'ai pas cherché. Par contre, ce que j'aimerais bien, c'est trouver des exemples plus frappants ou plus utiles.

↑Entry #2211 [older| permalink|newer] / ↑Entrée #2211 [précédente| permalien|suivante] ↑

↓Entry #2210 [older| permalink|newer] / ↓Entrée #2210 [précédente| permalien|suivante] ↓

(jeudi)

Quelques petits jeux avec l'algèbre commutative

Alice et Bob jouent au jeu suivant : dans l'anneau k[t1,…,tn] des polynômes en n indéterminées sur un corps k, chacun choisit à tour de rôle un polynôme f, la règle étant qu'on n'a pas le droit de choisir un polynôme de la forme g1·f1 + ⋯ + gr·frf1,…,fr sont les polynômes qui ont déjà été joués (et notamment, le polynôme nul) ; ou, si on préfère, l'idéal (f1,…,fr) = {g1·f1 + ⋯ + gr·fr} doit grandir strictement à chaque étape ; lorsque le polynôme 1 (donc, n'importe quel polynôme) peut s'écrire sous la forme g1·f1 + ⋯ + gr·fr, le joueur qui vient de jouer a perdu (autrement dit, on joue à la variante « misère » du jeu : celui qui ne peut pas jouer a gagné ; l'autre variante n'est pas intéressante, parce que qu'on gagne immédiatement en jouant le polynôme 1). Question : qui a une stratégie gagnante ? (En fonction de n et, éventuellement, du corps k.)

J'avoue ne pas savoir dire grand-chose d'intelligent sur ce problème. Si n=1, dans k[x], Alice (le premier joueur) a une stratégie gagnante évidente, consistant à jouer x (l'unique indéterminée). Si n=2, dans k[x,y], il me semble que le premier joueur gagne encore en jouant y² (si Bob réplique par y, Alice gagne parce qu'on est ramené au cas n=1 ; dans tout autre cas, l'intersection entre la droite y=0 comptée avec multiplicité 2 et la courbe algébrique d'équation définie par ce que Bob aura joué sera un nombre fini de points avec des multiplicités paires, et Alice peut alors sans difficulté au coup suivant tuer tous ces points sauf un qu'elle garde avec multiplicité 1, ce qui gagne le jeu), mais je suis loin d'avoir vérifié les détails et il n'est pas du tout improbable que je me sois trompé. Ce papier montre cependant qu'Alice a bien une stratégie gagnante, soit avec des arguments du même genre en jouant y²−x³ (§6.2), soit avec un argument différent et peut-être plus rigolo en jouant y²−x³+x−1 (corollaires 6.4–6.5). J'ai vaguement tendance à croire qu'Alice gagne toujours quand on part d'un anneau de polynômes, mais je ne sais vraiment pas le prouver. (Ce qui ne veut pas dire que ce soit très dur : je n'ai pas réfléchi très fort.)

Géométriquement, le jeu consiste à partir de l'espace affine de dimension n et à intersecter avec des hypersurfaces f=0 de façon à fabriquer des « sous-schémas fermés » de plus en plus petits, celui qui aboutit sur le vide ayant perdu (dans la variante « misère »).

Le jeu sous une forme un peu plus générale s'écrit ainsi : si R est un anneau [commutatif] nœthérien (on prend R=k[t1,…,tn] dans l'exemple ci-dessus), chacun des deux joueurs à son tour remplace R par le quotient de celui-ci par un de ses éléments non nuls (i.e., par un idéal principal non nul), et le premier qui tombe sur l'anneau nul a perdu (dans la variante « misère »). Le jeu termine nécessairement en temps fini car on construit une suite strictement croissante d'idéaux de l'anneau nœthérien R de départ (ceux par quoi on a quotienté jusqu'à présent). Bien sûr, je ne suis pas le premier à y penser, c'est vraiment tout naturel une fois qu'on se rappelle que tout processus terminant conduit à un jeu impartial. On peut bien sûr jouer avec toutes sortes d'autres structures algébriques nœthériennes (je suppose mes anneaux commutatifs parce que je suis géomètre algébriste, mais on peut évidemment faire des choses avec les non commutatifs et des idéaux à gauche — ou bilatères). Par exemple, Alice et Bob pourraient jouer alternativement des éléments de ℤm définissant des sous-ℤ-modules (=sous-groupes abéliens) de celui-ci, avec une inclusion stricte à chaque fois, et cette fois-ci on peut jouer à la variante normale du jeu (i.e., celui qui ne peut plus jouer a perdu) : il n'est pas extrêmement difficile — mais pas trivial non plus — de trouver montrer que Bob (le second joueur) a une stratégie gagnante si et seulement si m est pair.

On pourrait imaginer d'autres variations : par exemple, en revenant aux polynômes dans k[t0,…,tn], changer un peu la règle en imposant de jouer des polynômes homogènes et sans doute en terminant quand il y a une puissance de chaque indéterminée dans l'idéal qu'on a engendré, ce qui a aussi un contenu géométrique naturel : cette fois on joue avec des sous-schémas fermés de l'espace projectif de dimension n. On pourrait aussi jouer avec des monômes, auquel cas les coefficients n'existent plus et on est simplement en train de jouer au jeu de chomp multidimensionnel. Je trouverais satisfaisant d'arriver à plonger le jeu de chomp dans le jeu d'un anneau nœthérien sans restriction, mais j'avoue ne pas voir de façon de faire ça. (Je trouverais aussi satisfaisant d'arriver à résoudre le jeu de départ sur les polynômes en le ramenant au jeu de chomp par une utilisation astucieuse de bases de Gröbner qui feraient qu'on peut toujours supposer qu'on joue avec des monômes, c'est sans doute une idée naïve.)

Toujours est-il que ce jeu conduit à un invariant rigolo (quoique pas très sérieux) d'un anneau nœthérien, c'est ce que j'ai envie d'appeler sa fonction de Grundy-Gulliksen (je vais expliquer pourquoi Gulliksen, mais pour Grundy, voir mon entrée sur les jeux combinatoires que j'ai déjà liée, spécialement sa deuxième partie). La définition est très simple et très jolie :

Si R est un anneau [commutatif] noethérien, la fonction de Grundy-Gulliksen de R est le plus petit ordinal qui n'est pas égal à la fonction de Grundy-Gulliksen d'un R/(f) pour un élément f≠0 dans R.

La définition est récursive (définir la fonction de Grundy-Gulliksen de R demande de connaître celle de tous les quotients R/(f) de R par un idéal principal non nul), mais elle a quand même un sens par nœthérianité : c'est toute la beauté de l'induction nœthérienne.

Noter qu'il s'agit là de la fonction de Grundy pour la variante normale du jeu, qui (sur tout anneau non nul) vaut 1 plus la fonction de Grundy pour la variante misère. Donc la stratégie gagnante pour au jeu (variante misère) consiste à toujours jouer vers un anneau dont la fonction de Grundy-Gulliksen vaut 1.

Bon, je ne sais essentiellement rien dire d'intelligent sur ce nombre. En revanche, si au lieu de considérer R comme un jeu je le considère comme un processus terminant dont il faut évaluer la longueur (voir la première partie de mon entrée sur les jeux), on obtient une quantité très intéressante :

Si R est un anneau [commutatif] noethérien, la longueur de Gulliksen de R est le plus petit ordinal strictement supérieur à la longueur de Gulliksen de tout R/(f) pour un élément f≠0 dans R. (De façon équivalente, c'est le plus petit ordinal strictement supérieur à la longueur de Gulliksen de tout R/I pour un idéal I≠(0) de R.)

(L'équivalence dans la parenthèse finale n'a évidemment pas d'analogue pour la fonction de Grundy-Gulliksen : cela reviendrait à donner aux joueurs la possibilité de quotienter l'anneau autant de fois qu'ils veulent, auquel cas le jeu perd évidemment tout intérêt.)

On peut évidemment généraliser ça à d'autres choses : notamment, la longueur de Gulliksen d'un module nœthérien M sur un anneau R est le plus petit ordinal strictement supérieur à la longueur de Gulliksen de tout quotient M/N de M par un sous-R-module N (et en fait, on n'a pas besoin de supposer R commutatif, et d'ailleurs Gulliksen ne le fait pas) ; la longueur de Gulliksen d'un schéma nœthérien est le plus petit ordinal strictement supérieur à la longueur de Gulliksen de n'importe quel sous-schéma fermé strict. Toutes ces définitions ont un sens bien que récursives, grâce à la magie de l'induction nœthérienne.

Par exemple, l'anneau nul, comme il n'a aucun quotient non-trivial, a une longueur nulle (0 est le plus petit ordinal strictement supérieur à tout élément de l'ensemble vide), et c'est manifestement le seul ; un corps a une longueur 1, et réciproquement tout anneau de longueur 1 est un corps. Un espace vectoriel de dimension finie sur un corps a une longueur (en tant que module sur ce corps) égale à sa dimension. L'anneau k[t]/(t²) a une longueur 2, tandis que k[t] a longueur ω (parce que k[t]/(f) a une longueur égale au degré de f pour tout f non nul).

J'appelle cette notion longueur de Gulliksen parce qu'elle a été étudiée dans un très bel article par Tor Gulliksen en 1973. Elle généralise la notion classique de longueur (définie pour les modules à la fois nœthériens et artiniens, et en particulier pour les anneaux artiniens), mais avec une définition bien plus agréable, et des propriétés presque aussi sympathiques dans le cas infini (notamment, si 0 → M′ → MM″ → 0 est une suite exacte courte de modules nœthériens, la longueur de Gulliksen ℓ(M) de M est encadrée par la valeur de deux additions entre celles de M′ et M″, à savoir ℓ(M′) + ℓ(M″) ≤ ℓ(M) ≤ ℓ(M′) ⊞ ℓ(M″) où + désigne la somme usuelle des ordinaux, et ⊞ la somme naturelle ou somme de Hessenberg). Mais en même temps, la longueur de Gulliksen permet de retrouver la dimension (de Krull) d'un anneau, généralisée aux ordinaux non nécessairement finis : si on écrit la longueur de Gulliksen de M en forme normale de Cantor (c'est-à-dire en « base ω », voir par exemple cette entrée sur les ordinaux), alors le plus grand exposant de ω qui intervient définit la dimension de M — par exemple, la longueur de Gulliksen de k[t1,…,tn] vaut ωn. Entre autres propriétés dignes d'intérêt (elle n'est pas écrite noir sur blanc dans l'article de Gulliksen, mais elle s'en déduit assez facilement en considérant la dimension de Krull), un anneau [commutatif] nœthérien est intègre si et seulement si sa longueur de Gulliksen est une puissance de ω, ce qui est fort sympathique. Mieux, l'écriture en forme normale de Cantor de la longueur de Gulliksen d'un anneau [commutatif] R se relie à la décomposition primaire de R.

Je trouve la longueur de Gulliksen — et son écriture en forme normale de Cantor — beaucoup plus naturelle et élégante que la fonction de Hilbert-Samuel, et que la définition classique de la dimension de Krull : à mon avis, il serait profitable de s'en servir dans toute introduction ou tout livre sur l'algèbre commutative. Le fait que le concept ait été peu développé est sans doute le signe que les algébristes n'aiment pas les ordinaux (ou l'infini qu'ils ne contrôlent pas bien), ce qui est vraiment dommage.

Une chose qui me chagrine, cependant, c'est qu'on manque d'exemples d'anneaux nœthériens de dimension de Krull arbitraire (infinie) : essentiellement, je connais une construction, due à Nagata, qui a été raffinée par le même Gulliksen pour fabriquer des anneaux de dimension de Krull un ordinal quelconque (et du coup, de façon facile, de longueur de Gulliksen un ordinal quelconque) — cette construction n'est sans doute pas aussi simple qu'on voudrait, et, en tout cas, on manque (ou du moins, je manque) de variété dans les exemples.

↑Entry #2210 [older| permalink|newer] / ↑Entrée #2210 [précédente| permalien|suivante] ↑

↓Entry #2209 [older| permalink|newer] / ↓Entrée #2209 [précédente| permalien|suivante] ↓

(mardi)

De la force de Coq et d'autres systèmes, et de l'utilité de mettre les résultats mathématiques en contexte

À cause de la combinaison entre l'écriture de l'entrée précédente et de mon interaction avec des (enfin, surtout un) mathématicien constructiviste à la Bishop/Richman, j'ai tenté de me faire une idée sur la force logique des systèmes admis par les constructivistes. (L'idée est que — pour une raison qu'on ne comprend pas vraiment, mais que je suis tenté de prendre pour un indice empirique de l'existence platonique des entiers — toutes les théories logiques introduites « naturellement » en mathématiques semblent s'arranger selon une échelle totalement ordonnée de « force » convenablement définie. Je voulais savoir où, sur cette échelle, se situent les différents cadres des mathématiques constructives. On m'a recommandé de lire le texte introductif de Martin-Löf The Hilbert-Brouwer controversy resolved? — mais au final il me suggère plus de questions qu'il n'en clôt.) Mauvaise idée : je me suis retrouvé dans un labyrinthe de petits énoncés tordus, tous semblables — et surtout, de gens qui ne communiquent pas assez entre eux, et qui ne présentent pas leurs résultats dans le contexte des autres résultats du même genre.

Certes, le problème n'est pas évident, pour plusieurs raisons :

  • D'abord, ce n'est pas évident de définir ce qu'on appelle la force logique d'un système (j'agite pas mal les mains en parlant de ça). Il n'est pas du tout vrai que deux théories logiques naturelles T et T′ soient toujours comparables si on considère toutes leurs conséquences (i.e., il n'est pas vrai que soit toute conséquence de T découle aussi de T′ soit le contraire ; par exemple, la théorie des ensembles de Zermelo avec l'axiome du choix, ZC, est incomparable avec la théorie des ensembles de Zermelo-Fraenkel sans l'axiome du choix, ZF). Il faut pour cela se limiter au moins aux énoncés arithmétiques, sans doute même aux énoncés arithmétiques Π₂ (c'est-à-dire du type <telle> machine de Turing termine pour toute entrée) ou Π₁ (c'est-à-dire du type <telle> machine de Turing ne termine jamais), ou peut-être utiliser la relation une théorie très faible montre que si un énoncé Σ₁ (c'est-à-dire du type <telle> machine de Turing termine sur telle entrée) est démontré par T alors il l'est par T, voire une théorie très faible montre que si T est inconsistante alors T′ l'est, et si possible en explicitant un lien algorithmique entre une preuve effectuée dans T et une preuve dans T′. Je n'ai pas les idées totalement claires sur les rapports entre tous ces concepts (premier labyrinthe !), et même si je crois qu'au moins dans les cas qui m'intéressent cela ne fait aucune différence, je n'en suis pas totalement sûr. Premier reproche : les gens qui énoncent des comparaisons entre théories ne sont pas toujours très clairs sur ce qu'ils veulent dire, et en tout cas ne font pas du tout l'effort de remettre en contexte ce qu'on peut dire sur ces différentes relations. (À toutes fins utiles, je note quand même que le texte le plus clair que j'aie trouvé qui explique un peu comment les choses s'articulent est l'article introductif The Realm of Ordinal Analysis de Michael Rathjen — cf. notamment les définitions 2.4, 2.14, 2.15 et la remarque 2.16.)
  • De même, système admis par les constructivistes n'est pas bien défini, à part le fait qu'il doit s'agir de logique intuitionniste. En tout cas, moi, je ne comprends pas bien ce qui fait qu'on considère que tel ou tel postulat est constructif ou pas. (Comme je le mentionne ci-dessous, je crois comprendre de façon détournée que l'ensemble des parties d'un singleton existe ne peut pas être constructif, mais j'ai un peu du mal à voir ce qu'il y a de non constructif à l'ensemble des parties d'un singleton, ou en tout cas au simple postulat de son existence !) Ce qui est sûr, c'est qu'il y a un autre labyrinthe ici, de théories logiques, de systèmes de typage et de systèmes de preuves, etc., et que je suis loin d'avoir une idée clair du rapport entre toutes ces choses. L'article de Barendregt Introduction to generalized type systems (où il introduit le fameux λ-cube, et un cube analogue de théories logiques), par exemple, est très bon pour se faire une idée rapide de ce qu'est un système de typage, mais on reste ignorant du rapport précis entre ces systèmes de typage et les théories des types introduites par Martin-Löf. Un des problèmes est probablement que les informaticiens théoriciens et les matheux ne se parlent pas suffisamment, et disent des choses semblables dans des langages différents, ce qui fait qu'il est difficile de relier ce que disent les uns et ce que disent les autres.
  • L'ajout d'un axiome en apparence innocent peut changer complètement la force d'un système. (Parfois il peut même le rendre inconsistant : voir le paradoxe de Girard, expliqué par exemple ici, qui est une sorte de version du paradoxe de Burali-Forti en théorie des types. Pour le résoudre, on a tendance à introduire des notions d'univers, typiquement une suite infinie d'univers emboîtés, un peu comme Grothendieck l'a fait pour se débarrasser de difficultés en théorie des catégories, et ce genre de postulat n'est évidemment pas innocent du point de vue de la force logique.) Ainsi, l'axiome du tiers exclu (qui change la logique intuitionniste en logique classique) ne se contente pas de briser le constructivisme, il peut aussi augmenter considérablement la force logique d'une théorie — j'ai mis longtemps à le comprendre, surtout que superficiellement ceci semble contradictoire avec la traduction de Dragalin-Friedman. C'est le cas notamment de la théorie CZF, ou ZF constructif (décrite en détails dans ces notes sur CZF par Aczel et Rathjen), une des théories standard pour les maths constructives : toute seule, elle a la même force arithmétique Π₂ qu'une théorie classique assez faible, KP (je n'ai d'ailleurs pas de bonne référence pour ce fait, mais voir page 24 du texte de Rathjen que j'ai déjà cité), alors que quand on ajoute l'axiome du tiers exclu elle devient équivalente à ZF, qui est très forte (voir la proposition 7.5 des notes d'Aczel et Rathjen citées ci-dessus). De façon encore plus surprenante, l'axiome l'ensemble des parties d'un singleton existe (i.e., la classe des sous-ensembles de {∅} est un ensemble) change considérablement la force arithmétique de CZF (voir la preuve de 7.3(ii) dans le texte d'Aczel et Rathjen et le paragraphe ci-dessous). Et je crois comprendre que pour des théories des types (comme le calcul des constructions) c'est le cas de l'axiome d'irrelevance des preuves, ou de l'axiome du choix (alors que par rapport à ZF classique, l'axiome du choix apporte une force logique nulle : par un argument de Gödel, tout énoncé aritmétique prouvable dans ZFC l'est en fait dans ZF).

Une des choses que j'aimerais comprendre, par exemple, c'est quelle est la force logique du calcul des constructions inductives (une extension du calcul des constructions qui se situe au coin le plus complexe du cube de Barendregt mentionné ci-dessus) et du système Coq qui se base dessus. Et aussi de savoir si on doit le considérer comme « constructif ». (La réponse à ces deux questions dépendra sans doute, et de façon subtile, de ce qu'on met dedans : il est certain que l'ajout du tiers exclu augmente énormément la force logique, par exemple, mais j'ai les idées beaucoup moins claires sur l'introduction du type Prop « imprédicatif » ou de l'irrelevance des preuves.) J'ai toutes sortes de réponses partielles à ces questions, mais surtout un grand mal à les relier les unes aux autres, de nouveau, parce que les gens qui ont écrit ces réponses ne se citent pas les uns les autres pour expliquer le lien entre ce qu'ils disent. Pour commencer, j'apprends dans un vieil article de B. Werner intitulé Sets in Types, Types in Sets que Coq avec ω univers est (co-interprétable, donc) équiconsistant avec ZFC avec ω univers (de Grothendieck, i.e., cardinaux inaccessibles) — sauf qu'en fait, en lisant bien, on voit que c'est après ajout de l'axiome du tiers exclu (et peut-être un autre axiome bizarre), donc ça ne m'apprend qu'une borne supérieure (très faible) sur la force de Coq sans le tiers exclu. Un article de Rathjen intitulé Constructive Zermelo-Fraenkel Set Theory, Power Set, and the Calculus of Constructions (publié dans un volume en l'honneur de Martin-Löf) m'apprend, si je lis bien !, qu'une certaine théorie basée sur le calcul des constructions (et/ou la théorie des types de Martin-Löf — comme je l'ai dit, je ne comprends pas bien le rapport entre elles), comportant une règle d'« irrelevance des preuves », a une force logique équivalente à la fois (1) à CZF + l'axiome d'existence de l'ensemble des parties [d'un singleton, cela suffit], (2) à la théorie classique Power-KP (essentiellement, Kripke-Platek si on ajoute la fonction « ensemble des parties » au langage), ou encore (3) à la théorie des ensembles classique de Zermelo à laquelle on ajoute un nombre d'univers égal à l'ordinal de Bachmann-Howard. La thèse d'Alexandre Miquel émet (conjecture 9.7.6) une supposition qui pourrait sembler contradictoire avec ça, mais peut-être pas parce qu'il y a toutes sortes de subtilités techniques qui diffèrent entre les théories comparées (en tout cas, les deux sont d'accord sur le fait que la force logique dépasse celle de la théorie des ensembles de Zermelo) — en revanche, je ne comprends pas si l'axiome d'irrelevance des preuves a dû être postulé pour obtenir la borne inférieure. En tout cas, il s'agit de théories assez « fortes » car elles dépassent l'arithmétique du second ordre (qualifiée de fossé infranchissable dans le texte de Martin-Löf cité tout au début). A contrario, j'ai trouvé un texte d'Aczel, On Relating Type Theories and Set Theories ainsi qu'un plus vieux texte de Rathjen, The strength of Some Martin-Löf Type Theories, qui arrivent à la conclusion que diverses théories des types entre lesquelles je m'embrouille complètement sont, pour leur part, d'une force logique très modeste (en-deçà du fossé infranchissable évoqué par Martin-Löf). La différence doit donc bien être dans l'existence de l'ensemble des parties [d'un singleton], dans le type Prop de Coq que différents auteurs qualifient d'« imprédicatif » même si j'avoue ne jamais avoir compris ce que ce mot veut dire, et/ou dans l'irrelevance des preuves.

Mais bon, trève de détails techniques (que j'avoue avoir écrits surtout pour m'en souvenir plus tard) : ce dont je veux surtout me plaindre et de la façon dont les gens ne communiquent pas assez. Par exemple, j'ai trouvé extrêmement peu d'arêtes pour la relation être cité par entre les équipes d'informaticiens qui gravitent autour de Coq (du genre, B. Werner) et les équipes de matheux qui font de la théorie ordinale de la démonstration (comme le M. Rathjen que j'ai cité plusieurs fois ci-dessus, et dont les articles répondent très souvent aux questions que je me pose en théorie de la démonstration) : pourtant, ces deux groupes de gens font de la logique parfois intuitionniste et notamment de la théorie de la démonstration ; et pourtant, il est essentiel pour bien faire comprendre ses résultats de les mettre en perspective par rapport à d'autres résultats du même genre. Ceci me rappelle cette citation de Giancarlo Rota :

A leader in the theory of pseudo-parabolic partial differential equations in quasi-convex domains will not stoop to being understood by specialists in quasi-parabolic partial differential equations in pseudo-convex domains.

— Indiscrete Thoughts (XXI. Book reviews: Professor Neanderthal's World)

Résultat, moi qui ne suis spécialiste ni des équations différentielles pseudo-paraboliques dans les domaines quasi-convexes ni des équations différentielles quasi-paraboliques dans les domaines pseudo-convexes, je dois m'arracher les cheveux à me demander quel est le rapport entre tel résultat de la première théorie et tel résultat apparemment très semblable de la seconde, sachant qu'aucun ne mentionne l'autre pour m'éclairer sur le sujet.

[Ajout : il y a différents compléments dans les commentaires, grâce aux explications gentiment fournies par Arnaud Spiwack ; la moralité est qu'en tant que mathématicien classique non habitué aux maths constructives et/ou précatives, je m'embrouille complètement sur les nuances entre ce que ces différentes théories intuitionnistes prouvent ou interprètent (quelle est la force supplémentaire apportée par l'axiome du tiers exclu, par celle de l'axiome 0≠1, auquel je n'avais pas du tout pensé, la force de leur fragment de double négation, la difficulté à montrer leur cohérence versus leur normalisation : tout ça est très confus pour moi). Mais je retiens quand même que la comparaison entre Coq et ZFC n'est pas claire en l'état actuel des choses.]

↑Entry #2209 [older| permalink|newer] / ↑Entrée #2209 [précédente| permalien|suivante] ↑

↓Entry #2208 [older| permalink|newer] / ↓Entrée #2208 [précédente| permalien|suivante] ↓

(dimanche)

Comment calculer un grand nombre

J'ai déjà exploré assez en détail le sujet des (très très) grands nombres. Je ne vais pas revenir sur tout ce que j'ai dit (et comme d'habitude, je tenterai de ne garder mes posts indépendants les uns des autres), mais je veux me servir de cette question pour illustrer quelques faits de logique rigolo.

[Ajout : voir aussi ce message sur Reddit où j'explique de façon peut-être plus pédagogique (mais en anglais) en gros les mêmes choses que ci-dessous.]

Imaginons qu'un génie pervers nous mette devant un ordinateur et nous donne la tâche d'écrire — dans un langage de programmation idéalisé de notre choix — un programme qui calcule le nombre le plus grand possible avant de s'arrêter. (Le programme en question tournera sur l'Infinitiplex du génie, équivalent à une machine de Turing qui dispose de ressources de calcul illimitées : donc ni le temps de calcul ni la mémoire consommée ne sont à prendre en compte, seul importe le nombre renvoyé ; en revanche, évidemment, la taille du programme doit rester humainement gérable ; par ailleurs, le programme doit effectuer un calcul déterministe et s'arrêter de lui-même, sans intervention extérieure.)

↑Entry #2208 [older| permalink|newer] / ↑Entrée #2208 [précédente| permalien|suivante] ↑

↓Entry #2202 [older| permalink|newer] / ↓Entrée #2202 [précédente| permalien|suivante] ↓

(lundi)

Faut-il communiquer sur l'intuition en mathématiques ? — ici : le corps de classes

Une question qui fait régulièrement débat en ce qui concerne la rédaction mathématique est de savoir si l'auteur d'un livre ou article mathématique doit se contenter de définir des concepts et démontrer leur propriété ou si (ou plutôt, dans quelle mesure) il doit tenter de proposer une façon de les visualiser intuitivement et guider le lecteur sur la manière d'y penser.

Il va de soi qu'avec une formulation aussi générale, la réponse est difficile à donner. Tout le monde sera sans doute d'accord sur le fait qu'une définition vraiment bizarre ou surprenante, une clause qui risque particulièrement de prêter à confusion, une subtilité dans une démonstration qui pourrait ne pas être remarquée, etc., méritent d'être signalées ou expliquées. À l'inverse, tenter de communiquer toute intuition vague n'est pas forcément bénéfique et peut même être néfaste à la compréhension (car l'intuition qu'on se forge soi-même peut être meilleure que celle qu'on reçoit d'un autre mathématicien), ou à la détection d'erreurs de raisonnement (si on fait confiance à l'intuition d'un autre, on risque de faire les mêmes erreurs que lui, et donc de ne pas détecter celles-ci). Quelque part entre les deux, je trouve toujours irritant, quand un objet mathématique est défini dans un texte, de ne pas trouver la réponse aux questions les plus naturelles qu'on peut se poser sur ses propriétés (ou simplement l'affirmation que l'auteur ne sait pas si telle ou telle propriété est vraie) : par exemple, si un auteur devait définir un concept appelé para-anneau, je trouve qu'il serait de son devoir d'expliquer le rapport entre ce concept et celui d'anneau (et même si c'est complètement évident, écrire qu'un anneau est un para-anneau, ou attirer l'attention sur le fait que ce n'est pas le cas, ou peut-être dire qu'on ne sait pas et que de toute façon on n'en aura pas besoin, ou ce genre de choses) ; et si on met plusieurs clauses dans une définition, je trouve qu'il est généralement de bon ton d'expliquer pourquoi chacune est nécessaire et ce qui se passerait si on omettait celle-ci ou celle-là.

Je vais maintenant me plaindre de la façon dont est présentée la théorie globale du corps de classes. [Je suis sûr qu'il devait y avoir un rapport entre ce qui suit et ce qui précède, mais plus j'écris moins ce rapport est clair… enfin, ce n'est pas bien grave.]

En bref : la théorie du corps de classes prétend « expliquer » (c'est-à-dire décrire, classifier, permettre de comprendre) les extensions abéliennes finies (extension abélienne = extension [de corps] galoisienne de groupe de Galois commutatif) de certains corps. « Certains corps », à savoir, les « corps locaux » (auquel cas on parle de théorie locale du corps de classes) et les « corps globaux » (auquel cas, on l'aura deviné, on parle de théorie globale du corps de classes, qui est beaucoup plus intéressante et profonde que la théorie globale locale). Les corps locaux sont des choses comme les corps des réels et des complexes (mais sur ceux-ci la théorie est vraiment triviale), les corps des nombres p-adiques (et les extensions finies de ceux-ci) et les corps de séries formelles sur un corps fini. Des exemples de corps globaux sont le corps des rationnels (ou plus généralement toute extension finie de celui-ci, dit « corps de nombres ») et le corps des fonctions rationnelles sur un corps fini (ou plus généralement le corps des fonctions d'une courbe algébrique sur un corps fini).

↑Entry #2202 [older| permalink|newer] / ↑Entrée #2202 [précédente| permalien|suivante] ↑

↓Entry #2200 [older| permalink|newer] / ↓Entrée #2200 [précédente| permalien|suivante] ↓

(jeudi)

Sur la réalité des quaternions, quasars, quarks et quaggas

J'ai une fois de plus commis l'erreur de commencer (il y a deux-trois semaines) l'écriture d'une entrée que je pensais pouvoir faire courte et qui a grandi, grandi, grandi, jusqu'à prendre des proportions totalement délirantes. Comme je vais être assez débordé ces prochaines semaines, elle risque de rester indéfiniment dans les limbes, là où j'ai déjà mis tout ce que j'ai écrit sur les octonions et tant d'autres choses. Tout ceci m'énerve prodigieusement, et je ne sais pas quoi faire pour réussir à éviter ce problème.

Pour me distraire un peu, je voudrais juste faire une remarque sur la philosophie des mathématiques. Comme il n'aura échappé à personne, je suis férocement platoniste (au moins en ce qui concerne l'arithmétique), où par platonisme (voir aussi cette entrée, et notamment cette petite section de celle-ci pour plus de détails) j'entends le point de vue selon lequel les concepts mathématiques, ou au moins les plus « naturels » d'entre eux, ont une existence autonome, indépendante de l'esprit humain, qui ne fait que les découvrir, et même indépendante de l'univers matériel. (Il y a, bien sûr, toutes sortes de variantes[#][#2] de cette position, et on peut être d'accord avec certaines sans être d'accord avec d'autres, on peut d'ailleurs aussi considérer qu'il ne s'agit pas vraiment d'une différence d'opinion philosophique mais simplement de façon de dire les choses. À ce sujet, voir aussi cette autre entrée.)

Je crois avoir lu quelque part (mais je ne sais plus si un sondage précis à été fait dans ce sens ou si cette affirmation sortait d'un grand chapeau de magicien) que la majorité des mathématiciens, et l'écrasante majorité des logiciens, adhère au moins à une forme modérée de platonisme. A contrario, les neurologues semblent généralement persuadés (là aussi, il s'agit d'une statistique qui, comme 83.28% des statistiques, est purement et simplement inventée) que les mathématiques sont uniquement le résultat de processus cognitifs dans le cerveau humain et n'ont rien de « réel » ou d'« universel » (pas plus que, disons, la beauté de la musique de Bach).

Les arguments les plus souvent invoqués contre le platonisme mathématique, c'est-à-dire, pour montrer que les mathématiques viennent de l'esprit humain et pas d'un « paradis platonique », sont typiquement d'observer que les mathématiques ne sont pratiquées que par les humains (le summum des facultés mathématiques des animaux se limitant à savoir compter sur de tout petits entiers naturels), et aussi que celles-ci ont changé au cours de l'histoire (ce qui est de mauvais augure pour la découverte d'un monde censément intemporel).

Mais une chose que je ne comprends pas est pourquoi ce genre d'arguments, invoqué pour dire chers mathématiciens, les quaternions n'existent que dans votre cerveau ne l'est pas aussi pour dire chers astrophysiciens, les quasars n'existent que dans votre cerveau ou chers physiciens des particules, les quarks n'existent que dans votre cerveau, voire, chers zoologistes, les quaggas n'existent que dans votre cerveau. Après tout, si le problème est qu'on ne peut pas toucher un quaternion, qu'on ne peut les détecter qu'indirectement par le truchement de théories qui prédisent leur existence, et que seuls les humains sur cette Terre ont le moindre concept de quaternions dans leur tête, et encore, seulement depuis quelques siècles, exactement la même chose vaut pour les quarks et les quasars : jamais je ne pourrai toucher un quark ou un quasar, aucun animal autre que l'homme n'a affaire à eux ou de représentation mentale de ces choses-là, il y a simplement des scientifiques qui nous disent mon accélérateur de particules a vu trois quarks dans chaque proton, mon radiotélescope a détecté un quasar dans telle direction, mes calculs ont exhibé une structure abstraite de dimension 4 sur les réels qui se comporte comme une algèbre à divisions. Même les quaggas, je n'en ai, après tout, jamais touché, et comme c'est une espèce éteinte ça ne risque pas de se produire, et j'ai beau avoir des témoignages de gens qui en ont dessiné ou de biologistes qui assurent que ces bestioles ont existé, je ne vois pas pourquoi ils seraient plus (ou moins) crédibles que les physiciens qui disent que les quarks et les quasars existent ou les mathématiciens qui disent que les quaternions existent.

Or j'ai rarement entendu des gens transposer à la physique ou à d'autres domaines la position anti-platoniste qu'ils peuvent avoir au sujet des mathématiques. Y a-t-il des neurologues qui disent aux physiciens ce que vous appelez étoile à neutron n'est que le fruit de vos processus cognitifs [remplacer étoile à neutron par n'importe quoi de difficile à imaginer] ? Voire, qui disent aux autres neurologues ce que vous appelez neurone n'est que le fruit de vos processus cognitifs (de nouveau, on ne peut pas toucher directement un neurone, il faut faire confiance à la théorie du microscope).

Les tenants du platonisme mathématique sont souvent décriés comme religieux, parce qu'ils croient en une sorte de perfection intangible et accessible uniquement par l'esprit (et c'est vrai que le choix d'un terme comme paradis platonique n'est certainement pas pour aider). Je ne sais pas pourquoi ce reproche n'est pas fait aux physiciens des particules[#3] qui prétendent que les quarks et atomes sont vraiment les composants de toute notre matière.

Au contraire, l'attitude consistant à dire je ne crois réel que ce que je peux toucher (et notamment sa variante ultrafinitiste, les grands nombres n'ont pas de sens parce que je ne peux pas les voir) me semble être exactement la même que ceux qui prétendent je ne peux pas croire que la Terre soit plus vieille que 6000 ans environ, parce que je n'ai que des preuves indirectes des millions d'années censées nous avoir précédées. À partir du moment où on accepte l'épistémologie des mathématiques, sa démarche scientifique pour arriver à une vérité (et il est difficile de la nier compte tenu de l'extrême utilité pratique des mathématiques, entre la construction des ponts et la cryptographie !), il faut bien reconnaître qu'elles nous renseignent sur quelque chose qui n'est pas sujet à notre bon vouloir comme la poésie ou la musique — et qu'elles sont, sur le même plan, que les autres sciences, une entreprise visant à découvrir systématiquement une réalité préexistante.

Bref, je comprends la position extrême je ne crois réel que ce que je peux directement toucher (donc je ne crois réels ni les quaternions, ni les quarks, ni les quasars, ni les quaggas, ni la planète Jupiter, ni les virus, ni Louis XIV), mais je ne comprends pas ceux qui l'appliquent uniquement pour les mathématiques et aucune autre science.

[#] Par exemple, je suis tenté de distinguer le platonisme structural, qui serait la position selon laquelle les structures que nous pensons discerner (les groupes, par exemple) sont « naturelles » et « découvertes », et le platonisme logique, orthogonal, qui serait la position selon laquelle les fondements mêmes des mathématiques (entiers naturels ou ensembles), sur lesquels on pose ces structures, ont une existence. Essentiellement, le platonisme structural affirmerait qu'on découvre les définitions tandis que le platonisme logique affirmerait qu'on découvre les axiomes. On peut parfaitement croire à l'un mais pas à l'autre (que la définition d'un groupe est naturelle, mais que les mathématiques n'aient pas de fondement logique immatériel) ou à l'autre mais pas à l'un (que les entiers naturels ou une forme quelconque de monde platonique préexistent à l'univers matériel, mais que la façon dont on le structure est profondément humaine), ou au deux, ou ni à l'un ni à l'autre.

[#2] Concernant ce que j'appelle le platonisme logique dans la note précédente, on peut aussi tenir toutes sortes de positions, par exemple l'idée que les entiers naturels (ou n'importe quoi qui permet d'encoder les structures finies) ont une position spéciale et sont le seul substrat ayant une réelle « existence platonique », ou bien étendre cette position à des objets plus complexes, comme les ensembles d'entiers naturels, ou tous les ensembles (un platoniste ensembliste doit croire que l'hypothèse du continu à une valeur de vérité bien définie, même si les axiomes de ZFC ne permettent pas de la trouver) ; à l'inverse, s'agissant des ensembles, on peut croire à un multivers platonique plutôt qu'un univers platonique.

[#3] Peut-être parce que les physiciens des particules ont des détecteurs expérimentaux ? Mais les mathématiques expérimentales existent aussi, et je ne vois pas de différence importante entre faire s'entrechoquer des protons à haute énergie pour chercher à trouver, peut-être, un compagnon supersymétrique, ou bien faire tourner des ordinateurs à calculer les valeurs de la fonction ζ pour chercher à trouver, peut-être, un zéro non-trivial qui ne soit pas sur l'axe critique.

↑Entry #2200 [older| permalink|newer] / ↑Entrée #2200 [précédente| permalien|suivante] ↑

↓Entry #2189 [older| permalink|newer] / ↓Entrée #2189 [précédente| permalien|suivante] ↓

(jeudi)

L'étrange multiplication du bidual

Encore une surprise mathématique qui m'a fait tomber à la renverse…

Si k est un anneau commutatif et M un k-module (si ça peut aider certains à comprendre, on peut déjà imaginer le cas où k est un corps et M un k-espace vectoriel ; mais dans ce cas il faudra considérer M de dimension infinie pour que ce soit intéressant), on appelle dual de M, disons D(M), l'ensemble des applications linéaires Mk. Et on appelle bidual de M le dual du dual, DD(M). Il y a une application k-linéaire naturelle Φ:MDD(M) (à savoir x ↦ (uu(x))) : en général, elle n'est ni injective ni surjective (mais si k est un corps, elle est toujours injective, et si de plus M est un espace vectoriel de dimension finie sur k, elle est aussi surjective).

Maintenant supposons que A soit une algèbre commutative sur k (c'est-à-dire un k-module qui est aussi un anneau avec la même addition et une multiplication k-bilinéaire). Si on essaie de trouver une multiplication naturelle sur DD(A), on s'aperçoit que ce n'est pas du tout évident. Dans ce genre de situation, le réflexe mathématique est que les choses doivent être soit tout à fait évidentes soit impossibles. Pourtant, il existe bien une multiplication naturelle sur DD(A) : précisément, si ξ et η sont deux éléments de DD(A), on peut définir leur produit ξη comme l'application qui envoie uD(A) sur η(yξ(xu(x·y))) (ceci est bien un élément de k). C'est assez difficile à visualiser, mais rien qu'au niveau du typage on peut être assez convaincu que c'est la seule formule possible. Et entre autres propriétés, cette multiplication est k-bilinéaire, associative, et vérifie : Φ(a)•η = ηΦ(a) est la fonction uη(yu(a·y)), et en particulier, Φ(a)•Φ(b) = Φ(a·b). Je pense que la grande majorité des mathématiciens sera d'accord que dès lors qu'on a trouvé une formule simple, naturelle, qui définit sur DD(A) une structure de k-algèbre, et qui respecte Φ comme je viens de le dire, c'est forcément « la bonne » multiplication sur DD(A).

Eh bien ce qui surprendra certainement bon nombre de mathématiciens comme ça m'a surpris moi-même, c'est que cette multiplication n'est pas commutative en général !

Et ce n'est même pas facile du tout de donner un contre-exemple. (Déjà, il faut bien sûr un exemple où Φ n'est pas surjectif, puisque j'ai expliqué que les Φ(a) commutent les uns aux autres ; or si c'est le cas — grâce à l'axiome du choix — pour tout espace vectoriel de dimension infinie sur un corps k, le problème est qu'« on n'y voit rien » aux éléments du bidual qui ne viennent pas du primal.) Le contre-exemple que j'ai, c'est que si k est un corps fini et A=k[t] l'algèbre des polynômes en une variable t sur k, de sorte qu'en tant que k-espace vectoriel, A est la somme directe d'un nombre dénombrable de copies de k, et son dual est le produit d'un nombre dénombrable de copies de k, c'est-à-dire les suites à valeurs dans k : le bidual de A contient les fonctions qui à une suite u à valeurs dans k associent sa limite prise selon un ultrafiltre sur ℕ, et le produit de deux tels éléments (pour la multiplication que j'ai explicitée) est la limite selon l'ultrafiltre somme, pour l'addition usuelle sur le compactifié de Stone-Čech de ℕ — or il est « bien connu » que cette addition n'est pas commutative en général. (Si vous n'y voyez rien, c'est normal. Moi non plus.)

J'ai posé la question sur MathOverflow de trouver des critères intéressants pour que DD(A) soit commutatif, ou bien un exemple plus simple où il ne l'est pas. Je n'espère pas vraiment de réponse (et par ailleurs j'ai moi-même résolu le cas qui m'avait amené à m'intéresser à cette question : si A est finie sur un anneau k noethérien intègre), mais à vrai dire je serais déjà content si on pouvait me dire que cette multiplication a été explicitée à tel ou tel endroit dans la littérature. (Ah, en fait, on me souffle que ça s'appelle la multiplication d'Arens, au moins dans le contexte des algèbres de Banach.)

↑Entry #2189 [older| permalink|newer] / ↑Entrée #2189 [précédente| permalien|suivante] ↑

↓Entry #2179 [older| permalink|newer] / ↓Entrée #2179 [précédente| permalien|suivante] ↓

(mardi)

Qu'est-ce qu'une géométrie, et autres pensées de fin d'année

J'ai passé la fin de la semaine dernière principalement à comater (pendant que mon poussinet était parti rendre visite à sa tante à Majorque) : c'est surtout en voyant combien j'ai dormi que je prends conscience de combien j'étais fatigué. En particulier, je n'ai pas continué ma série sur la géométrie hyperbolique, et il est probable que ma motivation à le faire décroisse rapidement avec le temps (j'ai déjà parlé de la nature épisodique de mes intérêts ? je ne retrouve pas). Encore que je voudrais vraiment trouver le moment de comprendre un peu en profondeur ces histoires d'automates finis en lien avec les groupes de Coxeter (intéressants dans le cas hyperbolique), et il faut que j'en discute avec mon voisin de bureau qui est spécialiste des automates. Bref.

Il y a une question qui m'a tracassé même dans ma torpeur, cependant, une question posée innocemment à propos de ma description faisant un grand parallèle entre la géométrie euclidienne, sphérique et hyperbolique : pourquoi n'y a-t-il que trois types de géométrie ? — en fait, s'il n'est pas très difficile de répondre à cette question en dimension 2, la question sous-jacente qui me tracasse, c'est surtout : qu'est-ce que c'est, au fait, une géométrie ? — et je dois avouer que je n'ai pas vraiment de réponse satisfaisante.

Ce n'est pas vraiment une question mathématique, ou en tout cas ce n'est pas une question mathématiquement bien formulée : c'est une question de trouver une définition satisfaisante, c'est-à-dire, puisque c'est moi qui me pose la question, une définition qui me satisfasse, qui satisfasse mon sens de l'esthétique mathématique. Comme je crois profondément à la philosophie proposée par notre maître Felix Klein, je suis persuadé que la bonne réponse est une certaine sorte de quotient d'un groupe de Lie (réel, entendu qu'on parle de géométries réelles), ou du moins quelque chose de fortement lié à ça.

[Attention, ce qui suit est un rant de matheux qui ne sait pas trop ce qu'il raconte. Faites semblant de comprendre même si ce n'est pas le cas : vous n'y perdrez pas grand-chose.]

Il y a des gens (je pense notamment à Sharpe, dans son livre Differential Geometry: Cartan's Generalization of Klein's Erlangen Program) qui prennent une définition très générale : une géométrie de Klein, c'est un quotient d'un groupe de Lie par un sous-groupe fermé ; le livre de Sharpe explique ensuite qu'il est possible de se servir de ces géométries de Klein comme des modèles pour définir les géométries de Cartan, qui sont des courbures de celles-ci, c'est-à-dire des espaces qui localement ressemblent à une géométrie de Klein modèle mais qui peuvent varier de point en point (si on part de la géométrie euclidienne comme géométrie modèle, on obtient la notion de variété riemannienne, mais on peut définir énormément d'autres types de structure, un pour chaque géométrie de Klein) : je trouve ça fascinant, ça fait des années que je me dis que je voudrais bien trouver le temps d'y réfléchir plus et d'essayer de comprendre comment me représenter toute cette zoologie de structures et comment elles interagissent entre elles, mais je trouve ça un chouïa trop général pour que ça corresponde à ce que je voulais ici (déjà, en dimension 2, ça fait un peu trop de géométries de Klein).

Il y a aussi la notion classique d'espace symétrique, qui peut être défini comme une variété riemannienne possédant suffisamment de « symétries » ou comme un quotient bien particulier d'un groupe de Lie : ces espaces sont complètement classifiés, indiscutablement ils sont d'une grande beauté conceptuelle, et on peut les regrouper en une famille plutôt « sphérique » (les espaces symétriques compacts) et une famille plutôt « hyperbolique » ; bref, c'est certainement une bonne partie de la réponse de ce que doit être une géométrie, mais on n'y trouve pas forcément la combinatoire (points, droites, etc.) que je m'attends à trouver dans une « géométrie ». Je ne suis donc pas non plus complètement satisfait par cette réponse-là.

En fait, je voudrais bien qu'il y ait une notion de géométrie associée à chaque groupe de Lie semi-simple de sorte qu'on ait un type d'objet pour chaque nœud du diagramme de Dynkin : la géométrie projective (sous un avatar ou un autre) serait celle associée à la série An, les différents nœuds du diagramme de Dynkin de An correspondant aux points, droites, plans, etc., jusqu'aux hyperplans ; la géométrie sphérique ou elliptique serait celle associée aux séries Bn et Dn, avec les points, droites, etc., jusqu'à la moitié de la dimension (au-delà c'est redondant parce que la polarité fondamentale met en dualité points et hyperplans, droites et plans de codimension 2, etc.). Et, pour reprendre ma fascination avec E₈, il devrait y avoir une géométrie exceptionnelle avec 8 types d'objets et certaines notions d'incidence entre elles (mais au lieu que ce soient points, droites, plans, etc., jusqu'à la dimension 7, il y aurait un type qui « brancherait » des autres comme sur le diagramme de Dynkin).

Ce genre de choses existe et est bien connu : ce sont les sous-groupes paraboliques maximaux du groupe de Lie, ou plutôt les quotients du groupe de Lie par ses sous-groupes paraboliques maximaux (ou les « géométries de Klein » données par ces quotients) : il y a bien un sous-groupe parabolique maximal par nœud du diagamme de Dynkin, et dans le cas de An ils définissent naturellement les points, droites, plans, etc. Ces objets, ou les géométries de Cartan modelées sur eux (en les « courbant ») sont étudiés et s'appellent les géométries paraboliques. Mais je ne suis toujours pas content, parce que dans le cas du groupe spécial orthogonal (Bn ou Dn), les quotients paraboliques correspondent pas aux points, droites, etc., de la sphère (d'ailleurs, dans le cas réel compact, ils sont tout simplement vides) : ce sont les points, droites, etc., isotropes, c'est-à-dire dans le cas hyperbolique les points idéaux, droites idéales, etc. — ce n'est pas ce que je veux. (Même si je ne sais pas exactement ce que je veux.) Je veux quelque chose qui ressemble plus aux espaces symétriques classiques, mais qui ait une combinatoire naturelle comme les nœuds du diagramme de Dynkin (peut-être des orbites d'éléments assez généraux dans les représentations fondamentales, mais je n'ai pas les idées super claires sur ce que ça peut être).

La réponse à ma question est peut-être dans le très remarquable (rien que par son prix !) livre de Boris Rosenfeld, Geometry of Lie Groups, mais il faut bien le dire, ce livre est aussi brouillon qu'il est fascinant, et plus je le regarde plus j'ai les idées confuses.

Ce qui est sûr, c'est que parmi les géométries intéressantes, il y a les espaces projectifs réels, complexes et quaternioniques (de toute dimension), et la droite et le plan projectif octonionique (sur les octonions, on ne peut pas faire plus qu'un plan projectif, parce que le théorème de Desargues est automatique à partir de la dimension 3), ainsi que leurs analogues hyperboliques ; le plan projectif octonionique a d'ailleurs un lien intéressant avec plusieurs groupes de Lie exceptionnels (au moins F₄ et E₆), comme il sera raconté dans ma mythique page sur les octonions quand j'aurai fini de l'écrire (mais ce n'est pas forcément si loin que ça, justement !). Il existe aussi une géométrie intéressante, que je ne connaissais pas du tout : celle dont les « points » sont des algèbres de nombres complexes dans les octonions (ou, de façon équivalente, la sphère des des octonions imaginaires purs de module 1) et dont les « droites » (sans doute pas le meilleur terme) sont des algèbres de quaternions dans les octonions (ce sont les quotients de G₂ par quelque chose comme SU₃ et Spin₃×Spin₃), ça a l'air extrêmement joli, et je ne sais pas du tout quel est son nom classique.

Bon enfin bref. Bonne année.

↑Entry #2179 [older| permalink|newer] / ↑Entrée #2179 [précédente| permalien|suivante] ↑

↓Entry #2176 [older| permalink|newer] / ↓Entrée #2176 [précédente| permalien|suivante] ↓

(samedi)

Surprises et bizarreries de la géométrie hyperbolique

Cette entrée fait suite à celle-ci et celle-là. Je promets que ce blog ne parlera pas de géométrie hyperbolique jusqu'à la fin des temps ! Mais il me reste encore un certain nombre de choses rigolotes à raconter. Notamment, ce que sont les cercles (cycles et horocyles) en géométrie hyperbolique et les trajectoires de rotations et translations ; comment les projections de Poincaré et de Beltrami-Klein sont reliées (et comment les réconcilier en projetant le plan hyperbolique sur une demi-sphère) ; ce que sont la pseudo-sphère et les surfaces de Dini et pourquoi elles nous permettent de voir un petit bout de la géométrie hyperbolique sur un modèle euclidien (mais juste un petit bout) ; et surtout, parler un peu des groupes de Coxeter et de leur automaticité, des pavages du plan hyperbolique, et de comment ceci permet de travailler informatiquement avec le plan hyperbolique, par exemple s'il s'agit de faire des jeux ou des simulations dessus. (Je ne garantis pas du tout que j'aurai le courage ou le temps de développer chacun de ces thèmes, mais ce serait bien.) De nouveau, je promets d'essayer de rendre les différentes entrées de cette série aussi indépendantes que possible les unes des autres, de façon à ce qu'on puisse lire le sous-ensemble qu'on veut dans l'ordre qu'on veut. Et je vais essayer de rendre autant que possible les choses accessibles à un bas niveau de connaissances mathématiques (même si je n'arriverai pas toujours à rester au niveau lycée), quitte à commettres des imprécisions que les experts, j'espère, me pardonneront.

[Navigation : cette entrée fait partie d'une série de trois sur la géométrie hyperbolique : 1, 2, 3 (plus ou moins indépendantes) ; voir aussi celle-ci pour des illustrations de différentes projections ; voir aussi les jeux de labyrinthe hyperbolique que j'introduis ici, et , et dont j'explique le fonctionnement dans des transparents disponibles ici.]

Dans cette entrée je voudrais essayer de décrire quelques phénomènes très simples qui illustrent le comportement surprenant de la géométrie hyperbolique. Les quatre sous-parties sont d'ailleurs indépendantes les unes des autres. Le principe informel mais fondamental que je cherche à illustrer (au moins sur les deux premiers exemples) est le suivant :

Dans un espace hyperbolique, il n'y a essentiellement qu'une façon de relier deux points : même un chemin en zigzag ne peut pas se racourcir beaucoup.

Autrement dit, si vous allez de A à B par un chemin en zigzag et que quelqu'un d'autre y va en ligne droite, il ne gagnera pas énormément par rapport à vous. Ou, pour dire les choses de façon plus décourageante, si vous avez marché un peu au hasard à partir de A, que vous vous retrouvez en B, et que maintenant vous voulez revenir à votre point de départ, vous allez devoir refaire toutes les méandres de votre chemin. Raconté de façon aussi informelle, ce principe peut être complètement faux, bien sûr (essentiellement parce que le plus court chemin entre deux points est une droite, pas un zigzag, même en géométrie hyperbolique), comme il peut être tout à fait vrai : mais si dans l'espace euclidien il n'y a aucun doute qu'il est faux, dans l'espace hyperbolique je vais essayer d'expliquer de quelle façon il est néanmoins vrai.

Ceux qui ont suivi mon conseil et essayé de jouer à HyperRogue se seront sans doute rendu compte à quel point il est difficile de retracer un chemin dans l'espace hyperbolique (surtout si vous êtes allés jusqu'au bout du jeu !) : voici donc une tentative d'explication de ces phénomènes. J'utiliserai les formules fondamentales de l'entrée précédente, mais si on ne l'a pas lue on peut se contenter de me croire sur parole quand j'affirme une relation métrique.

↑Entry #2176 [older| permalink|newer] / ↑Entrée #2176 [précédente| permalien|suivante] ↑

↓Entry #2175 [older| permalink|newer] / ↓Entrée #2175 [précédente| permalien|suivante] ↓

(mardi)

La formule fondamentale de la trigonométrie du triangle

Je continue une série commencée dans l'entrée précédente sur la géométrie elliptique et hyperbolique, mais, en fait, je vais tâcher de faire en sorte que ces entrées soient aussi indépendantes que possible, pour qu'on puisse les lire dans l'ordre qu'on veut. Je continue à me placer environ au niveau lycée (au moins dans les passages qui ne sont pas en petits caractères ; enfin, j'espère). Ici je veux parler un peu de la trigonométrie du triangle (la loi des cosinus et la loi des sinus). Dans une entrée ultérieure [mise à jour : elle est ], je parlerai des spécificités et bizarreries de la géométrie hyperbolique.

[Navigation : cette entrée fait partie d'une série de trois sur la géométrie hyperbolique : 1, 2, 3 (plus ou moins indépendantes) ; voir aussi celle-ci pour des illustrations de différentes projections ; voir aussi les jeux de labyrinthe hyperbolique que j'introduis ici, et , et dont j'explique le fonctionnement dans des transparents disponibles ici.]

A B C b a c α β γ

Considérons un triangle ABC (euclidien, sphérique ou hyperbolique, on verra plus tard) : j'appelle a la longueur BC (c'est-à-dire la longueur du côté opposé au sommet A), b la longueur CA, et c la longueur AB ; j'appelle α l'angle CAB (c'est-à-dire l'angle interne au sommet A), β l'angle ABC, et γ l'angle BCA. (On remarquera que ces notations, qui sont standard, sont symétriques si on permute les noms tant qu'on le fait de la même manière sur (A,B,C), (a,b,c) et (α,β,γ), et bien sûr tout ce qu'on pourra dire sur le triangle le sera aussi.)

On s'intéresse aux différentes relations qui peuvent exister entre les six quantités a,b,c et α,β,γ, et éventuellement, du coup, au problème de trouver les trois manquantes si on en connaît trois parmi les six (problème de la résolution du triangle).

Le triangle euclidien

Dans le cas euclidien, les trois angles d'un triangle ne sont pas libres : leur somme vaut π (radians, c'est-à-dire 180°) ; ceci sera la plus évidente différence avec le triangle sphérique ou hyperbolique.

Le théorème le plus célèbre de l'univers (sans doute le théorème que le plus grand nombre de personnes est capable de citer vaguement correctement) affirme que, en géométrie euclidienne, si le triangle est rectangle en C, c'est-à-dire si γ=π/2 (soit cos(γ)=0), alors c² = a² + b². Si on retire cette hypothèse d'être rectangle en C, la relation qui relie a, b, c et l'angle γ est la suivante :

c² = a² + b² − 2a·b·cos(γ)

↑Entry #2175 [older| permalink|newer] / ↑Entrée #2175 [précédente| permalien|suivante] ↑

↓Entry #2174 [older| permalink|newer] / ↓Entrée #2174 [précédente| permalien|suivante] ↓

(vendredi)

Quelques notions de géométrie sphérique et hyperbolique

Comme promis, je voudrais écrire un petit memento de géométrie, en insistant sur le parallèle entre la géométrie sphérique, la géométrie euclidienne, et la géométrie hyperbolique. Je précise que, pour une fois, je suppose très peu de connaissances mathématiques de mes lecteurs : juste un peu de géométrie euclidienne telle qu'on la voit au lycée. Un peu à la fin, et dans des entrées à venir, je supposerai qu'on sait ce que sont les lignes trigonométriques hyperboliques (cosh(t) = ½(exp(t)+exp(−t)) et sinh(t) = ½(exp(t)−exp(−t)), ainsi que tanh(t)=sinh(t)/cosh(t)), mais c'est essentiellement tout.

[Navigation : cette entrée fait partie d'une série de trois sur la géométrie hyperbolique : 1, 2, 3 (plus ou moins indépendantes) ; voir aussi celle-ci pour des illustrations de différentes projections ; voir aussi les jeux de labyrinthe hyperbolique que j'introduis ici, et , et dont j'explique le fonctionnement dans des transparents disponibles ici.]

La géométrie euclidienne, donc je suppose que tout le monde sait ce que c'est : c'est celle qu'on apprend au lycée, la géométrie du plan (pour se limiter à la dimension 2), les points et les droites, avec la notion de distance et d'angle.

(Digression : Si on oublie la notion de distance mais qu'on garde celle d'angle, on pourra parler de géométrie (euclidienne) conforme ; si on oublie à la fois la notion de distance et celle d'angle et qu'on ne garde que l'incidence — c'est-à-dire le fait qu'un point soit sur une droite ou qu'une droite passe par un point — et le parallélisme, alors on parle de géométrie affine. Si on oublie aussi le parallélisme, ne retenant que l'incidence, quitte à ajouter une droite à l'infini sur laquelle les droites parallèles sont réputées se croiser, on parle de géométrie projective. Dans l'esprit du fameux « programme d'Erlangen » de Felix Klein, à chacune de ces géométries est associé un groupe de transformations, c'est-à-dire celles qui préservent la géométrie en question du plan, à savoir le groupe des isométries planes pour la géométrie euclidienne, le groupe des similitudes pour la géométrie conforme, le groupe des affinités pour la géométrie affine, et le groupe des transformations projectives pour la géométrie projective. Il est intéressant de se rappeler ces différents niveaux, par exemple comme indiqué à la fin de cette entrée. Mais a priori, c'est bien avec la géométrie euclidienne classique, angles et distances, que je comparerai la géométrie sphérique et la géométrie hyperbolique.)

↑Entry #2174 [older| permalink|newer] / ↑Entrée #2174 [précédente| permalien|suivante] ↑

↓Entry #2170 [older| permalink|newer] / ↓Entrée #2170 [précédente| permalien|suivante] ↓

(jeudi)

Un labyrinthe hyperbolique, et une nouvelle vidéo

J'écrivais récemment un petit TODO pour plus tard. Il faut que je dise un peu ce que j'en ai fait.

Je me suis rendu compte que faire un labyrinthe hyperbolique était à la fois mathématiquement plus intéressant, et aussi plus facile, que ce que j'imaginais. En fait, j'ai eu une sorte d'épiphanie mathématique en réfléchissant à la question de savoir à la fois comment mettre des coordonnées sur un pavage comme ça (je veux dire quelque chose qui soit informatiquement manipulable et numériquement robuste, pour étiqueter les cases) et comment limiter la taille du labyrinthe. Comme Knuth l'a dit, on ne comprend vraiment bien un objet mathématique que quand on l'a enseigné, et on comprend encore mieux quand on l'a enseigné à un ordinateur.

(Ceux qui ne sont pas intéressés par les maths peuvent sauter les quelques paragraphes suivants.)

Quand on fait un jeu informatique sur le bête plan euclidien, pour ne pas aller à l'infini, parfois on met des bords, mais parfois aussi on préfère cycler dans les deux directions (i.e., quand on va trop loin, on retourne à son point de départ), ce qui revient en fait, mathématiquement, à quotienter le plan, et le réseau du pavage Λ (presque toujours un réseau carré en informatique, pour des raisons de simplicité du tracé) par un sous-réseau Γ des périodes (presque toujours aussi un réseau carré, même si pour le coup il n'y a guère de raison à ça), de sorte que Λ/Γ soit un groupe fini (un (ℤ/m)⊕(ℤ/n)), qui est celui où les coordonnées du jeu prennent leurs valeurs : du coup, les coordonnées sont des entiers modulo m et n (généralement deux puissances de 2, souvent égales, ce qui simplifie encore les choses), donc faciles à manipuler en informatique, et le monde est un quotient du plan par Γ, ce qu'on appelle un tore plat (ou, si on veut être sophistiqué, une surface de Riemann compacte de genre 1, c'est-à-dire une courbe elliptique, dont Γ est le groupe fondamental et dont Λ/Γ est un sous-groupe de points de torsion).

Bon, mais voilà, que faire pour le plan hyperbolique ? Contrairement au plan euclidien, ses translations ne commutent pas (c'est très clair quand on joue au jeu vers lequel je fais un lien ci-dessous) : on peut certes le paramétrer par deux coordonnées (par exemple les coordonnées polaires : la distance à une origine arbitrairement choisie et le cap/azimuth), mais ce sera des coordonnées réelles, peu pratiques à manipuler, et dès qu'on s'éloigne un peu de l'origine, elles deviennent numériquement délicates à gérer. Notamment, pour étiqueter les cases d'un pavage, ce n'est pas commode. Ce qui joue le rôle dans le cas hyperbolique du réseau du pavage dans le cas euclidien, c'est le groupe Δ des isométries du pavage (ou éventuellement le sous-groupe Δ⁰ des isométries directes) : c'est un groupe de Coxeter (en l'occurrence un groupe de triangle, qui, pour le pavage que j'ai choisi, est Δ(2,4,5), engendré par la réflexion par rapport à un mur du pavage, la rotation d'angle π/2 autour du centre d'un « carré » et la rotation d'angle 2π/5 autour d'un sommet). Ce qui permet déjà de le manipuler un peu informatiquement (il y a toutes sortes d'algorithmes pour calculer dans les groupes de Coxeter).

Mais surtout, ce qu'il y a, c'est qu'on peut aussi trouver, et de façon très agréable, des sous-groupes distingués Γ de Δ⁰ qui agissent sans point fixe, et de sorte que le quotient du plan hyperbolique par Γ soit compact (c'est une surface de Riemann compacte de genre ≥2) et notamment que Δ⁰/Γ soit fini (c'est le groupe des isométries de cette surface de Riemann). J'avoue n'avoir pas une idée aussi claire que je voudrais de comment décrire « tous » ces Γ, mais ce n'est pas difficile d'en trouver (en l'occurrence, j'ai écrit les matrices des isométries hyperboliques de mon pavage dans les nombres algébriques, j'ai trouvé un nombre premier p, en l'occurrence 89, qui scindait toutes ces matrices, et j'ai réduit modulo p). Du coup, ce qui joue le rôle analogue aux coordonnées cycliques (ℤ/m)⊕(ℤ/n) dans le cas d'un jeu euclidien, sur mon jeu hyperbolique, c'est le groupe Δ⁰/Γ, qui dans mon cas est PSL(2,89) (le groupe projectif spécial linéaire des matrices 2×2 sur le corps des entiers modulo 89 ; il a 352440 éléments), et le labyrinthe est en fait un sous-graphe du graphe de Cayley de ce groupe.

Voilà donc que j'ai figuré informatiquement, sans trop m'y attendre, trois objets mathématiques dignes d'intérêt : le plan hyperbolique, le graph de Cayley d'un groupe simple fini, et une surface de Riemann compacte de genre 8812 ayant ce groupe de symétries (et le plan hyperbolique comme revêtement universel, Γ étant le groupe fondamental).

(Il faudra que j'essaie voir avec un groupe Δ⁰/Γ plus petit — à commencer par trouver le plus petit possible, d'ailleurs — ce qui rendra le jeu moins intéressant mais peut-être la géométrie plus facile à visualiser. Une autre question sur laquelle je n'ai pas les idées parfaitement claires, c'est de savoir, si je voulais calculer les périodes de ma surface de Riemann, quelle serait la difficulté de l'opération.)

Bref, voici mon petit jeu de labyrinthe hyperbolique en JavaScript (qui devrait marcher sur les navigateurs vaguement récents ; mais n'essayez pas depuis un téléphone, d'une part parce qu'on joue avec les touches et d'autre part parce que les calculs sont un peu lents au démarrage et que ça consomme pas mal de mémoire).

Je l'ai présenté sous forme d'un jeu (il faut d'abord chercher à atteindre le cercle vert, puis revenir à son point de départ en ayant fait une « boucle » : c'est très facile si on se fie aux indications de distance données à droite, un peu plus difficile si on n'utilise que la couleur des cercles comme indication, et ce serait quasiment impossible s'il n'y avait rien du genre). Mais en fait l'intérêt est surtout d'explorer le plan hyperbolique et de se rendre compte comment il fonctionne. Par exemple, on peut chercher à se déplacer avec uniquement des « translations », sans jamais faire de rotation, et s'apercevoir qu'on peut revenir à son point de départ avec une différence d'orientation. On peut aussi s'amuser à essayer d'appliquer l'algorithme de la main droite (garder toujours un mur à droite et le suivre) et ceci donnera une idée de la vastesse du plan hyperbolique. Je trouve ça très instructif, et ce fut tout à fait agréable à programmer (une heureuse surprise).

🌍

Pour ce qui est des projections cartographiques, je n'ai pas calculé celle dont je parlais dans mon TODO, parce qu'elle ferait intervenir les fonctions hypergéométriques de façon pas du tout évidente, et je n'ai pas vraiment envie de me farcir le Abramowitz & Stegun pour un résultat incertain. En revanche, j'ai calculé les mêmes projections que dans ma vidéo précédente mais pour la Terre, c'est amusant à voir (et comme cette fois-ci je n'ai pas fait de commentaire audio, ça a été beaucoup plus rapide à faire) :

Allez, je termine par une vue de la Terre en projection stéréographique depuis le pôle sud sur une grande distance, parce qu'on n'a pas l'habitude de la voir comme ça, je trouve ça vraiment rigolo (et on comprend ce que ça veut dire qu'une projection conforme préserve les formes mais pas les tailles ; clickez pour zoomer) :

[La Terre vue en projection stéreographique depuis le pôle sud]

À ce propos, mon poussinet et moi avons cherché à trouver s'il y avait des vols aériens qui passent au-dessus de l'Antarctique : Wikipédia prétend que non, mais c'est au moins amusant, et étonnamment difficile, de chercher quelles lignes droites entre deux grandes villes sur Terre, passent au-dessus de l'Antarctique (nous avons trouvé, si je me rappelle bien : Sydney–Rio, Auckland–Le Cap, ou encore Buenos Aires–Shanghaï, mais aucune de ces liaisons n'existe en vol direct). Encore une illustration du fait qu'il est difficile de visualiser la géographie sphérique.

[Ajout : pour en savoir plus sur la géométrie hyperbolique, voir les entrées ultérieures suivantes : 1, 2, 3 (plus ou moins indépendantes), et celle-ci pour des illustrations de différentes projections ; voir aussi un autre jeu de labyrinthe hyperbolique que j'introduis et , et dont j'explique le fonctionnement dans des transparents disponibles ici.]

↑Entry #2170 [older| permalink|newer] / ↑Entrée #2170 [précédente| permalien|suivante] ↑

↓Entry #2168 [older| permalink|newer] / ↓Entrée #2168 [précédente| permalien|suivante] ↓

(mercredi)

Petit TODO pour plus tard

Deux idées que m'ont données les deux dernières entrées, à retenir pour plus tard, peut-être, un jour, si jamais j'ai du temps :

  • Combiner mon intérêt pour les labyrinthes avec celui des projections du plan hyperbolique, et faire un jeu de labyrinthe (en JavaScript) qui se joue dans le plan hyperbolique. Probablement sur ce pavage-ci (le pavage régulier du plan hyperbolique par des quadrangles — peut-on les appeler carrés ? — d'angle 72° à chaque sommet), qui a l'avantage qu'on pourrait jouer de façon évidente avec les flèches pour avancer ou reculer d'un quadrangle, ou tourner de 90° vers la droite ou la gauche. Il y a des chances que ce soit encore plus labyrinthique que mon labyrinthe 3D, et ça donnera peut-être une bonne intuition de la « vastesse » du plan hyperbolique. Si quelqu'un est motivé pour coder ça et a besoin que je lui explique comment on pourrait faire, qu'il me contacte !
  • Calculer le rendu de ma projection élue de la Terre, qui sera définie de la façon suivante : c'est une projection sur un patron d'icosaèdre (comme la Dymaxion), sauf que, pour chaque face de l'icosaèdre, la projection du triangle sphérique vers le triangle euclidien, plutôt qu'être la projection gnomonique ou la projection de la Dymaxion, sera l'unique application conforme (comme je l'ai expliqué) qui respecte les symétries du triangle. (J'ai pensé à un moment que ce serait une projection stéréographique par bouts, mais c'est idiot, ça ne peut évidemment pas être ça.) Bref, un peu comme la projection quinconciale de Peirce, mais en icosaèdre. Pareil, si quelqu'un se sent motivé pour appliquer la formule de Schwarz-Christoffel, qu'il se dénonce.

↑Entry #2168 [older| permalink|newer] / ↑Entrée #2168 [précédente| permalien|suivante] ↑

↓Entry #2167 [older| permalink|newer] / ↓Entrée #2167 [précédente| permalien|suivante] ↓

(mardi)

Visualisation de la sphère et du plan hyperbolique

Écrire l'entrée précédente m'a motivé pour faire quelque chose dont je traîne l'idée depuis longtemps : produire de jolies illustrations de quelques projections de la sphère et du plan hyperbolique, et des analogies entre elles.

[Pavage heptagonal du plan hyperbolique] [Pavage pentagonal de la sphère (dodécaèdre)]

On trouve beaucoup d'images et de vidéos des projections de la sphère, mais elles utilisent généralement l'image des continents de la Terre, parce qu'elles ciblent la cartographie, et pour cette raison aussi elles ont tendance à omettre la projection gnomonique, ce qui est dommage parce qu'elle est mathématiquement intéressante (elle met en lumière le fait que la sphère quotientée par l'antipodie donne un plan projectif réel tandis que la projection stéréographique illustre le fait que la sphère peut être vue comme une droite projective complexe).

Il y a aussi beaucoup d'images et de vidéos du plan hyperbolique, mais presque exclusivement en utilisant les modèles du disque et demi-plan de Poincaré (les projections conformes standards), beaucoup plus rarement le modèle de Beltrami-Klein, et je crois que je n'ai jamais vu une projection équivalente (=préservant les aires) du plan hyperbolique, alors qu'on en montre souvent pour la sphère.

Enfin, les analogies entre la sphère et le plan hyperbolique sont rarement mises en valeur. Bref, j'ai fait cette vidéo pour essayer de combler les trous (le commentaire est en anglais ; apparemment YouTube ne permet pas de faire des vidéos bilingues, ce qui est tout de même con) :

Comme souvent, ce n'est pas ce qu'on pense qui a pris du temps : le programme pour calculer les images (qu'on peut trouver ici) est extrêmement simple et m'a pris nettement moins d'une heure à écrire (environ 150 lignes de C ! on fait difficilement plus simple, même s'il est vrai que j'ai dû faire quelques petits calculs de trigonométrie sphérique et hyperbolique pour calculer les constantes au début du programme). Le calcul lui-même a été aussi assez indolore. La lecture du commentaire, en revanche, a été abominable, et j'ai fini par craquer et renoncer à produire quelque chose de pas trop mauvaise qualité où je ne bégaierais pas sur plein de mots, où on n'entendrais pas les voisins qui passent dans le couloir, où je ne parlerais pas avec une voix différente entre chaque paragraphe, etc. Je ne suis vraiment pas doué pour ça.

[Ajout : une série de trois entrées ultérieures sur la géométrie hyperbolique : 1, 2, 3 (plus ou moins indépendantes) ; voir aussi les jeux de labyrinthe hyperbolique que j'introduis ici, et , et dont j'explique le fonctionnement dans des transparents disponibles ici.]

↑Entry #2167 [older| permalink|newer] / ↑Entrée #2167 [précédente| permalien|suivante] ↑

↓Entry #2166 [older| permalink|newer] / ↓Entrée #2166 [précédente| permalien|suivante] ↓

(lundi)

La magie du théorème de l'application conforme de Riemann

Parmi les théorèmes mathématiques que je trouve les plus magiques, le théorème de l'application conforme de Riemann est assez haut dans la liste.

Pour expliquer un peu au niveau grand public ce que ce théorème signifie, il faut d'abord expliquer application conforme : une application conforme (ou holomorphe — au niveau où je me place ce n'est pas la peine de faire de distinction) est simplement une transformation du plan qui conserve les angles (orientés). De façon encore plus simple, disons qu'une application conforme est une application qui préserve localement les formes sans les aplatir : elle peut plus ou moins les agrandir ou les rétrécir d'un point à l'autre, mais un tout petit cercle se transforme en quelque chose qui ressemble à un cercle, pas à une ellipse (voir plus loin ce que je dis sur les cartes de la Terre).

Pour ceux qui comprennent un peu plus de maths, je peux dire ceci : une application affine (c'est-à-dire, préservant l'alignement) qui conserve les angles (orientés) est ce qu'on appelle une similitude (directe), c'est-à-dire la composition d'une homothétie et d'une rotation (et éventuellement d'une translation) ; si on voit le plan comme l'ensemble des nombres complexes, alors une similitude (directe) est précisément une application de la forme za·z+b pour certains nombres complexes a et b (le module et l'argument de a déterminant le rapport de l'homothétie et l'angle de la rotation, tandis que b détermine la translation ou le « centre » de la transformation). Une application conforme est une transformation qui, au premier ordre, en tout point, est une similitude (directe), c'est-à-dire, une application (d'une région du plan vers le plan) qui est différentiable et dont la différentielle est partout une similitude (directe) : d'après ce que je viens de dire, cela revient à voir ça comme une application dérivable au sens complexe.

À titre d'exemple, l'application exponentielle complexe, c'est-à-dire l'application qui à un point (x,y) du plan (qu'on peut identifier au nombre complexe z = x+i·y) associe le point (exp(x)·cos(y), exp(x)·sin(y)) (qu'on peut identifier au nombre complexe exp(z)), où ici exp(•) désigne e, est une transformation conforme. J'ai tenté de la représenter sur la figure suivante :

[Grille cartésienne + polaire] [Image de la grille par l'exponentielle]

↑Entry #2166 [older| permalink|newer] / ↑Entrée #2166 [précédente| permalien|suivante] ↑

↓Entry #2164 [older| permalink|newer] / ↓Entrée #2164 [précédente| permalien|suivante] ↓

(mercredi)

C'est compliqué d'enseigner Fourier

Je donne à Télécom ParisChose un cours d'Analyse (cela me demande d'ailleurs beaucoup de travail parce que je ne suis pas du tout analyste) dont un des points centraux est la théorie de Fourier. J'avais l'an dernier fait un petit catalogue de quelques énoncés sur la théorie des séries de Fourier (dépassant largement le niveau du cours que j'enseigne, mais nécessaire pour me clarifier les idées). Mais il faudrait que je parle un peu aussi de la transformée de Fourier, pour expliquer à quel point c'est subtil à définir.

Si f est L¹ (=intégrable au sens de Lebesgue) sur ℝ, on définit sa transformée de Fourier ℱf par

f ( ξ ) = + f(x) e 2i πξx dx

(pour ceux qui ont un de ces vieux navigateurs qui ne comprennent pas le MathML, il s'agit de l'intégrale de f(x)·exp(−2iπξx) pour x allant de −∞ à +∞, vue comme fonction de la variable ξ). Cette fonction ℱf est continue (de ξ), de limite nulle à l'infini.

Si f est L² (=de carré intégrable) mais pas forcément L¹, la formule ci-dessus n'a pas de sens en général ; on peut cependant définir une transformée de Fourier sur L² : par exemple, on utilise la formule ci-dessus pour définir la transformée de Fourier sur L²∩L¹ (ou sur un espace plus petit, comme l'espace de Schwartz), dense dans L², on démontre que l'opération « transformée de Fourier » est une isométrie au sens L², et on la prolonge par continuité. C'est déjà quelque chose d'assez subtil pédagogiquement.

L'autre subtilité pédagogique, c'est que nos élèves sortent (généralement) de prépa et que si on leur y a défini une intégrale de −∞ à +∞, c'est comme limite des intégrales à bornes finies quand ces bornes tendent vers −∞ et +∞ ; alors que l'intégrale dont il est question ci-dessus est l'intégrale de Lebesgue, définie de façon holiste sur ℝ, et il se trouve que si elle existe, elle est effectivement égale à la limite des intégrales à bornes finies (par le théorème de convergence dominée), mais la réciproque n'est pas vraie.

Les choses deviennent catastrophiques parce que ces deux subtilités se combinent de façon encore plus subtile : si on considère la fonction fM = f×1[−M;+M] égale à f sur l'intervalle [−M;+M] et à 0 ailleurs, alors fM converge vers f au sens L² quand M→+∞, donc les transformées de Fourier des fM convergent vers celle de f au sens L² ; or fM est L¹ et sa transformée de Fourier est donc donnée par l'intégrale de −M à +M de de f(x)·exp(−2iπξx). On a donc (pour tout f∈L²) :

M +M f(x) e 2i πξx dx M+ f ( ξ )

mais il s'agit d'une convergence au sens L² (d'une fonction de ξ vers une autre fonction de ξ), qui ne dit rien sur ce qui se passe pour un ξ ou un autre. Et là où ça devient subtilissimement subtilissime, c'est que en fait, si, il y a bien convergence pour presque tout ξ, mais cette convergence p.p. est un théorème très difficile (le théorème de Carleson).

En revanche, je suis assez convaincu, même si je n'ai pas de contre-exemple, qu'il est parfaitement possible qu'une fonction f localement intégrable ait une transformée de Fourier au sens des distributions g elle aussi localement intégrable, c'est-à-dire qu'on ait ∫(f·ψ)=∫(g·φ) pour toute fonction φ de l'espace de Schwartz ayant transformée de Fourier ψ (automatiquement elle-même dans l'espace de Schwartz), et pourtant que la limite écrite ci-dessus n'existe pour aucun ξ. Je me demande bien, d'ailleurs, s'il est possible que la limite existe pour tout ξ mais ne soit jamais égale à g(ξ) ; mais je n'ai pas du tout le temps d'y réfléchir.

Comment faire pour enseigner quelque chose qui soit rigoureux et qui ne noie pas pour autant les élèves sous la subtilité ?

↑Entry #2164 [older| permalink|newer] / ↑Entrée #2164 [précédente| permalien|suivante] ↑

↓Entry #2162 [older| permalink|newer] / ↓Entrée #2162 [précédente| permalien|suivante] ↓

(Monday)

Gratuitous Literary Fragment #147 (referee report)

To whom it may concern:

This is a referee report on the thesis titled The Character Table of the Weyl Group of E8: Applications to the Arcane Arts, a dissertation submitted by M. Parry Hotter in partial fulfillment of the degree of Magiæ Doctor at the University of Hogsbridge.

Context: To put this study in its proper historical perspective, which M. Hotter himself does at length in the first chapter of the thesis under review, would require more space than can be afforded here. As the author aptly recalls, the E8 perspective on the arcane arts can be traced back to the unification, proposed by Leibniz in 1710, of six of the seven classical schools of magic (Earth, Water, Air, Fire, Macrocosm, Life and Spirit, arranged linearly by Paracelsus) with six of the seven oriental phases (Earth, Water, Wind, Fire, Heaven, Change and Unchange, with Change and Unchange branching from Heaven), by equating Heaven with Macrocosm and Change with Life (and renaming Unchange as Time). The asymetrical nature of the resulting diagram — which we now know as the Dynkin diagram of E8 — prompted a number of attempts to identify at least one more house — attempts that we presently understand to be misguided.

But it is in the year 1918, which saw the publication of Hermann Weyl's now classic Earth, Water, Air, Fire, Space, Time, Life and Spirit, that the 240 directions of the mysticohedron were put upon a firm theoretical footing. This represents a considerable paradigm shift, whose practical consequences were slow to come to fruition (starting with the startling realization of where the level grades, 1, 248, 3 875, 27 000, 30 380, 147 250, 779 247… appear). And as examined in more detail in Aldus Bumblebore's The Eight Elements (or: What's so Special about the Number 696 729 600?), it was only considerably recently that any attention was given to the profond interconnection between the largest exceptional Weyl group and the transmutations of magic.

As explained in the abstract, M. Hotter's work consists of two main parts. The first explores applications of the « pure » character theory of W(E8) beyond the mere, and previously known, identification of the 112 representations (lines of the character table) and conjugacy classes (columns) with the arcane circles and astral configurations. The second, and much deeper, part of this thesis, develops an invariant theory for E8 that is analogous to the classical Schur-Weyl duality between representations of the linear group and those of the symmetric group, and then applies this duality to obtain esoterica. Eight specific and illustrative examples are given in an appendix. We now review each of the chapters in greater detail.


Some commentary: (Generally I don't discuss the references in my texts but I've made exceptions before.)

I've often said that E8 (and the cohort of related objects) is so deeply fascinating and profoundly beautiful (see here and there) that if the Universe has any sense of æsthetics, it really should involve E8 in some way (some people have indeed tried to find it, but with dubious success: it is possible that the Universe we live in does not have the same notion of æsthetics as mathematicians). At any rate, in a world in which magic is real and wizards who (after their elementary and high school years spent doing more applied work like fighting supervillain wizards) go to university and write doctoral theses in pure and theoretical magic, I cannot conceive the mathematical foundations of magic to be anything but E8 (OK, maybe I'll take the largest Mathieu group as an acceptable substitute). Of course, in my vision of magic, the head magician does not look so much like this as like that (seriously, if anyone is a real world magician, it's John Conway). Anyway.

So if magic is to be built on E8, then the system at the root of all arcana is represented by the following diagram:

—and there should definitely be some labels attached here. (They would actually be of some use to real world mathematicians because nobody can agree on how to number the vertices of this diagram. Unfortunately, Conway, the great inventor of witty names, did not do his job here.) So I propose to name the seven on the bottom line, from left to right: Spirit, Life, Macrocosm, Fire, Air, Water and Earth, and the top one, Time. There isn't much rationale to my suggestion that the Europeans and Chinese(?) should have discovered the A7 (i.e., all but the top node) and D7 (i.e., all but the leftmode node) subdiagrams of the above, but it is true that Leibniz was fascinated by the Yi Jing and popularized it in Europe. (It seemed right to make Leibniz play a role here when I had had fun with Newton in a previous fragment.)

More importantly, Hermann Weyl, something of a magician himself, to whom we owe much of the theory of representations of compact Lie groups (and in particular the formula which allows to compute the sequence I mention), wrote a book called Space, Time, Matter, one of the first expositions of Einstein's theory of general relativity (and indeed one of the books — found in my father's library — through which I myself learned the subject): in an alternate universe, it would certainly have been a book on magic.

Incidentally, I wish someone would tell me how one can construct some kind of analogue of Schur-Weyl duality for the exceptional groups (or in any way relate the representations of E8 to those of W(E8)).

Addendum: as mentioned in the comments, I should probably link to this later entry, and perhaps enve more relevantly to this one, for various explanations on E₈ (and why it fascinates me).

↑Entry #2162 [older| permalink|newer] / ↑Entrée #2162 [précédente| permalien|suivante] ↑

↓Entry #2160 [older| permalink|newer] / ↓Entrée #2160 [précédente| permalien|suivante] ↓

(samedi)

Comment définir efficacement ce qu'est un schéma

[Cette entrée m'a pris environ deux mois à écrire : j'ai commencé à cause de ceci qui m'a donné envie de m'éclaircir les idées en les racontant. Au début je pensais que ça ne ferait que quelques lignes !]

Introduction

Il est souvent intéressant, en mathématiques, de prendre une notion, et d'essayer de la redéfinir (ou, s'il s'agit d'un théorème, de le redémontrer) de novo, je veux dire en partant de rien (ou du minimum). Ceci permet de se rendre compte de ce dont on a besoin pour arriver à ce point, de chercher les raccourcis pour y arriver, et de gagner en clarté sur la notion en question. Je vais prendre l'exemple de la notion de schéma, qui est une notion centrale de la géométrie algébrique depuis Grothendieck.

Je m'adresse ici au lecteur qui sait ce qu'est un anneau commutatif (ainsi que les notions qui vont forcément avec : morphisme d'anneaux commutatifs, idéal, quotient par un idéal ; et aussi : polynômes à coefficients dans un anneau commutatif ; je suppose aussi qu'on sait ce qu'est un corps), mais qui n'a pas de connaissances en géométrie algébrique et qui, en particulier, ne sait pas ce qu'est un schéma. Le but est de montrer qu'on peut définir ce terme-là de façon relativement efficace (c'est-à-dire : rapide, mais pas forcément pédagogiquement idéale) en évitant de parler de tout le fatras d'idéaux premiers, faisceaux d'anneaux, espaces localement annelés, etc., qui servent normalement à définir ce qu'est un schéma.

(L'expert en géométrie algébrique n'a certainement rien à apprendre ici, mais il pourra trouver intéressante ma définition d'une immersion ouverte comme tiré en arrière d'une immersion universelle ; et s'il n'a jamais réfléchi à la question, l'exercice de décrire le foncteur des points de la droite avec origine doublée et de quelques uns de ses avatars est amusante.)

La définition proprement dite est ce qui figure dans les parties encadrées (pour montrer qu'elle est effectivement plutôt courte même si cette entrée dans son ensemble est extrêmement longue) : 1, 2, 3, 4, 5, 6, 7, 8 et 9. On peut donc en principe se contenter de lire ces passages-là. Le commentaire autour sert à essayer de la rendre pas totalement incompréhensible, en donnant quelques exemples et quelques explications, et à motiver un peu ce qu'on fait.

Plan de cette entrée :

Préliminaires : Je vais fixer un anneau de base k (tous les anneaux considérés sont commutatifs et ont un élément unité). Si on ne sait pas ce qu'est une k-algèbre (sous-entendu : commutative), c'est simplement un anneau A accompagné d'un morphisme d'anneaux kA (appelé morphisme structural) ; et un morphisme AB de k-algèbres est défini comme un morphisme d'anneaux tel que la composée kAB avec le morphisme structural de A soit le morphisme stuctural de B. Mais pour lire ce qui suit, si on préfère, on peut par exemple imaginer que k est un corps (la notion de k-algèbre est sans doute mieux connue dans ce contexte), ou bien que k est l'anneau ℤ des entiers auquel cas il faut comprendre l'expression k-algèbre comme exactement synonyme de anneau. Cet anneau de base k est sous-jacent à toute la construction (ce qu'on définit, ce sont les k-schémas), il ne sera jamais varié, et parfois j'omettrai sa mention (je parlerai par exemple de l'espace affine pour ce qu'il conviendrait d'appeler l'espace affine sur k).

Motivation : Le type d'objets que la géométrie algébrique étudie, ce sont des solutions d'équations polynomiales (en plusieurs variables), considérés comme des objets géométriques ; pour prendre un exemple, je pourrais appeler cercle unité (affine) l'« objet » C d'équation x²+y²=1 : quand j'écris ça, cela signifie que pour n'importe quel corps K sur k, ou plus généralement n'importe quelle k-algèbre A, je vais considérer l'ensemble C(K) ou C(A) des solutions de cette équation dans K ou A, c'est-à-dire l'ensemble des couples (x,y) de deux éléments de K ou A tels que x²+y²=1. A priori ce sont plutôt les solutions sur un corps K qui ont historiquement intéressé les géomètres algébriques, mais pour toutes sortes de raisons il est pertinent (et utile pour la formalisation) de s'intéresser aux solutions sur une k-algèbre A quelconque (même si l'intuition et, dans une certaine mesure, la terminologie, viennent du cas des corps).

À titre d'exemples, C(ℤ) n'a que quatre points ((±1,0) et (0,±1)), tandis que C(ℝ) correspond vraiment à ce qu'on appelle naïvement un cercle (celui qu'on peut paramétrer de façon transcendante par (cos(θ),sin(θ)) ; C(ℚ) est infini même si ce n'est pas forcément si évident à première vue (il contient des points tels que (3/5, 4/5)) ; et on peut montrer que C(F), si F est un corps fini à q éléments (q nécessairement une puissance d'un nombre premier), a q−1, q+1 ou q éléments selon que q est respectivement congru à 1 modulo 4, congru à 3 modulo 4, ou bien une puissance de 2. (En fait, si K est un corps de caractéristique différente de 2 — c'est-à-dire que 2 y est inversible — alors C(K) peut être paramétré, à l'exception du seul point (−1,0), par t↦((1−t²)/(1+t²),2t/(1+t²)) tant que t²≠−1.)

↑Entry #2160 [older| permalink|newer] / ↑Entrée #2160 [précédente| permalien|suivante] ↑

↓Entry #2147 [older| permalink|newer] / ↓Entrée #2147 [précédente| permalien|suivante] ↓

(mercredi)

Remise en question d'une croyance mathématique

[Le fond de ce dont il est question dans ce post ne sera compréhensible que par mes lecteurs qui sont eux-mêmes géomètres algébriques, mais ce que je veux souligner c'est aussi et surtout l'effet psychologique produit sur moi par ce que j'ai appris, et cet aspect-là ne devrait pas être technique.]

Lundi j'ai appris l'existence d'un contre-exemple qui montre que quelque chose que j'avais tenu pour absolument et évidemment vrai en géométrie algébrique est, en fait, faux. Le fait (vrai) est que :

L'algèbre Γ(X,𝒪X) des fonctions régulières globales d'un schéma X de type fini (et même d'une variété quasiprojective — ou meme quasi-affine — lisse) sur un corps k n'est pas nécessairement de type fini. (Voici un contre-exemple, accompagné de quelques commentaires ; voir aussi ici pour un contexte un peu différent.)

(Pour dire les choses différemment, le foncteur X↦Spec(Γ(X,𝒪X)), adjoint à gauche du foncteur d'inclusion des k-schémas affine dans les k-schémas, ne préserve pas la type-finitude sur k.)

Et pour ceux qui savent un peu d'algèbre mais pas de géométrie algébrique : on peut trouver une algèbre A de type fini sur un corps k — c'est-à-dire qu'elle est engendrée en tant qu'algèbre par un nombre fini d'éléments —, disons même A intègre, et un nombre fini d'éléments f1,…,fr de A (je suppose que r=2 doit suffire), tels que l'intersection B, prise dans le corps des fractions K de A, des anneaux A[1/fi] (des fractions qui peuvent s'écrire avec une puissance de fi comme dénominateur), autrement dit les éléments de K qui peuvent s'écrire avec une puissance de n'importe quel fi comme dénominateur, bref, tels que cette algèbre B ne soit pas de type fini. (Le lien avec la formulation précédente est que B est l'anneau des fonctions régulières sur l'ouvert réunion des D(fi) dans Spec(A). Bon, évidemment, sous cette forme purement algébrique, ce n'est pas évident que ça soit choquant.)

Pour ceux pour qui c'est du chinois, imaginez que j'aie toujours cru que New York était la capitale des États-Unis et je viens d'apprendre que non. Ou peut-être, justement, que je viens d'apprendre que New York est la capitale des États-Unis. Toujours est-il qu'il est intéressant de se demander pourquoi j'ai cru le contraire ou quel effet cette révélation produit sur moi.

Pourquoi me suis-je fait cette idée fausse (que l'algèbre des fonctions régulières d'un schéma de type fini serait de type fini) ? Évidemment ce n'est écrit dans aucun livre ou cours, puisque ce n'est pas vrai. Mais quand on apprend une branche des mathématiques, on ne se contente pas de retenir par cœur quantité d'énoncés vrais et leur démonstration : comme on n'est pas un ordinateur, on essaie de se former une image mentale de la manière dont les choses fonctionnent, des mécanismes généraux par lesquels les choses sont nommées et selon lesquelles les démonstrations sont construites, etc. Un de ces mécanismes, en géométrie algébrique, pourrait s'énoncer en gros comme ceci :

Contexte : Il y a des objets algébriques classiques appelés anneaux ou algèbres. La géométrie algébrique transforme ces objets algébriques en des objets géométriques, appelés schémas affines, qui leur sont tout à fait équivalents — chaque anneau donne naissance à un schéma affine, son spectre, et le schéma affine permet de retrouver l'unique anneau dont il est le spectre comme son anneau des fonctions régulières globales. Puis, sur cette base plus géométrique, la géométrie algébrique définit des objets géométriques plus généraux, les schémas, qui s'obtiennent par « recollement » de schémas affines (lesquels sont le modèle local des schémas généraux).

Idée : Pour beaucoup de propriétés P des anneaux qui ont un nom standard (par exemple : réduit), on qualifie par le même nom les schémas affines qui correspondent à (=sont le spectre d')un anneau ayant cette propriété P ; puis on appelle aussi de la même manière les schémas généraux qui s'obtiennent en « recollant » des schémas affines qui sont P (éventuellement avec une condition de finitude supplémentaire sur le recollement) ; et enfin, on s'assure que cette terminologie est raisonnable en vérifiant que les objets algébriques qui se déduisent de ces objets géométriques P (leurs anneaux de fonctions régulières) sont encore P.

Personne n'énonce explicitement ce principe général, qui admet d'ailleurs certaines déclinaisons, et notamment les étudiants en géométrie algébrique doivent s'en apercevoir eux-mêmes (c'est sans doute dommage), mais je pense que personne de l'art ne contestera l'esprit général de ce que je viens d'écrire. (Voyez, par exemple, dans le Algebraic Geometry de Hartshorne, l'exemple 3.0.1 et la proposition 3.2 et la remarque qui précède, et plus généralement ce qu'il y a dans ce coin-là.) Il y a certainement des petites variations selon la propriété P (idéalement, c'est une propriété locale, et alors il suffit de la vérifier localement), mais généralement elles sont telles qu'avec un tout petit peu d'habitude on comprend immédiatement ce qu'il faut dire parce que les contre-exemples sont évidents (par exemple : la somme de deux schémas intègres non vide n'est pas intègre puisque l'opération algébrique correspondante est un produit d'anneaux, ce n'est pas une propriété locale, mais on devine alors immédiatement quand il faut qualifier un schéma d'intègre). Je pense que beaucoup de cours, faute de temps, s'abstiennent de faire le tour de chaque propriété qu'on énonce pour expliquer si elle est locale (et le démontrer !) ou donner les métapropriétés de cette propriété qui permettent exactement de contrôler le vocabulaire — au mieux, on a tendance à laisser ça en exercice au lecteur (voyez, par exemple, toujours chez Hartshorne, l'exercice 2.3).

Le cas que j'évoque est traître, parce que les vérifications qu'on va instinctivement faire pour s'assurer que « tout va bien » pour cette propriété P (être de type fini sur un corps) sont effectivement satisfaites (notamment : si on définit un schéma affine de type fini sur k comme le spectre d'une algèbre de type fini sur k, alors il est bien vrai qu'un schéma affine est de type fini sur k si et seulement si il est recouvert par un nombre fini d'ouverts affines dont chacun est de type fini sur k). Et aucun contre-exemple ne saute aux yeux. Donc je pense que beaucoup de géomètres algébristes auraient tendance, comme moi, à tomber dans le panneau et à se dire être de type fini sur un corps, pour un schéma nœthérien, est une propriété locale, donc tout va bien. En tout cas, j'aurais certainement laissé passer sans tiquer, dans un article dont je serais rapporteur, une affirmation de ce genre.

Bref, je croyais quelque chose de faux. Est-ce grave ? Formellement, probablement pas, parce que je ne pense pas avoir jamais utilisé ce fait dans un raisonnement : c'est plutôt quelque chose qui guide l'intuition — et apparemment mon intuition était mal guidée — que quelque chose dont on se sert dans une démonstration. C'est néanmoins très déstabilisant de se rendre compte tardivement d'une pareille erreur d'intuition. Parce que ça m'oblige maintenant à me demander si je n'en ai pas fait d'autres pour d'autres propriétés P : à me demander, par exemple, si l'anneau des fonctions régulières globales d'un schéma intègre est bien intègre, ou autres bêtises de ce genre. Ou a contrario, à me demander s'il n'y a pas une propriété P′ meilleure que P pour laquelle le problème ne se poserait pas (être une limite finie d'algèbres de type fini, par exemple, est-ce que ça se comporte bien de tout point de vue ?), ce qui soulève quantité de questions certes très instructives mais néanmoins chronophages et qui ne font pas vraiment progresser la recherche.

Pour éviter ça, les cours devraient faire l'effort de systématiquement proposer tous les exemples et contre-exemples qui permettent d'éviter de se faire des idées fausses sur la terminologie. Donc j'en veux beaucoup à ceux qui m'ont enseigné la géométrie algébrique de ne pas avoir attiré mon attention sur ce qui est un problème ou un contre-exemple classique (puisque Hilbert se l'était posé, sous une forme différente). Ou de ne rien m'avoir appris sur les propriétés du foncteur d'« affinisation » X↦Spec(Γ(X,𝒪X)), par exemple quelles propriétés il préserve, ou de la flèche canonique X→Spec(Γ(X,𝒪X)), par exemple quelles propriétés elle a ou à quelle condition elle effectue une descente de propriétés de X vers son affinisé, parce que ce sont des objets vraiments naturels et qu'on est en droit d'être curieux à leur sujet.

Ajout : On me signale que l'avertissement est fait dans le Red Book de Mumford (II.§3, en-dessous de la proposition 1). En effet, c'est un bon point pour mumford, comme le fait qu'il fait des tentatives pédagogiquement intéressantes pour dessiner, par exemple, Spec(ℤ[t]).

↑Entry #2147 [older| permalink|newer] / ↑Entrée #2147 [précédente| permalien|suivante] ↑

↓Entry #2130 [older| permalink|newer] / ↓Entrée #2130 [précédente| permalien|suivante] ↓

(vendredi)

Pourquoi je ne crois pas à l'ultrafinitisme

(Je développe ici quelque chose que j'ai écrit en commentaire sur une entrée du blog de David Monniaux sur l'épistémologie des mathématiques.)

Le finitisme en philosophie des mathématiques est l'idée que l'infini n'existe pas (avec différentes variations possibles selon ce qu'on entend au juste par n'existe pas), i.e., que seuls les objets finis — les entiers naturels ou ce qui peut se coder avec eux, comme les structures combinatoires finies — ont vraiment un sens. Disons qu'il s'agit d'une position qui met une limite sur le platonisme (cf. ce que j'en disais ici) qui postule que les objets mathématiques existent dans une sorte de paradis platonicien : le finitisme n'admet dans ce paradis que les entiers naturels et les structures finies. Cette position est illustrée par une phrase de Kronecker que j'ai déjà mainte fois citée : Die ganzen Zahlen hat der liebe Gott gemacht, alles andere ist Menschenwerk (Les entiers ont été faits par Dieu, tout le reste est l'œuvre de l'homme).

L'ultrafinitisme va plus loin en rejetant non seulement l'infini mais même les nombres absurdement grands comme ceux que je décris dans cette entrée (ajout : voir aussi celle-ci) (ou même des nombres beaucoup plus petits qu'eux). Pour un ultrafinitiste, se demander, par exemple, si le 10↑(10↑(10↑100))-ième nombre premier se termine par 1, 3, 7 ou 9, est une question à peu près dénuée de sens.

Évidemment, il est parfaitement sot de penser qu'il existe un plus grand entier naturel, auquel on ne peut pas ajouter 1 : ce serait là une caricature de la position ultrafinitiste. Ceux qui se revendiquent de cette idée pensent plutôt que les nombres ont de moins en moins de sens au fur et à mesure qu'ils grandissent, si bien que 100 existe certainement mais que 10↑(10↑(10↑100)) n'a à peu près aucun sens. Je ne sais plus où j'avais entendu cette anecdote de quelqu'un qui a voulu mettre en défaut un ultrafinitiste en essayant de trouver le plus grand nombre dont ce dernier reconnaîtrait l'existence : il a commencé par lui demander si 1000 existait, puis un million (10↑6), puis un milliard (10↑9), puis un gogol (10↑100), etc. La raison pour laquelle ce plan n'a pas marché est que l'ultrafinitiste a toujours répondu « oui », mais en réfléchissant de plus en plus longtemps à mesure que le nombre était grand : il a répondu presque instantanément qu'un milliard existait, mais il a mis une seconde à déclarer qu'un gogol existait, peut-être 10 secondes pour 10↑1000, et n'a jamais consenti à admettre que 10↑(10↑100) existât. [Précision : l'ultrafinitiste en question était Esenin-Vol'pin, l'autre était Harvey Friedman, et l'anecdote est racontée sur l'article Wikipédia su premier (et les nombres impliqués sont d'ailleurs plus petits que ceux que je cite).] Voilà qui devrait donner une meilleure idée de cette philosophie ; et globalement, on peut imaginer qu'un ultrafinitiste typique reconnaîtra l'existence d'un nombre dans un temps, ou avec une difficulté, proportionnel à ce qu'il faut pour l'écrire complètement — disons, en base 10 — sans artifice comme les exponentielles, c'est-à-dire, proportionnellement à son log. Cela correspond assez bien à l'usage qu'on fait des nombres ailleurs qu'en mathématiques, finalement.

L'ultrafinitisme, du coup, n'admet pas forcément que l'arithmétique de Peano, ou que ZFC, soient consistants. En fait, il n'admet même pas que la question ait un sens (la démonstration d'une contradiction pourrait être démesurément longue) : mais il constate certainement qu'aucune contradiction n'est connue dans ces systèmes, et admet peut-être, ou peut-être pas, qu'on pourrait raisonnablement en trouver une. De fait, il y a des gens qui cherchent à trouver, ou espèrent sérieusement qu'on trouve, une contradiction dans ces systèmes. (Voir notamment cette entrée passée.)

C'est là que je trouve que la position ne tient pas vraiment debout. Car de toute évidence les ultrafinitistes acceptent les conclusions arithmétiques de Peano (ou même de ZFC, mais ça ne fait guère de différence ici) concernant les entiers qu'ils considèrent comme raisonnables : si on considère les mathématiques comme une science expérimentale, on peut même dire qu'on dispose d'une quantité faramineuse de confirmation expérimentale de son cadre (à chaque fois qu'on paie quelque chose, on vérifie la commutativité et l'associativité de l'addition, et n'importe quel calcul sur ordinateur vérifie expérimentalement des quantités énormes de théorèmes mathématiques), et en tout cas, je vois mal comment on peut être un mathématicien si on ne croit pas un mininum aux théorèmes que l'on démontre. Et je ne vois rien dans la position ultrafinitiste qui permette d'expliquer ceci : comment expliquer que Peano (ou ZFC) fasse des prévisions si justes ? comment se fait-il qu'on n'arrive pas, au moins en pratique, à y trouver une contradiction ? Ou, pour dire les choses autrement : comment se fait-il, si les objets infinis ou même les très grands nombres n'existent pas, qu'ils donnent l'illusion d'exister ? Comment se fait-il que le monde mathématique se comporte comme s'ils existaient ?

Un platoniste pur jus n'aura pas de difficulté à expliquer que Peano soit consistant : si les entiers naturels existent et sont bien définis, il est normal qu'ils se comportent ainsi, il n'y a pas de contradiction dans Peano tout simplement car Peano est vrai. ((S'agissant de ZFC, ma position à moi a tendance à être que les ensembles existent sans être uniques : il s'agit de la position « multivers » où il n'existe pas un univers privilégié de la théorie des ensembles, mais le fait qu'il en existe au moins un fait que ZFC doit être consistant.))

La position ultrafinitiste en mathématiques me paraît semblable à celle de quelqu'un qui prétendrait que les galaxies lointaines que nous voyons dans le ciel nocturne n'existent pas : comme nous ne pouvons pas raisonnablement les atteindre, nous n'en avons qu'une image, qui pourrait être une illusion. Il n'y a pas forcément besoin pour cela de croire que l'Univers aurait une frontière bien nette comme il n'y a pas besoin pour être ultrafinitiste de croire qu'il existe un plus grand entier naturel. Néanmoins, il me semble beaucoup plus simple de penser que ces objets lointains et inatteignables (qu'il s'agisse des autres galaxies ou des très grands nombres) existent vraiment que de croire que, sans exister, ils parviennent à laisser une image dans notre expérience qui suggère qu'ils existent.

(Ajout : voir aussi une entrée ultérieure sur un thème proche. • Et une autre.)

↑Entry #2130 [older| permalink|newer] / ↑Entrée #2130 [précédente| permalien|suivante] ↑

↓Entry #2124 [older| permalink|newer] / ↓Entrée #2124 [précédente| permalien|suivante] ↓

(mardi)

Pourquoi l'univers constructible de Gödel est important mathématiquement et philosophiquement

[J'ai écrit ce texte par tout petits bouts sur plusieurs mois. J'espère que le processus fragmenté de son écriture ne le rend pas trop brouillon !]

Introduction

De façon inexplicable, quand j'ai listé certains des objets mathématiques qui me fascinent, j'ai oublié de citer l'univers constructible de Gödel : la lecture de l'article Wikipédia à son sujet ne parlera sans doute pas beaucoup au profane, mais je voudrais rattraper mon oubli en essayant d'expliquer un peu de quoi il s'agit, parce que cet objet me semble non seulement fondamental et extrêmement élégant, mais même, dans un certain sens, on peut dire qu'il donne un sens à toutes les mathématiques (avec néanmoins une question philosophique fondamentale : est-ce vraiment le sens que nous voulons ou devrions leur donner ? c'est-à-dire, veut-on ou doit-on accepter l'axiome de constructibilité qui affirme que cet univers constructible est l'univers mathématique ?).

Si on me donne dix secondes pour dire un peu de quoi il s'agit à quelqu'un qui ne connaît rien aux mathématiques, je pense que je tenterais de résumer l'axiome de constructibilité à ce slogan philosophique dont, malheureusement, le côté percutant enlève beaucoup à la précision :

Il n'existe pas de vrai hasard.

Si, en revanche, je dois expliquer de quoi il s'agit à quelqu'un qui connaît un peu les mathématiques, je dirai la chose suivante : l'axiome de constructibilité généralise — et non seulement implique, mais même explique — deux postulats célèbres de la théorie des ensembles que sont l'axiome du choix et l'hypothèse (généralisée) du continu. L'axiome de constructibilité rend explicites et « naturels » des objets dont l'axiome du choix ne fait que postuler l'existence, et il apporte suffisamment d'information sur les parties de ℕ pour démontrer et éclairer l'hypothèse du continu (et plus généralement sur les parties de n'importe quel ensemble pour l'hypothèse généralisée du continu). L'axiome de constructibilité ordonne (et « déroule ») tout l'univers mathématiques selon les ordinaux et permet donc de répondre à un nombre étonnant de questions combinatoires autrement indécidables, dont la véracité de l'axiome du choix et de l'hypothèse du continu ne sont que deux exemples : ce qui ne veut pas dire, cependant, que nous devions forcément accepter ces réponses comme correctes.

Plan de cette entrée :

↑Entry #2124 [older| permalink|newer] / ↑Entrée #2124 [précédente| permalien|suivante] ↑

↓Entry #2104 [older| permalink|newer] / ↓Entrée #2104 [précédente| permalien|suivante] ↓

(mercredi)

Qui peut dire le nombre le plus grand ?

Je suppose que beaucoup d'enfants, quand ils apprennent à compter, jouent au petit jeu de qui peut dire le nombre le plus grand. Bon, au début, c'est facile, ils ne connaissent qu'un nombre fini d'entiers naturels, donc il suffit de dire le plus grand. Puis un jour arrivent les milliers, les millions, les milliards, les idées pas très claires sur ce qui est le plus grand dans tout ça et tout de même la réalisation terrifiante de ce que c'est que l'infini, et qu'il n'y a pas de moyen de gagner à coup sûr à ce jeu : si l'un dit un milliard, l'autre peut dire un milliard de milliards ; si on écrit les nombres sur du papier, cela devient une question de qui aura le temps ou la patience d'écrire le plus de chiffres. Quand on devient plus sophistiqué, on se dit qu'on peut relaxer la règle, ce n'est peut-être pas la peine d'écrire tous les chiffres, on peut écrire dix puissance suivi de leur nombre, mais alors le même problème se repose. Un jour, un des enfants décide qu'on a le droit de jouer l'infini, mais alors l'autre réplique l'infini plus un et s'ensuit une dispute pour savoir si c'est légitime ou pas, pareil ou pas.

Mais les adultes jouent parfois encore à ce jeu, et notamment les mathématiciens (il y a même un Wikia consacré aux grands nombres). Les physiciens sont petits joueurs : à peu près le plus grand nombre qui doit intervenir en physique est le rapport de la densité de Planck sur la constante cosmologique, soit quelque chose comme 10 puissance 122, ce qui est un peu embarrassant si on pense que ce nombre devrait valoir pas loin de 1, mais pour un mathématicien ce n'est pas très impressionnant ; pour un cryptographe, c'est une estimation du nombre d'opérations qu'il faut faire, en cryptographie, pour casser une clé de 400 bits.

D'où naturellement la question un peu stupide ou enfantine, mais néanmoins amusante : quel est le plus grand nombre qu'on sache définir ?

↑Entry #2104 [older| permalink|newer] / ↑Entrée #2104 [précédente| permalien|suivante] ↑

↓Entry #2093 [older| permalink|newer] / ↓Entrée #2093 [précédente| permalien|suivante] ↓

(samedi)

Ce que « vrai » veut dire en mathématiques

Une des difficultés que rencontrent les gens qui font un peu de logique mathématique mais qui n'en ont pas trop l'habitude — et une difficulté qui sous-tend beaucoup de discussions sur la philosophie des mathématiques — c'est que les gens ne savent plus ce que « vrai » veut dire. Par exemple, quand on discute du théorème de Gödel ou de la différence entre l'ensemble des théorèmes de l'arithmétique de Peano et l'ensemble des énoncés vrais de l'arithmétique, ceci cause souvent un certain malaise (voir par exemple la note #2b de l'entrée précédente). J'ai souvent tourné autour de cette question dans ce blog, mais je n'ai jamais essayé de présenter les choses de façon synthétique. En réponse à une question, je vais donc essayer de dissiper la confusion (ce qui me permettra de renvoyer à la présente entrée quand la question se reposera à l'avenir).

(Eh oui, ce qui suit est très long. Je croyais avoir trois fois rien à raconter, et une fois de plus j'ai pondu des pages… et des pages… et des pages, en essayant de « parler » à la fois à plusieurs niveaux d'expertise différents. J'ai mis en plus petits caractères les passages qui sont une digression par rapport à l'essentiel de mon propos, mais de façon générale j'ai essayé de faire en sorte qu'on puisse comprendre un passage même en ayant lu ce qui précède en diagonale. Peut-être que j'aurais dû publier tout ça sous forme de feuilleton, en plusieurs entrées de blog, mais je trouvais que ça nuisait à la cohérence de l'ensemble.)

↑Entry #2093 [older| permalink|newer] / ↑Entrée #2093 [précédente| permalien|suivante] ↑

↓Entry #2092 [older| permalink|newer] / ↓Entrée #2092 [précédente| permalien|suivante] ↓

(lundi)

Problème de l'arrêt de problème de Post

Je voudrais à nouveau tenter un peu de vulgarisation autour de la calculabilité, cette fois-ci pour parler du problème de Post dont je regrette qu'il soit trop mal connu des matheux et informaticiens. Bon, déjà le nom pose une difficulté, parce qu'il y a deux choses différentes connues sous le nom de problème de Post (pas totalement sans rapport, toutes les deux du domaine de la calculabilité et liées au problème de l'arrêt, mais néanmoins bien distinctes). Celle dont je ne veux pas parler est le problème de correspondance de Post : comme ce dernier est nettement plus connu que le problème de Post dont il est question ici, cela cause un certain nombre de confusions désagréables. Une autre difficulté tient de façon plus générale à la terminologie du domaine (comme le faisait remarquer il n'y a pas longtemps mon ami David Monniaux) : ce qu'on appelle problème décidable s'appelle aussi ensemble récursif (et problème semi-décidable aussi ensemble récursivement énumérable). Bref, essayons d'y voir plus clair.

D'abord, il faut expliquer ce qu'on entend par un problème de décision : en bref, c'est une question mathématique bien définie et dont la réponse doit être oui ou non. En un peu plus précis, c'est un problème qui prend en entrée une donnée finie (c'est-à-dire, par exemple, un entier, ou une chaîne finie de caractères, ou une donnée combinatoire finie par exemple un graphe) et qui doit répondre à une question mathématiquement précise sur cette donnée. Un exemple de tel problème serait : le nombre p que voici (la donnée du problème) est-il un nombre premier ? Ou encore : exise-t-il un circuit hamiltonien dans le graphe que voici ? Il est toujours possible — c'est même la définition ce que j'entends par donnée finie — de coder[#] la donnée sous la forme d'un entier naturel (par exemple, une chaîne de caractères peut se coder comme une suite finie d'entiers, et une suite finie d'entiers peut elle-même se coder sous la forme d'un entier par exemple en utilisant la décomposition en facteurs premiers ou diverses manipulations sur l'écriture binaire ou décimale). À cause de ça[#2], on peut tout simplement considérer qu'un problème de décision est un ensemble d'entiers naturels : le problème est alors simplement de savoir si tel ou tel entier donné appartient ou non à l'ensemble. On peut aussi préférer considérer — de la même manière — qu'on a affaire à un ensemble de chaînes (finies) de caractères, ce qu'on appelle un langage : les notions de problème de décision, d'ensemble d'entiers naturels, ou de langage, sont essentiellement équivalentes.

↑Entry #2092 [older| permalink|newer] / ↑Entrée #2092 [précédente| permalien|suivante] ↑

↓Entry #2090 [older| permalink|newer] / ↓Entrée #2090 [précédente| permalien|suivante] ↓

(jeudi)

Personne n'aime les fonctions primitives récursives ?

Je me dis souvent que la classe des fonctions primitives récursives est le parent mal aimé de l'informatique théorique.

Les fonctions générales récursives — c'est-à-dire, calculables au sens de Church-Turing — figurent dans n'importe quel cours de base : ce sont celles qui peuvent être calculées par une machine de Turing (ou toutes sortes d'autres modèles de calculs, comme des machines à registres), ou encore définies dans le λ-calcul de Church (non typé) ou par le schéma de récursion générale, ou bien calculables par essentiellement n'importe quel langage de programmation idéalisé raisonnable (je recommande la lecture du livre de Hofstadter, Gödel, Escher, Bach, et la description du langage FlooP, comme exemple). Toutes ces descriptions sont équivalentes. Le cours va ensuite généralement souligner deux choses : (A) il est possible de réaliser une machine de Turing universelle (c'est-à-dire, qui prend en entrée une description d'une autre machine de Turing et l'entrée à fournir à cette dernière, et simule son exécution, donc termine ssi l'autre termine, et renvoie le même résultat si c'est le cas), ce qui revient, informatiquement, à écrire un interpréteur d'un langage de programmation dans lui-même (ou dans un autre, peu importe), et (B) il n'est pas possible pour une machine de Turing de résoudre le problème de l'arrêt des machines de Turing, c'est-à-dire de décider (en terminant toujours) si une machine de Turing spécifiée s'arrête (sur une entrée donnée, mais en fait peu importe). La plupart des cours s'arrêtent là.

↑Entry #2090 [older| permalink|newer] / ↑Entrée #2090 [précédente| permalien|suivante] ↑

↓Entry #2089 [older| permalink|newer] / ↓Entrée #2089 [précédente| permalien|suivante] ↓

(lundi)

Petit exposé sur la relativité

J'ai donné avant-hier dans le cadre du séminaire Mathematic Park, qui s'adresse surtout aux élèves de prépas ou de licences scientifiques, un exposé intitulé Relativité et Géométrie — malgré ce que le titre peut laisser penser, je n'ai parlé que de relativité restreinte (j'avais prévu quelques transparents sur la relativité générale s'il me restait du temps, mais il ne m'en est pas resté, ce qui valait sans doute mieux). J'aurais sans doute dû l'annoncer à l'avance sur ce blog, mais j'ai été un peu pris par le temps en commençant à préparer à la dernière minute. Bref. Mon propos a surtout été de présenter la relativité restreinte en insistant sur l'importance de la forme quadratique de Minkowski, et d'expliquer pourquoi il vaut mieux mesurer les vitesses en relativité en utilisant la notion de rapidité[#] (qui ont le bon goût de s'ajouter en dimension 1+1) et en quoi l'espace des vitesses de la relativité restreinte se comporte naturellement comme un espace hyperbolique ; tout ça, en faisant le parallèle entre trois sortes de relativités : la relativité restreinte ou minkowskienne/lorentzienne, la relativité galiléenne (antérieure à 1905), et le monde fictif de la relativité « euclidienne » (exploré dans la trilogie Orthogonal de Greg Egan, qui malheureusement utilise le terme « riemannien » que je trouve très mal choisi), cette dernière servant essentiellement comme contrepoint plus facile à visualiser, et où l'espace des vitesses correspond à la géométrie sphérique.

Mes transparents sont ici [lien cassé : ici une version locale], même si je ne sais pas s'ils seront très compréhensibles sans tout le blabla que je prononce pour les expliquer (le blabla a été enregistré, je ne sais pas encore s'il sera mis en ligne ; si c'est le cas, je tâcherai d'éditer cette entrée).

Comme souvent, je me rends compte après coup de quantité de choses que j'aurais pu expliquer et que je n'ai pas dites (ou même pas pensées) : le sens de la polarité par rapport à la conique fondamentale en géométrie hyperbolique ou elliptique ; ou comment utiliser les formules trigonométriques hyperboliques pour calculer l'aberration de la lumière relativiste (je ne la mentionne que brièvement au transparent 30/37) ; ou la notion d'aire sur le plan hyperbolique, sa conservation par le groupe de Lorentz, et l'analogue hyperbolique de la projection azimutale équivalente de Lambert. Mais la continuation logique de mon exposé — qui dressait le parallèle entre la relativité restreinte et certaines géométries de Klein — serait de faire le parallèle entre la relativité générale et les géométries de Cartan, et ça, je dois dire que je n'ai pas encore pris le temps de le comprendre en profondeur (même si j'ai commencé ici).

[#] Par exemple, plutôt que de dire que les protons du LHC vont à 99.9999991% de la vitesse de la lumière (jouez à compter les 9 !), ou qu'ils ont une énergie de 7 TeV, je trouve que c'est plus parlant de dire qu'ils ont une rapidité de 9.6 dans les unités naturelles.

↑Entry #2089 [older| permalink|newer] / ↑Entrée #2089 [précédente| permalien|suivante] ↑

↓Entry #2085 [older| permalink|newer] / ↓Entrée #2085 [précédente| permalien|suivante] ↓

(samedi)

Petite devinette mathématique : un damier irrégulier

Carré divisé en 12×12 rectanglesÀ part mon choix lamentable de couleurs, qu'est-ce que l'image ci-contre a de (mathématiquement) remarquable ?

Les couleurs sont juste censées être une indication. La question porte sur la manière dont le carré est divisé en 12×12 rectangles.

Si on veut des valeurs plus précises, à cinq chiffres significatifs, les largeurs des colonnes en fraction du tout sont :

0.10367, 0.02778, 0.15922, 0.00744, 0.13889, 0.06300, 0.06300, 0.13889, 0.00744, 0.15922, 0.02778, 0.10367

et les hauteurs des lignes sont :

0.06699, 0.11603, 0.06699, 0.06699, 0.11603, 0.06699, 0.06699, 0.11603, 0.06699, 0.06699, 0.11603, 0.06699

Voici quelques faits supplémentaires (peut-être trompeurs, mais vrais) :

  • les nombres (dont des approchants sont) écrits ci-dessus sont tous algébriques (avec des degrés et hauteurs assez petites),
  • le motif d'ensemble est plus ou moins unique (il n'y a rien d'analogue en taille moins de 12×12 sauf en taille 10×10 si on autorise une certaine forme de dégénérescence, et il n'y a pas d'autre solution en 12×12 sauf si on autorise cette dégénérescence).

Je donnerai la solution — qui ne fait pas appel à des choses compliquées — en éditant ce post.

(Les gens qui ont déjà eu la réponse — et notamment les lecteurs de forum maths à l'ENS — n'ont pas le droit de participer ! ☺️)

La réponse () (je ne la cache pas, parce que c'est vraiment trop long) :

Chacune des diagonales du damier irrégulier a la même aire qu'elle aurait dans un damier 12×12 régulier : le rectangle (rouge) du coin en haut à gauche a une surface égale à (1/12)² = 1/144 du carré tout entier, les deux (orange) à sa droite et en-dessous ont, ensemble, une aire de 2/144 = 1/72 du carré entier, les trois rectangles adjacents ont une aire totale de 3/144 et ainsi de suite — n'importe quelle suite de n rectangles consécutivement arrangés en diagonale de bord en bord a une surface totale égale à n/144 du carré. (Ainsi, dans les nombres donnés ci-dessus, 0.10367×0.06699=(1/144), 0.02778×0.06699+0.10367×0.11603=(2/144), et ainsi de suite.) En particulier, avec mon choix de couleurs, chaque couleur couvre une surface égale de l'image (mais la propriété que j'énonce est plus forte).

Ce qui rend la chose remarquable, c'est qu'il n'y a pas moyen de faire ça (en écartant bien sûr le damier régulier) pour une taille moindre que 12×12 si on impose aux largeurs des colonnes et hauteurs des lignes d'être strictement positives (en taille 10×10 il y a une solution ou certaines lignes/colonnes sont réduites à 0). Et en 12×12 c'est, à symétrie près, la seule solution (autre que le damier régulier et les deux qui ont des lignes/colonnes réduites à 0).

Le problème peut aussi se poser avec des dés : on se demande si on peut construire deux dés à m faces, c'est-à-dire deux distributions de probabilités sur m entiers consécutifs (de 0 à m−1 ou de 1 à m, ça ne change rien) de manière que si on tire ces deux dés simultanément et qu'on considère leur somme, on obtienne bien la distribution de probabilité attendue pour la somme sur un tirage de deux dés à m faces non pipés. Pour m<10 la seule façon d'y arriver est d'avoir deux dés non pipés. Pour m=12, les deux distributions de probabilité données par les deux suites de 12 nombres ci-dessus définissent deux dés pipés dont la somme est non pipée.

Maintenant, comment (p/t)rouve-t-on ça ? L'idée initiale est très séduisante : on considère le polynôme de degré m−1 dont les coefficients sont les largeurs des colonnes (soit a0 + a1·x + ⋯ + am−1·xm−1 où les ai sont les largeurs des colonnes) et celui dont les coefficients sont les hauteurs des lignes (b0 + b1·x + ⋯ + bm−1·xm−1) : en les multipliant, on obtient le polynôme (de degré 2m−2) dont les coefficients sont les aires des diagonales (le coefficient constant est a0·b0, c'est la surface du coin en haut à gauche, le suivant est a1·b0 + a0·b1, c'est la surface de la diagonale adjacente, et ainsi de suite). La contrainte demande donc que ce produit soit le même que le produit des deux polynômes représentant une distribution uniforme, autrement dit (1+x+x2+⋯+xm−1)/m (au carré, donc, pour le produit). La question qui se pose est donc d'écrire le polynôme (1+x+x2+⋯+xm−1)2, c'est-à-dire ((1−xm)/(1−x))2 (j'ai retiré le coefficient multiplicatif 1/m de normalisation globale, sachant qu'on peut toujours renormaliser) comme produit de deux polynômes de degré m−1 à coefficients positifs (voire strictement positifs).

Malheureusement, la suite est plus décevante. Le polynôme 1+x+x2+⋯+xm−1 se factorise dans ℝ[x] comme produit des (1 − 2·cos(2iπ/mx + x²) pour i allant de 1 à ⌊(m−1)/2⌋ ainsi que (1+x) lorsque m est pair. On va donc chercher à répartir les exposants (le total devant être 2) sur chacun de ces facteurs de manière à avoir le bon degré de part et d'autre, et n'avoir que des coefficients positifs (ou strictement positifs), et je ne vois pas de façon plus intelligente pour ça que d'essayer toutes les combinaisons possibles, typiquement par ordinateur. S'agissant de m=12, les facteurs de 1+x+x2+⋯+x11 sont (1−√3·x+x²), (1−x+x²), (1+x²), (1+x+x²), (1+√3·x+x²) et (1+x). Et le produit de

(1−√3·x+x²) × (1−x+x²) × (1+x²) × (1+x+x²)2 × (1+x) = 1 + 0.26795·x + 1.53590·x2 + 0.07180·x3 + 1.33975·x4 + 0.60770·x5 + 0.60770·x6 + 1.33975·x7 + 0.07180·x8 + 1.53590·x9 + 0.26795·x10 + x11

et de

(1−√3·x+x²) × (1−x+x²) × (1+x²) × (1+√3·x+x²)2 × (1+x) = 1 + 1.73205·x + x2 + x3 + 1.73205·x4 + x5 + x6 + 1.73205·x7 + x8 + x9 + 1.73205·x10 + x11

donne bien (1+x+x2+⋯+x11)2, les deux ayant des coefficients positifs ; en les renormalisant (pour que la somme des coefficients, c'est-à-dire la valeur en 1, soit 1), on obtient les deux suites de nombres annoncées.

On peut aussi faire varier m et chercher le nombre de manières dont le polynôme (1+x+x2+⋯+xm−1)2 s'écrit comme produit de deux polynômes unitaires de degré m−1 à coefficients positifs, ou strictement positifs : ceci revient à compter le nombre de façons dont on peut fabriquer un damier m×m à diagonales régulières (sans ou avec la contrainte de non-nullité des lignes et colonnes ; et en comptant une fois le damier régulier et deux fois chaque damier régulier à cause de la possibilité d'échanger lignes et colonnes). Ces suites sont majorées par 3m/2⌋ (qui est le nombre total de manières d'écrire le polynôme comme produit de deux polynômes unitaires à coefficients réels, sans autre contrainte). Je trouve :

m12345678 910111213141516 1718192021
pos.11111111 13175799 1315172537
s.pos.11111111 11135799 1311171933

La seconde suite est sans doute plus naturelle (parce que si on va autoriser des coefficients égaux à 0, autant supprimer la contrainte que le degré des polynômes soit exactement m−1 parce que ça correspond à demander la non-nullité du dernier coefficient). Je l'ai soumise à l'OEIS.

↑Entry #2085 [older| permalink|newer] / ↑Entrée #2085 [précédente| permalien|suivante] ↑

↓Entry #2084 [older| permalink|newer] / ↓Entrée #2084 [précédente| permalien|suivante] ↓

(jeudi)

Les labyrinthes de petits théorèmes tordus, tous semblables (ici : Fourier)

Je donne cette année un cours d'Analyse en première année à l'école-qui-s'appelait-ENST. Je ne suis pas du tout analyste, mais j'en profite pour essayer de me cultiver un peu sur le sujet, et apprendre à mettre dans leur contexte les résultats somme toute assez basiques que je leur enseigne. Aujourd'hui j'ai fait cours sur les séries de Fourier, et comme je voulais essayer de mettre au clair les différents résultats relatifs à la convergence et à l'estimation de séries de Fourier, j'ai commencé à essayer de me faire une liste systématique, et je suis tombé sur ce que j'aime appeler un labyrinthe de petits théorèmes tordus, tous semblables (le terme est une référence geek célèbre).

Ce que je veux dire par là est qu'on a un phénomène mathématique sur lequel on montre une propriété, qui suggère quelques nouvelles questions, auxquelles on répond par de nouveaux théorèmes ou des contre-exemples, mais ceux-ci suggèrent encore des questions, et le processus ne converge pas (ou du moins, ne converge pas dans les limites de la patience ou de la mémoire dont je dispose), et à la fin je me retrouve avec une masse de théorèmes que je confonds et où je ne vois plus rien. C'est loin d'être le seul cas où ce me soit arrivé, mais les séries de Fourier sont un exemple assez frappant (et le fait qu'un M. Zygmund ait réussi à écrire deux tomes de 350 pages chacun sur le sujet sans réussir à faire le tour non pas de toutes les questions mais de toutes celles que je me pose naturellement, suggère qu'il y a vraiment un labyrinthe). Petit apercu (tout ceci étant dit pour les fonctions périodiques d'une variable réelle) :

  • Si f est une fonction périodique L1 dont les coefficients de Fourier sont nuls, alors f=0. [Zygmund (I.6.2) ; Katznelson (I.2.7).] Plus généralement (par dualité), ceci vaut pour une mesure borélienne signée finie (coefficients de Fourier-Stieltjes) [Katznelson (I.7.1)] ou une distribution.
  • Les coefficients de Fourier définissent un isomorphisme entre l'espace de Hilbert des fonctions périodiques L2 et l'espace de Hilbert des suites ℓ2 (Parseval-Riesz). [Zygmund, (II.1.12) ; Katznelson (I.5.5).]
  • Si f est une fonction périodique L1, alors ses coefficients de Fourier tendent vers 0 (Riemann-Lebesgue). [Zygmund, (II.4.4) ; Katznelson (I.2.8).]
  • Si μ est une mesure borélienne signée finie sur le cercle, ses coefficients de Fourier-Stieltjes sont bornés (évident).
  • Si μ est une mesure borélienne signée finie sur le cercle, alors μ est sans atome si et seulement si ses coefficients de Fourier-Stieltjes tendent vers 0 en densité (i.e. pour tout ε>0 l'ensemble des coefficients de valeur absolue >ε est de densité nulle) (Wiener).
  • Si f est une fonction périodique dont les coefficients de Fourier sont ℓ1 (la série de Fourier de f converge absolument), alors f est continue.
  • Si f est une fonction périodique Lp pour 1≤p≤2 et q l'exposant conjugué à p, alors les coefficients de Fourier de f sont ℓq (Hausdorff-Young). [Zygmund, (XII.2.3.i) ; Katznelson (I.4.7) & (IV.2.1).] Ceci n'est pas vrai en général pour p>2.
  • Dualement : si 1≤p≤2 et q l'exposant conjugué à p, alors toute suite ℓp est la suite des coefficients de Fourier d'une fonction périodique Lq. [Zygmund, (XII.2.3.ii).] Ceci n'est pas vrai en général pour p>2.

↑Entry #2084 [older| permalink|newer] / ↑Entrée #2084 [précédente| permalien|suivante] ↑

↓Entry #2078 [older| permalink|newer] / ↓Entrée #2078 [précédente| permalien|suivante] ↓

(samedi)

Ce qu'on écrit — en maths — et ce qu'on pense

L'autre jour, dans le cadre de mon cours d'Analyse à Télécom, j'énonce un résultat sur le changement de variable dans les intégrales multiples. Quelque chose comme ceci : si φ:UV est un C¹-difféomorphisme entre des ouverts de ℝn, et si f est une fonction mesurable sur V, alors l'intégrale sur V de f est égale à (et converge dans les mêmes cas que) l'intégrale sur U de (fφ)×|J| où J désigne le déterminant de la différentielle de φ. L'ennui d'un tel énoncé, c'est qu'il est peut-être mathématiquement impeccable, mais ce n'est pas la manière dont on pense à un changement de variable, ça ne montre pas la manière dont on mène en pratique le calcul, parce qu'avec un tel énoncé on ne sait pas où écrire les φ et les J.

La manière dont on fait les choses en pratique, c'est qu'on écrit les variables y1,…,yn sur V en fonction de celles x1,…,xn sur U, on calcule les différentielles dyi, on exprime l'« élément de volume » sur V dy1∧⋯∧dyn en fonction de celui dx1∧⋯∧dxn sur U (le rapport étant justement J), et on remplace simplement ça dans l'intégrale. Par exemple, pour passer des coordonnées cartésiennes aux coordonnées polaires dans une intégrale double, j'écris x = r·cos(θ) et y = r·sin(θ), donc dx∧dy = (cos(θ)·drr·sin(θ)·dθ) ∧ (sin(θ)·dr + r·cos(θ)·dθ) = r·dr∧dθ et je sais tout de suite où et comment placer le facteur r sans me tromper.

Mais expliquer ça, c'est autrement plus difficile qu'énoncer un théorème bien propre. Comme mes élèves ne savent pas ce que c'est qu'une forme différentielle (même sur ℝn), on ne peut pas vraiment expliquer quel sens aurait cet élément de volume dy1∧⋯∧dyn, ni les subtilités sur l'orientation (autrement qu'en dimension 1) : on écrit juste dy1⋯dyn pour l'élément d'intégration, mais du coup on n'explique pas vraiment le mystère (et on n'oriente pas les intégrales).

Bref, j'ai essayé de donner une idée de ce qui se passait, mais je ne sais pas si j'ai réussi à faire passer quoi que ce soit. C'est malheureusement assez fréquent, en maths, qu'on soit tiraillé, pour la pédagogique, entre donner un énoncé rigoureux auquel se rattraper, ou donner une explication utile pour le calcul ou pour l'intuition, avec parfois un peu de mal à faire le lien entre les deux.

↑Entry #2078 [older| permalink|newer] / ↑Entrée #2078 [précédente| permalien|suivante] ↑

↓Entry #2072 [older| permalink|newer] / ↓Entrée #2072 [précédente| permalien|suivante] ↓

(dimanche)

Objets mathématiques fascinants

Un jour il faudra que je fasse un petit catalogue des objets mathématiques qui me fascinent le plus. Ça peut être pour l'élégance hypnotique de leurs symétries (E8, le réseau de Leech) ou pour l'universalité protéenne de leurs structures (l'ensemble de Mandelbrot, l'ensemble des degrés de Turing, le compactifié de Stone-Čech des entiers naturels), ça peut être pour leur existence exceptionnelle (les octonions) ou leur valeur de contre-exemple qui surprend l'imagination (la longue droite), pour la simplicité naturelle de leur construction (l'algèbre de Grassmann-Cayley), ou encore leur centralité pour tout un domaine (le groupe de Galois absolu des rationnels). Si je fais des mathématiques, c'est sans doute beaucoup pour avoir le droit de visiter et admirer ce petit musée des formes extraordinaires (l'ensemble de Mandelbrot est visible pour n'importe qui, il y en a quantité de vidéos, mais les autres objets que j'ai cités sont à mon avis encore plus beaux, simplement ils sont plus difficiles à voir).

À côté de ça, il y a aussi les situations où les maths sont surprenantes. L'exemple le plus bateau est le paradoxe de Banach-Tarski (on peut découper une boule en un nombre fini de morceaux, et déplacer ces morceaux sans changer leur taille de manière à les réassembler pour former deux boules chacune de la taille de la boule d'origine — sans laisser de trous), mais il y en a d'autres. Là aussi, je devrais faire un petit catalogue. Par exemple, saviez-vous que la somme de deux régions convexes du plan dont le bord est C (ou même analytique) est toujours C6 (=six fois continûment dérivable) mais pas forcément C7 ? (Un exemple est formé par les épigraphes de x4/4 et x6/6 ; en fait, la régularité de la somme est C20/3.)

Il y a une célèbre citation de von Neumann : Young man, in mathematics you don't understand things. You just get used to them. (Mon père prétend d'ailleurs que von Neumann l'a piquée à Robert Musil, mais il n'a jamais été capable de me trouver la référence précise, et Google ne semble pas lui donner raison, donc je reste sur von Neumann.) Je ne sais pas si c'est vrai, ou, plus exactement, je ne sais pas s'il y a une différence entre comprendre les choses et s'y habituer. En lisant une démonstration du paradoxe de Banach-Tarski (la référence classique à ce sujet est l'excellent petit livre de Stan Wagon), j'ai l'impression de comprendre pourquoi et comment ce truc fonctionne ; et certainement, depuis le temps que je le connais, il ne me surprend plus trop. Disons qu'on se fait une intuition de la manière dont les objets mathématiques fonctionnent, cette intuition est essentielle pour rechercher ce qui a des chances d'être vrai et ce qui ne l'est probablement pas, cette intuition est parfois prise en défaut et à ce moment-là il faut la modifier, ce qui est d'autant plus facile si on comprend un peu en détail le pourquoi et le comment.

Et parfois les mathématiques sont à la fois très surprenantes et élégantes. Dans mon petit musée des objets mathématiques fascinants, il faut que je mette la sphère de Gromoll-Meyer et la sphère de Kervaire.

En voici une définition concise pour ceux qui la comprendront : on considère le groupe Sp(2) des matrices 2×2 à coefficients dans les quaternions et qui sont unitaires ; là-dessus, on fait agir le groupe Sp(1) des quaternions unité (=de module 1) comme ceci : si u est un quaternion unité et T est dans Sp(2), on définit uT comme la matrice 2×2 obtenue en multipliant T à gauche par u (c'est-à-dire la matrice diagonale (u,u)) et à droite par la matrice diagonale (1,u*) où u* désigne le conjugué (=l'inverse) de u ; ceci définit une fibration de Sp(2) en Sp(1)≅S3, et la base de cette fibration est la sphère de Gromoll-Meyer. Ce qui est incroyablement surprenant, à mes yeux, c'est que l'objet ainsi obtenu est homéomorphe à la sphère de dimension 7 mais pas difféomorphe : il s'agit donc d'une sphère exotique. (Il y a plusieurs choses surprenantes dans l'histoire : l'existence même des sphères exotiques, mais aussi le fait qu'on puisse en donner une construction aussi élégamment algébrique. On peut aussi définir la sphère de Gromoll-Meyer, en tant que variété différentielle, comme l'ensemble des quintuplets (f,p,x,y,z) de nombres complexes tels que f5+p3+x2+y2+z2=0 — j'espère que le 5 est correct — et |f|²+|p|²+|x|²+|y|²+|z|²=1, ce qui est plus simple à comprendre mais assurément moins élégant.)

Quant à la sphère de Kervaire (de dimension 9, disons), elle s'obtient en prenant deux copies de l'espace total du fibré en 5-disques tangentes à la 5-sphère et en les recollant ensemble en identifiant, sur un voisinage d'un point identifié à un 5-disque (sur lequel on a trivialisé le fibré), le 5-disque base d'une copie avec le 5-disque fibre de l'autre copie : le bord de la variété ainsi obtenue est la sphère de Kervaire. (On peut aussi la définir comme l'ensemble des sextuplets (p,x,y,z,u,v) de nombres complexes tels que p3+x2+y2+z2+u2+v2=0 et |p|²+|x|²+|y|²+|z|²+|u|²+|v|²=1.) Elle aussi est homéomorphe mais non difféomorphe à la sphère standard (en dimension 9, cette fois).

Bref, il faudra que je parle un jour de structures exotiques sur les sphères et sur ℝ4, je mets ça dans ma TORANT-list. Parce que ça fait partie de ces choses que je n'arrive vraiment pas à comprendre, ou disons, auquelles je ne me suis pas habitué.

↑Entry #2072 [older| permalink|newer] / ↑Entrée #2072 [précédente| permalien|suivante] ↑

↓Entry #2068 [older| permalink|newer] / ↓Entrée #2068 [précédente| permalien|suivante] ↓

(mercredi)

Comment peut-on courber un espace galiléen ?

La convergence entre le hasard de divagations mathématiques auxquelles je m'étais livré récemment (et qui passaient notamment par le concept de géometrie de Cartan) et du fait qu'on me propose de faire un exposé de vulgarisation sur la relativité (j'en parlerai une autre fois) m'a conduit à d'autres divagations entre la physique et les maths, et à me poser la question parfaitement idiote — et assez technique — suivante, que je vais néanmoins tâcher de raconter : comment peut-on courber un espace galiléen ?

Grossièrement, l'idée est de faire à la « relativité galiléenne » (c'est-à-dire la cinématique de la physique classique, telle qu'elle existait avant Einstein) la même chose qu'on fait pour passer de la relativité restreinte à la relativité générale : courber l'espace-temps.

L'espace-temps galiléen (« plat ») correspond à l'idée naïve qu'on est censé se faire de l'espace et du temps, ou l'idée qu'on s'en faisait avant le passage de MM. Lorentz, Poincaré, Einstein et Minkowski (et le terme d'espace-temps n'était pas utilisé parce qu'il n'est pas particulièrement utile, dans le cadre galiléen, de mettre les deux ensemble : un point de l'espace-temps est juste la donnée d'un point de l'espace à un moment précis). À savoir : le temps est le même pour tous les observateurs ; et les lois de la physique sont invariantes par les transformations suivantes : (0) une translation dans l'espace ou dans le temps, (1) une rotation (constante) de l'espace, et (2) un changement de référentiel donné par un mouvement de déplacement uniforme (=à vitesse constante). Soit concrètement : le résultat d'une expérience physique ne doit pas changer lorsque (0) on la fait à un autre endroit ou un autre moment (à condition bien sûr de déplacer tout ce qui intervient dans l'expérience, y compris la Terre si elle intervient !), (1) on oriente différemment ce sur quoi on mène l'expérience (même remarque), ou (2) on effectue l'expérience dans un laboratoire se déplaçant à vitesse constante ; le point (2) est le moins évident, il constitue le génie de Galilée qui a (au moins selon la légende) effectué des expériences dans des bateaux pour le prouver (du genre : une balle lâchée du haut du mât touche le sol au pied du mât — au moins en l'absence de frottement de l'air — et pas un peu derrière comme on pourrait le penser). Remarquons qu'une conséquence du point (2) est que dire que deux points de l'espace-temps sont « au même endroit » n'a aucun sens à moins qu'ils soient aussi au même moment (j'ai peut-être l'impression d'être assis au même endroit qu'hier, mais la Terre, pendant ce temps, a parcouru quelque chose comme 2.5 millions de kilomètres par rapport au système solaire qui lui-même, etc.). Les transformations de l'espace-temps (translations, rotations, changements de vitesse uniforme) décrites ci-dessus engendrent un groupe appelé groupe de Galilée (ou en fait, deux groupes : le groupe de Galilée homogène, de dimension 6, engendré par (1) et (2), qu'on peut imaginer comme opérant sur les vitesses, et qui est d'ailleurs isomorphe au groupe des déplacements d'un espace euclidien de dimension 3 ; et le groupe de Galilée inhomogène, ou complet, de dimension 10, engendré par (0)–(2), qu'on peut imaginer comme opérant sur les points de l'espace-temps).

L'espace-temps de la relativité restreinte, ou espace-temps de Minkowski, est construit selon des principes analogues à ceci près que maintenant le temps n'est plus absolu, c'est la vitesse de la lumière qui l'est (i.e., quelle que soit la vitesse à laquelle je cours derrière un rayon de lumière, il avancera toujours aussi vite par rapport à moi), et ce simple principe, avec les invariances par translation, rotations et changement de référentiel, correctement interprétées, suffit à fonder toute la cinématique relativiste. Si on a la vision de Felix Klein dans son célèbre programme d'Erlangen, ce qui importe vraiment est le groupe des transformations sur l'espace-temps, et en relativité restreinte les analogues du groupe de Galilée sont le groupe de Lorentz (de dimension 6, analogue du groupe de Galilée homogène) et le groupe de Poincaré (la variante inhomogène, c'est-à-dire incluant aussi les translations, il est de dimension 10).

La relativité générale part du principe que l'espace-temps est un espace courbe (et sans torsion ; cf. une entrée précédente sur le sens de ces deux mots) qui « ressemble localement » à l'espace-temps de Minkowski, et que les objets en chute libre suivent des géodésiques (c'est-à-dire des courbes « aussi droites que possible ») dans un espace-temps courbe. Normalement, il n'y a pas grand-chose à dire, en plus de ça, pour arriver à la relativité générale (il faut cependant bien dire quelque chose de plus car, comme j'aime bien le rappeler, sinon la théorie de Nordström convient aussi). La question que je me suis ingénument posée est : que donnerait une théorie physique fictionnelle partant du principe que l'espace-temps est courbe (et sans torsion) mais ressemble localement à l'espace-temps galiléen (toujours avec le principe que les objets en chute libre suivent des géodésiques). Il est assez facile de se convaincre que cette théorie fictionnelle contient au moins la gravitation à la Newton, mais elle contient plus, parce que l'espace lui-même peut être courbe, parce qu'il y a une sorte de champ « gravitomagnétique », et par ailleurs, comme la relativité générale, elle permet de décrire des changements quelconques de coordonnées et de référentiels (y compris accélérés, en rotation, etc.).

↑Entry #2068 [older| permalink|newer] / ↑Entrée #2068 [précédente| permalien|suivante] ↑

↓Entry #2066 [older| permalink|newer] / ↓Entrée #2066 [précédente| permalien|suivante] ↓

(mercredi)

Le problème des chapeaux de couleur

Une petite addition à la liste que j'avais faite naguère (+1), dans le genre plus proche de ces problèmes-là (en fait, il s'agit d'une variation sur ce que j'appelais l'archi-classique problème des amazones qui tuent leurs maris quand elles savent qu'ils sont infidèles ; il est aussi apparenté au paradoxe de l'examen surprise) :

Le cruel Docteur No a capturé seize mathématiciens. Il les installe autour d'une table et met sur la tête de trois d'entre eux un chapeau blanc, cinq d'entre eux un chapeau rouge, et huit d'entre eux un chapeau noir. Puis il leur tient le discours suivant (rigoureusement exact) : Messieurs, vous êtes tous de parfaits logiciens, et vous savez que je ne vous dirai que la vérité. J'ai placé sur la tête de chacun d'entre vous un chapeau, soit blanc, soit rouge, soit noir. Chacun de vous peut voir la couleur des chapeaux des quinze autres mathématiciens, mais pas du sien. Votre but sera de déduire la couleur de votre chapeau. Pour cela, nous allons procéder en plusieurs tours : à chaque tour, vous écrirez secrètement dans une enveloppe la couleur que vous aurez déduite de votre chapeau, si vous le pouvez, sinon vous laisserez l'enveloppe vide — puis nous dévoilerons le contenu de chaque enveloppe et nous procéderons à un nouveau tour (et ainsi de suite). Vous n'aurez aucun autre moyen de communication. Si une enveloppe contient une mention incorrecte, je vous ferai tous mourir dans d'atroces souffrances (mais cela ne se produira pas car, en tant que parfaits logiciens, vous n'écririez qu'une couleur dont vous êtes logiquement certains). Si trop de tours passent sans que vous ayez tous déduit la couleur de votre chapeau, je vous mettrai aussi à mort (mais nous savons tous que vous écrirez une couleur dès qu'il est logiquement possible de la déduire). Bien, nous allons commencer. Puis, sentant bien que le problème est impossible, le Docteur No ajoute, comme indication : Toutes les couleurs de chapeaux sont représentées (i.e., au moins l'un d'entre vous a un chapeau de chacune des trois couleurs possibles). Comme vous pouvez d'ailleurs le voir. Que va-t-il se passer ?

Le nœud de ce problème existe sous un nombre infini de variantes (par exemple celle-ci), je l'ai souvent appelé le « problème des amazones » parce que j'ai dû le lire d'abord sous cette forme-là[#], mais il faut faire beaucoup de contorsions pour y mettre toutes les hypothèses nécessaires (que je crois avoir correctement réunies). Ici j'ai ajouté quelques petites subtilités pour rendre la chose plus amusante.

(Si on n'a jamais rencontré ce problème, on peut s'échauffer en réfléchissant à ce qui se passe s'il n'y a qu'un seul mathématicien, puis deux, puis trois, quatre, cinq, tous ayant un chapeau blanc, et l'indication du Docteur No est : au moins l'un de vous a un chapeau blanc. Chaque mathématicien supplémentaire apporte un niveau de profondeur supplémentaire dans le raisonnement, au sens où chacun raisonnera sur les raisonnements de tous les autres.)

Bref, que va-t-il se passer ? Réponse :

Le « paradoxe » avec ce problème, c'est que le Docteur No donne comme indication quelque chose qui est manifestement visible de tous les mathématiciens (tous voient bien qu'il y a un chapeau de chaque couleur, et même, puisqu'il y a trois chapeaux blancs, tous voient bien que tous le voient bien, donc tout le monde savait déjà que l'indication du Docteur No était correcte : elle n'apporte en elle-même aucune information nouvelle à qui que ce soit), mais elle est pourtant indispensable à une quelconque déduction (sans l'indication, aucun mathématicien ne peut jamais déduire quoi que ce soit). L'explication, c'est que si tous les mathématiciens savent que l'information donnée par le Docteur No était correcte, le fait qu'il la prononce apporte une information nouvelle, c'est que maintenant non seulement tout le monde le sait, mais tout le monde sait que tout le monde le sait, et tout le monde sait ça, et ainsi de suite. C'est ce qui rend la déduction possible.

En fait, si on veut faire une analyse vraiment rigoureuse du problème, le cadre est assez complexe : a priori, il s'agit de calcul propositionnel modal, avec 16 modalités correspondant à le mathématicien nº1 sait que jusqu'à le mathématicien nº16 sait que (heureusement vérifiant le système standard S5 de la logique modale). Et les modèles d'une telle chose sont, en vérité, assez compliqués. Par exemple, si on veut décrire l'ensemble de toutes les configurations possibles, ce n'est pas très aisé : rien qu'avec un mathématicien, il y a douze « états » possibles (le mathématicien a un chapeau blanc et ne sait rien sur la couleur de celui-ci, il a un chapeau blanc et il sait qu'il n'est pas rouge, il a un chapeau blanc et il sait qu'il n'est pas noir, il a un chapeau blanc et il sait qu'il est blanc, et de façon analogue pour chacune des deux autres couleurs) ; pour deux mathématiciens, il faut tenir compte non seulement de la couleur du chapeau de chacun, et de ce qu'il en sait, mais aussi de ce que l'autre sait qu'il en sait, et ainsi de suite jusqu'à un niveau arbitrairement élevé, ce qui fait une infinité de possibilités — il y a certainement des façons intelligentes de définir des états « utiles » en nombre fini, mais je n'ai pas les idées complètement claires.

On peut aussi penser au problème sous un angle vaguement algorithmique, si un mathématicien voit p autres mathématiciens ayant un chapeau blanc, q avec un chapeau rouge, et r avec un chapeau noir, en fonction de ce qui a été annoncé aux tours précédents, que doit-il mettre dans son enveloppe ? Ce qui est assez étonnant, c'est que si les mathématiciens avaient l'occasion de se concerter entre eux (avant de recevoir les chapeaux, évidemment, mais après avoir reçu l'indication il y a au moins un chapeau de chaque couleur, ou pour être exactement identique à ce que j'ai écrit, il y a au moins deux chapeaux de chaque couleur), leur stratégie serait de toute façon la même : cette concertation ne sert à rien. (Elle est utile, en revanche, s'ils ne reçoivent aucune indication : dans ce cas, ils peuvent décider de prendre le risque de mourir si le Docteur No a cité une couleur pour laquelle il n'a placé aucun chapeau.)

[#] Sans doute dans une des colonnes mathématiques de l'âge d'or de Scientific American : Mathematical Games de Martin Gardner, Metamagical Themas de Douglas Hofstadter ou bien Mathematical Recreations d'Ian Stewart.

↑Entry #2066 [older| permalink|newer] / ↑Entrée #2066 [précédente| permalien|suivante] ↑

↓Entry #2063 [older| permalink|newer] / ↓Entrée #2063 [précédente| permalien|suivante] ↓

(samedi)

Comment vulgariser la géométrie riemannienne ?

La laborieusement interminable écriture de mon texte de vulgarisation sur les octonions m'a amené, de fil en aiguille (géométrie octonionique → espaces projectifs sur les réels, complexes, quaternions et octonions → géométrie riemannienne réelle de ceux-ci) à lire ou relire des choses sur la géométrie riemannienne. Pour ceux qui n'ont aucune idée de ce dont je parle, disons qu'en zéroième approximation[#], il s'agit de la géométrie des espaces courbes (le mot courbe méritant lui-même d'être expliqué, puisqu'il s'agit de courbure intrinsèque) ; et il s'agit[#2] de l'ingrédient mathématique essentiel de la relativité générale, qui explique la gravitation comme une courbure de l'espace-temps[#3].

La géométrie riemannienne est aussi quelque chose qu'on a vraiment envie de vulgariser : parce qu'elle donne l'impression, peut-être trompeuse, qu'il est possible d'en expliquer les idées fondamentales « avec les mains », tant il s'agit d'idées géométriques souvent visuellement « concrètes » ; et aussi parce que la vulgariser aiderait à mieux la comprendre et, accessoirement, à comprendre la relativité générale[#4].

La première difficulté qui surgit, c'est sans doute d'expliquer de quoi il est question : la géométrie riemannienne s'intéresse à la courbure intrinsèque ; or si on propose au profane de visualiser un espace courbe, il va fatalement le visualiser comme une courbe ou une surface à l'intérieur d'un espace euclidien (i.e., plat) de dimension 3, cette tendance étant certainement accentuée par le fait que l'exemple le plus évident soit celui d'une sphère, qu'on imaginera volontiers plongée en dimension 3. Et de même, si on explique que la relativité générale présente l'espace-temps comme courbe, la première image qui vient à l'esprit de tout un chacun est de se dire qu'il doit y avoir un espace plus gros (i.e., ayant des dimensions en plus) à l'intérieur duquel il réside. Or, s'il est vrai qu'on peut toujours plonger une variété riemannienne dans un espace plat de dimension assez grosse (ajouter une seule dimension ne suffit pas toujours), ce n'est une opération ni naturelle, ni très intéressante, et ce serait un mauvais départ que de se servir de cet artifice.

↑Entry #2063 [older| permalink|newer] / ↑Entrée #2063 [précédente| permalien|suivante] ↑

↓Entry #2051 [older| permalink|newer] / ↓Entrée #2051 [précédente| permalien|suivante] ↓

(samedi)

Amplificateurs de probabilités

Dans la série David fait joujou avec les probas et les maths élémentaires, je me suis fait les réflexions à 15 microzorkmids suivantes :

Voilà un scénario typique : on considère un jeu sportif entre deux joueurs qui se joue, disons, par manches indépendantes les unes des autres (chacune étant gagnée par un des deux joueurs), et on souhaite définir une manière de combiner les manches en un match, c'est-à-dire définir une règle qui détermine en fonction des manches déjà jouées si on en joue une nouvelle ou si on déclare un gagnant et dans ce cas lequel. Des exemples de telles règles pourraient être : le gagnant est le premier joueur qui a emporté deux manches (« deux sets gagnants »), ou trois manches (« trois sets gagnants »), ou encore, le gagnant est celui qui a emporté deux manches de plus que son adversaire (le risque étant alors que la partie dure longtemps). On peut aussi imaginer empiler deux niveaux de telles règles, par exemple avoir un jeu qui se joue sous forme de jeux indépendants, une première règle définissant quel joueur emporte une manche en fonction des jeux qu'il a gagnés, et une seconde définissant quel joueur emporte le match en fonction des manches. Voire trois niveaux (points, jeux, manches, match) ou plus.

Faisons l'hypothèse suivante : le joueur 1 remporte une manche quelconque avec probabilité p, le joueur 2 l'emportant donc avec probabilité 1−p, et chaque manche est indépendante des autres. La probabilité que le joueur 1 emporte le match est une fonction f(p) qui dépend de la règle appliquée. La moindre des choses qu'on souhaite, c'est que la règle soit équitable a priori, favorise le meilleur joueur a posteriori, et d'autant plus qu'il est bon, i.e., f(1−p)=1−f(p) et f strictement croissante (ce qui implique notamment f(p)>½ si p>½), mais même f(p)>p pour ½<p<1. C'est ça que je vais appeler un amplificateur de probabilités.

La règle « deux sets gagnants » est sans doute la plus simple (à part celle qui consiste à dire qu'on ne fait qu'une seule manche) : il revient évidemment au même de faire toujours exactement trois manches et de prendre pour gagnant celui qui en a gagné le plus (c'est-à-dire, au moins deux), étant entendu qu'il n'a pas d'incidence mathématique qu'on décide de ne pas jouer la troisième manche lorsqu'elle ne peut pas influencer l'issue du match. La probabilité que le joueur 1 emporte le match est égale à la somme de la probabilité qu'il emporte les trois manches, soit p³, et de la probabilité qu'il emporte deux des manches et perde la troisième, qui peut être n'importe laquelle des trois, soit 3·p²·(1−p) ; ce qui fait, au total, f(p) = 3·p² − 2·p³.

↑Entry #2051 [older| permalink|newer] / ↑Entrée #2051 [précédente| permalien|suivante] ↑

↓Entry #2042 [older| permalink|newer] / ↓Entrée #2042 [précédente| permalien|suivante] ↓

(dimanche)

Matrice de report des voix : mes résultats

Je conclus (enfin, j'espère !, parce que je commence à en avoir un peu marre) la série des trois derniers posts (1, 2 et 3) avec les résultats promis.

D'abord, en petits caractères, voici de façon très détaillée la méthodologie que j'ai suivie.

Premièrement, le jeu de données. S'agissant de 2012, elles sont ici pour le 1er tour et pour le 2d ; s'agissant de 2007, elles sont ici pour le 1er tour et pour le 2d. Pour bien s'entendre, il y a 36791 lignes de données pour 2012, et 36698 pour 2007.

J'ai d'abord retiré tous les départements et autres collectivités d'outre-mer (mais gardé la Corse), au motif que les reports s'y effectuent sans doute de façon différente de la métropole, et aussi parce que certaines ne sont pas détaillées dans le fichier pour 2007. Ensuite, comme je voulais un fichier unifié entre 2007 et 2012, j'ai fusionné toutes les données selon les codes de département et de commune, en ne gardant que les clés qui étaient présentes à la fois en 2007 et 2012 : ceci implique que pour les communes qui ont fusionné entre les deux (par exemple 21084 Blessey et 21551 Saint-Germain-Source-Seine ont fusionné pour former 21084 Source-Seine), je n'ai gardé que la commune ayant le numéro donné à la commune fusionnée, et inversement en cas de scission (par exemple, je supprime des données de 2012 la commune de 52033 Avrecourt qui faisait en 2007 partie de 52332 Val-de-Meuse). Je supprime encore les 15 communes (05014 Barret-sur-Méouge, 05181 Villar-d'Arêne, 10298 Pont-sur-Seine, 14726 Vassy, 31019 Artigue, 31081 Bourg-d'Oueil, 39364 Montrond, 43122 Lissac, 50049 Besneville, 50105 Catteville, 50614 Le Valdécie, 63181 Joserand, 79076 La Chapelle-Saint-Laurent, 80197 Cizancourt et 86241 Saint-Rémy-sur-Creuse) dont la totalité des votes a été annulée par le Conseil constitutionnel sur l'un des quatre tours d'élection considérés. Au final, il reste 36538 lignes de données.

S'agissant des colonnes de données, je conserve, pour chaque élection : le nombre d'abstentions, le nombre de blancs+nuls, et le nombre de votes pour chaque candidat, le total étant toujours égal au nombre d'inscrits. Qui peut, en revanche, changer, même entre les deux tours d'une même élection, oui : mon fichier totalise 42057423 inscrits (sur 44472834 en vérité, c'est-à-dire surtout avec l'outre-mer) au premier tour en 2007, 42057755 au second tour (sur 44472733), et en 2012 : 43250761 inscrits (sur 46028542) au premier tour et 43253197 (sur 46066307) au second.

J'attribue à chaque commune et indépendamment pour chacune des deux élections, une répartition gauche-droite grossière, sur la base des suffrages exprimés au 1er tour, en sommant les voix des candidats que j'ai classés, de façon ad hoc, comme étant « de gauche » d'un côté, « de droite » de l'autre, les centristes comptant pour moitié de chaque côté. J'ai classé de cette manière : en 2007, Besancenot, Buffet, Schivardi, Bové, Voynet, Royal et Laguiller à gauche, Bayrou et Nihous au centre, de Villiers, Le Pen et Sarkozy à droite ; en 2012, Joly, Mélenchon, Poutou, Arthaud et Hollande à gauche, Cheminade et Bayrou au centre, Le Pen, Sarkozy et Dupont-Aignan à droite. Par exemple, ce score répartit Paris en 2007 à 48.8% à gauche (et donc 51.2% à droite) et en 2012 à 55.8% à gauche (et donc 44.2% à droite). Ce score ne sera pas utilisé directement mais simplement pour analyser plus finement les populations d'abstentionnistes et d'électeurs de Le Pen (l'idée étant que les électeurs de Le Pen ne sont pas vraiment les mêmes à Calais et à Cannes et n'ont pas le même comportement au second tour).

Je divise fictivement les populations d'abstentionnistes et de votants pour Le Pen au premier tour en deux dans les proportions données par la répartition gauche-droite grossière évoquée ci-dessus (par exemple, comme j'ai dit qu'à Paris en 2012 la répartition est de 55.8% à gauche et 44.2% à droite, je ferai deux sous-populations des abstentionnistes du premier tour avec 55.8% d'un côté et 44.2% de l'autre, et pareil pour les électeurs de Le Pen du premier tour). J'appellerai ces sous-populations des abstentionnistes de gauche et abstentionnistes de droite et de même électeurs de Le Pen gauche et électeurs de Le Pen droite, ce qui ne signifie pas que je préjuge des choses à leur sujet, encore moins leur vote, il s'agit juste de refléter l'environnement général où ils se trouvent pour modéliser la façon dont ils se comporteront au second tour.

J'ai donc divisé les électeurs du premier tour en N populations : il y a 16 populations en 2007 (une pour chacun des 12 candidats, sauf Le Pen qui en a récupéré deux, plus encore deux pour les abstentionnistes et une pour les blancs-ou-nuls), et 14 populations en 2012. Pour le second tour, je n'ai que 4 populations : les abstentionnistes, les votes blancs-ou-nuls, et les deux candidats. Pour éviter de s'ennuyer avec la variation du nombre d'inscrits, je multiplie les populations du second tour par le rapport d'inscrits pour faire comme si le nombre d'inscrits du second tour était égal à celui du premier tour : ce sera ça ma cible. (On aurait aussi pu imaginer créer une nouvelle population, les absents-au-premier-tour, mais je ne pense pas qu'ils soient suffisamment nombreux ou homogènes pour être modélisables de façon sensée.)

Maintenant, je vais chercher à trouver la matrice de report entre les N populations du premier tour et les 4 populations du second tour (ramenées proportionnellement au nombre d'inscrits du premier tour). Je cherche donc une matrice 4×N de nombres réels entre 0 et 1 (les N colonnes représentant la répartition des votes au second tour, en proportion, de chacune des populations du premier tour). Les contraintes exigées de cette matrice sont :

  • toutes les entrées sont comprises entre 0 et 1,
  • la somme de chaque colonne vaut 1 (i.e., la matrice est stochastique à gauche),
  • la matrice appliquée au vecteur total de chacune des N populations de premier tour pour toute la France doit donner le vecteur total des 4 populations de second tour (par exemple, pour 2012, la matrice appliquée au vecteur [abstention-gauche:3930068 abstention-droite:4126717 blancs-ou-nuls:648166 Joly:786363 Le-Pen-gauche:2923896 Le-Pen-droite:3397710 Sarkozy:9340798 Mélenchon:3899288 Poutou:399753 Arthaud:194859 Cheminade:84939 Bayrou:3173183 Dupont-Aignan:627892 Hollande:9717129] doit donner [abstention:7804808 blancs-ou-nuls:2076910 Hollande:17125029 Sarkozy:16244014]), ces deux vecteurs étant évidemment de même total à savoir le nombre d'inscrits du premier tour.

Les deux derniers points déterminent N+3 conditions linéaires indépendantes (une pour chaque colonne et une pour chaque ligne, sachant qu'une quelconque de ces conditions peut être éliminée comme découlant de toutes les autres).

Je cherche maintenant la matrice, vérifiant ces contraintes, qui parmi les matrices vérifiant ces contraintes réalise la plus petite somme des erreurs quadratiques sur toutes les communes (enfin, parmi les 36538 communes de mon tableau) : les erreurs étant la différence, en nombre total de voix, entre l'application de la matrice au vecteur donnant les populations de premier tour, et le vecteur de second tour (ramené au nombre d'inscrits du premier tour) — on somme donc les carrés des erreurs sur chacune des quatre composantes du vecteur et sur chacune des 36538 communes. Ce calcul est un problème de programmation quadratique en 4N variables, que j'ai résolu avec la fonction qp de GNU Octave.

On peut éventuellement ajouter une contrainte demandant que le report d'un candidat du premier tour qui est admis au second tour soit parfait sur ce candidat lui-même : je n'ai pas eu à ajouter cette contrainte pour 2012 (la solution trouvée vérifiait déjà cette contrainte) ; pour 2007, la solution vérifiait cette contrainte sur Nicolas Sarkozy, et seulement approximativement sur Ségolène Royal (la matrice trouvée reportait 3% des voix de Ségolène Royal du premier tour sur l'abstention au second tour, et seulement à 97% sur elle-même) : ajouter de force la contrainte ne changeait que de quelques pour cent les reports sur les autres candidats, une précision à laquelle je ne prétends de toute façon pas, donc je l'ai introduite pour plus de propreté.

Évidemment, pour indiquer les résultats finaux, il faut réagréger les populations qui ont été artificiellement séparées, c'est-à-dire les « abstentionnistes-de-gauche » et les « abstentionnistes-de-droite », et de même « Le Pen-gauche » et « Le Pen-droite ».

Par contre, je n'ai finalement pas agrégé ensemble dans les calculs les candidats trop semblables (Arthaud et Poutou, ou bien Cheminade avec les blancs-et-nuls), au motif que de toute façon les résultats calculés pour eux sont probablement dénués de sens de toute façon mais que les garder séparés dans les calculs permet peut-être d'obtenir de meilleurs résultats sur les autres candidats (après tout, tout prédicteur en entrée est bon à prendre, je suppose). J'agrège ces résultats uniquement dans le résultat que j'indique :

Pour 2012 :

1er tour→
↓2d tour
Abstentions Blancs+nuls + Cheminade Joly Le Pen Sarkozy Mélenchon Poutou + Arthaud Bayrou Dupont-Aignan Hollande
Abstentions 79% 0% 0% 23% 0% 0% 0% 0% 0% 0%
Blancs+nuls 0% 46% 0% 15% 0% 0% 40% 9% 44% 0%
Hollande 6% 43% 74% 6% 0% 98% 60% 42% 25% 100%
Sarkozy 15% 10% 26% 56% 100% 2% 0% 49% 31% 0%

Pour 2007 :

1er tour→
↓2d tour
Abstentions Blancs+nuls Besancenot + Schivardi + Laguiller Buffet Bayrou Bové + Voynet De Villiers Royal Nihous Le Pen Sarkozy
Abstentions 82% 0% 0% 0% 17% 0% 0% 0% 0% 0% 0%
Blancs+nuls 0% 75% 1% 0% 11% 1% 0% 0% 14% 9% 0%
Sarkozy 13% 0% 5% 0% 31% 0% 100% 0% 86% 85% 100%
Royal 5% 25% 94% 100% 41% 99% 0% 100% 0% 6% 0%

Évidemment, ces tableaux sont à prendre avec énormément de pincettes ! Pour avoir une idée de l'imprécision, on peut comparer le tableau pour 2007 ci-dessus avec celui donné dans une entrée précédente (où les principales différences étaient que (a) je n'avais pas scindé les abstentions et Le Pen, (b) je n'avais pas imposé les contraintes linéaires sur les lignes de la matrice, et (c) je n'avais pas exclu l'outre-mer) : je pense que mon nouveau tableau est un poil meilleur, mais il ne l'est sans doute pas énormément, donc il faut prendre l'énorme différence dans les scores de reports de Frédéric Nihous comme signifiant simplement on n'en sait rien (même si le nouveau tableau suggère quand même plutôt que son électorat était de droite). Évidemment il est invraisemblable que l'électorat de Bové et Voynet se soit reporté à 99% sur Ségolène Royal en 2007, ou celui de Mélenchon à 98% sur Hollande en 2012, ou autres colonnes de ce genre, donc ces reports doivent simplement être considérés comme signifiant que l'algorithme a correctement classifié ces candidats comme étant de gauche, ou a contrario De Villiers comme étant de droite (on pourra m'objecter que ma méthodologie supposait de toute façon de classifier a priori les candidats grossièrement à gauche ou à droite, mais en fait les scores dont je parle sont relativement robustes à cette classification).

↑Entry #2042 [older| permalink|newer] / ↑Entrée #2042 [précédente| permalien|suivante] ↑

↓Entry #2041 [older| permalink|newer] / ↓Entrée #2041 [précédente| permalien|suivante] ↓

(mercredi)

Encore des considérations sur les statistiques électorales

Pour ceux qui auraient la flemme de lire les deux entrées précédentes (ici et ), la question qui m'intéresse est la suivante : peut-on, à partir des données électorales détaillées des deux tours d'une élection présidentielle, estimer statistiquement la matrice de report des voix d'un tour sur l'autre, c'est-à-dire, la proportion de chacun des types d'électeurs du premier tour (électeurs de chacun des candidats + nuls + abstentionnistes) qui a eu tel ou tel vote au second tour (l'un des deux candidats, ou le nul, ou l'abstention) ? Je voudrais faire cette analyse pour les élections présidentielle de 2007 et de 2012[#].

Des commentaires éclairants sur les deux dernières entrées font que j'y vois plus clair : d'abord concernant la terminologie, on parle d'inférence écologique parce qu'il s'agit de reconstituer des comportements individuels à partir d'agrégats (on sait simplement le nombre total de votes de chaque type au premier et au second tour dans chacune des ∼37000 communes de France). Ce type d'inférence est hasardeux en général, et l'idée naïve d'appliquer une simple régression linéaire peut donner des résultats aberrants ou faux quoique apparemment plausibles. Un exemple célèbre du paradoxe écologique est celui souligné en 1950 par W. Robinson, qui observe qu'en faisant une régression entre (état par état) le nombre de personnes nées à l'étranger et le nombre de personnes sachant lire et écrire sur les données du recensement de 1930 aux États-Unis, il observe une corrélation positive, i.e., plus un état compte de personnes nées à l'étranger, plus il compte de personnes sachant lire et écrire : peut-on en conclure que les personnes nées à l'étranger savent plus lire et écrire (dans la population des États-Unis de 1930) ? non, l'explication est simplement que les personnes nées à l'étranger et ayant immigré aux États-Unis ont eu tendance à s'installer dans des états où la population (native) savait plus lire et écrire, par exemple parce que ces états étaient plus riches. Ce texte résume un peu le problème ainsi que différents éléments de réponse.

Les deux problèmes que je répertoriais dans l'entrée précédente sont : primo, que le nombre de coefficients que je cherche à estimer est important et que les données ne sont pas assez nombreuses, ou surtout pas assez dispersées pour permettre une estimation raisonnable (un commentateur me signale que la taille de l'échantillon pour avoir des coefficients peu bruités croît exponentiellement avec le nombre de coefficients à déterminer) ; secundo, qu'on a des effets que j'appelle non-linéaires et qui sont en vérité la même chose que le paradoxe écologique décrit ci-dessus : les abstentionnistes du premier tour, pour ceux qui se mobilisent au second tour, par exemple, n'ont pas le même profil dans les communes qui votent globalement à gauche que dans celles qui votent globalement à droite (si on y réfléchit, il s'agit du même phénomène que dans l'exemple de Robinson mentionné ci-dessus : une hétérogénéité des populations concernées).

Différentes méthodes ont été proposées pour essayer de faire quand même cette fameuse inférence écologique. La méthode consistant à faire une simple régression linéaire a été analysée par L. Goodman dans les années '50, qui montre (si je comprends bien, parce que je n'ai pas pu avoir accès aux articles) qu'elle fonctionne bien sous des hypothèses qui, dans mon cas, doivent dire grosso modo que la répartition des reports de chaque type d'électeurs du premier tour n'est pas corrélé à la configuration des votes dans la commune (comme je le signale ci-dessus, c'est sans doute assez faux, par exemple dans le cas de l'abstention ou dans une moindre mesure du vote pour Le Pen ; ça me semble plus plausible pour le vote Bayrou). Divers statisticiens ont eu, comme moi, l'idée de borner les coefficients et de faire une régression linéaire contrainte (je n'ai regardé que très sommairement, mais ce papier et celui-là ont l'air de faire des choses de ce genre : le second, d'ailleurs, semble extrêmement proche de ce que j'ai fait).

Des méthodes plus sophistiquées existent : on me signale que Gary King en a écrit tout un livre, où il propose une « solution » au problème ; comme je n'ai pas accès à ce livre, je dois me contenter d'en lire des recensions et commentaires, et des résumés de la méthode de King, et je note que cette dernière ne fait pas l'unanimité. Voir par exemple ce texte, écrit par un critique. Le principal problème que j'ai, moi, est d'ordre pratique : l'implémentation de la méthode pour [le programme de statistiques] R, écrite par King lui-même, est limitée à des matrices 2×2 (or celle qui m'intéresse est 4×14 pour 2007 et 4×12 pour 2012) ; de même, ce package, qui implémente une autre méthode « sophistiquée » d'inférence écologique, est limité de la même façon ; et toutes ces méthodes sont algorithmiquement bien trop pénibles pour être implémentées de novo en un temps raisonnable. Donc il faut bien que je me contente de quelque chose de plus simple.

D'un autre côté, je pense qu'il est raisonnable, pour le problème considéré, de se contenter de quelque chose de plus simple : notamment parce que les populations des différents votes au premier tour sont relativement homogènes quand il s'agit de prévoir le vote au second tour (contrairement aux exemples classiques de « paradoxe écologique » où on relie des variables très différentes et mal corrélées). Un exemple extrême est évidemment la population d'électeurs au premier tour d'un des candidats qui passent au second tour : il est évident qu'une énorme majorité d'entre eux votent encore pour le même candidat au second tour — ici, la prédiction est presque parfaite.

Je défends donc finalement mon idée de rester sur une régression linéaire, avec trois principales idées que j'ai déjà exposées pour améliorer la qualité des chiffres :

  1. Contraindre les coefficients de la régression à être entre 0 et 1 et de somme 1 pour chaque colonne. (Comme je l'ai signalé, je ne suis pas le premier à faire ça.) Autrement dit, j'effectue une régression linéaire contrainte (je minimiser la somme des erreurs quadratiques parmi les matrices vérifiant les contraintes ci-dessus). L'idée sous-jacente est que certains coefficients sont connus trop grossièrement, leur valeur calculée naïvement peut être délirante, cette contrainte assure qu'ils seront tronqués à quelque chose de raisonnable (ce qui, du coup, assure de répercuter une erreur déraisonnable sur d'autres coefficients).
  2. Regrouper les candidats du premier tour trop petits et ayant un profil sociologique proche (ça ne sert à rien d'essayer de déterminer séparément les reports du vote pour Poutou et du vote pour Arthaud, ou du vote pour Cheminade et du vote blanc/nul).
  3. À l'inverse, scinder les populations du premier tour qui risquent d'être sociologiquement trop inhomogènes (abstentionnistes et vote Le Pen) en sous-populations artificielles dans les mêmes proportions qu'une proportion gauche-droite approximative sur la commune. Ceci permet (au prix de nouveaux coefficients à déterminer !) d'introduire un effet non-linéaire relativement raisonnable et donc de diminuer l'effet d'inhomogénéité de ces populations (et le « paradoxe écologique » qui va avec).

Je donnerai les chiffres que j'obtiens dans la prochaine entrée (qui sera, j'espère, la dernière sur ce sujet !), parce que je suis fatigué d'avoir écrit tout ça. Mais disons qu'ils sont assez plausibles (évidemment, il faut imaginer qu'ils ne sont que des ordres de grandeur !) et qu'ils prédisent, par exemple, que les électeurs de François Bayrou du premier tour se sont plus reportés sur Sarkozy que sur Hollande en 2012, alors qu'en 2007 ils s'étaient plus reportés sur Royal que sur Sarkozy — or ceci est conforme aux sondages directs sur la question ainsi qu'aux analyses des politologues.

[#] J'ai fini par obtenir les données du second tour de 2012, qui étaient effectivement sur RegardsCitoyens.org comme on me l'avait soufflé, mais bien cachées et pas à l'endroit où on les attendait. Elles sont par ailleurs un peu incomplètes puisqu'il y manque la Corse, mais peu importe.

↑Entry #2041 [older| permalink|newer] / ↑Entrée #2041 [précédente| permalien|suivante] ↑

↓Entry #2039 [older| permalink|newer] / ↓Entrée #2039 [précédente| permalien|suivante] ↓

(lundi)

De la difficulté de faire une régression linéaire contrainte en politique

Dans l'entrée précédente, j'ai suggéré l'idée de faire une régression linéaire multivariée entre les deux tours des résultats de l'élection présidentielle, c'est-à-dire, essayer de calculer quelle combinaison linéaire des résultats du premier tour de la présidentielle (considérés comme un vecteur de N+2 nombres, à savoir le nombre de voix pour chacun des N candidats + bulletins blancs/nuls + abstentions) approche le mieux, sur l'ensemble des communes de France, les résultats du second tour (considérés comme un vecteur de 4 nombres, pour 2 candidats + blancs/nuls + abstentions). J'espérais[#] — un peu naïvement comme on va le voir — que ce calcul permettrait de connaître la matrice de reports des voix, c'est-à-dire, la proportion, dans chacun des N+2 votes possibles au premier tour, des 4 votes possibles au second tour : par exemple savoir que les électeurs de François Bayrou au premier tour se seraient reportés à 30% sur l'abstention, à 5% sur le vote blanc, à 35% sur Nicolas Sarkozy et à 30% sur François Hollande (chiffres imaginaires mais pas aberrants).

Les résultats du second tour n'étant pas encore disponibles sur www.data.gouv.fr au moment où j'écris, je me suis dit que j'allais m'exercer sur les résultats de 2007 (pour calculer les reports entre les deux tours de celle-ci soit, de façon plus osée, entre 2007 et 2012). Je passe sur les différentes petites crottes de ragondin rencontrées en chemin pour préformater les données sous une forme sympathique (par exemple les communes qui ont eu la fort sotte idée de fusionner ou de se séparer ; je passe aussi sur le fait qu'il n'y a pas de version détaillée des résultats de Paris, parce que Paris a le malheur d'être une unique commune). Disons que j'ai un gros tableau de données raisonnables, d'où j'ai retiré tout ce qui me chagrine.

Il n'est alors pas difficile de faire les régressions linéaires, avec un programme comme R[#2]. C'est-à-dire trouver les (2+2)×(12+2)=56 coefficients tels que, pour chaque vote possible au second tour, le nombre de ces votes soit au mieux prédit par la combinaison, affectée par les coefficients correspondants, des 14 votes possibles au premier tour (il y avait 12 candidats en 2007, ce qui fait 14 avec blancs et abstention). Il est assez facile de se convaincre, dans la mesure où le nombre d'inscrits ne change pas entre les deux tours (ce qui est quasiment vrai — pas rigoureusement, et ça fait partie des petites crottes de ragondin — mais suffisamment pour qu'on puisse faire comme si), que la somme des coefficients sur une colonne de cette matrice (c'est-à-dire pour chaque vote possible de premier tour) vaut 1. Maintenant, j'espérais que quelque chose ferait que ces coefficients seraient aussi tous positifs, et auraient l'interprétation naïve que j'ai décrite ci-dessus comme matrice de transfert des voix. Or ce n'est pas le cas, et voici la matrice des coefficients :

1er tour→
↓2d tour
Abstentions Blancs/nuls Besancenot Buffet Schivardi Bayrou Bové Voynet De Villiers Royal Nihous Le Pen Laguiller Sarkozy
Abstentions 0.8519 −0.4145 0.1060 0.0578 −0.2845 0.1705 0.5994 −0.5476 0.0281 0.0002 −0.5410 −0.0405 0.3789 0.0047
Blancs/nuls 0.0035 0.4859 0.1085 0.0113 0.5714 0.0867 −0.1229 0.0775 0.0596 −0.0019 0.2442 0.0119 0.2184 0.0153
Sarkozy 0.0910 0.5596 −0.0944 −0.0309 0.8717 0.3499 0.0586 0.9280 0.9056 −0.1129 0.7721 0.9979 −1.2629 1.0427
Royal 0.0532 0.3785 0.8656 0.9573 −0.1743 0.3938 0.5116 0.5719 0.0021 1.1131 0.5122 0.0283 1.6956 −0.0627

Le fit linéaire est excellent : même si je ne sais pas lire exactement les données de marges d'erreur que R me sort, je sais lire qu'elles sont très faibles (par exemple s'il me dit que 99.99% de la variance est expliquée par ce modèle linéaire, ou que dans 50% des communes l'écart est inférieur à 6 voix) ; bref, ces coefficients ont un sens. Mais pas exactement celui que je veux !

Il est relativement concevable que 85% des abstentionnistes du premier tour en 2007 l'aient encore été au second, tandis que 9% seraient allés voter Sarkozy et 5% Royal ; ou que les électeurs de Bayrou se soient reportés à 17% sur l'abstention, à 9% sur le vote blanc, à 35% sur Sarkozy et à 39% sur Royal : j'y crois assez ; ou encore que, comme le tableau le suggère, ceux de Villiers aient voté à 91% pour Sarkozy au second tour tandis que 3% se seraient abstenus et 6% auraient voté blanc. Mais il est impossible que 93% des électeurs de Voynet aient voté Sarkozy au second tour, 57% pour Royal, et un pourcentage négatif, −55%, se soient abstenus.

C'est assez perturbant : ce tableau montre des chiffres relativement sensés, dans un monde où un vote négatif serait possible. ☺️

Bon, ben si les chiffres ne veulent pas d'eux-mêmes être raisonnables, il n'y a qu'à les forcer à l'être : je peux demander à chercher, après tout, quelle est la matrice à coefficients positifs, où chaque colonne a pour somme 1, et qui réalise la meilleure approximation linéaire parmi celles vérifiant ces contraintes : on parle de régression linéaire avec contraintes. Il s'agit là d'un problème d'optimisation quadratique (avec contraintes linéaires, et terme quadratique positif défini) : quelque chose qu'on sait très bien faire. En principe, R a ce qu'il faut pour y arriver : mais nouvelle petite crotte de ragondin, ce package ne marche pas chez moi, il prétend que mes contraintes (=la positivité des variables) sont impossibles à satisfaire, je ne sais pas ce qu'il a fumé. À la place, j'ai dû passer par Octave, qui est encore plus pénible à manipuler et que je connais encore moins, mais enfin qui sait faire le boulot (quand on réussit à exporter les matrices du problème de R vers Octave, ce qui n'est pas la chose la plus agréable qui soit).

Voilà ce que ça donne :

1er tour→
↓2d tour
Abstentions Blancs/nuls Besancenot Buffet Schivardi Bayrou Bové Voynet De Villiers Royal Nihous Le Pen Laguiller Sarkozy
Abstentions 0.8424 0.0000 0.0000 0.0000 0.0000 0.1679 0.0275 0.0000 0.0000 0.0000 0.0000 0.0000 0.0000 0.0000
Blancs/nuls 0.0424 0.2600 0.0000 0.0000 1.0000 0.1093 0.0000 0.0000 0.0026 0.0000 0.5503 0.0004 0.0000 0.0000
Sarkozy 0.0495 0.0900 0.0000 0.0000 0.0000 0.3283 0.0000 0.0000 0.9974 0.0000 0.1999 0.9996 0.0000 1.0000
Royal 0.0657 0.6500 1.0000 1.0000 0.0000 0.3945 0.9725 1.0000 0.0000 1.0000 0.2498 0.0000 1.0000 0.0000

De nouveau, il est relativement raisonnable de penser que les électeurs de François Bayrou au premier tour en 2007 se seraient divisés au second tour entre l'abstention à 17%, le vote blanc à 11%, Sarkozy à 33% et Royal à 39% (les chiffres diffèrent très peu du tableau précédent, et sont toujours crédibles). À la limite, il n'est pas totalement délirant d'imaginer que, avec la précision des mesures, près de 100% des électeurs de Marie-George Buffet, ou même d'Olivier Besancenot, se soient reportés sur Ségolène Royal au second tour, comme d'ailleurs les électeurs du premier tour de Ségolène Royal elle-même. Mais alors croire que les électeurs de Gérard Schivardi au premier tour auraient tous voté blanc au second (sans s'abstenir, mais vraiment voté blanc), ou croire que ceux qui ont voté blanc au premier tour auraient été 65% à voter pour Royal au second, ce n'est, comme qui dirait, pas très crédible. Je suis aussi amusé du 99.96% de report calculé de Le Pen sur Sarkozy (les 0.04% restants ayant censément voté blanc, c'est très précis) !

Voici donc la question à 100¤ : ces chiffres ont-ils une quelconque signification en rapport avec la réalité, ou un quelconque intérêt pour l'analyse politique ? À défaut, y a-t-il un autre traitement statistique que je puisse mener pour en obtenir de meilleurs ? Et en tout état de cause, quand (et si) le ministère de l'intérieur se sortira les doigts du c** pour fournir les chiffres complets du second tour de 2012 en Open Data, sera-t-il intéressant de mener la même analyse ou doit-on considérer que c'est du temps perdu ?

[#] Pourquoi espérer ça ? Parce que si les reports de voix du premier vers le second tour se font à peu près de la même façon partout, et notamment, indépendamment de ce pour quoi votent les autres électeurs de la commune, ce qui a priori ne semblait pas une hypothèse délirante, alors on devrait bien retomber dessus en faisant une régression linéaire.

[#2] Programme au nom incroyablement stupide quand on pense à la difficulté que cela cause de chercher dans Google des informations sur un truc à une lettre.

↑Entry #2039 [older| permalink|newer] / ↑Entrée #2039 [précédente| permalien|suivante] ↑

↓Entry #2037 [older| permalink|newer] / ↓Entrée #2037 [précédente| permalien|suivante] ↓

(vendredi)

Le jugement de paris : comment établir une cote ?

Une obscure province des États-Unis d'Europe va bientôt tenir l'élection de son gouverneur. Les deux candidats encore en course s'appellent M. Sarlande et M. Holkozy. Toutes sortes d'instruments sont utilisés pour mesurer l'état de l'opinion de l'électorat avant cette échéance (sondages, pronostics de politologues et autres boules de cristal), mais au final on aimerait avoir des résultats lisibles sous la forme M. Sarlande a x% de chances d'être élu gouverneur, M. Holkozy a (100−x)% de chances. Déjà, il est un peu difficile de donner un sens à une telle affirmation : si je prétends que M. Sarlande a 85% de chances et M. Holkozy en a 15%, que l'un ou l'autre soit élu, on ne pourra pas me dire que j'avais tort (après tout, les deux nombres étaient strictement positifs) ; or l'expérience (=l'élection) n'a lieu qu'une fois, on ne va pas la répéter d'une manière qui permette de donner un sens statistique aux probabilités.

On pourrait cependant faire des statistiques pour savoir si je suis un fin analyste politique. Si, par exemple, à chaque fois qu'il y a une élection je fais un pronostic du style le candidat 1 a une probabilité q1 d'être élu, le candidat 2 en a q2, le candidat 3 en a q3, etc. (la somme ∑i(qi) des probas annoncées valant 1), si c'est le candidat numéro i qui est effectivement élu on m'attribue un score de fiabilité de valeur log(n)+log(qi) où n est le nombre total de candidats. (Pourquoi log(qi) ? Parce qu'il est facile de se convaincre que la stratégie optimale pour maximiser son succès dans ce contexte, si on connaît les « vraies » probabilités pi, consiste à annoncer effectivement qi=pi, auquel cas on a une espérance de gain de l'opposé de l'entropie de Shannon de la distribution, plus le terme ajouté log(n) (=l'entropie d'une distribution uniforme sur les candidats) qui est là pour assurer qu'on ne gagne ni ne perd rien en faisant la prévision triviale de donner la même proba qi=1/n à chaque candidat.) Par exemple, quand je prédis 85% de chances à M. Sarlande et 15% à M. Holkozy, il convient d'ajouter 0.77 logons à mon score de fiabilité si c'est le premier qui est élu et d'y retranche 1.74 logons si c'est le second qui est élu. Et si mes chiffres sont corrects, mon espérance de score est de 0.39 logons. (Le mot logon indiquant que j'ai pris des logs base 2.) Si on somme ce score fiabilité sur un grand nombre de prévisions, on peut comparer mes capacités d'analyse à celles d'autres analystes. Je me dis souvent qu'on devrait faire des concours de prévisions de ce genre entre analystes politiques.

Bon, maintenant, comme les gens aiment bien jouer aux jeux de hasard, inévitablement, on va vouloir transformer cette question d'évaluer les chances en un pari. La conversion est la suivante : dire que je considère que M. Sarlande a x% de chances d'être élu et que M. Holkozy en a (100−x)%, ça signifie que je suis prêt à accepter de payer x¤ pour un contrat qui me promet 100¤ si c'est M. Sarlande qui gagne, et dualement (100−x)¤ pour un contrat qui me promet 100¤ si c'est M. Holkozy qui l'emporte. Il y a donc moyen de mettre en place un marché de tels contrats, laisser faire l'axiome libéral de l'efficience des marchés, et voir ce qu'il en résulte. C'est ce que fait le site intrade.com (dont le fonctionnement est résumé ici), et sur lequel on peut notamment voir le cours de MM. Sarlande et Holkozy ici et (à moins que ce soit le contraire). Ces cours (le prix auquel s'échange un contrat je paie 10$ en cas d'élection de Untel) se lisent assez directement comme des probabilités, c'est assez agréable. Il serait intéressant de les évaluer sur un grand nombre d'élections selon le score de fiabilité que je propose plus haut. À vrai dire, je ne suis pas trop convaincu par l'efficience de ces marchés, qui ont des volumes assez petits dont les acteurs sont largement des Américains pas forcément bons analystes de la situation politique française (même si ceux qui parient, évidemment, doivent se renseigner). La logique voudrait que j'intervinsse moi-même dans le marché si je m'estime meilleur analyste (ou simplement pour acheter une assurance contre l'élection d'un candidat qui me déplairait), mais j'ai assez peu de confiance dans ce genre de site et dans mes chances de récupérer effectivement une grosse somme d'argent si je parie comme je le pense.

Un système apparenté mais différent est utilisé par les bookmakers anglais : il s'agit cette fois de cotations (on n'échange pas des contrats mais on place des paris à une certaine cote), et on peut voir ici une synthèse des cotes qu'ils attribuent (c'est un peu pénible à lire : le système traditionnel d'affichage de la cotation indique la fraction de la mise qu'on récupère en plus de celle-ci si on a raison sur la prévision — sachant que si on a tort on perd tout ; alors que le système décimal indique combien on récupère au total, mise comprise, si on a raison, comme un nombre décimal).

J'en viens à la question qui m'a pas mal tracassé : comment fait-on, au juste, pour établir une cote de paris ? (Autrement dit, je veux imaginer un système où chacun peut décider de placer un pari sur un des candidats, à une cote instantanée déterminée automatiquement en fonction des paris précédents, pari qui sera payé par une autorité centrale organisatrice, et pas un système de marché comme sur intrade.com ; notamment, une personne doit pouvoir parier même si elle est seule à le faire.)

Une première idée naïve pour un système de paris pourrait être ceci : tous ceux qui le veulent placent un pari de la somme qu'ils veulent sur un des deux candidats, toutes ces sommes sont mises en commun (mettons que u zorkmids aient été pariés sur Sarlande et v sur Holkozy), et lorsque le gagnant est connu, la somme totale u+v est redistribuée à ceux qui ont parié sur ce gagnant, proportionnellement à leur mise (donc par exemple si c'est Sarlande qui gagne, la mise de ceux qui ont parié sur lui est multipliée par (u+v)/u, autrement dit ils emportent v/u fois leur mise en plus de celle-ci). Ce système est extrêmement simple, mais souffre de défauts rédhibitoires : essentiellement, la cote est la même pour tous et n'est connue qu'à la clôture des paris et ne dépend pas du moment où on a parié — ce qui va conduire à des paris de dernière minute alors que le résultat de l'élection se précise, et pénaliser les parieurs de la première heure qui auraient une vision claire bien en avance. On peut imaginer un tel système où les paris seraient clos à une date butoir, ou renouvelés dans le temps, ou ce genre de choses, mais on ne résout pas vraiment le problème.

Ensuite, je me suis imaginé la chose suivante : lorsqu'on parie une somme sur l'un des deux candidats, la cote instantanée utilisée est donnée simplement par le rapport entre la somme totale qui a été pariée sur l'un et celle qui a été pariée sur l'autre. Plus exactement, le système serait le suivant : initialement, l'autorité centrale place 100¤ (disons) comme somme fictive pariée sur Sarlande et autant sur Holkozy ; puis, si à un instant donné u zorkmids ont été pariés sur le premier et v sur le second, et si je veux miser δ (une somme infinitésimale) sur Sarlande, je récupérerai δ·(u+v)/u (c'est-à-dire ma mise δ plus encore δ·v/u de bonus) si j'ai eu raison et 0 (=ma mise est perdue) si j'ai eu tort. On convient que les cotations sont modifiées instantanément : pour parier une somme non infinitésimale, il faut diviser celle-ci en mises infinitésimales et faire l'intégrale qui convient — je n'insiste pas là-dessus. L'ennui c'est qu'avec ce système, les pertes de l'autorité centrale ne sont pas bornées : si après la mise fictive initiale de 100¤ de chaque côté je suis seul à parier et que je mise A sur Sarlande, et si j'ai gagné, je récupère ma mise A plus un gain de 100¤·log(1+(A/100¤)) payé par la banque (comme on le vérifie en calculant l'intégrale 100¤ (A+100¤) ( u+100¤ u ) du — ici écrite en MathML — qui vaut A + 100¤ · log ( 1 + A100¤ ) ). La divergence est certes logarithmique, mais elle est là (sans regarder le détail de l'intégrale, on voit bien que la divergence doit être logarithmique parce que le gain varie comme l'inverse de u).

Voici comment on peut y remédier. Disons que la banque (=l'autorité qui mène les paris) veut limiter ses pertes à 100¤ dans le pire cas. Elle met donc initialement 100¤ dans deux comptes, le compte u somme pariée sur Sarlande et restant à distribuer et le compte v somme pariée sur Holkozy et restant à distribuer. Si je veux miser δ (une somme infinitésimale) sur Sarlande, ce δ est ajouté à u comme précédemment, et placé à la même cote que précédemment (je récupérerai δ·(u+v)/u en cas de victoire de Sarlande, c'est-à-dire ma mise plus δ·v/u), mais cette fois je déduis la somme δ·v/u du compte v, puisque c'est à partir de là que je paie les gains. Il est facile de se convaincre que dans ce système, le produit u·v (ou, si on veut, la moyenne géométrique entre les deux) reste constant ; la banque réalise un bénéfice net de v−100¤ si c'est Sarlande qui gagne, et u−100¤ si c'est Holkozy, ses pertes sont donc minorées dans le pire cas (le reste des gains éventuels venant des mises des autres joueurs). Cette fois, si après la mise fictive initiale de 100¤ de chaque côté je suis seul à parier et que je parie A sur Sarlande, et si j'ai gagné, je récupère ma mise A plus un gain de 100¤·(A/(100¤+A)) payé par la banque, puisque u vaut 100¤+A après mes mises et v vaut 10000¤²/(100¤+A). Cette fois il n'y a pas de divergence puisqu'on intègre quelque chose en v/u, c'est-à-dire en fait en 1/u² (précisément, l'intégrale est 100¤ (A+100¤) ( u + ( 100¤/u ) u ) du ce qui vaut A + 100¤ · ( A 100¤ + A ) ).

Ce système semble mathématiquement assez naturel (et se généralise assez bien à plus de 2 candidats), et il me rappelle l'apparition de la moyenne géométrique que j'avais vue dans la réalisation des paniers de monnaies. Mais je ne sais pas si elle porte un nom standard, ni si c'est ce qu'utilisent les bookmakers anglais (modulo leurs marges, et modulo le fait qu'ils ne remettent évidemment pas à jour leur cotation instantanément).

↑Entry #2037 [older| permalink|newer] / ↑Entrée #2037 [précédente| permalien|suivante] ↑

↓Entry #2034 [older| permalink|newer] / ↓Entrée #2034 [précédente| permalien|suivante] ↓

(samedi)

Le piège de Hadwiger-Nelson

Un collègue m'a poussé à réfléchir à des questions tournant autour du problème de Hadwiger-Nelson (ou problème du coloriage du plan, ou problème du nombre chromatique des unit distance graphs). Fatale erreur ! Ma maman m'avait pourtant toujours dit quand j'étais petit : Ne réfléchis jamais au problème de Hadwiger-Nelson. Ce problème est comme le Zahir de Borges : on commence à y penser, on se laisse tenter par son apparence si facile et si séduisante, sa faculté à relier des domaines mathématiques aussi divers que la théorie des graphes, la géométrie algébrique et la logique, et bientôt on ne peut plus songer qu'à lui, on le voit dans des rêves et on dessine partout des petits graphes avec des points à distance 1 les uns des autres.

Le problème de Hadwiger-Nelson (que j'avais d'ailleurs déjà évoqué, mais je n'en connaissais pas le nom à l'époque) est sans doute le problème mathématique ouvert le plus incroyablement simple à formuler (même si cette simplicité est peut-être un peu trompeuse). Et qui insulte l'honneur de l'esprit humain de ne pas être capable de répondre à une question aussi bête :

(On fixe une fois pour toutes une unité de distance.) Quel est le nombre minimal χ de couleurs qu'il faut pour colorier le plan de manière que deux points distants d'une unité ne soient jamais de la même couleur ?

Tout ce qu'on sait montrer est que 4≤χ≤7. Et c'est d'autant plus frustrant que la démonstration de ces deux faits est à la portée d'un collégien, et que toutes les maths sophistiquées qu'on a pu essayer de balancer contre ce problème n'ont pas amélioré d'un chouïa l'encadrement en question. La minoration par 4 est démontrée par exemple par la figure suivante, appelée Moser's spindle (le fuseau de Moser ?) :

Chaque arête a distance 1 dans le plan, ce qui détermine rigidement la figure. Il est facile de se convaincre qu'il n'est pas possible de colorier les sommets de ce graphe avec seulement 3 couleurs de manière que deux sommets reliés par une arête ne soient pas de la même couleur (en effet, si on n'a que trois couleurs, quelle que soit la couleur c du sommet en bas à gauche, les deux autres sommets de chacun des triangles équilatéraux dont il est un sommet doivent avoir les deux autres couleurs, du coup le sommet tout en haut et celui tout à droite ont la même couleur c, or ils sont reliés par une arête, une contradiction). La majoration par 7 est démontrée par la figure suivante :

L'unité de longueur (figurée par le trait noir en haut à gauche, et qui est à la même échelle que sur la figure précédente) est juste un tout petit peu trop courte pour relier les sommets les plus proches de deux hexagones de la même couleur, mais juste un tout petit peu trop longue pour tenir à l'intérieur d'un même hexagone : donc quelle que soit la manière dont on place cette règle sur le plan, les deux extrémités auront une couleur différente.

On ne sait rien dire de mieux, donc. Oh, on sait dire des choses sur des problèmes adjacents : par exemple, sur le nombre chromatique fractionnaire du plan pour la relation être-distant-d'une-unité (on sait qu'il est compris entre 3.55 et 4.36), ou sur les coloriages dans lesquels on impose de plus aux parties coloriées d'être mesurables (on sait qu'il faut alors au moins 5 couleurs), ou sur le coloriage des points à coordonnées rationnelles (seulement 2 couleurs suffisent alors) ; en fait, on sait dire assez de choses pour qu'Alexandr Soifer écrive un livre de 600 pages consacré à peu près entièrement à ce problème. Mais l'inégalité 4≤χ≤7 continue de nous narguer et de rappeler les mathématiciens à l'humilité. Personnellement je trouve ça beaucoup plus rageant que le problème de Collatz/Syracuse/3n+1 ou que le théorème de Fermat n'a jamais pu l'être.

Et ce que ce problème a de rageant, aussi, c'est qu'on ne sait pas trop quoi conjecturer. Un théorème d'Erdős et de Bruijn (qui est une conséquence immédiate du théorème de compacité du calcul propositionnel, ou — un petit peu moins immédiate — de la compacité des produits de compacts) assure que pour minorer le nombre de couleurs χ nécessaire pour colorier le plan, il « suffit » [OK, en fait ça veut dire « il faut », comme on me le signale en commentaire] d'exhiber un graphe fini réalisable avec distance 1 et qui n'est coloriable qu'avec ce nombre de couleurs. On pourrait donc se dire que puisque ces « unit distance graphs » ont été tant étudiés, s'il y en avait un qui ait nombre chromatique 5 ou plus, ça aurait fini par se voir : mais il est aussi parfaitement plausible qu'il ait un nombre de sommets faramineux, alors que la borne supérieure de 7 par le pavage hexagonal, elle, a l'air tellement naturelle, tellement appropriée a la situation, qu'on a envie de croire qu'elle est optimale. De fait, jusqu'à récemment, j'étais convaincu en mon for intérieur que la bonne valeur de χ est 7. Mais il est parfaitement possible que ce ne soit la bonne valeur que pour des parties coloriées possédant des propriétés de régularité sympathique (ceci dit, même avec l'hypothèse de mesurabilité on ne sait pas fermer le gap), et qu'un coloriage avec 4 couleurs existe mais nécessite l'axiome du choix (qui est caché derrière le théorème d'Erdős-de-Bruijn). Croire que χ=4 se défend aussi : elle consiste à croire que les configurations de points à distance 1 dans le plan ne peuvent pas trop nous jouer de tour ou de coïncidences inattendues, qu'ils ne sont pas trop éloignés des graphes de Laman (lesquels sont toujours 4-coloriables pour des raisons idiotes de degré). Entre les deux, il est aussi permis de croire que la bonne valeur est 5 ou 6, même si ça semble moins plausible qu'aucun des deux arguments élégants pour 4 ou 7 ne donne la bonne valeur, mais bon, c'est comme en politique, il faut bien qu'il y ait aussi des centristes qui croient qu'in medio stat virtus.

Mise à jour : Quelques années plus tard, j'ai écrit un article sur le problème de Hadwiger-Nelson où on se concentre sur les points à valeurs dans certains corps de nombres comme ℚ(√2) ou ℚ(√3).

Nouvelles mises à jour : Voir aussi cette entrée pour mes mésaventures liées à l'article mentionné ci-dessus, et celle-ci pour un progrès démontrant χ≥5 (et mes méditations à ce sujet).

Bon, à part ça, je continue à vouloir écrire un textes sur les octonions pour mettre dans ce blog, mais je retarde ça tant que je n'ai pas trouvé le temps de lire l'article attentivement fondamental Lie Groups in the Foundations of Geometry de Hans Freudenthal (qui expose généralement la raison pour laquelle les octonions sont naturels). Comme cet article est dense, ça risque de prendre du temps.

Et je maintiens aussi sur le feu mes éphérémides astronomiques (dans lesquels je mets des quaternions, comme beaucoup de gens l'ont deviné, mais pas d'octonions), mais comme dès que je m'y mets pour trop longtemps je m'énerve contre l'astronomie, il ne faut pas non plus y compter pour sitôt.

↑Entry #2034 [older| permalink|newer] / ↑Entrée #2034 [précédente| permalien|suivante] ↑

↓Entry #2030 [older| permalink|newer] / ↓Entrée #2030 [précédente| permalien|suivante] ↓

(dimanche)

J'essaie de comprendre comment la Terre tourne

Je me suis laissé convaincre par un ami d'écrire une application pour Android d'éphémérides astronomiques (il en existe certainement déjà plein, mais guère qui soient libres / open source) : c'est-à-dire un truc qui calcule au moins des choses comme la position et les heures de lever et coucher du Soleil, de la Lune et des planètes, les dates et heures des saisons et des phases de la Lune, et sans doute d'autres choses du même acabit. Normalement, ça ne devrait pas être difficile, il y a plein de code pour ça, presque déjà écrit, en fait, qu'il suffit de convertir en Java.

Mais le truc, c'est que je suis un chouïa maniaque (←ceci est un euphémisme) sur certaines choses, et j'ai une idée assez arrêtée sur la façon dont les choses devraient être faites. Notamment, un calcul astronomique ne se mène pas vraiment de la même façon si on vise une précision d'une fraction de minute d'arc ou de quelques microsecondes d'arc. Et je n'ai pas envie de faire silencieusement des approximations qui empêcheraient de passer à une précision nettement plus grande : i.e., je ne tiens pas spécialement à ce que mon application permette une précision énorme, mais je tiens à ce que le cadre logiciel le permette. Ça devient un peu un défi (stupide) en soi.

Or il se trouve que réaliser des calculs astronomiques permettant, même en principe, une très haute précision, c'est compliqué. (Ne serait-ce que parce qu'on ne va plus pouvoir négliger les effets relativistes, et qu'on commence à avoir mal à la tête juste pour définir ce que c'est que le temps.) J'ai une assez bonne vision des phénomènes les plus simples, mais si je m'impose le carcan de bien prendre conscience de toutes les approximations, je m'y perds assez.

Prenons l'exemple de la rotation de la Terre.

La zéroïème approximation, celle qu'on apprend à l'école primaire, c'est que la Terre tourne autour du Soleil, effectuant une révolution en une année dans un plan appelé écliptique, et qu'elle tourne aussi autour d'elle-même selon un axe de direction fixe appelé l'axe des pôles et dont le plan perpendiculaire s'appelle le plan équatorial ; l'angle entre les plans écliptique et équatorial, ou bien entre l'axe des pôles et l'axe perpendiculaire au plan écliptique, s'appelle l'obliquité ou inclinaison de l'axe terrestre, notée ε, et vaut 23°26′15.66″. L'angle entre l'axe de rotation et la droite Terre-Soleil est responsable des saisons, lesquelles sont limitées par les deux équinoxes lorsque l'axe est en quadrature avec cette droite, ou, si on préfère, que le Soleil se trouve dans le plan équatorial de la Terre, et par deux solstices lorsque le Soleil atteint ses latitudes minimale et maximale par rapport au plan équatorial terrestre, qu'on appelle tropiques du Capricorne (→été austral) et du Cancer (→été boréal). Ça c'est ce que tout le monde devrait savoir, sauf à être un sombre inculte.

↑Entry #2030 [older| permalink|newer] / ↑Entrée #2030 [précédente| permalien|suivante] ↑

↓Entry #2029 [older| permalink|newer] / ↓Entrée #2029 [précédente| permalien|suivante] ↓

(lundi)

Quelques réflexions sur l'inertie

Quand j'écris inertie dans le titre de cette entrée, je ne parle pas du phénomène psychologique mais scientifique : scientifique, c'est-à-dire notamment physique mais pas seulement. Dans ce sens, l'inertie, de façon volontairement très vague, c'est le mécanisme qui fait qu'un phénomène qui se produit a tendance à continuer à se produire (plutôt que, par exemple, cesser immédiatement que sa cause cesse).

En physique, il s'agit de la loi d'inertie, ou première loi de Newton, selon laquelle en l'absence de forces extérieures un objet continue à se déplacer en ligne droite et à vitesse constante : ce n'est pas une évidence, et historiquement il semble qu'on ait pu croire — dans la mesure où la physique aristotélicienne énonçait ces choses clairement, ce dont je ne suis pas sûr du tout — qu'une force était toujours nécessaire pour mouvoir un objet, i.e., le que fait qu'un objet en mouvement finisse par s'arrêter dans les situations concrètes n'était pas l'action des forces de frottement mais le phénomène normal, et que du coup l'inertie était ce qu'il fallait expliquer, ce qu'on a pu faire, semble-t-il, par des mécanismes du genre la poussée de l'air exercée par l'endroit que l'objet venait de quitter (je ne suis pas compétent en histoire des sciences, donc j'affabule peut-être en disant ça, ce sont des souvenirs de manuels de physique lus il y a longtemps, mais Wikipédia suggère des choses compatibles). Le principe général d'inertie, ce que j'ai appelé première loi de Newton, a été formulé clairement par Galilée, même s'il est sans doute exagéré de dire que c'est lui qui l'a dégagé.

Toujours est-il qu'on aurait tort de prendre ça pour une évidence. Il y a une célèbre anecdote racontée par Richard Feynman (qui vaut la peine d'être écoutée rien que pour son délicieux accent new-yorkais) sur la manière dont son père (Melville Feynman) lui a expliqué ce qu'est l'inertie : personne ne sait à quoi c'est dû. Un mathématicien va voir l'inertie comme le fait que la physique est décrite par des équations différentielles du second ordre (la force contrôle non pas la vitesse mais l'accélération, c'est-à-dire la variation de la vitesse), mais ce n'est que reformuler le problème ; ou encore, que si on ramène ces équations au premier ordre, cela se fait en introduisant de nouvelles variables en plus de la position, à savoir la quantité de mouvement des objets : l'état d'un système mécanique classique se traduit par la donnée non seulement des positions des objets mais aussi de leurs quantités de mouvement (ou de façon plus approximative, vitesses[#]). On peut reformuler ces choses de façon plus ou moins sophistiquée, parler d'espace des phases, de principes variationnels, de formulations lagrangienne ou hamiltonienne de la mécanique, on peut généraliser à la mécanique quantique ou à la relativité générale, mais il reste toujours ce même mystère qu'on pousse ou cache sous ces diverses formulations[#2].

Mais il y a d'autres domaines où la notion d'inertie peut être considérée, et c'est alors d'autant plus frappant qu'il ne faut pas la prendre pour une évidence.

Prenons l'économie. Voici une question qui me semble assez profonde : si vous avez une grandeur économique ou financière, peut-être le cours d'une action ou d'une monnaie, dont vous ne savez rien sauf sa valeur à l'instant présent, manifestement la meilleure chose que vous puissiez faire pour prévoir sa valeur demain, c'est de prévoir la même valeur (ce n'est évidemment pas une bonne prévision, mais si vous ne savez rigoureusement rien de plus, c'est certainement le mieux qu'on puisse faire) ; maintenant, je vous donne la valeur d'aujourd'hui et aussi la valeur d'hier : est-ce que la connaissance de cette valeur d'hier peut aider à faire une prévision meilleure ? Si on croit à une forme d'inertie en économie, on va se dire que si la grandeur a augmenté entre hier et aujourd'hui, elle risque d'augmenter de nouveau entre aujourd'hui et demain, et peut-être dans les mêmes proportions, donc on va peut-être prévoir pour demain la valeur symétrique de celle d'hier par rapport à celle d'aujourd'hui (de fait, en physique, si vous voulez prévoir le mouvement d'un objet, c'est exactement ça que vous prévoit la loi d'inertie en l'absence de forces, et donc ce sera une approximation sensée si vous ne savez rien du tout). Mais en fait, s'agissant du cours d'une action, cette idée n'est pas du tout bonne : au contraire, on a tendance à modéliser ces choses-là — en toute première approximation — par des objets mathématiques appelés des martingales, ce qui signifie essentiellement que connaître des choses sur le passé ne vous avancera absolument pas à prévoir l'avenir (par rapport à juste connaître le présent) ; ou, de façon plus concise mais moins précise, il n'y a aucune sorte d'inertie. C'est raisonnable si on pense au cours d'une action comme déterminé par des agents rationnels : ils ont connaissance du passé et ils en tiennent compte, donc si une prévision simpliste basée dessus peut donner une meilleure approximation pour l'avenir qu'une prévision simpliste seulement basée sur le présent, ils en tiendront compte déjà au présent, donc anticipent sur cette prévision !, qui du coup devrait être réalisée déjà maintenant et pas dans l'avenir.

Mais l'absence totale d'inertie signifie que l'idée que le cours d'une action est en train de monter est dénué de sens, ou, en tout cas, de sens prédictif : le fait que ce cours ait augmenté ces N derniers jours ne donnerait aucune information sur le fait qu'il risque d'augmenter encore demain, pas plus que le fait de savoir qu'une pièce non truquée est tombée 20 fois sur pile ne vous donne d'information sur le fait qu'elle tombera sur pile la fois suivante. Or on a quand même tendance à s'imaginer qu'il y a de l'inertie : c'est contraire à cette idée que les marchés anticipent sur toute prévision qu'ils peuvent faire quant à l'avenir. Même si le cours d'une action dépend de phénomènes (physiques, par exemple) qui, eux, peuvent avoir de l'inertie, si ces phénomènes sont connus, ils devraient être anticipés. Je ne sais pas si on peut exhiber des cas où il y a quand même incontestablement une forme inertie dans des cours économiques, mais j'ai toujours été perturbé par cette dissonance entre le fait qu'on soit censé croire à l'absence d'inertie si les agents sont rationnels et le fait qu'on dise, par exemple, que le cours du pétrole va certainement continuer à monter au cours des prochaines années (si cette prévision est si évidente, tout le monde va vouloir prendre des options dessus, ce qui va faire augmenter le cours du pétrole maintenant).

Mais ce qui a motivé cette réflexion à ¤0.02 sur l'inertie, c'est encore un autre domaine, celui de la sociologie : j'entends les commentateurs politiques (dont je ne pense pas forcément grand bien) expliquer que la progression ou régression de tel ou tel homme politique dans les sondages électoraux constitue une dynamique. Le fait de parler de dynamique suppose qu'il y a inertie. Mais est-ce le cas ? Je n'ai cette fois pas d'argument comme pour l'économie qui expliquerait qu'il ne dût pas y en avoir, mais je n'ai pas non plus d'explication vraiment convaincante au fait qu'il y en ait (à part que les électeurs seraient naturellement portés à apprécier en soi les hommes politiques qui enregistrent déjà une progression dans les sondages récents, ce qui est possible mais pas évident). En tout état de cause, je trouve qu'on ne devrait prendre ni l'existence de l'inertie, ni son absence, pour une évidence : c'est une question essentielle qu'on doit se poser sur tout phénomène auquel on est confronté.

[#] Quand on parle d'un seul objet sans interaction extérieure, la masse n'intervient pas du tout, et l'inertie au sens physique peut porter aussi bien sur la vitesse (c'est la manière dont Newton la formule) que sur la quantité de mouvement. Quand il y a plusieurs objets qui interagissent, la masse (inertielle) d'un objet devient, très grossièrement, la proportion avec laquelle l'inertie de cet objet est importante relativement à celle des autres, donc la difficulté des forces à agir sur cet objet.

[#2] La relativité générale est peut-être ce qui arrive le plus proche d'une réponse au mystère, aux yeux du matheux que je suis, parce que l'équation des géodésiques et les équations d'Einstein sont des équations du second ordre mathématiquement très naturelles alors qu'il n'y a rien de la sorte au premier ordre ; mais on peut difficilement prétendre avoir tout résolu en disant ça.

↑Entry #2029 [older| permalink|newer] / ↑Entrée #2029 [précédente| permalien|suivante] ↑

↓Entry #2015 [older| permalink|newer] / ↓Entrée #2015 [précédente| permalien|suivante] ↓

(vendredi)

Quelques formules en radicaux

On a (en MathML, donc à condition que votre navigateur sache l'afficher correctement) :

cos ( 2π 3 ) = 1 2

cos ( 2π 5 ) = 1 4 + 1 4 5

cos ( 2π 7 ) = 1 6 + 1 6 7 2 21 2 −3 3 + 1 6 7 2 + 21 2 −3 3

cos ( 2π 11 ) = 1 10 + ( 1 40 + 1 40 5 + 1 40 10 2 5 ) 979 4 275 4 5 55 10 2 5 + 275 4 10 + 2 5 5 + ( 1 40 + 1 40 5 + 1 40 10 2 5 ) 979 4 + 275 4 5 275 4 10 2 5 55 10 + 2 5 5 + ( 1 40 + 1 40 5 1 40 10 2 5 ) 979 4 + 275 4 5 + 275 4 10 2 5 + 55 10 + 2 5 5 + ( 1 40 + 1 40 5 1 40 10 2 5 ) 979 4 275 4 5 + 55 10 2 5 275 4 10 + 2 5 5

cos ( 2π 13 ) = 1 12 + 1 12 13 + ( 1 24 + 1 24 −3 ) 65 2 39 2 −3 3 + ( 1 24 1 24 −3 ) 65 2 + 39 2 −3 3 + ( 1 24 + 1 24 −3 ) 4381 2 195 2 −3 6 + ( 1 24 1 24 −3 ) 4381 2 + 195 2 −3 6

cos ( 2π 17 ) = 1 16 + 1 16 17 + 1 8 17 2 1 2 17 + 1 4 17 4 + 3 4 17 1 2 17 2 1 2 17 17 2 + 1 2 17

L'existence de ces formules n'a rien de nouveau ou d'extraordinaire (celles de cos(2π/3) et cos(2π/5) sont essentiellement connues depuis l'antiquité, celle de cos(2π/17) a été trouvée par Gauß en 1796, lequel a aussi trouvé la méthode permettant de calculer toutes les formules de ce genre ; j'ai d'ailleurs déjà écrit une formule de ce genre ici, et la formule la plus compliquée, celle de cos(2π/11), dans l'exercice 5 de cette feuille d'exercices que je donnais quand j'enseignais à l'ENS) ; il s'agit de résultats classiques tournant autour de la théorie de Galois, et d'ailleurs c'est parce que j'écrivais quelque chose sur la théorie de Galois que je les ai calculées (et aussi pour m'amuser avec Sage). Ceci dit, la formule de cos(2π/13) ou cos(2π/11), je ne l'ai jamais vue écrite nulle part dans un bouquin.

Mais une question qui me laisse modérément perplexe, c'est la question de formes plus canoniques que d'autres (plus naturelles, plus élégantes, ce que vous voudrez, bref, préférables) pour ces expressions. Je ne parle même pas de factorisations possibles (comme on peut factoriser une racine 5-ième de 11/4 dans l'expression de cos(2π/11)), mais de réécritures un peu plus profondes. Par exemple, l'expression de exp(2·i·π/17) donnée dans l'entrée liée ci-dessus n'est pas la même que celle donnée dans le livre Galois Theory (formule (9.11) page 239) : j'ai tendance à trouver que la mienne (avec deux racines sixièmes plutôt qu'une racine carrée d'expressions faisant intervenir des racines cubiques) est préférable. Mais une autre formule pour cos(2π/17), qui est assurément moins agréable que celle donnée ci-dessus, et qui apparaît pourtant plus naturellement quand on applique un algorithme systématique, est la suivante :

cos ( 2π 17 ) = 1 16 + 1 16 17 + 1 16 −1 255 136 −1 4 1 16 −1 255 + 136 −1 4 + ( 1 32 2 1 32 −2 ) 73185 + 39032 −1 + 3264 2 6120 −2 8 + 1 16 −1 73185 39032 −1 3264 2 6120 −2 8 1 16 −1 73185 + 39032 −1 3264 2 + 6120 −2 8 + ( 1 32 2 + 1 32 −2 ) 73185 39032 −1 + 3264 2 + 6120 −2 8

Une autre question sur laquelle je ne sais pas dire grand-chose, c'est comment produire de façon systématique de telles expressions en MathML (pour celles que je viens de donner, j'ai utilisé un mélange de techniques pas complètement automatisées, jusqu'à terminer quelques réécritures à la main). Rien que mettre les termes dans un ordre raisonnable, ou transformer quelque chose qui apparaît naturellement comme −1·x + (−3/2)·y + (2−tz (une somme pondérée par des coefficients) en −x − (3/2)·y + (2−tz (extraire les signes pour les mettre au niveau de la somme), ce n'est pas évident.

↑Entry #2015 [older| permalink|newer] / ↑Entrée #2015 [précédente| permalien|suivante] ↑

↓Entry #2001 [older| permalink|newer] / ↓Entrée #2001 [précédente| permalien|suivante] ↓

(mardi)

De la difficulté de visualiser trois dimensions ou plus

Je suis notoirement incapable de visualiser la géométrie déjà en trois dimensions. Pourtant, quand j'étais petit, mon papa avait fabriqué pour moi, et suspendu au-dessus de mon lit, un mobile qui faisait tournoyer les cinq solides platoniciens : ça m'a peut-être donné le goût de la géométrie, mais ça ne m'a pas aidé à voir dans l'espace.

Un exemple de quelque chose de très simple que je n'ai jamais réussi à correctement me représenter mentalement, c'est le réseau que les chimistes appellent cubique faces centrées (tiens, pour une fois, Wikipédia en français n'est pas mauvais) et les mathématiciens le réseau A3 : il s'agit simplement d'un arrangement régulier de cubes où on place des points au sommets des cubes et aux milieux de leurs faces — dit comme ça ce n'est pas difficile à visualiser, mais on est censé pouvoir se rendre compte que le réseau en question est engendré par les vecteurs arêtes d'un tétraèdre régulier (voyez notamment cette image), et par ailleurs qu'en le tournant juste de la bonne façon on arrive à une superposition de plans sur chacun desquels les points sont en arrangement hexagonal (ce que les matheux appellent A2). Malgré la quantité tout à fait impressionnante de pages web qui illustrent ces choses de quantités de façons différentes (par exemple ici), et bien que mathématiquement je comprenne parfaitement ce qui se passe, je n'arrive décidément pas à le « voir » : soit je vois les cubes, soit je vois les tétraèdres et les hexagones, mais jamais les deux à la fois. (C'est un peu comme la fameuse illusion qu'on peut voir tourner dans un sens ou dans l'autre mais qu'il est très difficile de faire passer de l'un à l'autre.) Remarquez, si j'en crois le nombre de pages consacrées au réseau cubique faces centrées, justement, je ne dois pas être le seul à avoir du mal.

Dans ces conditions, il n'est pas surprenant que je n'arrive pas à visualiser quatre dimensions ou plus. Et si déjà le réseau A3 est surprenant par sa capacité à avoir une symétrie cubique, tétraédrale et hexagonale à la fois, il n'est pas étonnant que E8 recèle aussi des surprises.

Parfois les gens vous disent qu'ils arrivent à voir en quatre dimensions parce qu'ils ont regardé un tesseract tourner pendant assez longtemps. Demandez-leur alors : existe-t-il un hyperplan qui coupe le tesseract selon un tétraèdre régulier ? (la réponse est évidemment oui, et même un tétraèdre arbitrairement petit, de la même façon qu'on peut couper un cube par un plan proche d'un sommet pour obtenir un triangle équilatéral) ; puis : et pour un octaèdre régulier ? (la réponse est encore oui, en prenant un hyperplan défini par six sommets du tesseract). Je pense que ces questions en embarrasseront plus d'un.

Pour prendre un exemple très simple de quelque chose sur quoi notre intuition de trois dimensions conduit à penser des choses fausses, considérons une simple rotation uniforme. Par rotation uniforme j'entends ici ce que les mathématiciens appellent un sous-groupe à un paramètre du groupe des rotations : un mécanicien aura plutôt tendance à dire qu'on a affaire à une rotation (à vitesse) constante ; si l'on veut, on fait une rotation infinitésimale entre les temps 0 et δt puis on répète indéfiniment cette même rotation. Je ne sais pas bien quelle terminologie adopter pour souligner que c'est le concept le plus simple qu'on puisse concevoir, on fait juste tourner un solide toujours de la même façon et à vitesse toujours égale. Sauf qu'en fait ce n'est pas si simple que ça, parce que notre intuition de la dimension trois nous induit facilement en erreur : en dimension trois, une telle rotation uniforme se fait autour d'un axe de rotation, qui est une droite de points laissés fixes lors du mouvement ; mais en dimension quatre, il n'y a généralement pas d'axe de rotation : si on applique une rotation uniforme à une boule, il n'y a en général qu'un seul point fixe (le centre de la boule), et c'est le cas en toute dimension paire. Mais il y a pire : en dimension trois, si on continue la rotation pendant suffisamment longtemps, le solide finit par revenir à son orientation de départ, i.e., le mouvement est périodique. À partir de la dimension quatre, ce n'est plus le cas : une rotation uniforme très générale n'a pas de période[#]. Autre idée fausse : le fait que le mouvement d'un point donné, sous une rotation uniforme, soit un cercle — ceci est vrai de façon évidente en dimension deux, et aussi en dimension trois où c'est un cercle centré sur, et perpendiculaire à, l'axe de rotation. En dimension quatre ou plus, la trajectoire d'un point sous l'effet d'une rotation uniforme est une sorte de courbe de Lissajous, qui en dimension paire va avoir tendance à être dense dans [correction] un tore de la sphère (c'est-à-dire à passer arbitrairement près d'un point quelconque de celui-ci).

On pourrait illustrer les choses comme ceci : en dimension quatre, les habitants d'une planète sphérique en rotation dans l'espace pourraient généralement connaître l'heure et la date par la simple observation de la position d'une seule étoile. Ou pourraient réaliser des pendules qui sont une simple boule qui tourne uniformément, avec un point marqué. (Bon, tout ceci ne serait pas très pratique, certes, parce que ce serait pénible de faire une lecture précise, mais au moins dans l'idée de nos horloges analogiques avec deux aiguilles qui tournent on pourrait faire des horloges sphériques qui tournent rapidement dans une direction pour indiquer la minute et plus lentement dans une autre pour indiquer l'heure.)

La notion implicite sous-jacente, c'est plus ou moins celle du rang d'un groupe de Lie : le groupe SO3 des rotations en trois dimensions est de rang 1, c'est à peu près ça qui fait qu'on a une seule vitesse de rotation, que les trajectoires des points sont des cercles, etc. Mais en général, SOn est de rang ⌊n/2⌋, c'est par exemple le nombre de vitesses de rotation différentes qu'il faudra donner (sans même chercher à savoir dans quelles directions elles se font), le nombre de paramètres des courbes de Lissajous décrites par les points, etc.

Bref, ce sont différentes réflexions qui me sont venues en réalisant une nouvelle vidéo de rotation du système de racines de E8, rotation cette fois uniforme tout du long. (Pour l'instant la vidéo est sur YouTube, mais la qualité en est tellement abominablement atroce que ça n'a vraiment pas grand intérêt de la regarder ; je publierai une version JavaScript dès que j'aurai fini d'écrire une petite introduction mathématique pour aller avec.) Comme en huit dimensions il y a beaucoup de directions dans lesquelles on puisse tourner (SO8 est de dimension 28 et de rang 4), j'ai fait un choix qui m'a semblé amusant — et je reviendrai dessus pour l'expliquer plus précisément — consistant à prendre une rotation qui appartienne au groupe de Lie exceptionnel G2 (de dimension 14 et de rang 2) formé des rotations qui laissent invariante une structure octonionique sur l'espace de dimension 8, structure octonionique avec laquelle le réseau de E8 a d'intéressants rapports. Du coup, la vidéo fournit une illustration de deux groupes de Lie exceptionnels à la fois, G2 par son action et E8 par son système de racines.

[#] Ce qui ne signifie pas pour autant qu'il soit compliqué : si je prends bêtement deux points tournants à vitesse uniforme sur deux cercles dans le plan, et que leurs vitesses de rotation sont irrationnelles entre elles — ce qui est la situation la plus générale — alors ce mouvement n'est pas périodique non plus, et c'est quand même quelque chose de très simple ; en vérité, la rotation uniforme d'une sphère en dimension quatre n'est pas très éloignée de cette idée (justement parce que le rang du groupe des rotations vaut 2).

↑Entry #2001 [older| permalink|newer] / ↑Entrée #2001 [précédente| permalien|suivante] ↑

↓Entry #1988 [older| permalink|newer] / ↓Entrée #1988 [précédente| permalien|suivante] ↓

(jeudi)

La beauté du système de racines E8

[Une projection de E8]Parmi les objets mathématiques qui me fascinent complètement, un des plus beaux et des plus remarquables est certainement le système de racines de E8 (ou du moins tout le cortège d'objets mathématiques plus ou moins liés à E8 : le système de racines, les polytopes associés et leur groupe de Weyl, le réseau des poids, les groupes de Lie ou de Chevalley associés, les immeubles de Tits qui en découlent et les variétés de drapeaux en question, etc. : il y a plein de choses auxquelles on pense quand on dit E8). Sans rentrer dans les détails mathématiques, disons qu'il s'agit ici d'un solide semi-régulier en dimension 8 (pas exactement régulier — il n'y a que trois solides réguliers à partir de la dimension 5 et ils ne sont pas très amusants — mais uniforme), le plus grand et le plus remarquable d'une famille de cinq objets exceptionnels ; il s'agit aussi des points les plus proches de l'origine dans un certain réseau cristallographique aux propriétés mirobolantes[#].

J'avais déjà fait une page interactive à son sujet (que je devrais d'ailleurs retravailler un peu), mais je reste sur ma faim : cette page ne laisse pas vraiment voir la beauté de l'objet, parce qu'on ne le voit pas bouger.

Bref, je voulais me faire une image du système de racines de E8 en train de tourner et d'exhiber quelques unes de ses impressionnantes symétries.

Le problème est que le polytope dont je parle a 240 sommets et 6720 arêtes[#2], et que tracer 6720 arêtes 25 fois par seconde ça a l'air un chouïa trop rapide pour une bête application en JavaScript sur Canvas (enfin, si quelqu'un trouve moyen d'optimiser ça et peut m'expliquer comment faire, je suis preneur ; le calcul des projections de 240 points, lui, devrait être très rapide). [Mise à jour : voir néanmoins cette page.] À défaut, je me suis fait une vidéo, que j'ai entre autres mise sur YouTube ; bon, l'ennui, c'est que la compression drastique que YouTube fait subir à ses vidéos fait que c'est en fait épouvantablement moche (surtout dans la deuxième partie de la vidéo, celle où ça tourne très vite), ce qui est dommage pour quelque chose censé être d'une beauté ineffable : j'en ai donc aussi fait une version en plus haute qualité à télécharger (170Mo ; le lien qui précède pointe sur un fichier BitTorrent[#3], si ça ne marche pas, vous pouvez réessayer sans l'extension .torrent pour télécharger directement le fichier WebM), ce n'est toujours pas très satisfaisant, il y a encore des artefacts de compression et aussi des artefacts d'aliasing dans le tracé, mais bon, c'est quand même joli à regarder (et certainement mieux que l'horreur qui a atterri sur YouTube).

Les premières 1′30″ montrent différentes petites rotations du polytope pour illustrer certains de ses plans de projection à haute symétrie (à 10″ on voit une symétrie d'ordre 30 appelée figure de Petrie, à 20″ une symétrie d'ordre 20, à 30″ une symétrie d'ordre 24, à 50″ une symétrie d'ordre 18, à 1′10″ une symétrie d'ordre 14). Les 2′30″ restantes sont différentes : cette fois, on revient toutes les 10″ à une projection équivalente, après avoir fait une rotation qui laisse le polytope symétrique (ça tourne donc beaucoup plus vite, et c'est cette partie-là qui a été le plus complètement massacrée par la compression vidéo sur YouTube).

Bizarrement, le plus difficile dans l'histoire a surtout été d'écrire le code qui interpole une rotation discrète en un mouvement continu (ou, de façon mathématiquement plus précise, qui inscrit une transformation orthogonale directe au bout d'un groupe à un paramètre de telles transformations[#4]).

[#] Par exemple, concernant le problème de savoir combien de sphères identiques on peut placer en contact avec une sphère donnée (sans qu'elles se chevauchent, bien sûr), la réponse est connue en toute petite dimension (≤4), en dimension 8 grâce à E8, en dimension 24 grâce au réseau de Leech (un autre réseau aux propriétés mirifiques), et c'est tout. Donc en fait je pourrais définir mon polyèdre E8 de la façon suivante : placez 240 sphères toutes identique autour d'une autre (également identique) en dimension 8, il n'y a essentiellement qu'une seule façon de faire ça, et les centres des 240 sphères forment le polyèdre dont je parle. Mais bon, il est plus simple de dire constructivement que mes 240 points sont ceux qui ont les coordonnées (±1,±1,0,0,0,0,0,0) (pour un choix quelconque de deux coordonnées non nulles et deux signes indépendants, ce qui fait 112 points) ou bien (±½,±½,±½,±½,±½,±½,±½,±½) (pour un nombre pair de signes moins, ce qui fait 128 points).

[#2] Il a aussi 60480 faces, qui sont des triangles équilatéraux, 241920 trois-cellules (c'est-à-dire les faces de dimension 3), qui sont des tétraèdres réguliers, 483840 quatre-cellules, qui sont des 4-simplexes réguliers, 483840 cinq-cellules, qui sont des 5-simplexes réguliers, 207360 six-cellules (dont 138240 relient une facette 7-simplexe à une facette 7-croix et 69120 relient deux 7-croix), et enfin 19440 facettes (=sept-cellules), 17280 étant des 7-simplexes réguliers et 2160 étant des octaèdres généralisés (des 7-croix). Enfin, son groupe de symétries (le groupe de Weyl de E8) est d'ordre 696729600 (et il est isomorphe, à un facteur 2 près, au groupe des transformations préservant une forme quadratique déployée de rang 8 sur 𝔽8).

[#3] Mon organisation BitTorrent, basée sur XBT est d'ailleurs épouvantablement bordélique, mal foutue, et probablement bourrée de trous de sécurité inquiétants. Mais je n'ai jamais réussi à trouver un tracker et client BitTorrent qui me satisfassent (notamment, sans PHP), à utiliser en ligne de commande (sur des machines qui sont essentiellement des serveurs) ; si quelqu'un a des suggestions, je suis preneur. Je devrais peut-être essayer la combinaison opentracker et rTorrent, ce sera peut-être plus agréable que l'horreur que j'ai actuellement.

[#4] En principe c'est très facile : on veut calculer Mt, pour M une transformation orthogonale directe, avec t variant de 0 à 1 : on calcule une matrice P de vecteurs-propres de sorte que D:=P·M·P−1 soit diagonale, et on calcule P−1·Dt·P pour différentes valeurs de t. Le problème est que M peut avoir la valeur-propre −1, auquel cas (−1)t a un problème de détermination (si on ne fait pas attention, on va se retrouver avec une matrice complexe et pas une matrice orthogonale réelle comme on le veut) : il faut donc trouver une base orthogonale de l'espace propre de −1 (et commencer par en trouver une base réelle, parce que les approximations numériques peuvent faire que le calcul initial donne des résultats complexes), puis fabriquer une matrice diagonale par blocs 2×2 de rotation d'angle 2π·t, bref, c'est lourdingue d'avoir quelque chose d'un peu robuste numériquement.

↑Entry #1988 [older| permalink|newer] / ↑Entrée #1988 [précédente| permalien|suivante] ↑

↓Entry #1984 [older| permalink|newer] / ↓Entrée #1984 [précédente| permalien|suivante] ↓

(lundi)

Un joli livre de géométrie

Je mentionnais récemment que je n'écrivais pas beaucoup sur ce blog de critiques de livres. Il est encore plus vrai que je n'écris pas beaucoup de critiques de livres de maths : ce n'est pas que je n'aie pas de livres de maths préférés, bien au contraire, mais la difficulté extrême que je trouve à critiquer un tel livre est que je ne parviens généralement pas à séparer mon appréciation du sujet de celle de la forme (au moins dans le cas où les deux me plaisent). Par exemple, un de mes livres de maths préférés est Algorithms in Invariant Theory de Bernd Sturmfels, dont j'ai déjà parlé, mais en vérité il est difficile de savoir si je l'aime parce que la présentation est excellente ou simplement parce que les théorèmes sont très beaux (auquel cas l'auteur n'y est pas pour grand-chose : c'est juste que je trouve que la théorie des invariants est un petit bijou de mathématiques). Il y a bien sûr des cas où on sait distinguer : par exemple, pour tout livre écrit par Conway, on sait que le sujet va être magnifique mais que l'exposition va être insupportable parce qu'il s'adresse à des génies comme lui et pas à des êtres humains comme vous et moi, et qu'en plus il fait des espèces de jeux de mots insupportables dans sa façon de nommer tous les objets.

Bref, je ne parle normalement pas trop de livres de maths, mais je vais faire une exception pour signaler un livre récent de Jürgen Richter-Gebert, Perspectives on Projective Geometry (A guided tour through real and complex geometry) (Springer 2011, ISBN 978-3-642-17285-4), sur lequel je suis tombé un peu par hasard il y a quelques jours dans la librairie Eyrolles. D'abord parce qu'il ne s'agit pas d'un livre de recherche : il s'agit d'un livre pédagogique qui peut s'adresser à un lectorat extrêmement varié, et même si le mathématicien professionnel n'y apprendra probablement pas grand-chose (en tout cas celui qui se spécialise en géométrie), je pense que beaucoup de gens peuvent l'apprécier, entre un bon lycéen passionné de géométrie et un agrégatif de maths à la recherche de développements originaux.

Pour être clair, et pour m'adresser à mes lecteurs non mathémeticiens qui ont peut-être l'idée que quand je dis géométrie je parle de quelque chose de complètement abscons (du style donnée une variété algébrique projective de dimension n et une section hyperplane dont le complémentaire est lisse, le morphisme de restriction de l'une à l'autre, sur la cohomologie à coefficients entiers, est un isomorphisme jusqu'en dimension n−2 et injectif en dimension n−1), là il s'agit vraiment de géométrie au sens où les gens normaux l'entendent, avec des points, des droites et des triangles. Ceci étant, il s'agit quand même d'un point de vue projectif, algébrique et très élégant : donc de la géométrie plutôt façon Poncelet et Klein que façon Euclide et Apollonios[#]. Donc on a à la fois des choses vraiment élémentaires sur des angles et des distances, et des outils plus sophistiqués venus justement de la théorie des invariants (bracket algebras — comment dit-on ça en français ?).

En vérité, et c'est surtout la raison pour laquelle je le mentionne, il s'agit d'un livre que j'aurais voulu écrire, et qui présente exactement la manière dont je pense la géométrie élémentaire. En tout cas, c'est certainement selon ces lignes que j'aurais fait ma présentation de la géométrie sur ce blog si j'avais eu le courage de la mener à terme. Ce qu'on m'a plusieurs fois reproché de ne pas faire, donc, ceux qui m'ont dit ça, lisez le livre de Richter-Gebert !

Qui plus est, c'est un très joli livre, avec des illustrations bien faites (ce qui n'est jamais mal pour un livre de géométrie, même si le proverbe dit qu'il s'agit de l'art de raisonner juste sur une figure fausse), et imprimé en couleur. Donc même si vous en trouverez certainement un exemplaire électronique diffusé par rayons cosmiques, je conseille vivement d'en prendre une version bouts d'arbres morts, qui n'est pas très chère et qui fera belle figure sur la table basse du salon.

⁂ Un autre livre, sur un sujet vaguement apparenté, que j'ai aussi acquis récemment, et que je ne recommande pas, en revanche, c'est d'Ernest E. Shult, Points and Lines (Characterizing the classical geometries), qui porte sur la géométrie d'incidence. J'espérais y lire des choses qui m'éclairent un peu sur les immeubles et les quotients paraboliques des groupes algébriques réductifs vus comme des géométries, et le genre d'idées sur lesquelles je ne connais que le trop pléthorique et assez indigeste livre de Boris Rosenfeld, Geometry of Lie Groups. L'intention pédagogique de Shult est excellente en ce qu'il a fait un livre self-contained, mais le résultat est malheureusement un fouillis abscons de termes ultra-techniques qui me laisse exactement aussi peu Éclairé qu'au début et beaucoup plus embrouillé, et où il ne parle même pas de groupes de Lie ; et indépendamment du fond, beaucoup de termes sont utilisés avant d'être définis et ne figurent pas dans l'index, ce qui est à peu près rédhibitoire : par exemple, il dit tout un tas de choses sur les espaces métasymplectiques et leur caractérisation, et je n'ai pas réussi à trouver où il en a caché la définition ! C'est d'autant plus dommage que je pense qu'il y aurait eu le moyen de faire quelque chose d'excellent.

[#] Anecdote gratuite : j'ai un ami qui a fait un développement d'agreg sur les coniques sans jamais parler d'ellipse, parabole ou hyperbole. Rached Mneimné, qui était dans son jury, le lui reprochant, lui a dit : Je pense que votre leçon n'aurait pas plu à Archimède. Et il aurait répondu : Mais peut-être qu'elle aurait plu à Poncelet ? (enfin, non, en vérité, malheureusement, il n'a pas eu le culot de dire ça — mais il aurait voulu et eu raison de le faire, et du coup je raconte sans vergogne l'anecdote ainsi arrangée en espérant qu'elle devienne une jolie légende urbaine).

❄ Tiens, et pour ceux qui aiment la géométrie projective, voici une question à 0.02 zorkmids à laquelle je cherche toujours une solution simple et élégante : soient C et D deux coniques planes en position assez générale, p1,p2,p3,p4 leurs quatre points d'intersection, et 1,2,3,4 leurs quatre tangentes communes (c'est-à-dire les intersections des coniques duales C* et D*). Montrer que, quitte à réordonner les points, le birapport de p1,p2,p3,p4 sur C est égal au birapport de 1,2,3,4 sur D*. (Ce dernier étant le birapport sur D des quatre points de tangence de 1,2,3,4. On peut aussi éventuellement remarquer que le premier est aussi le birapport, dans le pinceau linéaire L de coniques engendré par C et D, de C,X,Y,ZX, Y et Z désignent les trois coniques dégénérées passant par p1,p2,p3,p4 ; et de même, le second birapport est aussi celui, dans le pinceau M de coniques simultanément tangentes à 1,2,3,4 de D,U,V,WU, V et W désignent les duales dégénérées qu'on devine. Mais peut-être que cette observation ne fait qu'embrouiller les choses.)

[Ajout () par rapport à la question précédente : cela revient plus ou moins à montrer qu'il existe une conique Γ telle que C et D soient polaires l'une de l'autre par rapport à Γ (car alors la polarité par Γ transforme p1,p2,p3,p4 en 1,2,3,4 à l'ordre près, ce qui implique ce qu'on veut sur le birapport) ; la conique Γ doit nécessairement admettre le triangle autopolaire commun à C et D comme on s'en persuade assez facilement ; on peut montrer son existence en considérant des coordonnés (x:y:z) pour lesquelles ce triangle autopolaire est donné par (1:0:0), (0:1:0) et (0:1:0), ce qui revient à diagonaliser simultanément les formes quadratiques définissant C et D : leurs équations deviennent, disons, cx·x² + cy·y² + cz·z² = 0 et dx·x² + dy·y² + dz·z² = 0, et Γ peut être définie par γx·x² + γy·y² + γz·z² = 0 où chaque γi vaut ±√(ci·di). Mais je voudrais quelque chose de purement géométrique.]

↑Entry #1984 [older| permalink|newer] / ↑Entrée #1984 [précédente| permalien|suivante] ↑

↓Entry #1976 [older| permalink|newer] / ↓Entrée #1976 [précédente| permalien|suivante] ↓

(samedi)

Différentes manières de permuter six objets

Voici un petit gadget qui pourrait servir de décoration de Noël (choisissez une entrée au hasard dans le menu déroulant puis cliquez sur Start, avant de lire les explications ci-dessous) :

Il s'agit d'une représentation sous forme animation de n'importe lequel des seize (classes de) sous-groupes transitifs sur six objets. J'avais évoqué des questions semblables à propos des symétries possibles sur cinq objets dans une entrée récente, mais la discussion pour six objets est évidemment plus compliquée. Un groupe de permutations sur six objets (=sous-groupe de 𝔖6), c'est un ensemble de façon de permuter (réordonner) ces six objets de façon que si on effectue deux permutations du groupe à la suite (=on les compose), on obtient encore une permutation du groupe ; le nombre de permutations s'appelle l'ordre du groupe. Un tel groupe de permutations est dit transitif lorsqu'il y a moyen d'envoyer n'importe quel objet à n'importe quel emplacement par (au moins) un élément du groupe. On dit que deux sous-groupe de 𝔖6 sont conjugués lorsqu'on peut transformer l'un en l'autre en permutant les objets. À conjugaison près, il existe exactement seize groupes de permutation transitifs sur six objets, et c'est ça que cette petite animation représente : on choisit un groupe dans la liste, et le script va choisir aléatoirement une permutation du groupe, puis une autre, puis une autre, et ainsi de suite indéfiniment, et anime à chaque fois le déplacement des six objets. À une extrême, 𝔖6 contient toutes les permutations possibles, à l'autre, C6 ne contient que les permutations cycliques. Entre les deux, chacun des sous-groupes proposés correspond à une petite danse que peuvent faire mes six cercles colorés, je trouve ça assez envoûtant à regarder.

Le cas de 𝔖6 est intéressant, parce que 𝔖6 est l'unique groupe symétrique qui possède des automorphismes extérieurs (c'est-à-dire des façons d'associer à toute permutation une autre de façon à préserver la composition). Pour reprendre la terminologie de Sylvester (qui aimait bien les mots commençant par sy), on appelle pentade synthématique une façon de partitionner en 5 classes les 15 arêtes du graphe complet sur les six objets de façon que deux arêtes ayant un sommet commun ne soient jamais dans la même classe : il existe exactement 6 pentades synthématiques, et 𝔖6 réalise toutes les permutations possibles sur les pentades, ce qui signifie qu'en même temps qu'il agit sur les six objets, il agit aussi sur les six pentades, la correspondance entre les deux définissant un automorphisme extérieur. Beaucoup des sous-groupes transitifs de 𝔖6 se voient assez naturellement à travers cette description. [Ajout : voir cette entrée ultérieure pour une description de l'automorphisme extérieur de 𝔖6.]

Par ailleurs, je dois signaler que j'ai dû faire des choix de représentants dans mes classes de conjugaisons de sous-groupes. (Il n'est malheureusement pas possible de les faire de façon parfaite, c'est-à-dire de façon que deux sous-groupes inclus à conjugaison près soient effectivement représentés par des sous-groupes inclus exactement.) J'ai fait ces choix de façon à respecter la structure de l'hexagone, c'est-à-dire, techniquement, que le groupe diédral de l'hexagone soit dans le normalisateur de tous les représentants ici choisis (ça doit rendre mes choix uniques ou quasiment uniques, et ça les rend en tout cas assez naturels).

↑Entry #1976 [older| permalink|newer] / ↑Entrée #1976 [précédente| permalien|suivante] ↑

↓Entry #1973 [older| permalink|newer] / ↓Entrée #1973 [précédente| permalien|suivante] ↓

(dimanche)

Polynômes plus ou moins symétriques en cinq variables

Suite à des réflexions autour de la théorie de Galois (notamment des équations de degré 5, dans le cadre de l'écriture d'un livre sur le sujet) et autour de la théorie des invariants, je me suis amusé à regarder un peu la façon dont « fonctionnent » les polynômes plus ou moins symétriques à cinq variables. Par plus ou moins symétrique je veux dire qu'il y a des permutations échangeant les cinq variables qui laissent le polynôme invariant, mais pas nécessairement que le groupe G de ces permutations doive être le groupe symétrique 𝔖5 de toutes les (120) permutations possibles. Par exemple, le polynôme Q = Z1·Z2 + Z2·Z3 + Z3·Z4 + Z4·Z5 + Z5·Z1 (dans les cinq variables Z1, Z2, Z3, Z4, Z5) est symétrique sous l'effet de 10 permutations des variables, à savoir les 10 symétries d'un pentagone régulier dont les sommets seraient étiquetés par les cinq variables (dans l'ordre donné), i.e., ce qu'on appelle le groupe diédral D5 du pentagone : concrètement, Q est invariant si on permute cycliquement les variables (Z1 devient Z2, Z2 devient Z3 et ainsi de suite) ou si on les inverse (Z1 devient Z5 et réciproquement, Z2 devient Z4 et réciproquement, et Z3 reste) ou par n'importe quelle composition de ces symétries.

Pourquoi précisément cinq variables ? Parce que c'est le plus petit nombre pour lequel il commence à y avoir des choses intéressantes à dire, parce qu'historiquement cela a eu de l'importance (pour montrer que l'équation algébrique générale du cinquième degré n'est pas résoluble par radicaux et pour savoir précisément détecter si une équation donnée l'est), parce que ça fait une situation sur laquelle faire des calculs explicites, et parce que la situation pour plus de variable commence à devenir franchement compliquée. Bref, c'est un cadre sympathique pour faire quelques observations élémentaires de théorie des invariants ou de Galois, et un peu de vulgarisation comme j'aime en faire.

↑Entry #1973 [older| permalink|newer] / ↑Entrée #1973 [précédente| permalien|suivante] ↑

↓Entry #1964 [older| permalink|newer] / ↓Entrée #1964 [précédente| permalien|suivante] ↓

(dimanche)

Les nombres surréels sont-ils intéressants ?

J'ai évoqué brièvement dans une entrée passée les nombres surréels de Conway. En marge de ma saga sur les ordinaux (commencée ici), je voudrais essayer d'en parler un peu plus ici (comme d'habitude, je promets de faire en sorte de dépendre le moins possible des entrées passées et d'être largement self-contained), et discuter notamment de la question de savoir dans quel mesure ces objets sont naturels, ou intéressants. Je sais qu'ils fascinent beaucoup les mathématiciens amateurs ou moins amateurs, parce qu'ils sont une classe de nombres extrêmement généraux, unifiant à la fois les ordinaux et les nombres réels : on aime bien, en maths, trouver des généralisations communes à plusieurs choses (et c'est vrai que c'est assez rigolo de se dire qu'il y a des « nombres » comme ω√2 ou ε−½). Il y a aussi de jolies analogies entre ces nombres surréels et les nimbres, ces derniers étant une sorte de version en caractéristique 2 de la même chose. Ceci étant, je n'arrive pas vraiment à décider si je trouve les nombres surréels vraiment élégants ou insupportablement bricolés, et je veux présenter des arguments dans les deux sens.

Suites de signes

Je vais prendre la définition suivante : un nombre surréel est une suite de signes plus (+) et moins (−), dont la longueur peut être ou non finie, et est en général un ordinal. (Cet ordinal est appelé par Conway la date de naissance du nombre surréel en question.) Parfois on peut avoir envie d'imposer à cette longueur de ne pas être trop grande, j'y reviendrai.

0 −1 1 −2 −½ ½ 2 −3 −3⁄2 −¾ −¼ ¼ ¾ 3⁄2 3

Par exemple, la suite vide () servira à désigner le nombre 0 (c'est le seul nombre surréel né le jour 0), la suite de longueur 1 formée d'un seul plus (+) le nombre 1 tandis que la suite (−) sera le nombre −1 (ce sont les deux nombres surréels nés le jour 1). Les quatre suites possibles de longueur 2, (++), (+−), (−+) et (−−) seront les nombres 2, ½, −½ et −2 respectivement (les quatre nombres surréels nés le jour 2). Comme je n'ai pas encore expliqué comment on ajoute ou multiplie les nombres surréels, il est normal de ne pas comprendre pourquoi ils correspondraient à ces valeurs précises, mais je donne ces exemples juste pour situer. En fait, n'importe quelle suite finie de + et de − codera un nombre dyadique, c'est-à-dire de la forme p/2r (avec p et r entiers). Ces suites forment un arbre, représenté ci-dessus si votre navigateur gère le SVG (prendre la branche qui descend à droite revient à ajouter un + à la fin de la suite, et celle qui descend à gauche revient à ajouter un −), et dont on devine assez bien la règle de correspondance avec les dyadiques : pour construire le niveau r en ayant construit les niveaux antérieurs, chaque nœud représente le nombre qui est la demi-somme du nombre immédiatement à gauche et du nombre immédiatement à droite dans les niveaux précédents (je veux dire, l'ancêtre le plus récent qui soit situé sur la gauche, resp. sur la droite, du nœud considéré), sauf pour le nombre le plus à droite d'un niveau donné qui s'obtient en ajoutant 1 à celui du niveau précédent (c'est donc l'entier r) et pour le nombre le plus à gauche qui s'obtient en soustrayant 1 à celui du niveau précédent (c'est donc l'entier −r).

↑Entry #1964 [older| permalink|newer] / ↑Entrée #1964 [précédente| permalien|suivante] ↑

↓Entry #1949 [older| permalink|newer] / ↓Entrée #1949 [précédente| permalien|suivante] ↓

(dimanche)

Jeux combinatoires et ordinaux

Je continue ma série sur les ordinaux commencée ici et (tout en promettant de faire mon possible pour que chaque entrée soit au moins vaguement lisible si on n'a pas bien lu les précédentes !, sinon je n'aurai vite plus personne qui suive).

Je commence par évoquer un des jeux mathématiques les plus classiques (et dont j'ai déjà parlé par le passé), le jeu de nim (que j'ai parfois aussi entendu appeler jeu de Marienbad parce qu'il apparaît dans le film L'année dernière à Marienbad de Resnais). C'est un jeu extrêmement simple, que n'importe qui peut comprendre :

On dispose un certain nombre d'allumettes (bâtonnets, jetons, ce que vous voudrez) en un certain nombre de lignes, classiquement 1+3+5+7=16 allumettes dans la position de départ (cf. la figure ci-contre si votre navigateur supporte le SVG). Deux joueurs s'affrontent et chacun, tour à tour, retire des allumettes, autant qu'il en souhaite mais provenant d'une seule ligne (il peut retirer la totalité de la ligne s'il le souhaite, et il peut ne retirer qu'une seule allumette, ou n'importe quoi entre les deux ; mais il doit retirer au moins une allumette et ne doit affecter qu'une seule ligne). Le gagnant est celui qui retire la dernière allumette (de façon équivalente, celui qui ne peut plus jouer parce qu'il n'y a plus aucune allumette a perdu).

La variante misère, dans laquelle celui qui retire la dernière allumette a perdu, existe aussi, et est même peut-être plus fréquente, mais elle est moins satisfaisante mathématiquement et je ne vois aucune raison de la préférer.

Ce petit jeu peut avoir un certain succès auprès des gens qui ne le connaissent pas (et quand on connaît le truc, c'est rigolo de les faire perdre). Si vous ne connaissez pas, et si votre navigateur le supporte, la figure ci-contre est jouable (je vous laisse deviner le mode d'emploi, qui n'est peut-être pas terrible, mais je fais ce que je peux).

Évidemment, la seule chose qui importe pour définir un état du jeu est le nombre d'allumettes restant dans chaque ligne (on pourrait imposer que les joueurs retirassent les allumettes à partir de le droite, ça ne changerait rien). Mathématiquement, on peut dire que le jeu est basé sur les idées que (1) si on décroît un nombre, on finit forcément pas s'arrêter (parce qu'on tombe sur zéro) et que (2) on va faire perdre le joueur qui ne peut plus jouer (selon la logique que chaque joueur est obligé de jouer, donc celui qui ne le peut pas à perdu).

Décrit comme ça, et vu que j'ai présenté les ordinaux comme des lignes de bâtonnets et que j'ai lourdement insisté sur le fait que quand on décroît un ordinal on obtient un processus qui termine toujours, on devine bien sûr que je vais définir un jeu de nim transfini (ou ordinal), dans lequel les lignes peuvent comporter un nombre éventuellement infini de bâtonnets. Mais en fait, ceci sera mon deuxième thème : je commence par un type de jeu un petit peu différent selon le même genre d'idées.

(Les deux parties qui suivent sont indépendantes, et les deux sous-parties de la première le sont aussi à peu près.)

↑Entry #1949 [older| permalink|newer] / ↑Entrée #1949 [précédente| permalien|suivante] ↑

↓Entry #1946 [older| permalink|newer] / ↓Entrée #1946 [précédente| permalien|suivante] ↓

(dimanche)

Petit navigateur d'ordinaux

À la fois pour m'exercer à la programmation en JavaScript (dont j'ai déjà dû dire que c'est un langage dans lequel je trouve un mélange très étonnant de choix très élégants et puissants et de bizarreries insupportables), et pour exercer mon intuition sur les ordinaux, j'ai créé une petite page dynamique (devrais-je dire un jeu en HTML5 ?) d'exploration des ordinaux sous la forme de bâtonnets comme je les présentais dans une entrée récente.

La page est ici, et je l'inclus aussi dans une frame plus bas dans cette entrée (mais c'est moins commode de cette manière parce qu'on ne peut pas utiliser l'historique du navigateur et notamment le bouton back) ; son utilisation mérite sans doute quelques explications. L'idée est qu'un certain ordinal est présenté sous forme de bâtonnets (au départ, ε0, mais on peut en sélectionner d'autres), et plus précisément comme une somme infinie, toujours aux mêmes proportions géométriques, de différents termes (du genre ε0 = ω + ωω + ωωω + ωωωω + ⋯, les termes omis valant d'ailleurs tout autant que le total), et on peut sélectionner un de ces termes (soit un des quatre premiers termes, soit la totalité du reste de la série, ce qui fait cinq choix) pour zoomer dessus : cela se fait soit en cliquant sur le graphique, soit sur un des noms des ordinaux en-dessous (ce sont des liens) qui représentent les valeurs de ces différents termes. Le faire devrait déclencher une animation où on voit la partie en question s'étendre jusqu'à occuper toute la largeur de l'image. Lorsque ceci se produit, le titre reflète le nouvel ordinal qu'on est en train d'observer (la valeur de l'ordinal représenté dans l'image, le « nombre » de bâtonnets sur lesquels on a zoomé, si l'on veut).

En bas de l'image s'affiche aussi le « nombre » de bâtonnets qu'on a fait disparaître sur la gauche, c'est-à-dire la valeur de la somme des termes précédents de la série (à toutes les étapes), ou encore, si l'on préfère, l'ordinal du bâtonnet le plus à gauche à l'intérieur de l'ordinal initial : en quelque sorte, c'est notre position actuelle dans l'ordinal de départ alors que l'ordinal affiché dans le titre est en quelque sorte notre niveau de zoom ; il est donc possible, pour n'importe quel ordinal α plus petit que l'ordinal de départ (et multiple de ω, parce que mon programme refuse de descendre plus loin), de faire en sorte que cet ordinal removed at left vaille exactement α.

En cliquant sur la partie tout à droite (celle qui représente la somme de tous les autres termes de la série), on ne fait pas diminuer l'ordinal représenté, et le processus ne termine jamais (enfin, jusqu'à ce qu'on ait fait exploser JavaScript, ce qui ne devrait pas être long, j'imagine). Si on clicke sur d'autres parties, l'ordinal décroît toujours (je parle de celui indiqué dans le titre, i.e. la « largeur », pas celui du bas, la « position », qui elle augmente toujours), et donc on tombe forcément en un nombre fini d'étapes sur ω, mais à moins de clicker sur la partie la plus à gauche, ce nombre fini d'étapes risque souvent d'être considérablement plus long que la durée de vie de l'Univers, ou en tout cas que la mémoire de votre navigateur, donc il est facile de tomber sur des ordinaux impossiblement compliqués.

Bref, voilà le joujou, je fais d'autres remarques plus sérieuses en-dessous :

Quelques remarques d'ordre mathématique, maintenant, suivies de quelques remarques d'ordre informatique :

  • Tous les ordinaux dans l'histoire ressemblent un peu à la même chose. En fait, dès qu'ils sont plus que légèrement compliqués, ils doivent être identiques au pixel près. La fractale à laquelle ils ressemblent, qui a tout à voir avec mon choix d'arrangement des bâtons, et rien avec la nature fondamentale des ordinaux, peut être décrite comme ceci : si on appelle u la transformation affine qui fixe le point au milieu à gauche du dessin et qui transforme l'axe des abscisses par un coefficient 10/3 et laisse les ordonnées identiques (c'est la transformation qui sert à zoomer sur la partie la plus à gauche de l'ordinal) et v la similitude qui fixe la pointe droite du dessin et qui a pour rapport 10/7 (les transformations qui servent à zoomer sur les autres parties de l'ordinal sont alors uv, uv², uv³, et v4 pour la partie restante), alors il s'agit du plus petit fermé contenant le bâtonnet tout à droite et stable par u et v. Ça, je répète, c'est à quoi mes dessins d'ordinaux ressemblent, et ça signifie simplement que la complexité est telle qu'on ne peut pas résoudre les petits bâtons individuels à moins de clicker très souvent ; mais cette fractale comporte un nombre infini de « bâtonnets » entre deux abscisses distinctes, alors que mes dessins d'ordinaux ont, en principe, un bâtonnet distinct à droite de n'importe quel bâtonnet donné. Bref, voilà une première limitation sérieuse de mon système, c'est qu'on n'y voit rien, et qu'on se retrouve plus ou moins à zoomer tout le temps de la même façon sur le même dessin, avec juste les noms qui changent, ce n'est ni très intéressant, ni très éclairant. ☹️
  • Encore plus sérieux : ce système suggère, alors que c'est faux, qu'il existerait une façon unique, ou au moins très naturelle, de fabriquer n'importe quel ordinal comme somme d'une série d'ordinaux plus petits ; ou que la subdivision de l'ordinal a quelque chose d'intrinsèque. Or ce n'est pas le cas. Par exemple, l'écriture de ωω comme ω + ω² + ω³ + ⋯ laisse penser, comme elle est représentée, que le premier ω est spécial ou différent des autres ω qui peuvent se trouver à l'intérieur des autres termes : il n'en est rien, il a un rôle totalement semblable à celui que joue par exemple le premier ω du terme ω², c'est-à-dire que la séparation des termes n'a rien de magique ou d'intrinsèque. Je suis donc en train de donner une impression totalement fausse de certains ordinaux comme quoi ils seraient plus ou moins espacés à tel ou tel endroit, ce qui n'a aucun sens — seul l'ordre compte. J'avais fait plus attention à ce problème dans mon entrée précédente sur la question, notamment dans mon dessin de ωω, où j'ai fait au moins en sorte que rien ne distingue fondamentalement le premier ω du suivant, mais je ne sais pas généraliser naturellement ce genre d'astuce un peu ad hoc à des ordinaux considérablement plus grands (et de toute façon je ne suis pas persuadé que c'était vraiment plus clair).
  • Les noms que mon programme donne aux ordinaux ne sont pas forcément idéaux : ils sont systématiques et raisonnables, mais ce ne sont pas forcément ceux qu'on utiliserait le plus naturellement. Par exemple, on a ε0ω = ωε0+1, et ε02 = ωε0⁢2, et ε0ω = ωωε0+1, et ε0ε0 = ωωε0⁢2, et ε0ε0ω = ωωωε0+1 (tout cela est laissé en exercice au lecteur, c'est assez facile en se rappelant que ε0=ωε0 et en partant plutôt du membre de gauche) ; or dans chacune de ces égalités, mon programme utilisera la forme de droite pour nommer les ordinaux en question (parce qu'il utilise autant que possible la forme normale de Cantor) alors que la version de gauche (basée en premier lieu sur une écriture en base ε0) semble plus naturelle. Cela peut, au moins, surprendre.
  • Le programme en JavaScript contient un vrai petit système d'objets permettant de représenter les plus petits ordinaux (les plus petits signifiant ici, les ordinaux jusqu'à l'ordinal εεε, qui est le plus petit ordinal vérifiant l'équation α=εα, exclu). Il « sait » vraiment faire des additions, comparer deux ordinaux, trouver une suite qui converge vers un ordinal limite, fabriquer un nom HTML, etc. Ça m'a donné l'occasion de faire de la programmation orientée objet en JavaScript (mélangée à un petit peu de programmation fonctionnelle, puisque je n'hésite pas à renvoyer des fonctions qui capturent des variables internes, etc.). Et à comprendre un peu plus précisément comment fonctionne le système de prototypage de JavaScript (je savais le principe général que chaque objet avait un prototype, qui sert à déterminer la chaîne d'héritage, mais je ne comprenais pas bien la façon dont il était initialisé avec new ou Object.create).
  • C'était aussi l'occasion de faire joujou avec la manipulation des URL en JavaScript : je voulais faire en sorte que chaque état du programme (l'ordinal qu'on voit, mais aussi celui laissé à gauche) corresponde à une URL stable qu'on peut charger directement, du style http://www.madore.org/~david/math/drawordinals.html#?v=e pour la représentation initiale de ε0 (et de même, tous les liens dans la page activent des fonctions JavaScript mais comportent aussi des URL correctes et qu'on peut utiliser directement) ; j'ai donc suivi cette fameuse convention (souvent décriée, mais enfin je ne vois pas comment faire autrement) consistant à coder les paramètres à passer au JavaScript dans ce qui devrait normalement être l'identificateur de fragment. (J'ai utilisé un ‘?’ pour séparer ces paramètres, ça me semble plus propre que de ne rien mettre pour signaler qu'il est normal qu'il n'y ait pas de balise HTML ayant cet ID, mais en même temps je ne voulais pas utiliser le ‘!’ qu'on voit souvent dans cette fonction parce qu'il a un sens magique pour Google.) Ceci permet aussi que (si vous essayez le truc dans sa propre page et pas tel qu'il est inclus dans la présente entrée) les boutons de navigation dans l'historique (back/forward) marchent normalement, ce qui est toujours plus plaisant ; la magie, là, est de capter l'événement hashchange (ça ne marche pas sur tous les navigateurs, mais bon, c'est déjà ça).
  • Ah, et vous vous en foutez, mais si vous voyez des petites mains qui pointent pour marquer les items de cette liste de remarques, je suis assez fier de ma petite magouille en CSS qui m'a fait un peu suer (mais ça c'est parce que je suis mauvais en CSS, pas parce que c'était dur).

↑Entry #1946 [older| permalink|newer] / ↑Entrée #1946 [précédente| permalien|suivante] ↑

↓Entry #1944 [older| permalink|newer] / ↓Entrée #1944 [précédente| permalien|suivante] ↓

(mardi)

Edward Nelson prétend montrer que les mathématiques sont inconsistantes

On me signale cette esquisse d'une démonstration (dont la version complète formerait un gros bouquin), par Edward Nelson (qui est pourtant un matheux relativement renommé, pas un fou dans sa cave), du fait que les mathématiques usuelles, et en fait déjà l'arithmétique de Peano, serait contradictoire.

L'idée serait une sorte de variante du paradoxe bien connu de l'« examen surprise » :

Un prof annonce à ses élèves qu'ils auront un examen au cours de la semaine qui vient (lundi à vendredi) et qu'ils ne pourront pas savoir avant le jour même quel sera le jour de l'examen ; les élèves raisonnent alors que l'examen ne peut pas être le vendredi puisque sinon le jeudi soir ils sauraient que ce ne peut être que le lendemain, et du coup le vendredi est exclu donc l'examen ne peut avoir lieu que du lundi au jeudi, mais les élèves peuvent alors répéter le même raisonnement pour exclure le jeudi, et ainsi de suite, et du coup l'examen ne peut pas avoir lieu du tout ; pourtant, lorsque le mercredi l'examen a lieu, il est effectivement une surprise.

On peut gloser cent mille ans sur ce paradoxe, je ne vais pas le faire parce que ça m'énerve particulièrement (voyez l'article Wikipédia à ce sujet), mais la résolution n'est pas particulièrement compliquée : si on appelle T0 l'axiome il y aura un examen cette semaine et Ti+1 l'axiome (T0 et) on ne peut pas conclure sur la base de Ti quel jour l'examen aura lieu avant qu'il ait lieu, alors T1 implique que l'examen n'a pas lieu vendredi, T2 implique qu'il n'a pas lieu jeudi non plus, T4 implique que l'examen a forcément lieu le lundi, et T5 est contradictoire. Si l'examen a lieu le mercredi, c'est que T3 était faux, voilà tout : si on interprète l'énoncé du prof comme T défini comme (T0 et) on ne peut pas conclure sur la base de T quel jour l'examen aura lieu avant qu'il ait lieu, c'est contradictoire et faux, ce qui n'empêche que T2 peut être vrai, ce qui présente déjà un certain élément de surprise. Bref, je trouve que ce paradoxe n'est pas spécialement intéressant. Mais je veux surtout faire remarquer que ce paradoxe appelle naturellement à faire appel à différentes théories, de plus en plus complexes, dans lesquelles on sait (ou on peut conclure) des choses.

L'erreur technique de Nelson (parce que comme Randall Munroe je n'ai pas le moindre doute qu'il y en ait une, et je ferais bien de prendre son conseil et d'ouvrir les paris au lieu d'essaier d'expliquer les choses) est facile à trouver : même si je n'ai pas envie d'essayer de comprendre exactement ce qu'il prend comme théories faibles de l'arithmétique, il est clair que le qu'il considère en haut de la page 4 (de l'esquisse signalée au début de cette entrée) dépend de la complexité (de Kolmogorov) de la théorie T. Or page 5 il considère des preuves qui increase in rank and level (de nouveau, je n'ai pas envie de savoir exactement ce qu'il entend par là), donc dans des théories T dont la complexité varie, alors qu'il prétend garder fixe. Perdu.

Du moins c'était ma réaction immédiate en lisant son esquisse, et comme je vois ici que Terence Tao est arrivé à la même conclusion, je suis raisonnablement confiant que c'est bien là le problème (au moins dans la façon dont Nelson explique les choses). Les mathématiques sont sauves (et nous avec) !

Mais même si j'ai envie d'ironiser en disant que c'est un peu inquiétant qu'un membre de la National Academy of Science puisse prétendre des choses aussi sottes, il y a un certain intérêt à essayer de comprendre ce que croit en fait Nelson, parce que ce n'est pas idiot (même si quand il pense que l'arithmétique de Peano est contradictoire, je suis totalement et complètement convaincu qu'il se trompe), et c'est une question qui est revenue à diverses reprises sur ce blog. Il ne prétend pas que les mathématiques réellement pratiquées sont contradictoires (et encore moins que 0=1), seulement que tous les systèmes dans lesquels on les fait habituellement sont contradictoires, parce que le principe de récurrence est faux et contradictoire. (Et il pense pouvoir reformuler beaucoup de résultats mathématiques dans un système plus faible qui lui convient, ce qui est en soi intéressant, par exemple du point de vue des mathématiques à rebours, même si on ne croit pas une seule seconde que ZF soit contradictoire.)

Peut-être que pour comprendre sa thèse je peux inviter mon lecteur à lire un texte de vulgarisation sur l'infini que j'avais écrit il y a quelques années, où je commence par expliquer le principe de récurrence sous la forme : 0 est un nombre fini, si n est un nombre fini alors n+1 est aussi un nombre fini (et les entiers naturels sont exactement ce qui s'obtient de cette manière, cf. ce que je racontais récemment sur les ordinaux) ; de ça, je prétends conclure que 1000, mais aussi 101000 ou 10↑10↑10↑10↑⋯↑1000 (avec 1000 élévations à la puissance), ou encore d'autres choses beaucoup plus grandes, sont des nombres finis. Faux, me rétorquerait Nelson : la seule façon dont je pourrais montrer que 101000 est un nombre fini, c'est par une démonstration qui commencerait par 0 est fini, donc 1 est fini, donc 2 est fini, donc 3 est fini, donc 4 est fini… et qui terminerait 101000 par donc 101000 est fini. Or si on met en doute le fait que 101000 soit fini, cette démonstration ne vaut que si elle est écrite complètement, ce qui est manifestement impossible, et je ne peux pas agiter des mains en disant oui, je pourrais le faire en principe, mais c'est très long alors il n'en est pas question : la question est justement de savoir si on pourrait le faire en principe, et si je ne le fais pas, mon raisonnement est circulaire. (Le problème est sérieux, puisque si on permet des longueurs non-standard, il est connu et certain qu'il existe des démonstrations de contradiction dans les mathématiques, mais ces démonstrations ne sont justement pas de longueurs finie, ce ne sont pas du tout des démonstrations, donc tout repose crucialement sur la question de la finitude.)

Maintenant, dans l'arithmétique de Peano, il n'y a aucun problème : si x et y sont des entiers naturels, alors xy existe (=est fini, a bien un sens, est un entier naturel). Mais c'est justement ça que Nelson met en doute : dans les théories faibles de l'arithmétique qu'il considère (je n'ai pas regardé les détails, mais ce genre de choses est assez habituel, voyez par exemple la partie C de ce livre), la fonction exponentielle n'est pas forcément totale : il n'y a pas de raison que xy existe si x et y sont des entiers naturels. Du coup, il faudrait effectivement une démonstration démesurément longue pour montrer que 101000 est un nombre fini ; et ces théories faibles ont un intérêt certain en algorithmique (à cause d'un rapport profond entre leurs théorèmes et différentes hiérarchies de complexité).

Maintenant, je ne sais pas si Nelson croit vraiment que le nombre 101000 n'existe pas ([ajout : en fait, probablement pas, parce que la fonction de multiplication, elle, est bien totale, et on peut construire 101000 en multipliant 1000 fois par 10, ce qui constitue une démonstration assez courte pour être écrite] ; mais il le croit sans doute pour le nombre 10↑10↑10↑10↑⋯↑1000 avec 1000 élévations à la puissance). Cela ne signifie pas qu'il existerait un plus grand entier naturel : tout le monde est d'accord que si n est un entier naturel, alors n+1 en est un, juste qu'on n'atteindrait jamais des nombres comme ce que je prétends avoir écrit ; c'est une opinion provocatrice, que je ne partage pas du tout parce que je suis religieusement platoniste, mais qu'il est difficile de disqualifier, parce qu'il est vrai qu'il faut pour éviter des démonstrations ridiculement longues (et peut-être, justement, prétendra Nelson, infiniment longues !) des axiomes strictement plus forts que ce qu'il admet, et dont il peut tout à fait croire qu'ils sont contradictoires (même si, en l'occurrence, il s'est trompé).

Et c'est un problème philosophique que je considère comme assez sérieux, de savoir si vraiment ces nombres ridiculement grands existent, et comment, et dans quelle mesure et pourquoi on a besoin qu'ils existent, et si les mathématiques peuvent s'en passer. Si on pense qu'ils existent (ce qui est mon cas), la difficulté est d'éviter le côté religieux du paradis platoniste. À l'inverse, si on pense qu'ils n'existent pas (ce qui est le cas de Nelson et, je crois, dans une certaine mesure, d'au moins un lecteur de ce blog), la difficulté est d'expliquer pourquoi ils ne causent pas de contradiction (s'ils n'en causent pas, c'est une forme d'existence au moins potentielle : pourquoi des choses inexistantes auraient-elles des conséquences tangibles comme la non-contradiction de Peano ou de ZFC ?), ou sinon, de trouver cette contradiction (comme Nelson semble déterminé à faire). Les paris sont ouverts !

↑Entry #1944 [older| permalink|newer] / ↑Entrée #1944 [précédente| permalien|suivante] ↑

↓Entry #1939 [older| permalink|newer] / ↓Entrée #1939 [précédente| permalien|suivante] ↓

(dimanche)

Nombres ordinaux : une (longue) introduction

Encore une fois je vais tenter de communiquer mon enthousiasme pour un objet mathématique en essayant d'en parler de façon compréhensible par ma petite sœur[#] : et encore une fois je vais échouer lamentablement parce que les non-matheux n'y comprendront vite rien ou n'essaieront pas de lire, et les matheux n'y apprendront rien. Encore une fois j'écris un post de blog en promettant que c'est le premier d'une série : et encore une fois je vais échouer parce que cette série s'arrêtera probablement à un élément (c'est toujours mieux que zéro, certes).

Parmi les choses qui me fascinent dans les mathématiques (un jour, je publierai un petit catalogue de mes objets mathématiques préférés), il y a beauté qui émane de la symétrie (dans quoi je range toute structure algébrique assez rigide), mais il y a aussi quelque chose sur quoi j'ai plus de mal à mettre un mot, disons peut-être la grandeur. Les ordinaux sont très représentatifs de cette dernière catégorie, et ils forment d'ailleurs une échelle de grandeur à l'aune de laquelle on peut mesurer beaucoup de « phénomènes » mathématiques (souvent pour un résultat décevamment[#2] médiocre, d'ailleurs). J'ai déjà tenté d'en parler sur ce blog (et encore), mais sans vraiment faire l'effort de vulgariser de quoi il s'agit au juste : voici donc une nouvelle tentative, agrémentée de petits dessins.

Plan de cette entrée :

↑Entry #1939 [older| permalink|newer] / ↑Entrée #1939 [précédente| permalien|suivante] ↑

↓Entry #1928 [older| permalink|newer] / ↓Entrée #1928 [précédente| permalien|suivante] ↓

(mercredi)

Passage en HTML5

On va encore me reprocher de faire des annonces techniques qui n'ont aucune conséquence perceptible (mais bon, quand je parle d'autre chose, l'électro-encéphalogramme des commentaires enregistre un calme plat, alors que voulez-vous ?) : je viens de passer ce blog (et toutes les pages de ce site gérées par le même moteur) en HTML5[#]. C'est encore mal dégrossi, mais j'ai essayé de mettre les balises sémantiques comme article, header, footer, nav là où elles s'imposaient. Sur les navigateurs modernes, cela ne devrait faire aucune différence visible. Il y a peut-être un espoir que Google soit moins désorienté dans l'indexation, mais je n'y crois pas trop[#2].

En revanche, ce qui est bien est que je peux maintenant, enfin, librement inclure du SVG et du MathML dans mon blog. Et pour que mon annonce ne soit pas complètement creuse, donc, voici à gauche une étoile à sept branches en SVG (donc si vous ne voyez pas d'étoile à sept branches, c'est que votre navigateur n'est pas assez récent), et voici une formule en MathML permettant de montrer que l'étoile en question est constructible par origami :

e 2iπ 7 = 16 ( 1 +7 + 72 + 213 2 3 + 72 213 2 3 + 4972 + 2733 2 6 4972 2733 2 6 )

(La démonstration de la formule est laissée en exercice au lecteur. Je ne sais pas ce que j'ai trouvé le plus pénible dans l'histoire, d'ailleurs, entre la calculer et la taper en MathML.) Si vous voyez juste quelques nombres éparpillés mais pas de fraction ou de racine, même conclusion que pour le SVG : votre navigateur est trop vieux ou mauvais (et je note avec déception que, chez moi, aussi bien Chrome qu'Opera sont incapables d'afficher le MathML — ce qui est d'autant plus bizarre, s'agissant de Chrome, que WebKit est censé avoir au moins une implémentation partielle de MathML).

[#] Techniquement, et pour répondre à la première de mes questions techniques de l'autre jour, en polyglotte HTML5/XHTML5. J'ai validé un certain nombre de pages (certes pas toutes) contre le validateur expérimental, ce qui m'a d'ailleurs fait trouver un bug dedans (dû à la façon dont Java saucisonne l'Unicode en UTF-16 ; c'est assez ironique, parce que l'auteur de ce validateur écrit en faisant l'éloge de Java's notion of Unicode frozen as UTF-16 from to dawn of time until eternity).

[#2] En revanche, et cela devrait répondre du même coup à la deuxième de mes questions techniques de l'autre jour, si j'adopte, et je vais envisager de le faire, le microformat hAtom, j'ai bon espoir que cela permette de vraiment définir la limite des entrées et aux agrégateurs de fournir un contenu complet.

↑Entry #1928 [older| permalink|newer] / ↑Entrée #1928 [précédente| permalien|suivante] ↑

↓Entry #1922 [older| permalink|newer] / ↓Entrée #1922 [précédente| permalien|suivante] ↓

(mercredi)

Qu'est-ce qu'un panier pour les économistes ?

J'éprouve toujours une certaine satisfaction un peu puérile quand je redécouvre ou réinvente quelque chose qui était déjà connu ou inventé. Il y a longtemps je m'étais demandé comment on devrait fabriquer une monnaie fictive qui représente une sorte de moyenne de devises existantes pour pouvoir, par exemple, toutes les comparer à cette moyenne plutôt que les unes aux autres (notamment pour atténuer les fluctuations dans le point de comparaison lui-même), et j'avais conclu pour plusieurs raisons que la bonne chose à faire était indiscutablement de prendre une moyenne géométrique et pas arithmétique des monnaies. Les deux arguments les plus évidents sont : (1) que ce qui a un sens important, ce ne sont pas vraiment les cours relatifs des devises, ce sont leurs logarithmes[#] — d'ailleurs, il faudrait toujours afficher les graphiques boursiers sur une échelle logarithmique — et que prendre la moyenne des logarithmes, c'est justement prendre une moyenne géométrique ; et (2) une moyenne arithmétique n'a pas trop de sens parce qu'il faut savoir comment on la pondère : on voit bien que prendre la moyenne arithmétique également pondérée au sens idiot entre 1$ et 1¥, c'est-à-dire 0.50$+0.50¥, ne va refléter que les variations du dollar et pas celles du yen vu que celui-ci compte, au final, pour à peu près 1.3% du total. Ceci étant, je ne voyais pas trop la moyenne géométrique utilisée, et je me suis dit, bon, ça doit être une idée saugrenue de matheux que j'ai eue.

Je savais bien qu'il existait un truc appelé un panier de monnaies (par exemple l'écu[#2] était un panier des monnaies européennes) (il existe d'ailleurs des paniers de beaucoup de choses : un panier de biens pour mesurer l'inflation des prix, un panier d'actions pour constituer des indices, etc.), mais la (mauvaise) explication qu'on lit en général, c'est qu'un panier est obtenu en prenant les différentes valeurs constituantes dans différentes proportions, et ceci semble suggérer une moyenne arithmétique. Pourtant, à regarder d'un peu plus près, on se rend compte que les proportions sont déterminées en proportion du panier et pas comme des fractions absolues des valeurs du panier. Qu'est-ce que cela signifie ?

Imaginons que je cherche à constituer un panier, le zorkmid (symbole ¤), avec pour simplifier deux monnaies dedans, disons le dollar ($) et l'euro (€). A priori je vais poser : 1¤ :≡ u$ + v, où les coefficients u et v sont susceptibles de varier dans le temps : à ce stade-là il s'agit d'une moyenne arithmétique ou plutôt d'une combinaison linéaire, et je n'ai rien dit du tout. Après tout, l'euro lui-même a un cours en dollars (ou vice versa), donc je pourrais écrire 1€ = z$ (pour le coup, il est certain que z varie au cours du temps) et 1¤ = (u+v·z)$ et du coup j'ai plutôt trop de variables. (Noter que j'utilise ‘=’ pour indiquer une égalité des cours, alors que j'ai écrit ‘≡’ pour indiquer quelque chose de peut-être un peu plus fort, reste à savoir ce que ça veut dire au juste.) Cherchons à voir ce qu'on pourrait imposer de plus.

La première condition consiste à se dire, comme je le suggère plus haut, que les coefficients u et v, au lieu d'être fixés de façon absolue, sont contraints par le fait qu'on veut que le dollar et l'euro comptent pour des proportions respectivement p et q (fixes, cette fois, avec p+q=1) de la valeur totale du zorkmid. Autrement dit, la valeur de u$ et v€ est dans des proportions p et q du total : en introduisant le taux de change 1€=z$, on va donc imposer que v·z = (q/pu (je répète qu'ici, u, v et z sont des fonctions du temps et que p et q, a priori, sont constants). Cette condition dit donc qu'on va ajuster les coefficients u et v de sorte que les u$ et v€ constituant le zorkmid soient toujours dans les proportions p contre q (si l'une des monnaies s'effondre par rapport à l'autre, le coefficient dans lequel elle entre augmentera relativement à l'autre pour compenser).

Mais il me reste une condition à trouver, totalement indépendante de la précédente (et qui consiste, en quelque sorte, à expliquer ce que signifie le ‘≡’ ci-dessus). C'est l'affirmation que, quand la valeur du dollar et de l'euro change, la valeur du zorkmid change de la même façon comme prescrit par les coefficients u et v (c'est-à-dire, comme si on avait effectivement u dollars et v euros) et pas suite à des changements de u et v eux-mêmes. De façon peut-être plus claire : en imaginant qu'on décompose les choses en variations infinitésimales, dans un premier temps la valeur du dollar et de l'euro change, ce qui change la valeur du zorkmid comme u dollars plus v euros, puis dans un second temps on change u et v mais en préservant alors la valeur du zorkmid. De façon plus succincte : cette condition affirme qu'on n'apporte pas d'argent et qu'on n'en retire pas au panier, on se contente de changer les proportions u et v. De façon encore plus parlante : on a en permanence un capital (le zorkmid) formé de u dollars et v euros, la valeur de ce capital évolue au fur et à mesure que ces devises varient, et on joue à convertir l'une en l'autre ou vice versa (par exemple pour vérifier la condition précédente, ou peut-être une autre) mais c'est la seule opération qu'on se permet de faire pour modifier u et v — on n'apporte pas d'argent de l'extérieur. Cette condition se traduit de la façon suivante : (u+v·z)′=v·(z′) où ′ dénote la dérivée par rapport au temps (le membre de gauche représentant la variation de la valeur du zorkmid en dollars au cours du temps, et le terme de droite impose qu'il soit donné uniquement par la variation de l'euro puisqu'on a exprimé les choses en dollars ; ce serait heureusement équivalent de les exprimer en euros). Autrement dit (en développant la dérivée du membre de gauche), la seconde condition est : u′ + v′·z = 0 (on me souffle que ça s'appelle la condition d'autofinancement, justement parce que cela signifie qu'on n'apporte ni ne retire d'argent au panier).

Maintenant, si je mets ensemble les deux conditions que je viens d'exprimer, j'ai un système d'équations différentielles (les variables étant u et v, et z une fonction paramètre) dont la solution est : u = K·p·zq et v = K·q·zpK est une constante arbitraire (on rappelle que q=1−p), c'est-à-dire que le zorkmid vaut 1¤ = (K·zq)$ = (K·zp)€. C'est précisément ce qu'on appelle, à une constante multiplicative près, la moyenne géométrique (pondérée des coefficients p et q) entre le dollar et l'euro ! (La moyenne géométrique entre 1$ et 1€=z$ est (1p·zq)$ = (zq)$.) Ceci se généralise assez simplement à un nombre quelconque de monnaies.

On ne peut pas dire que ce soient des maths de haute sophistication, mais je suis quand même content d'avoir retrouvé la moyenne géométrique, et d'avoir redécouvert tout seul comme un grand ce qu'est un panier (et on me confirme que je ne me suis pas trompé). Mais au-delà de ça j'ai deux remarques à faire.

La première concerne la réalisabilité du panier. Si je dois réaliser une combinaison u$ + v€ avec u et v constants (une combinaison arithmétique/linéaire, quoi), c'est facile, on prned u dollars plus v euros, on n'y touche pas, et la combinaison est ainsi réalisée. Pour le panier décrit ci-dessus, on doit en permanence convertir les monnaies l'une en l'autre au fur et à mesure qu'elles fluctuent pour maintenir vraie la première condition, v·z = (q/pu. Or ceci n'est pas possible de façon infinitésimale. Mais ce qui est magique, c'est qu'en vertu d'une inégalité classique (de convexité ou de moyenne ou de Hölder ou je ne sais quoi, j'ai la flemme de réfléchir à celle qui sert vraiment), si on a 1¤ sous la forme u0$ + v0€ à un certain moment t0, et que les cours relatifs de l'euro et du dollar changent, quelle que soit la durée qu'on attende, on aura au moins 1¤ (c'est-à-dire que u0+v0·zu+v·z si u = K·p·zq et v = K·q·zp et pareil avec des indices 0). Autrement dit, bien qu'au niveau infinitésimal on ait mis une condition d'autofinancement, si j'attends un temps fini, j'extrais de l'argent de mon u$ + v€ pour conserver 1¤ (ou bien : si j'ai 1¤ sous la forme u0$ + v0€ au moment t0 et que je le garde sur ma table sous la forme de u0 dollars et v0 euros, une semaine plus tard quand les cours auront changé, si je fais les conversions qui s'imposent pour revenir dans les proportions p et q, j'aurai plus que 1¤). Je suppose même que c'est une façon certes très simple mais raisonnable de hedger contre les variations de monnaies, et je suppose que c'est un fait bien connu. (En revanche, plus ou fait souvent les rééquilibrages des proportions de l'euro et du dollar dans le panier, moins on extrait d'argent, et à la limite si on le fait en continu, on n'extrait rien puisqu'il y a autofinancement.)

L'autre remarque concerne une analogie dont je ne sais pas très bien quoi faire. Un fait bien connu en thermodynamique est le suivant : si je prends deux corps, de capacités calorifiques constantes, disons égales pour simplifier, qui sont à des températures différentes, et que je les mets bêtement en contact, à l'équilibre la température sera la moyenne arithmétique (naïvement, si je mélange 1kg d'eau à 20°C et 1kg d'eau à 80°C, j'obtiens à peu près 2kg d'eau à 50°C, même si ce n'est pas tout à fait exact parce que la chaleur spécifique de l'eau varie un peu avec la température) ; en revanche, si au lieu de les mettre bêtement en contact je fais tourner un moteur idéal pour extraire tout le travail que je peux de cette différence de température, c'est-à-dire que je fais le mélange sans produire d'entropie, alors la température résultante est la moyenne géométrique des températures thermodynamiques absolues des deux corps (qui est plus basse que la moyenne arithmétique justement par l'inégalité mentionnée ci-dessus, et la différence a été récupérée sous forme de travail par le moteur). Il y a une certaine analogie dans les formules avec ce que je viens de dire ci-dessus, ce qui n'est certes pas surprenant vu que la moyenne géométrique apparaît à cause d'une différentielle logarithmique quelque part (dans le cas de la thermo, c'est la quantité d'entropie δQ/T qui s'écrit c·dT/T lorsque la chaleur spécifique c est constante, et c'est de là que tout sort), mais il reste que la condition d'autofinancement évoquée ci-dessus évoque quand même de façon surprenante la condition de non production d'entropie. Peut-on dire qu'en récupérant de l'argent de la réalisation du zorkmid évoquée au paragraphe précédent on est en train de faire fonctionner un moteur de Carnot entre le dollar et l'euro ? Ou est-ce que je suis en train de virer au crackpotisme, là ?

Je vais m'arrêter sur cette angoissante question, et sur l'étonnement du nombre de mots que j'arrive à pondre sur un sujet aussi trivial, mais au moins j'aurai fait le service que je dois au fan-club de la moyenne géométrique (dont je suis un membre militant).

[#] Une « règle du pouce » est que quand une grandeur ne peut pas être négative, il est probable que ce soit une grandeur logarithmique (au sens où c'est son logarithme, éventuellement rapporté à une origine arbitraire, qui a un sens naturel). Une indication supplémentaire, c'est que ça ait beaucoup plus de sens que cette grandeur soit doublée ou divisée par deux que augmentée ou soustraite d'une quantité constante.

[#2] On apprend des choses amusantes en lisant cet article de Wikipédia, notamment que les états américains de l'Illinois et de New York ont passé des lois déclarant que l'euro est le successeur de l'écu, par prudence pour éviter que certains contrats rencontrent des difficultés légales.

↑Entry #1922 [older| permalink|newer] / ↑Entrée #1922 [précédente| permalien|suivante] ↑

↓Entry #1920 [older| permalink|newer] / ↓Entrée #1920 [précédente| permalien|suivante] ↓

(samedi)

Pourrait-on appliquer la logique linéaire aux produits financiers ?

Il faut que je me dépêche de parler de produits financiers, parce qu'il paraît que lundi le monde va s'écrouler parce qu'un truc est passé de AAAAA à AAAA++ ou quelque chose de ce genre.

Je n'ai jamais eu la patience (ou le temps à perdre) pour dépasser les quelques premiers chapitres du classique livre de Hull, Options, Futures, and Other Derivative Securities, et je n'ai pas énormément d'intérêt[#] en général pour la finance ou pour les maths financières, mais j'avoue qu'il y a dans la structure des produits dérivés quelque chose que je trouve intellectuellement intéressant. Pas tellement le détail des produits en question, mais l'idée sous-jacente qu'un produit dérivé, qui est un contrat visant une certaine transaction (par exemple, s'agissant d'un future, s'engageant à vendre ou à acheter quelque chose dans l'avenir à un taux déterminé dans le contrat), puisse lui-même être acheté ou vendu, donc faire l'objet d'autres transactions, y compris dans des contrats constituant d'autres produits dérivés.

Ces produits dérivés sont étudiés mathématiquement sous l'angle des probabilités (par exemple, des équations différentielles stochastiques s'il s'agit de modéliser comment pourrait évoluer le cours d'une action ou d'une commodité afin de pricer un future ou une option sur sa vente ou son achat). Mais je n'ai pas l'impression que qui que ce soit se soit vraiment penché sur la question plus basique (et peut-être pas du tout intéressante, c'est vrai) de la structure, algébrique si j'ose dire, des contrats qui peuvent être passés.

Pour donner une idée de ce dont je veux parler, mettons que je note €⊸¤ un contrat qui m'oblige et me permet à la fois, dès maintenant, d'acheter 1 zormid au prix de un 1 euro, et □2012-12-21(€⊸¤) un contrat (un future — ou peut-être un forward peu importe) qui m'oblige et me permet d'acheter au 21 décembre 2012, 1 zorkmid au prix de 1 euro. L'option qui consisterait à m'autoriser (sans que ce soit obligatoire ; les financiers diraient que je suis long sur l'option d'achat d'un zorkmid) à échanger 1€ contre 1¤ le 21 décembre 2012 s'appellerait □2012-12-21(1&(€⊸¤)), où le 1 (qui devrait sans doute être un 𝟏, en fait) signifie rien du tout (pour des raisons qui deviendront claires plus tard — ou pas — je choisis une notation multiplicative, dont c'est un 1 qui désigne le rien, et dans le même genre, 5 euros devraient normalement se noter €⊗5) et le & signifie que j'ai le choix entre les deux parties proposées. En revanche, si je suis le cocontractant (c'est-à-dire court) pour une option qui autorise quelqu'un à me vendre 1 zorkmid pour le prix de 1 euro et que je serais obligé d'accepter, je noterais ça □2012-12-21(1⊕(€⊸¤)), le signe signifiant cette fois que ce n'est pas moi qui choisis. L'action consistant à vendre pour 1 dollar cette dernière option (celle par laquelle je m'engage à accepter d'acheter 1 zorkmid pour le prix de 1 euro le 12 décembre 2012) se noterait (□2012-12-21(1⊕(€⊸¤)))⊸$, et ainsi de suite.

Mes notations sont un peu barbares, mais le lecteur qui aurait lu le titre ou qui les aurait reconnues comprendra qu'il s'agit de celles de la logique linéaire de Girard, avec l'interprétation intuitive classique des connecteurs de celle-ci : AB désigne un contrat qui oblige et permet de transformer A en B, tandis que A&B désigne un contrat qui garantit de pouvoir disposer au choix de A ou de B (au choix de celui du point de vue duquel on se place) alors que AB désigne un contrat qui garantit qu'on disposera soit de A soit de B sans garantir lequel des deux. Et AB indique qu'on a à la fois A et B (s'agissant de beaucoup de choses, comme des euros, je suppose[#2] que AB, pour beaucoup de valeurs de A et de B, par exemple des sommes en euros, serait en fait égal à AB). Reste que ma sémantique n'est pas aussi claire que je voudrais le faire croire, et surtout que le rapport avec le temps qui passe, que j'ai arbitrairement noté □ (comme une sorte de modalisateur) n'est pas évident : il faudrait une logique linéaire temporelle…

⚠ Là, le Club Contexte est très fier de lui : il existe un truc qui s'appelle la logique temporelle linéaire, et qui n'a rien à voir avec la logique linéaire ou avec la logique linéaire temporelle que je voudrais.

C'est peut-être une idée très fumeuse, mais la symétrie entre les positions longue et courte sur une option et les opérations & et ⊕ de la logique linéaire me semble assez frappante, et j'ai l'impression que beaucoup de clauses de contrats peuvent effectivement se formuler dans ce langage. En tout cas, bizarrement, je ne trouve aucun indice que quelqu'un ait eu cette idée avant moi (et il y a quelques années j'avais évoqué la question avec un ami qui a fait de la logique linéaire quand il était plus jeune puis, après un petit passage par les topoï, est devenu trader, et il a reconnu qu'il n'avait jamais fait le lien).

Il y a un peu plus d'un an, quelqu'un avait sorti un canular selon lequel l'analyse ordinale (un sujet qui m'intéresse beaucoup — et je suis le principal auteur de cet article de Wikipédia) aurait trouvé des applciations en finance. Ce n'était pas crédible, mais pas complètement absurde non plus : quiconque a lu les très étranges livres de Conway, On Numbers and Games et Winning Ways sait que de la théorie (combinatoire) des jeux aux ordinaux il n'y a pas très loin, et de la théorie (fût-elle combinatoire) des jeux à la finance, il n'y a pas très loin non plus. En tout cas, le canular était rigolo et bien trouvé (mais je n'ai jamais eu d'explication au juste sur qui l'avait lancé et pourquoi). Moi je propose la logique linéaire (quelque chose qui est aussi notoirement considéré dans le contexte de la théorie des jeux) à la place des ordinaux. Enfin, bien sûr, si je devais écrire un projet ANR, je n'hésiterais pas à expliquer que je vais mettre tout ça ensemble, la logique linéaire, les ordinaux, les équations différentielles stochastiques, la théorie des jeux, peut-être même les topoï, pour attaquer les marchés financiers, et gagner ‹ordinal de Bachmann-Howard› euros (mais qui ne vaudront plus rien parce que l'euro se sera écroulé et ce sera la fin du monde).

[#] Tout, bien sûr, est relatif. Par rapport au foot ou aux frasques des célébrités du show-biz, je trouve la finance fascinante (et cette comparaison n'est pas gratuite : dans mon club de muscu on nous distribue des journaux gratuitement, les plus fréquents étant La Tribune, un journal « people » dont je ne me rappelle pas bien le titre, et évidemment L'Équipe — dans ce contexte, clairement, La Tribune est ce qui m'intéresse le plus). Ceci étant, je crois que je préférerais le bulletin de la société slovène de philologie.

[#2] C'est toujours le ⅋ (par) qui est embêtant, quand on essaie de donner une sémantique, formelle ou intuitive, à la logique linéaire. Mais pour quelque chose de dénombrable et de fractionnable comme des euros, des dollars, des zorkmids ou des onces d'or, il est assez clair que AB et AB sont égaux ; une façon de s'en convaincre est de se dire que dans ce contexte, AA′ a manifestement la valeur somme de A et A′ (par exemple, si A et A′ sont un certain nombre de zorkmids, AA′ est la somme de ces nombres), et AB a la valeur de B moins celle de A (puisqu'on donne A pour recevoir B), or il se trouve que (AB)⅋(A′⊸B′) est exactement équivalent, en logique linéaire, à (AA′)⊸(BB′). En raisonnant sur cette base, on peut arguër que ⊥ (l'unité pour ⅋) ou AA pour tout A (dans le domaine considéré de choses dénombrables et fractionnables) représente la même chose que 1 (l'unité pour ⊗), à savoir une valeur nulle ; et du coup que AB et AB valent la même chose.

↑Entry #1920 [older| permalink|newer] / ↑Entrée #1920 [précédente| permalien|suivante] ↑

↓Entry #1909 [older| permalink|newer] / ↓Entrée #1909 [précédente| permalien|suivante] ↓

(samedi)

Ordres de succession, et encore de la généalogie mathématique

Je repense à ça en apprenant la mort du prince Otto von Habsburg (dernier prince hériter de l'empire d'Autriche) : j'ai cru un instant lire que la maison des Habsbourgs s'était éteinte, ce qui n'est bien sûr pas le cas.

Récemment j'avais lu un article qui m'avait bien fait rire sur une certaine Karin Vogel, assistante médicale vivant à Rostock en Allemagne, qui aurait l'honneur fort douteux, selon certains généalogistes amateurs, d'être la dernière (et 4973e), au moment où l'article a été écrit, dans l'ordre de succession à la couronne britannique. La liste est finie parce que l'Act of Settlement de 1701 ne désigne comme héritiers possibles que les descendants de la princesse électrice Sophie de Hanovre. La liste complète était maintenue sur Wikipédia dans le temps (Mme Vogel y figure bien en dernier, mais avec un rang très différent parce qu'elle ne numérote pas les catholiques qui sont expressément exclus par l'Act of Settlement). Comme Wikipédia a des choses plus sérieuses et importantes à maintenir que les héritiers de la couronne britannique, la liste a été remplacée par une version abrégée.

Il y a deux principales conceptions qui s'opposent pour déterminer l'ensemble des héritiers potentiels d'une couronne : la monarchie française utilisait une loi dite salique (inventée de toutes pièces par un bricolage historique, et appliquée plus ou moins rétroactivement) qui excluait de la succession non seulement les femmes mais surtout toute leur descendance. Selon cette règle, les héritiers potentiels du roi Truc (disons, Hugues Capet) sont les fils (légitimes) de celui-ci, et les fils de ceux-ci, et ainsi de suite récursivement. (Je discuterai plus loin de l'ordre de primogéniture.) D'autres sont moins difficiles et utilisent non seulement les fils mais tous les enfants : les héritiers potentiels du roi Machin sont les enfants, les enfants de ses enfants, et ainsi de suite récursivement. Appelons descendance patrilinéaire de Truc le premier ensemble (défini par la loi salique, donc les fils et les fils des fils et ainsi de suite) et descendance (tout court) de Machin le second ensemble. Et oublions commodément l'existence d'enfants illégitimes ou de toute autre complication généalogique (adoption ou n'importe quoi d'autre).

La descendance patrilinéaire se reflète par la tradition du nom de famille (modulo innovations récentes). Elle se reflète également par la transmission du chromosome Y. Elle a une propriété évidente, c'est qu'elle est réversible de façon unique : je n'appartiens à la descendance patrilinéaire que de mon père, de son père à lui, et ainsi de suite : donc en remontant le temps on obtient une suite unique d'individus dont je tiens mon chromosome Y. Faisons maintenant l'hypothèse d'une population constante, c'est-à-dire qu'on est à la limite ténue entre l'extinction et l'explosion exponentielle. Si à un instant donné dans le passé on note tous les individus de la population (appelons-les les têtes de clans), et qu'on appelle clans leur descendance patrilinéaire dans l'avenir, alors les clans sont en compétition pour partitionner la population : plus un clan est grand, plus il a de chances de s'étendre, tandis qu'un clan petit peut facilement s'éteindre. C'est la raison qui fait que les noms de famille se raréfient (c'est particulièrement frappant en Corée, qui utilise apparemment, et depuis très longtemps, la même règle patrilinéaire que dans de nombreux pays occidentaux). À la limite, si on attend assez longtemps, ou si on remonte assez haut la délimitation des têtes de clans, on s'attend à ce que tout le monde descende patrilinéairement de la même tête de clan, et que tous les autres clans se soient éteints. C'est la raison pour laquelle la loi salique a causé tant de difficultés dynastiques aux capétiens (le clan est toujours à la limite de l'extinction).

↑Entry #1909 [older| permalink|newer] / ↑Entrée #1909 [précédente| permalien|suivante] ↑

↓Entry #1882 [older| permalink|newer] / ↓Entrée #1882 [précédente| permalien|suivante] ↓

(dimanche)

Comment enseigner les rudiments de la géométrie algébrique ?

Je donne (pour la deuxième année) un petit cours de géométrie algébrique à Télécom Paris ; c'est un élément d'un cours appelé Techniques mathématiques avancées pour la cryptographie et le codage, donc il faut comprendre ça comme : éléments de géométrie algébrique pour la cryptographie et le codage. C'est un certain défi, parce que (1) les élèves ne sont pas spécialement orientés matheux, ils veulent surtout voir des applications (malheureusement, la géométrie algébrique demande de digérer une certaine quantité de jargon avant d'arriver aux applications), (2) ils n'ont pas forcément les réflexes et les habitudes (en algèbre) qu'auraient des élèves matheux suivant un cours de géométrie algébrique (par exemple, si je dis qu'avoir un morphisme surjectifs d'anneaux AA′, ça signifie que A′ peut se voir comme un quotient de A par un idéal, c'est le genre de choses qui demande une certaine explication), (3) j'ai peu d'heures (5 séances de 2×1h½, donc 15 heures au total ; l'an dernier j'en avais plus), et je veux arriver à des choses un peu compliquées, comme énoncer le théorème de Riemann-Roch et en faire comprendre le sens (j'ai le droit d'omettre toute démonstration, mais au bout d'un certain point ça fait vraiment magique). À cela s'ajoute, cette année : (4) il n'y a que cinq élèves inscrits au cours, sur lesquels, les bons jours, trois viennent effectivement.

Les notes que j'ai écrites sont ici (mais pas encore vraiment relues, donc probablement plein de fautes).

Il y a un certain nombre de difficultés sur la façon de présenter les choses. Par exemple : faut-il se cantonner fermement aux variétés (sur un corps parfait, disons, ce que j'ai fait) ou bien parler d'infinitésimaux et de schémas ? Avantage des variétés : les morphismes sont plus faciles à définir, parce qu'on peut les voir comme des applications des points sur un corps algébriquement clos, i.e., je peux tout présenter comme des parties de l'espace projectif sur la clôture algébrique, et les choses sont vaguement concrètes. Inconvénient : tester si des polynômes donnés définissent effectivement une variété — c'est-à-dire engendrent un idéal radical — devient un préliminaire indispensable à tout exercice, et ce n'est pas évident. Ou bien, comment présenter la définition des morphismes entre variétés quasiprojectives ? Un point de vue possible serait le point de vue fonctoriel, définir les points des variétés dans une algèbre quelconque (mais c'est justement assez lourd) et invoquer l'esprit du lemme de Yoneda pour définir les morphismes de variétés comme des morphismes de foncteurs (sans forcément prononcer ce mot) : c'est plus élégant, mais pour des gens pour qui l'idée que les morphismes de k-algèbres k[t]→A sont en correspondance avec les éléments de A n'est pas du tout naturelle, ce n'est vraiment pas intuitif. La version apparemment plus simple (et que j'ai suivie) consiste à définir les morphismes comme des applications des les points sur la clôture algébrique ; mais on perd toute la fonctorialité, et si les choses sont prima facie plus élémentaires, ce n'est pas évident qu'on arrive à des notions vraiment plus maniables.

↑Entry #1882 [older| permalink|newer] / ↑Entrée #1882 [précédente| permalien|suivante] ↑

↓Entry #1881 [older| permalink|newer] / ↓Entrée #1881 [précédente| permalien|suivante] ↓

(vendredi)

Que signifie : P=NP indépendant de ZFC ?

Je vais répondre ici à une question de David Monniaux sur son blog parce que ce n'est pas la première fois que j'entends quelqu'un la poser, ça me fera un lien à fournir pour la prochaine fois :

Qu'est-ce que cela voudrait dire que PNP indépendant des axiomes de la théorie des ensembles ?

(Attention, je vais m'adresser aux gens connaissant un peu de maths. Si vous ne savez pas ce qu'est le problème P=NP, commencez par lire Wikipédia.)

La remarque cruciale à faire, c'est que l'énoncé P=NP est un énoncé Σ2 de l'arithmétique. C'est-à-dire qu'il peut se réécrire sous la forme il existe n [en l'occurrence, codant un algorithme + une borne polynomiale explicite dessus] tel que pour tout k [codant une entrée du problème SAT, disons], <énoncé à quantificateurs bornés dont algorithmiquement testable en temps borné a priori> [en l'occurrence, que le problème codé par k est résolu dans le temps annoncé par l'algorithme n]. (Je discutais de ces choses-là ici par exemple.)

Mais commençons par un cas plus simple, celui d'un énoncé Σ1. Par exemple, la négation de l'hypothèse de Riemann. L'hypothèse de Riemann (HR) peut se mettre sous la forme Π1, en disant que pour tout n, <une certaine inégalité facilement testable sur des fonctions arithmétiques est vérifiée> (du genre σ(n) < exp(γ)·n·log(log(n)) pour tout n≥5041, avec σ la fonction somme des diviseurs et γ la constante d'Euler-Mascheroni ; mais peu importe le détail de l'inégalité n'est vraiment pas passionnant, et d'ailleurs il y en a plein). La négation ¬HR de l'hypothèse de Riemann est, dualement, Σ1 : elle dit qu'il existe n telle que <telle inégalité>.

Si on dit que HR (ou ¬HR, c'est pareil) est indécidable dans ZFC, cela signifie donc deux choses : (1) HR n'est pas réfutable dans ZFC, et (2) HR n'est pas démontrable dans ZFC (i.e., ¬HR n'est pas réfutable). Or la partie (1) implique[#] que HR est vraie. Pourquoi ? Parce que si HR est fausse, il existe un n qui viole l'inégalité, et donc qui la réfute (l'inégalité est quelque chose de trivialement testable). En fait, réciproquement, comme on croit à la véracité de ZFC (ou du moins de ses conséquences arithmétiques), si HR est vraie, elle ne peut pas être réfutable dans ZFC.

Autrement dit, si on prouve l'indécidabilité (1&2) de HR dans ZFC, on prouve au moins sa vérité (l'item (1)), et on prouve des choses en plus (à savoir l'item (2)). Du coup, évidemment, ce n'est pas dans ZFC qu'on a des chances de prouver (1&2) (puisque le (2) contredit précisément le fait qu'on puisse prouver (1), i.e. HR, dans ZFC). C'est donc dans un système plus fort, typiquement quelque chose comme ZFC+<un axiome de grand cardinal> qu'on peut arriver à prouver (1&2). Prouver (1&2) dans le système ZFC+<grand cardinal> revient exactement à (1) prouver HR dans ZFC+<grand cardinal> et (2) prouver qu'on ne pouvait pas se passer d'au moins une forme d'axiome supplémentaire (à tout le moins la consistance de ZFC !).

Généralement, quand j'explique ce genre de choses, les gens prennent un air gêné et demandent ce que vrai veut dire. Je pense que quelqu'un qui commence à demander qu'est-ce que vrai veut dire a un sérieux problème et ferait mieux de faire de la peinture que des mathématiques. La seule chose que je peux faire pour dissiper une source de confusion possible, c'est de souligner que quand on dit un truc comme ¬HR n'est pas démontrable dans ZFC, on est en train de faire un énoncé arithmétique (par ailleurs Π1 : pour tout n, le nombre n ne code pas une démonstration de ¬HR dans ZFC) exactement comme quand on dit HR pour commencer (lui aussi Π1). Donc si on sait ce que vrai veut dire dans un cas, ou qu'on est prêt à l'admettre, on sait ce que ça veut dire dans l'autre cas aussi !

Ajout : Voir cette longue entrée ultérieure pour toutes sortes de développements sur la notion de vrai.

Maintenant, prenons P=NP et l'affirmation qu'il n'est pas décidable dans ZFC. Cela signifie de nouveau (1) qu'il n'est pas réfutable (PNP n'est pas démontrable), et (2) que P=NP n'est pas démontrable. Mais cette fois, les choses ont plus subtiles. On peut distinguer deux cas :

  • (a) Soit PNP (i.e., P=NP est faux) : il n'existe pas d'algorithme qui résolve SAT en temps polynomial, mais ce fait-là n'est pas prouvable dans ZFC (sur chaque algorithme putatif donné, il serait toujours possible de trouver une instance du problème qui le met en défaut, mais c'est tout). Rien de plus à dire dans ce cas.
  • (b) Soit P=NP est vrai : il existe donc bien un algorithme et qui résout SAT en temps polynomial. On peut expliciter cet algorithme et cette borne (je ne dis pas que la démonstration sera forcément constructive, mais un algorithme est un truc fini, il est en principe possible de l'écrire). Les choses se comprennent alors comme : l'algorithme existe, mais même si un messager des dieux nous le donne (avec sa borne) sur un plateau d'argent, il n'est pas possible de prouver dans ZFC que cet algorithme résoudra toujours le problème dans le temps imparti (on ne peut que constater que sur chaque instance donnée il tourne effectivement en temps imparti et résout le problème). (Il est bien sûr concevable que dans ZFC+<tel grand cardinal> on puisse prouver ce qu'on veut sur l'algorithme.)

On peut donc imaginer que dans ZFC+<un grand cardinal> quelqu'un arrive à prouver l'indécidabilité (dans ZFC) de P=NP en montrant qu'on est dans le cas (a) (il prouve PNP et il prouve que ZFC ne suffisait pas), ou en montrant qu'on est dans le cas (b) (il prouve P=NP, peut-être en donnant explicitement un algorithme et une borne dessus et en prouvant leur correction dans le système ZFC+<grand cardinal>, puis il prouve que ZFC ne suffisait pas ; mais il pourrait aussi y arriver de façon moins constructive), ou même sans distinguer entre les deux cas (cela paraît hautement invraisemblable, mais ce n'est pas inimaginable).

[#] Pour être parfaitement précis, je devrais dire que je peux prouver dans des systèmes arithmétiques faibles que [l'énoncé arithmétique] HR n'est pas réfutable dans ZFC implique [l'énoncé arithmétique] HR. (Par systèmes arithmétiques faibles, je veux dire que l'arithmétique de Peano est bien plus qu'il n'en faut : l'induction sur les formules Σ1 suffit au moins (cf. Hájek & Pudlák, Metamathematics of First-Order Arithmetic, chapitre I, théorème 4.33), et je suppose que dans ce contexte, il faut beaucoup moins que ça. De même, on peut remplacer ZFC dans ce qui précède par des choses très très très faibles, comme l'arithmétique de Robinson, mais pour le coup c'est assez évident.) En revanche, l'implication dans l'autre sens, HR implique que HR n'est pas réfutable dans ZFC, lui, est un principe de réflexion (fût-il Σ1 et arithmétique) de ZFC, et c'est quelque chose de fort (mais cela découle trivialement de théories comme ZFC+<cardinal inaccessible>). Bref, ce genre d'énoncés qui parlent d'énoncés qui parlent d'énoncés sont souvent assez embrouillés parce qu'il faut bien préciser quelles sont les théories en jeu à chaque niveau (pis que ça : pour une théorie imbriquée, ce qui compte n'est pas seulement quelle théorie est imbriquée mais aussi comment elle est formalisée pour le niveau d'au-dessus).

↑Entry #1881 [older| permalink|newer] / ↑Entrée #1881 [précédente| permalien|suivante] ↑

↓Entry #1871 [older| permalink|newer] / ↓Entrée #1871 [précédente| permalien|suivante] ↓

(lundi)

Une expression régulière pour les multiples de 7

Écrits en base 10, les multiples de 7 sont exactement les nombres vérifiant l'expression régulière suivante (en syntaxe egrep) :

^([07]|6[29]*3|(5|6[29]*[18])(4|5[29]*[18])*(6|5[29]*3)|(4|6[29]*[07]|(5|6[29]*[18])(4|5[29]*[18])*(3|5[29]*[07]))(6|[18][29]*[07]|([07]|[18][29]*[18])(4|5[29]*[18])*(3|5[29]*[07]))*([29]|[18][29]*3|([07]|[18][29]*[18])(4|5[29]*[18])*(6|5[29]*3))|(3|6[29]*6|(5|6[29]*[18])(4|5[29]*[18])*([29]|5[29]*6)|(4|6[29]*[07]|(5|6[29]*[18])(4|5[29]*[18])*(3|5[29]*[07]))(6|[18][29]*[07]|([07]|[18][29]*[18])(4|5[29]*[18])*(3|5[29]*[07]))*(5|[18][29]*6|([07]|[18][29]*[18])(4|5[29]*[18])*([29]|5[29]*6)))([18]|4[29]*6|(3|4[29]*[18])(4|5[29]*[18])*([29]|5[29]*6)|([29]|4[29]*[07]|(3|4[29]*[18])(4|5[29]*[18])*(3|5[29]*[07]))(6|[18][29]*[07]|([07]|[18][29]*[18])(4|5[29]*[18])*(3|5[29]*[07]))*(5|[18][29]*6|([07]|[18][29]*[18])(4|5[29]*[18])*([29]|5[29]*6)))*(5|4[29]*3|(3|4[29]*[18])(4|5[29]*[18])*(6|5[29]*3)|([29]|4[29]*[07]|(3|4[29]*[18])(4|5[29]*[18])*(3|5[29]*[07]))(6|[18][29]*[07]|([07]|[18][29]*[18])(4|5[29]*[18])*(3|5[29]*[07]))*([29]|[18][29]*3|([07]|[18][29]*[18])(4|5[29]*[18])*(6|5[29]*3)))|([29]|6[29]*5|(5|6[29]*[18])(4|5[29]*[18])*([18]|5[29]*5)|(4|6[29]*[07]|(5|6[29]*[18])(4|5[29]*[18])*(3|5[29]*[07]))(6|[18][29]*[07]|([07]|[18][29]*[18])(4|5[29]*[18])*(3|5[29]*[07]))*(4|[18][29]*5|([07]|[18][29]*[18])(4|5[29]*[18])*([18]|5[29]*5))|(3|6[29]*6|(5|6[29]*[18])(4|5[29]*[18])*([29]|5[29]*6)|(4|6[29]*[07]|(5|6[29]*[18])(4|5[29]*[18])*(3|5[29]*[07]))(6|[18][29]*[07]|([07]|[18][29]*[18])(4|5[29]*[18])*(3|5[29]*[07]))*(5|[18][29]*6|([07]|[18][29]*[18])(4|5[29]*[18])*([29]|5[29]*6)))([18]|4[29]*6|(3|4[29]*[18])(4|5[29]*[18])*([29]|5[29]*6)|([29]|4[29]*[07]|(3|4[29]*[18])(4|5[29]*[18])*(3|5[29]*[07]))(6|[18][29]*[07]|([07]|[18][29]*[18])(4|5[29]*[18])*(3|5[29]*[07]))*(5|[18][29]*6|([07]|[18][29]*[18])(4|5[29]*[18])*([29]|5[29]*6)))*([07]|4[29]*5|(3|4[29]*[18])(4|5[29]*[18])*([18]|5[29]*5)|([29]|4[29]*[07]|(3|4[29]*[18])(4|5[29]*[18])*(3|5[29]*[07]))(6|[18][29]*[07]|([07]|[18][29]*[18])(4|5[29]*[18])*(3|5[29]*[07]))*(4|[18][29]*5|([07]|[18][29]*[18])(4|5[29]*[18])*([18]|5[29]*5))))(3|[07][29]*5|(6|[07][29]*[18])(4|5[29]*[18])*([18]|5[29]*5)|(5|[07][29]*[07]|(6|[07][29]*[18])(4|5[29]*[18])*(3|5[29]*[07]))(6|[18][29]*[07]|([07]|[18][29]*[18])(4|5[29]*[18])*(3|5[29]*[07]))*(4|[18][29]*5|([07]|[18][29]*[18])(4|5[29]*[18])*([18]|5[29]*5))|(4|[07][29]*6|(6|[07][29]*[18])(4|5[29]*[18])*([29]|5[29]*6)|(5|[07][29]*[07]|(6|[07][29]*[18])(4|5[29]*[18])*(3|5[29]*[07]))(6|[18][29]*[07]|([07]|[18][29]*[18])(4|5[29]*[18])*(3|5[29]*[07]))*(5|[18][29]*6|([07]|[18][29]*[18])(4|5[29]*[18])*([29]|5[29]*6)))([18]|4[29]*6|(3|4[29]*[18])(4|5[29]*[18])*([29]|5[29]*6)|([29]|4[29]*[07]|(3|4[29]*[18])(4|5[29]*[18])*(3|5[29]*[07]))(6|[18][29]*[07]|([07]|[18][29]*[18])(4|5[29]*[18])*(3|5[29]*[07]))*(5|[18][29]*6|([07]|[18][29]*[18])(4|5[29]*[18])*([29]|5[29]*6)))*([07]|4[29]*5|(3|4[29]*[18])(4|5[29]*[18])*([18]|5[29]*5)|([29]|4[29]*[07]|(3|4[29]*[18])(4|5[29]*[18])*(3|5[29]*[07]))(6|[18][29]*[07]|([07]|[18][29]*[18])(4|5[29]*[18])*(3|5[29]*[07]))*(4|[18][29]*5|([07]|[18][29]*[18])(4|5[29]*[18])*([18]|5[29]*5))))*([18]|[07][29]*3|(6|[07][29]*[18])(4|5[29]*[18])*(6|5[29]*3)|(5|[07][29]*[07]|(6|[07][29]*[18])(4|5[29]*[18])*(3|5[29]*[07]))(6|[18][29]*[07]|([07]|[18][29]*[18])(4|5[29]*[18])*(3|5[29]*[07]))*([29]|[18][29]*3|([07]|[18][29]*[18])(4|5[29]*[18])*(6|5[29]*3))|(4|[07][29]*6|(6|[07][29]*[18])(4|5[29]*[18])*([29]|5[29]*6)|(5|[07][29]*[07]|(6|[07][29]*[18])(4|5[29]*[18])*(3|5[29]*[07]))(6|[18][29]*[07]|([07]|[18][29]*[18])(4|5[29]*[18])*(3|5[29]*[07]))*(5|[18][29]*6|([07]|[18][29]*[18])(4|5[29]*[18])*([29]|5[29]*6)))([18]|4[29]*6|(3|4[29]*[18])(4|5[29]*[18])*([29]|5[29]*6)|([29]|4[29]*[07]|(3|4[29]*[18])(4|5[29]*[18])*(3|5[29]*[07]))(6|[18][29]*[07]|([07]|[18][29]*[18])(4|5[29]*[18])*(3|5[29]*[07]))*(5|[18][29]*6|([07]|[18][29]*[18])(4|5[29]*[18])*([29]|5[29]*6)))*(5|4[29]*3|(3|4[29]*[18])(4|5[29]*[18])*(6|5[29]*3)|([29]|4[29]*[07]|(3|4[29]*[18])(4|5[29]*[18])*(3|5[29]*[07]))(6|[18][29]*[07]|([07]|[18][29]*[18])(4|5[29]*[18])*(3|5[29]*[07]))*([29]|[18][29]*3|([07]|[18][29]*[18])(4|5[29]*[18])*(6|5[29]*3))))|([18]|6[29]*4|(5|6[29]*[18])(4|5[29]*[18])*([07]|5[29]*4)|(4|6[29]*[07]|(5|6[29]*[18])(4|5[29]*[18])*(3|5[29]*[07]))(6|[18][29]*[07]|([07]|[18][29]*[18])(4|5[29]*[18])*(3|5[29]*[07]))*(3|[18][29]*4|([07]|[18][29]*[18])(4|5[29]*[18])*([07]|5[29]*4))|(3|6[29]*6|(5|6[29]*[18])(4|5[29]*[18])*([29]|5[29]*6)|(4|6[29]*[07]|(5|6[29]*[18])(4|5[29]*[18])*(3|5[29]*[07]))(6|[18][29]*[07]|([07]|[18][29]*[18])(4|5[29]*[18])*(3|5[29]*[07]))*(5|[18][29]*6|([07]|[18][29]*[18])(4|5[29]*[18])*([29]|5[29]*6)))([18]|4[29]*6|(3|4[29]*[18])(4|5[29]*[18])*([29]|5[29]*6)|([29]|4[29]*[07]|(3|4[29]*[18])(4|5[29]*[18])*(3|5[29]*[07]))(6|[18][29]*[07]|([07]|[18][29]*[18])(4|5[29]*[18])*(3|5[29]*[07]))*(5|[18][29]*6|([07]|[18][29]*[18])(4|5[29]*[18])*([29]|5[29]*6)))*(6|4[29]*4|(3|4[29]*[18])(4|5[29]*[18])*([07]|5[29]*4)|([29]|4[29]*[07]|(3|4[29]*[18])(4|5[29]*[18])*(3|5[29]*[07]))(6|[18][29]*[07]|([07]|[18][29]*[18])(4|5[29]*[18])*(3|5[29]*[07]))*(3|[18][29]*4|([07]|[18][29]*[18])(4|5[29]*[18])*([07]|5[29]*4)))|([29]|6[29]*5|(5|6[29]*[18])(4|5[29]*[18])*([18]|5[29]*5)|(4|6[29]*[07]|(5|6[29]*[18])(4|5[29]*[18])*(3|5[29]*[07]))(6|[18][29]*[07]|([07]|[18][29]*[18])(4|5[29]*[18])*(3|5[29]*[07]))*(4|[18][29]*5|([07]|[18][29]*[18])(4|5[29]*[18])*([18]|5[29]*5))|(3|6[29]*6|(5|6[29]*[18])(4|5[29]*[18])*([29]|5[29]*6)|(4|6[29]*[07]|(5|6[29]*[18])(4|5[29]*[18])*(3|5[29]*[07]))(6|[18][29]*[07]|([07]|[18][29]*[18])(4|5[29]*[18])*(3|5[29]*[07]))*(5|[18][29]*6|([07]|[18][29]*[18])(4|5[29]*[18])*([29]|5[29]*6)))([18]|4[29]*6|(3|4[29]*[18])(4|5[29]*[18])*([29]|5[29]*6)|([29]|4[29]*[07]|(3|4[29]*[18])(4|5[29]*[18])*(3|5[29]*[07]))(6|[18][29]*[07]|([07]|[18][29]*[18])(4|5[29]*[18])*(3|5[29]*[07]))*(5|[18][29]*6|([07]|[18][29]*[18])(4|5[29]*[18])*([29]|5[29]*6)))*([07]|4[29]*5|(3|4[29]*[18])(4|5[29]*[18])*([18]|5[29]*5)|([29]|4[29]*[07]|(3|4[29]*[18])(4|5[29]*[18])*(3|5[29]*[07]))(6|[18][29]*[07]|([07]|[18][29]*[18])(4|5[29]*[18])*(3|5[29]*[07]))*(4|[18][29]*5|([07]|[18][29]*[18])(4|5[29]*[18])*([18]|5[29]*5))))(3|[07][29]*5|(6|[07][29]*[18])(4|5[29]*[18])*([18]|5[29]*5)|(5|[07][29]*[07]|(6|[07][29]*[18])(4|5[29]*[18])*(3|5[29]*[07]))(6|[18][29]*[07]|([07]|[18][29]*[18])(4|5[29]*[18])*(3|5[29]*[07]))*(4|[18][29]*5|([07]|[18][29]*[18])(4|5[29]*[18])*([18]|5[29]*5))|(4|[07][29]*6|(6|[07][29]*[18])(4|5[29]*[18])*([29]|5[29]*6)|(5|[07][29]*[07]|(6|[07][29]*[18])(4|5[29]*[18])*(3|5[29]*[07]))(6|[18][29]*[07]|([07]|[18][29]*[18])(4|5[29]*[18])*(3|5[29]*[07]))*(5|[18][29]*6|([07]|[18][29]*[18])(4|5[29]*[18])*([29]|5[29]*6)))([18]|4[29]*6|(3|4[29]*[18])(4|5[29]*[18])*([29]|5[29]*6)|([29]|4[29]*[07]|(3|4[29]*[18])(4|5[29]*[18])*(3|5[29]*[07]))(6|[18][29]*[07]|([07]|[18][29]*[18])(4|5[29]*[18])*(3|5[29]*[07]))*(5|[18][29]*6|([07]|[18][29]*[18])(4|5[29]*[18])*([29]|5[29]*6)))*([07]|4[29]*5|(3|4[29]*[18])(4|5[29]*[18])*([18]|5[29]*5)|([29]|4[29]*[07]|(3|4[29]*[18])(4|5[29]*[18])*(3|5[29]*[07]))(6|[18][29]*[07]|([07]|[18][29]*[18])(4|5[29]*[18])*(3|5[29]*[07]))*(4|[18][29]*5|([07]|[18][29]*[18])(4|5[29]*[18])*([18]|5[29]*5))))*([29]|[07][29]*4|(6|[07][29]*[18])(4|5[29]*[18])*([07]|5[29]*4)|(5|[07][29]*[07]|(6|[07][29]*[18])(4|5[29]*[18])*(3|5[29]*[07]))(6|[18][29]*[07]|([07]|[18][29]*[18])(4|5[29]*[18])*(3|5[29]*[07]))*(3|[18][29]*4|([07]|[18][29]*[18])(4|5[29]*[18])*([07]|5[29]*4))|(4|[07][29]*6|(6|[07][29]*[18])(4|5[29]*[18])*([29]|5[29]*6)|(5|[07][29]*[07]|(6|[07][29]*[18])(4|5[29]*[18])*(3|5[29]*[07]))(6|[18][29]*[07]|([07]|[18][29]*[18])(4|5[29]*[18])*(3|5[29]*[07]))*(5|[18][29]*6|([07]|[18][29]*[18])(4|5[29]*[18])*([29]|5[29]*6)))([18]|4[29]*6|(3|4[29]*[18])(4|5[29]*[18])*([29]|5[29]*6)|([29]|4[29]*[07]|(3|4[29]*[18])(4|5[29]*[18])*(3|5[29]*[07]))(6|[18][29]*[07]|([07]|[18][29]*[18])(4|5[29]*[18])*(3|5[29]*[07]))*(5|[18][29]*6|([07]|[18][29]*[18])(4|5[29]*[18])*([29]|5[29]*6)))*(6|4[29]*4|(3|4[29]*[18])(4|5[29]*[18])*([07]|5[29]*4)|([29]|4[29]*[07]|(3|4[29]*[18])(4|5[29]*[18])*(3|5[29]*[07]))(6|[18][29]*[07]|([07]|[18][29]*[18])(4|5[29]*[18])*(3|5[29]*[07]))*(3|[18][29]*4|([07]|[18][29]*[18])(4|5[29]*[18])*([07]|5[29]*4)))))(5|3[29]*4|([29]|3[29]*[18])(4|5[29]*[18])*([07]|5[29]*4)|([18]|3[29]*[07]|([29]|3[29]*[18])(4|5[29]*[18])*(3|5[29]*[07]))(6|[18][29]*[07]|([07]|[18][29]*[18])(4|5[29]*[18])*(3|5[29]*[07]))*(3|[18][29]*4|([07]|[18][29]*[18])(4|5[29]*[18])*([07]|5[29]*4))|([07]|3[29]*6|([29]|3[29]*[18])(4|5[29]*[18])*([29]|5[29]*6)|([18]|3[29]*[07]|([29]|3[29]*[18])(4|5[29]*[18])*(3|5[29]*[07]))(6|[18][29]*[07]|([07]|[18][29]*[18])(4|5[29]*[18])*(3|5[29]*[07]))*(5|[18][29]*6|([07]|[18][29]*[18])(4|5[29]*[18])*([29]|5[29]*6)))([18]|4[29]*6|(3|4[29]*[18])(4|5[29]*[18])*([29]|5[29]*6)|([29]|4[29]*[07]|(3|4[29]*[18])(4|5[29]*[18])*(3|5[29]*[07]))(6|[18][29]*[07]|([07]|[18][29]*[18])(4|5[29]*[18])*(3|5[29]*[07]))*(5|[18][29]*6|([07]|[18][29]*[18])(4|5[29]*[18])*([29]|5[29]*6)))*(6|4[29]*4|(3|4[29]*[18])(4|5[29]*[18])*([07]|5[29]*4)|([29]|4[29]*[07]|(3|4[29]*[18])(4|5[29]*[18])*(3|5[29]*[07]))(6|[18][29]*[07]|([07]|[18][29]*[18])(4|5[29]*[18])*(3|5[29]*[07]))*(3|[18][29]*4|([07]|[18][29]*[18])(4|5[29]*[18])*([07]|5[29]*4)))|(6|3[29]*5|([29]|3[29]*[18])(4|5[29]*[18])*([18]|5[29]*5)|([18]|3[29]*[07]|([29]|3[29]*[18])(4|5[29]*[18])*(3|5[29]*[07]))(6|[18][29]*[07]|([07]|[18][29]*[18])(4|5[29]*[18])*(3|5[29]*[07]))*(4|[18][29]*5|([07]|[18][29]*[18])(4|5[29]*[18])*([18]|5[29]*5))|([07]|3[29]*6|([29]|3[29]*[18])(4|5[29]*[18])*([29]|5[29]*6)|([18]|3[29]*[07]|([29]|3[29]*[18])(4|5[29]*[18])*(3|5[29]*[07]))(6|[18][29]*[07]|([07]|[18][29]*[18])(4|5[29]*[18])*(3|5[29]*[07]))*(5|[18][29]*6|([07]|[18][29]*[18])(4|5[29]*[18])*([29]|5[29]*6)))([18]|4[29]*6|(3|4[29]*[18])(4|5[29]*[18])*([29]|5[29]*6)|([29]|4[29]*[07]|(3|4[29]*[18])(4|5[29]*[18])*(3|5[29]*[07]))(6|[18][29]*[07]|([07]|[18][29]*[18])(4|5[29]*[18])*(3|5[29]*[07]))*(5|[18][29]*6|([07]|[18][29]*[18])(4|5[29]*[18])*([29]|5[29]*6)))*([07]|4[29]*5|(3|4[29]*[18])(4|5[29]*[18])*([18]|5[29]*5)|([29]|4[29]*[07]|(3|4[29]*[18])(4|5[29]*[18])*(3|5[29]*[07]))(6|[18][29]*[07]|([07]|[18][29]*[18])(4|5[29]*[18])*(3|5[29]*[07]))*(4|[18][29]*5|([07]|[18][29]*[18])(4|5[29]*[18])*([18]|5[29]*5))))(3|[07][29]*5|(6|[07][29]*[18])(4|5[29]*[18])*([18]|5[29]*5)|(5|[07][29]*[07]|(6|[07][29]*[18])(4|5[29]*[18])*(3|5[29]*[07]))(6|[18][29]*[07]|([07]|[18][29]*[18])(4|5[29]*[18])*(3|5[29]*[07]))*(4|[18][29]*5|([07]|[18][29]*[18])(4|5[29]*[18])*([18]|5[29]*5))|(4|[07][29]*6|(6|[07][29]*[18])(4|5[29]*[18])*([29]|5[29]*6)|(5|[07][29]*[07]|(6|[07][29]*[18])(4|5[29]*[18])*(3|5[29]*[07]))(6|[18][29]*[07]|([07]|[18][29]*[18])(4|5[29]*[18])*(3|5[29]*[07]))*(5|[18][29]*6|([07]|[18][29]*[18])(4|5[29]*[18])*([29]|5[29]*6)))([18]|4[29]*6|(3|4[29]*[18])(4|5[29]*[18])*([29]|5[29]*6)|([29]|4[29]*[07]|(3|4[29]*[18])(4|5[29]*[18])*(3|5[29]*[07]))(6|[18][29]*[07]|([07]|[18][29]*[18])(4|5[29]*[18])*(3|5[29]*[07]))*(5|[18][29]*6|([07]|[18][29]*[18])(4|5[29]*[18])*([29]|5[29]*6)))*([07]|4[29]*5|(3|4[29]*[18])(4|5[29]*[18])*([18]|5[29]*5)|([29]|4[29]*[07]|(3|4[29]*[18])(4|5[29]*[18])*(3|5[29]*[07]))(6|[18][29]*[07]|([07]|[18][29]*[18])(4|5[29]*[18])*(3|5[29]*[07]))*(4|[18][29]*5|([07]|[18][29]*[18])(4|5[29]*[18])*([18]|5[29]*5))))*([29]|[07][29]*4|(6|[07][29]*[18])(4|5[29]*[18])*([07]|5[29]*4)|(5|[07][29]*[07]|(6|[07][29]*[18])(4|5[29]*[18])*(3|5[29]*[07]))(6|[18][29]*[07]|([07]|[18][29]*[18])(4|5[29]*[18])*(3|5[29]*[07]))*(3|[18][29]*4|([07]|[18][29]*[18])(4|5[29]*[18])*([07]|5[29]*4))|(4|[07][29]*6|(6|[07][29]*[18])(4|5[29]*[18])*([29]|5[29]*6)|(5|[07][29]*[07]|(6|[07][29]*[18])(4|5[29]*[18])*(3|5[29]*[07]))(6|[18][29]*[07]|([07]|[18][29]*[18])(4|5[29]*[18])*(3|5[29]*[07]))*(5|[18][29]*6|([07]|[18][29]*[18])(4|5[29]*[18])*([29]|5[29]*6)))([18]|4[29]*6|(3|4[29]*[18])(4|5[29]*[18])*([29]|5[29]*6)|([29]|4[29]*[07]|(3|4[29]*[18])(4|5[29]*[18])*(3|5[29]*[07]))(6|[18][29]*[07]|([07]|[18][29]*[18])(4|5[29]*[18])*(3|5[29]*[07]))*(5|[18][29]*6|([07]|[18][29]*[18])(4|5[29]*[18])*([29]|5[29]*6)))*(6|4[29]*4|(3|4[29]*[18])(4|5[29]*[18])*([07]|5[29]*4)|([29]|4[29]*[07]|(3|4[29]*[18])(4|5[29]*[18])*(3|5[29]*[07]))(6|[18][29]*[07]|([07]|[18][29]*[18])(4|5[29]*[18])*(3|5[29]*[07]))*(3|[18][29]*4|([07]|[18][29]*[18])(4|5[29]*[18])*([07]|5[29]*4)))))*(4|3[29]*3|([29]|3[29]*[18])(4|5[29]*[18])*(6|5[29]*3)|([18]|3[29]*[07]|([29]|3[29]*[18])(4|5[29]*[18])*(3|5[29]*[07]))(6|[18][29]*[07]|([07]|[18][29]*[18])(4|5[29]*[18])*(3|5[29]*[07]))*([29]|[18][29]*3|([07]|[18][29]*[18])(4|5[29]*[18])*(6|5[29]*3))|([07]|3[29]*6|([29]|3[29]*[18])(4|5[29]*[18])*([29]|5[29]*6)|([18]|3[29]*[07]|([29]|3[29]*[18])(4|5[29]*[18])*(3|5[29]*[07]))(6|[18][29]*[07]|([07]|[18][29]*[18])(4|5[29]*[18])*(3|5[29]*[07]))*(5|[18][29]*6|([07]|[18][29]*[18])(4|5[29]*[18])*([29]|5[29]*6)))([18]|4[29]*6|(3|4[29]*[18])(4|5[29]*[18])*([29]|5[29]*6)|([29]|4[29]*[07]|(3|4[29]*[18])(4|5[29]*[18])*(3|5[29]*[07]))(6|[18][29]*[07]|([07]|[18][29]*[18])(4|5[29]*[18])*(3|5[29]*[07]))*(5|[18][29]*6|([07]|[18][29]*[18])(4|5[29]*[18])*([29]|5[29]*6)))*(5|4[29]*3|(3|4[29]*[18])(4|5[29]*[18])*(6|5[29]*3)|([29]|4[29]*[07]|(3|4[29]*[18])(4|5[29]*[18])*(3|5[29]*[07]))(6|[18][29]*[07]|([07]|[18][29]*[18])(4|5[29]*[18])*(3|5[29]*[07]))*([29]|[18][29]*3|([07]|[18][29]*[18])(4|5[29]*[18])*(6|5[29]*3)))|(6|3[29]*5|([29]|3[29]*[18])(4|5[29]*[18])*([18]|5[29]*5)|([18]|3[29]*[07]|([29]|3[29]*[18])(4|5[29]*[18])*(3|5[29]*[07]))(6|[18][29]*[07]|([07]|[18][29]*[18])(4|5[29]*[18])*(3|5[29]*[07]))*(4|[18][29]*5|([07]|[18][29]*[18])(4|5[29]*[18])*([18]|5[29]*5))|([07]|3[29]*6|([29]|3[29]*[18])(4|5[29]*[18])*([29]|5[29]*6)|([18]|3[29]*[07]|([29]|3[29]*[18])(4|5[29]*[18])*(3|5[29]*[07]))(6|[18][29]*[07]|([07]|[18][29]*[18])(4|5[29]*[18])*(3|5[29]*[07]))*(5|[18][29]*6|([07]|[18][29]*[18])(4|5[29]*[18])*([29]|5[29]*6)))([18]|4[29]*6|(3|4[29]*[18])(4|5[29]*[18])*([29]|5[29]*6)|([29]|4[29]*[07]|(3|4[29]*[18])(4|5[29]*[18])*(3|5[29]*[07]))(6|[18][29]*[07]|([07]|[18][29]*[18])(4|5[29]*[18])*(3|5[29]*[07]))*(5|[18][29]*6|([07]|[18][29]*[18])(4|5[29]*[18])*([29]|5[29]*6)))*([07]|4[29]*5|(3|4[29]*[18])(4|5[29]*[18])*([18]|5[29]*5)|([29]|4[29]*[07]|(3|4[29]*[18])(4|5[29]*[18])*(3|5[29]*[07]))(6|[18][29]*[07]|([07]|[18][29]*[18])(4|5[29]*[18])*(3|5[29]*[07]))*(4|[18][29]*5|([07]|[18][29]*[18])(4|5[29]*[18])*([18]|5[29]*5))))(3|[07][29]*5|(6|[07][29]*[18])(4|5[29]*[18])*([18]|5[29]*5)|(5|[07][29]*[07]|(6|[07][29]*[18])(4|5[29]*[18])*(3|5[29]*[07]))(6|[18][29]*[07]|([07]|[18][29]*[18])(4|5[29]*[18])*(3|5[29]*[07]))*(4|[18][29]*5|([07]|[18][29]*[18])(4|5[29]*[18])*([18]|5[29]*5))|(4|[07][29]*6|(6|[07][29]*[18])(4|5[29]*[18])*([29]|5[29]*6)|(5|[07][29]*[07]|(6|[07][29]*[18])(4|5[29]*[18])*(3|5[29]*[07]))(6|[18][29]*[07]|([07]|[18][29]*[18])(4|5[29]*[18])*(3|5[29]*[07]))*(5|[18][29]*6|([07]|[18][29]*[18])(4|5[29]*[18])*([29]|5[29]*6)))([18]|4[29]*6|(3|4[29]*[18])(4|5[29]*[18])*([29]|5[29]*6)|([29]|4[29]*[07]|(3|4[29]*[18])(4|5[29]*[18])*(3|5[29]*[07]))(6|[18][29]*[07]|([07]|[18][29]*[18])(4|5[29]*[18])*(3|5[29]*[07]))*(5|[18][29]*6|([07]|[18][29]*[18])(4|5[29]*[18])*([29]|5[29]*6)))*([07]|4[29]*5|(3|4[29]*[18])(4|5[29]*[18])*([18]|5[29]*5)|([29]|4[29]*[07]|(3|4[29]*[18])(4|5[29]*[18])*(3|5[29]*[07]))(6|[18][29]*[07]|([07]|[18][29]*[18])(4|5[29]*[18])*(3|5[29]*[07]))*(4|[18][29]*5|([07]|[18][29]*[18])(4|5[29]*[18])*([18]|5[29]*5))))*([18]|[07][29]*3|(6|[07][29]*[18])(4|5[29]*[18])*(6|5[29]*3)|(5|[07][29]*[07]|(6|[07][29]*[18])(4|5[29]*[18])*(3|5[29]*[07]))(6|[18][29]*[07]|([07]|[18][29]*[18])(4|5[29]*[18])*(3|5[29]*[07]))*([29]|[18][29]*3|([07]|[18][29]*[18])(4|5[29]*[18])*(6|5[29]*3))|(4|[07][29]*6|(6|[07][29]*[18])(4|5[29]*[18])*([29]|5[29]*6)|(5|[07][29]*[07]|(6|[07][29]*[18])(4|5[29]*[18])*(3|5[29]*[07]))(6|[18][29]*[07]|([07]|[18][29]*[18])(4|5[29]*[18])*(3|5[29]*[07]))*(5|[18][29]*6|([07]|[18][29]*[18])(4|5[29]*[18])*([29]|5[29]*6)))([18]|4[29]*6|(3|4[29]*[18])(4|5[29]*[18])*([29]|5[29]*6)|([29]|4[29]*[07]|(3|4[29]*[18])(4|5[29]*[18])*(3|5[29]*[07]))(6|[18][29]*[07]|([07]|[18][29]*[18])(4|5[29]*[18])*(3|5[29]*[07]))*(5|[18][29]*6|([07]|[18][29]*[18])(4|5[29]*[18])*([29]|5[29]*6)))*(5|4[29]*3|(3|4[29]*[18])(4|5[29]*[18])*(6|5[29]*3)|([29]|4[29]*[07]|(3|4[29]*[18])(4|5[29]*[18])*(3|5[29]*[07]))(6|[18][29]*[07]|([07]|[18][29]*[18])(4|5[29]*[18])*(3|5[29]*[07]))*([29]|[18][29]*3|([07]|[18][29]*[18])(4|5[29]*[18])*(6|5[29]*3))))))*$

J'avoue que je trouve ça un peu magique et fascinant : j'ai beau l'avoir construite (avec une méthode tout à fait facile et standard pour convertir un automate fini en expression régulière), je n'arrive pas à bien me faire une idée de comment elle fonctionne.

Je ne sais pas non plus si on peut faire (substantiellement) plus court que ces 16233 caractères (on peut certainement gratter un peu, mais je veux dire que je ne sais pas si on peut faire quelque chose de vraiment différent) : je suppose que non, mais je ne sais pas le prouver.

↑Entry #1871 [older| permalink|newer] / ↑Entrée #1871 [précédente| permalien|suivante] ↑

↓Entry #1862 [older| permalink|newer] / ↓Entrée #1862 [précédente| permalien|suivante] ↓

(vendredi)

Quelques résultats scientifiques

J'évoquais hier le fait que je travaillais sur deux questions à la fois : voici que ces deux questions se sont reliées de façon inattendue, chacune apportant la solution de l'autre. À savoir :

  • l'hypothèse de Riemann est indécidable (dans ZFC et dans des systèmes beaucoup plus forts, devrais-je préciser), et
  • il existe un algorithme en temps polynomial pour factoriser les entiers (mais la complexité de cet algorithme n'est pas démontrable dans ZFC).

L'idée-clé de la démonstration du premier fait est d'associer à chaque zéro de la fonction zêta une démonstration dans un certain système formel 魚 (un peu compliqué à définir) : si le zéro ne se trouve pas sur l'axe critique, la démonstration prouvera ⊥ (i.e., une contradiction) dans ce système formel 魚 ; a contrario, si une contradiction se trouve, alors on peut l'utiliser pour produire des zéros non situés sur l'axe critique. Donc, l'hypothèse de Riemann équivaut à la consistance du système formel en question. Encore faut-il pouvoir en dire quelque chose ! C'est là qu'intervient le second point : ce système formel peut se voir, en fait, comme lié un protocole cryptographique 𓆛 (là aussi, les détails sont un peu compliqués) tel que prouver la sécurité du protocole 𓆛 revienne exactement à prouver la contradiction du système formel 魚. Or il est relativement facile de ramener la sécurité du protocole 𓆛 à la difficulté de la factorisation des entiers. Reste la dernière pièce du puzzle : ce protocole peut se voir comme un jeu à deux joueurs et, interprété dans le cadre de la théorie des jeux à la Conway, il définit naturellement un ordinal, qui se décrit comme l'écrasement d'un certain grand cardinal que j'appelle icthy un (c'est le premier d'une famille infinie), et qui mesure précisément la force du système formel 魚. Tout tombe donc dans ZFC augmenté de l'hypothèse le cardinal icthy un existe, et si on croit à cette hypothèse, les résultats ci-dessus sont démontrés.

↑Entry #1862 [older| permalink|newer] / ↑Entrée #1862 [précédente| permalien|suivante] ↑

↓Entry #1845 [older| permalink|newer] / ↓Entrée #1845 [précédente| permalien|suivante] ↓

(mardi)

Le paradoxe du second as, et le sens du conditionnement

Voici un paradoxe célèbre des probabilités :

Je tire cinq cartes au hasard d'un jeu de cartes (ordinaire, de 52 cartes). Deux questions séparées :

  • Supposant que j'aie un as dans mon jeu, quelle est la probabilité que j'aie un second as ?
  • Supposant que j'aie l'as de pique dans mon jeu, quelle est la probabilité que j'aie un second as ?

On s'attend à ce que la réponse à ces deux questions soit la même, selon un raisonnement intuitif du genre : la probabilité que j'aie un second as ne dépend pas du fait que le premier (celui supposé exister) soit l'as de pique, cœur, carreau ou trèfle ; et si la probabilité est la même que l'on suppose avoir l'as de pique, cœur, carreau ou trèfle, cela devrait aussi être la même si on suppose juste avoir un as, sans préciser lequel. Pourtant, ce n'est pas le cas : la réponse à la première est 12.2% (exactement 2257/18472), tandis que la réponse à la seconde est 22.1% (exactement 922/4165). En revanche, c'est (évidemment) vrai que la seconde question admettrait la même réponse si on remplaçait pique par l'un de cœur, carreau ou trèfle.

Et ce n'est pas une question de probabilité. Parmi les 2598960 mains de cinq cartes, il y en a 886656 (34.1%) qui contiennent au moins un as, 249900 (9.6%) qui contiennent au moins l'as de pique, 108336 (4.2% du total, donc 12.2% de celles contenant au moins un as) qui contiennent au moins deux as, et 55320 (2.1% du total, donc 22.1% de celles contenant au moins l'as de pique) qui contiennent au moins deux as dont l'as de pique. La vérification de ces nombres devrait être du niveau du programme de maths des terminales scientifiques (à savoir : 2598960=C(52,5) ; 886656=C(52,5)−C(48,5)=4×C(48,4)+6×C(48,3)+4×C(48,2)+48 ; 249900=C(51,4)=C(48,4)+3×C(48,3)+3×C(48,2)+48 ; 108336=C(52,5)−C(48,5)−4×C(48,4)=6×C(48,3)+4×C(48,2)+48 ; 55320=C(51,4)−C(48,4)=3×C(48,3)+3×C(48,2)+48). Mais je voudrais expliquer comment on pouvait arriver sans aucun calcul à la conclusion que la réponse à la seconde question était nécessairement plus élevée que celle à la première ; et ensuite, essayer d'expliquer pourquoi ces nombres nous paraissent surprenants (et pourquoi, en fait, notre intuition avait peut-être raison).

On peut diviser l'ensemble de toutes les mains possibles en cinq ensembles exclusifs et recouvrant tous les possibles, selon que le nombre d'as dans la main vaut exactement 0 (la main n'a pas d'as du tout), exactement 1 (la main contient un unique as), exactement 2, exactement 3, ou enfin 4 (la main contient tous les as) : appelons N0 (qui ne nous intéresse pas), N1, N2, N3 et N4 les nombres de main correspondantes. (Il se trouve que ces nombres valent respectivement 1712304=C(48,5), 778320=4×C(48,4), 103776=6×C(48,3), 4512=4×C(48,2) et 48, mais peu importe. Je veux justement éviter ce genre de calculs.) La réponse à la première question, i.e., la proportion des mains ayant au moins deux as parmi celles en ayant au moins un, est donc le rapport de N2+N3+N4 sur N1+N2+N3+N4 ; ou si on préfère, le complémentaire, i.e. la proportion des mains n'ayant qu'un seul as parmi celles qui en ont au moins un, est le rapport de N1 sur N1+N2+N3+N4. Maintenant, on peut de même diviser les mains ayant l'as de pique en quatre ensembles selon que le nombre total d'as vaut 1 (il n'y a que l'as de pique), 2 (il y a l'as de pique et un seul autre), 3 ou enfin 4 (la main contient tous les as). Si on appelle P1, P2, P3 et P4 les quatre nombres de mains correspondantes, il n'est pas difficile de les relier à N1, N2, N3 et N4 : on a évidemment P4=N4 (dans les deux cas, il s'agit de l'ensemble des mains contenant tous les as) et on a P1=¼×N1 (car parmi les mains contenant exactement un as, il y en a autant pour lesquelles il s'agit de l'as de pique, de cœur, de carreau et de trèfle ; et en réfléchissant un peu on se rend compte que P2=½×N2 (parmi les six chois possibles de deux as, il y en a trois qui contiennent pique et trois qui ne le contiennent pas) et que P3=¾×N3 (si on a trois as sur quatre, il y en a un seul qui manque, donc trois chances sur quatre d'avoir le pique). La réponse à la seconde question, i.e., la proportion des mains ayant au moins deux as parmi celles ayant l'as de pique, est le rapport de P2+P3+P4 sur P1+P2+P3+P4, soit de ½×N2+¾×N3+N4 sur ¼×N1+½×N2+¾×N3+N4 ; pour y voir plus clair, son complémentaire (la proportion des mains ayant uniquement l'as de pique parmi celles ayant au moins l'as de pique) est donc le rapport de P1 sur P1+P2+P3+P4, soit ¼×N1 sur ¼×N1+½×N2+¾×N3+N4. En multipliant par 4 le numérateur et le dénominateur de cette expression, on trouve donc que c'est le rapport de N1 sur N1+2×N2+3×N3+4×N4. Or sous cette forme il est clair (puisque le dénominateur est strictement plus grand) que c'est strictement moins que le rapport de N1 sur N1+N2+N3+N4 qui était le complémentaire de la réponse à la première question. On a donc montré qu'il y a strictement moins de chances d'avoir un unique as si on a au moins l'as de pique que d'avoir un unique as si on a au moins un as.

Essayons de dire ça de façon plus simple : quand on dit ma main contient l'as de pique, on restreint d'un facteur 4 les possibilités pour les mains contenant un unique as (P1 comparé à N1), mais on restreint d'un facteur plus petit celles contenant exactement deux as, trois as, et s'agissant de celles en contenant quatre, on ne les restreint pas du tout (P4=N4, car si on a les quatre as, on a certainement l'as de pique). Par conséquent, ces possibilités deviennent relativement plus probables quand on dispose de l'hypothèse ma main contient l'as de pique que quand on dispose simplement de l'hypothèse ma main contient un as. C'est ce qui est explicité dans le calcul ci-dessus.

Toujours pas éclairé ? Alors simplifions à l'extrême : le jeu ne contient plus que trois cartes, à savoir l'as de pique, l'as de scoubidou, et le valet de patate, et j'en tire deux au hasard. Alors il y a trois mains possibles : ma main contient forcément un as, et la probabilité d'en avoir deux est de 1/3 ; en revanche, si je sais que j'ai l'as de pique, ceci exclut une des trois mains possibles (as de scoubidou et valet de patate) et il y a maintenant une probabilité de 1/2 d'avoir les deux as.

Très bien, mais quelle est la morale de l'histoire, au juste ?

Se donner une hypothèse, en probas ou en stats, et mesurer des probas ou des proportions relativement à cette hypothèse, cela s'appelle conditionner. Conditionner par l'hypothèse j'ai un as dans ma main signifie qu'on se restreint aux mains ayant au moins un as, et qu'on calcule des proportions relatives à celle-ci (avec les notations ci-dessus, ceci signifie qu'on écarte N0) ; tandis que conditionner par j'ai l'as de pique signifie qu'on se restreint aux mains contenant l'as de pique et qu'on calcule de même relativement à cette hypothèse (avec les notations ci-dessus, ceci signifie qu'on remplace les N par des P). Conditionner est une opération fondamentale en probabilités et statistiques, mais pour lui donner un sens dans la vraie vie, il faut souvent se demander pourquoi on conditionne et quelle est au juste l'hypothèse.

L'idée qu'on ait la seule information j'ai un as dans mon jeu ou j'ai l'as de pique dans mon jeu est inhabituelle. Cela correspondrait au protocole expérimental suivant : Alice pioche cinq cartes, Bob lui demande explicitement dis-moi oui si tu as au moins un as dans ton jeu (et ne fais aucun autre commentaire), respectivement dis-moi oui si tu as l'as de pique dans ton jeu (et ne fais aucun autre commentaire), Bob entend Alice faire la réponse oui, et Bob en tire les conclusions expliquées ci-dessus quant à la probabilité qu'Alice ait un second as dans son jeu. Elles sont alors correctes, et finalement peu surprenante : c'est plutôt le protocole expérimental qui est bizarre, et le fait, absolument essentiel, que Bob ait demandé à Alice d'énoncer uniquement et exactement la réponse à la question as-tu au moins un as dans ton jeu, respectivement as-tu l'as de pique dans ton jeu. En revanche, si Alice pioche cinq cartes et énonce j'ai un as dans mon jeu ou j'ai l'as de pique dans mon jeu, on ne sait pas trop quoi conclure : si Alice avait au moins deux as, elle l'aurait sans doute dit, non ? Donc peut-être doit-on conclure simplement que la probabilité qu'elle ait au moins deux as est nulle (ou du moins, très faible), car Alice n'est pas du genre à faire une affirmation de logicien.

Pour revenir à quelque chose de plus mathématique, voici le plus important : si Bob demande à Alice si tu as au moins un as dans ton jeu, dis-moi la couleur d'un des as de ton jeu (choisi aléatoirement parmi ceux qui y sont) et qu'Alice répond j'ai l'as de pique, la probabilité qu'elle ait un second as est de 12.2% et pas de 22.1% : cette fois, on ne conditionne plus par l'information il y a l'as de pique dans le jeu d'Alice mais par Alice a énoncé pique en réponse à la question de Bob, ce qui apporte l'information qu'il y a au moins un as, plus l'information totalement sans intérêt que c'est l'as de pique qui a été choisi au hasard parmi les as d'Alice. Bref, ce qui importe pour bien conditionner n'est pas seulement de savoir la réponse, mais aussi de savoir la question à laquelle on répond. Et la raison pour laquelle les probabilités annoncées plus haut nous semblent paradoxales est justement que nous avons plutôt en tête le protocole beaucoup plus naturel que je viens de décrire : quand j'ai l'information l'as de pique est dans la main, ce n'est pas qu'on cherchait spécifiquement l'as de pique avec un détecteur à as-de-pique, mais plutôt qu'on cherchait un as, et qu'il s'est trouvé que c'était l'as de pique. Dans ce cas, le raisonnement que j'ai qualifié d'intuitif plus haut est correct, et la probabilité d'avoir un second as est bien de 12.2%.

La moralité, quand on fait des stats en sciences expérimentales, en sciences sociales, ou dans n'importe quel domaine, c'est donc : qu'il faut toujours se demander non seulement quelle est l'information connue (qui conduit à un conditionnement), mais aussi par quel dispositif expérimental on aboutit à cette information. Par exemple, si je fais un sondage en demandant aux gens quel(s) as avez-vous dans votre main (réponses multiples possibles), ce n'est pas du tout pareil de me restreindre à ceux qui ont déclaré avoir l'as de pique que si je demande avez-vous un des as suivant (aucun/pique/cœur/carreau/trèfle). Dans les deux cas je saurai peut-être qu'Alice a l'as de pique, mais c'est très différent de savoir qu'elle est dans la population des gens ayant l'as de pique ou dans la population des gens ayant au moins un as et ayant déclaré l'as de pique au hasard parmi tous les as de leur main.

Ajout () : cf. ce fil Twitter.

↑Entry #1845 [older| permalink|newer] / ↑Entrée #1845 [précédente| permalien|suivante] ↑

↓Entry #1829 [older| permalink|newer] / ↓Entrée #1829 [précédente| permalien|suivante] ↓

(mercredi)

Pierre-Papier-Ciseaux, thème et variations

Le jeu de Pierre-Papier-Ciseaux est probablement le deuxième jeu le plus simple de l'Univers. Avant d'en parler, je devrais donc commencer par décrire le premier, qui n'a pas de nom à ma connaissance, et je manque d'inspiration pour lui en trouver un. Les règles sont : l'un des deux joueurs (convenu à l'avance, disons Alice) choisit en secret le nom de l'un des deux joueurs, tandis que l'autre joueur (disons Bob) choisit soit le mot perd soit le mot gagne ; si la phrase formée des deux mots choisis est Alice gagne ou Bob perd, alors Alice gagne, tandis que si c'est Alice perd ou Bob gagne, alors Bob gagne. De façon plus abstraite : chacun des deux joueurs choisit un bit en secret, on calcule le XOR de ces bits, et s'il vaut 0 c'est Alice qui gagne tandis que s'il vaut 1 c'est Bob (ou toute autre convention équivalente choisie à l'avance). Ce jeu fait partie des jeux dits à somme nulle, c'est-à-dire que les joueurs sont exactement adversaires, les gains de l'un étant par définition les pertes de l'autre (par opposition à un jeu comme le dilemme du prisonnier, où il est possible pour que les deux joueurs gagnent ou perdent dans différentes proportions, et il y a donc un sens à chercher à coopérer) ; et à information incomplète imparfaite, parce que les deux joueurs choisissent leur option simultanément, donc dans l'ignorance du choix de l'autre (par opposition aux jeux à information parfaite, comme les échecs, qui sont l'objet de la théorie combinatoire des jeux). En pratique, le choix simultané peut se faire en écrivant sur un papier (si on est très formaliste, dans un pli scellé), ou, si on est cryptographe, en utilisant une fonction de hachage pour s'engager sur un choix sans divulguer celui-ci. Ou en montrant simultanément un choix de la main (par exemple, Alice pourrait pointer du doigt l'un des deux adversaires, tandis que Bob fait le signe du pouce vers le haut ou vers le bas).

La stratégie à suivre est tout à fait évidente : si Bob avait connaissance du nom choisi par Alice, il aurait intérêt à choisir perd lorsque Alice choisit Alice et gagne lorsque Alice choisit Bob ; donc Alice ne doit pas divulguer la moindre information sur son choix, et doit choisir aléatoirement et de façon équiprobable, indépendante des choix précédents, le nom qu'elle choisit ; Bob, évidemment, a intérêt à choisir aléatoirement entre gagne et perd. On dit qu'on a affaire là à la stratégie (mixte) optimale[#] du jeu pour l'un et l'autre joueur (qui se caractérise formellement par le fait qu'il s'agit d'une distribution de probabilité sur les options du jeu dont l'espérance de gain contre n'importe quelle option de l'adversaire est au moins aussi bonne que celle de toute autre telle distribution). Les deux stratégies optimales jouées l'une contre l'autre constituent un équilibre de Nash, mais la notion en question est un peu idiote dans le cas d'un jeu à somme nulle à cause d'un théorème de von Neumann qui assure, pour parler de façon très vague, que dans ce cas les stratégies optimales des deux joueurs se répondent bien l'une à l'autre (en maximisant le pire gain possible on maximise précisément le gain contre la stratégie optimale de l'adversaire). Le jeu très simple dont j'ai parlé est à valeur nulle, c'est-à-dire que quand les deux joueurs jouent leur stratégie optimale, aucun n'est privilégié, ce qui est le cas si, mais pas seulement si, il existe une symétrie totale entre les joueurs.

Ce jeu a beau être très simple, et sa stratégie optimale évidente, quand on y joue dans la vraie vie, tirer des valeurs au hasard n'est ni très facile ni très satisfaisant, et on se retrouve à jouer sur la psychologie de l'adversaire (comme dans la célèbre Battle of Wits de Princess Bride), surtout quand on joue plusieurs coups de suite : chacun essaie de se prévoir quel sera le choix de l'autre (en tenant compte du fait que l'autre essaie de prévoir, etc.), et de choisir le coup gagnant en fonction de cela. Je me souviens avoir joué à une variante de ce jeu contre un ami quand j'étais petit (sous la forme : mon adversaire choisit une carte dans un jeu et je dois deviner si elle est rouge ou noire), et avoir gagné une vingtaine de fois d'affilée, si bien que mon ami commençait à se dire que je lisais dans ses pensées. Mais bon, ce petit jeu n'a de sens que parce qu'on joue contre un humain qui joue aussi au même petit jeu : si l'un des deux joueurs tire son choix aléatoirement, l'autre n'a rien d'intelligent à faire, dans tous les cas son espérance de gain sera nulle. Autant jouer à pile ou face, et, bien sûr, c'est essentiellement ce qu'on est en train de faire.

↑Entry #1829 [older| permalink|newer] / ↑Entrée #1829 [précédente| permalien|suivante] ↑

↓Entry #1825 [older| permalink|newer] / ↓Entrée #1825 [précédente| permalien|suivante] ↓

(dimanche)

Les centres (points remarquables) d'un triangle

[Un triangle et dix points remarquables]Quand j'étais petit, j'ai appris la chose suivante : si A, B et C sont trois points (ce qui s'appelle un triangle), alors les trois droites (appelées médianes) qui relient chacun de ces trois points au milieu des deux autres, ces trois droites concourent en un point (appelé centre de gravité du triangle, noté G dans la figure[#] ci-contre), qui a la propriété d'être le centre de gravité de la surface du triangle (ou aussi de ses trois sommets avec une masse égale en chacun) ; les trois droites (les bissectrices) qui bissectent les trois angles du triangle sont également concourantes en un point (appelé centre du cercle inscrit, noté I dans la figure ci-contre), qui a la propriété d'être le centre du cercle intérieurement tangent aux trois côtés du triangle ; les trois droites (les médiatrices) perpendiculaires aux côtés et passant par leurs milieux sont également concourantes en un point (appelé centre du cercle circonscrit, noté O dans la figure ci-contre), qui a la propriété d'être le centre du cercle passant par les trois sommets du triangle ; et les trois droites (les hauteurs) perpendiculaires aux côtés et passant par le sommet opposé respectif sont également concourantes en un point (appelé orthocentre, noté H dans la figure ci-contre) ; et qu'en plus, le centre de gravité, le centre du cercle circonscrit et l'orthocentre sont eux-mêmes alignés sur une droite (appelée droite d'Euler). J'étais fasciné d'apprendre qu'à n'importe quel triangle on pouvait associer des points aussi naturels et possédant des propriétés aussi élégantes, et même si j'étais un peu déçu pour la symétrie des choses que le centre du cercle inscrit ne fût pas aligné avec les autres, c'était pour moi une découverte profonde.

Je pense que ces faits sont un test intéressant pour savoir si quelqu'un aimera les maths : ils sont assez faciles à présenter (et ne font intervenir aucun calcul, même si on veut en présenter une démonstration ou du moins une ébauche d'explication), et même si on peut tout à fait les trouver sans grand intérêt et avoir quand même goût pour les maths, je pense que si on est fasciné c'est le signe d'un certain esprit géométrique et qu'il faudrait chercher à en savoir plus. La tendance qu'ont trois droites naturellement définies dans un triangle à vouloir toujours concourir a quelque chose d'assez magique, il faut le reconnaître.

Maintenant, la folie des points remarquables dans les triangles a atteint un niveau un peu extrême, il faut le reconnaître : la célèbre encyclopédie de Kimberling des centres des triangles recense (actuellement ?) 3597 points remarquables ; il est assez facile de se convaincre qu'on peut en produire une infinité si on décide que (1) la droite reliant deux points remarquables dans un triangle est une droite remarquable, et (2) l'intersection de deux droites remarquables dans un triangle est un point remarquable.

Néanmoins, les six points suivants dans l'encyclopédie de Kimberling sont encore assez remarquables, à savoir le centre du cercle de Feuerbach (des « neuf points ») (noté X5 sur ma figure), le point de Lemoine (intersection des symmédianes, c'est-à-dire des symétriques des médianes par rapport aux bissectrices, noté X6 sur ma figure), le point de Gergonne (intersection des droites reliant chaque sommet au point de tangence du cercle inscrit avec le côté opposé, noté X7 sur ma figure), le point de Nagel (idem mais avec les points de tangence des cercles exinscrits, noté X8 sur ma figure), le Mittenpunkt (point de Lemoine du triangle des centres des cercles exinscrits, ou bien intersection des droites (GX7) et (HX10), noté X9 sur ma figure) et le centre de Spieker (centre du cercle inscrit du triangle formé par les milieux des côtés, noté X10 sur ma figure). Concernant le point de Spieker, je trouve assez remarquable le fait que c'est le centre de gravité du périmètre du triangle : c'est assez frappant que le centre de gravité de la surface du triangle coïncide avec celui de ses trois sommets, mais que pour le périmètre on trouve un point différent. Les quelques points de Kimberling suivants sont encore assez remarquable, mais ma patience ne va pas jusque là.

Mais voilà, je suis géomètre algébriste, alors je me suis demandé : ne peut-on pas présenter ce petit zoo des centres d'un triangle d'une façon un peu plus systématique, quitte à être plus sophistiqué (et moins géométrie-élémentaire-à-la-façon-des-Grecs) ? Voici ce à quoi je suis arrivé (attention, ce qui suit demande pour être compris quelques connaissances d'algèbre et de géométrie projective) :

↑Entry #1825 [older| permalink|newer] / ↑Entrée #1825 [précédente| permalien|suivante] ↑

↓Entry #1809 [older| permalink|newer] / ↓Entrée #1809 [précédente| permalien|suivante] ↓

(dimanche)

Affichage de l'hémisphère « trans » de la 3-sphère

Petit complément à l'entrée précédente : j'ai refait des vidéos qui me semblent meilleures. (Disponible à la même adresse que les précédentes, lesquelles ont été reléguées au sous-répertoire old/ ; et aussi sur la même playlist YouTube.) Outre que j'ai corrigé un problème d'orientation entre les vues avant et les vues de côté (décidément, POVray m'embrouille complètement avec son système de coordonnées), la principale différence avec les précédentes est que je montre maintenant aussi ce qui se passe dans l'hémisphère éloigné de l'observateur (l'hémisphère « trans »). Par une combinaison de décision éclairée et de fainéantise, j'ai choisi de rendre ces choses-là comme de simples lignes rouges sans texture ni épaisseur (bon, rouges, elles ne le sont plus tellement après l'encodage vidéo, mais je pense qu'on devinera bien de quoi je parle). Normalement, dans cet hémisphère, plus les objets sont lointains (c'est-à-dire, proches des antipodes), plus ils devraient paraître épais : j'ai trouvé que ça rendait les choses vraiment trop incompréhensibles, alors j'ai préféré une vue en fil de fer. (De toute façon, comme un commentateur me faisait remarquer sur l'entrée précédente, l'épaisseur des tubes n'est pas exacte quand ils sont lointains.) C'est déjà intéressant de voir les polyèdres rapetisser quand on se rapproche d'eux (pour ensuite passer dans l'hémisphère « cis » en se matérialisant pour grandir de nouveau).

Cela vaut notamment la peine de regarder les vues de côté : il est amusant de constater que bien qu'en regardant en face on avance manifestement tout droit, quand on regarde sur le côté, on se trouve en train de tourner autour d'un axe (regardez ce qui se passe du côté du polyèdre qui est au centre de la vue et à la frontière entre l'hémisphère cis/gris et l'hémisphère trans/rouge). C'est ça, aussi, vivre dans une 3-sphère : suivre un grand cercle c'est tourner autour d'un axe (qui est lui-même un grand cercle, et il y a une jolie autodualité des grands cercles dans la 3-sphère). Finalement, je crois que réaliser ces vidéos m'a permis de mieux comprendre la 3-sphère mais pas vraiment de mieux comprendre les polytopes réguliers en dimension 4.

Ah, et à la demande de quelqu'un, j'ai fait le 24-cellule. Comme je le disais, il est totalement sans intérêt car vraiment trop vide (et les autres solides réguliers le sont encore plus).

Le programme Perl qui a servi à générer toutes ces vidéos est disponible depuis ici.

↑Entry #1809 [older| permalink|newer] / ↑Entrée #1809 [précédente| permalien|suivante] ↑

↓Entry #1808 [older| permalink|newer] / ↓Entrée #1808 [précédente| permalien|suivante] ↓

(vendredi)

Naviguons dans des pavages de la 3-sphère

J'avais évoqué autrefois la question suivante : comment vaut-il mieux visualiser les solides réguliers en dimension 4 ? (Je rappelle qu'ils sont au nombre de 6, contre 5 en dimension 3 : le 4-simplexe ou pentachore, qui est l'analogue du tétraèdre et qui est son propre dual ; le tesseract ou hypercube, qui est l'analogue du cube ; le 16-cellule ou hyperoctaèdre ou 4-orthoplexe, qui est l'analogue de l'octaèdre et le dual du précédent ; le 24-cellule ou icosatétrachore, qui n'a pas d'analogue en dimension 3 et qui est son propre dual ; le 120-cellule ou hécatonicosachore, qui est l'analogue du dodécaèdre ; et le 600-cellule ou hexacosichore, qui est l'analogue de l'icosaèdre.) Comme les solides réguliers sont très populaires (même ma maman sait qu'il y en a cinq en dimension 3, parce que j'avais un mobile au-dessus de mon lit qui les représentait, quand j'étais petit — une œuvre de mon papa), il y a beaucoup de gens qui ont essayé de représenter ces 4-polytopes réguliers (voyez par exemple sur Google images) : les façons populaires d'essayer consistent par exemple à projeter orthogonalement (en faisant éventuellement tourner en même temps), à utiliser une projection stéréographique, à faire des patrons, ou encore à utiliser le temps comme 4e dimension.

Mais il y a une autre façon de faire, que je n'ai encore jamais vue employée, et qui consiste à se rappeler qu'une autre façon de considérer les solides réguliers est de les voir comme des pavages de la sphère. Pour expliquer dans le cadre plus familier de la 2-sphère en dimension 3, le dodécaèdre, par exemple, peut être considéré comme un solide convexe en dimension 3, mais il peut aussi être considéré comme vivant sur la sphère (imaginez qu'au lieu d'inscrire votre dodécaèdre dans la sphère vous le gonfliez jusqu'à ce qu'il coïncide avec elle, mais en gardant le souvenir des limites des faces ; un peu comme un ballon de football traditionnel est cousu de pentagones et d'hexagones) : le dodécaèdre est alors une façon de paver la sphère avec 12 pentagones réguliers. Pour mémoire, les pentagones réguliers ne pavent pas le plan, et c'est bien le signe que la sphère a une courbure positive qu'on peut la paver avec des pentagones réguliers de sorte que trois se touchent à chaque sommet. De même, l'icosaèdre peut être vu comme une façon de paver la sphère avec 20 triangles équilatéraux (certes les triangles équilatéraux pavent déjà le plan, mais là on en met cinq autour de chaque sommet, alors que pour paver le plan on en met six : de nouveau, on voit que la courbure est positive), le cube comme une façon de paver la sphère avec six carrés, etc.

[Vue du 120-cellule dans la 3-sphère]Exactement la même chose fonctionne une dimension au-dessus : chacun des six solides réguliers de la dimension 4 correspond à un pavage régulier de la 3-sphère. Pour le 120-cellule, par exemple, c'est avec 120 dodécaèdres réguliers : les dodécaèdres réguliers ne pavent pas l'espace euclidien, mais grâce à la courbure de la 3-sphère ils arrivent à la paver, elle. Vous allez me dire que ça n'aide pas vraiment à visualiser les choses, mais en fait si : la 3-sphère étant un espace de dimension 3, fût-il courbe, on peut espérer l'appréhender. L'idée, donc, est de représenter ce que verrait d'un 120-cellule (par exemple) quelqu'un qui vivrait sur (ou faut-il dire dans ?) une 3-sphère : un pavage régulier de tout son univers par 120 dodécaèdres réguliers.

Quand j'ai proposé ça, je me disais que ce serait vraiment fastidieux à calculer : aucun programme de raytracing n'est prévu pour faire des calculs dans des espaces courbes, après tout. Mais j'avais une idée confusément à l'arrière de la tête, qui a fini par ressortir quand un ami s'est proposé d'essayer de réaliser ces calculs : la projection gnomonique.

La projection gnomonique est une façon de projeter une sphère sur un plan (ou les objets analogues en dimension supérieure). Elle consiste à projeter depuis le centre de la sphère sur un plan tangent à elle en un point (qu'on appelle le centre de la projection) ; c'est donc quelque chose de très facile à calculer, mais elle est bizarrement beaucoup moins populaire et moins connue que la projection stéréographique (avec laquelle il ne faut pas confondre) qui, elle, projette depuis le point de la sphère opposé au centre de projection. Alors que la projection stéréographique conserve les angles (on dit qu'elle est conforme), la projection gnomonique conserve l'alignement : les grands cercles de la sphère (c'est-à-dire ses géodésique, ou « droites », ce que les navigateurs appellent des orthodromies) deviennent des droites dans la projection gnomonique (c'est assez évident sur la description géométrique). Autrement dit, le plus court chemin pour aller de A à B (au moins s'ils sont dans le même hémisphère que le centre de projection) s'obtient juste en traçant une droite sur la projection gnomonique. La plupart des gens (et même pas mal de mathématiciens) sont intimement persuadés que c'est impossible : comme la sphère est courbe, on ne peut pas envoyer les droites sur des droites entend-on parfois dire. Pourtant, la projection gnomonique fait bien ça (et c'est la seule), et elle mériterait d'être plus connue même si elle a des défauts qui la rendent peu utilisable en cartographie. Pour les géomètres : alors que la projection stéréographique identifie naturellement la sphère à la droite projective complexe (on parle de sphère de Riemann), la projection gnomonique, elle, identifie naturellement la sphère modulo antipodie au plan projectif réel.

Quel rapport avec mon problème ? Le point important est que la projection gnomonique fonctionne aussi bien de la 3-sphère vers l'espace euclidien de dimension 3, et ce que voit de la 3-sphère quelqu'un qui vit dessus est donné justement par la projection gnomonique centré au point où se trouve cet observateur. Voici la façon dont je justifiais ce fait dans un mail à un ami :

Le truc c'est le suivant : (1) pour quelqu'un qui vit dans S³ (en un point P, disons), les grands cercles de S³ apparaissent comme des droites, et (2) plus précisément, ce qu'il voit d'une configuration de droites (=grands cercles) de S³ est exactement la même chose que verrait quelqu'un dans ℝ³ qui verrait la projection gnomonique de ces droites à condition que cette projection gnomonique soit centrée en P.

La projection gnomonique d'une sphère centrée en un point P de celle-ci, c'est la projection centrale depuis le centre de la sphère (et pas depuis le point antipodal à P qui serait la projection stéréographique, bien plus connue) et sur l'hyperplan tangent à la sphère en P. En fait, c'est bêtement l'identification de la sphère quotientée par l'antipodie avec l'espace projectif de même dimension. L'intérêt, c'est que (a) la projection gnomonique préserve les droites (parce que les droites sont les intersections de la sphère avec des plans passant par l'origine). Par ailleurs, (b) au point P (mais seulement lui), la projection gnomonique préserve les angles (parce que c'est clair que la différentielle en ce point est l'identité, vu qu'on a justement pris le plan tangent). Ces deux propriétés (a) et (b) prouvent mon point (2) (puisque ce que voit un observateur en P sur S³ est déterminé par les angles des grands cercles passant par P et des points de la configuration de grands cercles proposée). Et mon point (2) implique notamment mon point (1).

Bon, je m'explique sans doute comme un pied, mais en tout cas la morale est que pour représenter ce que voit d'un solide régulier dans S³ un observateur P dans S³, on peut faire la chose suivante : déterminer l'image (dans ℝ³) par projection gnomonique centrée en P de tous les sommets et toutes les arêtes, et filer ça à un raytracer. L'avantage, c'est qu'on n'a pas à raytracer soi-même.

Sauf qu'il y a quand même un problème […] : pour les points qui sont dans l'hémisphère « cis » (celui qui contient P et qui est centré en P), tout se passe bien, mais pour les points qui sont dans l'hémisphère « trans », ils devraient apparaître au-delà de l'infini et on ne peut pas demander ça à un raytracer. (Concrètement, un mec qui vit dans ℝ³ et qui regarde une droite ne voit qu'une demi-droite qui se termine à un « point de fuite » ; alors qu'un mec qui vit dans S³ et qui regarde un grand cercle va vraiment voir le truc se prolonger tout à travers son champ de vision.)

J'ai quand même fait l'expérience de filer les choses à un raytracer (POVray), en me limitant aux points de l'hémisphère « cis » (et lorsqu'un segment relie les deux hémisphères, je le fais partir à l'infini) : pour les solides un peu remplis (le 120-cellule (« hécatonicosachore ») et le 600-cellule (« hexacosichore »)), c'est très joli. On voit qu'il manque l'hémisphère trans, mais le fait que les rayons partent à l'infini donne l'impression qu'il est noyé dans une brume noire impénétrable. Pour les autres solides réguliers, par contre, ça n'a aucun intérêt : ça donne juste un grand truc essentiellement vide.

Et pour faire une vidéo, je fais tourner la 3-sphère, ça n'apporte aucune difficulté supplémentaire.

[Vue du 600-cellule dans la 3-sphère]Bref, j'ai réalisé des petites vidéos de tout ça : une visite du 120-cellule (qui pave la 3-sphère par 120 dodécaèdres réguliers, donc), et une visite du 600-cellule (qui la pave par 600 tétraèdres réguliers). Elles sont en basse qualité sur YouTube et téléchargeables en plus haute qualité ici (pour une fois elles sont assez petites pour que je ne m'embête pas à fabriquer des torrents pour le téléchargement). Si on regarde attentivement, on se rend bien compte que l'espace dans lequel on évolue est courbe : les polyèdres et polygones censés être réguliers ne le sont pas vraiment (ce qui est normal, vu que réguliers dans un espace plat ils ne peuvent pas paver), et les angles semblent se modifier subtilement quand la caméra se déplace. Et comme je l'explique ci-dessus, je ne montre malheureusement que ce qui se passe dans l'hémisphère « cis » de la 3-sphère (l'hémisphère proche de l'observateur). Mise à jour () : je montre maintenant aussi l'hémisphère « trans » avec des lignes rouges, cf. l'entrée suivante.

Je fournirai prochainement ici le programme Perl qui a servi à générer (les fichiers POVray de) ces animations. Mise à jour : voir ici.

Par ailleurs, il serait intéressant de faire des animations semblables pour des pavages de l'espace hyperbolique : la géométrie hyperbolique est en quelque sorte le pendant à courbure négative de la géométrie sphérique, par exemple on peut y faire des pentagones réguliers à angles de 90° alors que les pentagones réguliers du 120-cellule ont des angles de 120° et en géométrie euclidienne bien sûr c'est 108°. (Par ailleurs, les « vrais » géomètres nous apprennent que la courbure négative est beaucoup plus intéressant que la courbure positive.) Il y a une image de ce genre ici, mais j'aimerais faire mieux. La projection gnomonique existe aussi pour l'espace projectif, donc la même approche fonctionne (avec la difficulté des deux hémisphères en moins, remplacée par la difficulté que les pavages sont infinis).

↑Entry #1808 [older| permalink|newer] / ↑Entrée #1808 [précédente| permalien|suivante] ↑

↓Entry #1807 [older| permalink|newer] / ↓Entrée #1807 [précédente| permalien|suivante] ↓

(mardi)

Encore des petites conventions de matheux

Addendum à l'entrée précédente : on me fait remarquer que le maître de cérémonies d'un séminaire a, ou en tout cas devrait avoir, d'autres rôles : par exemple, quand le séminaire vise un certain public (et notamment si ce public est large), s'assurer que l'orateur reste compréhensible par ce public en interrompant, si besoin est, pour demander de rappeler la définition de tel ou tel concept, ou l'énoncé de tel ou tel théorème (même si le maître de cérémonies lui-même les connaît, probablement : le but est d'éviter aux gens qui ne connaissent pas d'être trop timides pour demander). Malheureusement, ce rôle-là est rarement tenu. De façon plus pragmatique, l'intérêt de la phrase d'introduction est aussi de demander à l'assistance de faire silence (dans la pratique, j'ai rarement l'impression que ce soit nécessaire, sauf dans de très grosses conférences), et de pousser un orateur peut-être un peu timide à se lancer.

Tant que j'y suis, je signale un autre aspect du « savoir-vivre » mathématique que je trouve amusant : quand dans un exposé de séminaire on cite un résultat, on écrit quelque chose comme Théorème (Duschnock, 1995) en donnant l'année de publication de l'article quand c'est effectivement publié ; mais si on cite un théorème qu'on a soi-même découvert, on met juste son initiale (ou ses initiales si on a l'habitude d'écrire les prénoms) et la date éventuelle. Il m'a fallu un certain temps pour comprendre ça.

Il y a sans doute quantité de règles comme ça que j'ai apprises et qui me semblent maintenant assez évidentes. Mais c'est un problème pour les gens qui débutent dans le métier : ce genre de règles n'est écrit nulle part — normalement ce devrait être le directeur de thèse qui les fait remarquer, mais il n'y pense pas toujours, ou parfois il n'est pas là. Ne pas les respecter peut être embarrassant (je me souviens avoir commis un impair une fois, mais je ne me rappelle plus de quoi il s'agissait).

↑Entry #1807 [older| permalink|newer] / ↑Entrée #1807 [précédente| permalien|suivante] ↑

↓Entry #1806 [older| permalink|newer] / ↓Entrée #1806 [précédente| permalien|suivante] ↓

(vendredi)

Le petit rituel des séminaires

Cet après-midi, j'ai assisté à trois heures d'exposés : une d'un groupe de travail géo.alg/crypto à Télécom (dont je suis vaguement un co-organisateur) et deux du séminaire Variétés rationnelles (qui pourrait s'appeler séminaire-des-élèves-de-Colliot-Thélène). Les séminaires de mathématiques (et probablement de beaucoup d'autres disciplines, mais je n'en ai pas vu assez pour pouvoir conclure de façon ferme) obéissent à un petit rituel qui m'agace (et que j'ai insisté pour éliminer à notre groupe de travail à Télécom) : ce rituel fait qu'il y a dans un exposé de séminaire deux personnes jouant un rôle distingué : l'orateur (ou conférencier), évidemment, qui est celui qui parle l'essentiel du temps, mais aussi quelqu'un qui est normalement un des organisateurs du séminaire et que j'appellerai (faute de connaître un nom plus standard à ce rôle) le maître de cérémonie (on pourrait aussi dire : le présentateur). Quand il y a plusieurs organisateurs d'un séminaire, ils exercent généralement la fonction de maître de cérémonie à tour de rôle, car ce rôle ne peut pas être collégial. Le maître de cérémonie doit impérativement être distinct de l'orateur : aujourd'hui, au séminaire Variétés rationnelles, comme un des organisateurs du séminaire était absent et que l'autre était un des exposants, il a fallu qu'une autre personne (en l'occurrence, Colliot-Thélène) joue ce rôle.

Le rôle en question, donc, est tout à fait rituel : lorsque le séminaire va commencer, le maître de cérémonie prononce l'incantation propitiatoire (quasiment verbatim) : Nous écoutons maintenant $nom_du_conférencier, qui va nous parler de $titre_de_l_exposé. Lorsque l'orateur a fini de parler, le maître de cérémonie dit sobrement merci, et l'assistance peut applaudir (cependant, au séminaire Variétés rationnelles, quand Colliot-Thélène était un des organisateurs, on n'applaudissait pas, selon le principe, avec lequel je suis plutôt d'accord, qu'un exposé de mathématiques n'est pas un tour de magie et qu'il n'y a pas spécialement lieu de manifester son émerveillement). Ensuite, le maître de cérémonie demande à l'assistance y a-t-il des questions ?, les gens les posent (sans attendre qu'on les invite individuellement à parler), et quand les questions sont taries, le maître de cérémonie a le droit d'en poser lui-même (ce qui est bien vu si l'assistance n'en a pas posé du tout, histoire qu'il y en ait au moins une), et enfin il invite l'assistance à remercier de nouveau l'orateur (on applaudit donc deux fois, y compris s'il n'y a eu aucune question). Tout ceci est extrêmement codifié.

Il y a, bien sûr, des variantes nationales. En anglais (langue dans laquelle se font la plupart des séminaires hors des mathématiques, et même en mathématiques un grand nombre de séminaires et la majorité des conférences), le maître de cérémonies annoncera : Our ${if_first_speaker?first:next} speaker for today is $speakers_name, who will talk about $title_of_talk ; et à la fin : Let us thank the speaker (on remarquera qu'en anglais le maître de cérémonie invite l'assistance à remercier l'orateur alors qu'en français il le remercie lui-même et l'assistance se joint spontanément à ces remerciements), et enfin : Let us thank the speaker again. En allemagne, on n'applaudit pas, on cogne la table avec l'articulation des doigts (la main étant repliée en poing) — cela surprend la première fois. (D'un autre côté, j'ai un ami qui m'a fait remarquer que si une seule fois on regarde des gens en train d'applaudir en se disant mentalement, mais qu'est-ce que c'est que ce rituel idiot ?, il est ensuite impossible de s'enlever cette idée de la tête et on a à tout jamais du mal à se retenir de rire quand on se trouve dans un groupe de gens en train d'applaudir.)

Bref. Je reconnais que le maître de cérémonie joue un rôle important, qui est justement celui que je n'ai pas décrit : il fait comprendre à un orateur qui déborderait son temps qu'il faut songer à arrêter. (Ou il répond à la semi-question s'il me reste $n minutes…? soulevée par l'orateur.) En revanche, le rôle de prononciateur de la phrase magique qui ouvre le séminaire, je trouve ça vraiment idiot. Je ne dis pas ça que par iconoclasme : cette façon de procéder (qui est essentiellement celle des soutenances de thèse) donne aux séminaires un côté plus formel, plus rigide, donc aussi plus intimidant (notamment pour les jeunes orateurs) que si l'orateur se contentait de sonder vaguement le premier rang pour savoir s'il peut commencer (et pareil quand il s'agit de savoir s'il reste du temps). Le nom de l'orateur n'a pas besoin d'être répété, a priori tout le monde l'a lu sur l'affiche ou l'annonce du séminaire (et il vaut mieux le voir écrit de toute façon, parce que sinon on va se tromper en l'écrivant), et quant au titre, l'orateur peut très bien l'écrire au tableau. De même, je préfère que l'orateur demande lui-même s'il y a des questions : c'est lui, après tout, qui vient de faire une communication, il est normal que ce soit lui qui veuille savoir si on veut lui en demander plus. Bref, je préfère qu'un exposé ressemble à une discussion informelle entre collègues (et égaux) qu'à une soutenance de thèse ou je ne sais quel cérémonial codifié.

Il y a d'ailleurs au moins un séminaire prestigieux où les choses se passent sans cérémonial, c'est le séminaire Bourbaki. Je suppose que la logique est que le maître de cérémonie serait logiquement Bourbaki, mais que bizarrement il n'est pas présent. Je regrette d'ailleurs qu'il n'y ait pas une petite blague récurrente qui consisterait à ce qu'à chaque séminaire Bourbaki quelqu'un aléatoirement se lève pour annoncer à l'assistance : Monsieur Nicolas Bourbaki, organisateur de ce séminaire, vous prie de bien vouloir l'excuser pour son absence, due à $raison_inventée_rigolote_et_différente_à_chaque_fois. L'humour mathématicien français a l'air d'avoir un peu décliné depuis les années 30. Au demeurant, j'ai entendu des gens se plaindre du séminaire Bourbaki du fait de son extrême opacité (on ne sait pas comment les orateurs sont choisis, ni d'où vient l'argent), donc je ne sais pas si c'est un modèle à suivre : mais j'apprécie l'absence du cérémonial religieux qui commence la grande majorité des autres séminaires.

↑Entry #1806 [older| permalink|newer] / ↑Entrée #1806 [précédente| permalien|suivante] ↑

↓Entry #1805 [older| permalink|newer] / ↓Entrée #1805 [précédente| permalien|suivante] ↓

(jeudi)

Onze vues du graphe de Higman-Sims

[Le graphe de Higman-Sims]Je continue dans mon exploration graphique un peu aléatoire d'objets mathématiques exceptionnels et mysérieux avec le graphe de Higman-Sims, dont je viens de produire cinq onze vues.

(Mise à jour () : J'avais initialement écrit cinq, alors que j'avais produit six vues, je ne sais pas pourquoi ce lapsus. Mais de toute façon, je me suis rendu compte entre temps que j'avais mal regardé les symétries et que ma méthode produit non pas cinq ou six vues différentes mais onze. Pour me faire pardonner, voici une jolie animation (également en mauvaise qualité sur YouTube) de ce graphe, qui passe par toutes ces différentes vues (comme d'habitude, si vous n'arrivez pas à télécharger par BitTorrent, vous pouvez retirer l'extension .torrent à la fin de l'URL pour accéder directement au fichier).

Ajout : Les vues en SVG sur Wikimédia Commons : 1, 2, 3, 4, 5, 6, 7, 8, 9, 10, 11.

Le graphe de Higman-Sims est l'unique graphe ayant 100 sommets, chacun étant adjacent à 22 autres (ce qui fait donc 1100 arêtes au total), de sorte que deux sommets adjacents n'ont jamais de voisin commun (i.e., le graphe n'a aucun triangle) et que deux sommets non adjacents ont exactement 6 voisins en commun. Comme tout objet mathématique exceptionnel qui se respecte, il admet quantité de définitions ou de constructions équivalentes. Son groupe d'automorphismes (d'ordre 88704000), c'est-à-dire l'ensemble des façons dont on peut permuter les sommets en préservant la relation d'adjacence entre eux, est, à une extension d'ordre 2 près, le groupe du même nom, l'unique groupe simple fini d'ordre 44352000 et un des vingt-six groupes sporadiques. Si on fixe un sommet quelconque, le groupe restant (le stabilisateur d'un sommet, donc) est, de nouveau à une extension d'ordre 2 près, le groupe M22 de Mathieu (qu'on rencontre régulièrement sur ce blog) — ce qui explique que l'ordre du groupe de Higman-Sims soit 100 fois celui du groupe M22.

Il y a une autre façon de décrire le groupe de Higman-Sims, c'est comme les automorphismes d'un certaine structure combinatoire appelée la géométrie de Higman (qui comporte 176 « points » et 176 « quadriques », chaque quadrique comportant 50 points et chaque point étant sur 50 quadriques — et l'automorphisme extérieur du groupe de Higman-Sims correspondant à échanger points et qudratiques). Cette description a été trouvée indépendamment, et on ne s'est pas rendu compte immédiatement qu'il s'agissait du même groupe. Là où le Club Contexte est très content, c'est que le Higman du groupe et du graphe Higman-Sims (Donald G. Higman, un Américain) n'est pas le même que le Higman de la géométrie de Higman (Graham Higman, un Britannique) ; il semble qu'ils n'aient même pas de lien de parenté évident, c'est tout de même très fort. (Les blagueurs les appelaient les Higmen.)

En fait, mes dessins représentent le graphe de Higman-Sims pas seulement comme un graphe abstrait mais comme un minuscule petit bout d'un autre objet mathématique exceptionnel et extraordinaire, le réseau de Leech.

Le réseau de Leech est réseau régulier de points en 24 dimensions qui a des propriétés absolument mirobolantes ; on peut par exemple le décrire comme la (seule) façon d'empiler des sphères de même rayon, en 24 dimensions, de façon que chaque sphère en touche 196560 autres (les sphères sont centrées sur les points du réseau de Leech) ; il est assez apparenté au réseau engendré par le système de racines E8 dont je parlais récemment (et qui permet d'empiler des sphères en 8 dimensions, de façon que chaque sphère en touche 240 autres), si ce n'est que le réseau de Leech n'est pas associé à un système de racines. Les isométries laissant invariant le réseau de Leech (et fixant l'origine) forment un autre groupe remarquable, le groupe ·0 (lire dot-oh) de Conway, qui est une extension par 2 de l'unique groupe simple fini (appelé ·1 par Conway) d'ordre 4157776806543360000, encore un sporadique. Malheureusement, il n'est pas question de représenter le réseau de Leech (ou même sa première couche), parce que déjà un polyèdre ayant 240 sommets en dimension 8 ayant 696729600 symétries, on n'y voit rien, alors représenter un polyèdre ayant 196560 sommets en dimension 24 et ayant 8315553613086720000 symétries c'est un peu désespéré.

Néanmoins, on peut au moins représenter des tout petits bouts du réseau de Leech. Il se trouve que le graphe de Higman-Sims (comme la géométrie de Higman, d'ailleurs) apparaît naturellement comme un tout petit bout du réseau de Leech : si on choisit trois points quelconques du réseau dont les distances entre eux valent 3, 3 et 2, alors il y a exactement 100 points du réseau qui sont à distance 2 de chacun des trois points choisis, et si on relie par une arête ceux qui sont à distance 3, on obtient le graphe de Higman-Sims ; ceci plonge le groupe de Higman-Sims dans le groupe ·1 de Conway comme le groupe des isométries du réseau de Leech fixant le triangle choisi. Les dessins que j'ai fait sont des projections orthogonales du graphe de Higman-Sims tel que je viens d'expliquer qu'il apparaît dans le réseau de Leech, en choisissant une projection qui révèle la symétrie d'ordre 11 (il n'y a qu'une seule classe de conjugaison d'ordre 11 dans ·0, et elle est dans le groupe de Higman-Sims et, en fait, dans M22). Les 100 points sont organisés en un point au centre et 9 endécagones réguliers autour de lui ; et comme il restait un peu de liberté j'ai fait en sorte que les deux endécagones formés des points directement reliés au point central soient symétriques l'un de l'autre (formant donc un 22-gone régulier) — ce qui ne laisse plus, je crois, que les six projections que j'ai tracées.

Ça ne rend pas très joliment sur l'écran, mais imprimé sur du papier A3 c'est vraiment très esthétique. Ce n'est qu'un minuscule aspect des élégantes symétriques du réseau de Leech, mais c'est déjà inspirant.

↑Entry #1805 [older| permalink|newer] / ↑Entrée #1805 [précédente| permalien|suivante] ↑

↓Entry #1802 [older| permalink|newer] / ↓Entrée #1802 [précédente| permalien|suivante] ↓

(samedi)

Jouons avec le système de racines E8

Comme j'étais déçu de ne pas avoir gagné la même célébrité et fortune avec mes taquins de Mathieu (244823040 = 210×33×5×7×11×23 combinaisons possibles) qu'Ernő Rubik avec son fameux cube (43252003274489856000 = 227×314×53×72×11 combinaisons), voici une nouvelle tentative de puzzle mathématique à 696729600 = 214×35×52×7 combinaisons, sur cette page-ci (qui ne marchera que sur un navigateur assez récent gérant l'élément <canvas>). On peut voir ça comme un petit jeu (appuyer sur scramble et cliquer sur les points jusqu'à réussir à tout remettre à sa place) ou comme un outil pour essayer de comprendre le groupe de Weyl de E8 (à ne pas confondre avec le groupe de Weyl de E6 dont je parlais récemment). Les explications mathématiques et le mode d'emploi sont au bout du lien. C'est plus simple que le Rubik's cube parce que finalement on ne fait que bouger l'objet en bloc sans le déformer, mais c'est plus compliqué parce que l'objet en question vit en huit dimensions.

Et c'est frustrant, décidément, quoi que je fasse, je n'arrive pas à visualiser huit dimensions (il faut dire que je n'arrive déjà pas à en visualiser trois, alors…). Le système de racines E8 est un de ces objets mathématiques exceptionnels et un peu mystérieux que je trouve complètement fascinants et qui devraient être d'une beauté insoutenable si on arrivait à les voir proprement à défaut de seulement les comprendre. En l'occurrence, celui-ci est un peu comme un solide régulier (même s'il n'est pas régulier avec la définition usuelle du terme, c'est-à-dire si on demande que le groupe de symétries soit transitif sur les drapeaux : il n'y a que trois solides réguliers en toute dimension à partir de 5), mais il a quantité d'autres propriétés remarquables.

Le système de racines E8 est le point de départ pour construire un autre objet remarquable qui en découle naturellement, et qui est E8 lui-même. Encore plus compliqué à visualiser, puisque lui a 248 dimensions (comme 8+240, où 8 est le nombre de dimensions dans lequel vit le système de racines E8 et 240 est le nombre de racines de ce dernier). Et un jour où je serai vraiment en forme, je ferai un puzzle basé sur E8(𝔽2), qui a, pour sa part, 337804753143634806261388190614085595079991692242467651576160959909068800000 = 2120×313×55×74×112×132×172×19×312×41×43×73×127×151×241×331 éléments[#].

[#] C'est beaucoup plus que les malheureux 808017424794512875886459904961710757005754368000000000 = 246×320×59×76×112×133×17×19×23×29×31×41×47×59×71 du Monstre, même si le Monstre est en fait beaucoup plus difficile à réaliser parce que pour E8(𝔽2) on peut utiliser des matrices de taille 248×248 alors que pour le Monstre il faut a priori des matrices de taille 196883×196883, ou peut-être 196882×196882 si on travaille sur 𝔽2 mais en tout cas c'est énorme. Par contre, c'est toujours moins que le nombre de protons de l'Univers, qui vaut notoirement exactement 15747724136275002577605653961181555468044717914527116709366231425076185631031296.

↑Entry #1802 [older| permalink|newer] / ↑Entrée #1802 [précédente| permalien|suivante] ↑

↓Entry #1801 [older| permalink|newer] / ↓Entrée #1801 [précédente| permalien|suivante] ↓

(mardi)

Powers of Ten

Beaucoup connaissent sans doute déjà le célèbre film scientifique Powers of Ten de Ray et Charles Eames, qui présente la taille relative des choses dans l'Univers, et des puissances de dix, par un zoom à travers le cosmos, entre un panorama qui englobe de nombreuses galaxies et l'intérieur d'un proton dans la peau d'un homme qui dort après un pique-nique à Chicago (ce pique-nique constituant la scène initiale du film, et le milieu du zoom). Sinon, je vous encourage à le voir (cf. aussi ici).

Je l'ai vu pour la première fois en 1984 au Ontario Science Centre (quand mes parents et moi habitions Toronto) — ce même musée des sciences dont je me plaignais il y a trois ans qu'il était devenu juste une attraction ludique pour gamins. Il y avait une petite salle où il passait en boucle, et mon père et moi (mon père surtout, mais moi aussi) en étions fans et nous l'avons vu de nombreuses fois.

Sauf que c'est un peu plus subtil : il y a deux versions du film. Celle que j'ai vue et revue en 1984, c'est la version de 1968, qui est en noir et blanc si je me rappelle bien. Plus tard, le Science Centre a changé et a mis la version de 1977, en couleur (je crois que je l'ai vue en 1988 quand nous sommes retournés à Toronto pour un été), et c'est cette version-là qu'on voit maintenant partout (y compris sur le lien vers YouTube que je donne plus haut). La différence notable entre la version de 1968 et son remake, c'est que la version ancienne, dans la partie du voyage des puissances de dix qui zoome vers l'extérieur et vers le cosmos, affichait les effets relativistes (le temps qui s'écoule pour le voyageur et le temps qui s'écoule sur Terre, notamment, au fur et à mesure que la vitesse s'approche de celle de la lumière). Cela a probablement été jugé trop difficile à comprendre et un peu hors sujet, et éliminé de la version suivante. Mais mon père aimait beaucoup mieux cette première version, et a été déçu quand le film a changé.

Toujours est-il que la version de 1968 est apparemment introuvable sur le Web. C'est dommage. Il y a cependant un DVD, trouvable sur Amazon (mais uniquement d'occasion), qui contient apparemment les deux versions : du moins si j'en crois un commentaire qui confirme mon souvenir à ce sujet :

The primary difference between the two versions is that in the first version, there is a side window kept running throughout the movie, which shows the effect of relativity on the time-keeping of ten seconds per order of magnitude of meters travelled. Around the time the "camera" pulls back from 10-to-the-13th to 10-to-the-14th meters, the subjective time-sense of the camera operator would start to be strongly affected by relativity, because the "camera" would start to be travelling at a significant fraction of the speed of light. Gradually, subjective and Earthly time-sense gets so far out of whack that ten seconds for the cameraman would be 100,000,000 years on Earth. This might have the effect of prompting the philosophically-inclined viewer to get the screaming meemies, but it's better not to sweat the phiosophical details too much. Just ride with it, baby. Anyway, evidently, the producers decided that the additional feature of the relativistic clock was too distracting, and they pulled it from the final version. Here in this video, we get to see both versions of the film, which is a pretty tremendous experience.

J'hésite un peu à l'acheter, mais bon, c'est quand même un peu cher (et généralement acheter un article d'occasion chez Amazon.com quand on n'habite pas aux États-Unis signifie passer par plein d'étapes très compliquées pour finalement s'entendre dire que ce n'est pas possible de livrer là où on est).

↑Entry #1801 [older| permalink|newer] / ↑Entrée #1801 [précédente| permalien|suivante] ↑

↓Entry #1798 [older| permalink|newer] / ↓Entrée #1798 [précédente| permalien|suivante] ↓

(mercredi)

Jouons avec le problème de Galois inverse

Le fait inutile du jour : le polynôme

27·x27 + 21·x26 − 484·x25 − 3228·x24 − 16572·x23 − 68675·x22 − 222360·x21 − 572820·x20 − 1173331·x19 − 1869830·x18 − 2244571·x17 − 1849271·x16 − 462819·x15 + 1776007·x14 + 4159041·x13 + 5483063·x12 + 4692322·x11 + 1636621·x10 − 2304856·x9 − 4629308·x8 − 3932160·x7 − 1462752·x6 + 471744·x5 + 948800·x4 + 591872·x3 + 205056·x2 + 38912·x + 3072

a pour groupe de Galois sur ℚ le groupe de Weyl de E6, d'ordre 51840 qui peut être décrit comme SO6(𝔽2) ou SO5(𝔽3) ou encore comme groupe des automorphismes de l'unique groupe simple d'ordre 25920 (qui peut lui-même être décrit comme n'importe lequel de : SU4(𝔽2), SΩ6(𝔽2), PSp4(𝔽3) ou SΩ5(𝔽3).

[Figure géométrique]Ci-contre, la figure de ses racines dans le plan complexe (j'ai cadré pour qu'on les voie toutes sauf une, la racine réelle valant environ 7.2113675276, qui déborde). Le graphe que j'ai représenté en reliant chacune des racines à 10 autres, et qui est isomorphe au graphe du polytope 221 de Gosset, est préservé par le groupe de Galois : ce dernier est justement l'ensemble des permutations des racines qui laissent connectées les racines qui l'étaient.

Le polynôme est scindé modulo 1564741, 2506421, 2842537, 2848051, 3116447, 3331217, 3728393…

Les mathématiques expérimentales, c'est rigolo ! (Mais non, je n'ai pas pêché ce polynôme en essayant au hasard jusqu'à en trouver un qui marche.)

↑Entry #1798 [older| permalink|newer] / ↑Entrée #1798 [précédente| permalien|suivante] ↑

↓Entry #1791 [older| permalink|newer] / ↓Entrée #1791 [précédente| permalien|suivante] ↓

(vendredi)

Oubliez millard, billion, trillion… dites giga, téra, exa

En français, en principe, un milliard désigne le nombre 109 (ou 1e9 dans la version informatique de la notation scientifique), c'est-à-dire 1000000000 (un un suivi de neuf zéros), ou mille millions ; un billion désigne 1012 (=1e12), c'est-à-dire 1000000000000, soit mille milliards ou un million de millions ; et un trillion désigne 1018 (=1e18), c'est-à-dire 1000000000000000000, soit un million de billions, ou un milliard de milliards. Il n'y a pas de mots pour désigner 1015 (=1e15), on doit juste dire mille billions (quand j'étais petit, je m'imaginais fort logiquement que c'était un billiard, qui a même une entrée sur Wikipédia en français, mais ce n'est pas très sérieux). De façon générale, un N-llion, c'est 106N (par exemple, un septillion, c'est 1042), ou, si on préfère parler en puissances de 1000, c'est 10002N : on dit donc que le français utilise la convention 2N. Les puissances de mille sont appelées, dans cette convention : mille, million, millard, billion, « billiard », trillion, « trilliard », quadrillion, « quadrilliard », quintillion, etc.

En anglais, le mot million a le même sens que son homologue français ; le mot milliard n'existe pas trop ; et un billion désigne le nombre 109 (ou 1e9 dans la version informatique de la notation scientifique), c'est-à-dire 1000000000 (un un suivi de neuf zéros), ou mille millions, ce qu'en français on nomme un milliard. Un trillion désigne 1012 (=1e12), c'est-à-dire 1000000000000, ce qu'en français on nomme un billion ; et un quadrillion désigne 1015 (=1e15), c'est-à-dire 1000000000000000, ce qui se nomme en français mille billions, ou un billiard si on consent à utiliser ce mot. Quant à 1018 (=1e18), le trillion du français, en anglais il se dit quintillion. De façon générale, un N-llion, c'est 103(N+1) (par exemple, un septillion, c'est 1024, ce qu'en français on appellerait quadrillion), ou, si on préfère parler en puissances de 1000, c'est 1000N+1 : on dit donc que l'anglais utilise la convention N+1. Les puissances de mille sont appelées, dans cette convention : thousand, million, billion, trillion, quadrillion, quintillion, sextillion, etc.

Pour que ce soit bien clair :

Notation scientifique10310610910121015101810211024
Notation informatique1e31e61e91e121e151e181e211e24
Convention 2N
(français)
millemillionmilliardbillion« billiard »trillion« trilliard »quadrillion
Convention N+1
(anglais)
thousandmillionbilliontrillionquadrillionquintillionsextillionseptillion
Préfixe SIkilomégagigatérapétaexazettayotta
Périphrasemilleun millionun milliardmille milliards
ou : un million de millions
un million de milliardsun milliard de milliardsmille milliards de milliardsun million de milliards de milliards

La convention 2N (celle du français) a l'avantage pour elle d'être un peu plus simple pour les calculs : un trillion de trillions est un sextillion comme trois et trois font six, et un quadrillion de quadrillions est un octillion comme quatre et quatre font huit.

Bon, ça c'est la théorie.

La pratique, c'est que c'est le bordel complet. D'abord, parce que le français eut utilisé (au moins partiellement) la convention N+1 : le Littré définit le billion comme : Terme d'arithmétique. Dix fois cent millions ou mille millions, un milliard, qui [le mot milliard] est plus particulièrement usité dans le langage de la finance et dans le langage ordinaire. Et trillion : Terme d'arithmétique. Mille billions, ou mille fois mille billions. (le Club Contexte vous remercie, Monsieur Littré ! Dans la première définition, ou veut dire c'est-à-dire, et dans la seconde il veut dire ou bien au contraire.) Je pense qu'il y a toujours eu confusion.

A contrario, l'anglais eut utilisé la convention 2N : voici ce que dit l'OED à ce sujet :

billion (ˈbɪljən). [a. F. billion, purposely formed in 16th c. to denote the second power of a million (by substituting bi- pref. for the initial letters), trillion and quadrillion being similarly formed to denote its 3rd and 4th powers. The name appears not to have been adopted in Eng. before the end of the 17th c.: see quot. from Locke. Subsequently the application of the word was changed by French arithmeticians, figures being divided in numeration into groups of threes, instead of sixes, so that F. billion, trillion, denoted not the second and third powers of a million, but a thousand millions and a thousand thousand millions. In the 19th century, the U.S. adopted the French convention, but Britain retained the original and etymological use (to which France reverted in 1948).]

Je ne sais pas ce qui a pu être décidé en 1948[#0] ou comment ça a pu se passer, mais au moins ça a le mérite d'être clair : d'après l'OED, la convention initiale est la convention 2N, et elle date du 16e siècle en France, puis a été adoptée en anglais au 17e siècle ; au 19e siècle la France est passée à la convention N+1 (d'où les définitions données par Littré), les États-Unis ont suivi, puis au 20e siècle la France est revenue à la convention d'origine 2N. Bon, en fait, mon édition de l'OED date des années '80 (c'est la dernière édition imprimée), et j'imagine qu'une édition plus récente signalerait que les Britanniques se sont maintenant plus ou moins pliés à l'impérialisme américain (donc N+1) et que billion désigne maintenant vraiment un milliard même en Angleterre.

La confusion était déjà bien grande, et l'inventeur de la convention N+1 (un Français, donc, si j'en crois la petite remarque de l'OED) mériterait d'être pendu haut et court s'il n'était pas déjà mort depuis bien longtemps. Mais voilà qu'elle est rendue encore pire par l'apparition d'une engeance qui tente sans cesse de nous faire sombrer plus bas dans la nullité intellectuelle : les journalistes. Les journalistes français reçoivent quantité d'informations en anglais et, dès qu'ils voient un mot comme trillion figurer dans l'original, ils ne se font pas chier, ils traduisent par trillion. De toute façon, ils ne savent pas compter, pour eux un trillion est quelque chose de complètement incompréhensible, alors peu importe. On les entend donc régulièrement dire, par exemple, que la dette des États-Unis est de treize trillions de dollars (pour traduire thirteen trillion), sans se rendre compte de l'énormité du chiffre qu'ils avancent[#]. Non, elle est de treize billions de dollars, ça suffit largement, merci. Certains ajoutent l'insulte à l'injure, pardon, ajoutent l'injure à la blessure, en précisant qu'un trillion c'est un milliard de milliards, ce qui est juste, mais ce qui n'est pas l'usage qu'ils ont fait de ce mot. Tiens, je parie qu'il y a au moins un journaliste français qui a été assez con pour prétendre qu'on aurait calculé pi à cinq trillions de chiffres (remarquez que déjà l'article que je cite est en anglais censément britannique et utilise la convention N+1).

Bref, c'est vraiment la confusion la plus complète.

Bon, eh bien j'ai une proposition simple pour éviter toute cette confusion : ne plus utiliser ces mots.

Pour utiliser quoi à la place ? Une idée serait la notation scientifique. Mais celle-ci souffre de ses propres problèmes : d'abord, j'ai souligné que 1012 s'écrit aussi 1e12, donc il y aura toujours des journalistes assez cons pour dire que 1012 c'est 10 suivi de douze zéros ou a contrario pour lire 1e12 comme 1 puissance 12 ou ce genre de choses ; ensuite, il y a quelque chose dans la façon dont fonctionne la mémoire humaine qui fait qu'on retient plus facilement la mantisse que l'exposant, alors que c'est l'exposant qui est le plus important, et parfois il n'est pas si facile que ça de retrouver l'ordre de grandeur à un facteur 10 près ; enfin, c'est pénible à lire de toute façon, et c'est peu littéraire.

J'ai donc une autre solution : utilisons les préfixes SI comme si c'étaient des noms de nombres. On peut évidemment garder mille et le million, et éventuellement en français le milliard, mais à partir de là, disons sans hésiter : un téra (=1e12), un péta (=1e15), un exa (=1e18) et un zetta (=1e21) ; voire, un yotta (=1e24), mais cela prête à confusion pour d'autres raisons et de toute façon on a rarement besoin de nombres aussi grands. Le téra (=1e12, un million de millions) est de toute façon le plus important dans cette histoire : c'est lui qui se dit en principe billion en français et trillion en anglais, et qui cause le plus de confusion parmi les choses qu'on rencontre vaguement. La dette des États-Unis est de treize téra [de] dollars (ou treize téradollars), un japonais vient de calculer cinq téra [de] décimales de pi, le corps humain compte environ dix téra [de] cellules humaines (et cent téra [de] cellules dans la flore intestinale), et ça se dira exactement de la même façon en anglais. Et n'importe quelle personne ayant un minimum de culture scientifique comprendra tout de suite ce que cette convention signifie, sans avoir besoin d'être informée à l'avance (quant aux autres, ils ne savent de toute façon pas ce que billion veut dire, donc ils ne seront pas plus embêtés).

Faites circuler le mème ! (Après tout, la façon dont les journalistes utilisent ce genre de mots, c'est en reprenant ce que des scientifiques leur disent. Donc s'il faut inséminer l'idée, c'est bien chez des scientifiques — et j'imagine qu'il y en a qui lisent mon blog.)

(Il faudra que je traduise cette entrée en anglais, puisque c'est notamment par là que la confusion vient.)

[#0] Ajout : Il s'agirait apparemment de la 9e Conférence générale des Poids et Mesures, mais je ne trouve rien à ce sujet dans les résolutions de cette dernière. Par ailleurs, cette page en dit un peu plus sur la façon dont le bordel est apparu. Mais elle prétend que l'usage de la convention 2N est (re?)devenu légal en France par le décret 61-501 du 3 mai 1961 (relatif aux unites de mesure et au controle des instruments de mesure), et je ne vois rien dans ce décret qui mentionne le nommage des grands nombres pas plus que dans les résolutions de la 9e CGPM.

[#] Faut-il une preuve de ce que j'avance ? Chercher "trillions de dollars" sur Google : toutes les réponses (sauf celles qui dénoncent justement le phénomène dont je parle) sont forcément des erreurs, parce qu'un vrai trillions de dollars (1018, un milliard de milliards, un exa de dollars, quoi) c'est tellement gigantesque que ça ne peut rien désigner de correct.

↑Entry #1791 [older| permalink|newer] / ↑Entrée #1791 [précédente| permalien|suivante] ↑

↓Entry #1764 [older| permalink|newer] / ↓Entrée #1764 [précédente| permalien|suivante] ↓

(mercredi)

Pourquoi la physique utilise-t-elle des mathématiques ?

Puisque mes quelques dernières entrées étaient décidément dans le mode « métaphysique et science », j'en fais encore une :

La filosofia è scritta in questo grandissimo libro che continuamente ci sta aperto innanzi a gli occhi (io dico l'universo), ma non si può intendere se prima non s'impara a intender la lingua, e conoscer i caratteri, ne' quali è scritto. Egli è scritto in lingua matematica. (La philosophie [c'est-à-dire : la physique] est écrite dans ce grand livre qui est continûment ouvert à nos yeux (je veux dire l'univers), mais on ne peut le comprendre que si d'abord on apprend à en comprendre la langue et à reconnaître les caractères dans lesquels il est écrit. Il est écrit en langue mathématique.) — Galileo Galilei, Il Saggiatore (1623), chap. VI

Voilà encore une question qui me fascine : pourquoi la physique fait-elle appel aux mathématiques ? Et les questions que cela sous-entend : est-ce un fait profond sur notre Univers (comme Galilée le suggère dans le passage que je cite ci-dessus), ou est-ce simplement lié à la façon dont nous comprenons la physique ? Est-ce un fait fondamental de la physique ou simplement lié à l'utilité des mathématiques pour comprendre n'importe quel phénomène émergent ? Pourquoi les autres sciences[#] n'utilisent-elles pas autant les mathématiques, ou pas de la même façon ou (pour reprendre la description un peu élitiste et provocatrice que Hardy fait dans l'Apologie d'un mathématicien) pas des mathématiques élégantes ? Est-ce parce que ces sciences sont plus complexes que la physique, trop pour être mathématisées ? Parce que nous ne les comprenons pas assez bien ? (Dans la vision comtéenne, elles n'auraient pas encore atteint le stade positiviste.) Ou parce qu'intrinsèquement elles ne se plient pas autant à l'analyse mathématique ? Parce que ce ne sont pas des sciences exactes ?

Et voici une question apparentée, et pas forcément plus facile : pourquoi la physique n'utilise-t-elle qu'un petit sous-ensemble des mathématiques, et celui-ci admet-il une description plus simple que la partie des mathématiques à laquelle la physique fait appel ?

Par exemple, la physique fait — au moins apparemment — abondamment appel à la notion de nombre réel. Le monde qui nous entoure a l'air de dépendre lourdement de la notion de nombre réel. Même ma maman a une idée de ce qu'est un nombre réel : pour le non-mathématicien, c'est un nombre à virgule, qui pourrait s'écrire en théorie avec une précision aussi grande que voulue (et plus on ajoute de décimales, plus on est précis). Toutes les grandeurs qui nous entourent, les tailles des objets, les durées dans le temps, les vitesses, les masses, les grandeurs électriques, etc., semblent mesurées par des nombres réels.

Pourtant, mathématiquement, il existe plein d'autres sortes de nombres sur lesquels on aurait pu imaginer a priori que la physique reposât. Les nombres p-adiques semblent le candidat le plus évident : les nombres p-adiques partagent beaucoup de propriétés en commun avec les nombres réels, il y a de très importantes et élégantes symétries entre eux (les nombres réels prenant essentiellement la place des nombres ∞-adiques, et je n'utilise pas le mot place au hasard). Mais, pour autant que je sache, les nombres p-adiques n'ont aucune application en physique (malgré des tentatives pour leur en donner, qui ressemblent plus à une volonté de les rechercher à tout prix qu'à une théorie basée sur l'expérience). Non seulement cela, mais même dans des sciences basées très indirectement sur la physique, les nombres p-adiques ne jouent aucun rôle alors que les nombres réels sont omniprésents : la somme d'argent présente sur mon compte en banque est peut-être un rationnel (de dénominateur divisant 100), mais il faut clairement le considérer comme un nombre réel et non comme un p-adique quel que soit p (par exemple, si c'était un 7-adique, il serait presque pareil d'avoir 403536.07€ sur son compte que d'avoir 0€ ce qui, de toute évidence, n'est pas le cas). Bizarrement, même l'informatique semble avoir très peu besoin de nombres 2-adiques alors qu'elle est intrinsèquement binaire (et les calculs avec débordements dans les nombres en représentation binaire sont exactement des calculs approchés dans les entiers 2-adiques).

Je peux imaginer plusieurs raisons pour lesquelles les nombres p-adiques ne semblent pas exister dans la nature, dont au moins les suivantes :

  • C'est un fait de notre Univers : il ne faut pas chercher pourquoi, c'est juste comme ça.
  • C'est un fait de notre Univers : il s'explique par un argument anthropique (on peut imaginer des Univers basés sur les p-adiques, mais ils ne peuvent jamais soutenir une forme de vie ou de conscience).
  • C'est un fait mathématique : on ne peut pas construire de lois de la physique raisonnables (en un sens qu'il faudrait définir) sur les p-adiques.
  • C'est un fait lié à l'observation de notre Univers : il existe des phénomènes décrits par des nombres p-adiques, mais on ne peut pas les observer à notre échelle.
  • C'est un fait lié à notre situation dans l'Univers : il existe des phénomènes décrits par des nombres p-adiques, mais nous-mêmes « sommes » des phénomènes liés aux nombres réels, ce qui nous interdit de « voir » les phénomènes p-adiques.
  • C'est un fait mathématique : les phénomènes fondamentaux de l'Univers ne sont liés ni aux réels ni aux p-adiques (par exemple, l'Univers pourrait être un énorme automate cellulaire à états discrets), mais les nombres réels sont plus adaptés pour décrire les phénomènes émergents liés à des lois de la physique fondamentales inobservées.
  • C'est un fait lié à notre propre point de vue : les nombres p-adiques seraient tout autant adaptés que les nombres réels à décrire l'Univers, mais nous ne sommes pas habitués à ce point de vue, qui nécessiterait de tout revoir autrement.

J'avoue avoir énormément de mal à imaginer à quoi pourrait ressembler un univers où (disons) les 2-adiques joueraient un rôle important (et ce n'est pas faute de bien comprendre ce qu'est un nombre 2-adique, je pense). Il est donc aussi possible que la question soit aussi stupide que de demander pourquoi je ne vois jamais −42 moutons dans un pré, chose également difficile à imaginer. Mais je préfère prendre le risque de poser des questions stupides que celui de ne pas en poser d'intelligentes. ☺️

Les p-adiques ne sont qu'un exemple : pourquoi la physique n'utilise-t-elle jamais de nombres ordinaux ? (D'ailleurs, pour commencer, pourquoi les mathématiques en-dehors de la logique n'utilisent-elles quasiment jamais de nombres ordinaux ?) Utilise-t-elle E8 ou les tentatives de le voir apparaître sont-elles du wishful thinking ? Je ne sais pas si la physique gagne à se poser ce genre de questions, mais j'ai du mal à concevoir qu'on puisse ne pas se les poser.

[#] Enfin, ce n'est pas vrai : il y a une autre science qui utilise aussi lourdement les mathématiques, c'est l'informatique. Mais il y a quelque chose à dire sur le fait que si la physique est vraiment une branche à part car elle étudie le monde matériel, l'informatique, elle, est finalement une branche des mathématiques — celle que les mathématiciens sont trop snobs pour reconnaître comme telle. — Comme le disait éloquemment Dijkstra : Computer science is no more about computers than astronomy is about telescopes.

↑Entry #1764 [older| permalink|newer] / ↑Entrée #1764 [précédente| permalien|suivante] ↑

↓Entry #1763 [older| permalink|newer] / ↓Entrée #1763 [précédente| permalien|suivante] ↓

(samedi)

Les mathématiques pourraient-elles être différentes ?

C'est l'exemple type de la question (archi-rabachée) sur laquelle on ne fera pas de progrès, mais qui ne cesse de me fasciner : les mathématiques sont-elles découvertes ou inventées ? — ou, pour poser la question différemment (ou peut-être pour poser une autre question plus provocatrice), les mathématiques pourraiennt-elles être différentes ?

On peut certainement imaginer un Univers parallèle où les lois de la physique sont différentes, d'ailleurs des physiciens apparemment tout à fait sérieux le font (les deux articles vers lesquels je viens de faire un lien ont été vulgarisés récemment de façon assez intéressante, c'est pour ça que je les ai choisis, mais les exemples ne manquent pas). Mais les mathématiques ? Peut-on imaginer un Univers où, sans aller jusqu'à demander que 2+2=5, les mathématiques seraient subtilement différentes ? Si on peut l'imaginer, peut-on imaginer communiquer avec un tel Univers ? (De toute évidence, il y a un problème : si on fait une démonstration dans notre Univers d'un fait mathématique vrai dans le nôtre et qu'on l'envoie dans l'autre, faut-il croire que la réalité cesse d'exister ?) Pourrait-on imaginer que, demain, les mathématiques soient différentes ? (Bon, ça, ça m'arrive souvent : je me couche le soir en ayant démontré un théorème, et le matin il n'est plus vrai.)

Avec des lunettes de logicien formaliste, on pourrait dire : oui, des univers mathématiques alternatifs existent, ça s'appelle des modèles de la théorie mathématique en train d'être considérée, et tout ce qui n'est ni démontrable ni réfutable, en fait, est vrai dans certains univers et faux dans d'autres. Mais c'est une fausse réponse : ce n'est pas de ça qu'on veut parler — quand on pense aux mathématiques, on ne pense pas aux conséquences d'un système d'axiome, et notamment quand on pense aux entiers naturels on ne pense pas à toute implémentation des axiomes de Peano mais bien à quelque chose de plus précis que ça, que les axiomes de Peano ne capturent qu'imparfaitement. La question de savoir dans quelle mesure ces entiers naturels sont intriqués dans la structure de l'Univers physique est d'ailleurs très subtile et très délicate, et liée à la question de l'existence de l'infini (un commentateur de ce blog qui se reconnaîtra, pense par exemple qu'ils ne le sont pas puisque l'entier naturel Ackermann(100,100) n'a apparemment pas d'existence physique) ; par exemple, on peut imaginer des Univers basés sur des lois de physique semblables à celles que nous croyons être les nôtres, mais construites à partir d'un modèle non-standard de l'arithmétique de Peano et se demander dans quelle mesure des êtres vivants dans un tel Univers pourraient se rendre compte qu'ils ne sont pas dans le modèle standard [pas au sens de la physique] — c'est une question provocatrice et difficile. Mais ce n'est pas vraiment de ça que je veux parler : même les modèles des logiciens ont l'air de vivre dans une sorte de grand tout des mathématiques (et, de fait, la théorie des modèles est une branche des mathématiques, qui s'étudie avec des méthodes et outils des mathématiques et qui utilise donc un Univers mathématique ambiant, qui peut lui-même être un modèle d'une autre théorie puisqu'il est habituel de regarder des modèles dans des modèles).

La pratique mathématique donne indubitablement l'impression qu'on ne contrôle pas complètement ce qu'on fait : on peut être surpris par ce qu'on trouve, émerveillé ou parfois frustré, l'impression produite n'est pas du tout celle d'un architecte, d'un maçon ou d'un artiste qui construit quelque chose, mais d'un aventurier qui explore un terrain complexe, un labyrinthe, un palais. Je crois que la grande majorité des mathématiciens penchent plutôt pour la solution les mathématiques sont découvertes, même si tous n'adoptent pas un point de vue religieusement platoniste, loin de là. Le fait que les mathématiciens le pensent ne signifie cependant pas qu'ils aient raison : ils pourraient souffrir d'une <anglicisme>délusion</anglicisme> collective (soit qu'elle soit la conséquence de leur travail soit qu'elle soit un prérequis pour devenir mathématicien). La question pourrait aussi très bien ne pas avoir de sens, ou ne pas être tout d'un côté ou de l'autre : s'il est impossible de dire que j'ai inventé les formes dans les vidéos de l'ensemble de Mandelbrot que j'ai faites, il est néanmoins vrai que je les contrôlais un peu, puisque je savais comment choisir le point de zoom pour obtenir tel ou tel type de forme. (Et à l'extrême, quand un romancien écrit un roman, il ne fait que choisir un point dans l'ensemble de toutes les suites finies de caractères Unicode, il serait absurde de dire que le roman est découvert pour autant.)

Parfois les mathématiques donnent l'impression de contenir des « coïncidences » ; il y a d'ailleurs des objets mathématiques dont l'existence même a l'air de reposer sur des coïncidences — soit des coïncidences qui donnent l'impression d'être complètement fortuites et locales, soit une sorte de connexion bizarre entre des objets mathématiques éloignés qui, si j'étais religieux, me semblerait être la marque directe de la main de Dieu. Est-ce une impression complètement naïve qui ne fait que montrer l'insuffisance de notre recul ? Sans doute.

On peut néanmoins se livrer à des expériences de pensée. La première imagine une civilisation complètement indépendante de la nôtre, par exemple extra-terrestre (mais pas forcément : cela pourrait être la civilisation qui apparaîtra sur Terre quand les poulpes seront devenus l'espèce intelligente une fois que nous nous serons détruits 😉). J'écris le nombre 808017424794512875886459904961710757005754368000000000 (en binaire, disons, codé avec des bosses et des creux sur un objet physique, pour rendre la chose aussi abstraite que possible), ou bien j'écris un autre nombre, à peu près de la même taille et vaguement aussi divisible, mais sans propriété remarquable (comme 808192228876161778520554895458069891466133504000000000). Je dépose cet objet où la civilisation va le trouver, et j'observe ce qui se passe. Bien sûr, très souvent il ne se passe rien (la civilisation n'est pas intéressée par les mathématiques, ou ne décode pas l'objet). Le test de pensée que je fais est : y a-t-il avec le « bon » nombre une réaction plus souvent différente qu'avec n'importe que autre nombre comparable mais non remarquable ? Si oui, je peux considérer que notre civilisation a découvert un phénomène préexistant. Alors que si tous les nombres provoquent des réactions comparables auprès des civilisations extra-terrestres, c'était purement une invention liée, d'une façon inexplicable, à notre cerveau ou à notre culture, ou à je ne sais quoi.

Deuxième expérience de pensée. Je rencontre Dieu, qui se prétend omnipotent. Je lui demande une preuve de son omnipotence (je sais, je sais, οὐκ ἐκπειράσεις κύριον τὸν θεόν σου) : je lance un défi qui me semble un peu plus intéressant à relever que de me donner des bijoux en caoutchouc qui brillent comme des vrais ou un rameau de macaronis en fleurs : je veux qu'il fasse en sorte que si je calcule les 1000000000 premières décimales de la racine carrée de 7 en binaire (je choisis ça parce que probablement personne ne s'est amusé à faire ce calcul pour l'instant) et que je les interprète comme une chaîne d'octets (encodés en UTF-8), alors quelque part dedans doit apparaître la phrase eh oui, j'ai relevé ton défi, David, et comme tu peux le voir, je contrôle même les mathématiques, et tu ne peux pas mettre en doute mon omnipotence (bien sûr, cette chaîne était de tous temps présents dans les décimales de ce nombre, mais personne ne l'a remarqué) (cette chaîne de caractères apparaît « très probablement » quelque part dans les décimales de la racine carrée de 7, mais il serait plus qu'un peu surprenant qu'elle apparût dans les un milliard premières). Dieu peut-Il faire cela ? (Sans tricher, bien sûr, c'est-à-dire sans faire faire juste l'erreur de calcul qui va bien à mes ordinateurs pour me le faire croire.) Ou est-Il limité à l'Univers physique ? Ou qui, des mathématiques et de Dieu, est le plus fort ? Je suis tenté de faire le calcul pour voir s'il a relevé mon défi. ☺️ (Je lui accorde des points de style si à la place de la phrase demandée il a écrit le verset 7 du chapitre 4 de l'évangile de Matthieu.)

On peut pousser plus loin cette dernière expérience de pensée et imaginer un certain calcul qui, quand on le fait, permet d'écrire dans les décimales de pi (à condition que personne ne les ait jamais calculées ou écrites jusqu'à présent). Et qu'ensuite les gens s'amusent à vandaliser les décimales de pi, à y mettre des photos pornos. Mais je garde ça pour un fragment littéraire gratuit ultérieur.

↑Entry #1763 [older| permalink|newer] / ↑Entrée #1763 [précédente| permalien|suivante] ↑

↓Entry #1758 [older| permalink|newer] / ↓Entrée #1758 [précédente| permalien|suivante] ↓

(jeudi)

Fragment littéraire gratuit #131 (divagations métaphysiques redux)

— Je vais essayer, mais ce n'est pas facile ! Alors, postulat numéro 1 : il existe une identité de notre conscience, qui est hors de ce monde.

— Ça part mal, pour ce qui est de l'approche scientifique… Le fantasme du dualisme, depuis qu'il a quitté la glande pinéale, on l'a cherché un peu partout, sans jamais le trouver : aucune expérience ne laisse ne serait-ce qu'un commencement de début de raison de penser que la conscience se loge ailleurs que dans le cerveau, ou que les lois de la physique seraient différentes dans celui-ci, ou qu'il existe une « âme », un ghost in the machine.

— D'accord : je me suis mal exprimé. Il n'y a pas de fantôme qui contrôle nos actions à travers notre cerveau, nous sommes d'accord sur ce point. La thèse serait plutôt qu'il existe un fantôme qui les observe, et qui observe ce monde à travers notre cerveau. Une sorte d'aperception transcendentale qui relie notre expérience du monde.

— Alors ce n'est pas un concept détectable ni vérifiable, et c'est à peine s'il est définissable. Quel intérêt ?

— L'intérêt est de se demander pourquoi le monde est tel qu'il est. Le fantôme n'est pas plus détectable que ne l'est le lecteur d'un roman pour un personnage de ce dernier, mais il peut pourtant faire des déductions à son sujet : si je suis un héros de roman policier, je peux en déduire que mon lecteur aime probablement les romans policiers.

— On doit pouvoir en déduire que ton lecteur aime la philosophie absconse ! Ou plutôt, vu l'éclectisme de notre Univers, qu'il aime tout et n'importe quoi, et si possible changer de sujet sans raison, peut-être des fragments de littérature gratuite et inutile. Toujours est-il que ce fantôme devrait passer au filtre du rasoir d'Ockham : si nous ressentons une unité de la conscience, c'est parce que le cerveau a besoin de cette illusion pour maintenir l'intégrité de nos perceptions ou la cohérence de nos raisonnements, ou, probablement, l'intériorisation de la préservation de soi, toutes choses favorisées par l'évolution naturelle d'un être intelligent. Pas besoin de faire intervenir le surnaturel.

— C'est une raison interne parfaitement valable, et nul doute qu'elle soit vraie : le cerveau a été sélectionné pour être conscient, parce que c'est soit une conséquence inévitable de la représentation de soi, soit le moyen le plus simple pour l'atteindre. Mais le fantôme que j'évoque ne pouvait percevoir le monde à travers lui qu'à cette condition. Son rôle n'est pas d'expliquer pourquoi nous sommes conscients mais de qui nous sommes conscients : pourquoi le monde est tel qu'il est, pourquoi je suis qui je suis. De tous les mondes possibles, mathématiquement possibles, réalisant les contraintes des lois de la physique ou peut-être d'autres lois de la physique, pourquoi est-ce celui-ci que nous appelons « réalité » ? Et pourquoi ici et maintenant ce que nous appelons le présent ?

— Je ne suis pas sûr que la question ait un sens. Dans la mesure où elle en a un, je ne suis pas sûr qu'elle mérite une réponse, et même si elle en mérite une, je ne suis pas sûr qu'elle en admette une. Et même si elle a un sens, mérite et admet une réponse, je ne suis pas sûr que la réponse aille plus profondément ou soit plus intéressante que : L'Univers est tel qu'il est, et c'est tout. À la rigueur, qu'on explique que l'Univers ne pourrait pas être trop différent, sans quoi nous ne serions pas là pour nous poser la question…

— Voilà ! c'est exactement ça. Il fallait un Univers où existassent des êtres conscients pour que nos fantômes pussent l'observer.

— Tu pousses plus loin que je ne l'ai dit, et quand bien même j'admets tout cela, tu n'expliques toujours rien. Ces fantômes eux-mêmes, on va se demander où ils vivent, ou pourquoi ou comment ils sont conscients…

— C'est justement là que j'en viens à un nouveau postulat. Le monde que nous observons, que j'ai comparé à un livre, mais qui devrait plutôt être comparé à un programme dans un ordinateur (un programme sans interaction, qui suit ses propres règles et où le programmeur ne fait qu'observer ce qui se passe), est une expérience menée depuis un monde supérieur. Nous sommes le résultat de cette expérience : notre existence ou notre conscience en est le but, et nous sommes à la fois les fantômes qui observons ce monde, et ce qu'ils en voient.

— Ouhlà ! Très beau délire pour donner la réponse à la vie, l'Univers, et tout le reste.

— Bon, tu me demandes de t'expliquer, je le fais. Tu veux que je continue, oui ou non ?

— Pardon. C'est amusant, continue. Mais j'ai du mal à imaginer qu'on puisse croire à quelque chose comme ça. L'Univers, une sorte expérience de pensée de fantômes qui vivent dans un monde supérieur ? Et puis quoi encore ? Je suppose qu'ils ont contrôlé l'apparition de l'homme par intelligent design ?

— Excuse-moi si je vais paraître direct, mais il y a des millions de gens qui croient que le créateur de l'Univers se fait manger par ses adorateurs sous l'apparence de petits morceaux de pain circulaire : à moins de penser que c'est leur nombre qui leur donne raison, je ne vois pas ce qu'il y a de ridicule à croire quelque chose de métaphysiquement plus vague, et qui ne postule même pas l'existence d'un Dieu. Non, les fantômes dont je parle ne contrôlent pas l'évolution : ce serait plutôt qu'ils auraient cherché, à travers les milliards de planètes dans un nombre incalculable d'Univers possibles, celles qui leur convenaient. Il y a tout de même peut-être quelque chose dans l'idée que Dieu a créé l'homme à Son image, mais ce serait plutôt qu'il a choisi ou préféré celui qui Lui ressemble.

— Tu as prononcé le mot D***, je devrais dire que tu as perdu. Mais continue : tout ceci ne fait-il que remonter le problème d'un niveau ? D'ailleurs, si nous sommes ces fantômes-dieux, pourquoi ne sommes-nous pas parfaitement conscients de tout cela ?

— Le problème est bien le même un niveau plus haut, c'est là toute la beauté de la chose. Nous vivons dans le monde de niveau zéro. Les fantômes qui nous observent et que nous sommes vivent dans le monde de niveau un. Mais ce monde-là lui-même est observé par des fantômes qui vivent dans le monde de niveau deux… et ainsi de suite.

— …Ou comment repousser à l'infini la solution des problèmes qu'on ne sait pas résoudre. À quoi ils ressemblent, ces mondes empilés ?

— Ils ne ressemblent à rien. Ou plutôt, le monde de niveau un nous est déjà incompréhensible parce qu'il est formé, justement, d'une sorte de collection ou un enchaînement de mondes de niveau zéro. Toute notre existence ici n'est qu'un minuscule fragment de notre existence dans le monde d'au-dessus, qui lui-même, et ainsi de suite. Voilà pourquoi nous ne sommes pas directement conscients des mondes d'au-dessus.

— Je suis scié par un tel niveau de mysticisme… Je suppose que tout ceci implique une croyance en une forme de métempsycose ?

— C'est une description possible, mais il faudrait plutôt la présenter comme une infinités de réveils successifs vers des mondes de plus en plus riches et complexes.

— Des paradis emboîtés, donc ? Avec des jardins délicieux, du nectar et de l'ambroisie ?

— De nouveau, tu te moques. Mais tu sais très bien que ce n'est pas ça l'idée. Le niveau zéro est caractérisé par la thèse de Church-Turing.

— La… thèse de Church-Turing ? Tu as trouvé une interprétation religieuse de la thèse de Church-Turing ? Dis-moi que c'est une blague !

— La thèse de Church-Turing exprime l'idée (qu'il est peut-être abusif d'attribuer sous cette forme à Alonzo Church ou Alan Turing) que tout calcul qui, dans cet Univers, peut être mené par un moyen physique quelconque, est réalisable par une certaine abstraction mathématique des calculs mécaniques et codifiée sous la forme de machine de Turing, ou de lambda-calcul.

— Je sais bien. Quel rapport avec la philosophie mystique dont on parlait ?

— Tout : la première chose qu'une machine de Turing (donc un ordinateur de cet Univers physique) ne peut pas faire, c'est décider du résultat du calcul d'une autre machine de Turing si celui-ci est continué jusqu'à l'infini, à commencer par savoir si le calcul va terminer, ce qu'on appelle le problème de l'arrêt. Le niveau un des Univers transcende le niveau zéro exactement de la façon dont le problème de l'arrêt transcende les machines de Turing : dans le monde de niveau un, il est possible de connaître le résultat d'un calcul infini dans un Univers tel que celui-ci, et cela est possible justement en le menant puisque le monde de niveau un est formé de mondes de niveau zéro. L'opération mathématique en question porte le nom de saut de Turing.

— J'ai peur de deviner ce qui va suivre… L'infinité de mondes dont tu parlais ne s'arrête pas là ?

— Précisément. Le saut de Turing peut être transfiniment itéré. Le monde de niveau omega serait constitué de l'emboîtement de tous les mondes de niveau 0, 1, 2, 3 et ainsi de suite, et il est, techniquement, le monde dans lequel tout problème arithmétique devient immédiatement décidable, mais la chaîne ne s'arrête pas là. Il existe un monde omega plus un, puis omega plus deux, et après tout ceux-là un omega fois deux, puis omega fois deux plus un… Après omega, omega fois deux, omega fois trois et ainsi de suite vient omega carré…

— Quousque tandem ?

— Au moins jusqu'à l'ordinal de Church-Kleene, qui permet de décider toutes les questions hyperarithmétiques ; il s'agirait du niveau à partir duquel on ne peut même plus décrire ou représenter la numérotation des niveaux dans cet Univers-ci. Mais il n'y a pas de raison de ne pas aller beaucoup plus loin : mathématiquement, on sait que le saut de Turing se prolonge au moins jusqu'à l'ordinal de l'analyse ramifiée, qui représente le monde auquel on obtient une réponse à toute question d'analyse classique, et même jusqu'au plus petit ordinal indénombrable dans l'univers constructible. Bon, j'ai gagné mon pari, à ce stade-là ?

— Haut la main.

Finalement, j'aime bien ce mode d'écriture sous forme de dialogues pour présenter des idées que je trouve amusantes ou provocatrices (dialogue sur deux systèmes du monde ? à vous de décider qui est Simplicio et qui est Salviati). Et puis j'aime bien l'eschatologie. Quant au pari dont il est question, on peut imaginer qu'il s'agisse d'inventer une religion qui repose sur la science au lieu d'avoir du mal à ne pas la contredire (même si sur ce sujet-là j'en ai déjà dit assez).

Si vous voulez en savoir plus sur la partie mathématique (qui n'a rien de pipo, elle), j'avais gribouillé quelques explications sur la calculabilité supérieure il y a quelque temps, et il y a l'excellent livre de Peter Hinman, Recursion-Theoretic Hierarchies (mais pour la description précise de l'itération transfinie maximale du saut de Turing, il faut aller voir dans des articles de recherche assez pointus, notamment les suivants : George Boolos & Hilary Putnam, “Degrees of Unsolvability of Constructible Sets of Integers”, J. Symbolic Logic 33 (1968), 497–513 ; Carl G. Josckusch Jr. & Stephen G. Simpson, “A Degree-Theoretic Definition of the Ramified Analytical Hierarchy”, Ann. Math. Logic 10 (1975), 1–32 ; Harold T. Hodes, “Jumping through the Transfinite: the Master Code Hierarchy of Turing Degrees”, J. Symbolic Logic 45 (1980), 204–220).

↑Entry #1758 [older| permalink|newer] / ↑Entrée #1758 [précédente| permalien|suivante] ↑

↓Entry #1740 [older| permalink|newer] / ↓Entrée #1740 [précédente| permalien|suivante] ↓

(mercredi)

Le groupe de Galois de x^5−5x+12

J'ai beau connaître la théorie de Galois depuis longtemps, et l'avoir enseignée pendant trois ans à l'ENS, je continue à trouver ça assez magique. Il y a quelque chose de vraiment difficile à admettre, intuitivement, dans le fait que certaines équations ont des groupes de Galois plus petits que ce qu'ils pourraient être : et même quand on connaît bien la théorie, et qu'on sait calculer les groupes de Galois en pratique, on a l'impression de ne pas vraiment avoir de réponse satisfaisante à la question, mais pourquoi, au juste, cette équation décide-t-elle de ne pas faire comme ses petites copines ? Ça n'arrange pas, en plus, que la majorité des cours sur la théorie de Galois vous laissent sans la moindre idée de comment au juste on peut calculer un groupe de Galois en pratique (surtout quand il est petit).

Je peux essayer d'illustrer ça par l'exemple du polynôme f=x5−5x+12 (sur ℚ), que j'ai commencé à utiliser comme exemple dans un texte que j'écrivais aujourd'hui : qu'est-ce qui peut bien faire que ses racines décident de ne pas être rigoureusement interchangeables comme dans les équations les plus générales (car c'est ça que signifie le fait d'avoir un groupe de Galois maximal — tout le groupe symétrique sur les racines) ?

On peut en avoir une mesure expérimentale en regardant ce qui se passe si on réduit ce polynôme modulo différents nombres premiers p (c'est-à-dire qu'on le regarde comme un polynôme à coefficients dans 𝔽p=ℤ/pℤ) et qu'on cherche à le factoriser : modulo les 10000 premiers nombres premiers, si on excepte p=2 (le polynôme f devient x·(x+1)4) et p=5 (le polynôme devient (x+2)5) où le polynôme a des facteurs multiples (ce sont les nombres premiers dits ramifiés), il y a 4016 nombres premiers (7, 11, 13, 19, 23, 37…) modulo lesquels f est irréductible, il y en a 5021 (3, 17, 29, 31, 43, 61…) tels qu'il se factorise en un facteur linéaire et deux de degré 2 (par exemple, modulo 3, le polynôme f devient x·(x²+x+2)·(x²+2x+2)), et enfin il y a 961 nombres premiers (127, 157, 197, 223, 251, 331…) tels que le polynôme f se décompose totalement en cinq facteurs linéaires (par exemple, modulo 127, le polynôme f devient (x+19)·(x+65)·(x+81)·(x+93)·(x+123)). Autrement dit, il y a en gros 40%, 50% et 10% des nombres premiers modulo lesquels on a des degrés de factorisations 5, 1+2+2 et 1+1+1+1+1 respectivement ; et aucune autre factorisation ne se produit (ou en tout cas, ne semble se produire si on regarde les petits nombres premiers). On est censé trouver ça surprenant, parce qu'un polynôme général de degré 5 (qui a pour groupe de Galois le groupe symétrique 𝔖5, par exemple x5−5x+11), il a ces trois factorisations modulo p dans des proportions respectives 20%, 12.5% et ~0.83%, mais surtout, il y a d'autres factorisations possibles (1+4 modulo 25% des nombres premiers, 2+3 et 1+1+3 modulo ~16.7% chacun, et 1+1+1+2 dans ~8.3% des cas) : donc la conséquence, ou la détection, expérimentale du petit groupe de Galois de f=x5−5x+12, c'est que sa réduction modulo p n'admet pas toutes les factorisations possibles, ou avec des proportions inattendues pour celles qui sont possibles (et notamment, le scindage complet en facteurs linéaires a lieu beaucoup plus souvent que dans le cas général). Le théorème qui sous-tend ces statistiques, c'est le théorème de Čebotarëv (qui implique que les proportions des différentes partitions en degrés pour la factorisation des réductions d'un polynôme sur ℚ sont, asymptotiquement, les proportions des différentes répartitions en cycles des éléments de son groupe de Galois agissant sur les racines) ; les proportions observées pour f doivent faire soupçonner comme groupe de Galois le groupe diédral du pentagone, car on cherche un groupe de permutations sur 5 éléments, dont le nombre d'éléments est multiple de 5 et approximativement 10 (l'inverse de la proportion de p modulo lesquels f se décompose complètement), et qui a 40% d'éléments sans points fixes et 50% d'éléments avec exactement un point fixe. Mais tout ceci ne permet de conclure qu'heuristiquement.

Comme f=x5−5x+12 est irréductible (par exemple parce qu'il l'est modulo 7), ses racines sont au moins une fois interchangeables (=le groupe de Galois opère transitivement dessus) : on peut appeler a une de ses racines. On peut se fixer les idées en disant que a est l'unique racine réelle (celle qui vaut environ −1.842085). Le polynôme s'écrit alors f=(xa)·(x4+ax3+a2x2+a3x+a4−5). Là où ce polynôme est surprenant, c'est que le second facteur se décompose plus loin : f=(xa(x2+¼(−a4a3a2+3a+4)x+¼(−a4a3a2−5a+8))·(x2+¼(a4+a3+a2+a−4)x+½(−a3a−2)). On peut vérifier cette expression en développant patiemment le produit (et en utilisant de façon répétée le fait que f(a)=0) ; la présence des deux facteurs quadratiques garantit d'ores et déjà que f vu modulo n'importe quel nombre premier p ne pourra jamais avoir de factorisation dont les facteurs auraient degrés 1+1+3, par exemple (i.e., deux facteurs linéaires et un de degré 3), et l'existence de nombres premiers modulo lesquels ces facteurs quadratiques sont irréductibles (3, par exemple) garantit qu'ils sont irréductibles tout court. Mais ce n'est pas tout : on voit qu'une fois qu'on a choisi (identifié, nommé, distingué) une racine a de f, les quatre autres ne sont plus interchangeables — elles viennent en deux paires différenciées, à savoir, celles qui sont racines de x2+¼(−a4a3a2+3a+4)x+¼(−a4a3a2−5a+8), et celles qui sont racines de x2+¼(a4+a3+a2+a−4)x+½(−a3a−2). Par exemple, dans les complexes, si j'ai choisi a≈−1.842085, alors les racines du premier facteur quadratique seront environ 1.272897+0.719799i et 1.272897−0.719799i, que je voudrai noter respectivement a1 et a4 pour des raisons qui apparaîtront plus tard, et les racines du second facteur quadratique seront environ −0.351854+1.709561i et −0.351854−1.709561i, que je voudrai noter a2 et a3 respectivement.

Cette fois, on a bien prouvé les choses suivantes sur le groupe de Galois : qu'il opère transitivement sur les 5 racines, et que le sous-groupe fixant une racine a quelconque opère sur les 4 autres en les séparant en deux orbites de 2 éléments chacune. On peut se convaincre en examinant toutes sortes de cas que le groupe diédral du pentagone est le seul groupe de permutations sur cinq objets qui réponde à ces conditions. On peut cependant être plus explicite : si je note a0=a une des cinq racines de f, et a1 une des deux racines de x2+¼(−a4a3a2+3a+4)x+¼(−a4a3a2−5a+8), alors en posant a2=[(a04a03+a02a0−4)a1+(−a04+a03a02+a0−4)]/8, on peut vérifier (de nouveau, en développant de façon fastidieuse, si on ne trouve pas mieux) que a2 est aussi racine de f (et plus précisément de x2+¼(a4+a3+a2+a−4)x+½(−a3a−2)) ; et il en va de même de a3=[(−a04+a03a02+a0+4)a1+(−a04−3a03a02−3a0+12)]/8 ; enfin, la dernière racine de f est alors a4=−a1+¼(a04+a03+a02−3a0−4) d'après l'équation vérifiée par a1 : ceci montre qu'une fois choisies les racines a0 (une parmi cinq) et a1 (une parmi deux), toutes les autres sont complètement déterminées, donc le groupe de Galois a dix éléments. Et avec ces conventions, si on permute cycliquement les cinq racines a0, a1, a2, a3, a4, alors les mêmes relations sont satisfaite, de même que si, à a0 fixé, on échange a1 et a4 et a2 et a3 : on s'est donc convaincu que le groupe de Galois de f est le groupe diédral du pentagone, le pentagone en question étant celui formé par les aj (abstraitement, car dans ℂ ce n'est pas du tout un pentagone régulier).

Pour résumer, donc, si j'essaie de jouer à bouger les racines de f entre elles, je peux en mettre une première n'importe où, je peux mettre une seconde parmi deux possibles (si les racines étaient adjacentes sur le pentagone abstrait a0, a1, a2, a3, a4, elles doivent le rester), et une fois que c'est fait toutes les autres tombent forcément en place.

Parmi les petites identités remarquables qui tombent de la théorie de Galois, on peut remarquer que comme le groupe diédral du pentagone a un sous-groupe d'indice 2 (les rotations du pentagone), il doit y avoir une extension quadratique de ℚ dans celle engendrée par les racines de f : c'est bien le cas, et spécifiquement, √−10=(a02+1)a1+¼(−a04+a03a02+5a0+8) est laissé invariant par les éléments du groupe de Galois qui permutent cycliquement les cinq racines, et envoyé en son opposé par les éléments qui laissent fixe une racine. Du coup, si on veut, toutes les racines de f peuvent s'exprimer à partir d'une seule racine a et de cette quantité √−10. On pourrait aussi jouer à exprimer explicitement toutes les racines de f avec des radicaux : on va dire que je laisse ça en exercice au lecteur, qui pourra s'inspirer de l'exercice 5 de cette feuille d'exercices. [Ajout : voir cette entrée ultérieure où la réponse est donnée.]

PS () : D'autres exemples de groupes de Galois un peu rigolos qui ne soient ni trop petits ni trop gros pour être intéressants : x7−7x+3 (groupe simple à 168 éléments, PGL2(𝔽7)≅PGL3(𝔽2)) ; x8−16x+28 (groupe d'ordre 1344 extension du précédent par (ℤ/2ℤ)3) ; x8−5x−5 (groupe d'ordre 1152 extension de (ℤ/2ℤ) par le produit de deux copies de 𝔖4) ; x9−9x2+9x−10 (groupe d'ordre 1296 produit en couronne de 3 copies de 𝔖3 avec action de 𝔖3, i.e., stabilisateur d'un système de 3 blocs de 3).

↑Entry #1740 [older| permalink|newer] / ↑Entrée #1740 [précédente| permalien|suivante] ↑

↓Entry #1711 [older| permalink|newer] / ↓Entrée #1711 [précédente| permalien|suivante] ↓

(lundi)

Logicomix, et faut-il 350 pages pour prouver 1+1=2 ?

[Couverture de Logicomix]Bertrand Russell est un de mes héros (ou quelque chose de ce genre), et comme je m'intéresse à l'épistémologie des mathématiques, il était logique que je sois au moins intrigué par l'idée d'une bande dessinée dont le principal personnage est Bertrand Russell et dont le thème est la quête des fondements des mathématiques. C'est peut-être surprenant, mais cette BD existe (pour l'instant seulement en anglais) : elle s'appelle Logicomix ; ses auteurs sont grecs, et l'un d'entre eux, Christos Papadimitriou (enfin, Χριστος Παπαδημητριου), chercheur en informatique à Berkeley, m'était d'ailleurs connu comme auteur de plusieurs bons ouvrages sur la théorie de la complexité algorithmique : j'ai été assez étonné de le savoir co-scénariste de cette BD. (Un autre des auteurs est connu pour un roman intitulé Oncle Petros et la Conjecture de Goldbach dans sa traduction française.) Bref, comme un effet Zahir assez peu surprenant fait que j'ai entendu parler plusieurs fois de Logicomix (et en bien !) ces dernières semaines je l'ai achetée.

On aura compris que je la recommande, mais il faut que je précise bien quelque chose : ce n'est pas une BD sur les mathématiques, et ce n'est donc pas en tant que mathématicien que je la recommande (d'ailleurs, en tant que mathématicien, j'ai plutôt quelques reproches à lui faire). Je la recommanderais aussi bien à ma maman, si ma maman aimait les BD. Il ne s'agit pas d'un livre portant sur la logique, donc, mais sur l'histoire de la logique ou plutôt, un chapitre particulier de l'histoire de la logique qui est celui des fondements des mathématiques. Les héros s'appellent Russell, Whitehead, Frege, Cantor, Hilbert, Poincaré, Wittgenstein et Gödel : même si de très brefs passages sont employés à expliquer sommairement une ou deux idées essentielles de logique, de mathématiques ou — comme on se l'imagine avec l'apparition de Wittgenstein dans la liste des personnages — de philosophie, ce n'est pas du tout le propos. Le propos est plutôt d'expliquer ce qui a motivé cette quête des fondements, comment Russell l'a personnellement vécue, et en particulier comment les Principia Mathematica ont été écrits (et le Tractatus Logico-Philosophicus). La dimension humaine est essentielle, par exemple sur l'opposition de Russell à la guerre ou la perte de sa foi, mais surtout dans deux idées : la logique comme façon d'éviter la folie, et la quête des fondements sous forme de mythe de Sisyphe avec quoi les protagonistes n'en ont jamais fini. Ce n'est pas non plus une biographie de Russell (ne serait-ce que parce que ça s'arrête, comme ça commence, en fait, en 1939, donc son engagement contre la guerre du Vietnam n'est pas mentionné). Et évidemment, on pourrait redire des choses sur l'exactitude historique (notamment de la thèse sur la folie des logiciens : par exemple, le fait que Cantor soit devenu fou est d'une part un peu exagéré, et d'autre part sans rapport avec son activité mathématique). Mais assez parlé de la BD elle-même.

[Page des *Principia Mathematica*] Les Principia Mathematica sont peut-être bien le livre le plus abscons[#] de l'univers. Non seulement c'est de la logique formelle aride au possible, mais en plus les notations ne sont plus du tout celles qu'on utilise de nos jours (par exemple, l'ensemble vide — enfin, la classe vide — est noté par un lambda majuscule, un système de points est utilisé là où nous mettrions des parenthèses [ajout : voir cette entrée ultérieure], le et logique est aussi noté par un point et l'implication par un symbole ⊃), sans même compter les notations spécifiques de l'ouvrage, les abréviations qu'ils utilisent pour « alléger » les démonstrations ou la numérotation un peu déroutante ; et, évidemment, les fondements des mathématiques ont évolué depuis cette première tentative. Que le lecteur non-mathématicien s'imagine donc que la page reproduite ci-contre (et on pourrait en trouver de bien pires) est aussi imperméable à 99% des mathématiciens qu'elle l'est à lui. Il est suggéré dans la BD (je ne sais pas si Russell a vraiment émis cette opinion) que la seule personne qui ait lu le texte était Kurt Gödel ; et, de fait, les Principia sont sans doute autant célèbres pour le fait que c'est sur ce système que Gödel a initialement fondé son théorème d'incomplétude que pour être la première axiomatisation parfaitement rigoureuse de ce qui pourrait en théorie englober l'ensemble des mathématiques.

Par contre, il est vrai que la proposition énoncée sous le numéro *110·643 en haut de la page que je reproduis signifie bien ce qu'elle semble signifier : 1+1=2 (et le commentaire qui suit la démonstration est succulent : the above proposition is occasionally useful). Cette proposition intervient à la page 83 du second tome des Principia (dans la seconde édition), sachant que le premier tome comporte lui-même quelque chose comme 680 pages[#2]. Cela a valu aux Principia une renommée particulière, celle d'être le livre qui prend énormément de pages pour montrer 1+1=2 ; le énormément est parfois placé dans les 350, parce que la proposition *54·43, qui dit essentiellement que si deux ensembles ont chacun un élément et sont disjoints, alors leur union a deux éléments, est située page 362 du premier tome — ou 360 ou 379 selon les éditions — et suivie du commentaire : From this proposition it will follow, when arithmetical addition has been defined, that 1+1=2.

Ceci a malheureusement donné naissance au mythe selon lequel, en logique formelle, pour prouver quelque chose d'aussi évident que 1+1=2, il faut des centaines et des centaines de pages. C'est faux pour plusieurs raisons. D'abord parce que Whitehead et Russell n'avaient pas pour but d'arriver à cette proposition de la façon la plus rapide possible (même dans leur système on aurait pu faire ça de façon beaucoup plus économique). Ensuite, parce que leur système semble impossiblement compliqué aux yeux d'un logicien mathématique moderne[#3] : il est vrai qu'on ne cherche plus tellement à produire des systèmes qui fondent « toutes les mathématiques », mais même dans la mesure où on en choisit un, on peut espérer que la démonstation de 1+1=2 ne sera pas immensément compliquée (s'agissant de ZFC, le système orthodoxe pour fonder les mathématiques, la difficulté sera surtout d'écrire 1+1=2, c'est-à-dire, de définir qui sont ce ‘1’, ce ‘2’ et ce ‘+’ qui interviennent dans cette affirmation ; une fois cela fait, l'énoncé devrait sans doute être assez évident). J'avais lu quelque part un texte qui se voulait un peu sensationnaliste sur la longueur des démonstrations en logique formelle (où il concluait qu'il faudrait des quadrilliards de symboles pour démontrer le théorème des nombres premiers ou je ne sais quoi de ce genre) ; mais j'avais fini par me convaincre que l'auteur parlait probablement de démonstrations dites sans coupures (ce qui signifie, en très très gros, sans utiliser de lemme ou proposition intermédiaire ou quoi que ce soit de ce genre, mais en réécrivant à chaque fois la démonstration complète de ce qui aurait tenu lieu de lemme) : s'il y a des moyens d'éliminer les coupures dans une démonstration, ce moyen fait exploser la taille des démonstrations, c'est même un fait essentiel de la logique, et personne ne veut lire une démonstration sans coupures du théorème des nombres premiers ou même de 1+1=2.

[#] D'accord, on peut toujours trouver pire. Finnegan's Wake, par exemple ? Alors, pour lancer un petit troll, disons que les Principia sont peut-être bien le livre le plus abscons parmi ceux qui ont un sens. ☺️ (Plusieurs trolls, d'ailleurs : sur le fait de savoir si Joyce écrivait du pur charabia, et sur celui de savoir si de la logique peut avoir un sens ou autres questions qui auraient irrité Wittgenstein.)

[#2] Enfin, la proposition de loi sur la réforme du healthcare aux États-Unis fait autour de 2000 pages : je ne sais pas ce qui est le plus impressionnant, en fait.

[#3] La théorie ramifiée des types de Russell est assez éloignée de la façon dont on conçoit, de nos jours, les fondements des mathématiques (depuis que Zermelo a convaincu tout le monde de l'opportunité d'une théorie purement du premier ordre, avec une seule sorte d'objets — les ensembles). Pour un point de vue moderne sur la théorie de Russell (et son rapport avec les théories modernes), voir ce texte de Harvey Friedman ; cet article n'est pas mal non plus, pour remettre les choses dans le contexte et expliquer ce qu'est l'axiome de réductibilité.

↑Entry #1711 [older| permalink|newer] / ↑Entrée #1711 [précédente| permalien|suivante] ↑

↓Entry #1707 [older| permalink|newer] / ↓Entrée #1707 [précédente| permalien|suivante] ↓

(lundi)

De quoi parlent les mathématiques ?

Mathematics may be defined as the subject in which we never know what we are talking about, nor whether what we are saying is true. (Les mathématiques peuvent être définies comme la discipline dans laquelle on ne sait jamais de quoi on parle, ni si ce qu'on dit est vrai.) — Bertrand Russell (Recent Work on the Principles of Mathematics)

Je vais peut-être décevoir (ou au contraire rassurer ?) mon lecteur en avouant que je n'ai aucune intention d'essayer de répondre à la question qui sert de titre à cette entrée ; je vais tout au plus essayer de vulgariser un élément de réponse à une minuscule partie de cette question (ou d'une question proche), sur laquelle on peut dire des choses « techniquement » (c'est-à-dire : logiquement) précises. C'est déjà tout un programme.

[Ajout : cette entrée ultérieure évoque vaguement les mêmes questions, mais sous un angle différent ; je ne sais pas dans quel ordre il vaut mieux les lire.]

La plupart des mathématiciens (et même si ce n'est pas vraiment mon avis, je dois reconnaître qu'il est très répandu) conviendront que l'activité d'un mathématicien est de produire des théorèmes et des démonstrations. Par opposition, disons, aux définitions, exemples ou conjectures, qui forment certainement aussi une partie importante de l'activité en question, mais à laquelle l'opinion dont je parle attribue moins d'importance ou de dignité. Une démonstration est un argument logique plus ou moins formel qui suit certaines règles codifiées pour partir d'axiomes ou d'hypothèses et arriver à une conclusion : un énoncé qui est la conclusion d'une démonstration (connue !) est un théorème (ou une proposition, un lemme, un corollaire, selon sa difficulté, son importance et sa relation logique ou didactique à d'autres énoncés du sujet en cours de développement). Les règles du raisonnement, un peu comme celles des scolastiques d'autrefois (barbara, celarent, darii, ferio), sont supposées assez évidentes pour qu'on doute assez peu qu'elles préservent la vérité lorsqu'elles sont correctement appliquées : si les hypothèses de la démonstration sont vraies alors la conclusion l'est aussi ; donc, tout théorème produit à partir d'axiomes vrais est également vrai.

Mais quels sont les axiomes ? Un certain consensus, apparu au cours du XXe siècle, et maintenant assez fermement enraciné dans, disons, le dogme officiel des mathématiques, est que les axiomes qui fondent les mathématiques sont ceux de la théorie des ensembles de Zermelo-Fraenkel (en abrégé ZFC : le ‘C’ précise l'inclusion de l'axiome du choix, qui n'est pas du tout ce dont j'ai envie de parler à présent), les règles de raisonnement étant celles de la logique du premier ordre. Autrement dit, sauf mention explicite du contraire, ce qu'un mathématicien appelle théorème est un théorème de ZFC, et sa démonstration pourrait être rendue complètement formelle (une manipulation syntaxique fondée sur des règles de réécritures à partir des axiomes de ZFC pour arriver à ce théorème comme conclusion). C'est du moins le dogme officiel parce que, dans la pratique, beaucoup de mathématiciens non logiciens seraient probablement incapables de citer les axiomes de ZFC (ou de d'expliciter les règles de raisonnement de façon formelle et automatique) ; et la tâche d'expliciter complètement la démonstration de n'importe quel théorème modérément compliqué à partir des axiomes fondamentaux et en suivant les règles mécaniques est au mieux titanesque (même si les progrès de la vérification formelle ont montré qu'on pouvait arriver à des choses). Mais le dogme a le bon goût d'éviter des discussions sur les fondements des mathématiques que beaucoup de mathématiciens trouvent oiseuses ; il asseoit les mathématiques sur des bases solides et non dénuées d'élégance (et où, par exemple, la notion d'infini n'a plus rien de mystérieux ou de précaire) :

Aus dem Paradies, das Cantor uns geschaffen, soll uns niemand vertreiben können. (Du paradis que Cantor nous a créé, nul ne doit nous chasser.) — David Hilbert (Über das Unendliche)

↑Entry #1707 [older| permalink|newer] / ↑Entrée #1707 [précédente| permalien|suivante] ↑

↓Entry #1650 [older| permalink|newer] / ↓Entrée #1650 [précédente| permalien|suivante] ↓

(mercredi)

Élections à la proportionnelle : illustrations

Quelques illustrations pour rendre plus claire mon entrée précédente : dans chacun des diagrammes suivants, on a 12 sièges à répartir à la proportionnelle entre 3 listes ; le triangle représente les différentes proportions de voix possibles entre ces trois listes (les sommets représentent l'unanimité pour une des listes et 0 voix pour les deux autres, les côtés du triangle représentent les répartitions où une liste a 0 voix, et plus généralement les nombre de voix recueillies par les trois listes sont proportionnels aux distances aux trois côtés). Les (centres des) 78 gros points de couleur claire marquent les endroits où on a une représentation proportionnelle exacte (par exemple, le point jaune au milieu de la 3e ligne de points en partant du haut — celle qui a trois points — représente une répartition où une liste a exactement 10/12=5/6 des voix et les deux autres chacune 1/12 ; tandis que le point blanc au centre exact du triangle représente la répartition où chaque liste a exactement 4/12=1/3 des voix). Enfin, les régions de couleur qui divisent le triangle représentent chacun une configuration possible des sièges dans l'assemblée : la région marque donc l'ensemble des répartition de votes pour lesquelles le mode de scrutin considéré attribue cette configuration des sièges à l'assemblée. Par exemple, la région jaunâtre vers le sommet supérieur du triangle (celle qui contient le point jaune précédemment mentionné) représente les répartitions possibles des voix pour lesquelles le mode de scrutin représenté attribuera 10 sièges sur 12 à une liste et 1 siège à chacune des deux autres. Comme a priori on veut que la représentation proportionnelle donne à l'assemblée le nombre de sièges entiers exact évident si les voix sont dans des proportions exactes en 12e, évidemment, chaque région contient un et exactement un des points marqués (les exceptions étant si on ne permet pas à une liste d'obtenir zéro sièges, dans la méthode de Huntington-Hill).

[Diagramme méthode de Hare-Niemeyer]Dans le cas de la méthode du plus fort reste de Hare-Niemeyer/Hamilton, les régions sont de bêtes hexagones réguliers, centrés sur les points de représentation exacte, chaque répartition de votes étant envoyée sur la configuration de l'assemblée correspondant au point de représentation exacte métriquement le plus proche (ou, si l'on veut, il s'agit du diagramme de Voronoï des points de représentation exacte). C'est pour cette raison que la méthode est assez naturelle et intuitive. Notons que les hexagones au bord du triangle sont tronqués (ils n'ont donc pas la même aire que les autres) : si on tire uniformément au hasard la répartition des votes, on a moins de chances de tomber sur une situation où la configuration de l'assemblée ne donnera aucun siège à une liste que pour les autres configurations.

↑Entry #1650 [older| permalink|newer] / ↑Entrée #1650 [précédente| permalien|suivante] ↑

↓Entry #1649 [older| permalink|newer] / ↓Entrée #1649 [précédente| permalien|suivante] ↓

(mardi)

Comment faire une élection à la proportionnelle

Si on a N sièges d'une assemblée à répartir de façon proportionnelle entre r listes qui ont obtenu des proportions p1,…,pr des suffrages exprimées dans une élection, il y a plusieurs façons de procéder. (Je parle de sièges à répartir entre des listes dans une élection, mais c'est un problème tout à fait général : on peut vouloir attribuer n'importe quoi de non fractionnable entre n'importe quelle sorte d'entités de façon proportionnelle à n'importe quelles grandeurs pi.) Évidemment, lorsque N a le bon goût d'être un diviseur commun de p1,…,pr (c'est-à-dire que chaque N·pi soit un entier), les choses sont faciles : on attribue N·pi sièges à la liste i, et c'est tout (ensuite, il y a éventuellement la question de savoir quels sièges on attribue ou à qui sur la liste, mais je ne veux pas parler de ça ici : normalement les sièges sont interchangeables et on choisit juste les premiers de la liste). Évidemment, cette coïncidence numérique n'arrive jamais. On peut au moins commencer par attribuer à chaque liste la partie entière (c'est-à-dire, l'arrondi à l'inférieur, noté :) ⌊N·pi⌋, du nombre en question, mais il reste ensuite des sièges à répartir. Comment fait-on pour choisir à qui les donner ? Il y a différentes méthodes pour ça, qui ont des propriétés mathématiques et/ou politiques différentes, et qui sont employées dans divers contextes. (Je ne m'intéresse ici qu'aux situations où on répartit effectivement les sièges de façon proportionnelle : s'il y a, par exemple, une prime à la majorité, alors je parle des sièges en plus de cette prime — j'en avais déjà parlé dans le cas des municipales. De même, je fais abstraction des règles qui imposent une barrière minimale pour être représenté à la proportionnel : s'il y en a on suppose qu'on ne considère que les listes qui ont dépassé cette barrière.)

Ajout : voir l'entrée suivante pour une illustration graphique des différentes méthodes décrites ci-dessous.

Méthodes de plus fort reste

La méthode la plus naïve est celle du plus fort reste (ou plus exactement, une de celles de ce type), parfois plus précisément appelée méthode de Hare-Niemeyer ou de Hamilton : une fois attribuées les parties entières ⌊N·pi⌋, on compare les parties fractionnaires N·pi−⌊N·pi⌋, c'est-à-dire en quelque sorte les surplus de voix (les restes) par rapport au nombre nécessaire pour avoir le nombre de sièges qu'on vient d'attribuer, et on attribue un siège supplémentaire (jamais plus) aux listes ayant le plus fort reste, jusqu'à avoir attribué tous les sièges restants. Cette méthode peut sembler intuitive, et c'est celle qu'on invente généralement quand on veut faire une répartition à la proportionnelle et qu'on n'est pas matheux ; mais elle souffre d'un grave défaut : elle n'est pas monotone — il se peut très bien qu'en augmentant le nombre N de sièges disponibles, à proportions pi constantes, on diminue le nombre de sièges obtenu par telle ou telle liste. C'est le fameux Alabama paradox, découvert en 1880 parce que les Américains utilisaient cette méthode pour attribuer le nombre de sièges à la chambre des Représentants entre les états de l'Union proportionnellement à la population de ces états : on s'est aperçu que si le nombre de représentants au total passait de 299 à 300, alors l'Alabama en obtenait un de moins. À cause de ce paradoxe, ou parce qu'elle a tendance à trop favoriser les petites listes, cette méthode du plus fort reste est assez peu utilisée en pratique (elle sert cependant en Russie, par exemple).

↑Entry #1649 [older| permalink|newer] / ↑Entrée #1649 [précédente| permalien|suivante] ↑

↓Entry #1645 [older| permalink|newer] / ↓Entrée #1645 [précédente| permalien|suivante] ↓

(vendredi)

Comment un mathématicien ouvre une infinité de boîtes

Les remarques faites en commentaire sur mon entrée récente me semblent mériter quelques précisions d'ordre mathématique, auxquelles je vais consacrer cette entrée. Les deux parties suivantes, et même leurs différentes sous-parties, sont indépendantes (et au moins formellement indépendantes de l'entrée précédente). Vue la longueur de ce post, ce n'est pas une mauvaise chose, d'ailleurs.

Sur la calculabilité, et l'utilisation de l'axiome du choix

Plutôt que discuter sur le problème d'origine, je vais utiliser celui-ci, qui est légèrement plus simple :

Le cruel Docteur No a capturé une infinité dénombrable de mathématiciens pour les soumettre à une épreuve pernicieuse. Il a créé des chapeaux de plusieurs couleurs différentes. Après avoir permis aux mathématiciens de se concerter, il va les soumettre à son épreuve dont il leur communique les termes : il empêchera toute communication entre eux, puis rangera les mathématiciens en file indienne dans un certain ordre (indexé par les entiers naturels), et mettra sur la tête de chacun un chapeau d'une certaine couleur (les couleurs possibles sont connues à l'avance), de sorte que chaque mathématicien puisse voir les couleurs des chapeaux de tous ceux situés devant lui (une infinité), mais pas de ceux (en nombre fini) situés derrière lui ni du sien propre. Chaque mathématicien devra noter sur un papier (et toujours sans aucune communication avec les autres) ce qu'il croit être la couleur de son chapeau. Le Docteur No tolérera un nombre fini d'erreurs, mais pas plus : si une infinité de mathématiciens s'est trompée en annonçant la couleur de leur chapeau, tous seront tués avec des tortures particulièrement raffinées — à l'inverse, si tous sauf un nombre fini avaient raison, ils seront tous libérés. Comment les mathématiciens se tirent-ils de ce mauvais pas (de façon certaine) ?

De façon mathématique, on demande de construire une fonction φ:XX (où X est l'ensemble des couleurs et ℕ l'ensemble des naturels) qui à une suite de couleurs de chapeau associe la suite des réponses des mathématiciens, telle que si u et v ne diffèrent que sur les n premiers termes alors φ(u) et φ(v) ne diffèrent que sur les n−1 premiers termes (car, pour tout n, seuls les n−1 premiers mathématiciens peuvent pas voir certains des n premiers chapeaux), et telle que pour tout u les suites u et φ(u) coïncident sauf en un nombre fini de termes (seuls un nombre fini des mathématiciens a le droit d'avoir tort). Dispensons-nous des généralités vaseuses et mettons que X={0,1} (il n'y a que deux couleurs de chapeau : noir et blanc).

Peut-on avoir une stratégie calculable ?

Supposons un instant que nos mathématiciens soient, en fait, des informaticiens : ils ne peuvent calculer que les fonctions calculables effectivement par une machine de Turing ; plus exactement, il doit exister une machine de Turing T qui, quand on l'exécute en lui fournissant un entier naturel n et un oracle u (dont elle peut interroger librement les valeurs mais seulement celles qui sont au moins égales à n+1) doit terminer et renvoyer φ(u)(n). (On souligne qu'une machine de Turing ne peut évidemment, au cours de son exécution, interroger qu'un nombre fini de valeurs de l'oracle.) Or, dans ces conditions, les informaticiens ne peuvent pas être certains d'être libérés : en effet, si le Docteur No a connaissance[#] de la machine de Turing T qui sera utilisée par les informaticiens, il peut lancer l'exécution de T sur le nombre n0=0 en fixant arbitrairement la valeur de u(k) lorsque la machine T interroge l'oracle en k (forcément différent de 0 par hypothèse), puis choisir une couleur de chapeau u(0) pour l'informaticien 0 qui soit différente de la valeur retournée par T pour ce calcul (qui se fait en temps fini puisque T est censée toujours terminer) ; puis il lance l'exécution de T sur le premier nombre n1 pour lequel la valeur de u n'a pas encore été fixée, et de nouveau, à chaque fois que la machine T interroge l'oracle sur une valeur non encore déterminée de u, elle est fixée arbitrairement, et à la fin u(n1) est choisi différent du nombre renvoyé par T. Et ainsi de suite : alors les informaticiens n0, n1, n2, etc., se tromperont tous, donc le Docteur No aura réussi son plan diabolique.

↑Entry #1645 [older| permalink|newer] / ↑Entrée #1645 [précédente| permalien|suivante] ↑

↓Entry #1643 [older| permalink|newer] / ↓Entrée #1643 [précédente| permalien|suivante] ↓

(samedi)

Le Docteur No continue ses méfaits

Dans l'esprit des énigmes de combinatoire cybernétique que j'avais déjà posées, celle-ci est particulièrement étrange et étonnante (je la décris donc de façon très détaillée et verbeuse, pour qu'il n'y ait aucun doute sur les termes de l'épreuve) :

Le cruel Docteur No a capturé 100 mathématiciens pour les soumettre à une épreuve démoniaque. Il dispose dans une pièce de son bateau d'une infinité (dénombrable) de boîtes, étiquetées par les entiers naturels (0,1,2,3,…), contenant chacune un nombre (disons un nombre réel pour fixer les idées, même si des naturels marcheraient tout aussi bien : en tout cas, il n'y a aucune contrainte sur la suite de nombres ainsi formée) ; il est évidemment impossible de connaître le contenu d'une boîte sans l'ouvrir. Après avoir permis aux mathématiciens de se concerter, il va les soumettre à son épreuve dont il leur communique les termes : il empêchera toute communication entre eux et les emmènera chacun, dans un certain ordre, dans la pièce où se trouvent les boîtes.

Lorsqu'un mathématicien est dans la pièce, il pourra ouvrir les boîtes qu'il souhaite pour en examiner le contenu, y compris une infinité d'entre elles (en fonction éventuellement des nombres lus dans les boîtes déjà ouvertes) ; il devra cependant laisser au moins une boîte sans l'ouvrir, et faire une prédiction sur le contenu exact d'une boîte qu'il n'aura pas ouverte. (Entre les passages de deux mathématiciens, les boîtes sont bien sûr refermées, puisque les mathématiciens ne doivent disposer d'aucun moyen de communication ; ou, si on préfère, on peut imaginer qu'il y a 100 pièces différentes contenant chacune une copie identique de la même suite de nombres, et que tous les passages ont lieu simultanément : c'est équivalent.)

Au final, les 100 mathématiciens auront chacun fait une prédiction sur le contenu d'une boîte sans en avoir regardé le contenu. (Les mathématiciens ont le droit de faire des prédictions sur des boîtes différentes les uns des autres, ou sur la même.) Le Docteur No tolérera une seule erreur parmi ces prédictions : si au moins 99 des 100 mathématiciens ont donné exactement le bon nombre pour la boîte qu'ils ont désignée, alors le Docteur No les libérera tous. Si deux mathématiciens ou plus se sont trompés, alors le Docteur No tuera tous les mathématiciens avec ses tortures particulièrement raffinées.

Comment les mathématiciens font-ils pour être certains d'être tous libérés ?

La raison pour laquelle l'énigme semble (si on essaie d'y réfléchir) impossible à résoudre, c'est bien sûr qu'ouvrir des boîtes autres que celle sur laquelle on va faire la prédiction n'apporte aucune information sur cette dernière (puisque le Docteur No a le droit d'avoir rempli les boîtes absolumnet comme il le veut !), et on ne voit pas ce que peut apporter le fait qu'il y ait 100 mathématiciens, puisqu'ils ne communiquent pas du tout entre eux ; certes, on a droit à une erreur, mais on voit mal comment exploiter ce droit.

Je vais donc donner trois indications (cachées ci-dessous : cliquez sur les liens qui suivent pour les dévoiler) : la première est mathématique et devrait clarifier ce qu'on s'autorise à faire d'une infinité de nombres. La seconde indication est une nouvelle énigme, beaucoup plus simple (mais qui peut éventuellement être intéressante en elle-même, notamment pour les gens qui n'aiment pas les infinis), censée démystifier comment utiliser le droit à faire une erreur. Les deux ensemble devraient rendre claire la façon dont on peut prédire quelque chose sur une boîte sans l'avoir ouverte. (Attention cependant, il est possible que ma seconde indication, prise seule, mette sur une fausse piste.) Enfin, la troisième indication propose le mode opératoire. Les trois indications mises ensemble devraient rendre la solution assez évidente. Au lecteur de choisir quelle(s) indication(s) il souhaite ouvrir pour en lire le contenu !

[Ajout : voir une entrée ultérieure pour des éclaircissements sur le sens des règles.]

[Ajout ultérieur () : voir cette question de 2013 sur MathOverflow pour une discussion.]

↑Entry #1643 [older| permalink|newer] / ↑Entrée #1643 [précédente| permalien|suivante] ↑

↓Entry #1640 [older| permalink|newer] / ↓Entrée #1640 [précédente| permalien|suivante] ↓

(vendredi)

Encore un taquin de Mathieu

J'avais déjà proposé (deux fois) des petits jeux comme ça : celui qui se trouve en bas de cette entrée (cliquez ici s'il n'apparaît pas bien ci-dessous, et maudissez avec moi la norme HTML qui ne permet pas d'inclure un document dans un autre en l'insérant à sa taille naturelle) est donc ma troisième tentative, et j'en suis un peu plus content parce que, contrairement aux deux précédents qui étaient sérieusement trop durs, je crois qu'il est vraiment jouable (et construit de façon un peu moins ad hoc). Par contre, je ne suis pas du tout content de l'interface, comme je vais l'expliquer.

Le but du jeu, donc, est de mélanger les nombres de 1 à 24 puis d'arriver à les remettre dans l'ordre évident dans lequel ils sont présentés initialement. La commande ¿ (un point d'interrogation à l'envers, ne me demandez pas pourquoi) mélange complètement le taquin, et la commande (une croix) le réinitialise à sa position de départ ; quant à ¡ (un point d'exclamation à l'envers), elle effectue un mélange partiel, que j'appelle un m12lange pour une raison que je vais expliquer, qui n'échange pas de nombres entre les deux moitiés (dodécades), et qui peut donc servir de puzzle plus simple pour s'entraîner avant de passer au problème complet. Bref, votre mission est de mélanger — ou au moins de m12langer — le puzzle, puis d'arriver à le remettre dans son état initial en n'utilisant que les commandes de mouvement que je vais expliquer (mais peut-être que le mieux est d'expérimenter avec plutôt que de lire les instructions). Avant que quelqu'un n'exprime sa déception à ce sujet : non, il n'y a pas de message de félicitation quand on arrive à remettre le puzzle en ordre.

Expliquons donc quels sont les mouvements autorisés. Tout d'abord, les nombres sont placés en deux tableaux de 3 lignes × 4 colonnes (qu'on pourrait appeler dodécades), le tableau d'en haut contenant initialement les nombres de 1 à 12 et celui d'en bas de 13 à 24. Parmi les mouvements possibles, on peut faire n'importe quelle permutation des trois lignes d'un tableau, à condition de faire la même sur l'autre tableau : pour concrétiser ça, j'ai mis des commandes et , qui permutent cycliquement les trois lignes de chaque tableau, et qui échange les deux lignes d'en bas de chaque tableau : avec ça, on arrive assez facilement à faire n'importe quelle permutation des lignes (la même sur les deux tableaux). Pour ce qui est des colonnes, on à le droit à ce qu'on appelle une permutation paire (toujours la même sur les deux tableaux) : concrètement, j'autorise à faire une permutation cyclique des trois colonnes de gauche, ou des trois colonnes de droite, avec les commandes et situées de part et d'autre du losange central — celles à gauche permutent cycliquement les trois colonnes de gauche et celles à droite les trois colonnes de droite. Ensuite, on a l'opération la plus importante, le flip de Mathieu, commandée par le losange  : elle échange huit paires de nombres (quatre en haut et quatre en bas, mais pas organisées de la même façon), rappelées par des couleurs sur les cases en question — le contenu de chaque case coloriée est échangé avec celui de la case de même couleur. (Chaque paire est composée de deux cases adjacentes en diagonale — du moins, pour le tableau du bas, si on regarde les colonnes cycliquement.)

↑Entry #1640 [older| permalink|newer] / ↑Entrée #1640 [précédente| permalien|suivante] ↑

↓Entry #1625 [older| permalink|newer] / ↓Entrée #1625 [précédente| permalien|suivante] ↓

(mardi)

Traces de matrices nilpotentes

La trace de la matrice générique[#] d×d nilpotente d'ordre n est elle-même nilpotente d'ordre d·nd+1.

(C'est-à-dire : le plus petit k tel que sur n'importe quel anneau (commutatif) A, si t est la trace d'une matrice d×d nilpotente d'ordre n alors t est elle-même nilpotente d'ordre k, ce plus petit k vaut exactement[#2] d·nd+1.)

Ça ne vous fait peut-être pas des masses d'effet, mais moralement, une affirmation comme ça ne devrait pas être difficile à démontrer (sur un corps, le fait qu'une matrice nilpotente soit de trace nulle, c'est de niveau math. sup.). Il devrait juste s'agir de développer (x1,1+⋯+xd,d)k, réorganiser les termes pour faire apparaître des xi,j de façon à mettre en évidence les entrées de la matrice puissance n-ième, et conclure que pour k assez grand tout s'annule. Eh bien non, bizarrement, il semble qu'on ne sache pas faire comme ça (en tout cas, moi, je ne sais certainement pas), pour n et d quelconques.

Il semble que Bernard Mourrain ait une démonstration (les deux endroits où j'ai trouvé une référence à ce problème, ce Monsieur était nommément cité comme ayant expliqué la solution), et que ça utilise des bases SAGBI (Subalgebra Analog of Gröbner Bases for Ideals), je n'en sais pas plus : mais c'est scandaleux que quelque chose comme ça ne soit pas plus facile.

Où est-ce qu'on envoie les pétitions pour réformer les mathématiques ?

Mise à jour : Une démonstration élémentaire de cet énoncé est l'objet de l'article Gert Almkvist's Generalization of a Mistake of Bourbaki de Doron Zeilberger (il attribue le résultat à Almkvist (1973)).

[#] Précision () : Le terme générique n'est sans doute pas approprié : il faudrait plutôt dire universel, en fait. (La matrice universelle d×d nilpotente d'ordre n, c'est celle dont les d² entrées sont les images des indéterminées de l'anneau des polynômes à d² variables dans son quotient A par les relations algébriques qui expriment le fait que la matrice puissance n vaut zéro ; la matrice générique correspondante, ce serait la même matrice vue sur le corps des fractions au lieu de l'anneau quotient A — mais cet anneau n'est pas intègre, donc on ne peut pas vraiment parler de matrice générique ; on peut cependant réduire l'anneau A et ensuite passer au corps des fractions — mais évidemment, à ce moment-là, la trace est nulle, comme celle de n'importe quelle matrice nilpotente sur un corps.)

[#2] Plus exactement : montrer que tout exposant de nilpotence k de la trace vaut au moins d·nd+1 est facile en considérant une matrice diagonale de coefficients diagonaux nilpotents d'ordre n ; montrer que la trace est nilpotente (sans déterminer k) est également facile en utilisant le cas des corps : ce qui est difficile, c'est de voir que l'exposant d·nd+1 annule effectivement la trace.

↑Entry #1625 [older| permalink|newer] / ↑Entrée #1625 [précédente| permalien|suivante] ↑

↓Entry #1624 [older| permalink|newer] / ↓Entrée #1624 [précédente| permalien|suivante] ↓

(vendredi)

Calculs sur ordinateur : Sage et compagnie

Pfiou, déjà quinze jours que je n'ai rien posté ici ? Mais — outre qu'à chaque fois que je commence à écrire une entrée mon poussinet m'appelle pour faire dodo — j'ai un alibi : j'étais occupé à m'arracher les cheveux pour faire marcher des programmes de calcul pour faire un peu de géométrie algébrique effective, notamment Sage et Macaulay2.

En l'occurrence il s'agissait de vérifier des calculs dans un article que je référais rapportais, mais le prétexte était aussi d'apprendre à mieux me servir de ces programmes. Sage, en fait, est une sorte de méta-programme : il ne fait pas énormément lui-même, il a surtout pour fonction de rassembler sous une interface commune divers autres programmes auxquels il délègue les calculs difficiles (notamment Singular, qui devait faire tout le boulot sérieux sur les bases de Gröbner que je confiais à Sage), et j'espère d'ailleurs que Macaulay2 arrivera lui aussi prochainement sous l'ombrelle Sage, de sorte que je n'aurai plus qu'un programme à utiliser pour ce genre de choses (parce que c'est un peu fastidieux, entre les Sage, Macaulay2, Singular, mais aussi parfois GP/Pari, Gap4, Maxima, Axiom et quelques autres, de se rappeler qui a besoin d'un ; à la fin de la ligne, qui utilise % pour rappeler le résultat précédent, qui utilise = ou := ou : pour l'affectation, et ainsi de suite). Sage est basé sur Python, sans doute pas mon langage de programmation préféré, mais, du moins, un vrai langage de programmation, ce qui est toujours plus agréable que les succédanés que certains logiciels nous servent.

Tous les programmes que j'ai cités ci-dessus ont la vertu d'être des logiciels libres. Malheureusement ce n'est pas du tout le cas de celui qui fait aujourd'hui référence en matière de calculs informatiques en algèbre et géométrie algébrique (et qui, même si Sage fait des progrès très rapides et très spectaculaires, domine tous les autres à peu près autant qu'un moine Shaolin m'éclaterait en combat singulier), à savoir Magma. Celui-ci, non seulement il n'est pas libre, mais il est même vendu très cher (neither Free as in Free Speech nor Free as in Free Beer—more like Free as in Free Tibet!, comme on dit). Je trouve ça d'ailleurs plus qu'un peu choquant, vu que le coup bas ne vient pas d'une compagnie à but très lucratif (on a sinon la résignation du moins l'habitude que les Springer, les Elsevier et d'autres, ou dans un autre genre les Wolfram Research, se fassent de l'argent à partir des subventions à la recherche en mathématiques) mais d'une université, en l'occurrence celle de Sydney[#].

En l'occurrence, ce n'est pas qu'une préférence abstraite pour le logiciel libre ni un souci d'argent qui me font m'agacer de Magma, c'est aussi une troisième raison, une inquiétude scientifique pour la reproductibilité des calculs. Car si un théorème dépend du résultat d'un calcul (comme c'était le cas dans l'article que je rapportais), et si ce calcul n'est faisable qu'avec un unique logiciel dont, de plus, le code source n'est pas visible, peut-on avoir complètement confiance en l'exactitude du théorème ? Il ne suffit pas que les algorithmes annoncés être utilisés par le logiciel soient publiés et validés, encore faut-il qu'ils soient correctement implémentés et exempts de bugs, et cela on ne peut pas s'en assurer même en théorie. Je proposerais donc comme critère pour qu'un calcul puisse être considéré comme tenant lieu de démonstration inconditionnelle (lorsqu'il repose, évidemment, sur des algorithmes garantis et qui prouvent effectivement la conclusion annoncée) qu'il puisse être reproduit sur au moins deux logiciels développés de façon complètement indépendante, dont au moins un ait un code source publié. Certes je n'irai pas relire le code complet du logiciel, mais je n'ai pas non plus lu la démonstration du théorème de Hironaka, je fais confiance à des spécialistes pour ça, et je ne me prive pas de l'utiliser : il m'importe cependant de pouvoir lire au moins en théorie tout ce sur quoi mes démonstrations reposent, que ce soit d'autres démonstrations ou des logiciels ; la vérification par un second logiciel tiendrait, dans cette analogie, lieu de vérification d'une démonstration par un rapporteur — cela ne valide pas les logiciels tout entiers, mais ça valide au moins le calcul précis qu'on a fait refaire.

(Dans l'article que je rapportais, fort heureusement, j'ai pu refaire dans Sage et Macaulay2 la totalité des calculs que les auteurs prétendaient avoir effectués avec Magma : ce fut au prix de pas tout à fait du sang mais du moins beaucoup de larmes et de sueur.)

Comme exemple de calcul qu'on peut faire avec ce genre de programmes, voici un exemple dont je ne suis pas peu fier : l'équation cartésienne du bord des composantes hyperboliques de période 3 dans l'ensemble de Mandelbrot (c'est-à-dire la cardioïde du plus gros « ensemble satellite », celui qui est situé autour de −1.755 sur l'axe réel, plus les deux plus gros bulbes symétriques sur la cardioïde mère, situés autour de −0.123±0.745i). Il s'agit de la courbe d'équation suivante (où x désigne la partie réelle et y la partie imaginaire du paramètre) :

x12 + 6x10y2 + 15x8y4 + 20x6y6 + 15x4y8 + 6x2y10 + y12 + 8x11 + 40x9y2 + 80x7y4 + 80x5y6 + 40x3y8 + 8xy10 + 28x10 + 116x8y2 + 184x6y4 + 136x4y6 + 44x2y8 + 4y10 + 60x9 + 192x7y2 + 216x5y4 + 96x3y6 + 12xy8 + (6015/64)x8 + (3199/16)x6y2 + (3709/32)x4y4 + (127/16)x2y6 − (129/64)y8 + (927/8)x7 + (1117/8)x5y2 − (35/8)x3y4 − (225/8)xy6 + (232639/2048)x6 + (116925/2048)x4y2 − (5315/2048)x2y4 − (20673/2048)y6 + (186941/2048)x5 − (9283/1024)x3y2 + (56637/2048)xy4 + (3851/64)x4 − (11473/512)x2y2 + (6615/512)y4 + (64071/2048)x3 − (32193/2048)xy2 + (3552255/262144)x2 − (1528065/262144)y2 + (250047/65536)x + (15752961/16777216) = 0

Si vous croyez que c'est facile de calculer ça, même avec un ordinateur, trouvez-moi la courbe limite correspondante pour les points de période 4 — je serai très impressionné.

[#] Remarquez, si je commence à dire du mal des universités qui ont des politiques honteuses quand il s'agit de se servir de leur droit d'auteur pour éviter que quelque chose d'utile au public ou à la recherche puisse leur échapper, j'en aurais un rayon à sortir… j'ai déjà parlé du OED ?

↑Entry #1624 [older| permalink|newer] / ↑Entrée #1624 [précédente| permalien|suivante] ↑

↓Entry #1608 [older| permalink|newer] / ↓Entrée #1608 [précédente| permalien|suivante] ↓

(dimanche)

Petite histoire du « gras de tableau noir »

Puisque je parlais de notations mathématiques dans la dernière entrée, je vais raconter un autre petite histoire à ce sujet : celle de la police qu'on appelle le gras de tableau noir (enfin, je ne sais pas ce qu'on est censé dire en français : je sais juste que les lettres se disent double barre — c'est en anglais qu'on parle de blackboard bold).

Il s'agit des caractères qui ressemblent à ceci :

(en l'occurrence le Z double barre, qui désigne l'ensemble des entiers relatifs {…, −3, −2, −1, 0, 1, 2, 3, …}, et qui pour Unicode se nomme U+2124 DOUBLE-STRUCK CAPITAL Z, soit ℤ si vous avez donné des polices correctes à votre navigateur). Il s'agit d'une désignation maintenant assez standardisée pour les ensembles de nombres (les plus communs étant : ℕ soit N double barre pour les entiers naturels, ℤ soit Z double barre pour les entiers relatifs, ℚ soit Q double barre pour les rationnels, ℝ soit R double barre pour les nombres réels, ℂ soit C double barre pour les nombres complexes). Quelle est leur histoire ?

La désignation des ensembles de nombres était assez fluctuante jusque dans la première moitié du XXe siècle : l'utilisation du N pour les entiers naturels remonte au moins à Peano, celle du Z pour les entiers — sans doute comme initiale du mot allemand Zahl — est sans doute due à Landau dans ses Grundlagen der Analysis vers 1930 (mais l'écrivait en gothique fraktur, ℨ, avec une barre au-dessus), celle du R pour les réels a été faite par plusieurs personnes indépendamment (mais le R était aussi souvent utilisé au début du XXe pour désigner les rationnels).

C'est Bourbaki qui a fixé, à la fin des années '40, au moins les lettres suivantes : le N pour les entiers naturels, le Z pour les entiers relatifs, le Q pour les rationnels (qu'il semble avoir été le premier à introduire), le R pour les réels et C pour les complexes (ainsi que H pour les quaternions — par contre, il n'utilise pas O pour les octonions, et il est sans doute difficile d'attribuer à quelqu'un de précis une notation aussi évidente). Il semble que ces conventions se soient imposées très rapidement. Cependant, si Bourbaki a fixé les lettres, il utilise une police grasse normale pour les désigner.

L'histoire du « gras tableau noir » est complètement contenue dans son nom : pour simuler du gras au tableau noir, le plus simple est de doubler les traits. La façon dont cette habitude est passée du tableau noir au papier n'est pas évidente à retracer : il semble que le vecteur ait été les textes imprimés à la machine à écrire, où on graissait une lettre en la frappant deux fois légèrement décalée. (Le premier texte comme ça est peut-être le Lectures on Riemann Surfaces de Gunning, publié en 1966 à Princeton, Gunning lui-même ayant tiré cette idée de l'habitude prise dans le séminaire Kodaira-Spencer à Princeton au début des années '60 ; mais ce n'est pas sûr.)

Je ne sais pas non plus quel est le premier document imprimé de façon plus sérieuse qu'à la machine qui a eu ces caractères, mais il y a eu plusieurs styles de polices « gras tableau noir » dans TeX, essentiellement développées par l'AMS :

(msym10) est devenu (msbm10) vers 1991

(mais ce ne sont pas les seules formes possibles).

Ce qui est amusant, c'est que cette police est apparue par accident (du gras de Bourbaki au gras « tableau noir » en passant par le doublement des traits sur un vrai tableau noir et le doublement des caractères sur la machine à écrire), mais que c'est une invention vraiment géniale (et maintenant entérinée par son introduction dans Unicode) : alors qu'en maths on manque souvent de lettres et de symboles pour désigner les objets, voici une police complètement unique et très facilement reconnaissable pour désigner des objets uniques. Car il n'y a pas que les ensembles de nombres : le gras « tableau noir » semble servir généralement à désigner un objet mathématique qui n'est pas seulement particulier au problème, mais plus généralement bien reconnu dans l'ensemble de la branche des mathématiques où on se place, voire l'ensemble de toutes les mathématiques — en probabilités on utilisera 𝔼 soit E double barre pour l'espérance, en géométrie ℙ soit P double barre pour l'espace projectif, etc. (tiens, le μ double barre n'est pas dans Unicode : il faudra sans doute que je propose celui-là aussi — il sert à désigner le groupe des racines de l'unité).

Un peu de polémique maintenant : les membres de Bourbaki, notamment Serre, et un certain nombre de matheux français qui s'en sentent proches, n'aiment pas cette police et refusent de l'utiliser dans les textes imprimés, arguänt que Bourbaki avait choisi le gras, pas le doublement des barres. Je trouve que ce refus est une obstination d'orgueil bien malheureuse, car même si le gras « tableau noir » est le fruit du hasard, il est vraiment utile, et il a l'avantage de libérer le gras pour d'autres usages (par exemple pour désigner des catégories, des foncteurs, que sais-je encore) ; en tout cas, l'explication c'est un malentendu, le doublement des barres n'est qu'une façon de faire du gras au tableau noir ne vaut rien (on est conscient que c'est un malentendu, mais beaucoup d'inventions utiles sont nées d'une erreur, ce n'est pas une raison pour ne pas s'en servir !), pas plus que Bourbaki a choisi le gras (comme Landau a choisi la fraktur avec une barre dessus, mais on ne s'en sert plus). Heureusement, je crois qu'ils ont complètement perdu cette bataille (les notations pour les ensembles de nombres en « gras tableau noir » s'enseignent maintenant au collège/lycée, en France et dans beaucoup d'autres pays, et l'immense majorité des mathématiciens les considèrent maintenant comme acquises).

PS : Si quelqu'un veut bien faire à ma place l'effort de rendre un peu plus corrects les articles de Wikipédia (It is frequently claimed that the symbols were first introduced by the group of mathematicians known as Nicolas Bourbaki. There are several reasons to doubt this claim…), ce serait bien. L'ennui, c'est qu'il est à peu près impossible de donner une référence pour tout ce que je viens de raconter.

↑Entry #1608 [older| permalink|newer] / ↑Entrée #1608 [précédente| permalien|suivante] ↑

↓Entry #1607 [older| permalink|newer] / ↓Entrée #1607 [précédente| permalien|suivante] ↓

(vendredi)

Je propose un N-ARY RESTRICTED PRODUCT

Aujourd'hui j'ai fait une tentative pour augmenter mon karma geek : j'ai soumis un caractère à Unicode. Il s'agit d'un caractère qui ressemble à ça :

— c'est-à-dire une combinaison de ça : et ça :

Les deux symboles de droite existent déjà dans Unicode : il s'agit du symbole du produit de suites ou familles (U+220F N-ARY PRODUCT) ∏ (le glyphe est juste un grand pi majuscule) et du symbole du coproduit de suites ou familles (U+2210 N-ARY COPRODUCT) ∐ (le glyphe est le même mais inversé). Le symbole que j'ai proposé d'ajouter (et qui pourrait recevoir le nom de N-ARY RESTRICTED PRODUCT) a un glyphe dont la partie supérieure est celle du symbole du produit et la partie inférieure est celle du symbole du coproduit (donc le même à l'envers). Il dénote le produit restreint (toujours de suites ou familles) et apparaît en théorie des nombres — dans un contexte où le coproduit d'une famille infinie d'objets (qui, en général, est une opération duale du produit) serait le sous-ensemble du produit dont toutes les coordonnées sont nulles sauf un nombre fini, le produit restreint est intermédiaire entre le coproduit et le produit, il correspond au sous-ensemble du produit dont toutes les coordonnées sauf un nombre fini appartiennent à un sous-ensemble des facteurs compris implicitement ; cela apparaît par exemple, dans la définition de l'anneau des adèles ou du groupe des idèles d'un corps de nombres. Ce produit restreint est souvent noté ∏′ (pi-prime, quoi) par les auteurs qui n'aiment pas ou ne savent pas typographier le symbole ci-dessus, mais cette notation, proposée par John Tate, est occasionnellement utilisée, graphiquement élégante, et il me semble important de l'ajouter à Unicode.

Ça faisait longtemps que je pensais soumettre ce caractère (depuis un exposé au séminaire Variétés rationnelles, il y a un an, où il était apparu sous la craie de David Harari). Mais pour cela, il me fallait trouver des exemples de son utilisation : c'est là qu'était la difficulté, parce que pour convaincre que le symbole existe vraiment, il faut des exemples imprimés, or il sert surtout au tableau noir où ce n'est pas difficile de le faire et où le prime dans ∏′ (pi-prime) risquerait de ne pas être vu — alors que dans un texte imprimé, c'est le contraire, écrire ∏′ est plus facile (le symbole inventé par Tate, non seulement il n'est pas dans Unicode, mais il n'est pas non plus dans les jeux de macros LaTeX usuels). Je savais que je l'avais vu quelque part dans un livre, mais encore fallait-il retrouver où. Un ami m'a rapidement trouvé un spécimen dans Galois Cohomology de Serre, et ce n'est que récemment que je suis retombé sur un autre exemple que j'avais oublié, dans Algebraic Number Theory de Neukirch. Avec ces deux références (de deux auteurs différents et imprimées par un éditeur connu), je pense qu'il y a de bonnes chances que le caractère soit inséré : plutôt que m'adresser directement au working group, sur la suggestion d'un ami j'ai confié le combat à une experte en typographie mathématique qui m'a répondu que : the unicode technical committee has accepted the principle that math notation is open-ended, so they are receptive to well-documented submissions of new symbols. your informants are correct that, at present, i'm the "fast track" entry point to the system, and that will probably continue for several more years.

Nous allons donc maintenant voir combien de temps il faudra pour que ce caractère apparaisse dans une version ultérieure du standard. (Il ne faut jamais perdre espoir, avec Unicode : la première proposition d'encoder les hiéroglyphes date de 1999, maintenant c'est quasiment acquis, au moins pour la partie la plus basique, mais il faut encore attendre la sortie de la prochaine version, donc ça aura pris environ dix ans.)

[Mise à jour () : j'ai essayé de rassembler sur une page différents caractères dont je voudrais proposer l'addition à Unicode (voir une entrée ultérieure). Au fait, les hiéroglyphes basiques ont bien fini par arriver dans le standard.]

↑Entry #1607 [older| permalink|newer] / ↑Entrée #1607 [précédente| permalien|suivante] ↑

↓Entry #1606 [older| permalink|newer] / ↓Entrée #1606 [précédente| permalien|suivante] ↓

(samedi)

Comment tirer des bosons d'une urne ?

Un classique des cours élémentaires de probabilités : on a trois cartes de même forme, l'une ayant deux faces noires, la deuxième ayant une face noire et une rouge, la troisième ayant deux faces rouges. (Les trois cartes, et les deux faces d'une carte, sont indiscernables sauf par leur couleur.) On mélange les cartes sans les regarder, on en tire une au hasard et, toujours sans la regarder, on la pose sur la table (une face aléatoire étant visible, donc, l'autre étant cachée). On observe que la face visible est rouge (pour que ce soit bien clair : si ce n'est pas le cas, on recommence le mélange depuis le début, et on répète autant de fois que nécessaire jusqu'à ce que ça soit le cas). Quelle est la probabilité que la face opposée soit également rouge ?

Autrement dit, si on tire une carte et une face au hasard et qu'on sait que cette face est rouge, quelle est la probabilité que la carte tirée soit la carte rouge-rouge ?

La plupart des gens (enfin, ceux qui ont une réponse à la question) répondent 1/2 suivant le raisonnement suivant : la carte noire-noire est exclue, il ne reste que deux possibilités, la carte rouge-noire et la carte rouge-rouge, et comme tout ce qu'on sait est qu'il y a une face rouge, ces deux possibilités sont équiprobables, donc la probabilité d'avoir affaire à la carte rouge-rouge est 1/2. Ce raisonnement est faux, parce que l'événement connu n'est pas la carte a une face rouge mais bien la face que j'ai tirée au hasard est rouge : la probabilité recherchée n'est pas 1/2.

↑Entry #1606 [older| permalink|newer] / ↑Entrée #1606 [précédente| permalien|suivante] ↑

↓Entry #1593 [older| permalink|newer] / ↓Entrée #1593 [précédente| permalien|suivante] ↓

(mardi)

Ensemble de Mandelbrot : programme, images fixes, vidéo nº2

Pour satisfaire la promesse que j'avais faite la semaine dernière, le programme que j'ai écrit pour calculer des vues de l'ensemble de Mandelbrot est maintenant disponible ici (j'en ai profité pour rafraîchir très sérieusement cette page listant mes programmes : peut-être qu'il y aura d'autres choses qui intéresseront les gens). Pour ceux qui veulent juste voir des jolies images, j'ai mis quelques vues fixes sur flickr, et j'ai créé une deuxième vidéo (de nouveau, il y en a une version de haute qualité, de 44Mo, téléchargeable par BitTorrent[#] en suivant ce lien — où vous pouvez retirer le .torrent final si BitTorrent ne vous est pas possible — et une version exécrable sur YouTube ; par ailleurs, comme je le disais dimanche, j'ai réencodé les vidéos pour avoir quelque chose que j'espère de meilleure qualité et d'ailleurs légèrement plus petit).

[#] J'ai d'ailleurs peut-être un problème avec mes torrents : il est censé y avoir deux seeds (semences ? graines ?), mais certains clients BitTorrent n'en voient qu'une, ou prennent un certain temps à voir la seconde. Je ne sais pas ce qui se passe.

↑Entry #1593 [older| permalink|newer] / ↑Entrée #1593 [précédente| permalien|suivante] ↑

↓Entry #1591 [older| permalink|newer] / ↓Entrée #1591 [précédente| permalien|suivante] ↓

(mardi)

Zoom dans l'ensemble de Mandelbrot

[Zoom dans l'ensemble de Mandelbrot]Ça faisait longtemps que je rêvais de faire une belle vidéo de zoom dans l'ensemble de Mandelbrot[#], mon poussinet m'a redonné cette envie en jouant avec Fraqtive (j'en parlais la semaine dernière) : me voilà enfin satisfait à ce sujet. Pour l'instant, vous ne pourrez voir de la vidéo en question que la version sur YouTube (ma première vidéo YouTube !), qui est d'une qualité épouvantablement exécrable[#2] (enfin, vous avez le choix entre la qualité normale, qui est épouvantablement exécrable, et la « haute » qualité, qui est seulement lamentablement exécrable), mais je promets de publier prochainement (1) la vidéo en haute qualité (ou en tout cas, beaucoup plus haute que ça), (2) le programme qui a servi à la calculer et peut-être (3) quelques petites notes de vulgarisation sur ce qu'est l'ensemble de Mandelbrot. Mais là je n'ai pas le temps.

Mise à jour () : J'ai uploadé la vidéo de haute qualité (64Mo pour 4′14″ en 640×480, 25fps) : vous pouvez la télécharger (en utilisant BitTorrent, BitTornado, Azureus ou un autre client de ce genre) en suivant ce lien (il s'aggit d'un petit fichier .torrent que vous passez ensuite à votre client BitTorrent pour qu'il récupère le tout). Pour ceux qui n'arrivent pas à utiliser BitTorrent ou équivalent (par exemple parce que vous êtes à la merci d'un administrateur réseau crétin qui croit que ça ne sert qu'à diffuser des contenus « piratés »), vous pouvez récupérer directement le .avi en effaçant l'extension .torrent du lien précédent (je fais exprès de ne pas faire un lien direct, parce que je veux m'assurer que les gens ne cliquent pas sans réfléchir).

Suite : voir cette entrée ultérieure.

[#] Vous me direz que ça manque pathétiquement d'originalité (il suffit de regarder le nombre de vidéos de ce genre déjà sur YouTube). Je dois bien reconnaître que c'est vrai ; mais mon amour-propre me poussera à prétendre, avec un tantinet de mauvaise foi, que ma vidéo à moi elle est — aux artefacts de compression près — beaucoup plus belle que toutes les autres : certes, elle ne vas pas aussi loin que certaines (il y en a une sur YouTube qui va à une magnification de 101000, et qui est d'ailleurs rigoureusement sans intérêt), mais elle montre plus de variété dans l'ensemble puisque j'ai judicieusement choisi le point autour duquel zoomer. (Par exemple, pour trouver des points intéressants, il faut résister à la tentation de descendre dans les spirales de ce qu'on appelle les points de Misiurewicz, qui n'apportent aucun motif nouveau mais seulement des itérations en plus dans les calculs.) Sinon, pour ce qui est de la musique, ce n'est peut-être pas ce que j'aurais mis idéalement, mais je voulais un enregistrement dans le domaine public (l'ensemble de Mandelbrot l'est forcément, puisqu'il est une pure construction mathématique, et ç'aurait été dommage que la seule musique apporte un copyright à la vidéo), et ce n'est pas évident à trouver.

[#2] J'en suis d'ailleurs un peu contrarié : YouTube fournit des explications sur les formats à utiliser, que j'ai suivies scrupuleusement (codec H.264, résolution 640×360, audio MP3, 30 images par seconde), en me disant que peut-être comme ça ils n'auraient pas à réencoder la vidéo, et évidemment, ça n'a pas manqué… Ils m'auraient donné une borne sur le débit, j'aurais peut-être pu produire un truc de meilleur qualité qu'eux avec cette contrainte !

↑Entry #1591 [older| permalink|newer] / ↑Entrée #1591 [précédente| permalien|suivante] ↑

↓Entry #1585 [older| permalink|newer] / ↓Entrée #1585 [précédente| permalien|suivante] ↓

(vendredi)

Une nouvelle soumission au Sloane

Je viens de soumettre à l'encyclopédie des suites d'entiers la séquence :

0, 0, 1, 1, 3, 2, 9, 9, 23, 29, 89, 72, 315, 375, 899, 1031, 3855, 3886

Pour ceux qui s'amuseraient à chercher ce que ça peut être avant que Sloane la valide (ou la rejette s'il trouve ça sans intérêt), voici trois indications : corps fini à 2n éléments • éléments primitifs • trace.

Mise à jour : La suite a été publiée sous le nom A152049.

↑Entry #1585 [older| permalink|newer] / ↑Entrée #1585 [précédente| permalien|suivante] ↑

↓Entry #1556 [older| permalink|newer] / ↓Entrée #1556 [précédente| permalien|suivante] ↓

(samedi)

Solides réguliers en quatre dimensions

On me signale un magnifique film de vulgarisation mathématique, en neuf parties : Dimensions, par Jos Leys, Étienne Ghys et Aurélien Alvarez[#] (téléchargeable[#2] et redistribuable sous licence Creative Commons by-nc-nd 3.0, et également commandable en DVD) ; il s'agit, notamment, d'essayer de donner une idée compréhensible par le grand public de ce à quoi la quatrième dimension (et les solides réguliers en dimension 4) preuvent ressembler, ainsi que d'autres choses (comme les transformations conformes du plan, les fractales…). C'est tellement rare de voir de la bonne vulgarisation mathématique qu'il faut vraiment signaler celui-ci.

Il y a six solides réguliers en dimension 4[#3] : c'est d'autant plus remarquable que pour toutes les dimensions à partir de 5 il n'y en a plus que trois (le simplexe, l'hypercube et le dual de ce dernier), et que si chacun des solides réguliers en dimension 3 a une généralisation naturelle en dimension 4 (tétraèdre→simplexe=5-cellule ; cube→hypercube=tesseract ; octaèdre→16-cellule ; dodécaèdre→120-cellule ; icosaèdre→600-cellule), il y en a un supplémentaire, le 24-cellule (formé de 24 octaèdres assemblés de façon complètement régulière), véritablement exceptionnel[#4], qui n'a pas d'analogue en dimension 3. Bref, la dimension 4 est la plus fertile en solides réguliers.

Je pense que la meilleure façon[#5] de visualiser les solides réguliers en dimension 4 — qui est proche[#6] mais pas identique à une de celles utilisée dans le film, à savoir la projection stéréographique — consiste à les mettre sur la 3-sphère (S3), et à voir celle-ci comme un espace courbe de dimension 3, et à faire du raytracing dans cet espace courbe. J'avais proposé ça il y a longtemps, mais je n'ai jamais eu la patience de faire. D'ailleurs, la raison pour laquelle ils n'ont pas fait ce que je propose dans le film, c'est probablement que ça oblige à mettre à la poubelle les outils comme Povray.

Quoi qu'il en soit, j'espère que ce film aura une large diffusion : si ça peut susciter des vocations, notamment.

[#] Je ne connais pas les autres auteurs, mais Étienne Ghys, qui semble avoir écrit le scénario de la partie proprement mathématique, est quelqu'un d'absolument impressionnant par sa culture mathématique (le genre qui est capable d'écouter parler un mathématicien de n'importe quel domaine et de poser des questions intelligentes après).

[#2] Enfin, si ce n'est qu'au moment où j'écris (2008-06-21T21:45+0200) le site est indisponible pour le téléchargement. (C'est de nouveau disponible.) Si on ne s'était pas tellement occupé de faire interdire autant que possible les protocoles de torrent ou de pair-à-pair (par exemple dans les universités et lieux de recherche), on se serait rendu compte qu'ils sont aussi drôlement utiles dans ce genre de cas pour distribuer des contenus légaux sans avoir un point d'échec systématique à cause des capacités des serveurs. En l'occurrence, je pressens que la vidéo va certainement fasciner les joyeux mutants de BoingBoing et les foules nerdesques de Slashdot quand ces deux sites s'en seront emparés, ce qui ne manquera pas d'arriver, donc il serait bon d'avoir un bittorrent d'ici là. (J'en aurais bien lancé un, mais malheureusement j'ai bêtement effacé les fichiers .zip une fois téléchargés.)

[#3] Le film dont je parle ne mentionne que cinq d'entre eux (sans doute par manque de temps ou de patience et parce que celui qu'ils ont omis — le 16-cellule qui généralise l'octaèdre — n'est ni le plus simple à comprendre ni le plus impressionnant ni le plus beau).

[#4] Il est fortement lié au système de racines exceptionnel de type F4, un de ces bijoux de symétrie qui existent dans le paradis mathématique, qu'on sait prédire et démontrer mais dont je ne pense pas qu'on puisse vraiment les expliquer. On peut cependant faire remarquer que le centre d'un hypercube (en dimension 4) est à la même distance des sommets que les sommets sont entre eux, ce qui laisse soupçonner que si on prend seize hypercubes se touchant en un sommet, les seize centres des hypercubes et les huit sommets adjacents dans les huit directions vont, tous ensemble, former quelque chose de joli : de fait…

[#5] Ce qui est certain est que la moins bonne façon de visualiser les solides réguliers est par sections successives, comme c'est d'ailleurs bien expliqué dans le film : c'est joli, mais ça n'aide vraiment pas à se faire une idée de l'objet. D'ailleurs, c'est un test terrible pour se rendre compte si on voit en quatre dimensions : Y a-t-il une section (hyperplane) de l'hypercube qui soit une pyramide à base quadrilatérale ?

[#6] La différence essentielle avec la projection stéréographique, c'est que les rayons de lumière qu'on utilise pour la projection, ils suivent eux-mêmes des géodésiques (i.e., des grands cercles sur S3) : du coup, on a beaucoup moins de degrés de liberté pour montrer l'objet, on peut seulement se balader dedans et tourner, c'est la même chose que de faire tourner l'objet en 4D, alors que si on prend la projection stéréographique comme ils font dans le film ça donne deux effets différents, du coup c'est un peu plus dur à visualiser. Par contre, la vision en espace courbe dont je parle a la propriété que le solide paraît infini dans toutes les directions (en fait c'est simplement qu'il y a des rayons qui reviennent après avoir fait le tour de la sphère) : il convient donc d'ajouter des marqueurs pour différencier une (ou un certain nombre de) face.

↑Entry #1556 [older| permalink|newer] / ↑Entrée #1556 [précédente| permalien|suivante] ↑

↓Entry #1544 [older| permalink|newer] / ↓Entrée #1544 [précédente| permalien|suivante] ↓

(mercredi)

Platonisme et mathématiques

Une assez longue réflexion en philosophie des mathématiques, que j'ai écrite dans un mail à un ami, et qui me semble suffisamment intéressante pour reproduire ici : il s'agit d'éclaircir un peu la question de savoir quel sens est-ce que ça a de qualifier un énoncé mathématique de vrai ou faux ?, i.e., dans quelle mesure les objets mathématiques existent-ils ? Et notamment : pourquoi est-ce qu'on sera généralement d'accord pour attribuer la valeur de vérité vraie à l'énoncé 252097800623 est un nombre premier mais qu'on sera beaucoup plus sceptique quant au fait que l'hypothèse du continu ait vraiment un sens dans le monde réel ?

[Ajout : on ferait mieux de commencer par lire d'abord cette entrée ultérieure, sans doute beaucoup plus pédagogique.]

On peut établir une hiérarchie dans laquelle s'inscrivent un certain nombre d'énoncés mathématiques (mais pas tous !) en fonction de la complexité de leurs quantificateurs. (Je crois que cette hiérarchie est due, sous une forme ou une autre, à Stephen Kleene et Azriel Lévy, mais je ne sais pas exactement quelles sont les contributions de l'un et de l'autre. Il en existe un certain nombre de variantes, arithmétiques ou ensemblistes : la variante qui suit est clairement du côté arithmétique d'ordre supérieur si on doit la qualifier plus précisément : ce que j'appellerai un énoncé Πn c'est précisément un énoncé arithmétique Πn.)

Tout en bas de la hiérarchie, il y a les énoncés et prédicats qu'on peut indifféremment appeler Σ0, Π0 ou Δ0. Il s'agit des affirmations dont tous les quantificateurs portent sur les entiers naturels et sont, de surcroît, bornés, ou gardés, c'est-à-dire sont de la forme il existe un n inférieur à t ou pour tout n inférieur à t avec t un terme en les variables libres à ce point-là (disons que le terme peut faire intervenir l'addition, la multiplication et l'exponentiation des entiers, ou peut-être n'importe quelle fonction primitive récursive, pour ce que je vais dire ce n'est pas important).

Par exemple, l'entier formé par les 10000000000 premières décimales de pi en base 10 est un nombre premier peut s'écrire sous la forme d'un énoncé Δ0 (modulo un tout petit peu de travail sur les écritures, i.e., l'implémentation d'un algorithme qui calcule pi). L'essentiel est que quand on a affaire à une telle affirmation, on peut la tester, de façon algorithmique, en temps fini, avec terminaison garantie (pour chaque valeur des variables libres, s'il y en a) : pour les connecteurs propositionnels c'est clair, et pour les quantificateurs, comme ils sont bornés, on n'a jamais qu'un nombre fini de cas à tester ; pour la même raison, un énoncé Δ0 n'est jamais indécidable. Ça c'est ce qui est censé justifier que, philosophiquement, à peu près tout le monde conviendra qu'il y a bien un sens à dire si un tel énoncé est vrai ou faux — il suffit d'essayer. Évidemment, comme les opinions philosophiques des gens sont très variés, il y aura des ultra-finitistes purs et durs pour me dire que d'après eux, non, ça n'a pas de sens de se demander si l'entier formé par les 10↑(10↑(10↑(10↑10))) premières décimales de pi est premier ou non, s'il est complètement inconcevable de tester aussi loin : j'appellerai ces gens des (0,0)-platoniciens (i.e., pas du tout platoniciens) ; les autres sont au moins (0,1)-platoniciens.

Juste au-delà des énoncés (et prédicats) Δ0, il y a les énoncés (et prédicats) Σ1 et Π1. Les premiers sont formés en ajoutant devant un prédicat Δ0 un quantificateur existentiel portant sur les entiers naturels, alors que les seconds ajoutent un quantificateur universel. En fait, il est assez facile de voir (en utilisant un codage des k-uplets d'entiers par des entiers) qu'on peut se permettre d'ajouter un nombre fini de quantificateurs de même nature pour le même prix. Bien sûr, la négation d'un énoncé Σ1 est Π1, donc pour ce qui est de donner une valeur de vérité on peut ne regarder qu'un d'entre eux.

↑Entry #1544 [older| permalink|newer] / ↑Entrée #1544 [précédente| permalien|suivante] ↑

↓Entry #1542 [older| permalink|newer] / ↓Entrée #1542 [précédente| permalien|suivante] ↓

(jeudi)

Qu'est-ce qu'un nombre aléatoire ?

Voici une question un peu provocante : qu'est-ce qu'un nombre aléatoire ? Pour un probabiliste, elle n'a pas de sens : un nombre n'est pas aléatoire (le nombre π, par exemple, ça n'a pas de sens de dire qu'il est aléatoire), ce qui a un sens, c'est de tirer aléatoirement un nombre, disons, dans l'intervalle [0;1]. Pourtant, il y a des branches des mathématiques où cela a bien un sens, de façon absolue, de dire qu'un nombre réel bien défini est — ou n'est pas — aléatoire (et, pour gâcher le suspens, le nombre π n'est pas aléatoire, en aucun sens qu'on sache définir à ma connaissance, ce qui est bien dommage parce que ça permettrait de prouver plein de choses merveilleuses à son sujet, mais c'est tout à fait intuitif parce qu'on arrive à le calculer — donc il n'est certainement pas tiré au hasard !). Je vais tâcher d'expliquer simplement de quoi il retourne, sans entrer dans trop de technicité mais en donnant tout de même des définitions complètes. L'idée qu'on cherche à développer est que :

L'aléatoire est ce qu'on ne peut prévoir (de façon non triviale) par le calcul.

↑Entry #1542 [older| permalink|newer] / ↑Entrée #1542 [précédente| permalien|suivante] ↑

↓Entry #1534 [older| permalink|newer] / ↓Entrée #1534 [précédente| permalien|suivante] ↓

(dimanche)

J'apprends à compter jusqu'à ψ(εΩ+1) et à dompter les hydres

Si j'ai été silencieux pendant deux semaines c'est qu'aussitôt abandonné le nim épicé j'ai été saisi d'une autre obnubilation intellectuelle (et qui me revient périodiquement[#]), c'est celle des ordinaux dénombrables : j'avais déjà écrit des articles de vulgarisation à ce sujet (celui-ci il y a très longtemps et celui-ci il y a un peu moins longtemps, le premier ayant même été traduit en chinois[#2] ; et j'ai beaucoup contribué à l'article de Wikipédia en anglais), cette fois je me suis intéressé aux notations ordinales — c'est-à-dire, en gros, la façon de donner un nom aux (plus petits des) ordinaux dénombrables, en essayant d'aller le plus loin possible, tout en sachant très bien qu'on ne pourra pas atteindre tous les ordinaux dénombrables (ni même tous les ordinaux récursifs). Et j'ai produit les jolis dessins que voici[#3] et surtout ce texte explicatif dont je vais dire un peu plus.

[Ajout : voir une entrée ultérieure, qui peut servir d'introduction à la suite.]

Quand on entend parler des ordinaux pour la première fois, on entend parler de ω, de ωω, de ωωω, et souvent de la limite de tout ça, qui est ε0 (qui le plus petit ordinal vérifiant ωξ = ξ, c'est-à-dire le plus petit point fixe de ξ↦ωξ). Manipuler les ordinaux jusqu'à ε0 (pour les ajouter, les multiplier ou les exponentier) est facile si on travaille en forme normale de Cantor (c'est-à-dire, informellement, en base ω) itérée pour les exposants ; à partir de ε0 ça devient un chouïa plus compliqué parce qu'il y a des égalités qui ne sautent pas immédiatement aux yeux (comme le fait que ωε0+1 = ε0·ω), mais on finit par s'y habituer. Le problème est plutôt de monter aussi loin que possible : si ε1 est la prochaine solution de ωξ = ξ, et ainsi de suite, jusqu'où ira-t-on ? On pourrait appeller ζ0 la limite de la suite ε0, εε0, εεε0, c'est-à-dire le premier point fixe de ξ↦εξ, on peut continuer à jouer ainsi avec les lettres grecques qui énumèrent les points fixes les unes des autres, si bien qu'on finit par avoir besoin d'un alphabet grec transfini : c'est ce qu'on appelle le schéma φ de Veblen (à deux variables, pour commencer), chaque rang énumérant les points fixes du rang précédent. Seulement, on arrive à court de ce schéma aussi lorsqu'on tombe sur la première lettre grecque qui est, pour ainsi dire, son propre rang dans l'alphabet. C'est cet ordinal (toujours dénombrable, bien sûr) qu'on appelle l'ordinal de Feferman-Schütte (et c'est la limite des ordinaux pour lesquels le schéma de Veblen à deux variables fournit des notations).

↑Entry #1534 [older| permalink|newer] / ↑Entrée #1534 [précédente| permalien|suivante] ↑

↓Entry #1533 [older| permalink|newer] / ↓Entrée #1533 [précédente| permalien|suivante] ↓

(dimanche)

Le jeu de nim épicé

Je disais récemment que les idées scientifiques ont tendance à ressembler aux tamagotchis : on commence à les élever et rapidement elles vous accaparent l'esprit en demandant sans arrêt votre attention.

Je réfléchissais à la théorie combinatoire[#] des jeux quand je suis tombé sur l'idée du jeu suivant (qui a certainement déjà été décrit et étudié, mais il est difficile de trouver sous quel nom), que j'appellerai le jeu de nim épicé.

Tout d'abord je rappelle ce qu'est le jeu de nim usuel : il s'agit d'un jeu (impartial, à connaissance complète, à deux joueurs) extrêmement simple, dans lequel l'état du jeu est donné par un certain nombre de lignes de bâtonnets (ou quelconques autres jetons : tas de pièces, rangées d'alumettes, ce que vous voudrez ; il en existe aussi des avatars un petit peu moins évidents où, par exemple, on avance des pions sur un échiquier sans avoir le droit de les reculer). Seul compte le nombre de bâtonnets présent sur chaque ligne (il n'y a pas d'ordre, pas de structure supplémentaire). La règle du jeu est simplissime : les joueurs jouent tour à tour et chacun, quand c'est son tour, peut et doit retirer des bâtonnets situés sur une ligne — il doit en retirer au moins un et peut en retirer autant qu'il veut, y compris vider (et donc supprimer) la ligne, mais il ne peut en un tour retirer des bâtonnets que d'une seule ligne (de son choix). Normalement, le joueur qui ne peut plus jouer a perdu (i.e., celui qui retire le dernier bâtonnet a gagné, puisque son adversaire est dans l'impossibilité de jouer) : en fait, la version misère, où celui qui retire le dernier bâtonnet perd, est peut-être plus commune pour le jeu de nim, mais elle est en fait très semblable[#2] et moins intéressante mathématiquement.

Dans n'importe quel jeu de ce genre (i.e., un jeu impartial, à connaissance complète, à deux joueurs, et qui termine toujours en temps fini sur le gain d'un des joueurs avec par convention perte du joueur qui ne peut plus jouer), l'un des deux joueurs a forcément une stratégie gagnante. En fait, on peut définir deux sortes de positions du jeu : les positions dites nulles[#3] — qui sont celles à partir desquelles le second joueur a une stratégie gagnante —, et les autres (généralement les plus nombreuses). La définition (récursive !) est qu'une position nulle est une position qui ne conduit qu'à des positions non nulles (y compris si elle ne conduit à rien du tout, car alors le joueur qui vient de jouer à gagné) et, a contrario, une position non nulle est une position qui conduit à au moins une position nulle. La stratégie gagnante consiste à jouer, autant que c'est possible, pour mettre le jeu dans une position nulle (auquel cas le joueur adverse sera obligé de jouer pour aller dans une position non nulle, donc on sera assuré de pouvoir jouer vers une position nulle, et ainsi de suite : comme on peut toujours gagner, on ne peut pas perdre). Connaître la stratégie gagnante revient donc à identifier les positions nulles (pour pouvoir jouer vers elles).

Dans le cas du jeu de nim, elles ont une description mathématiquement simple : sont nulles les positions telles que le « ou exclusif » du nombre de bâtonnets de chaque ligne (écrits en binaire) soit nul. Par exemple, la position de départ habituelle, (1,3,5,7) (ce qui signifie : un bâtonnet sur une ligne, trois sur une autre, cinq sur une troisième, et sept sur la dernière ligne) est nulle car 001⊕011⊕101⊕111=000 : cela signifie qu'au jeu de nim usuel, à partir de cette position, le joueur qui ne commence pas peut gagner le jeu à coup sûr (par exemple, si le premier joueur retire deux bâtonnets de la dernière ligne, c'est-à-dire joue vers (1,3,5,5), position non nulle, on répondra vers (1,1,5,5), position nulle, et il est d'ailleurs clair qu'on a alors une stratégie gagnante en reproduisant les coups d'une paire de lignes (1,5) sur l'autre).

J'en viens donc au jeu de nim épicé : la différence avec le jeu nim normal est qu'il existe deux sortes de lignes, des lignes épicées et des lignes normales (ou fades). La règle du jeu est la même (on peut retirer autant de bâtonnets qu'on veut, mais d'une ligne seulement) avec la seule différence que lorsqu'un joueur retire des bâtonnets d'une ligne épicée, son adversaire ne peut pas en faire autant au coup immédiatement après (il doit retirer des bâtonnets d'une ligne fade — et s'il n'en reste aucun, il a perdu). Autrement dit, on ne peut consommer deux coups de suite des bâtonnets épicés. Une position du jeu est déterminée par le nombre de bâtonnets dans les lignes de chaque sorte (je les écrirai en commençant par les lignes fades) et par le fait que le coup immédiatement précédet ait été épicé ou non.

Exemple : en commençant sur (1,2,11;4;4), c'est-à-dire qu'il y a trois lignes fades avec 1, 2 et 11 bâtonnets, et deux lignes épicées avec 4 bâtonnets chacune, le premier joueur pourrait jouer vers (1,2,5;4;4), le second joueur répliquerait avec (1,2,5;4,2)H (le H, comme hot signifiant qu'on vient de jouer épicé, donc que le coup suivant sera forcément fade), le premier joueur tenterait maladroitement (1,2,3;4,2), ce à quoi on rétorquera (1,2,3;2,2)H, et une tentative désespérée vers (1,2,1;2,2) provoquera inévitablement la réplique (1,2,1;2)H, et il n'y a plus de doute sur qui a gagné.

La question à cent zorkmids est donc : quelles sont les positions nulles du jeu de nim épicé ? Peut-on les décrire de façon synthétique comme pour le jeu de nim usuel ?

Je n'ai que des réponses partielles à cette question. Je peux facilement faire produire à mon ordinateur des tables gigantesques de positions nulles, mais ensuite trouver la logique ressemble à un test d'intelligence. Et si c'est un test d'intelligence, j'ai échoué : j'ai trouvé beaucoup de motifs partiels, mais aucune decription complète.

Il est clair que si le nombre total de bâtonnets épicés est strictement supérieur au nombre total de bâtonnets fades (et que la position n'est pas chaude, i.e., qu'on peut jouer épicé), alors la position est non nulle : en effet, le premier joueur a la stratégie gagnante consistant à manger les épices, c'est-à-dire retirer un quelconque bâtonnet épicé, auquel cas son adversaire devra en retirer un fade, puis le premier joueur retire de nouveau un épicé, et ainsi de suite jusqu'à épuisement des fades auquel moment le second joueur perd. Dans « beaucoup » de cas, une position nulle s'obtient en complétant une telle position jusqu'à ce que le nombre total de bâtonnets fades égale le nombre total d'épicés : notamment, s'il y a une seule ligne fade, il est facile de voir que la condition de nullité est précisément que le nombre de bâtonnets de cette ligne soit égal au nombre total de bâtonnets épicés ; il en va de même si les épicés sont dans des lignes d'un seul bâtonnet chacune (i.e., de nouveau, quelle que soit la répartition des fades, la condition de nullité sera que les fades soient aussi nombreux que les épicés). Mais parfois, il faut plus de fades : par exemple, s'il y a exactement deux lignes de fades dont une avec un seul bâtonnet, la condition de nullité est que le nombre total de fades soit deux de plus que le nombre total d'épicés si toutes les lignes épicées ont un nombre pair de bâtonnets (alors que c'est égalité s'il y a une ligne épicée avec un nombre impair de bâtonnets) — je sais le démontrer, mais je ne sais pas le généraliser correctement. Je ne sais pas « expliquer » de façon satisfaisante le fait que (2,9;6,4) ou (3,8;6,4) ou (1,2,8;6,4) (et pas (2,8;6,4) ou (2,n;6,4) pour un quelconque autre n) soient des positions nulles. C'est mystérieux : je cherche encore le motif, mais je commence à douter de son existence…

Évidemment, le jeu de nim se prête à quantité d'autres variations (on pourrait interdire de reprendre des bâtonnets de la ligne qui vient d'être diminuée, ou bien seulement pour certaines lignes, ou avoir plusieurs sortes d'épices, que sais-je encore…). Mais celle-ci m'est venue à l'esprit dans le cours d'une réflexion plus générale, alors maintenant elle m'obsède.

[#] Le terme combinatoire est là pour insister sur le fait qu'il ne s'agit pas de la théorie des jeux dans le sens où on l'entend d'habitude (jeux à la von Neumann, équilibres de Nash, tout ça) mais de jeux à information complète. Pour la bible sur le sujet, voir le très exotique Winning Ways de Berlekamp, Conway et Guy. On peut dire que c'est Conway qui a inventé la théorie combinatoire des jeux partiaux, la théorie combinatoire des jeux impartiaux — théorie de Sprague-Grundy, par exemple — est plus ancienne.

[#2] Pour jouer à la version misère, si on connaît la stratégie optimale de la version normale du nim décrite plus loin, il suffit de jouer comme pour la version normale sauf lorsqu'on ne va laisser que des lignes avec (zéro ou) un bâtonnet : à ce moment-là on en laisse un de moins (ou un de plus).

[#3] Le choix du mot nul, pour les jeux où le second joueur a une stratégie gagnante, pourra sembler bizarre. L'explication est que si on fait la somme d'un tel jeu et d'un jeu G quelconque — la somme étant à comprendre au sens où chaque joueur peut jouer dans une composante quelconque de la somme — alors la somme en question a la même caractéristique que le jeu G. Ou encore, les positions nulles sont celles dont la fonction de Grundy est nulle.

↑Entry #1533 [older| permalink|newer] / ↑Entrée #1533 [précédente| permalien|suivante] ↑

↓Entry #1515 [older| permalink|newer] / ↓Entrée #1515 [précédente| permalien|suivante] ↓

(samedi)

Fragment littéraire gratuit #107 (la Contradiction)

Au mur figurait l'inscription énigmatique suivante :

…945497946690011303871870040893554688 = 2∞−1

(Les chiffres sur la gauche devenaient de plus en plus petits et rapidement illisibles.)

L'homme resta silencieux pendant qu'Ack et Bel s'assirent dans les chaises qu'il leur désigna. Il ne parla qu'après les avoir longuement dévisagé comme s'il cherchait à lire dans leur visage la clé d'un mystère ancien.

Venons-en au fait, Messieurs, car la menace est terrible. Vous avez combattu des ennemis terrifiants qui mettaient en danger la reine, le royaume ou l'humanité tout entière… L'agent Bel voulut protester mais l'homme ne lui en laissa pas le temps. Vous et vos collègues avec combattu des ennemis terrifiants, mais aucun tel que celui-ci.

Voyant que l'autre attendait une question, Ack demanda poliment : Que compte-t-il faire ? Expliquez-nous donc.

Il ne veut pas seulement devenir maître du monde, ou de l'univers tout entier, mais de tous les univers possibles. Notre savant fou n'est pas un vulgaire biologiste qui menacerait de dominer l'humanité, ni un chimiste prêt à faire sauter la Terre ; ni même un physicien qui aurait découvert comment repolariser le vide et transformer ainsi le cosmos en une mer de bébé-univers. Non, il est bien plus dangereux que tout ça !

Venez-en au fait, je vous en prie. Que projette-t-il ?

Notre mathématicien s'apprête à modifier la logique même du monde. Son pouvoir serait alors sans limites.

Vous voulez dire, si je comprends bien, demanda 006, cachant avec peine son incrédulité, qu'il veut devenir une sorte de dieu ?

Au moins un dieu, oui : un dieu absolument omnipotent. Je ne parle pas seulement de voyager dans le temps, de vaincre la mort ou de ce genre de choses. Bien plus que ça. Car le Dieu chrétien lui-même, si on en croit Thomas d'Aquin, ne peut pas faire ce qui est logiquement impossible — ea vero quæ contradictionem implicant sub divina omnipotentia non continentur (Somme théologique, première partie, question XXV). Néanmoins, Descartes, dans sa première Méditation

Toutes passionnantes que sont ces considérations théologiques, interrompit 007, elles ne sont pas ce qui nous amène aujourd'hui. Est-il possible que notre savant fou atteigne son but ?

Non, bien sûr : c'est logiquement impossible. Mais son but est précisément d'y arriver bien que ce soit logiquement impossible, puisqu'il cherche à s'affranchir de la logique. Après une pause : Notre avis est que l'arme qu'il construit — la Contradiction — amènera une destruction complète dans laquelle périront non seulement l'univers et tout ce qu'il contient, mais aussi l'idée de l'univers, les triangles équilatéraux et le nombre 42.

Si je comprends, vous dites que c'est impossible mais qu'il y a tout de même un risque qu'il y parvienne. Comment va-t-il s'y prendre ?

L'homme plaça sur la table devant lui un jeu de cartes, le coupa, en retourna la première carte et la montra aux agents secrets : elle représentait un vieil homme s'appuyant sur un bâton et tenant dans l'autre main un objet qui pouvait être une lanterne ou un sablier ; au-dessus de la carte, le chiffre IX. Le neuvième arcane majeur.

En 1873, expliqua-t-il, Charles Hermite — le mathématicien — déposa un pli cacheté à l'Académie des sciences, contenant un lemme essentiel pour atteindre la Contradiction. Ce pli a été détruit sans être ouvert à la mort de Hermite, suivant des instructions qu'il avait laissées dans son testament. Mais nous savons aussi que, profondément troublé par le résultat qu'il avait découvert, il l'avait communiqué à son élève Jules Tannery ; et que celui-ci laissa à sa mort une copie de la démonstration au jeune Albert Châtelet. La piste se perd alors.

Ce papier est ce que cherche notre savant ?

Ce papier est ce qu'il vous faut détruire, ainsi que toute trace, tout souvenir, de ce lemme. Car réunir cette démonstration avec les résultats que notre ennemi possède déjà, cela lui donnerait la Contradiction. La logique du monde dépend de vous, Messieurs !

↑Entry #1515 [older| permalink|newer] / ↑Entrée #1515 [précédente| permalien|suivante] ↑

↓Entry #1483 [older| permalink|newer] / ↓Entrée #1483 [précédente| permalien|suivante] ↓

(Tuesday)

Converting latitude+longitude to Universal Transverse Mercator

[The following is a highly technical note (i.e, odds are you don't want to read it at all), which I'm writing in the form of a blog post mainly because I'm too lazy to start another Web page—but it might get copied elsewhere eventually (if someone wants to dump it on Wikipedia, feel free to).]

I've long been intrigued by the oft-used (especially in conjunction with GPS units) UTM coordinates: explanations as to how UTM coordinates are defined exactly is very hard to find (the spherical case is simple enough, but the ellipsoidal one is a much tougher nut), and although there are many online tools (such as this one) to convert from latitude+longitude to UTM and back, they use black box formulæ, converging power series, and looking at the source will give you very little insight on what is going on. So here is an attempt at a mathematically precise definition (it took me a whole day of angry formula-crunching before I came up with something entirely correct, so I won't spare the details).

First of all, what is a transverse mercator projection? It can be defined by starting from a central meridian (UTM uses 60 possible central meridians, one for each UTM zone) and constructing the mathematically unique projection from a spheroid (= rotation of an ellipse about one of its axes) to a plane which is conformal (= preserves angles) and maps the central meridian to a straight line with constant scale. So here are the defining features of Universal Transverse Mercator:

  • It divides the Earth in 60 longitude zones, each 6° wide: zone 1 ranges from 180° to 174°W, zone 2 from 174°W to 168°W and so on through zone 30 from 6°W to 0°, zone 31 from 0° to 6°E, up to zone 60 from 174°E to 180°. Each zone's central meridian is halfway between the limiting longitudes (e.g., zone 33's central meridian is 15°E). UTM coordinates are given relative to a longitude zone and a hemisphere (North or South).
  • In each zone, UTM is strictly conformal (= preserves angles). In practice, this means that it does not distort shapes anywhere, or that the UTM grid is a square grid everywhere, and it appears square when displayed on any conformal map projection. (Note that this does not imply that the lines of the grid are aligned north-south and east-west! But they are always perpendicular.)
  • Each zone's central meridian (which receives an easting coordinate of 500000, see below) is mapped to a vertical line along which the vertical (northing) coordinate is simply proportional to distance (on the ellipsoid) along that meridian; however,
  • the scale along the central meridian is not 1:1, it is 9996:10000. In other words, two points lying at 10km's distance from one another on the central meridian will have vertical (northing) coordinates differing by only 9996. The reason for this is that the map scale increases on either side of the meridian (as is unavoidable when mapping a curve surface) and the 0.04% decrease at the center is chosen to make the average scale on a 6°-wide zone roughly 1:1.
  • The two UTM coordinates, easting (given first) and northing (given second) are orthogonal and measured in meters (after the transverse mercator projection and the 9996:10000 scale are applied). Easting is measured horizontally, with the central meridian having value 500000 (in practice, it can take values from 166021 to 833979 at the equator, the range being more narrow at higher latitudes). Northing is measured vertically, with the equator having value 0 in the northern hemisphere and 10000000 in the southern hemisphere; in the northern hemisphere, it ranges from 0 at the equator to 9328094 at 84°N on the central meridian, or even 9997965 if used all the way to the pole (but this normally isn't supposed to happen: beyond 84°N, and beyond 80°S Universal Polar Stereographic coordinates should be used instead of UTM).
  • UTM is (nowadays) almost exclusively used with the WGS84 ellipsoid: this has a semimajor axis of a = 6378137m (exactly) and a flattening of f = 1 − b/a = 1/298.257223563 (exactly).

This is a complete definition, however it isn't a very usable one because it lacks a description of how to use the conformal part of the definition to actually convert geodetic coordinates to Universal Transverse Mercator. If we were to use a spherical Earth model, computations would be easy enough:

↑Entry #1483 [older| permalink|newer] / ↑Entrée #1483 [précédente| permalien|suivante] ↑

↓Entry #1477 [older| permalink|newer] / ↓Entrée #1477 [précédente| permalien|suivante] ↓

(lundi)

Le taquin de Mathieu : le retour du fils de la vengeance

Bon, le taquin proposé dans l'entrée précédente était assez infaisable. Celui qui suit a le même groupe (toujours le groupe M24 de Mathieu) mais il est sans doute beaucoup plus faisable (et en tout cas plus amusant à essayer) :

Je ne propose pas une configuration précise à atteindre : je propose plutôt de mélanger le taquin (ce qui se fait en choisissant le lien « point d'interrogation à l'envers ») et de chercher ensuite à le remettre en ordre. Cette fois-ci le taquin a la forme de 7+1 colonnes sur 3 lignes : on peut cycler les lignes avec les flèches vers le haut et le bas, cycler les 7 colonnes principales (celle qui est séparée des autres reste alors fixe) avec les flèches vers la gauche et la droite, ou enfin appliquer un flip de Mathieu qui échange huit paires de cases coloriées de la même façon (cinq de ces huit paires sont contiguës : 02–03, 08–09, 11–14, 12–15 et 13–16 ; et les trois cases 22, 23, 24 de la colonne spéciale sont échangées avec 07, 21 et 20 respectivement). Cette présentation n'est peut-être pas encore totalement satisfaisante, mais elle n'a pas l'air complètement folle et on peut vaguement la retenir et même développer quelques techniques de jeu comme l'utilisation judicieuse de la colonne séparée et des deux cases échangeant en diagonale. Avant de jouer avec le taquin tout entier, on peut se familiariser avec le sous-groupe d'ordre 21 engendré par les flèches (ça c'est vraiment très facile), avec le sous-groupe d'ordre 54 engendré par les flèches verticales et le flip (c'est encore plutôt facile), et enfin avec le sous-groupe d'ordre 21504 engendré par les flèches horizontales et le flip (il permet par exemple d'échanger exactement les deux lignes d'en bas sans toucher à celle d'en haut, ce qui n'était pas évident a priori : voyez-vous comment ?). Quant aux 244823040 états du puzzle complet, j'arrive pour ma part à placer en gros deux cases où je veux, parfois trois, mais pas plus (le groupe M24 de Mathieu, je l'ai déjà expliqué, permet de placer cinq cases où on veut).

J'ai aussi essayé de rajouter la possibilité de jouer au clavier (avec les flèches et la touche insertion pour appliquer le flip), mais c'était probablement une mauvaise idée vu que la possibilité de capturer les touches en JavaScript est complètement bizarre et dépend horriblement du navigateur et marche mal même sur un navigateur bien connu comme Firefox (il y a plein de problèmes de focus).

↑Entry #1477 [older| permalink|newer] / ↑Entrée #1477 [précédente| permalien|suivante] ↑

↓Entry #1476 [older| permalink|newer] / ↓Entrée #1476 [précédente| permalien|suivante] ↓

(samedi)

Le taquin de Mathieu

À force de méditer sur les groupes simples finis, j'ai imaginé le puzzle suivant, que j'appelle le Taquin de Mathieu :

Votre but est d'inverser complètement les nombres, c'est-à-dire de mettre le 24 là où est le 01 initialement et vice versa, le 23 à la place du 02, le 22 à la place du 03, etc. Pour ça, on dispose d'essentiellement deux opérations : l'une consiste à décaler cycliquement les douze colonnes du puzzle (vers la droite ou la gauche), l'autre — que j'appellerai le flip de Mathieu — réalise simultanément huit échanges entre deux cases (chaque paire étant coloriée d'une même couleur pour qu'on les voie facilement), les huit autres cases (marquées en gris) restant fixes. Les trois symboles immédiatement en-dessous du tableau sont cliquables, et réalisent ces opérations (et celui encore en-dessous remet le puzzle à son état de départ). Arriver à la situation inverse demandée est faisable — mais ce n'est pas très facile.

Le puzzle a 244823040 états possibles — c'est considérablement moins qu'un Rubik's cube, pourtant il me semble possible qu'il soit, d'une certaine manière, plus difficile à résoudre : en effet, pour résoudre le Rubik's cube on va chercher à trouver certaines combinaisons qui font des opérations faciles à comprendre, alors que dans le groupe de Mathieu, en un certain sens, ces opérations n'existent pas (aucune opération ne peut laisser plus de huit points fixes, par exemple). Peut-être que je me trompe. Si vous y arrivez sans l'aide d'un ordinateur, dites-le-moi !

En tout cas, il s'agit d'une illustration d'une des choses que j'affirmais dans ma précédente entrée : si on cherche des puzzles de ce genre qui ne permettent pas de réaliser toutes les permutations possibles des pièces, ou au moins toutes les permutations paires, alors il n'y a en gros que celui-ci et un analogue sur douze pièces qui offrent une certaine liberté dans le mouvement des pièces (vous pouvez placer cinq pièces quelconques aux endroits que vous voulez, et c'est le maximum).

↑Entry #1476 [older| permalink|newer] / ↑Entrée #1476 [précédente| permalien|suivante] ↑

↓Entry #1475 [older| permalink|newer] / ↓Entrée #1475 [précédente| permalien|suivante] ↓

(dimanche)

Numérologie mathématique monstrueuse

Je l'ai mentionné il y a quelque temps, nous avons eu il y a un deux semaines à l'ENS un séminaire du mathématicien John McKay sur un phénomène paranormal mathématique appelé le Monstrous Moonshine : même si ce n'est pas ma spécialité, je voudrais dire un mot de ce qui tourne autour. (En fait, je viens de finir la lecture d'un petit livre de vulgarisation sur le sujet, Symmetry and the Monster de Mark Ronan, qui n'est pas mal du tout — même si c'est très insatisfaisant pour un mathématicien de lire des livres de vulgarisation adressés au grand public vu qu'on veut toujours en savoir plus.)

Il y a un débat récurrent autour de la question de savoir si les mathématiques sont découvertes ou inventées : même si la réponse n'a pas à être complètement d'un côté ou de l'autre, je pense que la plupart des mathématiciens eux-mêmes sont d'avis qu'elles sont découvertes, et assurément la théorie des groupes finis est un des domaines où la réponse découvertes s'impose le plus naturellement. Après une tentative (sans doute assez lamentable) pour expliquer à ma mère ce qu'est le Monstre, elle a commenté : les mathématiciens sont vraiment doués pour inventer des choses complètement farfelues — mais à mes yeux c'est une mésinterprétation complète de la réalité : personne n'a inventé le Monstre, il a été découvert par des gens qui ont été stupéfiés de le voir s'imposer à eux de cette façon. (Un des termes qui revient parfois dans la description du sentiment qu'ils ont eu en le découvrant est qu'il y avait something out there ; je ne saurais pas rendre ça en français, mais je trouve que c'est très fort.)

Il y a certainement quelque chose dans certaines branches des mathématiques, et notamment dans la théorie des groupes finis qui rappelle la fascination que les hommes ont pu avoir pour des pseudo-sciences comme la numérologie ou l'astrologie : je pense que c'est ça qui peut donner à des gens (y compris certains mathématiciens de branches plus éloignées, d'ailleurs !) l'impression qu'il s'agit de mathématiques un peu suspectes, où l'on étudie les propriétés magiques des nombres tels que 196883 ou 244823040. Je crois que des matheux un peu joueurs comme Conway s'en amusent beaucoup, en fait. La différence, c'est que les douze signes du zodiaque sont le résultat du hasard de configurations d'étoiles interprétées par les yeux d'astronomes anciens et qui auraient très bien pu être autrement alors que la table des caractères de M24 est quelque chose qui s'imposerait de la même façon à tous extra-terrestres ayant la curiosité de s'intéresser aux façons de réordonner des objets.

↑Entry #1475 [older| permalink|newer] / ↑Entrée #1475 [précédente| permalien|suivante] ↑

↓Entry #1469 [older| permalink|newer] / ↓Entrée #1469 [précédente| permalien|suivante] ↓

(mardi)

Une page se tourne

Maintenant c'est officiel :

Monsieur et Cher Collègue,

Suite à votre audition le 26 juin 2007 par la Commission de recrutement pour un emploi d'enseignant-chercheur à l'ENST en « Cryptographie », j'ai le plaisir de vous faire savoir que vous avez été classé premier.

Vous voudrez bien prendre contact avec […] afin de mettre en œuvre votre recrutement.

En vous félicitant pour ce succès, je vous prie de croire, Monsieur et Cher Collègue, en l'expression de mes salutations les meilleures.

Je quitte donc l'ENS l'an prochain pour devenir maître de conférences à l'ENST (je gagne une lettre de plus, quoi, et je me rapproche un peu de chez moi).

Avec un double défi : celui de faire de la recherche qui soit intéressante et de haut niveau à la fois mathématiquement et informatiquement. Informatiquement parce qu'on m'a recruté pour faire de la crypto et que je compte bien honorer ce devoir. Et mathématiquement parce qu'être mathématicien est mon rêve d'enfant et que je ne le lâcherai pas.

Mais on ne quitte pas sans une larme à l'œil un endroit qu'on a fréquenté assidûment pendant onze ans. Madame notre Directrice organisait justement aujourd'hui un pot pour le départ de ceux qui s'en vont (principalement des élèves, bien sûr, ceux de la promotion 2003, et j'en connais aussi beaucoup de cette année-là, qui commencent une thèse ou deviennent enseignants du secondaire), l'occasion de nous dire que nous serions toujours les bienvenus. Ça tombe bien, j'ai un copain dans cette École et il y a une bibliothèque de maths extrêmement bien fournie donc j'y serai sans doute encore souvent.


Comme une bonne nouvelle ne vient pas seule, j'apprends que le Bulletin de la Société Mathématique de France engage enfin la publication d'un article que j'y avais soumis en août 2004, et qui avait été accepté en janvier 2005. Les mathématiques ne sont pas trop pressées, mais trois ans c'est tout de même exceptionnellement long : la raison en est apparemment des difficultés techniques liées à une réorganisation de la chaîne de production du journal. J'espère tout de même que la revue sera datée de 2006, parce que sinon on risque de dire que le résultat de Madore (2007) a été ultérieurement généralisé par Hassett et Tschinkel (2006), ce qui me rend quand même un peu ridicule dans l'affaire.


Sinon, cela n'a pas de rapport, mais je viens de tomber sur une jolie suite d'entiers assez naturelle qui ne figurait pas encore dans l'encyclopédie des suites d'entiers de Sloane : j'ai donc proposé son ajout. J'espère qu'elle sera acceptée, parce que c'est quelque chose dont je suis assez fier que d'avoir fait rajouter quelques suites dans cette fabuleuse mine de numérologie scientifique (en l'occurrence, A033623, A046873, A051917 et A100002[#]).

Je propose donc cette nouvelle suite comme une énigme mathématique du jour (mais je serais vraiment très impressionné si quelqu'un la résolvait avant que la suite passe dans le Sloane) :

1, 1, 2, 10, 64, 596, 8056, 130432, 2534960, 59822884, 1718480368, 56754444440

Deux indications, tout de même, pour que ce ne soit pas complètement infaisable : premièrement, ça a un rapport avec les tableaux de Young (ou avec les représentations du groupe symétrique 𝔖n sur n objets), deuxièmement on peut considérer que c'est la continuation logique de A000041, A000085 et A000142.

Pour savoir la réponse, il suffira d'attendre que la suite soit ajoutée à l'encyclopédie…

[#] La A100002, d'ailleurs, malgré sa très grande simplicité (son mode de construction est tout à fait explicable à un enfant), a eu l'honneur d'attirer un peu sérieusement l'attention de Neil Sloane lui-même : je suppose que c'est pour ça qu'elle a eu droit à un numéro aussi spécial. Et par ailleurs elle produit une musique vraiment très intéressante.

↑Entry #1469 [older| permalink|newer] / ↑Entrée #1469 [précédente| permalien|suivante] ↑

↓Entry #1466 [older| permalink|newer] / ↓Entrée #1466 [précédente| permalien|suivante] ↓

(mercredi)

Séries de Hahn, théorèmes de Kedlaya et algébricité

La série annuelle des exposés de maîtrise (d'initiation à la recherche) des normaliens de première année vient de commencer, donc cette année encore je vais assister à un grand nombre d'exposés mathématiques (pour la plupart très intéressants) sur des sujets très variés. Aujourd'hui il y en a un qui a particulièrement retenu mon attention puisqu'il était question d'une de ces constructions mathématiques que je trouve particulièrement remarquables. Et qui m'est très chère puisque je l'avais redécouverte il y a quelques années sans savoir que c'était un objet classique (j'étais allé, tout excité, en parler à mon directeur de thèse qui m'avait aussitôt sorti un préprint qui expliquait ça et qui répondait à certains des problèmes que je m'étais posés à ce sujet) : il s'agit des séries de Hahn, ou de Mal'cev-Neumann (les deux noms se trouvent, et je ne sais pas si l'un est plus correct). Même si j'ai pour habitude de ne pas parler de maths sur ce blog[#] je vais tout de même essayer de décrire de quoi il s'agit et pourquoi je trouve ça remarquable. En supposant que le lecteur est un peu familier avec des notions de base d'algèbre (disons, séries formelles et extensions de corps).

Pour introduire cette notion je pourrais commencer par partir de l'anneau F[t] des polynômes en une indéterminée t à coefficients dans un corps F (qui sera, pour l'essentiel, un corps fini). Il s'agit de généraliser cette notion : une première généralisation, puisque les polynômes s'écrivent comme des sommes formelles de termes ci·ti pour i parcourant un nombre fini d'entiers naturels, serait d'autoriser les sommes infinies de cette forme, c'est-à-dire que i parcourt tous les entiers naturels ; on obtient ainsi l'anneau F[[t]] des séries formelles en l'indéterminée t. Par exemple, 1+t+t²+t³+⋯ (somme de tous les ti avec tous les coefficients ci égaux à 1) est une série formelle, qui est l'inverse de 1−t [#2]. Une nouvelle intéressante, c'est que F[[t]] est un anneau de valuation discrète : sans chercher à définir cette notion, disons qu'il n'y a en gros qu'une seule série formelle qui n'est pas inversible, c'est t elle-même, au sens où si on lui donne un inverse alors toute série formelle (non nulle) en aura un.

On introduit donc l'anneau F((t)) des séries de Laurent : il s'agit toujours des sommes formelles de termes de la forme ci·ti, sauf que cette fois-ci on permet à l'exposant i de prendre des valeurs négatives (puisqu'on veut inverser t, il faut bien introduire un t à la puissance −1). Bien sûr, toute somme de cette forme n'est pas légitime : par exemple, il n'y a pas de sens à donner[#3] à la somme des ti sur tous les entiers relatifs i. La bonne condition est de demander que les i qui supportent la somme (c'est-à-dire ceux pour lesquels le coefficient ci est non nul) soient bornés inférieurement, ou de façon équivalente qu'il n'y ait qu'un nombre fini de termes à exposants négatifs. Il se trouve qu'on obtient ainsi un corps (toute série de Laurent non nulle est inversible), qui est le corps des fractions de l'anneau des séries formelles.

Pour aller plus loin, on cherche à résoudre certaines équations algébriques. Par exemple, dans le corps des séries de Laurent, la série 1+t a une racine carrée (si F est de caractéristique différente de 2), solution de l'équation algébrique f²−(1+t)=0, à savoir f = 1 + (1/2)t − (1/8)t² + (1/16)t³ − (5/128)t4 + ⋯ (bref, le développement asymptotique connu). Une autre nouvelle intéressante, c'est que F[[t]] est un anneau (local) hensélien : sans chercher à définir cette notion, disons qu'il n'y a en gros qu'une série de Laurent qui n'a pas de racine carrée, cubique ou je ne sais quoi, c'est t elle-même.

On introduit donc l'anneau F((t1/∞)) des séries de Puiseux : il s'agit toujours des sommes formelles de termes de la forme ci·ti, sauf que cette fois-ci on permet à l'exposant i de prendre des valeurs rationnelles (puisqu'on donner des racines à t, il faut bien introduire un t à la puissance ½ ou autres). Bien sûr, toute somme de cette forme n'est pas légitime : de même que pour définir les séries de Laurent on avait imposé aux exposants supportant la série (c'est-à-dire intervenant effectivement dedans) d'être bornés inférieurement, de même, cette fois, on demandera qu'ils aient un dénominateur borné (ou, de façon équivalente, qu'il y ait un même dénominateur qui convienne pour tous les exposants) — on continue bien sûr d'imposer que les exposants soient eux-mêmes minorés. C'est-à-dire, si on préfère, qu'une série de Puiseux est une série de Laurent en t1/k pour un certain k (qui dépend de la série, mais qui doit convenir pour tous les coefficients de celle-ci). Il est trivial que la somme de deux séries de Puiseux est encore une série de Puiseux, et il est facile de vérifier que cela vaut aussi pour le produit (qui s'écrit formellement comme on le devine), et que les séries de Puiseux forment un corps.

↑Entry #1466 [older| permalink|newer] / ↑Entrée #1466 [précédente| permalien|suivante] ↑

↓Entry #1462 [older| permalink|newer] / ↓Entrée #1462 [précédente| permalien|suivante] ↓

(mercredi)

Devinette mathématique du jour

Quelle est la logique derrière les suites binaires :

…101000001100011110110111010111100010001001110011100010010011010100001001100010110110100010101000101100100001000101011001

et

001111100001011011010000100100011010010101011110001110000101000110011000101100010011001100100010100110010001000100000110…

? (Précisons que j'aurais pu poser la question pour chacune séparément : il y a une certaine beauté à les mettre ensemble, mais elle n'est pas du tout évidente.) Ce n'est pas la peine de chercher dans le Sloane, aucune des deux n'y est (et c'est tout à fait normal).

↑Entry #1462 [older| permalink|newer] / ↑Entrée #1462 [précédente| permalien|suivante] ↑

↓Entry #1456 [older| permalink|newer] / ↓Entrée #1456 [précédente| permalien|suivante] ↓

(mercredi)

Célébrité de la semaine

Ciel ! Mon voisin de bureau (pour quelques jours) est Pierre Deligne ! je ne vais vraiment pas oser lui parler.

↑Entry #1456 [older| permalink|newer] / ↑Entrée #1456 [précédente| permalien|suivante] ↑

↓Entry #1429 [older| permalink|newer] / ↓Entrée #1429 [précédente| permalien|suivante] ↓

(samedi)

Géométrie plane : I. Géométrie projective

Comme promis, je vais tenter, en une série de posts dans ce blog, de faire un peu de vulgarisation de la géométrie plane, et je commence par parler de la géométrie projective.

Alors, qu'est-ce que c'est que la géométrie projective ? C'est un terme qui fait souvent peur (par exemple à mes agrégatifs 😉), et je n'arrive pas à comprendre pourquoi : la géométrie projective, au contraire, c'est la plus simple qui soit, parce qu'il n'y est question ni d'angles ni de distances, ni même de droites parallèles, mais uniquement de droites qui se rencontrent et de points alignés. La géométrie projective, donc, c'est celle que vous faites si vous avez pour seul instrument une règle non graduée (et un papier et un crayon, d'accord) : vous pouvez relier deux points par une droite, vous pouvez trouver le point d'intersection de deux droites, mais vous ne pouvez rien faire d'autre. Cela semble très facile ? Pourtant, on entre déjà dans un monde assez riche.

Avant d'en dire plus, il faut que je torde le coup à un des trucs qu'on associe typiquement à la géométrie projective : les points à l'infini. Si vous voulez faire de la géométrie projective, disons, sur une feuille de papier (qui est plutôt un modèle de la géométrie euclidienne), vous allez régulièrement tomber sur des droites qui se coupent loin hors de votre feuille de papier : ça ce n'est pas grave, vous savez qu'elles se coupent quand même (enfin, ce sera peut-être pénible pour faire la figure si vous devez relier ce point d'intersection à un autre, mais il y a des astuces pour y arriver quand même) ; et parfois les droites seront carrément parallèles, c'est-à-dire qu'elles ne se coupent pas dans le monde euclidien. Mais en géométrie projective (plane, bien sûr), deux droites sont censées toujours se rencontrer : alors pour faire quand même de la géométrie projective à partir d'un truc euclidien, on rajoute un point fictif, qu'on appelle point à l'infini, dans chaque direction de droite possible, et c'est le point où se coupent toutes les droites parallèles ayant cette direction. Vous pouvez l'imaginer très très loin dans un sens, ou aussi bien dans l'autre puisque, après tout, seule la direction de la droite compte, pas son sens. Et on regroupe tous ces points à l'infini sur une droite fictive, la droite à l'infini. Mais notez bien que la notion d'être à l'infini n'existe pas en géométrie projective : pour celle-ci, ces points à l'infini ou cette droite à l'infini n'ont rien d'inhabituel ou de différent des autres, c'est uniquement parce qu'on cherche à représenter la géométrie projective dans un contexte euclidien (ou, plus exactement, affine) qu'il y a des choses qui partent à l'infini.

[Figure géométrique] [Figure géométrique] [Figure géométrique]Pour essayer de donner une première image de la géométrie projective, je commence par les trois figures ci-contre (à gauche). La première est une figure euclidienne typique : une grille régulière, avec cinq droites parallèles horizontales (rouges) régulièrement espacées, cinq droites parallèles verticales (vertes) qui leur sont perpendiculaires et qui sont aussi régulièrement espacées, et neuf droites diagonales (bleues) toujours régulièrement espacées. Mais, je répète, aucune de ces notions (parallèles, régulièrement espacées, perpendiculaires) n'est une notion projective. La figure de droite est, pourrait-on dire, la vision qu'a la géométrie projective de la figure de gauche : projectivement c'est exactement la même figure (cinq droites rouges qui concourent, cinq droites vertes qui concourent, et neuf droites bleues, correspondant aux diagonales des 25 intersections, qui concourent aussi). Pour passer d'une figure à l'autre, on a appliqué ce qu'on appelle une transformation projective ; simplement, le point de rencontre des droites vertes, ou des droites rouges, ou des droites bleues, qui étaient à l'infini sur la figure de gauche, a été ramené à distance finie (je répète que projectivement ceci n'a pas de sens, j'explique simplement en quoi diffèrent les représentations de la même figure projective) ; la droite en pointillés est celle qui était à la droite à l'infini sur la première figure : on a la confirmation que les points de rencontre des droites rouges, des droites vertes et des droites bleues sont bien alignés (ceci confirme qu'on a raison de décréter que les points à l'infini constituaient collectivement une droite à l'infini). Et la figure du milieu, alors ? Je l'ai mise là pour comparaison, c'est la vision qu'a de la figure de gauche la géométrie affine, dont je ne parle pas plus aujourd'hui. (Pour résumer, en géométrie euclidienne, les trois figures sont distinctes ; en géométrie affine, les deux de gauche sont la même et la troisième est distincte ; et en géométrie projective les trois sont la même.)

↑Entry #1429 [older| permalink|newer] / ↑Entrée #1429 [précédente| permalien|suivante] ↑

↓Entry #1428 [older| permalink|newer] / ↓Entrée #1428 [précédente| permalien|suivante] ↓

(mercredi)

Géométrie plane

On m'a fait remarquer que je n'essaie quasiment jamais, sur ce blog, et c'est dommage, de faire de la vulgarisation mathématique : il y a à cela toutes sortes de raisons, comme le fait idiot qu'il est très fastidieux d'écrire des formules mathématiques, même très simples, dans une page HTML (donc je préfère encore le format PDF dès qu'il s'agit de faire quelque chose d'un minimum sérieux) ; et puis il y a le fait fondamental que la vulgarisation du savoir scientifique est un art très difficile, surtout quand on sait qu'on sera lu aussi bien par des experts et par des gens qui n'y connaissent rien, et qu'on doit s'arranger pour présenter les choses de façon que les uns ne trouvent rien à redire (voire, trouvent ça intéressant) et que les autres comprennent quand même ! Je me contente donc le plus souvent de remarques rapides (et pas forcément très compréhensibles) lorsque je tombe sur quelque chose qui m'excite mathématiquement et qui soit vaguement communicable au profane.

Il y a cependant un sujet dont j'aimerais parler — mais je ne sais pas encore sous quelle forme, parce que je risque d'avoir nettement trop à en dire pour faire juste une entrée de blog, alors ce serait peut-être un feuilleton, que je rassemblerais plus tard dans une unique page HTML (avec encouragements à recopier ce qu'on veut sur Wikipédia). Il s'agit de la géométrie plane.

Pourquoi la géométrie ? Parce que d'une part c'est un sujet qui parle à tout le monde, qui ne rebute pas d'emblée, même les plus réfractaires aux mathématiques : c'est généralement là que la notion de démonstration passe le mieux. Mais d'autre part parce qu'on peut quand même trouver énormément de choses mathématiquement intéressantes à dire, autour de la méthode axiomatique, de la géométrie projective (vue à la Artin et Coxeter), autour de la géométrie algébrique, même. Et, tout simplement, parce que je suis géomètre (c'est mon métier de chercheur, et c'est aussi quelque chose sur quoi j'interviens comme enseignant dans le cadre de la prépa agreg à l'ENS).

[Figure géométrique]Malheureusement, qui dit géométrie dit jolies figures à regarder. Et, mine de rien, c'est bigrement pénible à réaliser correctement avec un ordinateur, des figures de géométrie. À titre d'exemple, prenez la figure ci-contre, censée illustrer le théorème de Desargues (qui assure que si deux triangles, coloriés sur la figure, ont les sommets portés par trois droites concourantes, vertes sur la figure, alors les points de rencontre des côtés correspondants, en bleu sur la figure, sont alignés — enfin, il y a plein de façons de lire la même figure, bien sûr, notamment comme la réciproque de la même chose), un des théorèmes fondamentaux de la géométrie projective. Ça n'a l'air de rien, mais ça a été un boulot important de la produire (surtout que je voulais à la fois satisfaire mon idée de comment les points devaient être placés et l'exactitude des relations mathématiques d'incidence).

Et il n'y a pas que ça : il y a aussi la question quel format graphique utiliser ?. En l'occurrence, cette image est à l'origine un fichier SVG, un format d'images vectoriel : seulement, je ne sais pas dans quelle mesure ce format est nativement compris par les navigateurs Web typiques, donc, ci-contre, j'ai utilisé une image PNG plutôt que de mettre directement le SVG. Mais si vous cliquez sur l'image (faites-le !), vous obtiendrez le SVG, ce qui permettra de savoir si votre navigateur gère ce format (normalement l'image devrait être la même).

Voilà que je n'ai encore rien dit et qu'il faut déjà que je m'arrête. Mais je précise déjà quel serait mon but : il ne s'agirait pas vraiment de parler d'un sujet précis en géométrie, mais plutôt d'expliquer de façon compréhensible au non-mathématicien quels sont, dans l'esprit du programme d'Erlangen de Klein, les différents niveaux d'étude qu'on peut faire de la géométrie plane et des structures qui l'enrichissent, que je pourrais résumer par le tableau suivant :

Géométrie projective
d=8
Géométrie elliptique
d=3
Géométrie affine
d=6
Géométrie hyperbolique
d=3
Géométrie euclidienne
d=3 (voire 4)

Pour dire les choses très succinctement, plus on descend dans ce tableau plus on rajoute de la structure : alors que la géométrie projective ne connaît que les droites et les points et une relation d'incidence (un point est sur une droite, une droite passe par un point), la géométrie affine a une notion de droites parallèles, et la géométrie euclidienne a une notion de distances et d'angles. Du coup, cela diminue les transformations qui préservent cette structure : le nombre d que j'ai indiqué dans le tableau mesure le nombre de degrés de liberté dont on dispose encore, compte tenu de la structure imposée. Et de gauche à droite, très grossièrement, on passe de la géométrie elliptique où deux droites ne sont jamais parallèles, à la géométrie affine/euclidienne où c'est un cas possible mais limite, et à la géométrie hyperbolique où deux droites peuvent être, en quelque sorte, plus que parallèles.

↑Entry #1428 [older| permalink|newer] / ↑Entrée #1428 [précédente| permalien|suivante] ↑

↓Entry #1396 [older| permalink|newer] / ↓Entrée #1396 [précédente| permalien|suivante] ↓

(mardi)

Encore une curiosité mathématique

Comme je l'ai signalé à bien des reprises, mes goûts mathématiques sont assez portés vers les objets élégants et insolites qui pourraient figurer dans un musée des curiosités du paradis platonique. Parmi mes préférés, vu mon domaine de recherche, figure évidemment le( graphe de)s vingt-sept droites sur la surface cubique : c'est en effet quelque chose d'assez frappant que, même si les surfaces cubiques ne sont pas toutes identiques, toutes (enfin, toutes celles qui sont lisses) ont 27 droites géométriques tracées dessus et que la relation d'incidence sur ces droites (c'est-à-dire lesquelles coupent lesquelles) et toujours la même ; elles définissent donc une structure combinatoire (un graphe à 27 sommets, chacun étant relié à 10 autres) unique et remarquable[#]. Mais j'ai appris qu'il existe un polyèdre (le polyèdre 221 de Schoute) qui code de façon extrêmement élégante la structure de ces 27 droites : il s'agit d'un polyèdre (convexe) à faces carrées, ayant 27 sommets, correspondant aux 27 droites sur la surface cubique, de sorte que deux sommets sont à distance 1 si les droites correspondantes ne se coupent pas, et √2 si elles se coupent, et que les 45 triangles équilatéraux de côté √2 définis par trois droites deux à deux sécantes ont tous le même centre ; je ne peux malheureusement pas le tracer puisqu'il vit dans un espace euclidien de dimension 6. Il est toutefois très facile de donner les coordonnées de ses sommets : en représentant des points de l'espace euclidien de dimension 6 comme des triplets de nombres complexes, ce sont les (0,ωa,−ωb), les (−ωb,0,ωa) et les (ωa,−ωb,0), où a et b varient entre 1 et 3 et où ω est une racine cubique primitive de l'unité.

[#] Rien que son groupe d'automorphismes, le groupe de Weyl d'un système de racines de type E6, à 51840 éléments, est un objet remarquable en lui-même, et qui apparaît à des endroits inattendus des mathématiques.

↑Entry #1396 [older| permalink|newer] / ↑Entrée #1396 [précédente| permalien|suivante] ↑

↓Entry #1390 [older| permalink|newer] / ↓Entrée #1390 [précédente| permalien|suivante] ↓

(lundi)

De quelle science relèvent ces énigmes ?

On m'a posé aujourd'hui l'énigme suivante :

Le cruel Docteur No a capturé 100 mathématiciens pour les soumettre à une épreuve démoniaque. Il a placé (dans un ordre connu de lui seul) le nom de chacun des 100 mathématiciens dans autant de boîtes numérotées de 1 à 100. Après avoir permis aux mathématiciens de se concerter, il va les soumettre à son épreuve dont il leur communique les termes : il empêchera toute communication entre eux et les emmènera, dans un certain ordre, dans la pièce où se trouvent les 100 boîtes. Lorsqu'un mathématicien est dans la pièce, il pourra ouvrir une boîte de son choix pour lire le nom qui y figure, puis, s'il le souhaite, en ouvrir une autre, puis éventuellement une autre, et ainsi de suite jusqu'à 50 boîtes au maximum. (Les boîtes sont, bien entendu, refermées avant le passage du prochain mathématicien, puisque toute communication est interdite après la concertation initiale.) Le but de chaque mathématicien, lorsqu'il passe dans la pièce contenant les boîtes, est d'ouvrir la boîte contenant son nom (parmi les ≤50 boîtes qu'il peut ouvrir). Si chaque mathématicien a réussi à ouvrir la boîte contenant son nom, alors le Docteur No les libérera tous. Si ne serait-ce qu'un seul n'a pas réussi, alors le Docteur No tuera tous les mathématiciens avec ses tortures particulièrement raffinées.

Si chaque mathématicien ouvrait bêtement 50 boîtes au hasard, il aurait une chance sur deux de voir son nom dans l'une d'elles, et la probabilité que tous les mathématiciens réussissent le test (donc soient libérés) serait d'une chance sur 2 puissance 100 (en gros une chance sur un quintillion, ce qui ne pèse pas très lourd). Mais comme ce sont des mathématiciens, ils vont trouver une solution bien plus intelligente. 😎 En fait, ils élaborent une stratégie qui leur permet d'avoir plus de 30% de chances de s'en sortir. Comment font-ils ?

[Ajout () : voir ici.]

Mon propos n'est pas de donner la réponse à cette énigme (qui m'a demandé un bon moment de réflexion, d'ailleurs), je le ferai éventuellement plus tard (mais sans doute quelqu'un se dévouera-t-il dans les commentaires), mais de faire le parallèle avec d'autres. Il y a en effet toutes sortes d'énigmes bien connues dans ce genre, et qui font souvent appel au Docteur No et à nombre variable de mathématiciens.

↑Entry #1390 [older| permalink|newer] / ↑Entrée #1390 [précédente| permalien|suivante] ↑

↓Entry #1389 [older| permalink|newer] / ↓Entrée #1389 [précédente| permalien|suivante] ↓

(jeudi)

Nouvelles mathématiques

OK, ça fait une semaine que je n'ai rien écrit, et pourtant je me me suis pas pendu. Je plaide coupable, tout ça tout ça… J'ai fait des maths. ☺️

La situation de ma recherche n'est pas ce que je croyais : j'ai réussi à contourner les problèmes (1) et (2), et à démontrer un résultat vaguement intéressant en utilisant un théorème un peu différent et en considérant une situation un peu plus générale pour laquelle le problème n'est pas trivial. Tout est pour le mieux ? Pas tant que ça : le lemme technique que j'avais démontré, lui, s'avère être déjà connu (un lemme essentiellement équivalent est démontré dans un article de 1986, au moins, que mon directeur de thèse a déniché). Certes, c'est satisfaisant d'avoir redémontré indépendamment quelque chose de déjà connu (cela veut dire que je suis, d'une certaine manière, « sur la bonne piste »), mais cela fait aussi que mon résultat final, même s'il est vaguement intéressant, est quand même trop simple pour en faire un article (malheureusement, on ne juge pas les résultats uniquement à leur énoncé mais aussi à la longueur de leur démonstration). Solution possible : le généraliser, en utilisant une version plus forte du lemme qui a été démontrée depuis ; mais ça risque d'être extrêmement difficile, et en tout cas je vais devoir potasser beaucoup de grothendieckeries (la cohomologie étale de variétés singulières) avant d'avoir la moindre chance d'y arriver.

En attendant, je dis un mot d'un résultat (« classique », si j'ose dire) sur lequel je suis tombé par hasard dans un livre, qui a un rapport lointain avec ce sur quoi je réfléchis en ce moment, et que je trouve extrêmement joli. J'ai déjà dû raconter que j'éprouvais une fascination particulière, en mathématiques, non tant pour les théorèmes que pour certains objets remarquables et particulièrement élégants que l'on peut admirer dans, pour ainsi dire, le musée des curiosités du paradis platonique. En l'occurrence :

La variété sécante de la surface de Veronese dans P5 est une hypersurface cubique.

Alors j'essaie d'expliquer ce que ce charabia veut-dire. D'abord, Veronese, ce n'est pas le peintre, c'est le mathématicien. La surface de Veronese (dans P5, i.e., dans l'espace (projectif) de dimension 5) est un très joli objet mathématique, dont il est malheureusement difficile de donner une image vu qu'elle habite un espace à cinq dimensions (on peut cependant en représenter une projection, ou surface de Steiner) : il s'agit de la surface définie paramétriquement par les équations (v,w)↦(v²,w²,vw,w,v), autrement dit, l'image du plan par le système complet de degré deux, (sauf qu'il est plus commode et plus élégant de travailler dans l'espace projectif, c'est-à-dire en rajoutant les points à l'infini, auquel cas le paramétrage devient (U:V:W)↦(U²:V²:W²:VW:UW:UV) en coordonnées homogènes, et on voit bien apparaître tous les monômes de degré 2). Elle a notamment la propriété remarquable que n'importe quelle conique du plan en est une section hyperplane (i.e. : si vous coupez la surface de Veronese, qui vit dans l'espace de dimension 5, par un hyperplan de dimension 4, vous obtenez l'image — par le paramétrage ci-dessus — d'une conique plane, et toute conique s'obtient de la sorte) ; et si vous l'intersectez avec un espace linéaire de dimension 3, il reste quatre points dans l'espace (on dit donc que la surface de Veronese en question est de degré 4).

Maintenant, la variété sécante (de la surface de Veronese), c'est la réunion de toutes les droites passant par deux points de la surface (et les limites de telles droites). Comme on part d'une surface, donc d'un objet de dimension 2, et que pour chaque couple de points sur cette surface on prend la droite qui les relie, qui est de dimension 1, on s'attend à ce qu'en réunissant ces droites on obtienne un truc de dimension 2+2+1=5, donc tout l'espace. Eh bien non ! En fait on n'obtient qu'un truc de dimension 4, parce que n'importe quel point de la réunion est situé, en fait, sur une famille à un paramètre de telles droites (donc il y a une dimension qui tombe). Si vous voyez dans l'espace à cinq dimensions (ce qui n'est pas mon cas, hélas) ça vous fait une configuration géométrique extrêmement jolie. Et ce truc de dimension 4, cette variété sécante à la surface de Veronese, a le bon goût d'être une hypersurface cubique (une droite générale de P5 rencontre en trois points des droites reliant deux points de la surface de Veronese) ! Algébriquement, cela correspond à la relation cubique remarquable suivante : (AU²+BU′²)·(AV²+BV′²)·(AW²+BW′²) + 2(AVW+BVW′)·(AUW+BUW′)·(AUV+BUV′) = (AU²+BU′²)·(AVW+BVW′)² + (AV²+BV′²)·(AUW+BUW′)² + (AW²+BW′²)·(AUV+BUV′)², entre les six quantités AU²+BU′², AV²+BV′², AW²+BW′², AVW+BVW′, AUW+BUW′ et AUV+BUV′. Cette hypersurface cubique a pour points singuliers exactement les points de la surface de Veronese. Si on la coupe par un hyperplan général, on obtient une hypersurface cubique qui est la variété sécante d'une courbe rationnelle normale de degré 4 (qui sont ses points singuliers). Si on la coupe par un espace linéaire de dimension 3 général, on obtient une surface cubique de Cayley, la seule surface cubique ayant quatre points singuliers (donc si on veut, l'hypersurface de dimension 4 dont on parle est la réunion de toutes les surfaces de Cayley ayant pour quatre points singuliers n'importe quels quatre points sur la surface de Veronese).

Bref, voilà de la très jolie géométrie, où mes « yeux » de géomètre algébriste me permettent de voir quelque chose qui, vivant en dimension 5, n'est pas évident à visualiser, mais dont je perçois quand même la beauté.

↑Entry #1389 [older| permalink|newer] / ↑Entrée #1389 [précédente| permalien|suivante] ↑

↓Entry #1388 [older| permalink|newer] / ↓Entrée #1388 [précédente| permalien|suivante] ↓

(mercredi)

Les maths, c'est trop zinjuste

J'ai passé des semaines à réfléchir sur un problème, à élaborer une stratégie de démonstration, à commencer à la rédiger, à me rendre compte d'erreurs, à les corriger, à trouver des erreurs dans ma façon de corriger les erreurs, à contourner ces erreurs, à croire que je touchais au but, pour finalement m'apercevoir aujourd'hui que (1) le résultat principal sur lequel je comptais appuyer toute ma démonstration est faux (je m'étais trompé en croyant me souvenir d'un théorème censément bien connu) et (2) de toute façon, le problème sur lequel je réfléchissais admet une réponse en deux lignes complètement évidente (i.e., le problème lui-même est sans intérêt). Je ne sais pas ce qui est le plus frustrant, entre (1) et (2), d'ailleurs.

Je me retrouve avec sur les bras un lemme technique qui m'a coûté très cher et qui a l'air de ne pouvoir servir dans aucun autre contexte que celui qui s'avère sans objet.

Quelqu'un a une corde, pour que je puisse me pendre ?

↑Entry #1388 [older| permalink|newer] / ↑Entrée #1388 [précédente| permalien|suivante] ↑

↓Entry #1372 [older| permalink|newer] / ↓Entrée #1372 [précédente| permalien|suivante] ↓

(lundi)

Le DVD Surfaces cubiques est arrivé !

J'en avais déjà dit un mot, le voici arrivé : le DVD Surfaces cubiques, avec plein de jolies animations, est prêt à être téléchargé.

Malheureusement, je n'ai pas encore réussi à mettre les mains sur un lecteur de DVD de salon (avec une vraie télévision dessus, et tout et tout) pour le tester… (Enfin, plus exactement, j'ai pu faire un essai et ça n'a pas marché, mais c'était très vraisemblablement un problème de support, i.e., un DVD+R mal finalisé, et pas un problème avec l'image elle-même, donc ça ne me dit rien.) Je serais donc reconnaissant à quelqu'un qui pourrait faire l'essai et me confirmer qu'il marche bien : d'ici là, je m'abstiens de faire trop de publicité pour ce truc, puisque s'il s'avère qu'il ne marche pas bien sur les lecteurs de salon je devrai le refaire.

↑Entry #1372 [older| permalink|newer] / ↑Entrée #1372 [précédente| permalien|suivante] ↑

↓Entry #1370 [older| permalink|newer] / ↓Entrée #1370 [précédente| permalien|suivante] ↓

(mercredi)

Un DVD artistico-mathématique

J'ai eu une idée saugrenue : à force de faire des vidéos de surfaces cubiques, je vais en réaliser un DVD. En effet, je n'arrête pas, en ce moment, de calculer des vidéos de ce genre : pas seulement des surfaces cubiques qui tournent, mais aussi qui se déforment les unes en les autres, des singularités qui se créent ou qui fusionnent, etc. Au début, c'était pour m'aider à me faire une représentation mentale des singularités possibles, mais il y aussi un aspect artistique parce que certaines de ces vidéos sont extrêmement jolies. À vrai dire, même en ayant fait une thèse sur le sujet, je ne me rendais pas compte à quel point ces surfaces de degré trois admettaient une incroyable richesse de formes, donc je m'émerveille de voir tout ce que j'arrive à faire voir.

Bon, la réalisation technique risque de poser encore quelques problèmes. Pour l'instant, je calcule des animations de 300 images que je fais répéter 5 fois, ce qui fait une vidéo d'une minute chacune, qui met autour d'une demi-heure à calculer — enfin, je peux facilement en faire deux en parallèle puisque mon ordinateur est un dual-core, et je pourrais faire plus en faisant appel à d'autres machines, donc ce n'est pas vraiment ça le facteur limitant. Ce qui sera plus délicat, si je veux faire un « vrai » DVD (lisible sur un lecteur de salon) ce sera de faire une structure de menus, de rajouter des explications, et de trouver les foutues options à utiliser (dans mencoder) pour faire une vidéo lisible partout.

Ensuite il faudra voir comment distribuer le résultat, aussi : j'ouvrirai un torrent, mais ce serait bien de pouvoir proposer à des gens intéressés d'en avoir une version gravée.

↑Entry #1370 [older| permalink|newer] / ↑Entrée #1370 [précédente| permalien|suivante] ↑

↓Entry #1367 [older| permalink|newer] / ↓Entrée #1367 [précédente| permalien|suivante] ↓

(jeudi)

Encore des jolies images de surfaces cubiques

[Surface cubique de Clebsch]J'avais déjà réalisé des images de surfaces cubiques lisses, comme celle qui décore cette entrée ou les deux que j'ai mises au frontispice de ma thèse ; mais cette fois, pour m'aider à lire le texte de Cayley dont je parlais hier j'ai fait quelques vidéos de surfaces cubiques ayant des singularités. Vous pouvez donc admirer :

Ces vidéos sont au format AVI contenant du codec DivX;-). Si vous ne trouvez pas mieux, utilisez VLC pour les lire.

↑Entry #1367 [older| permalink|newer] / ↑Entrée #1367 [précédente| permalien|suivante] ↑

↓Entry #1366 [older| permalink|newer] / ↓Entrée #1366 [précédente| permalien|suivante] ↓

(mercredi)

Un peu d'archéologie mathématique

On raconte souvent que les mathématiques sont des choses qui sont éternellement vraies et que les textes mathématiques ne vieillisent donc pas… c'est un peu exagéré.

J'avais besoin de résultats sur les types de singularités possibles pour une surface cubique. De fil en aiguille, j'ai remonté des références jusqu'à un mémoire d'Arthur Cayley de 1869 où il classifie complètement (en vingt-trois cas) les lieux singuliers possibles d'une surface cubique, et je me suis dit que j'allais essayer de regarder ça (après tout, 1869, ai-je pensé, ce n'est pas si vieux). Premier souci, je n'avais pas le titre exact du mémoire en question (seulement une référence à Cayley's famous memoir on cubic surfaces ; de fait, il s'avère que le titre exact est bien : A Memoir on Cubic Surfaces, comme quoi M. Cayley n'avait pas énormément d'originalité dans les choix de ses titres). Je rentre juste le nom d'auteur dans le catalogue de notre bibliothèque, et il me renvoie sur les œuvres complètes du Monsieur, lesquelles sont en réserve, en 13 volumes plus un volume d'index. Je demande donc à la bibliothécaire si je peux consulter le volume d'index, mais au début elle ne le trouvait pas (probablement parce que c'est un volume un peu plus petit que les autres et qu'on vient de déménager récemment donc peut-être la réserve n'est-elle pas encore bien rangée) : elle m'a donc suggéré quelques pistes pour trouver une version numérisée sur Internet. De fait, on en a trouvé une, dans les collections mathématiques historiques de l'Université du Michigan ; mais, de toute manière, en cherchant un peu plus, on a fini par me retrouver l'index dans la réserve, et j'ai pu faire sortir le bon volume (le VI, pour être précis). Qui, bien sûr, n'était même pas coupé : donc j'ai dû déranger de nouveau les bibliothécaires pour qu'on me trouve un coupe-papier (et ça non plus, ça n'a pas été facile).

Bon, comme ça n'a pas été facile, je vais moi-même mettre ce « fameux » mémoire sur Internet (puisque Cayley est mort il y a nettement plus de 70 ans, ça devrait être bon, et je doute que l'Université du Michigan m'ennuie au sujet de la façon dont les pages sont scanées) : voici donc A Memoir on Cubic Surfaces, by Arthur Cayley (Phil. Trans. R. Soc. London, 159 (1869), 231–326) [PDF, 6.1Mo], et une version réduite à 2 pages sur 1 plus commode à imprimer sur du papier A4. (J'aurais voulu mettre une version DjVu plutôt que PDF, parce que c'est quand même plus adapté pour un document scané, mais je ne trouve rien pour convertir l'un en l'autre de façon propre.)

Mais bon, une fois qu'on a l'article, les difficultés ne sont pas finies : les notations et la terminologie mathématique utilisées par Cayley sont, pour une bonne part, incompréhensibles pour un géomètre algébriste moderne : par exemple, ce qu'il appelle un point de type C2 s'appellerait aujourd'hui une singularité de type A1, je ne suis pas sûr de comprendre ce qu'il appelle un bord torsal d'un point biplanaire, ni ce qu'est la courbe spinodale (même si je devine vaguement que c'est la courbe des points dont le plan tangent intersecte la cubique en une courbe ayant un cusp en ce point). Je ne sais pas si je vais vraiment pouvoir en tirer quelque chose, de cet article.

Je me disais que je pourrais peut-être faire des graphismes de ces différents types de singularités, mais, évidemment, quelqu'un y a pensé avant : il y a même un site Web consacré aux surfaces singulières, avec visualisation en Java. Et pour quelque chose de plus traditionnel, je suis sûr qu'en cherchant un peu à l'IHP ou au Palais de la Découverte on pourrait me trouver des jolis modèles poussiéreux comme on en faisait autrefois représentant « en dur » toutes sortes de surfaces cubiques singulières.

Pour en savoir plus sur l'histoire des surfaces cubiques, voyez ici.

↑Entry #1366 [older| permalink|newer] / ↑Entrée #1366 [précédente| permalien|suivante] ↑

↓Entry #1340 [older| permalink|newer] / ↓Entrée #1340 [précédente| permalien|suivante] ↓

(Sunday)

A counterexample to the Hodge conjecture?

I can't tell from a superficial inspection whether this is correct (such spectacular claims require a great deal of prudence, although superficially it seems to make sense, and the authors obviously aren't crackpots); but if it is, (a) its authors might be eligible for a $1000000 prize and (b) a number of people I know will be quite disappointed.

Update: The authors just posted a revised version of the article, with the following comment: We no longer claim to have disproved the Hodge conjecture. Section 5 is deleted except for 5.3, and Theorems 6.2,6.3 are deleted.

↑Entry #1340 [older| permalink|newer] / ↑Entrée #1340 [précédente| permalien|suivante] ↑

↓Entry #1324 [older| permalink|newer] / ↓Entrée #1324 [précédente| permalien|suivante] ↓

(mercredi)

Conformations du cyclohexane

Petit exercice de géométrie dans l'espace (a priori compréhensible, et pourquoi pas résoluble, par n'importe quel lycéen) qui me semble particulièrement joli :

Soient a et b deux longueurs. On considère (dans l'espace euclidien de dimension 3, donc) un triangle équilatéral P0 P2 P4 de côté 2a. On appelle respectivement C1, C3 et C5 les cercles de rayon b ayant pour centres respectifs les milieux de [P0 P2], [P2 P4] et [P4 P0] et pour axes respecifs les droites portant ces mêmes segments. (Ainsi, si P1 est un point quelconque de C1, par exemple, alors les distances P0 P1 et P1 P2 valent toutes les deux ca²+b²=c².)

Supposons maintenant que P1 soit un point de C1 tel qu'il existe deux points distincts sur C3 (et donc aussi sur C5) à distance 2a de lui. Appelons P3 un de ces points, et P5 le point de C5 qui n'est pas « le même » (j'espère qu'on me comprend[#]).

Montrer que P1 P3 P5 est équilatéral (c'est-à-dire que la distance de P3 à P5 est elle aussi 2a).

J'ai une solution purement calculatoire (que je ne présenterai[#2] pas ici, parce qu'elle est assez fastidieuse). Je félicite d'avance celui qui me trouvera une démonstration courte, élégante et purement géométrique (sans calcul) de ce fait.

J'ai trouvé ce truc en digérant un oral d'agreg auquel j'ai assisté hier. Je laisse donc en question subsidiaire le problème de savoir quel est le rapport entre cet exercice et le titre de l'entrée (les conformations du cyclohexane, et notamment la question de la rigidité d'une conformation : pour fixer les idées, on appellera cyclohexane un hexagone pas nécessairement plan, dans l'espace, dont tous les côtés valent c et dont tous les angles valent un certain angle φ, avec mettons cos(φ/2)=b/c et sin(φ/2)=a/c).

Il faut essayer d'imaginer la cinématique de la chose : on fixe le triangle équilatéral P0 P2 P4 et le triangle équilatéral P1 P3 P5, de même côté, bouge en gardant constantes toutes les distances P0 P1, P1 P2, P2 P3 jusqu'à P5 P0. J'ai vaguement montré, aussi, qu'on peut passer continûment de l'une à l'autre des deux configurations possibles pour P1 donné (le mouvement du triangle P1 P3 P5 suit un cycle que j'aimerais bien voir illustré graphiquement).

Selon toute probabilité, ce truc porte le nom d'un illustre mathématicien d'un siècle passé, mais je ne sais pas lequel.

PS : J'ai posé la question sur Usenet, on verra bien si quelqu'un trouve une réponse.

Mise à jour (2006-07-21) : Pierre Dehornoy a trouvé une solution, que je reproduis ici :

Soit r la rotation autour de l'axe (P1 P4) qui envoie P0 en P2. Elle envoie alors le milieu de [P0 P4] sur celui de [P2 P4], donc elle envoie C5 sur C3. Comme P1 est sur l'axe de r, et que P1 P5 = P1 P3, r envoie P5 sur P3 (et non sur l'autre point qu'on a pas choisit, ca ca se voit en regardant l'ordre des points sur leur cercle). r envoie donc P0 en P2 et P5 en P3.

Or ce qu'on veut montrer c'est P3 P5 = P0 P2, il suffit donc de montrer que la distance de P5 à l'axe de r est la même que celle de P0 à ce même axe.

Comme P1 P5 = P4 P0 = 2a, et que P1 et P4 sont sur l'axe, il suffit de montrer que l'angle P4 P1 P5 est égal à l'angle P1 P4 P0. Et pour cela une condition suffisante est que les triangles P4 P1 P5 et P1 P4 P0 soient isométriques dans cet ordre. Or on a P4 P1 = P1 P4, P1 P5 = P4 P0 = 2a et P5 P4 = P0 P1 = c.

[#] S'il y a vraiment besoin d'explication : P3 et P5 sont deux points de C3 et C5 respectivement situés chacun à distance 2a de P1, et on suppose que P5 n'est pas le point de C5 obtenu à partir de P3 par symétrie par rapport au plan contenant C1.

[#2] Voici cependant le principe : on écrit le paramétrage rationnel standard (c'est-à-dire du style ((1-u²)/(1+u²), (2u)/(1+u²))), en fonction d'une variable u disons, du point P1 décrivant le cercle C1, et pareil avec des variables v et w pour les points P3 et P5. On appelle P(u,v) le polynôme, de degré 2 en chacune de ses variables, tel que P(u,v)=0 exprime le fait que la distance P1 P3 vaille 2a, et bien sûr P(u,w)=0 exprime de même le fait que P1 P5 vaille 2a. Il s'agit donc de vérifier que si v et w sont les deux racines (à u fixé) de l'équation quadratique P(u,t)=0 (en t), alors automatiquement P(v,w)=0. Or le polynôme P est symétrique en ses deux variables, donc le fait que P(v,w)=0 se vérifie en connaissant la somme et le produit de v et w, et ces deux quantités sont connues, si v et w sont les deux racines de P(u,t)=0, en fonction des coefficients de cette équation, qui sont des polynômes de u. Il y a donc simplement un calcul facile, quoique fastidieux, à mener pour arriver au bout.

↑Entry #1324 [older| permalink|newer] / ↑Entrée #1324 [précédente| permalien|suivante] ↑

↓Entry #1323 [older| permalink|newer] / ↓Entrée #1323 [précédente| permalien|suivante] ↓

(mardi)

Oraux d'agreg

Je reviens du lycée Marcelin Berthelot de Saint-Maur des Fossés où se déroulent (pour encore un jour) les oraux de l'agreg de maths. Il y a cette année une nouvelle option à l'agreg de maths, algèbre et calcul formel (il s'agit d'une épreuve de modélisation, c'est-à-dire en gros de commentaire de texte avec simulation sur ordinateur), et comme je m'occupe de la préparation à celle-ci pour l'ENS, je joue l'espion en assistant à quelques oraux pour apprendre un peu les sujets des textes proposés par le jury (on ne peut pas voir les textes eux-mêmes, et les candidats doivent les rendre, mais les oraux sont publics et quand on y assiste on a une bonne idée de quoi ça parle — même s'il est interdit de prendre des notes, on a évidemment le droit de retenir tout ce qu'on veut dans sa tête).

Certains textes ont manifestement été préparés à la hâte. D'autres, en revanche, renferment des petites surprises : un de nos agrégatifs, par exemple, est tombé sur un texte sur les canons en musique (je crois que je devine d'où il est sorti, celui-là, il va falloir que je ré-épluche quelques numéros de la Gazette des mathématiciens) ; et un oral auquel j'ai assisté parlait des conformations de la molécule de cyclohexane (ça peut surprendre pour un texte de maths et surtout d'algèbre, mais en fait il s'agit d'un prétexte pour regarder la géométrie, sous l'angle de la géométrie algébrique, des hexagones dans l'espace dont tous les côtés ont la même longueur et tous les angles la même valeur). Le jury en question était celui présidé par Jean-François Mestre, un mathématicien qui est aussi très bon pédagogue (je le sais parce que c'est lui qui donnait les cours d'algèbre quand j'étais en première année à l'ENS) et qui aime parfois jouer au mathémagicien, si j'ose dire, c'est-à-dire montrer un phénomène mathématique sous forme un peu surprenante — voire ludique — et demander de l'expliquer, et là ça n'a pas manqué (et là votre résultant, numériquement, vous pensez qu'il est nul ? vous sauriez l'expliquer ?). D'ailleurs, sur le coup, j'ai été assez stupéfait de son tour de magie (j'attends de l'avoir mieux compris pour en dire plus).

Il y a aussi des moments un peu embarrassants. J'ai assisté à un oral (une leçon) d'informatique fondamentale (autre option nouvelle cette année, l'informatique), où le candidat a été tellement mauvais, et il s'en rendait compte, que j'en avais mal pour lui. Il y aussi un Monsieur gentil et tout timide, qui ne devait pas être bien loin de la retraite, qui passait l'agreg, et qui venait assister à d'autres oraux (souvent le public est, ainsi, formé d'autres candidats) : il m'a demandé si moi aussi je passais l'agreg, et j'ai répondu sans trop réfléchir que, non, moi je l'avais passée huit ans plus tôt ; à la réflexion, j'aurais sans doute dû mentir plutôt que de faire cette réponse.

↑Entry #1323 [older| permalink|newer] / ↑Entrée #1323 [précédente| permalien|suivante] ↑

↓Entry #1302 [older| permalink|newer] / ↓Entrée #1302 [précédente| permalien|suivante] ↓

(mardi)

Des maths : résultats, décision, exposé

Here is wisdom. Let him that hath understanding count the number of the beast: for it is the number of a man; and his number is Six hundred threescore and six. (Rev 13:18)

Une des bonnes nouvelles de la journée, pour moi, c'est la liste d'admissibilité de l'Agreg de maths, puisque nous avons eu 100% d'admissibles. Ce n'est pas comme si c'était une surprise énorme, certes, mais j'avais tout de même un peu d'inquiétude — notamment parce que le ministère a beaucoup baissé le nombre de places cette année — concernant un ou deux élèves[#]. Pour la suite j'ai par ailleurs quand même des doutes sur la possibilité de battre le record de l'an dernier (entre autres, la façon dont seront notées les nouvelles options reste un grand mystère).

Indépendamment de ça, j'ai pris une décision, qui est de rester à Paris l'an prochain (de ne pas partir à Lyon, donc). La proposition qu'on m'avait faite était très intéressante mais, tout bien considéré, pour probablement seulement un an le déplacement n'en valait pas la peine. Bon, me connaissant, je vais probablement le regretter, mais il fallait faire un choix, alors voilà, il est fait.

Tant que j'y suis à parler boulot : je donne un exposé vendredi en quinze (le 23, donc) au séminaire Variétés rationnelles sur un très beau résultat de Kollár. Non seulement le théorème est joli, mais la démonstration a été l'occasion pour moi de comprendre à quoi pouvait servir le théorème d'annulation de Kawamata-Viehweg (dont l'énoncé me semblait, a priori, assez sibyllin[#2]).

Hors de ça, j'ai l'impression de passer un temps dingue à écrire des mails. Pas que j'en écrive tant que ça, mais je peux passer des heures à écrire trois lignes, parfois, pour trouver exactement les mots qui conviennent.

[#] Inquiétudes tout à fait injustes, n'est-il pas : personne n'oserait suggérer que certains agrégatifs passent en fait leurs journées à jouer à WoW… quand même ? 😉 Mais bon, à tout hasard, si quelqu'un a une suggestion pour faire apparaître l'agreg comme un jeu vidéo en ligne où on dégomme des développements et quand on arrive au niveau 60 on est admis, je suis preneur…

[#2] Le mieux que j'aie entendu est : Kawamata-Viehweg, c'est Kodaira pour les log-structures ; mais bon, je ne trouve pas que ça Éclaire tant que ça, en fait.

↑Entry #1302 [older| permalink|newer] / ↑Entrée #1302 [précédente| permalien|suivante] ↑

↓Entry #1297 [older| permalink|newer] / ↓Entrée #1297 [précédente| permalien|suivante] ↓

(mercredi)

Conversations de matheux, corps à un élément, apprentissage et petits gâteaux

Le mercredi après-midi, au département de maths de l'ENS, nous avons notre thé hebdomadaire : tout le monde (enseignants, chercheurs, invités de passage, étudiants… et quelques informaticiens déçus qu'il n'y ait rien de tel dans leur département), enfin, tous ceux qui veulent venir, se retrouve en salle Jean-Louis Verdier[#] pour partager thé[#2], jus d'orange et petits gâteaux achetés sur les fonds secrets du département (enfin, aujourd'hui, on était à court de petits gâteaux, mais normalement il y en a). Et c'est décidément un moment très convivial.

La conversation, naturellement, outre les sujets récurrents chez les normaliens[#3], tourne autour de potins et de ragots mathématiques[#4], mais aussi autour de mathématiques proprement dites, souvent de façon plus ou moins ludique. Par exemple on évoque régulièrement des résultats mathématiques qui nous semblent particulièrement surprenants, ou choquants : le fait que la somme de deux convexes du plan de frontière C a une frontière C6 mais pas C7 en général ; le fait que toute variété différentiable homéomorphe à Rn est difféomorphe à Rn sauf pour n=4 ; le fait qu'une série de distributions delta, en des points tous distincts, pondérées par des coefficients tous non nuls peut converger vers la distribution nulle ; etc. Nous avons également débattu de savoir quel style de rédaction mathématique offre la plus grande clarté (vaut-il mieux une démonstration compréhensible ligne à ligne et dont la rigueur est inattaquable mais dont on ne parvient pas à dégager les idées directrices générales, ou au contraire une démonstration qui fasse clairement ressortir les idées sous-jacentes mais demeure perfectible dans beaucoup de détails parfois fastidieux à compléter ? et comment parvenir à allier les qualités de ceux styles tout en évitant leurs défauts ?).

Aujourd'hui, nous[#5] nous sommes mis à papoter sur le corps à un élément et sur le corps résiduel des réels. Cela va sembler complètement sibyllin aux non-mathématiciens, mais il s'agit (surtout pour le premier) d'une fantaisie récurrente des algébristes (dont le statut varie, selon les circonstances, entre de la recherche sérieuse et une blague de matheux) ; la plupart des personnes ayant fait des maths au niveau du second cycle savent bien qu'un corps doit avoir au moins deux éléments (à savoir 0 et 1, qui ne coïncident que dans l'anneau nul, ce dernier n'étant pas un corps) : mais il se trouve qu'un nombre important de résultats d'algèbre ou de théorie des nombres semblent pouvoir s'expliquer, par analogie, comme si elles provenaient de l'existence d'un objet que, à l'heure actuelle, personne ne sait définir correctement (et qui n'est certainement pas un corps au sens usuel, et qui n'a probablement pas non plus un élément en aucun sens naïf[#6]) mais que suite à ces analogies on appelle corps à un élément (sur lequel, notamment, l'anneau des entiers serait une algèbre), F1. Là où il s'agit d'une blague, c'est quand on se met à explorer les analogies les plus fumeuses sur le corps à un élément, du style : comme tout corps fini, le corps à un élément à une extension de degré n, qui est le corps à 1n éléments, qui n'est bien sûr pas la même chose que le corps à un élément ; et il a un frobenius, qui est l'élévation à la puissance 1, qui n'est bien sûr pas l'identité… si vous êtes un être humain normal, donc pas un algébriste, il est sans doute naturel que vous ne trouviez pas ça drôle. 🤪 Là où c'est plus sérieux, c'est quand on espère qu'une définition rigoureuse de cet objet mystérieux permettrait de tirer correctement des analogies : notamment, l'hypothèse de Riemann (dont la tête est mise à prix) aurait des chances de pouvoir être abordée comme l'analogue des conjectures de Weil (qui, comme leur nom ne l'indique pas, sont maintenant des théorèmes) pour le spectre des entiers vu comme variété sur le corps à un élément. Malheureusement, si des définitions partielles ont été proposées pour le corps à un élément (ici et par exemple), non seulement aucune n'est complète (aucune, notamment, ne permet de donner un sens intelligent au produit tensoriel de Z avec lui-même au-dessus de F1) mais en plus elles ne sont pas d'accord entre elles. Quant au corps résiduel des réels, c'est quelque chose dans le même style… ce serait un corps fini qui aurait la bizarre propriété d'avoir une unique extension, de degré deux (le corps résiduel des complexes) qui soit algébriquement close ; là, personne n'a trop d'idée de combien d'éléments il serait censé avoir (on peut donner des arguments pour 0, 1, 2, 3, ou même 2.718… ou une infinité ; personnellement, j'ai tendance à croire qu'il a un seul élément mais dont deux sont non nuls !).

Bon, heureusement, pendant que certains mathématiciens se demandent combien d'éléments a un corps à un élément, d'autres font des choses utiles, comme se pencher sur les manières de construire un filtre à spam efficace : mes collègues statisticiens organisent à l'ENS un colloque sur les fondements mathématiques de l'apprentissage, ou comment apprendre (à un ordinateur, disons), à partir d'un échantillon de données et de réponses associées (du style, ceci est un spam, ceci n'est pas un spam — mais je ne voudrais pas donner l'idée que l'apprentissage ne sert qu'à trier le spam !), à tirer des réponses correctes sur d'autres données. Par exemple, le filtre à spam que j'utilise, qui est essentiellement un filtre bayesien avec quelques améliorations (comme je ne suis pas statisticien, je ne comprends pas grand-chose, là), a tendance à se faire avoir à cause du problème suivant : quasiment tous les mails que je reçois en anglais sont du spam, contre très peu des mails en français — du coup, au lieu d'apprendre à reconnaître le spam et le ham (= non-spam), il a surtout appris à reconnaître l'anglais du français, et quand on m'envoie un mail en anglais, très souvent il passe à la poubelle… alors je me sens assez concerné par ce genre de questions !

[#] Note mentale : il faudra créer un article Wikipédia sur Verdier.

[#2] Il est vrai que c'est plutôt le café que les matheux sont censés transformer en théorèmes, mais il faut un peu de tout : avec du thé on produit des conjectures, l'espresso donne des lemmes, le capuccino des corollaires, le jus d'orange des définitions, etc. En revanche, il faut éviter le déca : avec ça, on produit des contre-exemples — ça a la saveur d'un théorème, mais on send bien qu'il manque quelque chose. Et évidemment, le Coca-Cola, lui, donne des programmes informatiques.

[#3] Comme l'incompréhension totale devant les dernières mesures prises par les responsables hygiène et sécurité de l'établissement, ou la difficulté de trouver un quatrième partenaire pour jouer au bridge.

[#4] En cette saison de l'année, ce genre de potins prennent assurément une tournure la vie est dure, mais ce n'est pas toujours le cas. Un de mes collègues soutient bientôt sa thèse, alors on discutait de comment il faut décoder les phrases dans les rapports du jury : si c'est écrit a prouvé de bonnes qualités d'enseignant, ça veut dire qu'ils pensent que tu es un mauvais chercheur…

[#5] Nous, en l'occurrence, c'est surtout Xavier Caruso et moi. Il peut tout nier mais des gens l'ont vu.

[#6] D'ailleurs, on remarquait justement que, selon les auteurs, le corps a un élément semble avoir (au sens du nombre de points de le droite affine sur ce corps…) deux ou trois éléments. 😉 Mais si on tient absolument à avoir une réponse intuitive, l'idée serait que le corps à un élément a un 1 mais n'a pas de 0 — explication à ne pas prendre très au sérieux, cependant !

↑Entry #1297 [older| permalink|newer] / ↑Entrée #1297 [précédente| permalien|suivante] ↑

↓Entry #1274 [older| permalink|newer] / ↓Entrée #1274 [précédente| permalien|suivante] ↓

(mardi)

Field Arithmetic

J'ai encore fait une folie en achetant (à plus de 150€ — argh !) le volumineux Field Arithmetic[#] de Michael Fried et Moshe Jarden (2e édition). Une folie, certes, mais un rapide coup d'œil à la table des matières a suffi à me convaincre que je devais acheter ce livre, et après avoir examiné un peu plus longuement le contenu je suis conforté dans cette conclusion : je ne vais pas le regretter.

Quand j'étais petit, je pensais naïvement que la structure mathématique de corps était une structure si rigide qu'il n'y avait pas grand-chose à dire (ou que la théorie de Galois nous disait à peu près tout ce qu'il y avait à savoir, puisque les corps sont construits à partir des rationnels ou des corps finis en prenant des extensions algébriques — qui sont bien comprises — et des extensions transcendantes « sur lesquelles il ne doit pas y avoir grand-chose à dire »). Ha ! S'il y avait besoin de montrer que ce raisonnement était idiot, le fait que ce livre fasse plus de 700 pages, et densément peuplées, est significatif.

Ce qui est fascinant, c'est comme le sujet — l'arithmétique des corps — dresse un pont entre l'arithmétique, la géométrie algébrique, la théorie des groupes, et la logique (la théorie des modèles), en passant évidemment par la question centrale du problème de Galois inverse (j'en ai dit un mot récemment). Il y a là ce qui suscite mon intérêt mathématique le plus profond : et ce n'est pas une coïncidence ; d'ailleurs, un résultat récent de Kollár, dont j'ai déjà parlé ici (et que j'exposerai à la séance de juin du séminaire Variétés rationnelles), s'inscrit complètement dans ce cadre. Beaucoup des résultats énoncés dans ce livre me sont déjà familiers, bien sûr, mais parfois seulement comme « folklore », et ce qui est certainement remarquable c'est de les trouver aussi commodément rassemblés dans un seul tome.

(Il faut dire que j'ai un faible pour les livres d'exposition mathématique qui se veulent aussi complets et encyclopédiques que possible : qui tentent de faire le tour d'un sujet donné, et dont on apprend vite à savoir, quand on cherche un résultat sur ce sujet, qu'on l'y trouvera. Celui-ci a l'air de correspondre assez bien à cette idée.)

[#] Field Arithmetic ne signifiant pas — comme un ami me l'a suggéré en plaisantant — arithmétique de terrain. 😉

↑Entry #1274 [older| permalink|newer] / ↑Entrée #1274 [précédente| permalien|suivante] ↑

↓Entry #1258 [older| permalink|newer] / ↓Entrée #1258 [précédente| permalien|suivante] ↓

(mardi)

Mathématiques et compétition

Les résultats du concours du CNRS seront probablement connus d'ici un ou deux jours. Je n'ai que très peu d'espoir pour moi-même (ne serait-ce que parce que, comme m'a averti un jour mon directeur de thèse : votre thèse ne contient pas de cohomologie, et donc n'impressionnera jamais les Français), donc je ne suis pas trop stressé[#], mais cela m'offre l'occasion de réfléchir à ce système bizarre de recrutement des mathématiciens par concours.

Outre que le nombre de places est ridiculement faible eu égard au nombre de candidats (12 places pour autour de 260 candidats), je vois au moins deux problèmes graves à recruter par concours. Le premier, c'est que concours implique classement et qu'il est impossible de comparer deux candidats, étant donné qu'ils ne passent pas une épreuve mais présentent un dossier, et que le jury en est donc réduit à l'absurdité de savoir s'il vaut mieux avoir démontré le théorème foo ou le théorème bar[#2], sachant qu'en général l'un n'implique pas trivialement l'autre donc un classement ne correspond pas à un ordre logique, bref, de faire un tri plus entre les domaines de recherche qu'entre les candidats. Le second problème, encore plus grave, c'est que c'est fondamentalement contraire à l'esprit de la recherche, telle que je la conçois, que de placer les chercheurs dans une situation de compétition : le principe même de la science est d'être une collaboration entre les hommes contre, disons, globalement, l'adversité (les forces de la nature ou, dans le cas des mathématiques, la difficulté à mettre de l'ordre dans le paradis platonique). Et même si le métier du mathématicien est largement solitaire (ce en quoi il diffère radicalement de celui qui travaille dans les sciences expérimentales), il n'en demeure pas moins que nous travaillons pour une cause commune et que nous mettre en concurrence les uns avec les autres est exactement opposé à ce que nous voudrions faire.

J'ai néanmoins l'impression, pour ce que je vois d'autres branches de la science, que les mathématiques sont très gentlemanly, c'est-à-dire qu'on ne se tire pas dans les pattes (en refusant de communiquer des résultats, ou ce genre de choses), du moins beaucoup moins qu'ailleurs. C'est sans doute une des raisons qui m'ont poussé dans cette direction (après mon inclination naturelle, bien entendu) : je m'en réjouis donc. Mais même : personnellement j'ai découvert que je travaillais bien plus efficacement lorsque j'ai l'impression que ma réflexion est dénuée de tout enjeu — et surtout celui de ma carrière — lorsque je travaille, donc, gratis pro amore arithmeticæ[#3] ; je suppose que je suis loin d'être le seul dans ce cas, et, par conséquent, cela doit faire beaucoup de productivités qui sont réduites par le simple fait de placer les gens en situation de concurrence.

Je n'irais pas jusqu'à honnir celui qui travaillerait pour la gloire : je comprends que, pour certains, c'est un stimulant utile, voire nécessaire. Ce n'est pas mon cas, et je trouve que la satisfaction d'avoir démontré un théorème prime sur toute réputation qu'il peut vous valoir. (Ou, pour dire les choses autrement, si un génie pervers m'offrait le choix entre réussir par moi-même à démontrer l'hypothèse de Riemann mais devoir n'en tirer aucune gloire, ou bien en trouver une démonstration toute cuite par magie dans mon tiroir et pouvoir la publier à mon nom, je n'hésiterais pas une seconde à choisir le premier.) Je suis donc partagé quant au bon goût de nommer les théorèmes d'après les mathématiciens qui les ont trouvés — c'est une chose, d'ailleurs, que Bourbaki a toujours refusée. Et si un jour j'estime ma carrière suffisamment avancée, je pense que je ferai publiquement savoir que toutes mes publications seront désormais anonymes (ce qui ne veut pas dire que l'auteur soit totalement secret[#4], mais qu'il ne figure pas sur l'article et qu'on doive donc citer ce dernier par son simple titre) et j'inciterai d'autres à en faire de même : l'idéal étant même d'être complètement oublié sauf dans la mesure où cela aide à la recherche[#5] (par exemple, pour savoir qui est compétent pour répondre à telle ou telle question).

J'allais dire que la compétition devrait être laissée à l'esprit combatif des plus jeunes, mais même dans ce cas c'est douteux. Plutôt qu'organiser des olympiades de mathématiques, ne devrait-on pas concevoir des défis où des groupes de jeunes reçoivent des problèmes à résoudre collectivement, se les répartissent comme ils veulent, partagent leurs idées pour arriver à une solution, et sont collectivement récompensés s'ils parviennent au bout d'un nombre important de problèmes ? Car l'idée du concours, une fois qu'elle rentre dans les esprits, n'est pas si facilement délogée (ma maman, par exemple, n'a toujours pas compris que c'est une bêtise dangereuse que sa fierté maternelle d'avoir eu un petit garçon qui réussissait bien).

Hélas, mille fois hélas ! Si je dis que le concours est gravement délétère pour les mathématiques (et sans doute pour les autres sciences, même si je ne peux pas vraiment parler pour elles), je ne sais pas quoi proposer à la place. Je me suis dit un moment que ce serait peut-être un moindre mal d'avoir un examen avec un numerus clausus roulant sur plusieurs années, mais au mieux cela reviendrait au même et au pire cela conduirait à des spéculations malsaines sur qui pourrait venir les années suivantes. Je reste du moins persuadé que tant qu'à avoir des concours, il faut qu'ils soient placés relativement en amont dans la carrière (donc, si possible, avant la thèse), pour éviter que des jeunes se retrouvent devant la situation où, ah, vous avez passé dix ans de votre vie à travailler pour ça ? merci d'avoir joué, nous n'avons pas de place pour vous

[#] Ce qui m'inquiète plus, en fait, est de savoir combien de places de maîtres de conférences seront libérées par le fait que les candidats déjà admis au CNRS sont essentiellement rayés des listes.

[#2] Déjà, il est douteux que la qualité d'un mathématicien (c'est-à-dire sa capacité à faire avancer la recherche) se réduise à sa production de théorèmes (qui mesure sans doute, plutôt sa consommation de café) : c'est faire l'impasse sur sa capacité à reformuler des démonstrations qui existent déjà, à discuter avec d'autres mathématiciens pour les aider à éclaircir leurs propres idées ou leur proposer des pistes intéressantes, etc. Et bien sûr, à poser les bonnes questions : car la recherche, c'est au moins autant de poser les bonnes questions que d'y trouver la réponse.

[#3] Certains pourraient être tentés de me rétorquer que j'ai bien réussi des concours, dans ma jeunesse. En vérité, je n'ai jamais travaillé pour eux : j'ai travaillé avant, et j'ai passé ces concours pour voir ce qu'ils donnaient.

[#4] Je ne veux pas priver les historiens des mathématiques de leur travail, en le rendant impossible !

[#5] Ou à l'enseignement, d'ailleurs… un effet positif inattendu de sa relative déconsidération dans le système français est qu'il n'y a pas de compétition à ce niveau-là : enseigner, c'est vraiment se mettre dans le même camp que les autres enseignants et aussi que ceux à qui on enseigne.

↑Entry #1258 [older| permalink|newer] / ↑Entrée #1258 [précédente| permalien|suivante] ↑

↓Entry #1255 [older| permalink|newer] / ↓Entrée #1255 [précédente| permalien|suivante] ↓

(vendredi)

Séminaire, pas séminaire…

J'ai très mal dormi la nuit dernière (m'étant couché vers 2h je n'ai réussi à m'endormir que vers 5h30, pour me lever à 9h30), ce qui fait que je n'étais pas trop en forme en TD ce matin. J'ai quand même rassemblé ma motivation[#] pour assister au séminaire Variétés rationnelles, parce que les sujets me semblaient vraiment beaux et intéressants[#2]. Donc je m'élance de l'ENS à 14h, et lorsque mon RER arrive à Cité universitaire, on nous apprend que des manifestants sur les voies à Arcueil font que les trains n'iront pas plus loin que Laplace. Du coup, je n'ai plus eu qu'à faire demi-tour, et je suis revenu à mon point de départ après avoir perdu une heure (plus le prix d'un ticket de RER jusqu'à Orsay) ainsi que l'occasion d'assister à un exposé auquel je tenais vraiment. Quelle plaie.

Les deux exposés tournaient autour du problème de Galois inverse (ou, donné un groupe fini, comment construire une extension galoisienne des rationnels ayant ce groupe de Galois — ou au moins prouver qu'il en existe), un sujet qui me fascine assez en ce moment (surtout que mes TD concernent justement la th'eorie de Galois), et qui fait converger des approches très différentes, dont les intrigants dessins d'enfants[#3] mais aussi des techniques apparentées aux variétés rationnelles comme l'approximation faible.

[#] Assister à un exposé mathématique quand on a du sommeil en retard est assez pénible, il faut bien le dire : même quand on est sincèrement intéressé par ce que raconte l'orateur, il est difficile de se retenir de somnoler, et à plus forte raison de prêter une attention convenable !

[#2] Évidemment, c'est un séminaire qui correspond bien à mes centres d'intérêts mathématiques en général, mais il faut avouer que parfois — c'est-à-dire essentiellement quand il y a le mot motif dans le titre ou dans le résumé — je me demande vraiment ce que je fais là.

[#3] Un dessin d'enfant au sens de Grothendieck est (la structure combinatoire plongée de) l'image réciproque du segment [0;1] par un revêtement de la sphère de Riemann ramifié en au plus 0, 1 et ∞. D'après la réciproque du théorème de Belyj, un tel revêtement peut toujours être défini par un polynôme à coefficients algébriques : le groupe de Galois absolu opère sur les dessins d'enfants en opérant sur les coefficients, et les orbites pour cette action sont quelque chose d'encore très mal compris.

↑Entry #1255 [older| permalink|newer] / ↑Entrée #1255 [précédente| permalien|suivante] ↑

↓Entry #1227 [older| permalink|newer] / ↓Entrée #1227 [précédente| permalien|suivante] ↓

(jeudi)

Feuilles de TD

Je rédige intégralement (énoncés et corrections) les feuilles des TD que je donne à l'ENS : outre que ça rend service aux élèves (enfin, j'espère…), c'est important pour moi, parce que je ne veux pas me pointer en me disant que je sais faire tel ou tel exercice et en me rendant compte finalement que non, il y a un détail qui m'a échappé ou que j'ai oublié ou un petit problème dans l'énoncé. (En DEUG je n'avais pas ce souci, mais là, le niveau est quand même un petit peu plus élevé…) Ça prend tout de même beaucoup de temps.

Je comptais cette année réutiliser les mêmes feuilles que l'an dernier. Mais voilà que je ne suis pas content des exercices que j'ai donnés, et j'en cherche de nouveaux… Je n'en suis toujours pas très content, en fait : là, pour illustrer les concepts d'algèbre tensorielle, symétrique et extérieure, qui ont beau être des notions excessivement classiques et importantes, je n'ai pas vraiment réussi à trouver des choses qui me satisfassent, ni par moi-même ni en fouillant dans des livres. (La solution de facilité, c'est, comme toujours, d'aller chercher dans les sorites au niveau n+1 les exercices du niveau n : mais je pense que pédagogiquement ce n'est pas bon.)

↑Entry #1227 [older| permalink|newer] / ↑Entrée #1227 [précédente| permalien|suivante] ↑

↓Entry #1218 [older| permalink|newer] / ↓Entrée #1218 [précédente| permalien|suivante] ↓

(mardi)

Les ordinaux dénombrables

De même qu'il m'arrive d'avoir des lubies dans d'autres domaines, j'en ai parfois en mathématiques. J'ai déjà mentionné il y a longtemps trois objets mathématiques (parmi de nombreux autres) me fascinent pour leur beauté et leur élégance : les ordinaux (encore que ce n'est pas clair si les ordinaux sont un objet mathématique) méritent aussi d'y figurer. Je suis en train de faire plein d'éditions à l'article Wikipédia à ce sujet (j'ai même fabriqué une image de ω² pour illustrer).

Il y a fort longtemps j'avais écrit un texte de vulgarisation à ce sujet, puis un autre. C'est quelque chose qu'on a vraiment envie de vulgariser parce que l'idée est excessivement simple : dès lors qu'on ordonne un ensemble (éventuellement infini) de choses de façon à ce qu'il ne soit pas possible de décroître indéfiniment, on a défini un ordinal. Ou encore : décrire un ordinal, c'est exactement décrire tous les ordinaux plus petits que lui (et comment les comparer). Tout le jeu, ensuite, consiste à se demander jusqu'où on peut monter — sachant que, quelle que soit l'intelligence avec laquelle on s'y prendra, il y aura toujours un plus petit ordinal qu'on n'atteindra pas (c'est ça qui fait que les ordinaux donnent une vision terrifiante de l'infini — un infini plus grand, si j'ose dire, que tout ce que vous pouvez construire par n'importe quelle méthode).

Si on veut, les ordinaux sont le défi que se lancent les petits enfants, tu sais compter jusqu'à combien ?, prolongé dans le transfini, et appliqué aux mathématiques.

Tout le monde connaît, a priori, l'ordinal ω, celui des entiers naturels : même si on ignore la terminologie “entiers naturels” (et même si on ignore le zéro, ce qui, du point de vue de l'ordinal, ne change rien du tout), à part quelques peuplades perdues qui n'ont pas la notion de nombre, tout le monde sait compter : 0, 1, 2, 3… jusqu'à l'infini, peut-être pas effectivement jusqu'à l'infini, mais au moins potentiellement, et ceci définit justement l'ordinal ω.

Tout le monde peut encore comprendre ω2, soit deux copies successives de ω : il suffit de prendre les entiers naturels, et, après tous les entiers naturels n, mettre les ω+n, en décrétant que les ω+n se comparent comme les n et que les ω+n sont toujours supérieurs aux n :

0, 1, 2, 3, 4, … ω, ω+1, ω+2, ω+3, …

Une fois qu'on a compris ω, ω2, ω3 et ainsi de suite, on a logiquement compris ω², qui n'en est que la limite. De même, ω³ ne pose pas de problème conceptuel particulier. Les choses deviennent compliquées, a priori, avec ωω, mais on ne peut pas vraiment capituler puisque le truc génial avec les ordinaux, c'est que dès qu'on a compris tous les ordinaux inférieurs à un ordinal donné, on a compris celui-là (et ceci permet de conclure qu'on a compris tous les ordinaux…). Ainsi, ωω c'est l'ordre de toutes les expressions du genre ω³·42 + ω·1729 + 18. Mais ensuite, ce n'est pas un pas conceptuel si compliqué d'imaginer ωω2, voire ωω², et ainsi de suite jusqu'à ωωω. En empilant les ω on arrive jusqu'à l'ordinal où les problèmes de visualisation commencent sérieusement : ε0. Avant ε0, la forme normale de Cantor (« écriture en base ω ») résout tous les problèmes, après lui, trouver une écriture n'est plus aussi facile. (D'ailleurs, quand le petit enfant demande : tu sais compter jusqu'à combien ?, Monsieur Peano est forcé de reconnaître qu'il ne sait pas compter jusqu'à ε0.) Pourtant, dans la mesure où on a compris cette forme normale de Cantor, on a compris ε0. En faisant un petit effort, on comprend aussi ε1, qui correspond à l'ordre de toutes les expressions du même type que la forme de Cantor pour les ordinaux avant ε0 mais en ajoutant ε0 comme un nouveau symbole.

Après tous les ε0, ε1, εω, εε0, η0εε et autres lettres de l'alphabet grec (transfini) il vient un ordinal appelé l'ordinal de Feferman-Schütte, parfois noté Γ0. Là il se passe quelque chose de significativement nouveau, parce que c'est, dans un sens difficile à préciser mais très important, le plus petit ordinal qui ne peut pas se décrire en utilisant des ordinaux plus petits que lui. C'est l'ordinal que Feferman décrit comme mesurant la puissance de la prédicativité, et certains systèmes mathématiques importants cessent de savoir compter à partir de là. Au-delà il y a pourtant encore des ordinaux qu'on peut décrire : j'avoue que je n'y vois plus grand-chose, mais l'ordinal de Bachmann-Howard, par exemple, est très important, il mesure la force d'un système axiomatique notable, la théorie des ensembles de Kripke-Platek. Encore au-delà il y aurait un ordinal qui mesurerait la force de la théorie des ensembles de Zermelo-Fraenkel (mais personne ne sait le décrire précisément, autrement que par une arnaque qui ne nous apprend rien). Tout ceci a un sens difficile à expliquer mais très précis (et notamment, lorsqu'un énoncé arithmétique, bien que vrai, n'est pas démontrable, c'est parce qu'on ne sait pas « compter assez loin », au sens des ordinaux, pour arriver jusqu'à lui).

Tout ceci n'est rien, cependant, par rapport à l'ordinal de Church-Kleene (c'est amusant, tous les noms sont doubles, dans ce domaine) : il représente le plus petit ordinal à partir duquel il devient théoriquement impossible de représenter les ordinaux de façon informatisée, par exemple, ou de façon arithmétique. Malgré cela, on reste dans les ordinaux dénombrables, c'est-à-dire représentables, en théorie, par des petits dessins comme j'ai fait pour ω². Personne ne pourrait donner un sens à une représentation de ce genre pour l'ordinal de Church-Kleene (déjà, il faudrait pouvoir voir les détails infiniment fins), mais en principe il existe (et aucun ordinateur ne peut le produire). Alors que dire de ω1, le plus petit ordinal indénombrable, celui qui renferme tous les ordinaux dénombrables ? Le plus petit qui a la propriété — totalement démentielle — que toute suite dedans est bornée.

Comme les théologiens hindous qui, émerveillés d'avoir le système décimal à leur disposition, s'amusaient à écrire de (passablement) grands entiers et à raconter des choses avec, j'éprouve une fascination semblable pour les ordinaux, cette échelle du monde mathématique.

↑Entry #1218 [older| permalink|newer] / ↑Entrée #1218 [précédente| permalien|suivante] ↑

↓Entry #1193 [older| permalink|newer] / ↓Entrée #1193 [précédente| permalien|suivante] ↓

(samedi)

Un théorème de János Kollár

Ceci n'est pas censé être un blog mathématique, mais je veux quand même prendre la peine de signaler ce résultat récent (annoncé il y a un mois) de János Kollár : tout corps pseudo-algébriquement clos de caractéristique zéro est C1 (et même C′1).

On dit qu'un corps K est pseudo-algébriquement clos lorsque, pour tout polynôme f à deux varibles sur K qui est irréductible sur la clôture algébrique de K, il existe une infinité de (x,y)∈K² pour lesquels f(x,y)=0. (En termes plus sophistiqués : toute variété algébrique géométriquement intègre sur K a un K-point ; et il suffit, comme je viens de le faire, de le postuler pour une courbe.)

Par ailleurs, un corps K est dit C1 lorsque tout polynôme f homogène de degré n en >n variables possède un zéro non trivial (non trivial signifiant autre que (0,…,0)). (Quant à la condition C′1, elle énonce le même fait pour une famille de polynômes homogènes dont la somme des degrés est n. On conjecture que c'est équivalent, mais on ne sait pas le prouver.)

Ce résultat était conjecturé depuis longtemps (moralement, si j'ose dire, la propriété C1 est beaucoup plus faible que celle d'être pseudo-algébriquement clos). Néanmoins, arriver le démontrer, surtout de façon aussi courte, est un tour de force de Kollár, qui fait beaucoup avancer notre compréhension de ces propriétés importantes, à la définition simple, mais sur lesquelles on sait encore, somme toute, très peu (et le fait qu'on ne sache toujours pas prouver C1⇔C′1 est assez symptomatique).

Le principe est extrêmement joli : Kollár démontre que sur un corps K de caractéristique zéro si X dans Pn est une hypersurface de degré ≤n, ou plus généralement une intersection (schématique) d'hypersurfaces de somme des degrés ≤n, alors X contient une sous-variété Y définie sur K et qui est géométriquement irréductible. Cela se fait en écrivant (trivialement) X comme fibre d'un morphisme dont la fibre générale est une variété de Fano lisse et en étudiant (c'est là tout le travail) la dégénérescence des variétés de Fano. Je n'ai pas encore regardé les détails de la démonstration, mais c'est assez court et ça a l'air plutôt simple (et bien rédigé).

↑Entry #1193 [older| permalink|newer] / ↑Entrée #1193 [précédente| permalien|suivante] ↑

↓Entry #1157 [older| permalink|newer] / ↓Entrée #1157 [précédente| permalien|suivante] ↓

(vendredi)

Séminaires mathématiques

Le séminaire Variétés rationnelles (le séminaire mathématique où je suis le plus assidu[#]) a repris ses activités, mais il a maintenant[#2] lieu à Orsay et non plus à l'ENS. J'espère que cela n'entraînera pas un changement de style pour converger vers le SAGA (Séminaire d'Arithmétique et de Géométrie Algébrique) d'Orsay, un séminaire très prestigieux mais où le commun des mortels ne comprend souvent pas grand-chose aux exposés ; parce que le séminaire Variétés rationnelles, lui, est beaucoup plus compréhensible pour quelqu'un comme moi (évidemment, j'ai fait ma thèse dans le domaine).

Les deux exposés de cet après-midi étaient d'ailleurs tous les deux d'une clarté remarquable. Le second, notamment, m'a beaucoup impressionné : l'orateur (Arnaud Beauville) a réussi, tout en tenant les temps (c'est assez rare pour être mentionné…) à expliquer avec à la fois une grande précision et une grande simplicité un résultat[#3] qui n'avait rien d'évident. Il a fait le modeste en disant que son résultat n'avait pas d'autre intérêt que de répondre à une question que Serre lui avait posée. Serre étant dans l'assistance, d'ailleurs.

C'est bien quand il y a quelques grands mathématiciens dans l'assistance : je ne parle pas pour le prestige du séminaire, mais simplement parce qu'ils osent poser des questions parfois tout à fait terre-à-terre. Je veux dire, quand un petit jeune comme moi a une question dans un séminaire, il ose rarement la poser, par peur que ce soit une bêtise, et pour ne pas passer pour un idiot devant l'assemblée de pontes (même quand il n'y a pas deux médailles Fields dans la salle, il y a toujours des gens devant qui on ne veut pas avoir l'air plus bête que nécessaire…). Serre, lui, il sait que la question n'est pas idiote, et quand bien même elle le serait il n'aurait pas peur de demander. Il y a une certaine satisfaction, quand on pense très fort mais pourquoi est-ce que machin ? et que quelqu'un d'autre (dont la compétence mathématique n'est pas à remettre en cause) demande à haute voix mais pourquoi est-ce que machin ?, à se dire qu'on avait au moins une question qui montre qu'on a suivi. (Il ne s'agit pas forcément de questions qui impliqueraient un manque de clarté de la part de l'orateur, d'ailleurs : souvent, en fait, ça porte sur un énoncé adjacent à celui qui vient d'être affirmé.) Enfin, c'est encore plus satisfaisant quand on trouve soi-même la réponse avant que quelqu'un d'autre pose la question, évidemment.

Mais globalement je me rends compte qu'il y a quelque chose de vraiment commun entre l'entrée dans le monde de la recherche mathématique et l'apprentissage d'une langue étrangère : au début, quand les gens parlent, ça a l'air d'être du charabia, ils vont beaucoup trop vite, etc. Puis progressivement des automatismes se mettent en place comme des règles de grammaire, on voit venir les choses, on sait que telle situation doit inciter à se poser tel type de question, à rechercher tel type de méthode… Et à force, on arrive à s'efforcer à mentalement précéder un peu ce que les gens vont dire.

Il y a une personne extraordinaire dans l'assistance (à la fois au séminaire Variétés rationnelles et au SAGA), c'est le génial Ofer Gabber, sans doute le mathématicien le plus vif d'esprit que je connaisse. Il est du genre, quand un orateur énonce un théorème, à lever immédiatement la main pour dire quelque chose comme : là, vous allez démontrer ce théorème en passant par cette étape-ci, puis celle-là, puis celle-là, mais je ne comprends pas comment, dans votre conclusion, vous allez faire pour traiter le cas suivant… (Modulo la formulation ; en fait, il pose généralement la question en anglais.) Bref, il n'a souvent même pas besoin qu'on lui donne la démonstration pour voir immédiatement quels sont les points difficiles ; parfois c'est ce sur quoi l'orateur voulait insister (et alors il arrive qu'il s'agace parce qu'il voulait y venir en temps utile), parfois c'est ce qu'il voulait cacher sur le tapis (et alors il s'excuse de devoir admettre ce point ou ne faire qu'esquisser la technique), et parfois il n'avait pas vu du tout la difficulté. Depuis que j'assiste à ce séminaire, j'ai vu au moins deux fois Gabber démolir en direct une démonstration (trouver une erreur dedans, je veux dire), et une fois réfuter du tac au tac une conjecture que l'orateur énonçait. Tout à fait impressionnant (et du coup, on tremble à l'idée de faire un exposé devant lui). Un ami me disait même qu'il y avait une notion plus forte que le « vrai », en mathématique, il y avait « Gabber-vrai » (autrement dit, le résultat a été suggéré à lui et il a répondu que ça marchait). Au cours du premier exposé de cet après-midi, le monde mathématique a tremblé, donc, parce que Gabber a plaisanté : but then there's a contradiction in mathematics, puisqu'il avait émis une objection à un énoncé formulé par l'orateur, et que l'orateur a répondu à l'objection, et les deux semblaient avoir raison. Toute l'assistance a bien ri. Il faut dire que l'exposé tournait autour du fait que 240 n'est pas égal à 248, ce qui est déjà original.

Et ce week-end, je vais à (certains exposés du) séminaire Bourbaki.

[#] Il est vrai qu'il est mensuel : ça doit aider.

[#2] La raison est qu'un des trois organisateurs, auparavant chargé de recherche au CNRS en poste à l'École, a été recruté professeur à Paris XI — où sont déjà les deux autres. Il n'y a donc plus de prétexte pour que ça ait lieu rue d'Ulm.

[#3] Si p est un nombre premier ≥7 alors tout sous-groupe abélien de p-torsion du groupe de Crémona du plan (sur un corps algébriquement clos de caractéristique ≠p), i.e. tout groupe de la forme (Z/pZ)r d'automorphismes birationnels du plan projectif, est de rang r≤2 et contenu dans un tore standard. (Et il y a des résultats un petit peu plus techniques pour p valant 2, 3 ou 5.) Bon, la formulation peut paraître effrayante au non mathématicien, il me faudrait un petit peu de temps pour expliquer les termes, mais cet énoncé n'est pas très compliqué : c'est de la « vraie » géométrie (il s'agit d'un énoncé sur les transformations du plan, après tout ; d'ailleurs, j'ai déjà décrit ici ce qu'était le groupe de Crémona — dans le cas de R, certes), d'une façon que l'école italienne n'aurait pas reniée, et je trouve ça très beau.

↑Entry #1157 [older| permalink|newer] / ↑Entrée #1157 [précédente| permalien|suivante] ↑

↓Entry #1141 [older| permalink|newer] / ↓Entrée #1141 [précédente| permalien|suivante] ↓

(lundi)

Exposition mathématique

Aujourd'hui j'assistais à un exposé (dans le cadre du séminaire de théorie des nombres de Chevaleret) sur Model Theory of the Witt Frobenius, with Emphasis on an Ax-Kochen-Eršov Principle : le titre peut paraître barbare, mais en fait c'était remarquablement clair et bien présenté. Peut-être justement parce que c'était de la logique (de la théorie des modèles, comme le titre de l'exposé le dit) exposée à des non-logiciens (des théoriciens des nombres, comme le titre du séminaire le dit) : je soupçonne que quand un mathématicien s'adresse à des spécialistes d'autres disciplines que la sienne il fait un effort particulier dans la clarté de sa présentation, il ne s'embarrasse pas de détails techniques encombrants (et là, apparemment, la démonstration en regorge, même si l'énoncé final — ou bien un corollaire fondamental — est tout à fait simple et beau).

↑Entry #1141 [older| permalink|newer] / ↑Entrée #1141 [précédente| permalien|suivante] ↑

↓Entry #1120 [older| permalink|newer] / ↓Entrée #1120 [précédente| permalien|suivante] ↓

(jeudi)

Un peu de masturbation (intellectuelle)

Apparemment la question suivante (on peut appeler ça des maths, je suppose) est un problème ouvert (j'aime collectionner les problèmes ouverts dont l'énoncé est aussi simple que possible, et celui-là sera bien placé dans ma collection) : partez d'un mot (fini, quelconque) sur l'alphabet de trois lettres a, b et c, par exemple baacabbabc, et, aussi souvent que vous voulez, remplacez deux lettres identiques consécutives (par exemple aa), s'il y en a, par les deux autres dans l'ordre alphabétique (donc aabc, bbac et ccab) ; la question est : peut-on, en suivant ces règles, revenir sur le mot de départ (peut-on faire une boucle, quoi, sachant qu'on choisit comme on veut le mot initial et qu'on applique les règles comme on veut) ? Il semble que non, on ne boucle jamais, on se retrouve toujours coincé dans une situation où il n'y a plus deux lettres consécutives identiques (exemple : baacabbabcbbccabbabcacccabbabcaabcabbabcaabcaacabcaabcbccabcbcbcbccabcbcbcbababc), mais allez le prouver… (En termes d'informatique théorique, la question est de savoir si la grammaire de réécritures {aabc, bbac, ccab} est fortement normalisante.)

Mise à jour (2005-11-22) : En fait, c'est démontré. Mais on notera que c'est très récent !

Avec la règle bbca à la place de bbac (le reste étant identique), j'arrive à le prouver, mais c'est très différent. Je laisse ça en exercice au lecteur intéressé (ce n'est pas complètement trivial, mais ce n'est pas non plus excessivement difficile, et ça ne demande aucune connaissance mathématique particulière, seulement une certaine habitude du raisonnement mathématique et un certain pouvoir d'abstraction).


Je fais un coq-à-l'âne, mais toujours dans le domaine de la masturbation intellectuelle, pour évoquer le droit théorique (le terme n'est pas terrible : je devrais plutôt dire méta-droit parce que c'est au droit ce que la métaphysique est à la physique, sauf que méta-droit ça pourrait évoquer le droit du droit, ce qui serait autre chose). Comme la conservation de l'information, c'est quelque chose qui plait souvent aux geeks : il s'agit, en gros, de se demander comment le droit juridique répond à des situations qu'il suppose impossible, ou qui sont totalement farfelues ou bizarres. Voici quelques exemples de problèmes sur lesquels on pourra plancher :

  • Est-il légal de survoler Paris à basse altitude si on vole de ses propres ailes et pas dans un engin quelconque ?
  • Est-il légal de se promener nu si on n'a pas d'organes sexuels ?
  • Est-il légal de courir sur une route de campagne à une vitesse supérieure à la vitesse maximale autorisée pour les véhicules à moteur ?
  • Si deux hommes (ou deux femmes, mais c'est moins rigolo) ont un enfant sans l'aide d'une femme et sans assistance médicale, qui sont les parents légaux de l'enfant ?
  • Si je trouve que les décimales de pi à partir de la 101729-ième représentent exactement le contenu de l'édition 1991 du petit Robert, le petit Robert tombe-t-il ipso facto dans le Domaine Public ? et/ou est-il interdit de publier des cartes postales contenant les (je ne sais combien) décimales de pi à partir de la 101729-ième ?
  • Si un mort ressuscite, peut-il réclamer son héritage ? et d'ailleurs, quel est son statut légal ?
  • Si je me téléporte à travers la porte d'entrée de quelqu'un qui est fermée à clé, suis-je entré par effraction ?
  • Si un mouton écrit une lettre au Procureur de la République pour porter plainte contre son berger, le Procureur doit-il donner suite ?
  • Si je m'opère moi-même de l'appendicite sans être médecin, risqué-je d'être condamné pour pratique illégale de la médecine ? Et si je me retire moi-même un rein, que je le donne à quelqu'un, et qu'il se le greffe lui-même, dans des circonstances où ce serait normalement interdit de faire un don d'organe (en gros, il n'est pas de ma famille), pouvons-nous être condamnés ?
  • Si on a une mémoire absolument parfaite (comme celle décrite par Borges dans sa nouvelle Funes el memorioso), est-on soumis à la loi Informatique et Libertés ? A-t-on le droit d'ouvrir les yeux dans un endroit où il est interdit de photographier ?
  • Quelqu'un qui naîtrait avec une tache qui reproduirait exactement les formes d'une œuvre soumise à droit d'auteur a-t-il le droit d'être pris en photo ? De diffuser des photos de lui-même ?
  • Si je trouve une contradiction dans la Constitution, ai-je le droit de tout faire ? [Cet exemple, en fait, est historiquement célèbre, parce que Kurt Gödel (le logicien), quand il avait passé un examen pour obtenir la nationalité américaine, avait étudié la Constitution américaine et prétendait avait trouvé comment exploiter des failles logiques dedans pour transformer les États-Unis en dictature de façon tout à fait légale. Albert Einstein avait dû le persuader de ne pas en souffler mot lors de l'entrevue… Plus de détails sur cette anecdote ici et .]
  • Si le parlement vote une loi selon laquelle 2+2=5, le Conseil constitutionnel peut-il, ou doit-il, la déclarer non conforme à la Constitution ? Et s'il ne le fait pas (ou qu'il n'est pas saisi), comment cette loi doit-elle être appliquée ? [Autre exemple célèbre, parce que la chambre des représentants de l'état de l'Indiana avait voté, en 1897, une loi qui fixait la valeur de π à quelque chose (du genre 3.2) demandé par un crackpot quelconque ; heureusement, cette loi avait été rejetée par le sénat.]

C'est une sorte de test de geekitude, en fait : si ces questions vous amusent ou vous intriguent, vous avez probablement une mentalité au moins un peu geek. Si vous vous dites simplement je ne comprends pas, ce n'est pas possible, alors non.

(Et encore, je n'ai pas parlé du droit international théorique, qui est encore plus rigolo.)

↑Entry #1120 [older| permalink|newer] / ↑Entrée #1120 [précédente| permalien|suivante] ↑

↓Entry #1088 [older| permalink|newer] / ↓Entrée #1088 [précédente| permalien|suivante] ↓

(lundi)

Devinette mathématique

On m'a posé cette question assez récemment, et je la trouve assez jolie ; en plus, elle est compréhensible par tout le monde :

On vous donne une règle (non graduée) d'environ 30cm (et un crayon, quand même), et deux points distants d'environ 35cm : comment faire pour tracer la droite qui les relie ? Et si les points sont très éloignés, peut-on toujours s'en sortir ?

Corrigé (2005-09-06T21:30+0200) : Voici la solution que j'avais à l'esprit.

Soient A et B les points à relier, distants d'à peu près 35cm. On trace une droite dà peu près parallèle à (AB) et distante d'elle de quelques centimètres (au maximum), et sur cette droite on marque deux points A′ et B′ tels que B′ soit à peu près aussi loin que possible de A, et A′ à peu près aussi loin que possible de B, mais pour que les droites AB′ et BA′ soient quand même traçables. Soit C′ un point sur la droite d′=(AB′) à peu près au milieu de [AB′]. On appelle C″ le point où AB′ et BA′ se rencontrent. On trace une droite d″ passant par C″ et à peu près parallèle à (AB) et (AB′) : soit B″ l'intersection de cette droite d″ et de (AC′) et A″ l'intersection de d″ et (BC′). Soit C le point d'intersection de (AB″) et (BA″) (si on a bien choisi C′, ça doit être faisable). Alors le théorème de Pappus assure que A, C et B sont alignés, et C est à peu près au milieu de [AB], donc on trace (AC) (c'est assez court) et (CB) et on a notre droite (AB) recherchée.

Cette solution doit marcher lorsque le rapport entre la distance à tracer et la longueur de la règle n'excède pas quelque chose comme 4/3 (je n'ai pas vérifié, mais c'est de ce genre-là). Mais du coup, on peut faire comme si on disposait d'une règle, disons, 30% plus longue, et en itérant la construction on peut faire comme si on disposait d'une règle arbitrairement longue. (Sauf que ça devient complètement théorique parce que la perte de précision est rapidement énorme.)

↑Entry #1088 [older| permalink|newer] / ↑Entrée #1088 [précédente| permalien|suivante] ↑

↓Entry #1046 [older| permalink|newer] / ↓Entrée #1046 [précédente| permalien|suivante] ↓

(dimanche)

Un tour de cartes (mathématique)

Le tour de cartes suivant est très facile à réaliser, puisqu'il ne demande aucune adresse et aucune manipulation (c'est une propriété purement mathématique), et pourtant il arrive à surprendre certains.

Utiliser un jeu de 104 cartes (c'est-à-dire deux jeux de 52 cartes identiques) ; ça marche déjà plutôt bien avec un seul jeu de 52, mais la probabilité d'échec est quand même relativement importante, alors qu'avec 104 ça marche beaucoup mieux. Donner le jeu à quelqu'un, qui devra les battre soigneusement, puis choisir (sans le révéler) un nombre entre 1 et 13. Il retire alors le nombre correspondant de cartes du dessus du paquet (mais en les gardant dans l'ordre) et regarde la dernière de celles-ci. Puis, selon la valeur de cette dernière carte (en interprétant un valet comme 11, une dame comme 12, un roi comme 13), il retire de nouveau le nombre en question de cartes du dessus du paquet. Encore une fois, il regarde la dernière carte pour savoir de combien avancer. Il répète ainsi l'opération jusqu'à ce que, vers la fin du paquet, il reste moins de cartes que la valeur de la dernière regardée. Cette dernière carte regardée (où on ne peut plus avancer) sera la carte qu'il devra retenir. Tout ceci est fait sans que le magicien voie, ou même, soit présent. (Il faut juste prendre garde au fait que beaucoup de gens ne savent pas compter, se trompent parfois de ±1 carte, regardent la suivante au lieu de la dernière retirée, mélangent les valeurs des figures, ou ne remettent pas les cartes rigoureusement dans l'ordre où ils les ont prises ; le mieux est sans doute de montrer la manipulation une fois, puis de battre le paquet pour que le cobaye la fasse tout seul.) Ensuite, le paquet est rendu au magicien, qui l'examine attentivement et annonce quelle était la dernière carte regardée.

C'est bien sûr un non-tour de magie, puisqu'il n'y a aucun truc : c'est juste que, avec une bonne probabilité, la dernière carte ne dépend pas du nombre initial choisi. Le magicien a donc simplement à refaire mentalement l'opération en regardant les cartes (peut-être avec plusieurs choix initiaux, pour vérifier que ça n'en dépend pas ou pour prendre la carte finale la plus probable si par malchance il y en a plusieurs en concurrence), et il annonce la bonne carte. Évidemment, il y a des gens qui ne seront pas impressionnés, parce que la propriété leur paraît évidente, mais généralement, quand on n'y a pas réfléchi, ça ne saute pas aux yeux.

Voici un exemple : supposons que les valeurs des cartes soient dans l'ordre suivant, que je viens de tirer au hasard (je note ‘A’ pour un as, ‘X’ pour un dix, ‘V’ pour un valet, ‘D’ pour une dame et ‘R’ pour un roi) :

64AD4V9436952A8DRA86R562X8AR76XVD7684RR3AV258XD6V97X952A84R2VV937D552XD7R37D384VV4X28XAA4729R96X53373D59

—on pourra vérifier que, quelle que soit la carte initiale choisie parmi les trente premières au moins, on atteint forcément le valet que j'ai marqué en vert (la carte précédente pouvant être soit le 2 deux cartes avant soit le 9 neuf cartes avant), et ensuite la succession roi, dix, dix est automatique. Tous les jeux ne sont pas aussi chanceux : dans le suivant

5273X6A92A937653A4XD8R483AADD4A7858D4927D27V66V59DD5D356A294X8VARVVX42R932V78V2X38369945R7R5XXRXR8R467V6

—on pourra vérifier que, si on part d'une des treize premières, on a quatre chances sur treize de tomber sur la séquence (en rouge) as-4-roi-8-7-dame-8-2-9-3-6-7-roi et neuf sur treize de tomber sur (en vert) dame-7-2-dame-dame-roi-valet-roi-7. Cela donne tout de même mieux de deux chance sur trois de réussir le tour même dans ce cas-là qui est malchanceux (et si on se trompe on pourra annoncer une deuxième carte qui sera la bonne).

Comme je suis mauvais en probabilités (et aussi, flemmard), je n'essaierai pas de faire de calcul précis (j'imagine que si on veut trouver la probabilité que deux valeurs initiales convergent, et en supposant qu'au lieu d'avoir un battage des cartes chacune a une valeur indépendante de toutes les autres, ça doit se faire). Mais le principe heuristique, au moins, c'est que si on prend deux valeurs initiales et qu'on itère l'algorithme, l'écart va varier aléatoirement jusqu'au moment où par hasard on tombe sur la même carte, et alors on est sûr d'y rester jusqu'au bout — et si on fait assez d'étapes, c'est-à-dire si le jeu est assez long, ça a une grande chance de se produire. Expérimentalement, la probabilité de tomber sur la même carte finale entre deux cartes initiales tirées au hasard a l'air d'être 2/3 pour un jeu de 52 cartes, et 8/9 pour un jeu de 104 (je ne pense pas que ce soient des valeurs exactes, mais en tout cas ça doit être très proche).

↑Entry #1046 [older| permalink|newer] / ↑Entrée #1046 [précédente| permalien|suivante] ↑

↓Entry #1042 [older| permalink|newer] / ↓Entrée #1042 [précédente| permalien|suivante] ↓

(mercredi)

Écriture mathématique

Comme j'ai appris ces derniers temps un certain nombre de choses (que j'étais censé savoir déjà, mais passons) sur la cohomologie des faisceaux, je me suis dit que j'allais essayer de les coucher par écrit, pour ne pas les oublier ensuite. Grave erreur ! Car au lieu de prendre de rapides notes précises, j'ai voulu remonter trop en amont, j'ai commencé à écrire un texte qui débute avec la définition d'un faisceau : avec ça, évidemment, j'ai de quoi pondre des pages et des pages avant d'en venir à ce que je voulais surtout écrire (et, sans doute, l'oublier entre temps si c'était ça le risque) — car sur les faisceaux il y a de quoi écrire des livres (d'ailleurs, des gens l'ont déjà fait, mais il faut dire qu'aucun ne me satisfait vraiment).

Je me suis donc mis en mode « écriture automatique ». Car c'est bien de ça qu'il est question : pisser du texte mathématique presque sans réfléchir (pour l'instant, car je connais trop bien ce que je raconte). Sans doute encore un projet à la David Madore, qui commence en fanfare et qui terminera en queue de poisson, c'est-à-dire qu'il restera moribond, puis sera définitivement abandonné, sans avoir jamais été achevé. Peut-être dès demain, si demain j'ai le bon sens de ne pas continuer cette insensée tentative. Enfin, toujours est-il que j'ai pondu dix pages aujourd'hui, en lesquelles je n'ai d'ailleurs même pas encore trouvé moyen de définir le germe d'un faisceau en un point, comme quoi ça ne va pas très vite.

↑Entry #1042 [older| permalink|newer] / ↑Entrée #1042 [précédente| permalien|suivante] ↑

↓Entry #1040 [older| permalink|newer] / ↓Entrée #1040 [précédente| permalien|suivante] ↓

(lundi)

Tétraèdres et miroirs (réponse au mystère)

L'image de l'entrée d'avant-hier est la vue de l'intérieur d'un tétraèdre (régulier) dont les faces sont des miroirs (et, pour qu'on voie quand même quelque chose, ils sont aussi intrinsèquement lumineux : en rouge, vert, bleu et blanc pour les quatre faces). La caméra est positionnée en un sommet du tétraèdre et regarde la face opposée, ce qui explique que la figure soit aussi symétrique. Peut-être qu'elle devient plus claire si je rajoute une boule au centre du tétraèdre et que je décale légèrement la caméra (la réflectivité des miroirs a également été un peu augmentée) :

[Image de l'intérieur d'un tétraèdre]

J'ai également un petit film (format DivX;-)-dans-AVI, je crois — j'ai laissé l'encodeur prendre ses options par défaut —, une douzaine d'images, six-sept secondes) où la caméra se déplace un peu, ce qui aide à visualiser la géométrie de la chose. Personnellement, j'ai du mal, mais je vois intrinsèquement très mal en trois dimensions (ce n'est apparemment pas rédhibitoire pour passer une thèse en géométrie).

On peut imaginer que sur chaque face du tétraèdre initial on a posé un tétraèdre qui lui est identique (son image par le miroir, juste légèrement teinté), puis sur chaque face de chaque nouveau tétraèdre (exceptées celles qui reviennent au tétraèdre initial) encore un tétraèdre et ainsi de suite à l'infini. Mais les tétraèdres réguliers ne pavent pas l'espace — loin de là : les volumes vont se chevaucher bizarrement. Mais imaginons que (et c'est justement le propre des miroirs que de multiplier les tétraèdres, les boules et le nombre des hommes) l'espace ne se chevauche pas : on obtient cette étrange vue.

La propriété mathématique intéressante, c'est que les réflexions autour des quatre faces du tétraèdre sont en produit libre : autrement dit, si je pars du tétraèdre initial et que j'empile comme je veux des nouveaux tétraèdres sur les faces des précédents, je ne vais jamais revenir à un tétraèdre translaté du tétraèdre initial (sauf à être revenu à lui, justement, ce qui ne peut se faire qu'en parcourant exactement le chemin inverse de celui qu'on a pris). Hum, je ne sais pas si c'est très clair, malheureusement.

Quoi qu'il en soit, l'apparence de la figure m'a un peu surpris. Évidemment, les choses s'expliquent. Par exemple, on remarque facilement deux boules très proches mais pas tout à fait confondues. Cela vient du fait que l'angle diédral entre deux faces adjacentes d'un tétraèdre est 70°32′, très près, donc, de l'angle au centre d'un pentagone régulier (71°) : donc en alternant les symétries par rapport à deux faces jusqu'à en faire cinq au total, on revient presque au point de départ (notamment, et c'est ce qui se voit le mieux, la symétrique de la boule centrale par rapport aux faces bleue puis blanche puis bleue est très proche de celle par rapport aux faces blanche puis bleue). Si au lieu de prendre un espace plat j'avais rajouté un peu de courbure (isotrope positive), pour qu'on puisse mettre exactement cinq tétraèdres autour de chaque arète, en pentagone régulier, alors on obtiendrait un joli pavage, un des six polytopes réguliers en quatre dimension, le 600-cellules ou « hypericosaèdre », qui, comme son nom l'indique, a 600 hyperfaces qui sont des tétraèdres réguliers. (Si j'ai la patience, un des jours, je tenterai de faire une représentation du 600-cellules, et peut-être des autres polytopes réguliers en quatre dimensions, inscrits dans l'hypersphère vue comme un espace 3D courbe.)

Je ne suis pas sûr de bien comprendre, en revanche, les espèces de pentagones clairs qui sont clairement visibles sur ma figure initiale (et pas sur la nouvelle, donc ça dépend de la position de vue) autour des trois sommets visibles du tétraèdre. Ce que j'explique encore moins, c'est pourquoi, si on augmente considérablement la réflectivité des miroirs, on obtient quelque chose comme ceci :

[Kaléidoscope de couleurs]

Décidément psychédélique.

↑Entry #1040 [older| permalink|newer] / ↑Entrée #1040 [précédente| permalien|suivante] ↑

↓Entry #1038 [older| permalink|newer] / ↓Entrée #1038 [précédente| permalien|suivante] ↓

(samedi)

Encore des figures psychédéliques

[Motif coloré]

[English translation follows.] Encore une image mathématique calculée sur ordinateur : qui saura reconnaître ce dont il s'agit (il n'y a pas besoin de notions mathématiques très poussées pour décrire la chose — en tout cas nettement moins que pour la précédente) ? J'avoue que je ne pensais pas trop que ça ressemblerait à ça, et je ne sais pas si je peux expliquer toutes les propriétés de l'image.

[Traduction anglaise de ci-dessus.] Another mathematical image drawn by computer : who can recognized what it is (one doesn't need very sophisticated mathematical notions to describe it — at least not nearly as much as the previous one) ? I must admit I didn't think it would look like that, and I'm not sure I can explain all of the features.

↑Entry #1038 [older| permalink|newer] / ↑Entrée #1038 [précédente| permalien|suivante] ↑

↓Entry #1036 [older| permalink|newer] / ↓Entrée #1036 [précédente| permalien|suivante] ↓

(jeudi)

C'est terrifiant, les maths

J'avais décidé de passer l'été à colmater un peu l'immensité de mon ignorance mathématique : pour l'instant, le résultat est assez catastrophique. Pas que je n'apprenne pas des choses, au contraire[#] : mais dès que j'apprends une chose il se pose cinq questions à moi, et elles se rajoutent aux questions que d'autres me posent incidemment. Du coup, je n'arrive pas à lire des livres systématiquement, je les parcours à la recherche d'informations sur ceci ou cela, je m'énerve souvent de ne pas les trouver, je perds un temps fou à essayer de résoudre le problème moi-même avant de me rendre compte qu'il est conséquence évidente d'un résultat que je devrais savoir.

Bon, c'est peut-être normal, diront certains : la science, c'est toujours comme ça, toute réponse doit déboucher sur au moins cinq questions. Mais bon, d'une part c'est un peu un cliché, d'autre part je ne fais pas avancer la science, là, juste mon savoir personnel (ou en tout cas la prise de conscience de mon ignorance).

C'est vraiment déprimant, tout ça.

[#] Je me fais par exemple, maintenant, une certaine idée de ce qu'est une classe de Chern (je n'oserais pas aller jusqu'à dire j'ai compris ce que c'est, parce que je suis toujours persuadé qu'il y a un plan de compréhension bien supérieur qui est accessible aux gens comme mon ami Joël Riou, mais j'ai réussi à faire quelques calculs sur ces bébêtes qui me semblaient si opaques à un moment) ; certes, il n'y a pas de quoi s'en vanter : ça fait bien longtemps que je suis censé avoir assimilé ce que ce sont. Je plaide néanmoins que c'est remarquablement mal expliqué dans tous les livres existant (et aucun ne prend la peine d'expliquer gentiment par quelques exemples comment on manipule ces machins). Les classes de Chern ne sont qu'un exemple parmi plein d'autres.

↑Entry #1036 [older| permalink|newer] / ↑Entrée #1036 [précédente| permalien|suivante] ↑

↓Entry #1016 [older| permalink|newer] / ↓Entrée #1016 [précédente| permalien|suivante] ↓

(Thursday)

David A. Cox's Galois Theory

[Traduction française ci-dessous.]

David A. Cox is renowned (besides for having been one of the “discoverers” of toric varieties, one of the most elegant objects in algebraic geometry) for the clarity of his mathematical writing: his Primes of the form x²+ny² is an excellent prolegomenon to algebraic number theory and class field theory. In a recent book-buying compulsion, I got a copy of his latest book, a teatise on Galois theory (ISBN 0-471-43419-1), and I wish to mention how remarkably good I find it—even after I've done hardly more than glance through it. True, there isn't much for me to learn in it (I should hope so!), but the book is so well written and full of interesting notes, both historical and mathematical, that it is very enjoyable even for those who think they don't have any more to discover about Galois theory. Among the features found in this treatise which are not common in similar presentations, let me mention his account of geometric constructions by origami (not every algebraist knows that the regular heptagon or enneadecagon can be constructed by paper folding!) or his very nice chapter on the division of the lemniscate (hinting at Kronecker's Jugendtraum for Q(i) and class field theory); his explanations on how to compute Galois groups also appear quite excellent. I merely regret that he didn't write a little something on Galois cohomology, but I guess that would have been beyond the intended scope of the book; and for those looking for a Bourbakist treatment of the Galois correspondance—as an equivalence of category between étale algebras and actions of the Galois group—Douady & Douady's also excellent (but utterly different in style and approach) Algèbre et Théories galoisiennes exists.

Anyway, I would heartily recommend Cox's book to anyone who knows basic algebra and wishes to learn about this fascinating and beautiful subject, Galois theory, and some of its nice applications (in elementary geometry and elsewhere): not just to students, but also to math hobbyists, mathematicians from other domains with a leisurely interest in algebra, etc.

[French translation of the above.]

David A. Cox est renommé (à part pour avoir été un des « découvreurs » des variétés toriques, un des objets les plus élégants de la géométrie algébrique) pour la clarté de son écriture mathématique : son Primes of the form x²+ny² est un excellent prolégomène à la théorie algébrique des nombres et la théorie du corps de classes. Dans une récente frénésie d'achat de livres, je me suis procuré une copie de son dernier livre, un traité de théorie de Galois (ISBN 0-471-43419-1), et je voudrais mentionner à quel point je le trouve remarquable — même si je ne l'ai pas parcouru beaucoup plus qu'en diagonale. Vrai, je n'ai pas énormément à y apprendre (il faut espérer !), mais le livre est tellement bien écrit et plein de notes intéressantes, tant historiques que mathématiques, qu'il est très appréciable même pour ceux qui croient qu'ils n'ont rien de plus à découvrir en théorie de Galois. Parmi les choses qu'on trouve dans ce traité qui ne sont pas communes dans des présentations semblables, signalons son compte-rendu des constructions géométriques à l'origami (tous les algébristes ne savent pas que l'heptagone ou l'ennéadécagone réguliers peuvent être construits en pliant du papier !) ou son chapitre très agréable sur la division de la lemniscate (en tirant vers le Jugendtraum de Kronecker pour Q(i) et la théorie du corps de classes) ; ses explications sur la manière de calculer les groupes de Galois semblent également excellentes. Je regrette simplement qu'il n'a pas écrit un petit quelque chose sur la cohomologie galoisienne, mais je suppose que ç'aurait été au-delà de la portée désirée de son livre ; et pour ceux qui cherchent un traitement bourbachique de la correspondance de Galois — comme une équivalence de catégorie entre les algèbres étales et les actions du groupe de Galois — l'également excellent (mais totalement différent par le style et l'approche) Algèbre et Théories galoisiennes de Douady & Douady existe.

Quoi qu'il en soit, je recommanderais chaleureusement le livre de Cox à quiconque connaît l'algèbre de base et souhaite apprendre ce sujet fascinant et beau, la théorie de Galois, et certaines de ses applications plaisantes (en géométrie élémentaire et ailleurs) : pas seulement aux étudiants, mais aussi aux mathématiciens amateurs, mathématiciens d'autres domaines avec un intérêt de dilettante pour l'algèbre, etc.

↑Entry #1016 [older| permalink|newer] / ↑Entrée #1016 [précédente| permalien|suivante] ↑

↓Entry #1005 [older| permalink|newer] / ↓Entrée #1005 [précédente| permalien|suivante] ↓

(samedi)

Blues du mathématicien

Une remarque (d'un de ces courageux commentateurs qui ne daignent pas signer de leur nom) sur une entrée précédente m'a fait réagir d'abord par un autre commentaire, mais je pense que le sujet mérite une entrée : la remarque était

La lecture de ce blog laisse un peu sceptique. On a plus l'impression d'avoir affaire à un amateur amusé par les mathématiques qu'à un mathématicien. Il serait temps de grandir un peu. La Mathématique apporte nettement plus de plaisir si on veut bien lui consacrer une relation suivie que si on vient tirer son coup de temps en temps…

Je réponds donc, d'abord, que ce n'est pas trop l'intérêt de ce blog — d'une part parce que les maths sont mon travail (et j'écris a priori pour parler de ma vie extra-professionnelle) et d'autre part parce que l'idée est qu'il (mon blog) soit généralement lisible par tous : je ne parle donc de maths que quand je me sens une envie soudaine de vulgariser ce que je fais ou lorsque j'ai appris quelque chose d'exceptionnellement frappant. Il est certainement vrai, aussi, que mon intérêt est trop dispersé dans des milliers de domaines différents, et même à l'intérieur des mathématiques dans des milliers de problématiques, et que ceci tend à être problématique pour faire de la recherche : mais qu'y puis-je ? on ne choisit pas ses goûts, et je reste persuadé que les mathématiques arrivent à progresser en se servant de toutes sortes de types de cerveaux, aussi bien les « panoramiques » que les « focalisés ».

Mais il y a une autre réponse, plus importante. Le « système » dans lequel on rentre, actuellement (en France au moins, mais je soupçonne que c'est pareil ou pire partout dans le monde) en voulant devenir chercheur en maths (et encore, les maths sont une des disciplines les moins atteintes par ce fléau) est une véritable machine à broyer l'individu. On parle des ravages de la prépa, mais pour avoir vécu les deux j'affirme que la prépa n'est rien par rapport à ce qui vient ensuite pour ceux qui ont la folie de vouloir consacrer leur vie à la Science. Ce système vous répète à l'envi que vous êtes insignifiant et mauvais, que vous devez faire vos preuves (encore, et toujours, et encore, et toujours), que vous n'avez aucun droit, et surtout pas celui de (prendre le temps de) vivre, et que vous n'avez aucune place nulle part. Alors il y a plusieurs réactions possibles : régresser en enfance, ignorer/refuser le système (voire, le quitter complètement), sombrer dans la frénésie du travail (et je conteste que ça soit plus sain ou même plus productif mathématiquement)… Ceux qui survivent ne sont pas forcément les meilleurs ou les plus utiles à la Science (même ceux qui passent en mode « frénétique »), ce sont les plus chanceux ou les plus résistants.

Moi je n'ai pas été broyé — pour l'instant — parce que la chance m'a souri (la chance, par exemple, dans les qualités remarquables de mon directeur de thèse, la chance d'avoir la place que j'ai, etc.) — pour l'instant. Mais je témoigne quand même de l'anxiété ressentie. À tel point que je me demande parfois, quand je vois des jeunes prêts à s'engager dans les maths, si je ne dois pas les en décourager : les maths sont quelque chose d'incontestablement magnifique, mais tout sera fait pour vous mettre des bâtons dans les roues.

Le fait est que mon parcours m'a donné l'occasion de rencontrer des jeunes incroyablement brillants, et aussi d'être témoin d'un incroyable gâchis de cerveaux. Je n'en dirai pas plus parce que je ne veux pas citer de noms, mais quand je pense à ce que sont devenus, ou ce que s'apprêtent à devenir, certains qui ne rentrent pas vraiment « dans le moule », j'ai envie de pleurer.

↑Entry #1005 [older| permalink|newer] / ↑Entrée #1005 [précédente| permalien|suivante] ↑

↓Entry #0935 [older| permalink|newer] / ↓Entrée #0935 [précédente| permalien|suivante] ↓

(vendredi)

L'arbre de Stern-Brocot

Ce que je trouve fascinant dans les mathématiques, ce sont les objets infiniment élégants qu'on peut y rencontrer : je ne m'enthousiasme pas tant pour les démonstrations que pour les créatures qui peuplent le paradis platonique et dont on a vraiment l'impression qu'elles existent et qu'on ne fait que les découvrir et non les inventer. La créature que j'ai rencontrée en l'occurrence (pas aujourd'hui, mais c'est aujourd'hui que j'ai appris son nom), c'est l'arbre de Stern-Brocot : il se trouve que je l'ai redécouvert (et je suppose que je ne suis pas le premier ; déjà, il porte un double nom en l'honneur de deux personnes — Moris Stern et Achille Brocot — qui l'ont découvert indépendamment) dans ma thèse (voyez page 26 de mon manuscrit) en étudiant la résolution par éclatement des morphismes depuis la droite projective, mais je suppose que j'aurais pu le rencontrer en mille et une circonstances tellement il est naturel. J'en discutais avec un ami (Arthur), qui m'a fait remarquer que cet arbre était déjà connu et signalé dans Concrete Mathematics (le livre de Graham, Knuth et Patashnik), et c'est là que j'en ai appris le nom.

Il s'agit d'un arbre binaire dont les feuilles sont exactement les nombres rationnels (du moins tel que je le conçois, avec zéro pour racine ; d'autres présentations prennent des sous-arbres de ce que j'évoque) : il a pour racine le nombre 0, dont partent deux feuilles étiquetées −1 (à gauche) et 1 (à droite) ; au niveau suivant, les feuilles sont −2 et −½ (filles de la feuille −1), ½ et 2 (filles de la feuille 1) ; au niveau suivant ce sont −3, −3/2, −2/3, −1/3, 1/3, 2/3, 3/2 et 3. À chaque niveau, la règle de construction est de placer entre deux fractions déjà formées la fraction dont le numérateur et le dénominateur sont chacun somme de ceux des deux fractions gauche et droite entre lesquelles on intercale la nouvelle (par exemple, entre 1/2 et 2/3 on mettra 3/5, qui sera fils gauche de 2/3). Peut-être que les explications de PlanetMath ou celles de MathWorld, avec dessin, seront plus claires. Cet arbre contient une et une seule fois chaque rationnel, qui apparaît sous forme réduite, et les irrationnels correspondent à des branches dans l'arbre de Stern-Brocot ; de plus, il a des propriétés miraculeuses liées à l'algorithme d'Euclide (d'écriture des réels en fractions continuées) : si on suit une branche de l'arbre, les fractions qui apparaissent sont exactement les meilleures approximations rationnelles du réel limite, et les changements de direction (de la gauche vers la droite ou vice versa) se font exactement aux réduites de l'écriture du réel en fraction continuée. Par exemple, si on part de 1 et qu'on alterne branche droite et branche gauche, on trouve les rapports successifs des nombres de Fibonacci (2, 3/2, 5/3, 8/5, 13/8…) qui sont les approximants d'Euclide convergeant vers le nombre d'or.

[Graphe de la fonction de Stern-Brocot]Je ne m'arrête pas là : supposons que je mette en correspondance l'arbre de Stern-Brocot (mettons le sous-arbre qui a pour sommet ½, entre 0 et 1) avec l'arbre dyadique de même intervalle (qui a pour sommet ½ dont partent deux branches vers ¼ et ¾ et ensuite vers les huitièmes et ainsi de suite). On obtient une fonction continue croissante (représentée ci-contre) qui à tout rationnel (entre 0 et 1) associe un nombre dyadique (c'est-à-dire un rationnel dont le dénominateur est une puissance de 2) : elle envoie par exemple 1/2 sur 1/2, 1/3 sur 1/4, 2/3 sur 3/4, 3/5 sur 5/8 et 5/8 sur 11/16. Cette fonction se prolonge (de façon unique) en une fonction croissante continue φ de l'intervalle [0;1] sur lui-même, qui fait correspondre exactement non seulement les rationnels avec les dyadiques mais aussi — par une propriété bien connue des développements en fractions continuées — les réels algébriques quadratiques avec les rationnels (par exemple, l'inverse du nombre d'or, (√5−1)/2, ou 0.61803…, s'envoie sur 2/3 exactement, car ils s'obtiennent en alternant branche droite et branche gauche dans un cas sur l'arbre de Stern-Brocot et dans l'autre sur l'arbre dyadique). Ceci suggère toutes sortes de questions. Que peut-on dire, par exemple, du nombre dont l'image par φ est (√5−1)/2 (et donc l'image par φ² — l'itérée double de φ — est 2/3) ? Que peut-on dire des nombres dont l'image par un nombre fini d'application de φ (à la partie fractionnaire) donne un rationnel (ou, de façon équivalente, un dyadique) ? Sont-ils stables par addition et multiplication ? Peut-on les caractériser ? L'image d'un algébrique par φ est-elle un algébrique ? Le nombre 0.42037… qui est un des deux points fixes irrationnels de φ (autrement dit, son parcours dans l'arbre de Stern-Brocot est le même que son parcours dans l'arbre dyadique, c'est-à-dire que son développement en fraction continuée et son écriture binaire sont directement liés) a-t-il des propriétés remarquables ? Est-il transcendant ? Bon, je n'ai pas réfléchi à tout ça, et sans doute beaucoup de ces questions sont-elles stupides (soit parce que leur réponse est évidente soit — ce qui me semble plus probable — parce qu'elle est hors de portée et peu intéressante), mais je suis sûr qu'il y a tout de même quantité de choses fascinantes à dire sur cette fonction φ (tiens, sa dérivée s'annule en tous les dyadiques, mais que peut-on dire de φ′ ailleurs).

Ajout (beaucoup plus tard) : Renseignement pris, la fonction φ en question est également classique et s'appelle la fonction ‘?’ [point d'interrogation] de Minkowski. Dans un même esprit, voir aussi la fonction de Fabius.

↑Entry #0935 [older| permalink|newer] / ↑Entrée #0935 [précédente| permalien|suivante] ↑

↓Entry #0929 [older| permalink|newer] / ↓Entrée #0929 [précédente| permalien|suivante] ↓

(dimanche)

Les affres du nilradical inférieur

C'est un sentiment vraiment horrible de se retrouver un dimanche à onze heures du soir passées à vouloir savoir ce que c'est que le nilradical inférieur (d'un anneau non commutatif) et de n'avoir aucun moyen d'accéder à cette information. Je suis très sérieux — enfin presque.

J'ai la fâcheuse manie de travailler hors des heures d'ouverture de la bibliothèque de maths à laquelle j'ai accès ; j'ai une petite collection de livres de maths dans mon bureau (et une collection bien plus importante chez mes parents à Orsay, mais je n'y suis plus trop), qui souvent ne suffit pas à fournir les réponses que je cherche. Reste le Web : mais ce qui est frustrant, avec le Web, c'est qu'on y trouve des articles de recherche (et un article de recherche en algèbre non commutative ne va jamais prendre la peine de rappeler quelque chose censément aussi connu que la définition du nilradical), des cours de niveau vraiment basique, et quelques informations éparses au niveau intermédiaire, mais rien de cohérent.

Alors voilà, pris dans un méandre de ma pensée j'ai voulu retrouver la définition du nilradical inférieur d'un anneau non commutatif. (Plus exactement, il y a un nilradical supérieur et un nilradical inférieur qui généralisent la notion de nilradical d'un anneau commutatif — l'ensemble des éléments nilpotents. Je connais une définition possible, parfois appelée nilradical tout court, à savoir la somme des idéaux bilatères nils — c'est-à-dire dont tous les éléments sont nilpotents — mais je ne sais pas si c'est le supérieur ou l'inférieur : je penche plutôt pour le supérieur, et j'aimerais bien savoir quel est l'autre définition — peut-être la somme des idéaux bilatères nilpotents mais ça pourrait être tout autre chose.) Impossible : tous les livres que j'avais sous la main traitent essentiellement (ou uniquement) d'algèbre commutative, et personne sur le Web ni sur Usenet n'a jamais écrit noir sur blanc la définition du nilradical inférieur. Je sais que la réponse est dans le livre de Lam sur les anneaux non commutatifs, qui se trouve chez mes parents, mais je ne l'ai pas avec moi.

C'est absolument affreux. Pas pour le nilradical lui-même, mais ce sentiment que l'information est là, quelque part, mais inaccessible.

↑Entry #0929 [older| permalink|newer] / ↑Entrée #0929 [précédente| permalien|suivante] ↑

↓Entry #0915 [older| permalink|newer] / ↓Entrée #0915 [précédente| permalien|suivante] ↓

(Tuesday)

The twenty-seven lines on the cubic surface

[Clebsch Cubic surface]I spent a good part of the afternoon creating this image (click to enlarge) of one of the remarkable inhabitants of the platonic heaven: the Clebsch cubic surface; specifically, this is the Euclidean form of the latter which has the greatest possible group of symmetries (24 of the 120 symmetries of the Clebsch cubic are realized as Euclidean isometries).

Every smooth cubic surface has twenty-seven lines on it (sometimes poetically known as Solomon's seal; I do not know who coined the term or whether it is related to the plant Polygonatum biflorum, which also goes by that name). But in general they exist only as complex lines and might not all be realized: the number of real lines can be three, seven, fifteen or twenty-seven, and on the Clebsch cubic all twenty-seven lines exist in a real sense. You can only see twenty-four lines in the picture (can you?), however, because the last three lines are away at infinity.

Furthermore, it is possible for three lines on a cubic surface to all meet in a single point, in which case the point in question is known as an Eckardt point (this is a remarkable feature, and while all cubic surfaces have lines on them, not all have Eckardt points, even in the complex sens): the Clebsch cubic surface is unique in that it has ten Eckardt points, and all are real (on my particular Euclidean realization, four are the vertices of a regular tetrahedron, two of which can be clearly seen, and six are at infinity).

Another way to represent a smooth cubic surface (at least one which has all twenty-seven lines real) is as a set of six points in the (projective) plane (in general position, that is, such that no three are aligned and all six do not line on a common conic). It is not easy to describe precisely the relation between the six marked (or blown up) points and the cubic surface[#], but it is quite easy to explain how the twenty-seven exceptional lines are seen: consider the six marked points, the fifteen lines connecting any two of them, and the six conics going through five of the marked points — now 6+15+6=27, and they correspond exactly to the exceptional lines on the cubic surface; and even intersection is preserved if we agree that intersection at the marked points in the plane is only taken into account when it is tangential[#2]. Eckardt points are also easily seen that way: when three lines defined by three pairs of marked points meet in a common point, that point is an Eckardt point; also when the conic through five of the six marked points has a tangent at one of said marked points which goes through the sixth, then that tangent direction is an Eckardt “point”. Under this correspondance, the Clebsch cubic is the most remarkable configuration of six points in the plane, namely, a regular pentagon and its center. The ten Eckardt points are then obvious.

So one of the answers I might give when asked what my thesis is about is: six points in the plane.

[#] Each point on the cubic surface corresponds to a point on the plane and, if it is one of the six marked points, a line direction throught that point. (This is what is meant by blowing up: replacing a marked point by the set of all directions through that point.)

[#2] For example, given three of the six marked points, the three lines connecting them are thought not to intersect; however, each of them intersects the two marked points which it joins. This is in accordance with the idea that the marked points have been replaced by the set of directions through them.

↑Entry #0915 [older| permalink|newer] / ↑Entrée #0915 [précédente| permalien|suivante] ↑

↓Entry #0892 [older| permalink|newer] / ↓Entrée #0892 [précédente| permalien|suivante] ↓

(lundi)

Encore quelques petites nouvelles mathématiques

Mon premier TD s'est plutôt bien passé. Ou alors, s'ils ont trouvé que j'étais décidément nul, ils ont été trop timides pour me le faire savoir (mais j'y crois assez peu ; en revanche, bouh hou hou, il y en a qui trouvent moyen de me vouvoyer). Mais je n'ai fait que moins de la moitié de ce que je pensais faire (donc moins du quart de la feuille, vu que j'avais prévu large dans l'autre sens).

Sinon, je me suis (stupidement ?) engagé à faire un exposé jeudi après-midi au séminaire des doctorants de géométrie algébrique à Paris XIII (Villetaneuse) sur quelques questions autour de l'arithmétique des variétés rationnellement connexes. Je n'ai encore aucune idée de ce que je vais bien pouvoir raconter.

J'ai eu un premier écho du rapporteur américain de ma thèse, qui ne dit pas grand-chose (il pose quelques questions qui n'ont pas l'air d'être une façon diplomatique de signaler une erreur) mais qui finit par une appréciation positive. Je vais sans doute bientôt sortir une version revue et corrigée de mon manuscrit. Avec tout ça je n'ai pas vraiment le temps de faire de la recherche… je me sens un peu débordé, en fait.

Je tiens toujours à apprendre comment on résout le problème de Post (exhiber des degrés de calculabilité récursivement énumérables strictement intermédiaires entre le degré récursif et celui du problème de l'arrêt — voir l'article sur Wikipédia que j'ai un peu complété). Je trouve ce problème absolument fascinant. Je viens d'avoir une discussion sur la question de savoir où il pourrait être enseigné, d'ailleurs, discussion d'où il résulte que peut-être il n'existe aucun cours de calculabilité (niveau master, en gros) en région parisienne dans le cadre duquel ce problème serait traité. Si c'est bien le cas je trouve ça vraiment dommage.

Une conjecture fumée qu'on pourrait imaginer est que le dixième problème de Hilbert pour l'existence d'une solution rationnelle à des équations diophantiennes serait, justement, indécidable mais non ramenable au problème semi-décidable universel (le problème de l'arrêt). Ce serait surprenant, et, pourtant, ça ne contredit rien de ce qu'on sait (ou en tout cas, de ce que je sais) sur la question, notamment le fait qu'on ne sache pas trouver un algorithme mais qu'on n'arrive pas non plus à ramener le problème à la décision sur les entiers en définissant un Z de façon diophantienne dans les rationnels.

↑Entry #0892 [older| permalink|newer] / ↑Entrée #0892 [précédente| permalien|suivante] ↑

↓Entry #0882 [older| permalink|newer] / ↓Entrée #0882 [précédente| permalien|suivante] ↓

(Friday)

Reverse Mathematics

Reverse mathematics is a strange subject (AMS Mathematics Subject Classification: 03B30, 03F35) that I didn't even know existed until I stumbled upon it by chance a few days ago. It was founded by Harvey Friedman and one of its leading experts is Stephen G. Simpson. Let me try to explain briefly what it is about.

Ordinary (“forward”?) mathematics, at least from the formalist's point of view, consists essentially of the following: start with a set of axioms, which is usually the Zermelo-Fraenkel set theory (although one of the things reverse mathematics teaches us is that in fact there is no need to make such incredibly strong assumptions), and try to derive theorems from them. Reverse mathematics goes the other way around, it tries to assess the “strength” of each theorem by seeing how much of the axioms has been captured in it and can be derived back from its content. Perhaps this is a big too vague to make much sense, so I'll try to be clearer.

No mathematical proof ever makes use of the full strength of the axioms[#]. Now what happens if we restrict the system—either by omitting some axioms or by weakening them in various ways? Some theorems will cease to be theorems, and if we continue that way we lose more and more theorems until the system becomes so weak that it is essentially useless and uninteresting and the irrelevant becomes a barrier (for example, irrelevant differences in the formulation of this or that axiom might cease to be irrelevant if the other axioms are weakened too much, because the equivalence between the formulations is no longer provable). Stop somewhere before you get to that point where the system is unmanageably weak, and call the resulting set of axioms the “core system”. So far this is not rigorous, we're just setting up the framework. Everything will then be done over that core system. Many results which “used to be” theorems (they are so in the full axiom system) are no longer so in the core system. A first step toward reverse mathematics would be to show that this or that theorem (of the full system) is indeed unprovable in the core system (the fact that the usual proof requires strong axioms doesn't mean that there doesn't exist a smarter proof which could do with weaker ones).

But reverse mathematics goes farther than this: one isn't content with giving bounds on the “logical strength” of a theorem over the core system (such as: the core system cannot prove statement T, but the full system can— or even this-or-that weakened system, intermediate between the core and full systems, cannot prove statement T, whereas this-or-that weakened system can), one seeks to determine the strength exactly. Indeed, it turns out, rather surprisingly, that over reasonable core systems, many theorems (of stronger systems) are in fact equivalent to one another, and equivalent to a certain reasonably formulated logical statement (an axiom or axiom scheme which is a weakening or a subset of some axioms of the full system omitted in the core one). When one can show that statement T not only follows from a certain natural set of axioms intermediate between the core and full systems, but also, conversely, implies (together with the core) the axioms in question, then one can be satisfied that one has precisely assessed the logical strength of T.

↑Entry #0882 [older| permalink|newer] / ↑Entrée #0882 [précédente| permalien|suivante] ↑

↓Entry #0856 [older| permalink|newer] / ↓Entrée #0856 [précédente| permalien|suivante] ↓

(Saturday)

Gödel was here

I've just learned of this mathematical result which I guess I could nominate as the most elegant theorem of all times (though probably the most useless ever). Actually, I had already read the statement in Douglas Hofstadter's Gödel, Escher, Bach a long time ago, but I hadn't paid much attention to it then; and now (in the course of thinking carefully about Gödel's incompleteness theorem, Löb's theorem, iterated Gödelization and self-consistency statements) I figured out how to prove it, and the proof is simply wonderful. I can't remember at the time whose name is associated to it (it is not Gödel), but here goes the theorem:

Theorem: This statement is a theorem.

That's all. Statement T merely states that T is a theorem (say, of first-order Peano arithmetic); and T is, in fact, true (and is a theorem: for once, we don't have to distinguish T being true and T being a theorem, because the two are exactly identical, by definition of T!). This sound provokingly much like the famous riddle:

Q: What is the ultimate question in the Universe, and what is its answer?

A: The ultimate question is that which you have just asked and its answer is that which I have just given.

So maybe we can try proving statement T by saying something like this is the proof of statement T — but any mathematician will see that this is a fraud (error 42: attempting to apply a fixed point operator on proofs!). Yet the proof that works is not entirely different either, but it is more intricate, a cleverly crafted jewel.

I won't give that proof here, though. I believe any mathematician who correctly understands Gödel's theorem (not just in the handwaving way it is sometimes — and to much damage — explained to laymen) should be able to have the pleasure of figuring the proof by himself, and anyone who does not understand Gödel's theorem sufficiently well would miss the point entirely. But maybe I'll write a little survey on the incompleteness theorem(s) sometime soon, which would try to go through all the subtleties and intricacies of how it works (e.g., do we need the set of axioms to be Σ1, or does arithmetically definable suffice? why?) and what happens when we try to push it to its limits (iterating Gödelization and reflexions to transfinite heights as Solomon Feferman has done in the most brilliant way); in which case I would also, as a matter of course, give a proof of the above-mentioned theorem. Just so people can't accuse me of lacking the appropriate rigor, here is the (“Quine-unfolding”) mathematically precise way of stating T:

Fix a usual Gödel numbering scheme (of the language of first-order arithmetic). Let P(n) be the predicate such that, for any predicate Q(n) with Gödel number ⌜Q⌝, P(⌜Q⌝) asserts the existence of a proof, in the first-order theory of Peano's axioms, of “Q(⌜Q⌝)” (that is, of the statement obtained by substituting Q's Gödel number, ⌜Q⌝, in place of the free variable). Then we have P(⌜P⌝) [and the proof works in first-order Peano arithmetic].

As an extra bonus, we can do everything in intuitionist logic (that the proof of the above statement can be given in intuitionist logic is trivial, since in the end we exhibit a proof; but actually we can replace the statement by the stronger statement that this statement is provable in intuitionist logic and the statement is again true — and indeed provable in intuitionist logic).

↑Entry #0856 [older| permalink|newer] / ↑Entrée #0856 [précédente| permalien|suivante] ↑

↓Entry #0826 [older| permalink|newer] / ↓Entrée #0826 [précédente| permalien|suivante] ↓

(mardi)

Deux exposés mathématiques

J'ai assisté à deux exposés à Orsay, aujourd'hui. Le premier, par Pierre Cartier (auquel j'avais fait référence déjà) concernait Bourbaki. Cartier raconte toujours de façon très intéressante, il a juste le défaut de partir parfois dans un peu trop de digressions (même si ses anecdotes sont amusantes). Je n'essaierai pas de résumer tout ce que j'ai appris, parce que ce sont plutôt une multitude de petits faits isolés que des grandes idées. De façon très succincte, son propos était : le bilan strictement scientifique de Bourbaki est globalement positif, mais son influence sur l'enseignement (dont il n'est d'ailleurs pas directement responsable), c'est-à-dire les maths modernes, a été catastrophique. Cela pose d'ailleurs la question tout à fait générale de comment on doit au mieux enseigner les mathématiques (entre autres — j'en discutais tout à l'heure avec une amie qui est prof en collège — comment présenter la notion de démonstration et faire ressentir son intérêt ? cela me semble très délicat), question dont je ne pense malheureusement pas qu'elle admette de réponse satisfaisante, en fait. J'y reviendrai peut-être un jour.

Le second était un exposé scientifique (par Bjorn Poonen, un des mathématiciens qui donne les séminaires les plus clairs que je connaisse) et concernait le dixième problème de Hilbert.

Sous sa forme originale, le dixième problème de Hilbert est quelque chose comme ceci : considérons un polynôme f à N variables et à coefficients entiers, on cherche à savoir s'il existe des entiers x1,…,xN tels que f(x1,…,xN)=0 (autrement dit, à savoir si l'équation diophantienne f=0 est résoluble), et on cherche à le faire de façon algorithmique : autrement dit, on demande s'il est possible de trouver un programme informatique (peu importent les détails, si on croit à la thèse de Church-Turing) qui, quand on lui fournit le polynôme f, va déterminer si l'équation est résoluble. La réponse apportée par Yuri Matiyasevič est négative : il n'existe pas de tel algorithme (il démontre même que, quel que soit l'ensemble d'entiers E récursivement énumérable, il existe f à 1+N variables pour un certain N — qui peut être choisi uniformément — tel que aE si et seulement si f(a, x1,…,xN) = 0 admet une solution dans les variables x1,…,xN ; et comme on sait qu'on peut trouver des ensembles récursivement énumérables et non récursifs, il ne peut pas y avoir d'algorithme — en fait, on atteint le niveau maximal de non-récursivité au sens de Post, et par ailleurs, ce qui est peut-être encore plus frappant, on peut trouver des f tels que l'existence d'une solution soit indécidable au sens de Gödel, dans n'importe quel système formel récursif contenant au moins l'arithmétique).

Ce qui est remarquable, en revanche, c'est que si on remplace la recherche de solutions entières par des solutions rationnelles (on peut aussi permettre aux coefficients de f d'être rationnels, ça ne change rien dans un cas comme dans l'autre car on peut toujours chasser les dénominateur), on ignore si la question est décidable ou non. La plupart des mathématiciens pensent que la réponse est négative comme sur les entiers, mais certains sont d'un avis différent (comme Swinnerton-Dyer, notamment, ou, semble-t-il, Mazur : ce ne sont pas de petits noms), et d'ailleurs un exposé récent du même Poonen au séminaire Variétés rationnelles laissait entrevoir un espoir possible de réponse positive ; cependant, l'exposé d'aujourd'hui laissait plutôt penser à une réponse négative. En tout cas, c'est une situation un peu analogue au fameux problème P=NP : la plupart des gens pensent qu'il n'y a pas d'algorithme, ce qui est sans doute dommage, mais on ne sait pas le démontrer, ce qui est également dommage. Sur les complexes ou les réels (ou les p-adiques), c'est-à-dire si on cherche des solutions dans un de ces corps-là, il existe bien un algorithme. L'exposé d'aujourd'hui portait sur le résultat suivant : il existe un ensemble S de nombres premiers de densité naturelle 1 (c'est-à-dire que pour x grand, le nombre de nombres premiers inférieurs à x qui sont dans S est asymptotiquement équivalent au nombre de nombres premiers inférieurs à x tout court) tel qu'il n'existe pas d'algorithme pour détecter l'existence de solutions rationnelles dont le dénominateur n'a que des facteurs premiers dans S ; si on pouvait dans cette affirmation prendre pour S l'ensemble de tous les nombres premiers (ou même, en fait, tous sauf un nombre fini), ce serait la réponse négative sur les rationnels, mais obtenir un ensemble de densité naturelle 1 est déjà une indication importante dans cette direction.

Clarification () : Yann Ollivier me fait remarquer que le résultat tel que je l'annonce est trivial si S n'est pas supposé récursif. Confirmation prise auprès de Bjorn Poonen, l'ensemble S qu'il construit est bien récursif : donc, rajouter cette précision dans les énoncés ci-dessus.

↑Entry #0826 [older| permalink|newer] / ↑Entrée #0826 [précédente| permalien|suivante] ↑

↓Entry #0790 [older| permalink|newer] / ↓Entrée #0790 [précédente| permalien|suivante] ↓

(vendredi)

Comment je fais des maths (ou pas)

C'est quelque chose qui me fascine assez, comment j'arrive à « faire des maths » pendant des heures d'affilée et ne pas avancer d'un pouce — mais pas parce que je butte sur une difficulté, simplement, je m'égare, je pense à autre chose (des mathématiques toujours, mais pas la question sur laquelle je suis censé réfléchir), je me pose des questions sans importance et je tiens obstinément à les résoudre avant de continuer. J'ai déjà signalé que j'ai tendance à me laisser trop facilement piéger par les petits problèmes à l'apparence élégante : je me demande dans quelle mesure c'est le cas des autres matheux. La rédaction d'une introduction de thèse est particulièrement redoutable pour ce qui est de tomber dans ce genre de pièges : quand on veut dresser un panorama de ce qu'on sait, c'est le moment de se poser énormément de Questions À La Con® et de passer un temps fou à les résoudre. J'ai noirci des pages et des pages de calculs pour appliquer des concepts abstraits à des situations triviales (vous serez tous ravis de savoir, par exemple, qu'une courbe cubique plane singulière irréductible tracée sur une surface cubique lisse est très libre si et seulement si sa singularité est un point double ordinaire : c'est ce qui résulte d'un calcul bovin et sans subtilité, et ça ne sert absolument à rien, même dans ma thèse).

Bref, alors que le temps commence à me manquer vraiment sévèrement, je le perds allègrement. Mais le pire, c'est que c'est en faisant des maths ! (Simplement, pas comme il le faudrait.)

↑Entry #0790 [older| permalink|newer] / ↑Entrée #0790 [précédente| permalien|suivante] ↑

↓Entry #0781 [older| permalink|newer] / ↓Entrée #0781 [précédente| permalien|suivante] ↓

(Friday)

Math is hard

One of the reason I didn't study computer science is that it is a pit of despair: as far as practical problems are concerned, nobody has yet come up with a programming language that is really usable (so everyone writes stuff in C, which is the moral equivalent of writing in English in iambic pentameters without the letter ‘e’—that is, sheer masochism—,or C++, which is worse; and high-level languages simply don't work); as for theoretical problems, they are sitting in a kind of limbo between mathematics and theology. Anyway.

But math is also a source of despair, when one reflects on how little one can prove and how many simple questions are left open and probably will remain so for a long time.

It is easy to concoct a math problem which is understandable by anyone and which is probably bound to remain unsolved for a really long time. Perhaps the simplest example is the so-called Syracuse problem (on the Collatz sequence), namely: start with a positive integer n, and, so long as it is not equal to 1, repeat the following: if it is even, divide it by two, and if it is odd, multiply it by three and add one (for example: 7 → 22 → 11 → 34 → 17 → 52 → 26 → 13 → 40 → 20 → 10 → 5 → 16 → 8 → 4 → 2 → 1); the conjecture states that, whatever the starting integer, the sequence will eventually reach 1 (after which it would go into the loop 1 → 4 → 2 → 1 if it is continued; the question is essentially whether there exist other loops than that which starts from 1). It has been verified experimentally for a huge number of starting values, but so far as I know, nobody has been able to come up with a proof (or, at least, a published proof which has passed peer-review: because there are certainly hundreds of crackpot-generated proofs on the Web). Of course, nobody (almost no real mathematician, I mean) is seriously interested in this particular problem, so it isn't a surprise that no progress is being made; all the same, it is probably a very hard problem.

Here's one, however, that is of much greater theoretical interest, which is “probably not false” (as I heard Swinnerton-Dyer call it), and which is hopelessly out of reach of the present techniques of mathematics. Let F1,…,Fk be k polynomials (in one variable x) with integer coefficients and positive leading coefficient, and assume the product F1·…·Fk of all the Fi is not constantly divisible by some prime p (for example, x²+x is always even—assume that sort of thing doesn't happen): then [the conjecture states that] there are infinitely many integer values of x such that the Fi(x) are all simultaneously prime. This is called Schinzel's (H) hypothesis (or sometimes the Schinzel-Sierpiński hypothesis). Even in the simple case where one takes the two polynomials x and x+2, the Schinzel hypothesis implies the twin primes conjecture, which is far from proven. Similarly, Schinzel's hypothesis implies that there are infinitely many primes of the form 6q²+1 where q itself is prime, and many other things of the same form.

Here is another conjecture, which is about of the order of difficulty of Schinzel's hypothesis (i.e., probably not false, but unattainable): assume t1,…,tr are r complex numbers that are linearly independent over the rationals, meaning that if c1,…,cr are r rationals such that c1·t1 + … + cr·tr = 0, then all the ci are zero (for example, 1, √2 and 2iπ are linearly independent over the rationals); then [the conjecture states that] out of the 2r complex numbers t1,…,tr,et1,…etr (in other words, add the exponentials of the ti to the list), there are at least r which are algebraically independent (over the rationals), meaning that if some polynomial with rational coefficients in r variables vanishes when applied to the r quantities in question, the polynomial is identically zero. This is known as Schanuel's conjecture. If applied in the particular case of 1, √2 and 2iπ, Schanuel's conjecture implies that the three quantities e, e√2 and π are algebraically independent, which in turn implies the transcendence of such quantities as e+π which is still an open problem. Schanuel's conjecture is a kind of holy grail in transcendental number theory.

In comparison to Schinzel's hypothesis or Schanuel's conjecture, the seven problems whose head is at stake for $1000000 are presumably quite easy (in fact, at least one—namely the Poincaré conjecture—can now be considered as good as solved, thanks to Perelman).

It is not unthinkable that these problems should be undecidable; however, the tools available for proving the undecidability of a mathematical statement are even more inadequate, in this case, than those that might attempt to prove the statement… (I let those with some knowledge in logic reflect upon what happens when one iterates the construction is undecidable to transfinite heights).

↑Entry #0781 [older| permalink|newer] / ↑Entrée #0781 [précédente| permalien|suivante] ↑

↓Entry #0637 [older| permalink|newer] / ↓Entrée #0637 [précédente| permalien|suivante] ↓

(mardi) · Dernier Quartier

Comment piéger un David Madore

Une façon de piéger un David Madore, c'est de lui poser une question de maths qui a l'air parfaitement innocente et qu'il va avoir envie de résoudre rapidement et élégamment, et qui en fait s'avère être un piège redoutable. Je suis capable de passer un temps invraisemblable sur certains problèmes de ce genre : en fait, c'est quasi obsessionnel — je n'arrive plus à penser à autre chose tant que je n'ai pas résolu la question ou que je ne me suis pas convaincu (parfois avec une certaine mauvaise foi, heureusement, sinon je pourrais y rester bloqué indéfiniment) que le problème n'a pas autant d'intérêt que je le pensais.

Un exemple de tel problème qui m'a bien eu il y a quelque temps était celui-ci :

Considérons un polyèdre (convexe, pas forcément régulier). Sur chaque face du polyèdre il y a une fourmi, qui parcourt les arêtes de la face en question (au rythme qu'elle veut, mais de façon continue, bien sûr ; elle a le droit de s'arrêter, de ralentir ou d'accélérer, mais pas de revenir en arrière) toujours en tournant dans le sens trigonométrique (le sens contraire des aiguilles d'une montre). On suppose qu'entre deux instants donnés, chaque fourmi a accompli un nombre entier non nul de tours (autrement dit, chaque fourmi a fait au moins un tour, dans le sens trigonométrique, de la face dont elle parcourt les arêtes, et est revenue à son point de départ, qui est quelconque). Il faut montrer que, pendant ce laps de temps entre les instants considérés, deux fourmis (au moins) se sont croisées.

Cela a l'air parfaitement innocent, mais c'est absolument diabolique. Je me suis torturé pendant des heures sans rien trouver (pourtant, ce n'étaient pas les pistes qui manquaient). Un ami a fini par trouver une démonstration, mais elle est sophistiquée et peu intuitive, et utilise le théorème de l'indice[#] de Hopf. C'est décevant, parce que le problème est compréhensible par ma maman et je voudrais une solution qui le soit aussi. Et c'est décevant parce je n'ai pas trouvé, moi. ☹️ Notons au passage que l'hypothèse que le nombre de tour de chaque fourmi est entier est indispensable.

Mais récemment, on m'a posé un problème qui me semble encore pire : son énoncé n'est peut-être pas aussi élémentaire que celui des fourmis, mais il est extrêmement naturel et semble très joli :

Trouver la dimension maximale (si elle existe, ou même simplement un majorant de la dimension) d'un espace vectoriel de matrices réelles (je veux dire, un sous-espace vectoriel des matrices n×n réelles, pour n non précisé) dans lequel la seule matrice singulière (de déterminant nul) soit la matrice nulle.

Je sèche complètement. Je suis arrivé à la conclusion, et plusieurs autres ayant réfléchi au même problème y sont parvenus indépendamment, que la dimension 8 est possible (consulter n'importe quelle introduction aux octonions pour en savoir plus), mais quant à savoir si c'est ou non le mieux possible… Je ne trouve vraiment rien. Et j'y ai passé déjà un certain nombre d'heures.

C'est dur, les maths !

(Bon, là, on va voir si les lecteurs de mon blog sont des torscheurs. À chaque fois que je parle de quelque chose, il se trouve un commentateur qui connaît parfaitement le sujet pour intervenir avec une expertise impressionnante. Voyons donc si je vais me faire ridiculiser par une résolution en trois lignes de ces deux problèmes ! En vérité, ça me plairait bien.)

[#] Il est certain, au moins, qu'il faut employer quelque part une propriété topologique de la sphère, parce que le résultat n'est pas vrai si le polyèdre a la forme d'un tore, comme on le voit assez facilement. Donc l'hypothèse de genre zéro est cruciale. Ceci étant, il devrait y avoir des façons plus simples de caractériser ce fait que le théorème de l'indice de Hopf.

↑Entry #0637 [older| permalink|newer] / ↑Entrée #0637 [précédente| permalien|suivante] ↑

↓Entry #0606 [older| permalink|newer] / ↓Entrée #0606 [précédente| permalien|suivante] ↓

(Saturday)

On a theorem by Max Noether

Today I feel like explaining a little bit of algebraic geometry, namely a theorem by Max Noether and Castelnuovo on the so-called Cremona group of the plane. (My intention is to explain what the theorem states, not to prove it, which would take far more space than an entry here would allow. I hope I can make what follows clear to people with a relatively modest knowledge of mathematics—perhaps an undergraduate level.) Incidentally, let me mention that Max Noether is the father of Emmy Noether, who was certainly the most remarkable woman mathematician ever (and far more important to mathematics than Max Noether).

We concern ourselves with birational transformations of the plane, so let us first explain what this means. A rational map of the plane to itself is given, by definition, by two expressions, (x′,y′), giving the coordinates of an image point in function of those, (x, y), of the original point, in which we impose that x′ and y′ be rational functions of x and y, in other words quotients of two polynomials in these two variables (which we can assume, after elimination, to have no common factor), say, with real coefficients, the denominator being—of course—not identically zero. For example, letting x′=0 and y′=0 maps the entire point to the origin (0,0); letting x′=−x and y′=y defines the symmetry with respect to the vertical axis; letting x′=x and y′=x² gives a rational map that projects the entire plane onto the parabola with equation y=x², and so on. Note that a rational map is not always defined everywhere, since we have allowed polynomials in the denominator: for example letting x′=1/x and y′=1/y gives a rational map that is defined only so long as x and y are both nonzero (i.e. away from the coordinate axes), but we still call this a rational map of the plane to itself; since the denominator cannot be identically zero, there are always some points in the plane (many points, in fact: in a topological sense they are “dense”) for which the mapping is defined. Now we can generally compose two rational maps of the plane to itself, merely by replacing the x and y variables in the “outer” map by the x′ and y′ given by the “inner” map; for example, composing the former map (x′=1/x and y′=1/y) with itself gives the identity map (x′=x and y′=y), simply because 1/(1/t)=t. (Note that we cannot always compose two rational maps, because the composition might make sense nowhere, in other words it might end up giving an identically zero denominator, something we have excluded.)

When a rational map of the plane to itself is such that there exists another map which, when composed with it, (makes sense and) gives the identity map (viz. x′=x and y′=y), we say that the map (either of the two maps, actually) are birational transformations of the plane. For example, our previous example (x′=1/x and y′=1/y) is a birational transformation of the plane, since, as we have explained, when composed with itself, it gives the identity map. On the other hand, the constant rational map (x′=0 and y′=0) is not a birational transformation of the plane, since composing it with anything (in any order) gives a constant map if it makes sense at all. In perhaps more intuitive terms, a birational transformation of the plane is a rational map of the plane to itself which can be inverted by another rational map. (It is not exactly a bijection in the function sense, because it might not be defined everywhere. However, it is “mostly” a bijection.) The misfortune of composition not always being defined does not happen for birational transformations: the composition of two birational transformations of the plane always exists (makes sense) and is always itself a birational transformation; and, of course, by definition, any birational tarnsformation has an inverse which is also a birational transformation. (For those who know what that means, birational transformation of the plane form a group, the Cremona group of the plane. Note that it does not suffice for a rational map to be bijective in order for it to be a birational transformation: for example, x′=x³ and y′=y³ defines a rational map which is bijective because cube roots always exist and are unique, but which is not a birational transformation because cube roots are not rational functions.)

Now let us consider some particular birational transformatins. First of all, we have those of the following form: x′ = (a1·x+b1·y+c1) / (a0·x+b0·y+c0) and y′ = (a2·x+b2·y+c2) / (a0·x+b0·y+c0), in which the ai, bi and ci are real numbers: in other words, x′ and y′ can be expressed as quotients of first degree (aka affine) polynomials, with equal denominator; I'm not claiming that any such form defines a birational transformation, but it turns out that almost all do (all one needs to impose is the nonvanishing of the determinant of the 3×3 matrix of coefficients, and then the inverse transformation is given in the same form by the inverse matrix); those that are of this form are called projective transformations of the plane. For example, all plane symetries, rotations, translations, and much else, are all projective transformations. The composition of two projective transformations of the plane is a projective transformation, and the inverse of a projective transformation is a projective transformation (in technical terms, this means that projective transformations form a subgroup of the Cremona group). Basically, the projective transformations are the “uninteresting” birational transformations: they take lines into lines, so they are geometrically boring. (Another fact worthy of note is that given four distinct points in the plane, no three of which are aligned, and given another quadruplet of points satisfying the same condition, there is a unique projective transformation taking one set in the other in the prescribed order.)

Now let us consider two very simple examples of birational transformations which are not projective transformations: the first one, which has x′=1/x and y′=1/y, we have already mentioned; the second one is x′ = x/(x²+y²) and y′ = y/(x²+y²). Both give the identity when composed with themselves, as is easily checked, so both are, indeed, birational transformations of the plane. The first one corresponds to inverting the two (cartesian) coordinates independently, and the second corresponds to inverting the radial coordinate of polar coordinates (in other words, the distance to the origin) while keeping the angular coordinate fixed. Call these two transformations the basic Cremona transformations of the plane.

Well, the remarkable fact, which is the statement of Max Noether's transformation theorem, is that these two basic Cremona transformations are essentially the “only” ones we can form, or, rather, they are the building blocks for all other birational transformations; the precise statement is: any birational transformation of the plane can be written as the composition of projective transformations and the two basic Cremona transformations which we have given (of course it may be necessary to use either of them multiple times in the composition). So any birational plane transformation, no matter how complex, can be reduced to these basic transformations. (In reality, the theorem is somewhat simpler if we work over the complex numbers instead of the reals as we have done: then we need only one basic Cremona transformation, and either one will do. Over the reals we need two. But I did not want to make this entry more complicated by considering the plane with complex coordinates.)

To give an idea of why this is remarkable, we can consider the situation in other dimensions: is is straightforward to define birational transformations of the line, of three-dimensional space, and in fact, of n-dimensional space for any n, and among these are the obviously defined projective transformations. Now it turns out that in dimension one, the only birational transformations are the projective transformations (also called homographies): there are simply no others (and there is no Cremona transformation). In dimension three or more, it is not possible to find a finite number of particular birational transformations which, when composed with the projective transformations, will span all other birational transformations: the Cremona group is simply too complicated, and it is hardly possible to say anything useful about it.

Well, I don't know whether this is all very enlightening; at least, it is a little but elegant bit of algebraic geometry.

↑Entry #0606 [older| permalink|newer] / ↑Entrée #0606 [précédente| permalien|suivante] ↑

↓Entry #0511 [older| permalink|newer] / ↓Entrée #0511 [précédente| permalien|suivante] ↓

(Friday) · Last Quarter

Math on the Web

One thing which regularly annoys me is how difficult it is to find mathematics on the Web at the level I'm interested in. One can easily find a lot of undergraduate-level stuff, and a lot of research papers, but there seems to be a gap between the two at which there is little available material online. For example, just now, I wanted the precise definition of a Gorenstein ring (or a Gorenstein singularity, or whatever: it's the word Gorenstein which interests me, in any context). Now there are tons of research papers which deal with resolution of Gorenstein singularities or classifications of them, or whatever: but they never bother to recall what a Gorenstein singularity is, because everyone is supposed to know that (in order to make any sense of the paper or to have any interest in it). And on the other hand, this is way too advanced for most online courses: what I would need is a general treatise on Cohen-Macaulay rings (such as this book by Bruns and Herzog, of which I have a copy at my parents' house but none where I am now).

Of course, there is MathWorld, which is often useful (especially when it comes to formulæ of kinds): here it gives a definition of a Gorenstein ring (I won't link to it because I don't wish to Google-feed it), but I can't make heads or tails of it — I'm sure they omitted something. And this isn't just bad luck in this particular case: it's really quite typical of their definitions in relatively high-brow domains. When there is an entry in the Wikipedia, it's usually more useful, but, unfortunately, the word Gorenstein doesn't appear in the Wikipedia (I guess I'll have to write the entry when I find out precisely). In general, the Wikipedia isn't very good when it comes to mathematics because there isn't a convenient way to typeset mathematical formulæ in it.

(That's what research libraries are for, you say? Yes, but it is presently 4AM, and all research libraries I have access to are closed. And how can I go to sleep without knowing the correct definition of a Gorenstein ring? Now you understand my dilemma!)

↑Entry #0511 [older| permalink|newer] / ↑Entrée #0511 [précédente| permalien|suivante] ↑

↓Entry #0425 [older| permalink|newer] / ↓Entrée #0425 [précédente| permalien|suivante] ↓

(Tuesday) · Premier Quartier

Popularizing mathematical concepts

Some time ago I got into my mind that I should try to write a little text (perhaps entries into this 'blog, perhaps a standalone document) explaining something about the meaning of three mathematical objects, in a way understandable to the layman (whose a priori level of mathematical culture remains to specify: it should include a basic understanding of natural numbers and plane geometry but no knowledge of ordinals).

The three mathematical objects, which I chose because they fascinate me, because I think there is genuine beauty in them, and I would like to share with others the vision of that beauty, and because of their tantalizing relation to “infinity” (in a broad sense) are: the Long Line, the Stone-Čech compactification of the natural numbers (also known as beta-omega), and Zariski's Ciel Étoilé (Starry Vault). These objects all convey a geometrical, almost graphical, intuition: yet they defy us (in a way akin to the bewilderment caused by Escher's prints, but much stronger) as they cannot be represented accurately by any pictorial representation, they cannot be “embedded” in our Universe.

↑Entry #0425 [older| permalink|newer] / ↑Entrée #0425 [précédente| permalien|suivante] ↑

↓Entry #0377 [older| permalink|newer] / ↓Entrée #0377 [précédente| permalien|suivante] ↓

(mardi)

1.0293753730039641320569866…

Encore un problème qui me rend fou :

Quelle est l'espérance du maximum de quatre variables gaussiennes indépendantes chacune de moyenne zéro et écart-type un ?

Numériquement, une valeur approchée est celle qui sert de titre à cette entrée. Si on remplace quatre par trois ou deux (ou bien sûr une) variables, je sais trouver une valeur symbolique exacte (enfin, surtout Mathematica sait si je le dirige un peu) : 1/√π (désolé, c'est moche, mais je veux dire un sur racine de pi, je ne vais pas mettre de MathML ici) pour deux variables, et 3/2 fois ça pour trois variables. Mais quatre… Péter m'a suggéré une façon de faire le calcul qui devrait aboutir à un résultat exact, mais c'est assez atroce à mener explicitement (de toute façon le problème est de délimiter une région sur la sphère de dimension trois qui a pour mesure un 24e de la mesure totale, et de faire une intégrale dessus).

Le merveilleux et fascinant inverseur de Plouffe ne connaît pas ce nombre.

Mise à jour () : Complètement par hasard, j'apprends que la valeur exacte de ce nombre est : (3/√π)[½+arcsin(1/3)/π]. Et pour cinq variables : (5/(2√π))[½+3·arcsin(1/3)/π]. Voir cette page, qui prétend que des expressions en forme close existent aussi pour six et sept variables en utilisant des dilogarithmes (et huit et neuf avec des trilogarithmes).

Nouvelle mise à jour () : Voir aussi ce lien, qui explique un peu comment cqlcuer ces choses.

Encore une mise à jour () : Voir ce fil Twitter où je récapitule (sans rien dire de nouveau) l'état de l'art sur ce problème.

↑Entry #0377 [older| permalink|newer] / ↑Entrée #0377 [précédente| permalien|suivante] ↑

↓Entry #0324 [older| permalink|newer] / ↓Entrée #0324 [précédente| permalien|suivante] ↓

(Friday) · Halloween

Freakish intellectual masturbation

[Traduction française ci-dessous.]

Don't try to solve this one without a computer:

Mr Magic says, I'll secretly choose two integers between 2 and 3000, and I'll tell their sum to Steven and their product to Peter, and, of course, does so. Peter comments, I don't know what the two integers are. Steven remarks, yeah, I knew that. Whereupon Peter says, oh? well now I know what they are. And immediately Steven says, now so do I. Mr Magic then asks Alice (who was also listening to the conversation), do you know what the two numbers are?, and Alice answers, of course not. So Mr Magic tells Alice the smaller of the two integers. And Alice replies, now I know what the other one is.

What are the two numbers?

This is a sick—pervert—problem if I ever heard one. (I'm not sure I should call this mathematics, either, though there is certainly some mathematical ground here, and it requires logical reasoning; but there is no theory behind it, as far as I can see.) Yet it can be solved (by brute force, as it were). In case you'd like to know, the answer (click here to reveal it if it is invisible) is . But the problem itself isn't so much interesting as the pattern. For example, the following problem is much clearer (and you definitely don't need a computer to solve this one):

Mr Magic announces, I'll roll two (ordinary, six-sided) dice in secret, and tell the smaller of the two figures (between one and six) to Minnie and the larger to Max, and he does so. The sum of the two numbers is neither six nor eight, he also says. Max remarks, I don't know what Minnie's number is. Minnie replies, I knew that. Then she adds: now do you know my number?; to which Max replies, yes. And Minnie says, then I know yours.

What are the two numbers?

Just follow the reasoning with pencil and paper: when it is done right, it is child's play. The answer (click here) is . I find much more elegance in this simpler problem than in the original one. But the basic logic is the same. (On the other hand, I did spend a tremendous amount of time coming up with a situation where every reaction would be more or less natural, which wasn't too gory to explain or too trivial to solve, etc. If you think it's easy, just try it for yourself!)

[French translation of the above.]

N'essayez pas de résoudre ce truc sans ordinateur :

M. Magie dit, je vais secrètement choisir deux entiers entre 2 et 3000, et j'en dirai la somme à Stéphane et le produit à Pierre, et, bien sûr, il le fait. Pierre observe, je ne sais pas quels sont les deurs entiers. Stéphane remarque, ouais, je le savais. Sur quoi Pierre dit, ah ? eh bien maintenant je sais ce qu'ils sont. Et immédiatement Stéphane dit, maintenant moi aussi. M. Magie demande alors à Alice (qui écoutait aussi la conversation), savez-vous quels sont les deux nombres ?, et Alice répond bien sûr que non. Alors M. Magie donne à Alice le plus petit des deux entiers. Et Alice répond, maintenant je sais quel est l'autre.

Quels sont les deux nombres ?

Voilà un problème cinglé — pervers — si j'en ai jamais vu. (Je ne suis pas certain que j'appellerais ça des maths, non plus, même s'il y a assurément des bases mathématiques là, et cela exige un raisonnement logique ; mais il n'y a pas de théorie derrière, pour autant que je voie.) Pourtant, il peut être résolu (par force brute, en tout état de cause). Si vous voulez savoir, la réponse (cliquez ici pour la révéler si elle est invisible) est . Mais le problème en lui-même n'est pas aussi intéressant que le motif. Par exemple, le problème suivant est beaucoup plus clair (et vous n'avez assurément pas besoin d'un ordinateur pour résoudre celui-ci) :

M. Magie annonce, je vais jeter deux dés (ordinaires, à six faces) en secret, et dire le plus petit des deux chiffres (entre un et six) à Minnie et le plus grand à Max, et il le fait. La somme des deux nombres n'est ni six ni huit, dit-il encore. Max observe, je ne sais pas quel est le nombre de Minnie. Minnie réplique, je le savais. Puis elle ajoute : maintenant, sais-tu quel est mon nombre ? ; ce à quoi Max répond, oui. Et Minnie dit, alors je sais le tien.

Quels sont les deux nombres ?

Suivez juste le raisonnement avec papier et crayon : quand on le fait correctement, c'est un jeu d'enfant. La réponse (cliquez ici) est . Je trouve beaucoup plus d'élégance dans ce problème plus simple que dans l'original. Mais la logique de base est la même. (D'un autre côté, j'ai effectivement perdu un temps fou pour trouver une situation où chaque réaction serait plus ou moins naturelle, qui ne soit ni trop pénible à décrire ni trop triviale à résoudre, etc. Si vous croyez que c'est facile, essayez par vous-mêmes !)

↑Entry #0324 [older| permalink|newer] / ↑Entrée #0324 [précédente| permalien|suivante] ↑

↓Entry #0308 [older| permalink|newer] / ↓Entrée #0308 [précédente| permalien|suivante] ↓

(Thursday)

On the beauty of the Steiner system of index (5,8,24)

If there were a beauty prize for mathematical objects, I think the Steiner system of index (5,8,24) (I will describe what this means in a moment) would be one of the most serious candidates. It is something extremely easy to define (but not so easy to exhibit or represent!) but of breathtaking intellectual elegance and having absolutely unique and “magical” properties. This is the sort of mathematical objects that holds (for me) all the fascination that numerology can have for some people, except that there is “really something” there (when I get mystical, I think: some deep insight into the fabric of reality).

To define naïvely what a Steiner system of index (5,8,24) means is, as I just said, very easy: it is a set of 24 “points” (objects, elements, whatever) together with 759 “blocks”, each consisting of 8 points, having the property that any 5 of the 24 points lie in one and only one of the 759 blocks. I know this doesn't sound impressive when said like that.

Maybe to give the feel of things I should explicitly describe the Steiner system of index (2,3,7) (the projective plane over F2, to be precise, also called the “Fano plane”). Consider 7 objects (“points”) which will be labeled (1:0:0), (0:1:0), (1:1:0), (0:0:1), (1:0:1), (0:1:1) and (1:1:1) (the labels are unimportant: I could just as well be calling these objects “red”, “green”, “yellow”, “blue”, “magenta”, “cyan” and “white”, or “Valor”, “Compassion”, “Sacrifice”, “Honesty”, “Honor”, “Justice” and “Spirituality” or whatever I wanted); and define the following seven blocks of three objects each: {(0:1:0), (0:0:1), (0:1:1)}, {(1:0:0), (0:0:1), (1:0:1)}, {(1:1:0), (0:0:1), (1:1:1)}, {(1:0:0), (0:1:0), (1:1:0)}, {(0:1:0), (1:0:1), (1:1:1)}, {(1:0:0), (0:1:1), (1:1:1)} and {(1:1:0), (1:0:1), (0:1:1)}. Then any choice of two of the seven points belongs to one, and only one, of the seven blocks. Try it!

The previous example has a classical geometric representation: draw an equilateral triangle, label its vertices (1:0:0), (0:1:0) and (0:0:1); label (1:1:0) the middle of the edge joining (1:0:0) and (0:1:0), label (0:1:1) the middle of the edge joining (0:1:0) and (0:0:1) and label (1:0:1) the middle of the third edge of the triangle; finally, label (1:1:1) the center of the triangle, and draw the inscribed center of the triangle as well as the three medians (which are also at once mediators, heights and bisectors since the triangle is equilateral). This defines seven points in the plane, and seven “lines” between them (the three sides of the triangles, the three medians and the seventh “line” is the inscribed circle), each of them joining exactly three points, and it is easily seen that any two points are joined by exactly one line.

Now the Steiner system of index (5,8,24) does the same with different numbers: there are 24 points and blocks of 8 points (“octads”) are defined such that any set of 5 different points belongs to exactly one of the octads. The first remarkable, and by no means obvious fact, is that there is only one Steiner system of index (5,8,24): if you find two of them, then there is some way of reordering the 24 points so that in fact they agree exactly. So it is justified to speak of the such Steiner system. (There is also a unique Steiner system of index (2,3,7); however not all Steiner systems are unique for their index: for example, there are exactly 2 Steiner systems of index (2,3,13), 18 of index (2,4,25). Also, not all Steiner systems exist even if there is no evident impossibility; for example, there is no Steiner system of index (2,7,43); it is not known whether there exists one of index (2,13,157), although it is conjectured that there is none. But very elementary knowledge of combinatorics suffices to prove that there is no Steiner system of index (6,9,25), say: the number of blocks could not be an integer.)

The Steiner system of index (5,8,24) has a large number of automorphisms, in other words, manners of permuting the 24 points in such a way that if eight points formed a block before the permutation, the new eight points which take their place after permutation still form a block. As a matter of fact, there are 244823040 automorphisms of the system: these constitute what is known as the Mathieu group M24. This group is five times transitive, meaning that if you take any five of the twenty-four points and place them in any five new places, there is a way (in fact, exactly 48 ways) to complete this choice into an automorphism of the system. Now this property is remarkable in the highest extent: indeed, apart from the full group of permutations on n objects (which is n times transitive) and the so-called “alternating group” of even permutations (which is n-2 times transitive), there are only two (permutation) groups which are 5-transitive: the Mathieu groups M24 (automorphisms of a Steiner system of index (5,8,24), as explained) and M12 (automorphisms of a Steiner system of index (5,6,12)), and neither is 6-transitive or more. And also, the Mathieu groups are among the twenty-six sporadic simple groups; but describing what this means (even by removing the word “sporadic”) would take just a bit too long for me to try it now.

Furthermore, I mention that the Steiner system (5,8,24) can be used to construct the (binary) Golay code, the most remarkable (and “powerful”!) error-correcting code ever, in the following way. Consider the 759 blocks (octads) of the Steiner system (5,8,24) as words of 24 bits, by putting a ‘1’ in a given place if the corresponding point is in the octad, and ‘0’ otherwise (so each of these blocks will have exactly 8 bits to ‘1’ and the others to ‘0’). Now combine these words in every possible way using (bitwise) XOR (eXclusive OR). This gives a total of 4096 words (a vector space of dimension 12 over the finite field with 2 elements), the words of the Golay code, none of which (except the entirely zero word) has fewer than 8 bits with value ‘1’; it is then possible to judiciously choose twelve columns out of the twenty-four in such a way that every combination of ‘0’'s and ‘1’'s in these columns matches one and exactly one of the 4096 words of the code. So we can code any word of 12 bits by a word of 24 bits in the Golay list: this code can correct an arbitrary error on 3 bits out of 24, and detect an arbitrary error on 4 bits. Out of the 4096 words of the Golay code, 759 have 8 bits with value ‘1’, 2576 have 12 bits with value ‘1’, 759 have 16 bits with value ‘1’ (and are the complements of the 759 with weight 8) and the last two are the fully zero and fully set words.

So far I have described some properties of the Steiner system (5,8,24) or the Golay code, but I have not described them explicitely. There is a plethora of ways to construct them, more or less intuitively understandable and more or less pleasant; however, there is at least one way I find truly remarkable: the dodecahedron construction of the Golay code, which works as follows. Take a (regular) dodecahedron and two colors of ink (say, black and red). Using the black ink, write ‘0’'s and ‘1’'s arbitrarily on the dodecahedron, one bit per face. Now using the red ink visit each of the dodecahedron's faces in turn, and compute the parity of all the black bits except those on the five faces immediately adjacent to the one on which the red bit is being written; in other words, each red bit on a given face of the dodecahedron indicates the parity of the black bits for the seven faces of the dodecahedron which are not immediately ajacent to the face in question: the red bit is ‘1’ when there are an odd number of black bits at ‘1’ among the seven faces in question, or ‘0’ when there are an even number of them. Thus, starting from twelve arbitrary black bits we get twelve red bits, to a total of twenty-four bits, two on each face of the dodecahedron: well, this is exactly the Golay code: the 4096 twenty-four bits words obtained by trying all possible combinations of black bits give exactly the 4096 words of the Golay code; and to construct the Steiner system of index (5,8,24), just take those 759 words having eight bits at ‘1’.

One of my bizarre dreams would be to find some way to construct a puzzle similar to Ernő Rubik's famous cube—only it would probably be shaped more like a dodecahedron—that has the Mathieu group M24 as group of transformations. [Update: I wrote a couple of such JavaScript games: see here, here and more importantly here; also see this link which was suggested in the comments.]

↑Entry #0308 [older| permalink|newer] / ↑Entrée #0308 [précédente| permalien|suivante] ↑

↓Entry #0300 [older| permalink|newer] / ↓Entrée #0300 [précédente| permalien|suivante] ↓

(Saturday) · Last Quarter

My mathematical diary

[300th entry in this 'blog! Hurray, hurray, hurray!]

At the end of 2001, I had started a mathematical diary, which I kept active through most of 2002. The idea is not to write down ideas that are important for my current mathematical research activity (writing my thesis, that is), but, on the contrary, to evacuate by committing them on paper various side ideas that I have from time to time. It's the same sort of force that drives me to write this 'blog: once I become infected with a meme, say a mathematical problem, it will keep haunting me until I either solve the problem or save it on paper (electronic paper will do as well, of course) for later. So this is how this diary should be regarded: as a meme pool of weird ideas. There is nothing that could be called “mainstream mathematics” in it, I guess. Some of these memes are “abstract nonsense” as mathematicians call it, embryos of theories that are trivial to work through the basic definitions, which produce pages and pages of easy writing. Some are (presumably hard) questions.

In a way, this diary could be shown as evidence of my mathematical angst: probably nothing it contains would be deemed of any interest by any other mathematician, and I guess the questions which obsess me are very much alien to mainstream mathematical culture. This is one of the reasons why I have solid doubts as to whether I should pursue the academic career in mathematics.

Anyway, whatever it's good for, I've decided to start writing in this diary again. Probably not nearly as actively as this 'blog, but it'd be nice to hold a one-entry-a-week minimum average. There are lots of goofy thoughts that I won't be rid of until I've written them down.

↑Entry #0300 [older| permalink|newer] / ↑Entrée #0300 [précédente| permalien|suivante] ↑

↓Entry #0211 [older| permalink|newer] / ↓Entrée #0211 [précédente| permalien|suivante] ↓

(Sunday)

Behold the cubic surface!

[Cubic surface]Here on the left (click to enlarge) you can see—to contradict an earlier statement I made that they are difficult to picture—a (nonsingular) cubic surface, one of the beasts that I've spent a good part of my thesis studing (the arithmetic of). Specifically, this is the surface with (affine) equation y³-3x²y+z³-3z=0. The white rods (fifteen of them, if you count well) are not part of the surface itself—or rather, they are, but they've been emphasized for clarity: they represent the straight lines lying on the surface. There are always twenty-seven straight lines on a nonsingular cubic surface, but all might not be “real” in the sense that some are actually pairs of complex conjugate lines; and this particular surface has fifteen real lines and six pairs of complex conjugate lines. Sometimes three lines on the surface (necessarily in the same plane) meet in one point: then that point is called an Eckardt point; this surface happens to have six Eckardt points (all real), three of which have been shown on the picture as small bright pink spheres (the other three are at infinity so you cannot see them); one of them (namely (0,0,0)) is at the center of the image. I've already mentioned Eckardt points on this 'blog; they have many remarkable properties, but they make the arithmetic of the surface rather harder to study when they exist. The sort of question one might ask is this: given that the surface has one point with rational coordinates (namely (0,0,0)), and since its equation has rational coefficients, is it true that there are points with rational coordinates arbitrarily close to any real point? (And the answer, for this surface, is yes.)

The image was made with the Persistance of Vision (“POV-ray”) raytracer. I don't deserve much credit since POV-ray has a primitive (quite appropriately called cubic!) which draws a cubic surface. My work as a mathematician in composing this image was limited to finding the equation of a nice cubic surface having some Eckardt points and then computing the equation of all the lines on it (a horrendous task in general, but relatively easy for this one surface since it has a very simple equation in which variables are separated). Note incidentally that the colors are not on the surface but come from three colored light sources.

I've also made a little animation of the rotation of the cubic surface (984kbyte AVI file) from the images computed by POV-ray. (Don't ask me how to read it or what codec it uses, I don't know anything about this stuff: I just fiddled around with MPlayer/MEncoder, randomly tweaking the command line options until it produced something that seemed like it was an animation.)

Would you believe it? Cubic surfaces have their own Web site!

↑Entry #0211 [older| permalink|newer] / ↑Entrée #0211 [précédente| permalien|suivante] ↑

↓Entry #0166 [older| permalink|newer] / ↓Entrée #0166 [précédente| permalien|suivante] ↓

(Saturday)

Seven colors

[Plane coloring]This image represents a periodic tiling of the plane with hexagons colored with seven colors, having plenty of nice mathematical properties. One of them is this: if you take any two points the distance between which is given by the little black rod shown on the upper left, then these two points cannot have the same color. In other words, no matter how you move the rod in the picture, so long as you don't change its size, the two ends will fall on different colors.

Can this be done with fewer than seven colors? In other words, is there a way to color the plane (not necessarily perodically) using six colors or less, in such a way that any two points separated by a certain distance d fixed a priori are always of different colors? Interestingly enough, the answer to this problem is not known: I believe this is still an open problem. (I don't know who asked the question in the first place. It sounds typical of Pál Erdős, but that's mere guessing.)

Probably this is the mathematical problem with the simplest formulation which is still open.

It can be shown, however, that no plane coloring with three or less colors can do the trick: if you color the plane, in any way whatsoever, using three colors or less, and you fix an arbitrary distance d, then there are always, somewhere, two points separated by that distance d that are of the same color. This is a very nice mathematical exercise, because it requires almost no knowledge of geometry (let alone of any other domain of mathematics), and yet it is not at all obvious—almost an exercise of pure reasoning. I remember hitting my head against the walls for quite some time before I found the trick; and I occasionally gave it to students to try, nearly driving some of them mad. The problem with two colors (showing that two colors or fewer cannot suffice to color the plane in such a way that two points at distance d are not of the same color), however, is very simple, and anyone should be able to solve that; and it holds the key to solving the case of three colors.

So I can't be held responsible for people banging their head against walls, I'll provide the answer to the two- and three-color problems now. (Click here to reveal the following text if it is invisible.) which completes the proof.

Perhaps this is a nice test to determine whether a person is mathematically inclined, or could be, even if that person has no knowledge of mathematics: anyone so inclined will not resist thinking about such a simple and elegant problem for some time.

Update: The problem in question is known as the Hadwiger-Nelson problem: see this later entry (in French) and the ones linked from there.

↑Entry #0166 [older| permalink|newer] / ↑Entrée #0166 [précédente| permalien|suivante] ↑

↓Entry #0120 [older| permalink|newer] / ↓Entrée #0120 [précédente| permalien|suivante] ↓

(Sunday) · My Birthday

I have defeated the Eckardt points!

(The title is mostly a joke, because Eckardt points really weren't the main difficulty.) Well, after another bunch of hours rewriting this proof in all aspects, I seem to have come to a final-so-far version of it: the statement is that weak approximation holds in places of good reduction for smooth cubic surfaces over the function field of a curve over an algebraically closed field. If that's Chinese to you, here's a slightly more understandable version of the same: consider a homogeneous cubic equation in four unknowns (say, X, Y, Z and T) whose coefficients are rational functions of one indeterminate (say, λ) having themselves complex coefficients: something like c3000(λX³ + c2100(λX²·Y + c2010(λX²·Z + … + c0003(λT³ = 0 where the cijkl are rational functions in λ with complex coefficients, and we are trying to solve for the variables X, Y, Z and T also as rational functions in λ; well, the idea is that if you fix a finite number of λ and for each one choose a solution of the corresponding equation (when specialized to that value), then there should be a parametric solution (one where X, Y, Z and T are rational functions in λ; I insist, rational) which interpolates them all, and actually, in a way I won't try to describe, one can fix the derivatives to any order of the variables X, Y, Z and T, not just their values at certain points; now actually this doesn't work for all λ: there are a certain finite number of them which can't be handled (places of “bad reduction”), essentially those in which the specialized equation becomes too degenerate in certain ways.

Now that I've finished that, I can perhaps start working on the piles and piles of emails that I still have to reply to.

↑Entry #0120 [older| permalink|newer] / ↑Entrée #0120 [précédente| permalien|suivante] ↑

↓Entry #0081 [older| permalink|newer] / ↓Entrée #0081 [précédente| permalien|suivante] ↓

(Sunday)

Some mathematics

I've mentioned several times in this 'blog my joint paper with Jean-Louis Colliot-Thélène on Del Pezzo surfaces over fields of cohomological dimension one, paper which I presented in Besançon. I would now like to try to explain something of what it is about for the benefit of non-experts. Unfortunately, I won't make my explanations accessible to all laymen, because that would either take too much space or require to remain at a level of generality such that it wouldn't be interesting to anyone; but I have plans for writing a general introduction to arithmetical geometry, at some point, that would be understandable by everyone and his dog. For the moment, I'll assume that the reader has undergraduate (math major) knowledge of algebra, including knowing what a field is (though I'll recall it in a minute), and also a polynomial in several indeterminates and a vector space. I guess that excludes a lot of people, but at least I'm not assuming that you know what an étale cohomology group (and, therefore, a field of cohomological dimension one) is. Anyway, here we go.

A (commutative) field (I recall just to make sure everyone agrees, but I assume the reader knows this already) is a domain E with two operations, addition and multiplication, so that addition puts an abelian group structure on E (in other words, addition is associative and commutative and has a neutral element written 0 (zero), and for every element x of E there exists an element written -x and called the negative of x, such that x+(-x) = (-x)+x = 0), multiplication distributes over addition, multiplication is associative and commutative and has a neutral element written 1 (one), and for every element x of E other than 0 there exists an element x-1 such that x·x-1 = x-1·x = 1. Examples of fields are the set R of real numbers or that, C, of complex numbers; or the set Q of rational numbers, or the set R(t) of rational functions in one indeterminate t with real coefficients; we also have the set Z/pZ of integer (residues) modulo a prime number p, or, for those who know what that is, the set of p-adics Qp.

If we take this definition of a (commutative) field and remove just the assumption that multiplication is commutative, leaving everything else as it is, we get the definition of a division algebra, or skew-field (some people just say “field”). The simplest example is probably the skew-field of quaternions, H: it is obtained by adjoining three symbols, i, j and k to the real numbers subject to the relations that i² = j² = k² = ijk = -1 (this implies, for example, that ij = k whereas ji = -k). It is a famous theorem of Wedderburn that every finite division algebra (in the sense that it has only finitely many elements) is, in fact, commutative (in other words, is a field). In any division algebra D, the set of elements x which commute with every other element y, that is, so that xy=yx for every y, forms a commutative field (a subfield of D) called the center of D (for example, the center of H is just R).

Recall that a field E is said to be algebraically closed iff every non-constant polynomial equation (with coefficients in E) in one indeterminate has a solution. For example, the equation x²-2=0 (in the indeterminate x) should have a solution (“the square root of two” up to a sign) in any algebraically closed field: this shows that the field Q of rational numbers is not algebraically closed, and neither is the field R of real numbers (because the equation x²+1=0 has no root in R, that is, -1 has no real square root); on the other hand, the field C of complex numbers is algebraically closed, a theorem due to Gauß (based on preliminary work by D'Alembert) and often referred to as the “fundamental theorem of algebra”.

A mere matter of notation: rather than considering polynomial equations in one indeterminate such as x²-2=0, algebraic geometers prefer to consider polynomial equations in two indeterminates which are homogeneous, meaning that the total degree (sum of the degree in each indeterminate) of each monomial es the same, such as X²-2T²=0: this can be arranged by the procedure (homogenizing the equation) consisting of adding a new indeterminate (here T) and multiplying each monomial by the appropriate power of it so that the total degree is the same everywhere. Now we look for non-trivial solutions of the homogeneous equation, that is, solutions in which all indeterminates are not simultaneously zero: it will readily be seen that (X,T) is a solution to the homogeneous equation X²-2T²=0 if and only if the ratio x=X/T is a solution of the initial equation x²-2=0. So it is the same to ask that every homogeneous polynomial equation in two indeterminates of (strictly) positive degree have a non-trivial solution as to ask that every non-constant polynomial equation in one indeterminate has a solution: both mean that the working field is algebraically closed. Actually it turns out that the restriction on the number of variables is irrelevant: over an algebraically closed field, any (one) homogeneous polynomial equation of (strictly) positive degree has a non-trivial solution so long as there are at least two indeterminates (and any (one) non-constant polynomial equation has a solution as long as there is at least one indeterminate). In fact, we can do better: over an algebraically closed field, if we consider at least m homogeneous equations of (strictly) positive degrees in n indeterminates, then so long as n-m is at least 1, they have a common non-trivial zero.

Homogeneous polynomials of degree two are called quadratic forms, and of degree three, cubic forms. When they have a non-trivial zero (in a given field) they are called isotropic, otherwise anisotropic. So the above paragraph tells us, in particular, that a form (quadratic, cubic or whatever) in at least two variables over an algebraically closed field is always isotropic. For quadratic forms, this is not true over the field of real numbers: in fact, there are quadratic forms with arbitrarily many variables over the real numbers that are not isotropic, for example X²+Y²+Z²+T² cannot be zero over the reals unless all of X, Y, Z and T are zero (the trivial solution), for reasons of positivity; on the other hand, any cubic form over the reals in at least two variables is always isotropic.

↑Entry #0081 [older| permalink|newer] / ↑Entrée #0081 [précédente| permalien|suivante] ↑

↓Entry #0064 [older| permalink|newer] / ↓Entrée #0064 [précédente| permalien|suivante] ↓

(Thursday) · Corpus Christi

Sociology and mathematics

Some is trying to make mathematical sense of a formula by Claude Levi-Strauss. Amusing.

↑Entry #0064 [older| permalink|newer] / ↑Entrée #0064 [précédente| permalien|suivante] ↑

↓Entry #0022 [older| permalink|newer] / ↓Entrée #0022 [précédente| permalien|suivante] ↓

(Sunday)

A mathematical amusement: bidecimal numbers

Here's a little mathematical fact that may amuse the reader. I thought about this some time ago, and was reminded of it while proving some statements about “weak approximation” today.

Start with a concrete case: on 2002-01-01, various European countries switched to a new currency, the euro. Exchange rates had been fixed several months earlier, by defining the euro's value as an exact number of the various earlier currencies (deutschemarks, liras, French francs and so on), these numbers being the then market exchange rate with six significant digits. For example, the euro was defined as (exactly) 6.55957 French francs. Naturally, this makes the French franc equal to approximately 0.152449 euro(s); however, in that direction, the ratio is not exact, because it is even more precisely 0.15244902 euro(s), and even that is not exact (though at that point, the further decimals become quite meaningless, but we are interested in mathematics, so we can't call it exact).

Call a number decimal iff it can be written with a finite (that is, ending) number of digits after the decimal point; in other words, it can be written as the quotient of an integer by some power of ten (as in: 6.55957 = 655957/100000). For example, 1/2 (one half) is a decimal number because it is 0.5 (or 5/10), and so is 1/5, but 1/3 and 1/7 are not decimal numbers (although they are, of course, rational numbers). So the value of the euro in all of the former European currencies was defined exactly as a decimal number (the euro's value in French francs, 6.55957, is decimal); on the other hand, the value of the currencies in euro(s) is not decimal (for example, 1/6.55957 is not decimal, because 0.152449 or 0.15244902 are only approximations, and no finite number of digits will give an exact value). This can be annoying (in practice it never is, or was, a problem, but assume for the sake of argument that it is).

Now call a number bidecimal iff it is decimal and its reciprocal (or “inverse”, that is, one divided by that number) also is a decimal number. There exist bidecimal numbers: for example, 4 is a bidecimal number, because 4 is decimal (as are all integers) and 1/4=0.25 is also decimal. Similarly, 1.25 is bidecimal (its reciprocal is 0.8). Now assume we had wanted the currency rates in both directions (the euro's value in the former currencies and the former currencies' value in euro(s)) to be decimal: in other words, for the currency rates to be bidecimal. Could we have done this? Of course, it is not the matter of picking an arbitrary bidecimal number: the fixed chosen currency rates need to be very close to the market rates at fixation time. The question, then, is: is it possible to find bidecimal numbers that are arbitrarily close to any given number? Or, in mathematical terminology: are the bidecimals dense in the real numbers? And the answer is yes. If it had been desirable to make the currency rates bidecimal, that could have been done while keeping adequation to an arbitrary precision with the market rates of the day. For instance, one can find a bidecimal number which is between 6.559565 and 6.559575, thus being close to 6.55957; or between 6.5595695 and 6.5596705; or to any desirable precision. I'm not claiming, though, that to fix the rate's approximation of market rates to six significant figures we need only six digits in the bidecimals! Actually, a bidecimal number between 6.559565 and 6.559575 needs quite an impressive number of digits to be written exactly (or its inverse); but it can be done.

As for why that wasn't done in practice, the last sentence above is the answer. But it is an amusing mathematical curiosity. I'll let the mathematically inclined reader figure out the proof.

Note that it doesn't work in binary, for instance: can you explain why that is?

↑Entry #0022 [older| permalink|newer] / ↑Entrée #0022 [précédente| permalien|suivante] ↑


List of all categories / Liste de toutes les catégories: glf, math, phys, ego, html, gay, relig, philo, polit, lex, libri, lang, unicode, cinema, linux, comp-hw, econom, auto-moto, gardinia, meta

[Index of all entries / Index de toutes les entréesLatest entries / Dernières entréesXML (RSS 1.0) • Recent comments / Commentaires récents]